You are on page 1of 4053

Introduction

NOTE: The intent of Knowledge Testing review books is to improve


performance on board and certification exams. Review books are not intended to
be a source of the knowledge base of medicine. The authors or editors do not
warrant the information is complete or accurate. The reader is encouraged to
verify each answer in several references.

All drug indications and dosages should be verified before administration.

Copyright 2015 by Knowledge Testing, Inc.

All rights reserved, including the right of reproduction, in whole or in part, in


any form.

The authors and editors would like to extend thanks to Thomas Johnson and
Mary Nan Farber for their editorial support.

http://www.knowledgetesting.com

INTRODUCTION

Congratulations! This Knowledge Testing book will help improve your fund of
knowledge. Our books are designed to help you improve your score on board
and certification exams. To achieve this goal the text is written in a multiple-
choice format with answers and explanations. Explanations contain additional
information intended to reinforce your knowledge. Emphasis has been placed on
covering facts that are easily overlooked, quickly forgotten, and often show up
on board and certification exams. For further information, all explanations are
electronically linked to eMedicine(c)(free), Google Scholar(c) (free), or
UpToDate(c) (subscription). We have no affiliation with these medical websites.

We suggest you note all questions answered incorrectly to allow you to rapidly
review the content prior to your exam.

We appreciate your comments, suggestions and criticisms. Some answers and


explanations may be in variance with your own knowledge. This is usually
attributable to variability among sources; however, please make us aware of any
potential errors you find as we update our books often. We appreciate your input
in regard to format, content, or presentation. Please contact us at
support@knowledgetesting.com or better yet, click on the flag button after each
question to give us specific input on any errors or omissions. We look forward to
hearing from you.

Good luck with your board and certification exam!

Knowledge Testing eBooks

Knowledge Testing books were created exclusively for use on eBook readers.
Because we don't publish our books in paper format, we have the luxury of
optimizing them for eBook use.

How are our books optimized for the viewing on an eBook Reader?

1. Efficient User Interface: Each multiple choice question is on a page by


itself so you can see the question and four multiple choice answers, but you
can't see the correct answer until you flip the page. This would be a
complete waste of paper in an ordinary book, but it makes for an easy to
use eBook. You don't have to cover up the correct answer with a piece of
paper. Most devices have a dedicated button for page turning so with the
click of a button, you can see the correct answer.
2. Research Links: After each explanation we include web links to search for
more information on the key concepts for that question. If you miss a
question, this is when you probably want to do more research on the
question concepts. This would be onerous and require a lot of typing using
a paper book, but is a single click on an eBook reader, PC, or Mac. Our
customers have asked for many search options, and we have chosen the
following for "assisted research":
eMedicine (now Medscape/WebMD) -
http://emedicine.medscape.com/
UpToDate - http://www.uptodate.com
Google Scholar- http://scholar.google.com/
We have no affiliation with any of these three information providers.
Also, to look at more than one page you may have to sign up for WebMD
and Uptodate. Google Scholar requires no signup.
3. Flag a Question: Medical standards and drug standards change. Also,
human errors can occur resulting in Questions that have issues. Knowledge
Testing strongly believes in Continuous Quality Improvement (CQI).
Flagging a question gives you, the medical professional, the ability to tell us
if something is wrong with a question and we will correct it and get back to
you with an updated book (providing you give us a phone number or email
address.) If you don't provide us with contact information, you can check
for a book update using the normal methods from the books store and you
will soon find an update with the question corrected by one of our health
professionals.
4. Volume of Questions: When we make a book we publish every question
we have on a topic (up to 6,000 per book). This would be ridiculously
expensive to do with traditional book publishing, but in the eBook world it
costs no more to publish 6,000 questions than 1,000 questions. You will
find that in most of our 177 specialties, we dwarf the competition with
breadth of material. This makes our books "just what the doctor ordered"
for those wishing to go the extra mile when preparing for your Board
Review, Certification Review, or other test.

KNOWLEDGE TESTING AUTHORS AND EDITORS

Over 1,000 physician, nursing, and allied health professionals authored a


database of over a million classified multiple-choice questions and explanations.
Each question has been peer reviewed by two health professionals and a
pharmacist. All questions are reviewed annually and updated through a peer
review process.

WE APPRECIATE YOUR COMMENTS!

We appreciate comments, suggestions corrections and additional contributing


authors and editors. Please e-mail us at: support@knowledgetesting.com.
Cardiothoracic Surgery Board
Review
Edited by Josh Helman

KnowledgeTesting Book ID: MD_SURGERY_CARDIAC-2015-01-06

Version: 1/6/2015
Table of Contents
Introduction

Section 1 ( Questions 1 - 200 )

Section 2 ( Questions 201 - 400 )

Section 3 ( Questions 401 - 600 )

Section 4 ( Questions 601 - 800 )

Section 5 ( Questions 801 - 1000 )

Section 6 ( Questions 1001 - 1200 )

Section 7 ( Questions 1201 - 1400 )

Section 8 ( Questions 1401 - 1600 )

Section 9 ( Questions 1601 - 1800 )

Section 10 ( Questions 1801 - 2000 )

About Knowledge Testing


Section 1
Question 1: Which statement is false about local anesthetics?
Choices:
1. They have CNS toxic effects
2. They have cardiovascular effects
3. They cause depression of the SA node
4. First sign of CNS effect is seizure
Answer: 4 - First sign of CNS effect is seizure
Explanations:
The first sign of toxic CNS effects of local anesthetics is perioral numbness.

Go to the next page if you knew the correct answer, or click the link images
below to further research the concepts in this question (if desired).

Research Concepts:
Local Anesthetics:

Tap flag to report any problems with this question.


Question 2: A 66-year-old elderly female is admitted for pneumonia in a
debilitated state. She is started on IV vancomycin and low molecular weight
heparin subcutaneously for DVT prophylaxis. On day seven of her admission,
she complains of left-sided calf pain. A subsequent Doppler confirms the
presence of a deep vein thrombosis, but her platelet count has now dropped to
33,000/mm3. Which of the following is the next step in her management?

Choices:
1. Change LMWH to unfractionated heparin
2. Give one dose of pulsed IV methylprednisolone
3. Start on coumadin
4. Discontinue all heparin products and start lepirudin
Answer: 4 - Discontinue all heparin products and start lepirudin
Explanations:
Heparin-induced thrombocytopenia is a rare but serious immune
thrombocytopenia. It is a complication of heparin therapy and is,
paradoxically, associated with venous thrombosis in approximately half of
patients.
Treatment involves the removal of all heparin products. Because this patient
has thrombosis, it is appropriate to switch to an alternative anticoagulation
strategy involving lepirudin, bivalirudin, or argatroban.
Warfarin does not act quickly enough to prevent DVT extension and
complications.

Go to the next page if you knew the correct answer, or click the link images
below to further research the concepts in this question (if desired).

Research Concepts:
Thrombocytopenia, Heparin Induced:

Tap flag to report any problems with this question.


Question 3: Which of the following managements is inappropriate for a
patient with right ventricular infarct?

Choices:
1. Maintain ventricular preload
2. Lower afterload
3. Reestablish early reperfusion
4. Venous dilation
Answer: 4 - Venous dilation
Explanations:
The primary goal for right ventricular infarct is to reestablish perfusion
Also to lower after load and maintain preload pressure
Preload reduction with venous dilator is inappropriate

Go to the next page if you knew the correct answer, or click the link images
below to further research the concepts in this question (if desired).

Research Concepts:
Right Ventricular Infarction:

Tap flag to report any problems with this question.


Question 4: Which is the best method to evaluate pericardial effusion?
Choices:
1. CT scan
2. Pericardial window
3. Echocardiogram
4. Physical exam
Answer: 3 - Echocardiogram
Explanations:
Echocardiogram is the best method to diagnose pericardial effusion
It will also reveal any structural abnormalities

Go to the next page if you knew the correct answer, or click the link images
below to further research the concepts in this question (if desired).

Research Concepts:
Pericardial Effusion:

Tap flag to report any problems with this question.


Question 5: Medicare covers what patient population?
Choices:
1. Low income
2. Children
3. Students
4. Elderly
Answer: 4 - Elderly
Explanations:
Medicaid is designed to help low income families
It is funded by the state
Medicare is for elderly population

Go to the next page if you knew the correct answer, or click the link images
below to further research the concepts in this question (if desired).

Research Concepts:
Medicare Coverage:

Tap flag to report any problems with this question.


Question 6: Which endocrine system is often affected by etomidate?
Choices:
1. Blood sugar
2. Thyroid hormone
3. Adrenal gland
4. Posterior pituitary
Answer: 3 - Adrenal gland
Explanations:
Etomidate suppresses corticosteroid synthesis in the adrenal cortex.
There is some evidence that prolonged use of etomidate in the ICU is
associated with increased mortality because of adrenal suppression.

Go to the next page if you knew the correct answer, or click the link images
below to further research the concepts in this question (if desired).

Research Concepts:
Adrenal Glands:

Etomidate:

Tap flag to report any problems with this question.


Question 7: What is true about Cheyne-Stokes breathing?
Choices:
1. It is due to fixed high levels of carbon dioxide
2. It is mostly a COPD breathing pattern
3. Considered normal during sleep in children and the elderly
4. Alternating periods of hypoventilation and apnea
Answer: 3 - Considered normal during sleep in children and the elderly
Explanations:
Cheyne-Stokes respirations can be normal during sleep in children and the
elderly.
It is alternating hyperpnea and apnea.

Go to the next page if you knew the correct answer, or click the link images
below to further research the concepts in this question (if desired).

Research Concepts:
Cheyne-Stokes Respirations:

Tap flag to report any problems with this question.


Question 8: Which of the following is not caused by Histoplasmosis?
Choices:
1. Fibrosing mediastinitis
2. Broncholithiasis
3. Superior vena cava syndrome
4. Restrictive pericarditis
Answer: 4 - Restrictive pericarditis
Explanations:
Histoplasmosis causes fibrosing mediastinitis. Consolidated granulomas
cause obstruction of adjacent mediastinal structures.
The reactive fibrosis causes the superior vena cava syndrome, tracheo-
esophageal fistulas, and broncholiths eroding into the bronchus.
Symptoms of superior vena cava syndrome do not occur because the
process is slow and leads to development of collaterals. Pericarditis is rare
but can occur.
It may present as either an effusion or tamponade. The pericardial
pathology is a constrictive process not restrictive.

Go to the next page if you knew the correct answer, or click the link images
below to further research the concepts in this question (if desired).

Research Concepts:
Histoplasmosis:

Tap flag to report any problems with this question.


Question 9: An infant is being treated with indomethacin (Indocin) for a
patent ductus arteriosus (PDA). Which finding should be reported immediately?

Choices:
1. Bleeding from prior heel pricks
2. Capillary refill of less than 2 seconds
3. Presence of a faint murmur that sounds machinery-like, which is unchanged
from previous assessment
4. Respiratory rate of 40 breaths per minute and heart rate of 130 beats per
minute
Answer: 1 - Bleeding from prior heel pricks
Explanations:
Bleeding from prior heel prick sites should be reported to the physician
immediately. Indomethacin is a non-steroidal anti-inflammatory agent used
as an alternative to surgery for patients with a patent ductus arteriosus.
Indomethacin inhibits platelet function. Therefore, the nurse should observe
for bleeding. Capillary refill of less than 2 seconds indicates good
peripheral perfusion and low probability of congestive heart failure.
The presence of a faint machinery-like murmur is typical of a patent ductus
arteriosus (PDA). A respiratory rate of 40 breaths per minute is within
normal limits (30 to 50) for an infant.
A heart rate of 130 beats per minute is within normal limits (120 to 160) for
an infant.

Go to the next page if you knew the correct answer, or click the link images
below to further research the concepts in this question (if desired).

Research Concepts:
Indomethacin:

Patent Ductus Arteriosus:


Tap flag to report any problems with this question.
Question 10: In what percentage of cases in pregnancy do the classic triad
seen in pulmonary embolism of hemoptysis, pleuritic chest pain, and dyspnea
occur?

Choices:
1. 10%
2. 20%
3. 50%
4. 75%
Answer: 2 - 20%
Explanations:
The classic triad: hemoptysis, pleuritic chest pain, and dyspnea-appear in
only 20% of cases. The most common sign on physical examination is
tachypnea (> 16 breaths/min)

Go to the next page if you knew the correct answer, or click the link images
below to further research the concepts in this question (if desired).

Research Concepts:
Pulmonary Embolism:

Tap flag to report any problems with this question.


Question 11: In which of the following west zones must the PA catheter be
placed for optimum readings?

Choices:
1. Zone 1
2. Zone 2
3. Zone 3
4. Zone 4
Answer: 3 - Zone 3
Explanations:
In Zone III, the pulmonary artery pressure is greater than the pulmonary
venous pressure which is greater than the pulmonary alveolar pressure.

Go to the next page if you knew the correct answer, or click the link images
below to further research the concepts in this question (if desired).

Research Concepts:
Pulmonary Artery Catheterization:

Tap flag to report any problems with this question.


Question 12: When should coronary bypass surgery be recommended in a
65 year old?

Choices:
1. Presence of coronary artery disease
2. Presence of left main stenosis more than 50%
3. Single vessel disease in the right coronary artery
4. Two vessel coronary disease with normal LV function
Answer: 2 - Presence of left main stenosis more than 50%
Explanations:
Indications for open heart surgery include left main stenosis > 50%, 2-3
vessel coronary artery disease, and LV dysfunction.
In a patient with severe symptoms despite maximal medical therapy,
surgery should be recommended.
Surgery should be recommended in the presence of poor LV function.
Surgery should be recommended if there are signs of LV dysfunction on
non-invasive testing and evidence of coronary artery disease.

Go to the next page if you knew the correct answer, or click the link images
below to further research the concepts in this question (if desired).

Research Concepts:
Revascularization Procedures In Coronary Artery
Disease:

Tap flag to report any problems with this question.


Question 13: Which of the following statements is NOT true about REDO
surgery for CABG?

Choices:
1. Totally occluded vein grafts are rarely source of emboli
2. Patent vein grafts are a source of emboli
3. Previous grafts should be dissected first out to prevent prolonged time on
CPB
4. All anastomosis should be done with a single cross clamp
Answer: 3 - Previous grafts should be dissected first out to prevent
prolonged time on CPB

Explanations:
It is easier and safer to reconstruct all the anastomosis on a single cross
clamp in REDO surgery.
REDO mortality is reduced by minimal handling of vein grafts and all
patent vein grafts should be minimally handled.
Prompt ligation of grafts before cardioplegia infusion should be done.

Go to the next page if you knew the correct answer, or click the link images
below to further research the concepts in this question (if desired).

Research Concepts:
Redo CABG:

Coronary Artery Bypass Graft:

Tap flag to report any problems with this question.


Question 14: Postganglionic sympathetic fibers use which of the following
agents for neuron transmission?

Choices:
1. Glutamate
2. Norepinephrine
3. Acetylcholine
4. Epinephrine
Answer: 2 - Norepinephrine
Explanations:
Postganglionic sympathetic fibers use norepinephrine for transmission
Glutamate is the major excitatory neurotransmitter in the CNS
Acetylcholine is a major transmitter of parasympathetic system and
neuromuscular junction
Epinephrine is released from adrenal medulla, which is a modified
postganglionic cell

Go to the next page if you knew the correct answer, or click the link images
below to further research the concepts in this question (if desired).

Research Concepts:
Sympathetic Nervous System:

Norepinephrine:

Tap flag to report any problems with this question.


Question 15: Which of the following does not lead to a decrease in surgical
site infections?

Choices:
1. Infusion of leukocyte-enriched blood
2. Maintaining blood sugar levels at less than 150 mg/dl
3. Keeping oxygen saturation above 95%
4. Not shaving the night before surgery
Answer: 1 - Infusion of leukocyte-enriched blood
Explanations:
Leukocyte filtration, or leukocyte-enriched blood, has made no difference
in surgical site infections.
The best way to reduce infection rates is to control blood sugar, provide
adequate oxygen, maintain core body temperature above 36.5 degrees
Celsius, and refrain from shaving the night before surgery.

Go to the next page if you knew the correct answer, or click the link images
below to further research the concepts in this question (if desired).

Research Concepts:
Wound Infection:

Tap flag to report any problems with this question.


Question 16: Which statement about banked blood is NOT correct?
Choices:
1. Citrate phosphate is added to bind to the calcium
2. Phosphate slows down breakdown of 2, 3, DPG
3. Levels of nitric oxide increase with time of storage
4. The risks of transfusion include viral infection
Answer: 3 - Levels of nitric oxide increase with time of storage
Explanations:
As soon as blood is donated, it loses it ability to retain nitric oxide. This can
cause a patient to develop a heart attack or a stroke.
Without nitric oxide, the blood vessels cannot open up and the tissues fail to
receive oxygenated blood.
Factor 8 also gets depleted in banked blood.
Levels of carboxy-hemoglobin do increase with time in banked blood.

Go to the next page if you knew the correct answer, or click the link images
below to further research the concepts in this question (if desired).

Research Concepts:
Blood Transfusion:

Tap flag to report any problems with this question.


Question 17: Select the most common complication of acute myocardial
infarction.

Choices:
1. Ventricular aneurysm
2. Cardiogenic shock
3. Arrhythmias
4. Pericarditis
Answer: 3 - Arrhythmias
Explanations:
90% of acute myocardial infarction patients have arrhythmias
Congestive failure is present in about 60%, pulmonary embolism in about
30%, cardiogenic shock in 10 to 15%, pericarditis in 15%, and ventricular
aneurysm in 3 to 20%

Go to the next page if you knew the correct answer, or click the link images
below to further research the concepts in this question (if desired).

Research Concepts:
Acute Myocardial Infarction:

Arrhythmias:

Tap flag to report any problems with this question.


Question 18: Which is least likely to cause pericardial effusion?
Choices:
1. Systemic lupus erythematosus, chronic renal failure
2. Hypertension
3. Lung cancer
4. AIDS, trauma, aortic dissection
Answer: 2 - Hypertension
Explanations:
Hypertension is least likely cause pericardial effusion

Go to the next page if you knew the correct answer, or click the link images
below to further research the concepts in this question (if desired).

Research Concepts:
Pericardial Effusion:

Tap flag to report any problems with this question.


Question 19: Bosentan mimics actions of what molecule?
Choices:
1. Angiotensin
2. Bradykinin
3. Nitric oxide
4. Prostaglandins
Answer: 4 - Prostaglandins
Explanations:
Bosentan is a duel endothelial receptor antagonist used to treat pulmonary
hypertension.
Bosentan acts at the endothelial receptor and dilates the pulmonary arteries.
The drug is now being investigated for its use in systemic sclerosis and
digital ulcer disease.

Go to the next page if you knew the correct answer, or click the link images
below to further research the concepts in this question (if desired).

Research Concepts:
Pulmonary Hypertension, Primary:

Tap flag to report any problems with this question.


Question 20: Why are phosphates are given to treat hypercalcemia?
Choices:
1. Increase excretion
2. Decrease absorption
3. Increase bone uptake
4. Increase binding
Answer: 4 - Increase binding
Explanations:
Steroids decrease gastrointestinal absorption of calcium
Plicamycin increases bone up take of calcium
Phosphates bind serum calcium

Go to the next page if you knew the correct answer, or click the link images
below to further research the concepts in this question (if desired).

Research Concepts:
Hypercalcemia:

Tap flag to report any problems with this question.


Question 21: How often should PT/INR be checked in reliable patients on
warfarin with a stable INR who have a good understanding of the medication
and dietary implications?

Choices:
1. Every 2 weeks
2. Every 4 months
3. Every month
4. Every 6 months
Answer: 3 - Every month
Explanations:
Every month, although studies are looking at extending that time

Go to the next page if you knew the correct answer, or click the link images
below to further research the concepts in this question (if desired).

Research Concepts:
Warfarin:

International Normalized Ratio (INR):

Tap flag to report any problems with this question.


Question 22: A 65-year-old patient presents with basilar rales and is also
found to have cool, clammy skin, elevated jugular venous pressure, rales and a
third heart sound. The physical findings are best characterized as which of the
following?

Choices:
1. Dry and cold
2. Wet and warm
3. Dry and warm
4. Wet and cold
Answer: 4 - Wet and cold
Explanations:
In CHF, there is fluid accumulation in the body and the perfusion of organs
is decreased.
Physical findings in CHF include elevated JVP, basilar rales, tachycardia,
pedal edema and cool extremities.
Such patients are usually described ad wet and cold. The skin may appear
dusky and feel cold to touch.
Auscultation may reveal and S3 or a gallop.

Go to the next page if you knew the correct answer, or click the link images
below to further research the concepts in this question (if desired).

Research Concepts:
Heart Failure, Congestive:

Tap flag to report any problems with this question.


Question 23: Which therapy is not indicated in the acute management of
congestive heart failure caused by a left-to-right shunt?

Choices:
1. Oxygen
2. Digoxin
3. Furosemide
4. Dobutamine
Answer: 1 - Oxygen
Explanations:
Supplemental oxygen may exacerbate heart failure that is caused by a left-
to-right shunt by decreasing pulmonary vascular resistance and increasing
the left-to-right shunting
The acute management of heart failure due to a left-to-right shunt includes
inotropic support and diuresis
The definitive treatment is to eliminate the left-to-the right shunt with either
interventional cardiac catheterization or surgery

Go to the next page if you knew the correct answer, or click the link images
below to further research the concepts in this question (if desired).

Research Concepts:
Heart Failure, Congestive:

Congenital Heart Disease:

Tap flag to report any problems with this question.


Question 24: Which is not a modifiable risk factor for coronary artery
disease?

Choices:
1. Hypertension
2. Smoking
3. Family history
4. Obesity
Answer: 3 - Family history
Explanations:
All of the options are risk factors for coronary artery disease.
Hypertension, smoking, and obesity are modifiable.

Go to the next page if you knew the correct answer, or click the link images
below to further research the concepts in this question (if desired).

Research Concepts:
Risk Factors For Coronary Artery Disease:

Tap flag to report any problems with this question.


Question 25: The antihypertensive agent associated with depression is:
Choices:
1. Propranolol
2. Clonidine
3. Verapamil
4. Lisinopril
Answer: 1 - Propranolol
Explanations:
Beta blockers are associated with depression and erectile dysfunction

Go to the next page if you knew the correct answer, or click the link images
below to further research the concepts in this question (if desired).

Research Concepts:
Beta-Blockers:

Tap flag to report any problems with this question.


Question 26: A patient with congestive heart failure is prescribed
furosemide (furosemide) 20 mg po every morning. The reason for this
prescription is to:

Choices:
1. Prevent hyperkalemia
2. Diurese the patient of extra fluid while he is awake
3. Increase myocardial contractility
4. Help wake the patient up in the morning
Answer: 2 - Diurese the patient of extra fluid while he is awake
Explanations:
When diuretics are given early in the day, the patient will void frequently
during the daytime hours and will not need to void frequently during the
night

Go to the next page if you knew the correct answer, or click the link images
below to further research the concepts in this question (if desired).

Research Concepts:
Heart Failure, Congestive:

Tap flag to report any problems with this question.


Question 27: Which of the following can cause atelectasis?
Choices:
1. Tumor
2. Mucus plug
3. Foreign body
4. All of the above
Answer: 4 - All of the above
Explanations:
Obstructive atelectasis can occur at any level of the trachea or bronchus.
Typical causes of atelectasis include tumor, foreign body, or mucus plug.
Direct signs of atelectasis include opacification of a collapsed lobe and
fissural displacement.
Indirect signs of atelectasis include hilar displacement, ipsilateral
mediastinal shift, ipsilateral volume loss, ipsilateral diaphragm elevation,
adjacent rib crowding, and compensatory hyperlucency of the non-
atelectatic lung.

Go to the next page if you knew the correct answer, or click the link images
below to further research the concepts in this question (if desired).

Research Concepts:
Atelectasis:

Tap flag to report any problems with this question.


Question 28: What is the most effective way to prevent postoperative
respiratory problems?

Choices:
1. Incentive spirometry
2. Bronchoscopy
3. Bronchodilator therapy
4. Effective pain control
Answer: 1 - Incentive spirometry
Explanations:
In preventing atelectasis, incentive spirometry is the most efficient way to
reduce respiratory complications.
Incentive spirometry should be taught to all patients before surgery and
encouraged soon after surgery.

Go to the next page if you knew the correct answer, or click the link images
below to further research the concepts in this question (if desired).

Research Concepts:
Post-op Respiratory Therapy:

Postoperative Recovery:

Tap flag to report any problems with this question.


Question 29: A 6 year old girl is noted to have a grade II/VI vibratory,
midsystolic murmur at the left midsternal border without radiation. What is the
most probable diagnosis?

Choices:
1. Mitral valve prolapse
2. Innocent murmur
3. Pulmonic stenosis
4. Aortic regurgitation
Answer: 2 - Innocent murmur
Explanations:
Innocent murmurs are most common in children from 3 to 7 years
They are often musical or vibratory, short, systolic ejection murmurs at the
left midsternal or low sternal border
There is no radiation

Go to the next page if you knew the correct answer, or click the link images
below to further research the concepts in this question (if desired).

Research Concepts:
Innocent Murmur:

Tap flag to report any problems with this question.


Question 30: Which analgesic is not used in patient controlled analgesia
pumps?

Choices:
1. Fentanyl
2. Morphine
3. Hydromorphone
4. Oxycodone
Answer: 4 - Oxycodone
Explanations:
Fentanyl is a good analgesic for PCA
Morphine has multiple adverse effects
Hydromorphone has high potency
Oxycodone is not commonly used

Go to the next page if you knew the correct answer, or click the link images
below to further research the concepts in this question (if desired).

Research Concepts:
Patient Controlled Anesthesia (PCA):

Tap flag to report any problems with this question.


Question 31: Which of the following is not used to ensure correct
placement of an endotracheal tube?

Choices:
1. Ultrasound
2. Bilateral breath sounds
3. Sustained end-tidal CO2
4. Sustained end-tidal O2
Answer: 4 - Sustained end-tidal O2
Explanations:
After ET tube placement, breath sounds should be present bilaterally and
absent at the stomach
Ultrasound can be used to confirm placement
Condensation in the tube indicates placement in the trachea and not the
stomach
Sustained end-tidal O2 is not a method used to evaluate placement

Go to the next page if you knew the correct answer, or click the link images
below to further research the concepts in this question (if desired).

Research Concepts:
Endotracheal Tube:

Tap flag to report any problems with this question.


Question 32: An 18-month-old asymptomatic child is found to have a
systolic murmur, upper extremity hypertension, and diminished femoral pulses.
Which of the following is most likely true about this child's disorder?

Choices:
1. Atrial septal defects
2. Atherosclerotic disease
3. Rib notching on x-ray
4. Claudication on walking
Answer: 3 - Rib notching on x-ray
Explanations:
The physical exam is consistent with the congenital anomaly coarctation of
the aorta. There is most often narrowing of the aorta just distal to the left
subclavian artery.
The intercostal vessels provide collateral circulation causing x-ray findings
of rib notching. If not surgically corrected, complications in adults include
death from cardiac failure, bacterial endocarditis, and rupture of aortic
aneurysms or of a cerebral artery.
Ventricular septal defects and aortic stenosis are often seen in association
with coarctation.
Claudication is a symptom of peripheral vascular .

Go to the next page if you knew the correct answer, or click the link images
below to further research the concepts in this question (if desired).

Research Concepts:
Aortic Coarctation:

Tap flag to report any problems with this question.


Question 33: What needs to be taught to a patient receiving an implantable
cardioverter defibrillator?

Choices:
1. They will need to know how to activate EMS in an emergency
2. They will need to know how many joules they can be defibrillated with
3. Patients will need to resume activities slowly following surgery
4. Patients will need to carry extra batteries at all times
Answer: 1 - They will need to know how to activate EMS in an emergency
Explanations:
Implanted defibrillators may not always capture, causing the patient to need
EMS services in event of emergency

Go to the next page if you knew the correct answer, or click the link images
below to further research the concepts in this question (if desired).

Research Concepts:
Implanted Cardiac Defibrillator Malfunction:

Tap flag to report any problems with this question.


Question 34: After an uneventful coronary artery bypass, the patient
complains of loss of sensation in his leg from where he had the vein taken.
Which of the following is the most likely etiology?

Choices:
1. Damage to sural nerve
2. Damage to saphenous nerve
3. Damage to femoral nerve
4. Damage to sciatic nerve
Answer: 2 - Damage to saphenous nerve
Explanations:
Damage to the saphenous nerve will result in loss of sensation to the medial
aspect of the leg.

Go to the next page if you knew the correct answer, or click the link images
below to further research the concepts in this question (if desired).

Research Concepts:
Saphenous Vein Grafts:

Tap flag to report any problems with this question.


Question 35: A post operative patient with a developmentally delayed child
at home should be referred to which professional?

Choices:
1. Psychologist
2. Home health nurse
3. Physical therapist
4. Social worker
Answer: 4 - Social worker
Explanations:
A social worker can aid the patient in coordinating care for her child while
she is recovering.

Go to the next page if you knew the correct answer, or click the link images
below to further research the concepts in this question (if desired).

Research Concepts:
Postoperative Care:

Tap flag to report any problems with this question.


Question 36: Cardioversion delivery is synchronized with:
Choices:
1. P wave
2. R wave
3. S wave
4. T wave
Answer: 2 - R wave
Explanations:
Cardioversion is synchronized with the R wave
VT is shock with nonsynchronized energy

Go to the next page if you knew the correct answer, or click the link images
below to further research the concepts in this question (if desired).

Research Concepts:
Synchronized Electrical Cardioversion:

Tap flag to report any problems with this question.


Question 37: An 18-year-old Asian female suddenly dies and an autopsy
reveals a small jelly-like mass in the left atrium. Prior symptoms included a low-
grade fever, weight loss, and mild shortness of breath. The mass appears fibrous
with papillary projections and an organized thrombus. It appears to be growing
all over the mitral valve. There is no evidence of a malignancy. She may have
which of the following conditions?

Choices:
1. Endocarditis
2. Cardiac myxoma
3. Rheumatic fever
4. Ruptured chordae
Answer: 2 - Cardiac myxoma
Explanations:
Myxomas are most common in young Asian females and the majority occur
in the left atrium.
Gross features will reveals a jelly-like mass with papillary extensions with
an organized thrombus. The mass can produce a ball valve effect on the
mitral valve orifice.
The myxoma may produce constitutional symptoms and can be mistaken
for vegetation from endocarditis.
Often the mass can embolize and cause stroke. Sudden deaths are
uncommon.

Go to the next page if you knew the correct answer, or click the link images
below to further research the concepts in this question (if desired).

Research Concepts:
Atrial Myxoma:

Tap flag to report any problems with this question.


Question 38: In a patient with acute cardiac tamponade, which of the
following should NOT be done?

Choices:
1. Give 100% oxygen
2. Leg elevation
3. Diuretic
4. Volume expansion
Answer: 3 - Diuretic
Explanations:
Patients with acute tamponade do not have blood being pumped out of the
heart because of the cardiac compression, and thus fluid is vital.
Other treatment measures include oxygen, bed rest, and leg elevation.
Immediate pericardiocentesis can be life saving.
Surgery requires making a hole in the pericardium and draining the fluid.

Go to the next page if you knew the correct answer, or click the link images
below to further research the concepts in this question (if desired).

Research Concepts:
Cardiac Tamponade:

Tap flag to report any problems with this question.


Question 39: Which of the following interventions should be avoided in a
patient with Wolff-Parkinson-White syndrome experiencing atrial fibrillation?

Choices:
1. Cardioversion
2. Verapamil
3. Procainamide
4. Amiodarone
Answer: 2 - Verapamil
Explanations:
Verapamil and digitalis may speed up conduction through the accessory
pathway.

Go to the next page if you knew the correct answer, or click the link images
below to further research the concepts in this question (if desired).

Research Concepts:
Atrial Fibrillation:

Tap flag to report any problems with this question.


Question 40: Which of following is commonly seen in cardiogenic shock?
Choices:
1. Hypertension
2. Hyperthermia
3. Decreased urine output
4. Fever
Answer: 3 - Decreased urine output
Explanations:
Oliguria occurs during cardiogenic shock because there is reduced blood
flow to the kidneys.

Go to the next page if you knew the correct answer, or click the link images
below to further research the concepts in this question (if desired).

Research Concepts:
Cardiogenic Shock:

Tap flag to report any problems with this question.


Question 41: A 62 year-old man is booked for open-heart surgery. He has
osteoarthritis for which he takes NSAIDs. Which of the following is the most
appropriate treatment prior to surgery to minimize the risk of bleeding from
NSAID use?

Choices:
1. Begin vitamin K 1 week prior to surgery
2. Give FFP few hours before surgery
3. Stop the NSAIDs 3 weeks prior to surgery
4. Stop the NSAIDs 1 week prior to surgery
Answer: 4 - Stop the NSAIDs 1 week prior to surgery
Explanations:
The patient would not benefit from vitamin K since he does not have
abnormal clotting factors.
The patient would not benefit from FFP since he does not have abnormal
clotting factors.
NSAIDs block platelet function by causing a reversible defect in the
enzyme cyclooxygenase. Unlike aspirin, which permanently acetylates
cyclo-oxygenase and leaves affected platelets dysfunctional throughout
their 7-day life span, NSAIDs cause a reversible defect
NSAIDs with short half lives can be stopped 3 or 4 days before surgery
while those with long half lives require a week

Go to the next page if you knew the correct answer, or click the link images
below to further research the concepts in this question (if desired).

Research Concepts:
NSAIDS:

Preoperative Evaluation And Management:

Tap flag to report any problems with this question.


Question 42: What does prothrombin time measure?
Choices:
1. Intrinsic pathway
2. Extrinsic pathway
3. Platelet clumping time
4. Bleeding time
Answer: 2 - Extrinsic pathway
Explanations:
Prothrombin time evaluates the extrinsic coagulation pathway.
The intrinsic pathway is a measure of PTT, or partial thromboplastin time.

Go to the next page if you knew the correct answer, or click the link images
below to further research the concepts in this question (if desired).

Research Concepts:
Prothrombin Time:

Tap flag to report any problems with this question.


Question 43: Which anti-hypertensive agent is associated with hemolytic
crises?

Choices:
1. Alpha methyldopa
2. Clonidine
3. Propranolol
4. Hydralazine
Answer: 1 - Alpha methyldopa
Explanations:
Alpha methyldopa (Aldomet) is a centrally acting agent used in the
treatment of essential hypertension.
Alpha methyldopa is associated with hemolysis, which is believed to be
immune mediated.

Go to the next page if you knew the correct answer, or click the link images
below to further research the concepts in this question (if desired).

Research Concepts:
Antihypertensive Medications:

Tap flag to report any problems with this question.


Question 44: Which of the following medications may impair glycemic
control in diabetic patients?

Choices:
1. Hydrochlorothiazide
2. Verapamil
3. Enalapril
4. Furosemide
Answer: 1 - Hydrochlorothiazide
Explanations:
Thiazide diuretics should be used with caution in diabetic patients
Insulin resistance and hyperglycemia may result from thiazide diuretic
therapy
ACE inhibitors are very helpful in diabetic patients as they decrease the risk
of nephropathy

Go to the next page if you knew the correct answer, or click the link images
below to further research the concepts in this question (if desired).

Research Concepts:
Thiazide Diuretics:

Insulin Resistance:

Tap flag to report any problems with this question.


Question 45: Which statement is true about tetralogy of Fallot?
Choices:
1. The shunt is predominantly from left to right
2. Squatting worsens cyanosis
3. ECG will reveal left ventricular hypertrophy
4. There is usually cyanosis that presents at 6 weeks to 6 months
Answer: 4 - There is usually cyanosis that presents at 6 weeks to 6 months
Explanations:
Children with TOF instinctively squat or assume the knee chest position to
increase oxygenation.
By increasing the intra abdominal pressure, a brief but non-sustained rise in
venous return to the heart is produced
At the same time, an increase in systemic arterial resistance favors
pulmonary rather than systemic flow
The shunt is right to left. The ECG shows right ventricular hypertrophy.

Go to the next page if you knew the correct answer, or click the link images
below to further research the concepts in this question (if desired).

Research Concepts:
Tetralogy Of Fallot:

Tap flag to report any problems with this question.


Question 46: Which of the following is an acceptable reason to disclose
Protected Health Information (PHI).

Choices:
1. Child or elder abuse
2. Request by a family member
3. Professional disputes
4. By request of an attorney
Answer: 1 - Child or elder abuse
Explanations:
In cases of suspected child or elder abuse PHI must be disclosed as required
by law.
Facilitation of treatment, payment, or health care operations allows for
disclosure of Protected Health Information-PHI.
Authorization from the individual allows for PHI to be disclosed.
Only the pertinent and minimum information related to the covered entity
may be disclosed under Protected Health Information-PHI.

Go to the next page if you knew the correct answer, or click the link images
below to further research the concepts in this question (if desired).

Research Concepts:
Protected Health Information-PHI:

Tap flag to report any problems with this question.


Question 47: Which is not a risk factor for a lower extremity deep venous
thrombosis (DVT)?

Choices:
1. Hip surgery
2. Smoking
3. Ambulation
4. Dehydration
Answer: 3 - Ambulation
Explanations:
Ambulation helps to reduce the risk of a lower extremity DVT.
Risk factors for DVT include major bone surgery, smoking, bedrest, and
dehydration.

Go to the next page if you knew the correct answer, or click the link images
below to further research the concepts in this question (if desired).

Research Concepts:
Deep Venous Thrombosis, Lower Extremity:

Tap flag to report any problems with this question.


Question 48: What is the most common causative organism for infective
endocarditis (IE)?

Choices:
1. Streptococcus viridans
2. Staphylococcus aureus
3. Pseudomonas aeruginosa
4. HACEK
Answer: 2 - Staphylococcus aureus
Explanations:
S. aureus is the most common causative agent for bacterial endocarditis
It is 3X more common in males than females
Echocardiography has become the indirect diagnostic method of choice for
IE
Valvular vegetations on transthoracic or transesophageal echocardiograms
portend a worse prognosis

Go to the next page if you knew the correct answer, or click the link images
below to further research the concepts in this question (if desired).

Research Concepts:
Endocarditis, Bacterial:

Staphylococcus Aureus:

Tap flag to report any problems with this question.


Question 49: In addition to maintaining the body's water, the renal system
also balances the electrolyte concentration. Which of the following signs would
alert you to an elevation of your patient's serum potassium?

Choices:
1. Change in mental status, cardiac arrhythmias, and tall and peaked T waves
on the electrocardiogram
2. Changes in mental status, cardiac arrhythmias, and depressed or inverted T
waves on the electrocardiogram
3. Changes in mental status and muscle weakness or flaccidity
4. Cardiac arrhythmias and neuromuscular irritability
Answer: 1 - Change in mental status, cardiac arrhythmias, and tall and
peaked T waves on the electrocardiogram

Explanations:
Potassium affects the conduction of the myocardium and its contractibility.
As a result, markedly elevated serum potassium levels cause arrhythmia
(dysrhythmias), electrocardiogram changes (tall T waves), and changes in
mental status.
Changes in mental status or cardiac arrhythmias and depressed or inverted
T waves on the electrocardiogram are signs of hypokalemia, not
hyperkalemia. Changes in mental status or muscle weakness and flaccidity
are signs of hypercalcemia, not hyperkalemia.
Cardiac arrhythmias (dysrhythmias) and neuromuscular irritability are signs
of hypocalcemia and hypokalemia, not hyperkalemia
Patient Need: Physiological Integrity

Go to the next page if you knew the correct answer, or click the link images
below to further research the concepts in this question (if desired).

Research Concepts:
Hyperkalemia:

Tap flag to report any problems with this question.


Question 50: Which of the following ECG findings are inconsistent with
hyperkalemia?

Choices:
1. Loss of the P wave
2. Widening of the QRS complex
3. Peaked T wave
4. Shortened PR interval
Answer: 4 - Shortened PR interval
Explanations:
The ECG progression of hyperkalemia is initially an increase in the T wave
amplitude.
It then causes a prolonged PR interval and loss of P waves.
Widening of the QRS complex occurs next.
Sine wave pattern occurs when the QRS complex merges with the T wave,
followed by asystole.

Go to the next page if you knew the correct answer, or click the link images
below to further research the concepts in this question (if desired).

Research Concepts:
Hyperkalemia:

Electrocardiogram:

Tap flag to report any problems with this question.


Question 51: A child with Down syndrome and atrioventricular canal
defect will have surgery when she reaches 7 kg but she is having difficulty
gaining weight. Which of the following is not a reasonable plan of management?

Choices:
1. Adding NG feedings at night
2. Explaining that patients with Down syndrome often have difficulty with
breast feeding but not bottle feeding
3. Weigh the infant before and after breast-feeding and observe the feeding
4. Offer pumped breast milk by bottle
Answer: 2 - Explaining that patients with Down syndrome often have
difficulty with breast feeding but not bottle feeding

Explanations:
Infants with Down syndrome may have feeding difficulties by breast or
bottle
In addition, children with congenital heart disease have feeding difficulties
Involvement and communication is needed between the family, primary
provider, cardiologist, and developmental therapist

Go to the next page if you knew the correct answer, or click the link images
below to further research the concepts in this question (if desired).

Research Concepts:
Atrioventricular Canal Defects:

Down Syndrome:

Tap flag to report any problems with this question.


Question 52: Which is true of pulmonary embolism (PE)?
Choices:
1. It never occurs in children
2. It can cause pulmonary infarction
3. The diagnosis is easily made based on ECG
4. Most chest radiographs obtained in acute PE are abnormal, though often
non-specific
Answer: 2 - It can cause pulmonary infarction
Explanations:
Presentation with PE is based on the severity of pulmonary artery
occlusion. Categories include massive embolus, acute pulmonary infarct, or
multiple pulmonary emboli.
Acute pulmonary infarction occurs in 10% of PE patients. It presents with
pleuritic chest pain, hemoptysis, and dyspnea.
The most common symptoms in the PIOPED study were dyspnea, pleuritic
chest pain, cough, and hemoptysis.
Pulmonary infarction following embolism is seen as a peripheral wedge
shaped pleural-based opacification on a chest CT. However, it is well
known that most chest radiographs obtained in patients with acute PE are
normal.

Go to the next page if you knew the correct answer, or click the link images
below to further research the concepts in this question (if desired).

Research Concepts:
Pulmonary Embolism:

Tap flag to report any problems with this question.


Question 53: Which beta blocker can be used in a patient with asthma and
hypertension?

Choices:
1. Pindolol
2. Propranolol
3. Phenoxybenzamine
4. Esmolol
Answer: 1 - Pindolol
Explanations:
Pindolol has partial agonist sympathetic activity, which is useful in the
treatment of patients with high blood pressure and asthma.
The intrinsic sympathetic activity of pindolol helps to dilate the
bronchioles.
In general, beta blockers are avoided with asthma.

Go to the next page if you knew the correct answer, or click the link images
below to further research the concepts in this question (if desired).

Research Concepts:
Hypertension:

Asthma:

Tap flag to report any problems with this question.


Question 54: Patient was found to be hypotensive, lethargic and diaphoretic
due morphine overdose. Breathing rate was 4-6/min and pulse oximetry showed
O2 sat-87%. What should be the initial treatment?

Choices:
1. Flumazenil
2. Oxygen by nasal cannula
3. Mechanical ventilation
4. Naloxone
Answer: 3 - Mechanical ventilation
Explanations:
This patient has respiratory failure due to depressed respiratory rate
The most urgent treatment is to intubate and ventilate until the morphine
wears off.
Mechanical ventilation may still be required even if naloxone is given.
Naloxone may require multiple doses.

Go to the next page if you knew the correct answer, or click the link images
below to further research the concepts in this question (if desired).

Research Concepts:
Opioid Toxicity:

Tap flag to report any problems with this question.


Question 55: What is the acute treatment for paroxysmal atrial tachycardia?
Choices:
1. Adenosine
2. Procainamide
3. Lidocaine
4. Verapamil
Answer: 1 - Adenosine
Explanations:
Adenosine is a very short acting AV node blocking agent and is indicated if
vagal maneuvers are not effective.
Other drugs that also work in treatment for PAT include calcium channel
blockers or metoprolol.
In pregnancy, adenosine is treatment of choice.
If patient is unstable, cardioversion may be used and is very effective.

Go to the next page if you knew the correct answer, or click the link images
below to further research the concepts in this question (if desired).

Research Concepts:
Paroxysmal Atrial Tachycardia:

Tap flag to report any problems with this question.


Question 56: Which is FALSE about bupivacaine?
Choices:
1. It is long acting
2. It is an amide
3. Toxicity is related to sodium channel blockage
4. It is degraded by cholinesterase
Answer: 4 - It is degraded by cholinesterase
Explanations:
The drug is an amide degraded by the liver.

Go to the next page if you knew the correct answer, or click the link images
below to further research the concepts in this question (if desired).

Research Concepts:
Bupivacaine:

Tap flag to report any problems with this question.


Question 57: The left ventricle wall is supplied by which artery?
Choices:
1. Right coronary artery
2. Left anterior descending artery
3. Coronary sinus
4. Left circumflex artery
Answer: 2 - Left anterior descending artery
Explanations:
The left anterior descending is a branch off the left coronary artery

Go to the next page if you knew the correct answer, or click the link images
below to further research the concepts in this question (if desired).

Research Concepts:
Heart Anatomy:

Tap flag to report any problems with this question.


Question 58: Which statement is most accurate regarding a chest tube
connected to a water seal drainage system?

Choices:
1. Bubbles in the water indicate that the chest tube is no longer needed
2. The water level should fall slightly with each spontaneous inspiration
3. The drainage system should be kept below the level of chest tube insertion
4. The chest tube should be clamped at all times when the patient is
ambulatory
Answer: 3 - The drainage system should be kept below the level of chest
tube insertion

Explanations:
Bubbles in the water arise from air in the pleural cavity.
They indicate that the chest tube is still necessary.
The water level in the tube should rise slightly with each spontaneous
inspiration.
The chest tube should generally remain open at all times; it is sometimes
clamped for a trial period prior to removal.

Go to the next page if you knew the correct answer, or click the link images
below to further research the concepts in this question (if desired).

Research Concepts:
Chest Tube:

Tap flag to report any problems with this question.


Question 59: Which lead is not important for diagnosis of an inferior wall
myocardial infarction?

Choices:
1. ll
2. lll
3. aVL
4. aVF
Answer: 3 - aVL
Explanations:
Diagnosis of an inferior wall MI based on the ECG is done by looking at
leads 2,3, and aVF

Go to the next page if you knew the correct answer, or click the link images
below to further research the concepts in this question (if desired).

Research Concepts:
Inferior Wall Myocardial Infarction:

Tap flag to report any problems with this question.


Question 60: The BEST test to study the aortic arch is:
Choices:
1. Transthoracic cardiac US
2. Transesophageal echocardiography (TEE)
3. CT angiography
4. Conventional angiography
Answer: 3 - CT angiography
Explanations:
Once viewed as the gold standard, the conventional angiogram is more
invasive, harder to obtain, and provides less information than the CT
equivalent.
One of the deficiencies of routine cardiac US is its limited view of the arch
and its vessels.
TEE has a better chance of evaluating the aortic arch, but it is clearly not as
informative as a CT.
MR angiography (a choice not offered) is competitive in its information to
CT and avoids ionizing radiation, but is technically more difficult to
perform and not as universally available 24/7.

Go to the next page if you knew the correct answer, or click the link images
below to further research the concepts in this question (if desired).

Research Concepts:
Aortic Arch:

Tap flag to report any problems with this question.


Question 61: What diagnostic test should be ordered for a patient who
develops fever and diarrhea after five days of antibiotic therapy?

Choices:
1. Colonoscopy with biopsy
2. Testing of stool for fecal leukocytes
3. Testing of stool for Clostridium difficile toxin
4. Gram stain of stool for curved rods
Answer: 3 - Testing of stool for Clostridium difficile toxin
Explanations:
Clostridium difficile is an aerobic, spore-forming bacillus that causes
pseudomembranous colitis due to overgrowth when the normal colonic
flora is reduced from antibiotic therapy
Fecal leukocytes are present in about half of cases and are nonspecific
Choice 4 is seen in Campylobacter jejuni infections
Pseudomembranes can be seen on colonoscopy but a less invasive test
should be ordered first

Go to the next page if you knew the correct answer, or click the link images
below to further research the concepts in this question (if desired).

Research Concepts:
Clostridium Difficile:

Tap flag to report any problems with this question.


Question 62: Which of the following is not true about morphine?
Choices:
1. Mu and kappa receptors are stimulated
2. Morphine causes ptosis
3. Morphine causes miosis
4. Morphine causes excitatory stimulation to the ventral tegmentum
Answer: 2 - Morphine causes ptosis
Explanations:
Morphine does not cause ptosis directly.
Morphine does stimulate mu and kappa receptors.
Morphine causes miosis.
Stimulation of the ventral tegmentum causes euphoria

Go to the next page if you knew the correct answer, or click the link images
below to further research the concepts in this question (if desired).

Research Concepts:
Morphine:

Tap flag to report any problems with this question.


Question 63: Select the medication that can cause necrotizing myopathy.
Choices:
1. Zidovudine
2. Colchicine
3. Simvastatin
4. Amiodarone
Answer: 3 - Simvastatin
Explanations:
HMG CoA reductase inhibitors can cause necrotizing myopathy. The
incidence is increased if combined with a fibrate
Antimicrotubular myopathy can occur with colchicine
Mitochondrial myopathy can be caused by zidovudine
Lysosomal related myopathy can be caused by amiodarone

Go to the next page if you knew the correct answer, or click the link images
below to further research the concepts in this question (if desired).

Research Concepts:
Statin Medication:

Necrotizing Myopathy:

Tap flag to report any problems with this question.


Question 64: Why is isoproterenol not indicated for a patient experiencing
a myocardial infarction?

Choices:
1. It acts on beta receptors to stimulate the heart
2. It acts on beta receptors to decrease respiratory rate
3. It can cause bradycardia
4. It can cause arrhythmias
Answer: 1 - It acts on beta receptors to stimulate the heart
Explanations:
Isoproterenol is a beta agonist that stimulates the heart, an action not
indicated in treatment of an MI

Go to the next page if you knew the correct answer, or click the link images
below to further research the concepts in this question (if desired).

Research Concepts:
Isoproterenol:

Acute Myocardial Infarction:

Tap flag to report any problems with this question.


Question 65: Which of the following is the least common site for invasive
arterial monitoring?

Choices:
1. Radial artery
2. Ulnar artery
3. Dorsalis pedis artery
4. Axillary artery
Answer: 4 - Axillary artery
Explanations:
The most common site for arterial monitoring is the radial artery, followed
by the ulnar
The least recommended site is the axillary artery
The dorsalis pedis artery should not be used in diabetics or those who have
peripheral vascular disease

Go to the next page if you knew the correct answer, or click the link images
below to further research the concepts in this question (if desired).

Research Concepts:
Arterial Cannulization:

Tap flag to report any problems with this question.


Question 66: A patient is receiving total parenteral nutrition through a
central line. A small amount of milky fluid is noted to be leaking at the site of
the central line dressing. Which of the following should be the initial action?

Choices:
1. Stop the infusion and change the dressing
2. Secure the junction where lipids are connected
3. Place the patient in a Trendelenburg position
4. Change the dressing when the infusion is completed
Answer: 3 - Place the patient in a Trendelenburg position
Explanations:
There is a break in the line that may cause air embolism
The patient should be placed on their left side with the head lower than the
feet
The infusion should then be stopped and the dressing changed
The break in the line is not at the junction but rather at the IV site

Go to the next page if you knew the correct answer, or click the link images
below to further research the concepts in this question (if desired).

Research Concepts:
Central Line Complications:

Tap flag to report any problems with this question.


Question 67: Which is the best description of orthostasis?
Choices:
1. A sensation of spinning
2. Lightheadedness upon standing
3. A sensation of loss of balance
4. A sensation of impending fainting
Answer: 2 - Lightheadedness upon standing
Explanations:
Orthostasis is lightheadedness upon standing or from sitting up from a
recumbent position.
Orthostatic hypotension is a common cause.
Presyncope is a sensation of impending fainting.
A sensation of spinning is vertigo.

Go to the next page if you knew the correct answer, or click the link images
below to further research the concepts in this question (if desired).

Research Concepts:
Orthostasis:

Tap flag to report any problems with this question.


Question 68: Which cephalosporin is usually given as a preoperative
antibiotic in elective cases?

Choices:
1. Ceftriaxone
2. Cefotetan
3. Cephalexin
4. Cefepime
Answer: 3 - Cephalexin
Explanations:
Cephalexin is usually administered 1 hr before the skin incision

Go to the next page if you knew the correct answer, or click the link images
below to further research the concepts in this question (if desired).

Research Concepts:
Cephalexin:

Preoperative Antibiotic Prophylaxis:

Tap flag to report any problems with this question.


Question 69: Structurally, what does digitalis glycosides resemble?
Choices:
1. Catecholamines
2. Steroids
3. Prostaglandins
4. Glycogen
Answer: 2 - Steroids
Explanations:
Digoxin has a structure that resembles a steroid.

Go to the next page if you knew the correct answer, or click the link images
below to further research the concepts in this question (if desired).

Research Concepts:
Digoxin:

Tap flag to report any problems with this question.


Question 70: How could starting an intravenous (IV) line without a
patient's permission be interpreted?

Choices:
1. Sexual harassment
2. Assault
3. Battery
4. None of the above
Answer: 3 - Battery
Explanations:
Starting an IV line without permission could be judged as battery
Battery is unpermitted physical contact
An assault is an intentional act that creates an apprehension of imminent
harmful contact
Sexual harassment does not pertain to this situation

Go to the next page if you knew the correct answer, or click the link images
below to further research the concepts in this question (if desired).

Research Concepts:
Patient Rights:

Tap flag to report any problems with this question.


Question 71: Select the factor that does not independently predict coronary
artery disease.

Choices:
1. Low HDL
2. High LDL
3. Obesity
4. Diabetes mellitus
Answer: 3 - Obesity
Explanations:
Obesity itself is not an independent risk factor for CAD
Other risk factors include hypertension, smoking, and family history
This said, multiple risk factors can be addressed by encouraging weight loss

Go to the next page if you knew the correct answer, or click the link images
below to further research the concepts in this question (if desired).

Research Concepts:
Risk Factors For Coronary Artery Disease:

Tap flag to report any problems with this question.


Question 72: When transesophageal echocardiogram is done to diagnose an
aortic dissection, the critical finding that leads to a diagnosis is:

Choices:
1. An enlarged aorta
2. An incompetent aortic valve
3. An intimal flap
4. Left ventricular function
Answer: 3 - An intimal flap
Explanations:
TEE is done to identify the intimal flap, and thus helps to distinguish the
dissection from an ordinary aortic aneurysm

Go to the next page if you knew the correct answer, or click the link images
below to further research the concepts in this question (if desired).

Research Concepts:
Dissection, Aortic:

Tap flag to report any problems with this question.


Question 73: What is the inotropic agent that inhibits phosphodiesterase?
Choices:
1. Digoxin
2. Quinidine
3. Amrinone
4. Dobutamine
Answer: 3 - Amrinone
Explanations:
Amrinone and milrinone are inotropic agents, which act by inhibiting
phosphodiesterase.
These agents increase the levels of cyclic AMP, which then stimulates the
heart.

Go to the next page if you knew the correct answer, or click the link images
below to further research the concepts in this question (if desired).

Research Concepts:
Amrinone:

Tap flag to report any problems with this question.


Question 74: Which of the following is NOT a requirement for competency
to refuse medical treatment?

Choices:
1. Drug screening
2. Ability to comprehend and communicate information
3. Ability to reason and deliberate about choices
4. Attainment of legal age
Answer: 1 - Drug screening
Explanations:
Competency to refuse medical treatment entails attainment of legal age,
ability to comprehend and communicate and ability to reason and deliberate
Adherence to religious beliefs does not make one incompetent
Only recourse in a competent patient refusing medical treatment is trying to
convince them of importance of treatment
Decision-making capacity is impaired by psychosis

Go to the next page if you knew the correct answer, or click the link images
below to further research the concepts in this question (if desired).

Research Concepts:
Refusal of Care:

Competency:

Tap flag to report any problems with this question.


Question 75: Your patient is receiving a loading dose of digoxin 0.5 mg po
in divided dosages over a 24-hour period. What is the most common and earliest
adverse reaction(s) to digoxin?

Choices:
1. Paresthesia
2. Nausea and anorexia
3. Dry eyes
4. Dizziness and vertigo
Answer: 2 - Nausea and anorexia
Explanations:
Nausea, vomiting, and anorexia are common early adverse reactions to
digoxin (Lanoxin) administration. Paresthesia (a sensation of numbness or
tingling) is an uncommon adverse reaction to digoxin (Lanoxin)
administration.
Dry eyes is an uncommon adverse reaction to digoxin (Lanoxin)
administration.
However, yellow-green halos around images, flashes of light, diplopia
(double vision), and photophobia (unusual intolerance of light) are common
adverse reactions.
Dizziness and vertigo are adverse reactions; however, they are not early
reactions.

Go to the next page if you knew the correct answer, or click the link images
below to further research the concepts in this question (if desired).

Research Concepts:
Digoxin:

Tap flag to report any problems with this question.


Question 76: On opening the right atrium, you see large amounts of blood
coming via the coronary sinus, which is also very enlarged. Which of the
following conditions should be suspected?

Choices:
1. Persistent left superior vena cava
2. Unroofed coronary sinus
3. Unrecognized atrial septal defect
4. Inadequate drainage from bicaval cannulation
Answer: 1 - Persistent left superior vena cava
Explanations:
When one sees a large amount of blood via the coronary sinus, it is
important to first assess the size of the coronary sinus.
A large coronary sinus is associated with an absent innominate vein and a
persistent left superior vena cava. This is the most benign anomaly of the
superior vena cava.
The left superior vena cava runs behind the left atrial appendage and joins
the coronary sinus just behind it. It does need to be cannulated if intra-
cardiac work is being done.
Occasionally the left superior vena cava will enter the left atrium, which
causes a right to left shunt and a mild cyanosis. This is now called an
unroofed coronary sinus, a version of an atrial septal defect.

Go to the next page if you knew the correct answer, or click the link images
below to further research the concepts in this question (if desired).

Research Concepts:
Congenital Heart Disease:

Tap flag to report any problems with this question.


Question 77: Orthostatic hypotension is most likely to be a common
complication of:

Choices:
1. Furosemide
2. Spironolactone
3. Triamterene
4. Hydrochlorothiazide
Answer: 1 - Furosemide
Explanations:
Furosemide is a potent diuretic that can cause orthostatic hypotension
Its effect may worsen when used with other antihypertensive medication
Thiazide diuretics and spironolactone are weaker diuretics when compared
with furosemide

Go to the next page if you knew the correct answer, or click the link images
below to further research the concepts in this question (if desired).

Research Concepts:
Loop Diuretics:

Orthostatic Hypotension:

Tap flag to report any problems with this question.


Question 78: Which of the following does not possess any antagonist
activity at the opioid receptor?

Choices:
1. Naloxone
2. Pentazocine
3. Butorphanol
4. Codeine
Answer: 4 - Codeine
Explanations:
Codeine is an alkaloid of opium.
Codeine is an effective cough suppressant and is also addictive
It is a pure agonist and does not possess any antagonistic properties

Go to the next page if you knew the correct answer, or click the link images
below to further research the concepts in this question (if desired).

Research Concepts:
Opioid Antagonists:

Opioid Receptor:

Tap flag to report any problems with this question.


Question 79: Which of the following statement is false?
Choices:
1. The BARI trial showed that for 2-vessel disease, there was no difference
between CABG and angioplasty
2. Compared to angioplasty, bypass provided a significant decrease in survival
in diabetics
3. CABG was associated with fewer subsequent revascularization procedures
4. Patients in the PTCA group required more interventions in the first year
Answer: 2 - Compared to angioplasty, bypass provided a significant decrease
in survival in diabetics

Explanations:
The BARI trial showed that CABG produces longer survival in diabetics
than PTCA

Go to the next page if you knew the correct answer, or click the link images
below to further research the concepts in this question (if desired).

Research Concepts:
Revascularization Procedures In Coronary Artery
Disease:

Tap flag to report any problems with this question.


Question 80: Which is an adverse effect of amiodarone?
Choices:
1. Ototoxicity
2. Pulmonary fibrosis
3. CHF
4. Nephrotoxicity
Answer: 2 - Pulmonary fibrosis
Explanations:
Gentamicin can cause ototoxicity
CHF is related to doxorubicin
Nephrotoxicity is caused by cisplatin
Amiodarone can cause pulmonary fibrosis

Go to the next page if you knew the correct answer, or click the link images
below to further research the concepts in this question (if desired).

Research Concepts:
Amiodarone:

Pulmonary Fibrosis, Interstitial (Nonidiopathic):

Tap flag to report any problems with this question.


Question 81: In a patient with new onset clubbing, the first study of choice
is?

Choices:
1. Pulmonary function tests
2. Liver function tests
3. Cardiac enzymes
4. Chest x-ray
Answer: 4 - Chest x-ray
Explanations:
Clubbing of both hands is an asymptomatic pulmonary process that requires
chest x-ray for further evaluation.

Go to the next page if you knew the correct answer, or click the link images
below to further research the concepts in this question (if desired).

Research Concepts:
Nails, Clubbing:

Tap flag to report any problems with this question.


Question 82: Which pressor drug is used to increase blood flow to the renal
arteries?

Choices:
1. Norepinephrine
2. Epinephrine
3. Dopamine
4. Acetylcholine
Answer: 3 - Dopamine
Explanations:
Dopamine at low doses can increase renal blood flow.
At doses of 2-5 mcg/kg/hr, dopamine is frequently used to increase renal
blood flow.
At high doses, dopamine can stimulate the heart and increase blood
pressure.

Go to the next page if you knew the correct answer, or click the link images
below to further research the concepts in this question (if desired).

Research Concepts:
Dopamine:

Tap flag to report any problems with this question.


Question 83: In an individual who has lost 5 percent intravascular volume
loss, which clinical feature will be present?

Choices:
1. Hypotension
2. Hypertension
3. Tachycardia
4. Bradycardia
Answer: 3 - Tachycardia
Explanations:
Tachycardia is the first sign of intravascular volume loss
Intravascular volume loss can be secondary to blood loss or dehydration

Go to the next page if you knew the correct answer, or click the link images
below to further research the concepts in this question (if desired).

Research Concepts:
Hemorrhage:

Dehydration:

Tap flag to report any problems with this question.


Question 84: Which medication is capable to lyse blood clots?
Choices:
1. Tissue plasminogen activator (TPA)
2. Heparin
3. Warfarin
4. Lepirudin
Answer: 1 - Tissue plasminogen activator (TPA)
Explanations:
Heparin, lepirudin, and warfarin are anticoagulants
Only TPA can lyse blood clot

Go to the next page if you knew the correct answer, or click the link images
below to further research the concepts in this question (if desired).

Research Concepts:
Tissue Plasminogen Activator:

Tap flag to report any problems with this question.


Question 85: A 55-year-old with atrial fibrillation presents to the clinic with
complaints of fatigue, anorexia, vomiting, and a headache. He claims that he has
not been feeling well for a few days and his vision is blurred and he is seeing
"green." This morning he has pain in his eyes in the presence of sunlight. He
takes several medications but cannot remember the names. Physical findings
include a slow heart rate that is irregular. He most likely is suffering from which
condition?

Choices:
1. Transient ischemic attacks
2. Migraine
3. Digoxin toxicity
4. Gastro-enteritis
Answer: 3 - Digoxin toxicity
Explanations:
Digoxin toxicity can present with many symptoms including palpitations,
mental changes, blurred vision, tendency to see green halos, diplopia,
photophobia, nausea, vomiting, headaches, and agitation.
Heart block may be present with digoxin toxicity. The cause of digoxin
toxicity may include dehydration, electrolyte changes, acute over dose,
acidosis, myocardial ischemia, erroneous dosing, and drug interactions.
Digoxin toxicity does not correlate with the levels of the drug and
endogenous digoxin-like substances can cause false positives, especially in
neonates and in people with renal insufficiency.
Treatment includes monitoring and atropine for bradycardia. Lidocaine,
phenytoin, or digoxin antibodies can be used. Quinidine and procainamide
are contraindicated as they can increase digoxin levels in the blood.
Magnesium can be used if there is ventricular fibrillation.

Go to the next page if you knew the correct answer, or click the link images
below to further research the concepts in this question (if desired).

Research Concepts:
Digoxin Toxicity:

Atrial Fibrillation:
Tap flag to report any problems with this question.
Question 86: Which of the following statements is not true in regards to
infectious endocarditis?

Choices:
1. Valve leaflets have higher rate of infection than myocardium
2. Murmurs are more detectable on the right than left side
3. Streptococcus is the most common organism for left sided endocarditis
4. Antifungal agents should be considered in immunocompromised patients
Answer: 2 - Murmurs are more detectable on the right than left side
Explanations:
Due to limited blood supply, heart valve leaflets are more susceptible to
infection
Murmurs are detected in 80 percent of patients with diseased valve on the
left side, while only 40 percent on the right side

Go to the next page if you knew the correct answer, or click the link images
below to further research the concepts in this question (if desired).

Research Concepts:
Endocarditis:

Tap flag to report any problems with this question.


Question 87: Which of the following drugs used for treating congestive
heart failure do not improve survival?

Choices:
1. Enalapril
2. Furosemide
3. Atenolol
4. Spironolactone
Answer: 2 - Furosemide
Explanations:
ACE inhibitors and beta blockers have well-demonstrated mortality
reduction benefits.
Spironolactone is unique among the diuretics in that it does carry an
advantageous mortality benefit.
Loop diuretics, though useful for fluid management, do not have a
demonstrated mortality benefit.

Go to the next page if you knew the correct answer, or click the link images
below to further research the concepts in this question (if desired).

Research Concepts:
Heart Failure, Congestive:

Tap flag to report any problems with this question.


Question 88: Which of the following is correct about formation of the
cardiac septa?

Choices:
1. If the ostium secundum does not close, there is a patent foramen ovale
2. The intraventricular septum is formed from the endocardial cushions
3. The septum primum and the septum secundum do not completely fuse in
about 20% of people
4. If there is an endocardial cushion defect the septum secundum can
completely separate from the atria
Answer: 3 - The septum primum and the septum secundum do not
completely fuse in about 20% of people

Explanations:
The septum primum grows from the roof of the atrium towards the
endocardial cushions at the end of the 4th week
Endocardial cushions assist in formation of the atrioventricular septum,
atrioventricular canals and the aortic and pulmonary channels
The opening between the septum primum and the endocardial cushions is
called the ostium primum
Perforations in the septum primum are the ostium secundum that allow
blood flow from right to left in the primitive atria

Go to the next page if you knew the correct answer, or click the link images
below to further research the concepts in this question (if desired).

Research Concepts:
Heart Embryology:

Tap flag to report any problems with this question.


Question 89: Besides the ECG, what other studies are required to study
Brugada syndrome?

Choices:
1. Stress test
2. Holter monitor
3. Angiogram
4. Echocardiography
Answer: 4 - Echocardiography
Explanations:
Both echo and MRI are used to study the right ventricle to exclude other
cause of right ventricular dysfunction.
Other disorder which may mimic Brugada syndrome include hypertrophic
cardiomyopathy, myocarditis, or aberrant original of coronary vessels.
All patients must have analysis of serum potassium, calcium, and
magnesium.
Both potassium and calcium can generate ECG patterns that mimic Brugada
syndrome.

Go to the next page if you knew the correct answer, or click the link images
below to further research the concepts in this question (if desired).

Research Concepts:
Brugada Syndrome:

Tap flag to report any problems with this question.


Question 90: Which of the following is not a true statement about the
phrenic nerve?

Choices:
1. The phrenic nerve originates at the 4th cervical nerve and gets contributions
from the 3rd and 5th cervical nerves
2. The phrenic nerve can be found anterior to the scalene muscles
3. Damage to one phrenic nerve means that the patient will no longer be able
to breathe without the use of ventilator
4. During surgery, radial incisions on the diaphragm should be made to
prevent injury to phrenic nerve branches
Answer: 3 - Damage to one phrenic nerve means that the patient will no
longer be able to breathe without the use of ventilator

Explanations:
The phrenic nerve contains both motor, sensory, and sympathetic fibers.
When there is a subphrenic abscess beneath the right diaphragm, the patient
will feel pain in the right shoulder (Kehr sign).
The phrenic nerve passes anterior to the hilum in the chest and must be
identified.
Severing one phrenic nerve can cause diaphragmatic paralysis, but the
patient can survive if the other nerve is intact.

Go to the next page if you knew the correct answer, or click the link images
below to further research the concepts in this question (if desired).

Research Concepts:
Phrenic Nerve:

Tap flag to report any problems with this question.


Question 91: A patient undergoes an open appendectomy and 4 days later
presents with a wound with diffuse erythema and clear watery discharge. He has
a high-grade fever, a wound develops margin of diffuse erythema, necrosis,
bullae, and clear watery discharge. What is the most likely cause?

Choices:
1. Staphylococcus and Streptococcus
2. Clostridia
3. Pseudomonas
4. Polymicrobial
Answer: 4 - Polymicrobial
Explanations:
Necrotizing fasciitis is a rapidly progressive inflammatory infection in the
deep fascia.The presence of gas forming organisms causes subcutaneous
gas.
The most common etiology is polymicrobial. Anaerobic bacteria with
aerobic Gram-negative organisms is common. In the past, group A beta-
hemolytic Streptococcus (GABS) was the most common isolate.
Contributing factors include diabetes and use of steroids
The infection usually begins with pain and erythema, followed by dusky
purple patches. Without treatment, myonecrosis and death is common.

Go to the next page if you knew the correct answer, or click the link images
below to further research the concepts in this question (if desired).

Research Concepts:
Necrotizing Fasciitis:

Tap flag to report any problems with this question.


Question 92: What is the most likely cause of acute postoperative onset of
pain in a single joint with edema and erythema?

Choices:
1. Rheumatoid arthritis
2. Gout
3. Osteoarthritis
4. Septic joint
Answer: 2 - Gout
Explanations:
Gout can develop when excess uric acid deposits in joints. The majority of
patients who develop gouty joint is because of an inability to excrete uric
acid in sufficient amounts.
Individual flare-ups can occur after excess alcohol ingestion. Indulgence in
foods high in purine, rapid weight loss, starvation and trauma are other
causes of acute gout.
Medications such as aspirin can also increase uric acid levels via effects on
renal tubular transport.
Gout can also flare up with use of radiocontrast dyes and uricosuric agents.

Go to the next page if you knew the correct answer, or click the link images
below to further research the concepts in this question (if desired).

Research Concepts:
Gout:

Tap flag to report any problems with this question.


Question 93: What drug can only be given intravenously for congestive
heart failure?

Choices:
1. Digoxin
2. Captopril
3. Quinidine
4. Dobutamine
Answer: 4 - Dobutamine
Explanations:
Dobutamine is an inotropic agent with beta 1 receptor agonist activity.
Besides increasing the force of contraction of the heart, it can also be a
vasodilator for the pulmonary arteries.

Go to the next page if you knew the correct answer, or click the link images
below to further research the concepts in this question (if desired).

Research Concepts:
Heart Failure, Congestive:

Tap flag to report any problems with this question.


Question 94: A 65-year-old man presents complaining of abdominal pain.
His vital signs are stable. Examination reveals a pulsatile mass in the mid-
abdomen. What is the most appropriate test for this patient?

Choices:
1. Magnetic resonance imaging (MRI) of the abdomen
2. Laboratory tests, including complete blood count, electrolytes, and
erythrocyte sedimentation rate
3. Ultrasound examination of the abdomen
4. Upper gastrointestinal series
Answer: 3 - Ultrasound examination of the abdomen
Explanations:
The patient most likely has an abdominal aortic aneurysm.
Urgent imaging with a B mode ultrasound or CT is needed.
Aneurysms greater than 5.5 cm are usually treated either by open method or
by endovascular repair.
Open operations have a mortality rate < 5%. An aneurysm of > 6 cm is
associated with a mortality rate of 50% within one year.

Go to the next page if you knew the correct answer, or click the link images
below to further research the concepts in this question (if desired).

Research Concepts:
Abdominal Aortic Aneurysm:

Tap flag to report any problems with this question.


Question 95: In infants and young children, the most common cause of
multiple brain abscesses is?

Choices:
1. Central lines
2. Congenital heart disease
3. Untreated GI disorders like hirschsprung disease
4. Over use of antibiotics for URTI
Answer: 2 - Congenital heart disease
Explanations:
Cyanotic heart disease is a very common cause of multiple brain abscesses
in children
Other causes of multiple abscesses include alcoholism, steroid use and
diabetes

Go to the next page if you knew the correct answer, or click the link images
below to further research the concepts in this question (if desired).

Research Concepts:
Pediatric Multiple Brain Abscesses:

Cyanotic Heart Disease:

Tap flag to report any problems with this question.


Question 96: Chest pain is positional in patients with which of the
following conditions?

Choices:
1. Tension pneumothorax
2. Acute myocardial infarction
3. Pericarditis
4. Pleurodynia
Answer: 3 - Pericarditis
Explanations:
The chest pain associated with pericarditis changes with position.
Patients get some relief when sitting forward rather than lying back.

Go to the next page if you knew the correct answer, or click the link images
below to further research the concepts in this question (if desired).

Research Concepts:
Pericarditis, Acute:

Tap flag to report any problems with this question.


Question 97: Which medication is most likely to precipitate congestive
heart failure?

Choices:
1. ACE inhibitor
2. Thiazide
3. Digoxin
4. Calcium channel blocker
Answer: 4 - Calcium channel blocker
Explanations:
Congestive heart failure may be precipitated, or exacerbated, by beta
blockers, calcium channel blockers, NSAIDS, and alcohol.
Other medications that can cause CHF include pioglitazone, doxorubicin,
and cocaine.

Go to the next page if you knew the correct answer, or click the link images
below to further research the concepts in this question (if desired).

Research Concepts:
Heart Failure, Congestive:

Calcium Channel blockers:

Tap flag to report any problems with this question.


Question 98: Which of the following would reverse the effects of a
morphine overdose?

Choices:
1. Atropine
2. Naloxone
3. Activated charcoal
4. Flumazenil
Answer: 2 - Naloxone
Explanations:
Naloxone is indicated in overdose with opioids, such as morphine or heroin,
and can quickly reverse respiratory depression
Atropine is indicated for toxicity to organophosphate insecticides
Activated charcoal is indicated for many oral poisons
Flumazenil is indicated for reversal of benzodiazepine overdose/poisoning

Go to the next page if you knew the correct answer, or click the link images
below to further research the concepts in this question (if desired).

Research Concepts:
Opioid Toxicity:

Tap flag to report any problems with this question.


Question 99: Which antihypertensive agent blocks the angiotensin II
receptor?

Choices:
1. Lisinopril
2. Diltiazem
3. Losartan
4. Clonidine
Answer: 3 - Losartan
Explanations:
Angiotensin I is converted by angiotensin converting enzyme to angiotensin
II.
Angiotensin II is a vasoconstrictor so blockage of angiotensin II lowers
blood pressure and is also used to treat CHF and diabetic nephropathy.
Losartan is one of several ARBs (angiotensin receptor blockers).

Go to the next page if you knew the correct answer, or click the link images
below to further research the concepts in this question (if desired).

Research Concepts:
Antihypertensive Medications:

Angiotensin II Receptor Blockers:

Tap flag to report any problems with this question.


Question 100: Which of the following is NOT a reason that epinephrine is
used for anaphylaxis?

Choices:
1. Elevation of blood pressure
2. Augmentation of the release of mediators
3. Inhibits the release of mediators
4. Reverses bronchospasm
Answer: 2 - Augmentation of the release of mediators
Explanations:
Epinephrine raises blood pressure, antagonizes mediators in the system, and
inhibits the release of mediators.
Bronchospasm decreases with epinephrine.
Typically epinephrine administered for anaphylaxis is given
intramuscularly, with the routine adult dose of 0.3 to 0.5 ml (1:10,000
concentration). This is usually administered in the lateral thigh at the vastus
lateralis.
Administration of epinephrine can also be given intravenously. This should
only be done in situations of cardiovascular collapse with profound
hypotension. The IV concentration is much stronger than the IM
concentration (1:1000). It is of course important to not administer the
1:1000 concentration IM.

Go to the next page if you knew the correct answer, or click the link images
below to further research the concepts in this question (if desired).

Research Concepts:
Anaphylaxis:

Tap flag to report any problems with this question.


Question 101: In a 67-year old, the dose of beta-blockers should be
increased until which of the following hemodynamic parameters is achieved?

Choices:
1. Heart rate of 70-80 bpm
2. Heart rate of 50-60 bpm
3. Blood pressure of 95/70 mm Hg or less
4. Blood pressure of 120/85 mm Hg or less
Answer: 2 - Heart rate of 50-60 bpm
Explanations:
The dose of beta-blockers can be increased until a heart rate between 50-60
bpm in achieved.

Go to the next page if you knew the correct answer, or click the link images
below to further research the concepts in this question (if desired).

Research Concepts:
Beta-Blockers:

Tap flag to report any problems with this question.


Question 102: The diagnosis of congestive heart failure (CHF) depends on
which of the following key diagnostic tests?

Choices:
1. Echocardiogram
2. Chest x-ray
3. ECG
4. Cardiac catheterization
Answer: 1 - Echocardiogram
Explanations:
The best test for confirming the diagnosis of CHF is the echocardiogram.
A definition of CHF caused by left ventricular systolic dysfunction is an
ejection fraction of less than 40%.

Go to the next page if you knew the correct answer, or click the link images
below to further research the concepts in this question (if desired).

Research Concepts:
Heart Failure, Congestive:

Tap flag to report any problems with this question.


Question 103: What is the most dangerous side effect of atropine in
infants?

Choices:
1. Tachycardia
2. Fever
3. Decreased salivation
4. Glaucoma
Answer: 2 - Fever
Explanations:
Antimuscarinic agents should be used with caution in children because of
the danger of hyperthermia.
Blockade of the thermoregulatory sweating may result in hyperthermia or
atropine fever.
Atropine toxicity is described as feeling dry as a bone because sweating,
salivation, and lacrimation are all significantly reduced.

Go to the next page if you knew the correct answer, or click the link images
below to further research the concepts in this question (if desired).

Research Concepts:
Atropine:

Tap flag to report any problems with this question.


Question 104: Which ACE inhibitor would be most appropriate in a patient
with severe liver disease?

Choices:
1. Lisinopril
2. Ramipril
3. Benazepril
4. Quinapril
Answer: 1 - Lisinopril
Explanations:
Captopril, lisinopril, and enalapril are the only ACE inhibitors that are not
prodrugs.
All other ACE inhibitors are prodrugs that require hepatic activation for
pharmacologic activity.
Using an ACE inhibitor that does not require hepatic activation would be
advantageous in patients with severe liver disease.

Go to the next page if you knew the correct answer, or click the link images
below to further research the concepts in this question (if desired).

Research Concepts:
Angiotensin Converting Enzyme Inhibitors (ACEI):

Chronic Liver Disease:

Tap flag to report any problems with this question.


Question 105: Which factor can promote wound healing?
Choices:
1. Hypoalbuminemia
2. Infrared therapy
3. Hydrocortisone
4. Zinc
Answer: 4 - Zinc
Explanations:
Zinc is an essential trace element and necessary for proper enzymatic
functions in the body.
Zinc deficiency has been associated with depressed growth, diarrhea,
alopecia, impaired wound healing, delayed sexual maturation, and a variety
of skin lesions
Zinc is readily available from food. Foods rich in zinc include sesame,
celery, mustard, and a variety of seeds.

Go to the next page if you knew the correct answer, or click the link images
below to further research the concepts in this question (if desired).

Research Concepts:
Wound Healing:

Tap flag to report any problems with this question.


Question 106: What is the most common benign tumor of the heart in
adults?

Choices:
1. Rhabdomyoma
2. Lipoma
3. Myxoma
4. Pheochromocytoma
Answer: 3 - Myxoma
Explanations:
Myxoma is the most common benign tumor of the heart.
Majority of myxomas occur in the left atrium near the mitral valve and may
mimic mitral stenosis.
Myxomas are pedunculated on a stalk and can present with embolic
symptoms and constitutional features like fever, general malaise and weight
loss.
Surgery is the only treatment for a myxoma. Most patients go on to lead a
normal life. Even though recurrences are reported, these usually occur when
the surgeon has not adequately removed the mass at surgery.

Go to the next page if you knew the correct answer, or click the link images
below to further research the concepts in this question (if desired).

Research Concepts:
Atrial Myxoma:

Tap flag to report any problems with this question.


Question 107: In a patient with restrictive cardiomyopathy secondary to
amyloidosis, which treatment is NOT recommended?

Choices:
1. Phlebotomy
2. Heart transplant
3. Aspirin
4. Digoxin
Answer: 2 - Heart transplant
Explanations:
Heart transplantation is contraindicated in patients with amyloidosis
because of recurrence cardiomyopathy in allograft.

Go to the next page if you knew the correct answer, or click the link images
below to further research the concepts in this question (if desired).

Research Concepts:
Cardiomyopathy, Restrictive:

Amyloidosis:

Tap flag to report any problems with this question.


Question 108: Which of the following is a property of aspirin?
Choices:
1. Antipyretic
2. Anti-inflammatory
3. Antiplatelet
4. All of the above
Answer: 4 - All of the above
Explanations:
Aspirin has an antipyretic effect and reduces fever
It also has an anti-inflammatory and analgesic effects
The most common effect for usage is its antiplatelet properties and it
reduces MI and stroke
Aspirin is also known as acetylsalicylic acid

Go to the next page if you knew the correct answer, or click the link images
below to further research the concepts in this question (if desired).

Research Concepts:
Aspirin:

Tap flag to report any problems with this question.


Question 109: Cheyne Stokes breathing can be seen in which of the
following conditions?

Choices:
1. Heart failure
2. Stroke patients
3. People living at high altitudes
4. All of above
Answer: 4 - All of above
Explanations:
Cheyne Stokes breathing is often seen in patients with heart failure, stroke,
brain injury, and brain cancers.
This breathing is often seen in individuals living at high altitudes.
Cheyne stokes is often a feature of carbon monoxide poisoning.

Go to the next page if you knew the correct answer, or click the link images
below to further research the concepts in this question (if desired).

Research Concepts:
Cheyne-Stokes Respirations:

Tap flag to report any problems with this question.


Question 110: How long should an emergent coronary artery bypass graft
be delayed after a dose of abciximab?:

Choices:
1. Not at all
2. Platelet gtt on call to OR
3. Delay 12 hours
4. Delay 24 to 48 hours
Answer: 1 - Not at all
Explanations:
Abciximab has not been associated with excess bleeding and platelets
should be given only if bleeding is observed

Go to the next page if you knew the correct answer, or click the link images
below to further research the concepts in this question (if desired).

Research Concepts:
Coronary Artery Bypass Graft:

Tap flag to report any problems with this question.


Question 111: What does an increased v wave in the central line tracing
indicate?

Choices:
1. Cardiogenic shock
2. Increased right atrium pressure
3. Increased left ventricular pressure
4. Infarction
Answer: 2 - Increased right atrium pressure
Explanations:
The v wave is always associated with the pressure in the right atrium.

Go to the next page if you knew the correct answer, or click the link images
below to further research the concepts in this question (if desired).

Research Concepts:
Swan-Ganz Catheterization:

Tap flag to report any problems with this question.


Question 112: Side effects of muromonab CD3 can include:
Choices:
1. Fever
2. Hallucinations
3. Dehydration
4. Hemorrhage
Answer: 1 - Fever
Explanations:
muromonab CD3 can cause patients to develop fever and pulmonary edema

Go to the next page if you knew the correct answer, or click the link images
below to further research the concepts in this question (if desired).

Research Concepts:
Muromonab CD3:

Tap flag to report any problems with this question.


Question 113: Regarding internal mammary artery grafts, all choices are
correct, EXCEPT:

Choices:
1. Graft failure is higher in patients with <50% stenosis in the target vessel
2. Long-term patency rates of LIMA to LAD are >95% at 10 years
3. Long-term patency rates are improved in distally minimally involved target
vessels
4. Graft thrombosis is the most common cause of poor function in the
perioperative period
Answer: 4 - Graft thrombosis is the most common cause of poor function in
the perioperative period

Explanations:
Spasm of the graft the most common cause of poor function and is treated
with nitrates and verapamil.

Go to the next page if you knew the correct answer, or click the link images
below to further research the concepts in this question (if desired).

Research Concepts:
Internal Mammary Artery Bypass:

Tap flag to report any problems with this question.


Question 114: Which of the following is NOT an appropriate preventative
measure for penicillin-induced anaphylaxis?

Choices:
1. Avoidance of penicillin
2. Education in the use of epinephrine
3. Pretreatment with diphenhydramine before administering penicillin
4. Education of family members about the dangers of anaphylaxis
Answer: 3 - Pretreatment with diphenhydramine before administering
penicillin

Explanations:
Patients with a history of anaphylaxis to penicillin should not be given the
antibiotic, or related antibiotics.
Penicillin induced anaphylaxis may be fatal.

Go to the next page if you knew the correct answer, or click the link images
below to further research the concepts in this question (if desired).

Research Concepts:
Anaphylaxis:

Tap flag to report any problems with this question.


Question 115: How deep should the provider depress the patient's sternum
during CPR in adults?

Choices:
1. 0.5 - 1.0 inches
2. 1.5 - 2.0 inches
3. 2.0 - 3.0 inches
4. At least 3.5 inches
Answer: 3 - 2.0 - 3.0 inches
Explanations:
Compression of at least 2.0 inches is the recommended depth for adults.
The chest should be allowed to completely recoil before the next
compression.
The rate should be 100/minute.
For CPR without rescue breaths the provider hould compress to a depth of
1-1.5 inches.

Go to the next page if you knew the correct answer, or click the link images
below to further research the concepts in this question (if desired).

Research Concepts:
Cardiopulmonary Resuscitation:

Tap flag to report any problems with this question.


Question 116: A 65-year-old patient presents with a low-grade fever, chest
discomfort, and mild shortness of breath. Quick exam reveals that she has
pulsations of the capillary bed in the nails. She may have which of the
following?

Choices:
1. A right to left shunt
2. Aortic regurgitation
3. Atrial septal defect
4. Arsenic poisoning
Answer: 2 - Aortic regurgitation
Explanations:
There are several physical signs of aortic insufficiency. These are related to
high pulse pressures and rapid decreases in blood pressure during diastole.
Quincke's sign is the pulsations of the capillary beds in the nails.
This sign is seen in acute AI in about 10% of patients.

Go to the next page if you knew the correct answer, or click the link images
below to further research the concepts in this question (if desired).

Research Concepts:
Aortic Regurgitation:

Tap flag to report any problems with this question.


Question 117: Which is not a determination for mechanical ventilation in a
pressure control mode?

Choices:
1. Breath rate
2. Tidal volume
3. Inspiratory time
4. Airway resistance
Answer: 2 - Tidal volume
Explanations:
In pressure control mode, the tidal volume changes.

Go to the next page if you knew the correct answer, or click the link images
below to further research the concepts in this question (if desired).

Research Concepts:
Ventilator Management:

Tap flag to report any problems with this question.


Question 118: An 18-year-old with an inherited disorder presents to the
cardiology clinic for a routine exam. He is a tall, thin male who had a
pneumothorax two months ago. Auscultation reveals that the chest is clear but he
has a "clicking" sound in his heart. He most likely has which of the following
conditions?

Choices:
1. Marfan syndrome
2. Down syndrome
3. Pericardial effusion
4. Mitral stenosis
Answer: 1 - Marfan syndrome
Explanations:
The above are presenting features of Marfan syndrome.
These individuals also have arachnodactyly, dissecting aortic aneurysms,
ectopia lentis (subluxation of lens), mitral valve prolapse, and recurrent
pneumothoraces.
Marfan syndrome is an autosomal dominant disorder.
The basic defect is fibrillin deficiency, which results in poor development
of aortic tissues.

Go to the next page if you knew the correct answer, or click the link images
below to further research the concepts in this question (if desired).

Research Concepts:
Marfan Syndrome:

Tap flag to report any problems with this question.


Question 119: In general, chronic transplant rejection occurs in what
period of time?

Choices:
1. Minutes
2. Days
3. Years
4. Seconds
Answer: 3 - Years
Explanations:
Hyperacute transplant rejection occurs within minutes.
Acute rejection occurs within months.
Chronic rejection usually occurs after several years.

Go to the next page if you knew the correct answer, or click the link images
below to further research the concepts in this question (if desired).

Research Concepts:
Acute Transplant Rejection:

Tap flag to report any problems with this question.


Question 120: Patients with hypovolemic shock may have all of these
findings except:

Choices:
1. Decreased cardiac output
2. Acidosis
3. Bradycardia
4. Restlessness
Answer: 3 - Bradycardia
Explanations:
Hypovolemic shock patients may experience tachycardia, hypotension,
confusion or restlessness
Without correction can lead to acidosis

Go to the next page if you knew the correct answer, or click the link images
below to further research the concepts in this question (if desired).

Research Concepts:
Hypovolemic Shock:

Tap flag to report any problems with this question.


Question 121: Which of the following do/does NOT occur with dissection
of the ascending aorta?

Choices:
1. Pericardial tamponade
2. Detachment of the commissure and aortic regurgitation
3. Detachment of the left coronary ostia and ischemia
4. Occlusion of the innominate artery take-off and stroke
Answer: 3 - Detachment of the left coronary ostia and ischemia
Explanations:
Dissection of the aorta can do all of the above except the left coronary is
very rarely involved. The right coronary is more involved.
The dissection can interrupt a blood supply and cause malperfusion.
Rupture into the pericardium is the most common cause of death in the first
week.

Go to the next page if you knew the correct answer, or click the link images
below to further research the concepts in this question (if desired).

Research Concepts:
Dissection, Aortic:

Tap flag to report any problems with this question.


Question 122: Why is knowing the therapeutic index of digoxin important?
Choices:
1. Safety
2. Toxicity
3. Bioavailability
4. Penetration across blood brain barrier
Answer: 1 - Safety
Explanations:
The therapeutic index is comparison of the dose that causes therapeutic
effects and the dose that causes death.
The higher the therapeutic index, the safer the drug.
Generally, drugs with low therapeutic index need monitoring of blood
levels.
Drugs with narrow range of therapeutic index include digoxin, lithium,
gentamycin, and amphotericin.

Go to the next page if you knew the correct answer, or click the link images
below to further research the concepts in this question (if desired).

Research Concepts:
Digoxin:

Tap flag to report any problems with this question.


Question 123: In a 65 year old patient with CHF, symptoms can be treated
with several medications. What is often combined with diuretics?

Choices:
1. Aspirin
2. Warfarin
3. ACE inhibitors
4. Nitroglycerin
Answer: 3 - ACE inhibitors
Explanations:
CHF is often associated with fluid overload and thus diuretics are the
mainstay of treatment.
To prevent decompensation, other first line drugs include ACE inhibitors.
ACE inhibitors have been shown to improve survival and quality of life in
patients with CHF.
Other drugs for CHF include beta blockers and spironolactone.

Go to the next page if you knew the correct answer, or click the link images
below to further research the concepts in this question (if desired).

Research Concepts:
Heart Failure, Congestive:

Tap flag to report any problems with this question.


Question 124: Which of the following is a common cause of flail chest?
Choices:
1. Crushing rollover injury
2. Blunt trauma to the rib cage
3. 1 and 2
4. None of the above
Answer: 3 - 1 and 2
Explanations:
The most common cause of flail chest is a crushing rollover injury or blunt
trauma to the rib cage with a transfer of a significant amount of kinetic
energy
By definition, flail chest results from 3 ribs, typically adjacent, being
broken in 2 or more places
The mediastinum shifts to the unaffected side during inspiration in flail
chest
Flail chest can be seen with motor vehicle accidents, falls and assaults

Go to the next page if you knew the correct answer, or click the link images
below to further research the concepts in this question (if desired).

Research Concepts:
Flail Chest:

Tap flag to report any problems with this question.


Question 125: A pericardial friction rub is commonly associated with:
Choices:
1. Uremia
2. Hypertension
3. Cardiac hypertrophy
4. Mitral stenosis
Answer: 1 - Uremia
Explanations:
Pericarditis often presents with a friction rub
Characterized by inflammatory changes of pericardial space

Go to the next page if you knew the correct answer, or click the link images
below to further research the concepts in this question (if desired).

Research Concepts:
Pericarditis, Uremic:

Tap flag to report any problems with this question.


Question 126: Which is the least important step in preparing for
emergency surgery under general anesthesia?

Choices:
1. Verify the correct operation with the patient and surgeon
2. Administer anti-anxiety medication
3. Obtain informed consent
4. Insert an intravenous line
Answer: 2 - Administer anti-anxiety medication
Explanations:
It is important to obtain informed consent, when possible, for every patient
having emergency surgery.
It is also necessary to insert an intravenous line to administer fluids and
anesthesia drugs.
If used, anti-anxiety medication should not be administered until after
informed consent is obtained.

Go to the next page if you knew the correct answer, or click the link images
below to further research the concepts in this question (if desired).

Research Concepts:
Informed Consent:

Emergency Operations:

Tap flag to report any problems with this question.


Question 127: Which catecholamine can be used to dilate the pulmonary
arteries?

Choices:
1. Isoproterenol
2. Dopamine
3. Norepinephrine
4. Dobutamine
Answer: 1 - Isoproterenol
Explanations:
Prior to the availability of nitric oxide and nitroglycerin, isoproterenol was
sometimes used to dilate pulmonary vessels.
Isoproterenol can only be given intravenously.
Isoproterenol elevates systolic blood pressure, but lowers diastolic blood
pressure
Excess use of isoproterenol can cause tachycardia.

Go to the next page if you knew the correct answer, or click the link images
below to further research the concepts in this question (if desired).

Research Concepts:
Catecholamines:

Tap flag to report any problems with this question.


Question 128: Which of the following is the most common valvular
disease in children in the U.S.?

Choices:
1. Rheumatic cardiac disease
2. Myxomas
3. Mitral valve prolapse
4. Aortic regurgitation
Answer: 3 - Mitral valve prolapse
Explanations:
Mitral valve prolapse is the most common valvular defect in children in the
United States
Rheumatic valvular diseases is the most common valvular disease in the
developing world
Myxomas are usually found in the left atrium

Go to the next page if you knew the correct answer, or click the link images
below to further research the concepts in this question (if desired).

Research Concepts:
Mitral Valve Prolapse:

Tap flag to report any problems with this question.


Question 129: A high volume, low pressure cuff can do which of the
following?

Choices:
1. Prevent aspiration
2. Depress cough
3. Better oxygenate
4. More secure tubing
Answer: 3 - Better oxygenate
Explanations:
The main reason to use high volume cuff is to obtain a good seal for better
oxygenation.

Go to the next page if you knew the correct answer, or click the link images
below to further research the concepts in this question (if desired).

Research Concepts:
Endotrachial Tube:

Tap flag to report any problems with this question.


Question 130: Which test with the highest sensitivity for detection of
ischemic myocardium after an MI in a 71 year old?

Choices:
1. Adenosine echocardiography
2. Stress treadmill test
3. CT scan
4. Holter monitor
Answer: 1 - Adenosine echocardiography
Explanations:
Medications used in stress test are coronary vasodilators
Coronary vasodilators cause steal phenomenon and this exacerbates the
hypoperfusion of clogged areas
Adenosine echocardiography has the highest sensitivity

Go to the next page if you knew the correct answer, or click the link images
below to further research the concepts in this question (if desired).

Research Concepts:
Coronary Artery Disease:

Tap flag to report any problems with this question.


Question 131: Primary treatment of Prinzmetal angina should NOT include
which of the following?

Choices:
1. Nitrates
2. Calcium Channel blockers
3. PTCA
4. CABG
Answer: 4 - CABG
Explanations:
CABG for non-ischemic CAD is associated with higher morbidity,
mortality, postoperative MI, decreased graft patency, and earlier recurrence
of angina.

Go to the next page if you knew the correct answer, or click the link images
below to further research the concepts in this question (if desired).

Research Concepts:
Angina, Prinzmetal:

Tap flag to report any problems with this question.


Question 132: Which of the following beta blockers is a class 3 anti-
arrhythmic agent?

Choices:
1. Metoprolol
2. Atenolol
3. Esmolol
4. Sotalol
Answer: 4 - Sotalol
Explanations:
Sotalol is in the same anti-arrhythmic class as amiodarone.
Sotalol is used in the treatment of ventricular arrhythmias.

Go to the next page if you knew the correct answer, or click the link images
below to further research the concepts in this question (if desired).

Research Concepts:
Antiarrhythmic Medication:

Beta-Blockers:

Tap flag to report any problems with this question.


Question 133: What percentage of patients allergic to penicillins have a
cross over allergy to cephalosporins?

Choices:
1. <1%
2. 1 to 5%
3. 10 to 15%
4. 100%
Answer: 2 - 1 to 5%
Explanations:
It was once thought that there was a 10 to 15% cross reactivity between
penicillin and cephalosporins. Further study has shown that the actual
number is much smaller.
Most patients who report penicillin allergy are not actually allergic.
The overestimation was due to contamination of cephalosporins with
penicillin.
The risk is greatest with first generation cephalosporins and approaches 0 in
third generation preparations.

Go to the next page if you knew the correct answer, or click the link images
below to further research the concepts in this question (if desired).

Research Concepts:
Penicillin Allergy:

Cephalosporins:

Tap flag to report any problems with this question.


Question 134: Which of the following is not a function of ACE inhibitors?
Choices:
1. Treat hypertension
2. Afterload reduction
3. Decrease long-term mortality
4. Reduce the risk of stroke
Answer: 4 - Reduce the risk of stroke
Explanations:
ACE inhibitors have a lot of potential actions
Reduction of preload and afterload is one of the desired effects of this class
of medication
ACE inhibitors are also known to decrease long-term mortality

Go to the next page if you knew the correct answer, or click the link images
below to further research the concepts in this question (if desired).

Research Concepts:
ACE Inhibitors:

Tap flag to report any problems with this question.


Question 135: Which heart valve is most often affected by acute rheumatic
fever?

Choices:
1. Aortic
2. Pulmonary
3. Tricuspid
4. Mitral
Answer: 4 - Mitral
Explanations:
Cardiac involvement is very common in acute rheumatic fever. It causes
inflammation of the endocardium and often affects the mitral valve. The
mitral valve becomes dysfunctional over time and leads to arrhythmias and
ventricular failure.
Vegetations can occur on damaged valves and later become infected.
Carditis is a major criterion for diagnosis of ARF and occurs in 40% of
individuals. This many include cardiomegaly, new murmur, CHF,
pericarditis or a pericardial friction rub.
PR interval prolongation does occur in more than 20% of patients but is not
specific for ARF. Echocardiogram is used to confirm carditis and assess the
degree of mitral valve involvement.

Go to the next page if you knew the correct answer, or click the link images
below to further research the concepts in this question (if desired).

Research Concepts:
Rheumatic Fever, Acute:

Tap flag to report any problems with this question.


Question 136: Which is not a risk factor for atherosclerosis?
Choices:
1. Male
2. Cigarette smoking
3. Hypotension
4. Diabetes mellitus
Answer: 3 - Hypotension
Explanations:
Being a male is a risk factor for atherosclerosis.
Other risk factors include obesity, hypertension, positive family history, and
hypercholesterolemia.

Go to the next page if you knew the correct answer, or click the link images
below to further research the concepts in this question (if desired).

Research Concepts:
Atherosclerosis:

Tap flag to report any problems with this question.


Question 137: What is the best treatment for an elevated INR caused by
excess warfarin?

Choices:
1. Urokinase
2. Ticlopidine
3. Vitamin K
4. Cryoprecipitate
Answer: 3 - Vitamin K
Explanations:
When patients present with elevated INR following treatment with oral
anticoagulants like warfarin (Coumadin), the best antidote is vitamin K.
The route of administration depends on the patient's condition.
It can be given orally if there is no active bleeding.
It can be given subcutaneously in a more acute situation. Intravenous
administration is reserved for severe active bleeding.

Go to the next page if you knew the correct answer, or click the link images
below to further research the concepts in this question (if desired).

Research Concepts:
Warfarin:

Tap flag to report any problems with this question.


Question 138: Which of the following is the major adverse effect of
massive blood transfusion after open-heart surgery?

Choices:
1. Hypothermia
2. Hyperkalemia
3. Hypocalcemia
4. Coagulopathy
Answer: 4 - Coagulopathy
Explanations:
Coagulopathy is common with massive blood transfusion. The cause of
dilutional thrombocytopenia
Clinically significant hypocalcaemia does not occur in normal people as
long as the liver is normal
Hypothermia can occur but most blood products are warmed up prior to
transfusion
The most common abnormality in acid base is metabolic alkalosis.
Metabolic acidosis from poor perfusion quickly resolves and the lactate is
converted into bicarbonate in the liver

Go to the next page if you knew the correct answer, or click the link images
below to further research the concepts in this question (if desired).

Research Concepts:
Blood Transfusion:

Open-heart Surgery:

Tap flag to report any problems with this question.


Question 139: Which antihypertensive medication has been associated
with a persistent dry cough?

Choices:
1. Angiotensin converting enzyme Inhibitors (ACEI)
2. Beta blockers
3. Alpha blockers
4. Calcium channel blockers
Answer: 1 - Angiotensin converting enzyme Inhibitors (ACEI)
Explanations:
Side effects from using ACEI include headache, dizziness or
lightheadedness, excessive tiredness, cough, upset stomach, loss of taste
sensation, unusual bruising, and diarrhea.
A persistent dry cough is also seen in a few patients and may require a
change in the medication. The cough may be secondary to the bradykinin
receptor.
The incidence of cough has been estimated from 3 to 35%, but is probably
at the lower end of this range.
The most serious side effect with these medications is angioedema.

Go to the next page if you knew the correct answer, or click the link images
below to further research the concepts in this question (if desired).

Research Concepts:
Angiotensin Converting Enzyme Inhibitors (ACEI):

Tap flag to report any problems with this question.


Question 140: Why is nitroglycerin administered to patients with angina?
Choices:
1. Inhibits formation of clots within the coronary arteries
2. Vasodilates coronary arteries as well as the peripheral vasculature
3. Has an analgesic effect
4. Increases cardiac contractility
Answer: 2 - Vasodilates coronary arteries as well as the peripheral
vasculature

Explanations:
Nitroglycerin produces peripheral vasodilation, which reduces myocardial
oxygen consumption and demand.
It also dilates coronary arteries to improve blood flow to the myocardium.

Go to the next page if you knew the correct answer, or click the link images
below to further research the concepts in this question (if desired).

Research Concepts:
Nitroglycerin:

Angina, Unstable:

Angina, Stable:
Tap flag to report any problems with this question.
Question 141: Which of the following signs on auscultation is most
common in the presence of a left atrial myxoma?

Choices:
1. Fixed splitting of second heart sound
2. Loud S1
3. Systolic ejection click
4. Austin flint murmur
Answer: 2 - Loud S1
Explanations:
Left atrial myxomas generally tend to present with signs and symptoms of
mitral stenosis.
A loud S1 is commonly heard because the mass tends to prolapse into the
orifice and mimics mitral stenosis.
Other findings may include elevated JVP, prominent A wave, or an S3.
A diastolic rumble or a holosystolic murmur can also be auscultated.

Go to the next page if you knew the correct answer, or click the link images
below to further research the concepts in this question (if desired).

Research Concepts:
Atrial Myxoma:

Tap flag to report any problems with this question.


Question 142: Percutaneous coronary artery angioplasty in a patient with
stable angina and one vessel disease has been shown to be superior to medical
therapy in which manner?

Choices:
1. Survival at 6 months
2. Exercise performance at 6 months
3. Overall risk of MI
4. No advantage
Answer: 2 - Exercise performance at 6 months
Explanations:
PTCA offers better symptom control and exercise performance in this
patient group

Go to the next page if you knew the correct answer, or click the link images
below to further research the concepts in this question (if desired).

Research Concepts:
Single Vessel Disease:

Percutaneous Transluminal Coronary Angioplasty:

Angina, Stable:

Tap flag to report any problems with this question.


Question 143: What parameter is usually not contained in an arterial blood
gas report?

Choices:
1. Hydrogen concentration
2. PaCO2
3. PaO2
4. Base excess
Answer: 1 - Hydrogen concentration
Explanations:
An arterial blood gas report contains pH, PaC02 and Pa02

Go to the next page if you knew the correct answer, or click the link images
below to further research the concepts in this question (if desired).

Research Concepts:
Arterial Blood Gas:

Tap flag to report any problems with this question.


Question 144: Which is usually true concerning aortic dissection?
Choices:
1. 10-20% of patients present without pain
2. Dissection does not involve the aortic valve or coronary arteries
3. Beta blockers are contraindicated
4. Treatment involves raising blood pressure to perfuse coronary arteries
Answer: 1 - 10-20% of patients present without pain
Explanations:
10-20% of patients present without pain
Dissection can involve the aortic valve producing an aortic insufficiency
murmur and coronary arteries producing an ECG with an ischemic pattern
Treatment is directed towards lowering blood pressure with beta blockers

Go to the next page if you knew the correct answer, or click the link images
below to further research the concepts in this question (if desired).

Research Concepts:
Aortic Dissection:

Tap flag to report any problems with this question.


Question 145: A 44-year-old on heparin requires urgent surgery. How long
must heparin be discontinued before one can safely undergo surgery?

Choices:
1. 1 hour
2. 6 hours
3. 24 hours
4. 48 hours
Answer: 2 - 6 hours
Explanations:
Heparin should be discontinued 6 hours prior to surgery.
The heparin can be restarted 12-24 hours after surgery if postoperative
hemorrhage is no longer a threat.
Heparin can be reversed with protamine.

Go to the next page if you knew the correct answer, or click the link images
below to further research the concepts in this question (if desired).

Research Concepts:
Preoperative Evaluation And Management:

Anticoagulation:

Tap flag to report any problems with this question.


Question 146: A 62 year old female complains of recurrent episodes of
palpitations lasting 3 minutes associated with near syncope. The episodes have
occurred about once a week for 3 weeks. Exam, blood work, and ECG are
normal. Which of the following tests is most likely to provide the diagnosis?

Choices:
1. Echocardiography
2. 24 hour holter monitor
3. Exercise stress test
4. Event monitor or loop monitor
Answer: 4 - Event monitor or loop monitor
Explanations:
Near syncope or syncope associated with palpitations are likely due to a
cardiac arrhythmia
The prognosis for cardiac causes of these complaints shows up to a 33%
one year mortality rate as compared to only 10% for non-cardiac causes
A 24 hour holter monitor is less likely to show the abnormality than an
event monitor or loop monitor.
Stress testing and echocardiography are less likely to provide the diagnosis

Go to the next page if you knew the correct answer, or click the link images
below to further research the concepts in this question (if desired).

Research Concepts:
Palpitations:

Tap flag to report any problems with this question.


Question 147: Which of the following is not a complication of myocardial
infarction?

Choices:
1. Arrhythmias
2. Congestive heart failure
3. Contractile dysfunction
4. Mitral valve stenosis
Answer: 4 - Mitral valve stenosis
Explanations:
Cardiac valve stenosis is not a complication of an MI

Go to the next page if you knew the correct answer, or click the link images
below to further research the concepts in this question (if desired).

Research Concepts:
Acute Myocardial Infarction:

Tap flag to report any problems with this question.


Question 148: Chronic use of hydrochlorothiazide can result in:
Choices:
1. Hypotension
2. Decreased urine output
3. Hyperthyroidism
4. Bradycardia
Answer: 1 - Hypotension
Explanations:
Hydrochlorothiazide is a diuretic
It depletes intravascular volume

Go to the next page if you knew the correct answer, or click the link images
below to further research the concepts in this question (if desired).

Research Concepts:
Thiazide Diuretics:

Tap flag to report any problems with this question.


Question 149: Which of the following is associated with the Bowditch
effect?

Choices:
1. Bow shaped legs
2. Bony protrusion of the ankle joint
3. Increased myocardial contractility with increased heart rate
4. Decreased breathing rate with opioids
Answer: 3 - Increased myocardial contractility with increased heart rate
Explanations:
The Bowditch effect is also known as the Treppe phenomenon.
With an increase in heart rate there is also an increase in myocardial
contractility.
The explanation is the inability of the Na+/ K+ ATPase to keep up with
sodium influx at high heart rates.

Go to the next page if you knew the correct answer, or click the link images
below to further research the concepts in this question (if desired).

Research Concepts:
Bowditch Effect:

Tap flag to report any problems with this question.


Question 150: A patient with congestive heart failure is hemodynamically
stable but still symptomatic. Which of the following should be started?

Choices:
1. Dobutamine
2. Hydralazine
3. Terazosin
4. Lisinopril
Answer: 4 - Lisinopril
Explanations:
ACEI can rapidly can improve symptoms in patients with congestive heart
failure.
ACEI are recommended for all patients with re-current or other symptoms
of CHF or reduced left ventricular function. ACEI decreases afterload and
permits LV remodeling. The drugs have been shown to increase survival
and decrease rate of hospitalization.
Side effects of ACEI include hyperkalemia, hypotension, cough, rash,
changes in mental state, angioedema, and dry cough
If cough develops, patients can be switched to an angiotensin receptor
blocker.

Go to the next page if you knew the correct answer, or click the link images
below to further research the concepts in this question (if desired).

Research Concepts:
Heart Failure, Congestive:

Tap flag to report any problems with this question.


Question 151: Which medication is an endogenous potent vasoconstrictor?
Choices:
1. Dobutamine
2. Isoproterenol
3. Phenylephrine
4. Angiotensin II
Answer: 4 - Angiotensin II
Explanations:
Angiotensin II is a potent vasoconstrictor that acts through G-protein
coupled receptors.
Dobutamine is used in refractory congestive heart failure.
Isoproterenol stimulates beta-1 and beta-2 receptors and increases heart rate
and contractility.
Phenylephrine is a potent, direct-acting alpha-adrenergic stimulator with
weak beta-adrenergic activity.

Go to the next page if you knew the correct answer, or click the link images
below to further research the concepts in this question (if desired).

Research Concepts:
Renin Angiotensin System:

Tap flag to report any problems with this question.


Question 152: What is the most common cause of myocardial infarction?
Choices:
1. Atherosclerosis
2. Smoking
3. Stress
4. Hypertension
Answer: 1 - Atherosclerosis
Explanations:
Atherosclerosis is the most common etiology of MI
MI is result from rupture of plaque
Others are risk factors

Go to the next page if you knew the correct answer, or click the link images
below to further research the concepts in this question (if desired).

Research Concepts:
Acute Myocardial Infarction:

Coronary Artery Atherosclerosis:

Tap flag to report any problems with this question.


Question 153: Data suggest that occurrence of a stroke after a TIA is
usually within what time period?

Choices:
1. 24 hours
2. 20 days
3. 90 days
4. 180 days
Answer: 3 - 90 days
Explanations:
About 10% of patients with a TIA will have a stoke
Most of these strokes occur within 90 days
However more than 50% of these strokes occur within 48 hours

Go to the next page if you knew the correct answer, or click the link images
below to further research the concepts in this question (if desired).

Research Concepts:
Transient Ischemic Attack:

Stroke:

Tap flag to report any problems with this question.


Question 154: Which of the following is the MOST common symptom in
patients who develop coronary artery disease?

Choices:
1. Chest pain
2. Dyspnea
3. Dizziness
4. Arm pain
Answer: 1 - Chest pain
Explanations:
Chest pain is the most common feature in patients with coronary artery
disease.
To differentiate anginal pain from other causes, it is important to ask about
the onset, provocative factors, quality of pain, region of pain and if the pain
is radiating.

Go to the next page if you knew the correct answer, or click the link images
below to further research the concepts in this question (if desired).

Research Concepts:
Coronary Artery Disease:

Tap flag to report any problems with this question.


Question 155: After coronary angioplasty a patient develops decreased
pedal pulses, livido reticularis, abdominal pain, and confusion. Laboratories
show BUN 120 mg/dL, creatinine 5.0 mg/dL, and phosphate 9.2 mg/dL.
Urinalysis has 8 to 10 WBC/HPF, 5 10 to 15 RBC/HPF, and few hyaline casts.
Select the most likely diagnosis.

Choices:
1. Acute tubular necrosis
2. Cholesterol embolisms
3. Renal artery thrombosis
4. Acute tubular necrosis secondary to hypotension
Answer: 2 - Cholesterol embolisms
Explanations:
Disruptions of plaques in the aorta or renal arteries during left heart
catheterization can dislodge cholesterol
This can affect peripheral vasculature, bowel, pancreas, and kidneys
The urinalysis can show amorphous sediment or hyaline casts but with
ATN, granular casts would be more likely
Eosinophiluria and decreased complement levels are possible

Go to the next page if you knew the correct answer, or click the link images
below to further research the concepts in this question (if desired).

Research Concepts:
Cholesterol Embolism:

Tap flag to report any problems with this question.


Question 156: A woman is found with no pulse by the housekeeper at her
residence. What is the next step after calling 911?

Choices:
1. 2 breaths for ventilation
2. Chest compressions at 100 beats/min
3. Perform cricothyrotomy
4. None of the above
Answer: 2 - Chest compressions at 100 beats/min
Explanations:
Revised CPR guidelines call for only chest compressions at 100 beats/min
Circulation is more important than ventilation
Breaths are not mandated anymore when performing CPR
Cricothyrotomies are not part of CPR

Go to the next page if you knew the correct answer, or click the link images
below to further research the concepts in this question (if desired).

Research Concepts:
Cardiopulmonary Resuscitation:

Tap flag to report any problems with this question.


Question 157: Which is most likely to cause alterations in serum
electrolytes?

Choices:
1. Thiazide diuretics
2. Loop diuretics
3. Spironolactone
4. Acetazolamide
Answer: 2 - Loop diuretics
Explanations:
Loop diuretics act on the thick ascending loop of Henle.
Electrolyte reabsorption mostly occurs in the thick ascending loop of Henle.
Acetazolamide acts on proximal tubules. Most of the filtered electrolyte are
reabsorbed later.
Thiazide and spironolactone are weak diuretics when compared with loop
diuretics.

Go to the next page if you knew the correct answer, or click the link images
below to further research the concepts in this question (if desired).

Research Concepts:
Loop Diuretics:

Tap flag to report any problems with this question.


Question 158: Which beta-blocker has intrinsic sympathomimetic activity?
Choices:
1. Pindolol
2. Propranolol
3. Labetalol
4. Esmolol
Answer: 1 - Pindolol
Explanations:
Pindolol is a nonspecific beta blocker.
It also has partial agonist properties at beta 2 receptors.
This is termed intrinsic sympathomimetic activity.
It is indicated for treatment of hypertension. It is also used off label for
stable angina and hyperthyroidism.

Go to the next page if you knew the correct answer, or click the link images
below to further research the concepts in this question (if desired).

Research Concepts:
Pindolol:

Tap flag to report any problems with this question.


Question 159: Which of the following is an absolute indication for a
Greenfield filter?

Choices:
1. Complication of anticoagulation
2. Presence of a free-floating iliofemoral thrombus
3. Prophylaxis for deep venous thrombosis
4. Contraindication of anticoagulation therapy
Answer: 4 - Contraindication of anticoagulation therapy
Explanations:
Inferior vena cava filters are not often used because there is not much solid
data to support their use. The most common indication for inferior vena
cava filters is in individuals who have a contraindication to anticoagulation
therapy.
Contraindications to anticoagulation may be due to hemorrhagic strokes,
trauma, brain tumors, pregnancy, or poor patient compliance. These
individuals may benefit from an inferior vena cava filter.
An inferior vena cava filter is less commonly indicated for patients who
have a complication from anticoagulation therapy.
A free-floating thrombus was once an indication for an inferior vena cava
filter, but today evidence does not show that such patients are at high risk
for pulmonary embolism.

Go to the next page if you knew the correct answer, or click the link images
below to further research the concepts in this question (if desired).

Research Concepts:
Inferior Vena Cava Filters:

Tap flag to report any problems with this question.


Question 160: Which is not a part of treatment of a surgical patient who
develops ventricular fibrillation in the operating room?

Choices:
1. External defibrillation
2. Precordial thump
3. Cardiopulmonary resuscitation
4. Consider epinephrine every 3-5 minutes
Answer: 2 - Precordial thump
Explanations:
Precordial thump is only used in witnessed monitored arrest where no
defibrillator is available
Always check airway, start CPR and administer defibrillation shock at 200
joules
Amiodarone and epinephrine can be administered during the arrest
Post resuscitation; one should maintain lidocaine at 1-4 mg/min or start an
amiodarone drip

Go to the next page if you knew the correct answer, or click the link images
below to further research the concepts in this question (if desired).

Research Concepts:
Ventricular Fibrillation:

Tap flag to report any problems with this question.


Question 161: What is the best intervention for a patient with high FIO2
requirements?

Choices:
1. Positive end expiratory pressure
2. Intermittent mandatory ventilation
3. Synchronous intermittent mandatory ventilation
4. Assist control ventilation
Answer: 1 - Positive end expiratory pressure
Explanations:
PEEP increases alveolar recruitment
It also decreases shunting improving gas exchange and reduces FIO2
requirements
PEEP can reduce cardiac filling by increasing intrathoracic pressure

Go to the next page if you knew the correct answer, or click the link images
below to further research the concepts in this question (if desired).

Research Concepts:
Positive End-Expiratory Pressure (PEEP):

Tap flag to report any problems with this question.


Question 162: What is the mechanism of action of amlodipine?
Choices:
1. Sodium channel blocker
2. Potassium channel blocker
3. Calcium channel blocker
4. Magnesium channel blocker
Answer: 3 - Calcium channel blocker
Explanations:
Amlodipine is a calcium channel blocker
Commonly used for hypertension

Go to the next page if you knew the correct answer, or click the link images
below to further research the concepts in this question (if desired).

Research Concepts:
Amlodipine:

Tap flag to report any problems with this question.


Question 163: When compared to young adults, elderly patients:
Choices:
1. Have a slower rate of drug elimination
2. Weigh less
3. Have a slower heart rate
4. None of the above
Answer: 1 - Have a slower rate of drug elimination
Explanations:
When compared to young adults, elderly patients have a slower rate of drug
elimination
The dosages of common medications may be reduced up to 50% in elderly
patients
The phenomenon is directly related to a decreased rate of drug elimination
by the liver and kidneys
The elderly usually weight more than younger patients. The resting heart
rate is comparable.

Go to the next page if you knew the correct answer, or click the link images
below to further research the concepts in this question (if desired).

Research Concepts:
Drug Metabolism:

Drug Elimination:

Tap flag to report any problems with this question.


Question 164: What is the treatment for beta blocker toxicity?
Choices:
1. Acetylcholine
2. IV fluid
3. Glucagon
4. Albuterol
Answer: 3 - Glucagon
Explanations:
Glucagon is the treatment option for beta blocker overdose

Go to the next page if you knew the correct answer, or click the link images
below to further research the concepts in this question (if desired).

Research Concepts:
Toxic, Beta-Blocker:

Tap flag to report any problems with this question.


Question 165: For long-term control of torsades, which class of drugs is
often used?

Choices:
1. Beta-blockers
2. Calcium channel blockers
3. ACE inhibitors
4. Class 3 antiarrhythmics
Answer: 1 - Beta-blockers
Explanations:
Beta-blockers are the drugs of choice for long-term control of torsades.
Beta-blockers should be used with caution if bradycardia is present.
Asymptomatic patients may be observed without drug treatment.
Permanent pacing is also used in some patients who are on maximal doses
of beta-blockers.

Go to the next page if you knew the correct answer, or click the link images
below to further research the concepts in this question (if desired).

Research Concepts:
Torsade de Pointes:

Tap flag to report any problems with this question.


Question 166: Which of the following is false regarding scimitar
syndrome?

Choices:
1. There is anomalous systemic arterial supply
2. There is hypoplasia of right lung
3. Venous drainage is to the superior vena cava
4. There may be associated congenital heart defects
Answer: 3 - Venous drainage is to the superior vena cava
Explanations:
Scimitar syndrome has anomalous venous drainage to the inferior vena
cava. This vein appears as a "Turkish Sword" on chest x-ray.
There is anomalous arterial supply from the aorta or the pulmonary artery.
The lung is usually hypoplastic. In most cases, an atrial septal defect is not
present.
Surgery may involve a pneumonectomy if the right lung is not functional.

Go to the next page if you knew the correct answer, or click the link images
below to further research the concepts in this question (if desired).

Research Concepts:
Scimitar Syndrome:

Tap flag to report any problems with this question.


Question 167: Which is false about isoproterenol?
Choices:
1. Acts on beta 1 receptor
2. Causes bradycardia
3. Inotropic agent
4. Acts on beta 2 receptor
Answer: 2 - Causes bradycardia
Explanations:
Isoproterenol is a non-selective beta 1 and 2 agonist

Go to the next page if you knew the correct answer, or click the link images
below to further research the concepts in this question (if desired).

Research Concepts:
Isoproterenol:

Tap flag to report any problems with this question.


Question 168: The hemodynamic collapse in septic shock is best treated
with which medication?

Choices:
1. Norepinephrine
2. Atropine
3. Milrinone
4. Fluids
Answer: 1 - Norepinephrine
Explanations:
Norepinephrine is a catecholamine mediator of the sympathetic nervous
system.
It is the best treatment of hemodynamic collapse in septic shock and should
be considered if the condition is refractory to fluid resuscitation.
It stimulates alpha and beta-1 adrenergic receptors causing vasoconstriction
and increased cardiac output.
Other essential components of treatment include respiratory support to
insure adequate oxygenation, antimicrobial therapy, and metabolic support.

Go to the next page if you knew the correct answer, or click the link images
below to further research the concepts in this question (if desired).

Research Concepts:
Shock, Septic:

Tap flag to report any problems with this question.


Question 169: Which of the following is not a characteristic of amiloride?
Choices:
1. Decreased calcium excretion
2. Increased sodium loss
3. Increased potassium retention
4. Increased magnesium loss
Answer: 4 - Increased magnesium loss
Explanations:
Amiloride is potassium sparing diuretic that acts on the distal tubules and
collecting ducts
It causes increased potassium retention and increased sodium loss
It also decreases magnesium loss by enhancing absorption
Calcium excretion is also decreased with the use of amiloride

Go to the next page if you knew the correct answer, or click the link images
below to further research the concepts in this question (if desired).

Research Concepts:
Potassium Sparing Diuretics:

Tap flag to report any problems with this question.


Question 170: What is the currently recommended first test for pulmonary
embolism?

Choices:
1. Ventilation perfusion scintigraphy
2. Arterial blood gas
3. CT pulmonary angiogram
4. Cardiac US
Answer: 3 - CT pulmonary angiogram
Explanations:
The current first recommended test to diagnose a pulmonary embolism is
CT pulmonary angiography.
CT pulmonary angiography has a sensitivity and specificity as good as
conventional pulmonary angiography with far less morbidity, invasiveness,
cost, and need for angiographic expertise.
V/Q scintigraphy remains an alternative in those patients allergic to
iodinated IV contrast administration.
CT pulmonary angiography is the initial recommended study of choice even
in the context of pregnancy.

Go to the next page if you knew the correct answer, or click the link images
below to further research the concepts in this question (if desired).

Research Concepts:
Pulmonary Embolism:

Tap flag to report any problems with this question.


Question 171: Which of the following is FALSE about aztreonam?
Choices:
1. Inhibits cell wall synthesis
2. Synergistic with aminoglycosides
3. It has excellent Gram positive coverage
4. It has no cross reactivity to penicillin
Answer: 3 - It has excellent Gram positive coverage
Explanations:
Aztreonam is a monobactam that is resistant to beta lactamase and has
excellent activity against gram-negative bacteria.
Aztreonam inhibits cell wall synthesis and is synergistic with
aminoglycoside antibiotics.
Aztreonam is administered intravenously and is excreted via the kidneys. It
has no cross reactivity to penicillin.
Aztreonam has almost no Gram-positive bactericidal effects.

Go to the next page if you knew the correct answer, or click the link images
below to further research the concepts in this question (if desired).

Research Concepts:
Aztreonam:

Tap flag to report any problems with this question.


Question 172: Which of the following is true of interpleural analgesia?
Choices:
1. It involves injection of local anesthetic into visceral pleura
2. It produces analgesia primarily by diffusion of local anesthetic into spinal
nerves
3. It is usually not effective in the presence of pleural adhesions
4. It is not recommended in the presence of thoracostomy tubes
Answer: 3 - It is usually not effective in the presence of pleural adhesions
Explanations:
Interpleural analgesia involves instillation of local anesthetic into the
interpleural space located between the visceral and parietal pleura.
Interpleural analgesia produces pain relief primarily by diffusion of local
anesthetic into the adjacent intercostal nerves.
Pleural adhesions may restrict flow of local anesthetic in the pleural space,
thus limiting the pain relief.
Interpleural analgesia is not contraindicated in the presence of
thoracostomy tubes, although it may be less effective due to loss of local
anesthetic through the tube hole site.

Go to the next page if you knew the correct answer, or click the link images
below to further research the concepts in this question (if desired).

Research Concepts:
Pleural Anesthesia:

Tap flag to report any problems with this question.


Question 173: A 6-month-old infant with Down syndrome has an
atrioventricular canal defect and had pulmonary artery banding at age 4 months.
She weighs 4.5 kg and takes 30 minutes to drink 4 oz. of formula mixed with
Polycose. The patient has significant head lag and does not roll. What would be
the primary goal of Early Intervention?

Choices:
1. Neck muscle strengthening exercises
2. Daycare outside the home
3. Interventions on feeding with feedback
4. Starting solid foods
Answer: 3 - Interventions on feeding with feedback
Explanations:
Early Interventions would involve a pediatric developmental feeding team
The most important goal with this patient is improved nutrition and weight
gain

Go to the next page if you knew the correct answer, or click the link images
below to further research the concepts in this question (if desired).

Research Concepts:
Atrioventricular Canal Defects:

Down Syndrome:

Tap flag to report any problems with this question.


Question 174: Which of the following symptoms is most likely in a young
patient with aortic stenosis?

Choices:
1. Palpitations, dyspnea, and sweating
2. Chest pain, syncope, and fatigue
3. Syncope, fatigue, and palpitations
4. Diaphoresis, syncope, and dizziness
Answer: 2 - Chest pain, syncope, and fatigue
Explanations:
Aortic stenosis can present with chest pain syncope and symptoms of heart
failure.
Aortic stenosis in young people is rare and symptoms develop gradually.
Bicuspid aortic valve is the most common cause of aortic stenosis.
Degenerative aortic stenosis occurs in elderly people.

Go to the next page if you knew the correct answer, or click the link images
below to further research the concepts in this question (if desired).

Research Concepts:
Aortic Stenosis:

Tap flag to report any problems with this question.


Question 175: A 5 year old is found to have mild aortic insufficiency and a
small subaortic ventricular septal defect. Which of the following is the most
appropriate plan?

Choices:
1. Oral loading of digoxin
2. Surgical closure of defect and repair of valve
3. Annual echocardiograms to assess progression of the lesions
4. Cardiac catheterization to measure chamber pressures
Answer: 2 - Surgical closure of defect and repair of valve
Explanations:
The earlier the repair the lower chance of progressive aortic insufficiency.
The defect should be closed and the valve repaired.
In some cases, only the VSD is closed and the valve observed for
progression.

Go to the next page if you knew the correct answer, or click the link images
below to further research the concepts in this question (if desired).

Research Concepts:
Ventricular Septal Defect:

Tap flag to report any problems with this question.


Question 176: What is the antihypertensive medication of choice if cost is
the primary concern?

Choices:
1. Calcium channel blockers
2. Beta blockers
3. ACE inhibitors
4. Diuretics
Answer: 4 - Diuretics
Explanations:
The least expensive antihypertensive medications are diuretics.

Go to the next page if you knew the correct answer, or click the link images
below to further research the concepts in this question (if desired).

Research Concepts:
Antihypertensive Medications:

Tap flag to report any problems with this question.


Question 177: In a patient with atrial fibrillation and a high ventricular
rate, what drug is recommended prior to conversion?

Choices:
1. Esmolol
2. Captopril
3. Amiodarone
4. Lidocaine
Answer: 1 - Esmolol
Explanations:
When a patient has atrial fibrillation with a high ventricular rate, the rhythm
can be converted to VT and thus a beta-blocker or a calcium channel
blocker must be used.
Either esmolol or diltiazem can be used prior to cardioversion.

Go to the next page if you knew the correct answer, or click the link images
below to further research the concepts in this question (if desired).

Research Concepts:
Atrial Fibrillation:

Tap flag to report any problems with this question.


Question 178: Diagnostic criteria for malignant hypertension:
Choices:
1. Papilledema
2. Presence of a malignant tumor
3. DBP > 120 mm Hg
4. SBP > 180 mm Hg
Answer: 1 - Papilledema
Explanations:
The development of papilledema evident on eye exam is present when
making a diagnosis of malignant hypertension
Accelerating hypertension is evidenced by blood pressure of >120 mm Hg
diastolic or >180 mm Hg systolic without end organ damage

Go to the next page if you knew the correct answer, or click the link images
below to further research the concepts in this question (if desired).

Research Concepts:
Hypertensive Emergencies:

Tap flag to report any problems with this question.


Question 179: During repair of an ascending aortic dissection, which
period is associated with the highest risk of malperfusion?

Choices:
1. Initiation of bypass
2. Termination of bypass
3. Cannulation of femoral artery
4. Cannulation of aortic graft
Answer: 1 - Initiation of bypass
Explanations:
With an ascending aortic dissection, the femoral vessels must be cannulated
first and the patient must be on femoral-femoral bypass. Later, a cannula
can be inserted into the right atrium.
The femoral artery cannula must be in the true lumen. Otherwise
malperfusion will occur.
The perfusion pressure will be very low if you cannulate the false lumen.
Transesophageal echocardiography is used nowadays to check for
malperfusion.
It is this phase, the initiation of perfusion, that represents the period of
maximal risk of malperfusion, as circulation shifts from antegrade to
retrograde via the femoral artery.

Go to the next page if you knew the correct answer, or click the link images
below to further research the concepts in this question (if desired).

Research Concepts:
Dissection, Aortic:

Tap flag to report any problems with this question.


Question 180: A patient's quality of life is of importance when deciding if:
Choices:
1. To offer them a research study
2. The families expectations
3. Their care is becoming futile
4. They are able to withstand rehabilitation
Answer: 3 - Their care is becoming futile
Explanations:
Futility of care is decided on based on the patient's wishes, cost of
continuing treatment and their quality of life

Go to the next page if you knew the correct answer, or click the link images
below to further research the concepts in this question (if desired).

Research Concepts:
Quality of Life:

Futile Treatment:

Tap flag to report any problems with this question.


Question 181: A 67 year old male has not passed urine since his inguinal
hernia repair 12 hours ago. He feels the urge and complains of lower abdominal
discomfort. What is the next step in his management?

Choices:
1. Suprapubic cystotomy
2. Foley catheter
3. Re-explore patient
4. Give fluids
Answer: 2 - Foley catheter
Explanations:
In a post operative patient who has lower abdominal discomfort and has not
voided for 12 hours, urinary retention should be the first diagnosis.
A foley catheter can be inserted at the bedside.
If one is not able to place a foley, then a urology consult may be required
for placement of a suprapubic catheter.

Go to the next page if you knew the correct answer, or click the link images
below to further research the concepts in this question (if desired).

Research Concepts:
Urinary Retention:

Tap flag to report any problems with this question.


Question 182: Which of the following is on the acute coronary syndrome
(ACS) continuum?

Choices:
1. Unstable angina
2. Non-ST segment elevation myocardial infarction (NSTEMI)
3. ST segment elevation myocardial infarction (STEMI)
4. All of the above
Answer: 4 - All of the above
Explanations:
The ACS continuum consists of unstable angina, NSTEMI and STEMI
Most patients with STEMI develop pathological Q waves
Patients with NSTEMI do not develop pathological Q waves

Go to the next page if you knew the correct answer, or click the link images
below to further research the concepts in this question (if desired).

Research Concepts:
Acute Coronary Syndromes:

Tap flag to report any problems with this question.


Question 183: The valsalva maneuver causes the murmur of aortic stenosis
(AS) to:

Choices:
1. Decrease
2. Increase
3. Disappear
4. No effect at all
Answer: 1 - Decrease
Explanations:
The valsalva maneuver is a moderately forceful attempted exhalation
against a closed airway
The maneuver causes the murmur of AS to decrease
It also decreases the murmurs of pulmonic stenosis and tricuspid
regurgitation

Go to the next page if you knew the correct answer, or click the link images
below to further research the concepts in this question (if desired).

Research Concepts:
Valsalva Maneuver:

Aortic Stenosis:

Tap flag to report any problems with this question.


Question 184: Cardiogenic shock is the most common complication of:
Choices:
1. Congestive heart failure (CHF)
2. Atrial fibrillation
3. Acute myocardial infarction (AMI)
4. Torsade de pointes
Answer: 3 - Acute myocardial infarction (AMI)
Explanations:
Cardiogenic shock is the most common complication of AMI
5-10% of patient with AMI develop cardiogenic shock
Hemodynamically it is defined by a systolic blood pressure <90 mm Hg for
at least 30 minutes, a cardiac index <2.2 L/min/m2 and pulmonary capillary
wedge pressure >15 mm Hg

Go to the next page if you knew the correct answer, or click the link images
below to further research the concepts in this question (if desired).

Research Concepts:
Cardiogenic Shock:

Acute Myocardial Infarction:

Tap flag to report any problems with this question.


Question 185: A patient with angina is taking isosorbide mononitrate for
chronic stable angina but still has elevated blood pressure. Which of the
following should be added?

Choices:
1. Amlodipine
2. Metoprolol
3. Diltiazem
4. Verapamil
Answer: 2 - Metoprolol
Explanations:
Patients with known coronary artery disease benefit from beta blockers and
aspirin
Calcium channel blockers are generally avoided

Go to the next page if you knew the correct answer, or click the link images
below to further research the concepts in this question (if desired).

Research Concepts:
Angina, Stable:

Tap flag to report any problems with this question.


Question 186: A patient in cardiac rehab is doing submaximal exercise.
What is the usual response of diastolic blood pressure with this level of activity?

Choices:
1. It decreases significantly
2. It does not change
3. It increases significantly
4. It is variable but does not change significantly
Answer: 4 - It is variable but does not change significantly
Explanations:
Submaximal exercise does not change diastolic blood pressure significantly
It may increase or decrease less than 10 mm Hg

Go to the next page if you knew the correct answer, or click the link images
below to further research the concepts in this question (if desired).

Research Concepts:
Cardiac Rehabilitation:

Therapeutic Exercise:

Tap flag to report any problems with this question.


Question 187: The artery that arises as a branch of right coronary artery
opposite the origin of the posterior interventricular branch supplies which of the
following structures?

Choices:
1. SA node
2. Bundle of hiss
3. Pectinate muscle
4. AV node
Answer: 4 - AV node
Explanations:
The AV node is supplied by the posterior interventricular artery.
This arises from the right coronary artery in 70% of patients making them
"right dominant".
It arises from the left circumflex artery in 30% of patients making them
"left dominant".

Go to the next page if you knew the correct answer, or click the link images
below to further research the concepts in this question (if desired).

Research Concepts:
Heart Anatomy:

Tap flag to report any problems with this question.


Question 188: INR for all indications should be kept in which of the
following ranges?

Choices:
1. 2.0-3.0
2. 1.0-1.5
3. 2.5-3.5
4. 1.5-2.5
Answer: 1 - 2.0-3.0
Explanations:
INR should be kept between the ranges of 2.0 to 3.0 for most indications
For mechanical prosthetic heart valves INR of 2.5 to 3.5 is recommended
An INR greater than 5 is likely to result in spontaneous bleeding
If the INR is very high or the patient is bleeding, fresh frozen plasma
should be given

Go to the next page if you knew the correct answer, or click the link images
below to further research the concepts in this question (if desired).

Research Concepts:
International Normalized Ratio (INR):

Tap flag to report any problems with this question.


Question 189: Which of the following is least effective for preventing
atrial fibrillation after cardiac surgery?

Choices:
1. Digoxin
2. Amiodarone
3. Sotalol
4. Beta blockers
Answer: 1 - Digoxin
Explanations:
Postoperative atrial fibrillation occurs in anywhere from 20 to 40 percent of
cases. It is often difficult to prevent.
Preoperative use of beta blockers, sotalol, and amiodarone may help reduce
atrial fibrillation in most patients.
Other data indicate that atrial pacing may also reduce the risk of developing
atrial fibrillation.
Most postoperative atrial fibrillation does resolve by the 6th postoperative
week.

Go to the next page if you knew the correct answer, or click the link images
below to further research the concepts in this question (if desired).

Research Concepts:
Atrial Fibrillation:

Tap flag to report any problems with this question.


Question 190: Which group of drugs has been shown to reduce mortality
in patients with congestive heart failure?

Choices:
1. Alpha blockers
2. Calcium channel blockers
3. Diuretics
4. ACE inhibitors
Answer: 4 - ACE inhibitors
Explanations:
ACE Inhibitors have been shown to reduce mortality and morbidity in
congestive heart failure.
ACE inhibitors reduce aldosterone secretion, reduce salt and water
retention, and decrease afterload.
ACE inhibitors are now considered first line drugs for congestive heart
failure.
Beta blockers also decrease mortality.

Go to the next page if you knew the correct answer, or click the link images
below to further research the concepts in this question (if desired).

Research Concepts:
Heart Failure, Congestive:

Tap flag to report any problems with this question.


Question 191: The internal mammary artery arises from what artery?
Choices:
1. Brachiocephalic artery
2. Intercostal artery
3. Mesenteric artery
4. Subclavian artery
Answer: 4 - Subclavian artery
Explanations:
The internal mammary artery arises from the subclavian artery close to its
origin.
The subclavian artery can vary in height, origin, and course.
It usually is above the sternoclavicular articulation but not always.
The left comes off the aorta, is deeper than the right, and usually not as
high.

Go to the next page if you knew the correct answer, or click the link images
below to further research the concepts in this question (if desired).

Research Concepts:
Internal Mammary Artery:

Tap flag to report any problems with this question.


Question 192: Which of the following is not a side effect of protamine?
Choices:
1. Hypertension
2. Bradycardia
3. Flushing
4. Dyspnea
Answer: 1 - Hypertension
Explanations:
Protamine is used to antagonize the effects of heparin.
Protamine does have a few side effects that include hypotension, dyspnea,
bradycardia, and flushing.

Go to the next page if you knew the correct answer, or click the link images
below to further research the concepts in this question (if desired).

Research Concepts:
Anticoagulation:

Tap flag to report any problems with this question.


Question 193: Which is not true regarding anxiety associated with medical
illness?

Choices:
1. Seen with infection, metabolic and endocrine imbalances, pregnancy,
2. Rarely seen with prescription medications
3. Seen with cardiac problems, such as arrhythmias, CHF, MI, hypovolemia,
angina
4. Seen with substance abuse or dependence or withdrawal, even caffeine,
MSG
Answer: 2 - Rarely seen with prescription medications
Explanations:
Many prescription medications cause anxiety-bronchodilators, stimulants,
akathisias from phenothiazine related medications, and antinausea meds,
anticholinergic and digoxin toxicity
Also seen in respiratory problems such as asthma, COPD, pneumothorax
Seen in neurologic problems
Seen in secreting tumors such as carcinoid, pheochromocytoma, insulinoma

Go to the next page if you knew the correct answer, or click the link images
below to further research the concepts in this question (if desired).

Research Concepts:
Anxiety:

Tap flag to report any problems with this question.


Question 194: What is the most effective medication to increase HDL
level?

Choices:
1. Gemfibrozil
2. Niacin
3. Atorvastatin
4. Ezetimibe
Answer: 2 - Niacin
Explanations:
Nicotinic acid increases HDL an average 20 percent
Unfortuntely, this does not seem to change risk of cardiac events and may
increased risk of stroke.

Go to the next page if you knew the correct answer, or click the link images
below to further research the concepts in this question (if desired).

Research Concepts:
Niacin:

Tap flag to report any problems with this question.


Question 195: The lesser saphenous vein usually drains into the:
Choices:
1. Femoral veins
2. Popliteal vein
3. Anterior tibial vein
4. Greater saphenous vein
Answer: 2 - Popliteal vein
Explanations:
The lesser saphenous vein runs behind the leg and passes between the heads
of gastrocnemius muscles and drains into the popliteal vein.
It is a superficial vein and sometimes used for coronary artery bypass
surgery.
Lesser saphenous vein thrombosis is a superficial vein thrombosis, but it is
frequently associated with DVT, which should be evaluated before deciding
to treat with anticoagulation or only with hot compresses and pain relief.
The lesser saphenous vein starts in the lateral marginal foot vein. It moves
up the leg posterior to the lateral malleolus, where it is accompanied by the
sural nerve, both on the lateral aspect of the Achilles tendon.

Go to the next page if you knew the correct answer, or click the link images
below to further research the concepts in this question (if desired).

Research Concepts:
Lesser Saphenous Vein:

Tap flag to report any problems with this question.


Question 196: A 54 year old has a stent placed in the LAD for a proximal
lesion. He has previously had 2 other stents and an angioplasty done to the same
lesion. He now presents with recurrent chest pain. What is the best procedure for
this patient?

Choices:
1. Re stenting
2. Rotablade with thrombolytics
3. Angioplasty with radiation
4. Surgery
Answer: 4 - Surgery
Explanations:
After numerous percutaneous procedures on the same vessel and recurrent
stenosis, it is best to refer this patient for surgery. A LIMA to LAD will
suffice.

Go to the next page if you knew the correct answer, or click the link images
below to further research the concepts in this question (if desired).

Research Concepts:
Restenosis:

Percutaneous Transluminal Coronary Angioplasty:

Tap flag to report any problems with this question.


Question 197: An obese 45-year-old undergoes gastrectomy. Five days
later, he has a foul smelling discharge from his wound. Which of the following is
NOT a contributing factor in this case?

Choices:
1. Malnutrition
2. Use of corticosteroids
3. Diabetics
4. Type of suture
Answer: 4 - Type of suture
Explanations:
Anyone can develop a wound infection postoperatively.
Individuals at high risk for wound infections include diabetics, those on
corticosteroids, and the malnourished.
Both radiation therapy and use of chemotherapeutic drugs are also risk
factors.
There is no correlation between type of suture used and wound infections,
as long as the proper technique of closure has been utilized.

Go to the next page if you knew the correct answer, or click the link images
below to further research the concepts in this question (if desired).

Research Concepts:
Wound Infection:

Tap flag to report any problems with this question.


Question 198: The thoracic duct usually empties into which of the
following vessels?

Choices:
1. Left subclavian vein
2. Right subclavian vein
3. Right atrium
4. Aorta
Answer: 1 - Left subclavian vein
Explanations:
The thoracic duct empties into the left subclavian vein.

Go to the next page if you knew the correct answer, or click the link images
below to further research the concepts in this question (if desired).

Research Concepts:
Thoracic Duct:

Tap flag to report any problems with this question.


Question 199: During an emergency room thoracotomy, tamponade is
suspected and the pericardium is opened. What is the safest technique of opening
the pericardium?

Choices:
1. Make a vertical incision from the top to bottom of pericardium
2. Make a cut in the posterior pericardium to drain blood
3. Make a cut in the horizontal fashion anteriorly
4. Make three cruciate incisions
Answer: 3 - Make a cut in the horizontal fashion anteriorly
Explanations:
Phrenic nerve injury is a real possibility and often this structure is not
visualized
The pericardium must be opened anteriorly in a parallel fashion to the heart
The incision should never be vertical as the chances of cutting the nerve are
high

Go to the next page if you knew the correct answer, or click the link images
below to further research the concepts in this question (if desired).

Research Concepts:
Emergency Room Thoracotomy:

Cardiac Tamponade:

Tap flag to report any problems with this question.


Question 200: Which of the following is not an extubation criterion?
Choices:
1. Mean inspiratory pressure > 20
2. Tidal volume > 5 cc/kg
3. RR > 6 < 22
4. FEV > 1 liter
Answer: 4 - FEV > 1 liter
Explanations:
FEV is not a criterion for extubation.

Go to the next page if you knew the correct answer, or click the link images
below to further research the concepts in this question (if desired).

Research Concepts:
Ventilator Management:

Tap flag to report any problems with this question.


Section 2
Question 201: Advance directives are completed when the patient is:
Choices:
1. In an emergency situation
2. In a coma
3. Hallucinating
4. Competent
Answer: 4 - Competent
Explanations:
Advance directives are to be respected at all time

Go to the next page if you knew the correct answer, or click the link images
below to further research the concepts in this question (if desired).

Research Concepts:
Advance Directives:

Tap flag to report any problems with this question.


Question 202: A 56 year old male is being evaluated for recurrent syncopal
episodes present for the past four months. Cardiac workup is unrevealing until
he becomes symptomatic in the middle of the night. EKG reveals atrial
fibrillation with heart rate of 144/minute. What is the appropriate next step?

Choices:
1. Amiodarone drip
2. IV heparin with immediate transthoracic cardioversion
3. IV metoprolol
4. Adenosine push
Answer: 3 - IV metoprolol
Explanations:
In atrial fibrillation with rapid ventricular response, the most important
immediate issue is rate control best achieved by metoprolol or diltiazem
Cardioversion is an appropriate step, however anticoagulation is generally
advised with heparin for 24 hours prior to transesophageal cardioversion
Adenosine is used in supraventricular tachycardia
Amiodarone is appropriate for maintaining rate control in a converted
patient

Go to the next page if you knew the correct answer, or click the link images
below to further research the concepts in this question (if desired).

Research Concepts:
Paroxysmal Atrial Fibrillation:

Tap flag to report any problems with this question.


Question 203: Which of the following drugs is most efficacious for
treatment of hypertriglyceridemia?

Choices:
1. Cholestyramine
2. Gemfibrozil
3. Niacin
4. Pravastatin
Answer: 3 - Niacin
Explanations:
Niacin is the first drug of choice for high triglycerides as it decreases
cardiovascular events but not overall mortality
Statins are second choices for hypertriglyceridemia
Statins are first choice to lower high LDL levels
Fibrates are used for hypertriglyceridemia but have not been shown to
affect cardiovascular mortality

Go to the next page if you knew the correct answer, or click the link images
below to further research the concepts in this question (if desired).

Research Concepts:
Hypertriglyceridemia:

Tap flag to report any problems with this question.


Question 204: Delirium tremens can be prevented with:
Choices:
1. Ziprasidone
2. Haloperidol
3. Methadone
4. Chlordiazepoxide
Answer: 4 - Chlordiazepoxide
Explanations:
DT is a condition that is caused by alcohol withdrawal in chronic alcoholics
It is characterized by agitated behavior and waxing and waning mentation
Chlordiazepoxide or diazepam are typically used to treat delirium tremens
Methadone is a substitute drug for heroin addicts

Go to the next page if you knew the correct answer, or click the link images
below to further research the concepts in this question (if desired).

Research Concepts:
Delirium Tremens:

Tap flag to report any problems with this question.


Question 205: Which of the following is the best method to evaluate fluid
status?

Choices:
1. Blood pressure
2. Neurological status
3. Weight
4. Skin turgor
Answer: 3 - Weight
Explanations:
Fluid status is best evaluated with daily weight measurements.

Go to the next page if you knew the correct answer, or click the link images
below to further research the concepts in this question (if desired).

Research Concepts:
Fluid Status:

Tap flag to report any problems with this question.


Question 206: In an intubated patient who starts to desaturate, what is the
first step?

Choices:
1. Check pulse oximetry
2. ABG
3. Hand bag the patient and check the machine
4. CXR
Answer: 3 - Hand bag the patient and check the machine
Explanations:
Machine malfunction should be the first thing ruled out.

Go to the next page if you knew the correct answer, or click the link images
below to further research the concepts in this question (if desired).

Research Concepts:
Desaturation:

Mechanical Ventilation:

Tap flag to report any problems with this question.


Question 207: Pulsus paradoxus is:
Choices:
1. Pulses different on each arm
2. Pulses changing with inspiration
3. Pulses not palpable
4. Irregular pulse between upper and lower extremities
Answer: 2 - Pulses changing with inspiration
Explanations:
Pulsus paradoxus is when a pulse becomes more weak upon inhalation and
stronger when exhaling
Can represent disease processes such as COPD, cardiac tamponade, or
pericarditis

Go to the next page if you knew the correct answer, or click the link images
below to further research the concepts in this question (if desired).

Research Concepts:
Pulsus Paradoxus:

Tap flag to report any problems with this question.


Question 208: Digoxin is useful in atrial fibrillation because it slows the
heart down. By what mechanism does digoxin decrease the heart rate?

Choices:
1. It stimulates the SA node.
2. It blocks sodium uptake.
3. It enhances vagal tone on the AV node.
4. It acts centrally and decreases sympathetic drive.
Answer: 3 - It enhances vagal tone on the AV node.
Explanations:
Digoxin acts at the AV node and is thought to enhance the vagal tone,
thereby decreasing the heart rate.

Go to the next page if you knew the correct answer, or click the link images
below to further research the concepts in this question (if desired).

Research Concepts:
Digoxin:

Tap flag to report any problems with this question.


Question 209: Which of the following patients should not receive
influenza virus vaccine?

Choices:
1. HIV patients
2. Young children
3. Elderly
4. A patient who develop Guillain-Barre syndrome after vaccination in the
past
Answer: 4 - A patient who develop Guillain-Barre syndrome after
vaccination in the past

Explanations:
Influenza vaccine is recommended for HIV infected people, the young, the
elderly, and pregnant patients
Contraindications to the inactivated influenza virus include GBS within 6
weeks of immunization, severe allergies to the vaccine or components, or
patients with current infections
If a patient can eat loosely cooked eggs they can receive the vaccination
Adults with comorbidities are given priority for influenza vaccination

Go to the next page if you knew the correct answer, or click the link images
below to further research the concepts in this question (if desired).

Research Concepts:
Influenza Vaccine:

Tap flag to report any problems with this question.


Question 210: A 14 year old boy is seen in the clinic for mild dyspnea.
Echo reveals that he was born with a single atrioventricular valve and yet has
survived without any previous hospital admission. What other anatomical
finding may have helped improve his prognosis?

Choices:
1. Presence of ASD
2. Coarctation of aorta
3. Pulmonary stenosis
4. Right ventricular hypertrophy
Answer: 3 - Pulmonary stenosis
Explanations:
With a single AV valve, there is rapid development of pulmonary
hypertension because of the massive left to right shunt.
When pulmonary hypertension develops at birth, survival is impossible
without a transplant.
If there is pulmonic stenosis, the lungs remain protected but the patient can
still survive.
In congenital heart disease, protection of the lungs from excessive blood
flow is critical. When pulmonary hypertension develops, one needs to
transplant both the lungs and the heart- with a decline in organ donation, the
prognosis is poor.

Go to the next page if you knew the correct answer, or click the link images
below to further research the concepts in this question (if desired).

Research Concepts:
Congenital Heart Disease:

Tap flag to report any problems with this question.


Question 211: pH 7.50; PO2 100 mm Hg; PCO2 23 mm Hg; and HCO3 25
mEq/L. What is the ABG interpretation?

Choices:
1. Uncompensated metabolic alkalosis
2. Compensated respiratory alkalosis
3. Uncompensated respiratory alkalosis
4. Compensated metabolic alkalosis
Answer: 3 - Uncompensated respiratory alkalosis
Explanations:
ABG interpretation is uncompensated respiratory alkalosis
pH is elevated indicating alkalosis
HCO3 is normal ruling out a metabolic component
PCO2 is decreased indicating a respiratory component

Go to the next page if you knew the correct answer, or click the link images
below to further research the concepts in this question (if desired).

Research Concepts:
Respiratory Alkalosis:

Tap flag to report any problems with this question.


Question 212: Which of the following tests is NOT useful in making a
diagnosis of Ehlers-Danlos syndrome?

Choices:
1. Skin biopsy
2. MRI
3. Molecular testing
4. Ultrastructure examination of collagen fibrils
Answer: 1 - Skin biopsy
Explanations:
Skin biopsy findings are non-diagnostic and should not be done.

Go to the next page if you knew the correct answer, or click the link images
below to further research the concepts in this question (if desired).

Research Concepts:
Ehlers-Danlos Syndrome:

Tap flag to report any problems with this question.


Question 213: Which pathology is found in the autopsies of more than
50% of hospitalized patients?

Choices:
1. Cancer
2. Pulmonary embolus
3. Pneumonia
4. Ulcer
Answer: 2 - Pulmonary embolus
Explanations:
More than 650,000 patients have pulmonary embolisms in the U.S. each
year making it the 3rd most common cause of death.
It is possibly the second most common cause in most age groups of
unexpected death. It is common in hospitalized patients.
Autopsy studies show that up to 60% of patients that die in the hospital had
pulmonary embolisms but it had only been diagnosed in 30%.
It is recognized as a common complication postoperatively but also is
common in medical inpatients.

Go to the next page if you knew the correct answer, or click the link images
below to further research the concepts in this question (if desired).

Research Concepts:
Pulmonary Embolism:

Tap flag to report any problems with this question.


Question 214: In a patient with known hereditary angioedema, which of
the following symptoms would be concerning?

Choices:
1. Chest pain
2. Itchy hives
3. Difficulty swallowing
4. Abdominal pain
Answer: 3 - Difficulty swallowing
Explanations:
Hereditary angioedema can often worsen with any type of trauma.
The major course of morbidity is laryngeal edema. A patient complaining of
oral cavity symptoms is an ominous warning.
Angioedema can rapidly progress and cause complete airway obstruction in
no time at all.
Other then swallowing difficulties, patients may complain of changes in
voice, cough, or excess secretions. Intubation and/or tracheostomy are not
uncommon.

Go to the next page if you knew the correct answer, or click the link images
below to further research the concepts in this question (if desired).

Research Concepts:
Hereditary Angioedema:

Tap flag to report any problems with this question.


Question 215: Nonsurgical acute cardiac tamponade is typically a
complication of:

Choices:
1. Rupture of the left ventricle due to myocardial infarction
2. Mural thrombosis
3. Ventricular aneurysm
4. Rupture of the papillary muscle
Answer: 1 - Rupture of the left ventricle due to myocardial infarction
Explanations:
An acute MI can result in rupture of the left ventricular wall and acute
tamponade.
This may end up as a left ventricular aneurysm if the rupture is not
complete.

Go to the next page if you knew the correct answer, or click the link images
below to further research the concepts in this question (if desired).

Research Concepts:
Cardiac Tamponade:

Tap flag to report any problems with this question.


Question 216: A patient smoked 1/3rd pack per dayfor 15 years How many
pack years is this?

Choices:
1. 15 pack years
2. 12.5 pack years
3. 10 pack years
4. 5 pack years
Answer: 4 - 5 pack years
Explanations:
Pack years = number of years the patient smoked X the number of packs
smoked per day.

Go to the next page if you knew the correct answer, or click the link images
below to further research the concepts in this question (if desired).

Research Concepts:
Smoking:

Tap flag to report any problems with this question.


Question 217: Anxiety can:
Choices:
1. Increase myocardial oxygen demand
2. Decrease pulse rate
3. Decrease myocardial oxygen demand
4. None of the above
Answer: 1 - Increase myocardial oxygen demand
Explanations:
Anxiety can increase pulse rate and as a result increase myocardial oxygen
demand
Pain can also increase myocardial oxygen demand
Relieving pain and anxiety are critical for cardiac patients

Go to the next page if you knew the correct answer, or click the link images
below to further research the concepts in this question (if desired).

Research Concepts:
Anxiety:

Myocardial Oxygen Demand:

Tap flag to report any problems with this question.


Question 218: Which of the following about quinidine and procainamide is
FALSE?

Choices:
1. Both can be used to treat atrial and ventricular arrhythmias
2. Both can prolong the PR interval
3. Both can prolong the QRS interval
4. Both can cause lupus
Answer: 4 - Both can cause lupus
Explanations:
Procainamide can cause a lupus like syndrome.
Quinidine can cause cinchonism.
Procainamide induced lupus can be distinguished by checking for
antihistone antibodies.
Other risk factors include use of vitamin D supplements.

Go to the next page if you knew the correct answer, or click the link images
below to further research the concepts in this question (if desired).

Research Concepts:
Antiarrhythmic Medication:

Tap flag to report any problems with this question.


Question 219: Which statement is TRUE concerning pulmonary artery
catheters and central venous pressure lines?

Choices:
1. Pericardial tamponade can be differentiated from tension pneumothorax
using pulmonary artery waveform characteristics
2. Unless there is structural heart disease, a pulmonary capillary wedge
pressure higher than the diastolic pulmonary artery pressure is most likely
an artifact
3. A central venous pressure is as accurate as pulmonary capillary wedge
pressure for monitoring volume status in young healthy patients
4. Pulmonary artery occlusion pressure directly measures left ventricular
filling pressure
Answer: 2 - Unless there is structural heart disease, a pulmonary capillary
wedge pressure higher than the diastolic pulmonary artery pressure is most likely
an artifact

Explanations:
Generally, pulmonary artery diastolic pressure is 1-5 mmHg higher than the
pulmonary artery wedge pressure. A pulmonary capillary wedge pressure
higher than the pulmonary artery diastolic pressure is seen in mitral
regurgitation, otherwise suspect artifact
Both pericardial tamponade and tension pneumothorax would result in
flattened waves and equalization of pressures.

Go to the next page if you knew the correct answer, or click the link images
below to further research the concepts in this question (if desired).

Research Concepts:
Pulmonary Artery Catheterization:

Tap flag to report any problems with this question.


Question 220: Which malignancy is most associated with pericardial
effusions?

Choices:
1. Ovary
2. Pancreas
3. Liver
4. Lung
Answer: 4 - Lung
Explanations:
Lung cancer is most associated with pericardial effusion
Responsible for 37 for the cases

Go to the next page if you knew the correct answer, or click the link images
below to further research the concepts in this question (if desired).

Research Concepts:
Pericardial Effusion:

Tap flag to report any problems with this question.


Question 221: A 69 year old female is found to have a 6 cm abdominal
aortic aneurysm (AAA). Select the correct statement.

Choices:
1. Survival rate for ruptured AAA is 40%
2. Over the next 5 years the patient has a 40% risk of rupture
3. Surgical repair presents normal cardiac risk
4. The AAA should be monitored yearly with ultrasound
Answer: 2 - Over the next 5 years the patient has a 40% risk of rupture
Explanations:
AAA can be followed if there is no pain and if it is <5 to 5.5 cm
The risk of surgery is increased as these patients have CAD but the risk of
rupture outweighs this risk at 5 cm
Of patients with ruptured AAAs only 18% survive surgery
Risk factors for AAA are PVD, hypertension, smoking, family history, male
gender, and increasing age

Go to the next page if you knew the correct answer, or click the link images
below to further research the concepts in this question (if desired).

Research Concepts:
Abdominal Aortic Aneurysm:

Tap flag to report any problems with this question.


Question 222: Which of the following is a potential sign of deep venous
thrombosis (DVT)?

Choices:
1. Waddell sign
2. Trousseau sign
3. Homans sign
4. Popliteal sign
Answer: 3 - Homans sign
Explanations:
Homans sign is a potential indicator of DVT
A positive sign is indicated by pain in the calf with abrupt dorsiflexion of
the foot with the knee flexed 90 degrees
The sign has poor positive and negative predictive value for DVT
As a result, its use has fallen into disfavor

Go to the next page if you knew the correct answer, or click the link images
below to further research the concepts in this question (if desired).

Research Concepts:
Deep Venous Thrombosis, Lower Extremity:

Tap flag to report any problems with this question.


Question 223: Which of the following procedures increase risk for
aspiration pneumonia?

Choices:
1. Nasogastric (NG) tube
2. Placement of an arterial line
3. Chest MRI
4. None of the above
Answer: 1 - Nasogastric (NG) tube
Explanations:
NG tube placement increases risk of aspiration pneumonia
Other procedures increasing risk for aspiration pneumonia include
endotracheal intubation, bronchoscopy, gastrostomy feeding tubes,
tracheostomy and upper endoscopy
All can contribute to colonization of gastric contents or introduce gastric
contents directly into the lungs
Placement of an arterial line or chest MRI do not contribute to aspiration
pneumonia

Go to the next page if you knew the correct answer, or click the link images
below to further research the concepts in this question (if desired).

Research Concepts:
Aspiration Pneumonia:

Nasogastric Tube:

Tap flag to report any problems with this question.


Question 224: Patent ductus arteriosus is closed with:
Choices:
1. Aspirin
2. Indomethacin
3. Prostaglandin
4. Alcohol
Answer: 2 - Indomethacin
Explanations:
The best medical treatment to close patent ductus arteriosus is indomethacin

Go to the next page if you knew the correct answer, or click the link images
below to further research the concepts in this question (if desired).

Research Concepts:
Patent Ductus Arteriosus:

Tap flag to report any problems with this question.


Question 225: Which of the following drugs has the longest half-life?
Choices:
1. Amiodarone
2. Digoxin
3. Gentamicin
4. Lidocaine
Answer: 1 - Amiodarone
Explanations:
The half-life of digoxin is about 36 hours.
The half-life of amiodarone is 26-107 days for parent drug but 61 days for
the active DEA metabolite.
Loading doses are required for drugs with long half-lives
Steady state is reached after 4-5 half lives so it takes up to a week to reach
steady state with digoxin therapy.

Go to the next page if you knew the correct answer, or click the link images
below to further research the concepts in this question (if desired).

Research Concepts:
Digoxin:

Tap flag to report any problems with this question.


Question 226: What is the second most common site of traumatic aortic
injury?

Choices:
1. Aortic arch
2. Ascending aorta
3. Descending aorta
4. Abdominal aorta
Answer: 2 - Ascending aorta
Explanations:
About 10% of aortic transections following trauma occur in the ascending
aorta. Vertical forces of deceleration tend to lead to rupture of the ascending
aorta and arch, whereas horizontal forces lead to rupture of the descending
aorta.
The aortic transection is most common in the distal ascending aorta, near
the origin of the innominate artery. Less commonly, it can occur in the
proximal ascending aorta.
As in the descending aorta, the transection may be complete or partial.
When patients survive the acute transection period, the periaortic hematoma
usually begins to liquefy about 2 weeks after the traumatic event. It is
gradually absorbed back into the aorta and a false aneurysm develops. The
false aneurysm may remain stable for a long period and even calcify, but
eventually it enlarges and ruptures.

Go to the next page if you knew the correct answer, or click the link images
below to further research the concepts in this question (if desired).

Research Concepts:
Aorta, Trauma:

Tap flag to report any problems with this question.


Question 227: Which of the following is an angiotensin II receptor
blocker?

Choices:
1. Captopril
2. Lisinopril
3. Enalapril
4. Olmesartan
Answer: 4 - Olmesartan
Explanations:
Olmesartan is an angiotensin II receptor blocker used in hypertension
Angiotensin II receptor blockers do not affect bradykinin metabolism
These medications do not cause dry cough that is common with use of ACE
inhibitors
Losartan is another angiotensin II receptor blocker used for hypertension

Go to the next page if you knew the correct answer, or click the link images
below to further research the concepts in this question (if desired).

Research Concepts:
Angiotensin II Receptor Blockers:

Tap flag to report any problems with this question.


Question 228: A young female presents to you with a mass in the posterior
mediastinum. She has been complaining of back pain for 3 months. A CT scan
reveals a mass very close to the vertebral column and an MRI is ordered. Which
of the following is incorrect?

Choices:
1. Neurogenic tumors are more common in females
2. Even if these lesions are asymptomatic, they should be removed
3. Infiltration of the vertebral column suggests malignancy
4. Surgery involves a combined neurosurgical and a thoracic procedure
Answer: 3 - Infiltration of the vertebral column suggests malignancy
Explanations:
Neither symptoms nor the invasion of the spinal cord indicates malignancy
with neurogenic tumors.
Neurogenic tumors are more common in females.
Thoracic surgery is done in combination with neurosurgery with the
neurosurgical part taking place first.
An MRI should be done to assess spinal cord invasion.

Go to the next page if you knew the correct answer, or click the link images
below to further research the concepts in this question (if desired).

Research Concepts:
Neurogenic Tumors:

Tap flag to report any problems with this question.


Question 229: Which of the following statements about propofol is true?
Choices:
1. Propofol is water soluble
2. Propofol is ideal for long procedures because it has rapid disappearance of
action
3. Propofol can be used without concern in patients with cardiovascular
disease
4. Propofol increases the risk of postoperative nausea and vomiting
Answer: 2 - Propofol is ideal for long procedures because it has rapid
disappearance of action

Explanations:
Propofol is a lipid soluble intravenous sedative hypnotic
It has rapid onset and rapid disappearance of action
It decreases postoperative nausea and vomiting at low dose
It can cause vasodilatation and hypotension and should be used with
caution in patients with cardiovascular disease

Go to the next page if you knew the correct answer, or click the link images
below to further research the concepts in this question (if desired).

Research Concepts:
Propofol:

Tap flag to report any problems with this question.


Question 230: Which of the following is not a side effect of heparin?
Choices:
1. Thrombosis
2. Hypersensitivity reactions
3. Osteoporosis
4. Bronchoconstriction
Answer: 4 - Bronchoconstriction
Explanations:
A decrease in number of circulating platelets may occur after about 7-10
days after therapy.
Heparin does not cause bronchoconstriction.

Go to the next page if you knew the correct answer, or click the link images
below to further research the concepts in this question (if desired).

Research Concepts:
Anticoagulation:

Tap flag to report any problems with this question.


Question 231: In adults, naloxone doses can be repeated to a maximum of:
Choices:
1. 10 mg
2. 20 mg
3. 30 mg
4. 40 mg
Answer: 1 - 10 mg
Explanations:
In adults, a dose between 0.4 mg to 2 mg can be given every 2-3 minutes
for a maximum of 10 mg
Pediatric dosing for kids less than 5 years or 20 kg is 0.1 mg/kg
Pediatric dosing for kids greater than 5 years or 20 kg is a 2 mg dose is
recommended

Go to the next page if you knew the correct answer, or click the link images
below to further research the concepts in this question (if desired).

Research Concepts:
Naloxone:

Tap flag to report any problems with this question.


Question 232: An elevated international normalized ratio, caused by
excess warfarin, is best treated with which medication?

Choices:
1. Urokinase
2. Ticlopidine
3. Vitamin K
4. Cryoprecipitate
Answer: 3 - Vitamin K
Explanations:
When patients present with an elevated international normalized ratio
following treatment with oral anticoagulants like Warfarin or Coumadin,
the best antidote is vitamin K.
Vitamin K can be given orally, intravenously, or intramuscularly as a 10 mg
dose.
Parenteral administration will correct the international normalized ratio
more quickly.

Go to the next page if you knew the correct answer, or click the link images
below to further research the concepts in this question (if desired).

Research Concepts:
Anticoagulation:

Tap flag to report any problems with this question.


Question 233: Which condition must be excluded before one can make a
definitive diagnosis of tamponade?

Choices:
1. Aortic dissection
2. Hypertrophic cardiomyopathy
3. Constrictive pericarditis
4. Pneumothorax
Answer: 3 - Constrictive pericarditis
Explanations:
Constrictive pericarditis can mimic cardiac tamponade. The diagnosis is
made by obtaining pressures during cardiac catheterization.
If the membrane is thick, it may be visualized on echo.
Other causes that may mimic cardiac tamponade include tension
pneumothorax, pulmonary embolism, effusion pericarditis, and tension
pneumopericardium.
Acute cases of cardiac tamponade need urgent drainage.

Go to the next page if you knew the correct answer, or click the link images
below to further research the concepts in this question (if desired).

Research Concepts:
Cardiac Tamponade:

Tap flag to report any problems with this question.


Question 234: What maintains the patency of the ductus arteriosus?
Choices:
1. Prostaglandins
2. Hypoxia
3. Indomethacin
4. Premature birth
Answer: 1 - Prostaglandins
Explanations:
The ductus arteriosus is patent during fetal life
The patency of the ductus is maintained by continual production of
prostaglandin E2 by the placenta
Prostaglandin antagonism, such as maternal use of NSAIDS, can cause
premature closure of the ductus arteriosus
After birth, the placenta is removed and the major source of prostaglandin
disappears

Go to the next page if you knew the correct answer, or click the link images
below to further research the concepts in this question (if desired).

Research Concepts:
Patent Ductus Arteriosus:

Tap flag to report any problems with this question.


Question 235: How long should an emergent CABG be delayed after a
dose of abciximab?

Choices:
1. Not at all
2. Platelet gtt on call to OR
3. Delay 12 hours
4. Delay 24 to 48 hours
Answer: 1 - Not at all
Explanations:
Abciximab has not been associated with excess bleeding and platelets
should be given only if bleeding is observed.

Go to the next page if you knew the correct answer, or click the link images
below to further research the concepts in this question (if desired).

Research Concepts:
Coronary Artery Bypass Graft:

Abciximab:

Tap flag to report any problems with this question.


Question 236: Diabetes commonly does not damage which of the
following organ systems?

Choices:
1. Nerves
2. Eyes
3. Kidneys
4. Liver
Answer: 4 - Liver
Explanations:
The liver is not affected by diabetes.

Go to the next page if you knew the correct answer, or click the link images
below to further research the concepts in this question (if desired).

Research Concepts:
Diabetes Mellitus, Type 2:

Diabetes Mellitus, Type 1:

Tap flag to report any problems with this question.


Question 237: Which of the following agents is used in hypertensive
emergency?

Choices:
1. Nitroprusside
2. Furosemide
3. Prazosin
4. Nifedipine
Answer: 1 - Nitroprusside
Explanations:
In hypertensive emergency, the high blood pressure needs to be dealt with
swiftly
Nitroprusside is a rapid vasodilating agent that can be used in such cases
Diuretics like furosemide are helpful in long term management
Alpha blockers and dihydropyridines are not suitable for management of
hypertensive emergency

Go to the next page if you knew the correct answer, or click the link images
below to further research the concepts in this question (if desired).

Research Concepts:
Hypertensive Emergencies:

Tap flag to report any problems with this question.


Question 238: What is it called when a practitioner performs a procedure
outside of his/her scope of practice and a negative result occurs?

Choices:
1. Negligence
2. Critical incident
3. Lack of standards
4. Malpractice
Answer: 4 - Malpractice
Explanations:
In order to claim malpractice one must prove professional misconduct
and/or performing a procedure outside the practitioner's scope of practice
Negligence refers to the failure to use a level of care determined to be
reasonable and appropriate under certain circumstances
Critical incidence is an incident that can lead to sever harm if not
recognized and corrected

Go to the next page if you knew the correct answer, or click the link images
below to further research the concepts in this question (if desired).

Research Concepts:
Medical Malpractice:

Tap flag to report any problems with this question.


Question 239: What is the most common adverse reaction to a blood
transfusion?

Choices:
1. Fever
2. Chills
3. Tachycardia
4. Dyspnea
Answer: 1 - Fever
Explanations:
The most common adverse reaction to a blood transfusion is a fever.
Hemolytic reactions include tachycardia, chills, chest pain, back pain,
headache, and cyanosis.

Go to the next page if you knew the correct answer, or click the link images
below to further research the concepts in this question (if desired).

Research Concepts:
Blood Transfusion:

Tap flag to report any problems with this question.


Question 240: The drug used to prevent strokes by inhibiting platelet
aggregation is:

Choices:
1. Warfarin
2. Dextran
3. Clopidogrel
4. Streptokinase
Answer: 3 - Clopidogrel
Explanations:
Clopidogrel, antiplatelet drug, is used to prevent strokes and coronary
restenosis by inhibiting platelet aggregation

Go to the next page if you knew the correct answer, or click the link images
below to further research the concepts in this question (if desired).

Research Concepts:
Clopidogrel:

Tap flag to report any problems with this question.


Question 241: What is the first step in a patient with a tension
pneumothorax on the medical floor?

Choices:
1. Administer IV fluids
2. Chest tube
3. Needle decompression
4. CXR
Answer: 3 - Needle decompression
Explanations:
The first step with tension pneumothorax is to release the air.
Needle thoracotomy is the best method.
Chest tube is placed after needle decompression.

Go to the next page if you knew the correct answer, or click the link images
below to further research the concepts in this question (if desired).

Research Concepts:
Pneumothorax:

Tap flag to report any problems with this question.


Question 242: Which of the following hearts has the best ability to revert
to normal after successful cardiopulmonary bypass and revascularization?

Choices:
1. Stunned myocardium
2. Hibernating myocardium
3. Infarcted myocardium
4. Both 1 and 3
Answer: 2 - Hibernating myocardium
Explanations:
Infarcted myocardium is irreversibly damaged and can not improve with
any interventions, save for transplantation.
Hibernating myocardium is a state of an impaired left ventricle at rest due
to decreased coronary blood flow, which if restored to normal leads to
recovery. This is a salvageable myocardium and the injury is reversible.
Hibernating myocardium is a less severe form of injury and reverts to
normal after coronary artery bypass grafting.
In stunned myocardium, there is left ventricular dysfunction without cell
death. This occurs following restoration of blood flow after an ischemic
episode. This injury is reversible and is thought to occur most likely as a
result of calcium overload, free radicals, and a generalized inflammatory
state. The treatment is supportive, including pharmacology and an intra-
aortic balloon pump.
Stunned myocardium has decreased contractility and decreased diastolic
compliance. In patients without hibernating or stunned myocardium,
medical management or transplant is a better option.

Go to the next page if you knew the correct answer, or click the link images
below to further research the concepts in this question (if desired).

Research Concepts:
Hibernating And Stunned Myocardium:

Coronary Artery Bypass Graft:


Tap flag to report any problems with this question.
Question 243: Which of the following evidence based treatments can help
decrease the risk of coronary disease in a patient known to have atherosclerosis?

Choices:
1. Blood pressure control
2. Control diabetes
3. Exercise
4. Statins
Answer: 4 - Statins
Explanations:
Statins have been shown to lower cholesterol and reduce the risk of
coronary artery disease.
Statins also help reduce weight if the diet is also modified.
Surgery for coronary artery disease is fraught with complications. The
results are average at best.
Exercise and control of blood pressure is also recommended.

Go to the next page if you knew the correct answer, or click the link images
below to further research the concepts in this question (if desired).

Research Concepts:
Atherosclerosis:

Tap flag to report any problems with this question.


Question 244: Which of the following forms a bridge between the
interventricular septum and the base of the anterior papillary muscle?

Choices:
1. Septomarginal trabecula
2. Sinus venarum
3. Ascending aorta
4. Crista terminalis
Answer: 1 - Septomarginal trabecula
Explanations:
The septomarginal trabecula conveys the atrioventricular bundle.
It is a thickening of the right ventricle running from the intraventricular
septum to the anterior papillary muscle.
It is the border between the trabeculated inflow tract and the smooth
outflow tract.
The septal attachment can connect with the supraventricular crest.

Go to the next page if you knew the correct answer, or click the link images
below to further research the concepts in this question (if desired).

Research Concepts:
Heart Anatomy:

Tap flag to report any problems with this question.


Question 245: An unlicensed assistant may not perform which of the
following?

Choices:
1. Take a pulse
2. Take a blood pressure
3. Assess a surgical site
4. Record output
Answer: 3 - Assess a surgical site
Explanations:
A surgical site must be assessed by a licensed member of the health care
team.

Go to the next page if you knew the correct answer, or click the link images
below to further research the concepts in this question (if desired).

Research Concepts:
Licensing:

Preoperative Evaluation And Management:

Tap flag to report any problems with this question.


Question 246: A patient is going for elective laparoscopic
cholecystectomy. How many days before surgery should aspirin be stopped?

Choices:
1. Fifteen days
2. Ten days
3. Three days
4. One day
Answer: 2 - Ten days
Explanations:
Patients are advised to stop NSAID three days before surgery.
Patients are advised to stop aspirin ten days before surgery.

Go to the next page if you knew the correct answer, or click the link images
below to further research the concepts in this question (if desired).

Research Concepts:
Preoperative Evaluation And Management:

Aspirin:

Tap flag to report any problems with this question.


Question 247: Which of the following medical disorders can increase risk
of developing atrial fibrillation?

Choices:
1. Diabetes
2. Renal failure
3. Hyperkalemia
4. Hyperthyroidism
Answer: 4 - Hyperthyroidism
Explanations:
Hyperthyroidism always increases the risk of precipitating atrial fibrillation.
While most patients with atrial fibrillation are investigated for
hyperthyroidism, less than 3% are actually diagnosed with the disorder.
In many cases, if the atrial fibrillation has been going on for more than 12
months, it does not reverse even with normalization of thyroid hormone.
Most patients, however, do benefit from use of beta-blockers.

Go to the next page if you knew the correct answer, or click the link images
below to further research the concepts in this question (if desired).

Research Concepts:
Hyperthyroidism:

Atrial Fibrillation:

Tap flag to report any problems with this question.


Question 248: A patient is experiencing chest pains and has an anterior
myocardial infarction. He begins to show frequent multifocal premature
ventricular contractions (PVCs) on his EKG. Which medication is appropriate?

Choices:
1. Atropine sulfate
2. Adenosine
3. Lidocaine 2 mg po
4. Lidocaine 2 mg/min by continuous infusion
Answer: 4 - Lidocaine 2 mg/min by continuous infusion
Explanations:
Lidocaine 2 mg/min by continuous infusion should be started. Lidocaine
hydrochloride is an anti-arrhythmic medication whose impact on ventricular
arrhythmias is due to its suppression of automaticity (automatic action) and
excitability of the myocardium.
It may be given as a loading dose bolus, then by continuous infusion.
Atropine sulfate is used to treat bradyarrhythmias. Adenosine is used to
treat supraventricular tachyarrhythmias.
Lidocaine is not given in pill form.

Go to the next page if you knew the correct answer, or click the link images
below to further research the concepts in this question (if desired).

Research Concepts:
Myocardial Infarction:

Tap flag to report any problems with this question.


Question 249: Which of the following medications is a potent full agonist
at opioid receptors?

Choices:
1. Propoxyphene
2. Pentazocine
3. Nalbuphine
4. Methadone
Answer: 4 - Methadone
Explanations:
Methadone is a prescription drug that is a full agonist of opioid receptors
It has a long half-life, making it a suitable substitute for heroin
Pentazocine and nalbuphine are partial agonists of opioid
Propoxyphene is a weak full agonist

Go to the next page if you knew the correct answer, or click the link images
below to further research the concepts in this question (if desired).

Research Concepts:
Methadone:

Tap flag to report any problems with this question.


Question 250: The most common valvular disorder in the elderly is which
of the following?

Choices:
1. Mitral stenosis
2. Aortic stenosis
3. Tricuspid regurgitation
4. Pulmonic regurgitation
Answer: 2 - Aortic stenosis
Explanations:
Aortic valve disease is the most common valvular disease in the elderly.
Calcification and thickening of the aortic valve occurs with age.
Aortic sclerosis is a common phenomenon with aging but does not cause
significant obstruction.
About 9% of the geriatric population suffers from hemodynamically
significant aortic stenosis.

Go to the next page if you knew the correct answer, or click the link images
below to further research the concepts in this question (if desired).

Research Concepts:
Aortic Stenosis:

Tap flag to report any problems with this question.


Question 251: Which statement is TRUE about progression of
atherosclerosis after vein graft bypass?

Choices:
1. Most severe progression occurs proximal to site of anastomosis in a grafted
vessel
2. Most severe progression occurs distal to site of anastomosis in a grafted
vessel
3. Most severe progression occurs in the grafted vessel
4. Most severe progression occurs in the proximal part of the grafted vessel
Answer: 1 - Most severe progression occurs proximal to site of anastomosis
in a grafted vessel

Explanations:
Vein graft atherosclerosis progresses faster than native coronary artery
disease.

Go to the next page if you knew the correct answer, or click the link images
below to further research the concepts in this question (if desired).

Research Concepts:
Saphenous Vein Grafts:

Tap flag to report any problems with this question.


Question 252: The major goal in treatment of a patient with heart failure is
to:

Choices:
1. Increase cardiac output
2. Increase vascular tone
3. Decrease hypoxemia
4. Decrease lower extremity edema
Answer: 1 - Increase cardiac output
Explanations:
Increasing cardiac output is the main goal of therapy for patients with heart
failure
Correction of hypoxemia from pulmonary edema and lower extremity
edema will occur as cardiac output is increased

Go to the next page if you knew the correct answer, or click the link images
below to further research the concepts in this question (if desired).

Research Concepts:
Heart Failure:

Cardiac Output:

Tap flag to report any problems with this question.


Question 253: A vasoconstrictor medication is added at an injection site to:
Choices:
1. Increase absorption
2. Decrease absorption
3. Increase elimination
4. Decrease pain
Answer: 2 - Decrease absorption
Explanations:
Vasoconstrictors are added at injection site to slow absorption
Local anesthetics often are mixed with epinephrine to decrease absorption
Procaine is mixed with penicillin for the same reason

Go to the next page if you knew the correct answer, or click the link images
below to further research the concepts in this question (if desired).

Research Concepts:
Vasoconstrictor Medication:

Tap flag to report any problems with this question.


Question 254: Which of the following is true regarding potency of a drug?
Choices:
1. A drug with high potency also has high efficacy
2. A drug with high potency also has high affinity
3. A drug with high potency also has higher steady state of drug concentration
4. A drug with high potency has both high affinity and efficacy
Answer: 2 - A drug with high potency also has high affinity
Explanations:
Potency refers to the amount of drug needed to produce an effect
Highly potent drugs have high affinity
A highly potent drug may not necessarily have a high efficacy

Go to the next page if you knew the correct answer, or click the link images
below to further research the concepts in this question (if desired).

Research Concepts:
Efficacy:

Drug Potency:

Tap flag to report any problems with this question.


Question 255: A patient with a previous TIA has a significant carotid
lesion. Cardiac catheterization reveals critical triple coronary vessel disease.
How do you proceed?

Choices:
1. Carotid surgery first followed by triple vessel bypass in same setting
2. Stage surgery with carotid first followed by the CABG later
3. Stage surgery with CABG first followed by carotid later
4. Stent coronary lesions and do carotid only
Answer: 1 - Carotid surgery first followed by triple vessel bypass in same
setting

Explanations:
If there is critical stenosis in both the carotid artery and the coronary
vessels, it is best to approach both these lesions in the same setting.
The carotid should be done first, the neck left open and then the CABG is
done next. After protamine is given the neck wound is closed.
One can also open the chest, put the sutures in for cannulation and then
proceed with the carotid. If the patient decompensates, the patient is ready
for bypass.

Go to the next page if you knew the correct answer, or click the link images
below to further research the concepts in this question (if desired).

Research Concepts:
Coronary Artery Bypass Graft:

Carotid Artery Stenosis:

Tap flag to report any problems with this question.


Question 256: Which of the following vascular structures are
intrapericardial in location?

Choices:
1. Main pulmonary bifurcation
2. Junction of middle and inferior pulmonary veins on the right
3. Right interlobar artery
4. Azygous arch
Answer: 1 - Main pulmonary bifurcation
Explanations:
The main pulmonary arterial bifurcation into right and left main branches is
within the pericardium
The other listed structures are extrapericardial in location

Go to the next page if you knew the correct answer, or click the link images
below to further research the concepts in this question (if desired).

Research Concepts:
Heart Anatomy:

Tap flag to report any problems with this question.


Question 257: Which findings are not usually seen in pericardial
tamponade?

Choices:
1. Decreased diastolic filling, decreased venous return, RV collapse
2. ECG with decreased voltage, electrical alternans
3. Chest pain, dyspnea, syncope, shock
4. Decreased breath sounds on left
Answer: 4 - Decreased breath sounds on left
Explanations:
Decreased breath sounds on left would be suggestive of pneumothorax or a
R mainstem bronchus intubation
All the others, including CXR with "water bottle" heart, suggest tamponade

Go to the next page if you knew the correct answer, or click the link images
below to further research the concepts in this question (if desired).

Research Concepts:
Cardiac Tamponade:

Tap flag to report any problems with this question.


Question 258: Which of the following does NOT cause cardiac toxicity?
Choices:
1. Daunorubicin
2. Cocaine
3. Adriamycin
4. Marijuana
Answer: 4 - Marijuana
Explanations:
Marijuana is a psychoactive plant. The major active chemicals in marijuana
include tetrahydrocannabinol and cannabinol.
Marijuana can have both physical and psychological effects.
Marijuana is also addictive and may cause episodic memory loss.
Marijuana has no detrimental effect on the heart.

Go to the next page if you knew the correct answer, or click the link images
below to further research the concepts in this question (if desired).

Research Concepts:
Toxic, Cocaine:

Chemotherapy:

Marijuana:
Tap flag to report any problems with this question.
Question 259: Internal cardiac defibrillators are implanted on:
Choices:
1. Midsternum
2. Right chest
3. Left chest
4. Any of the above
Answer: 3 - Left chest
Explanations:
Most ICD pulse generators are implanted on the left side of the chest
Left side requires lower energy

Go to the next page if you knew the correct answer, or click the link images
below to further research the concepts in this question (if desired).

Research Concepts:
Pacemaker, Automatic Internal Cardiac Defibrillator:

Tap flag to report any problems with this question.


Question 260: A common EKG finding in patients with a pulmonary
embolism is:

Choices:
1. Left bundle branch block
2. Supraventricular tachycardia
3. Sinus tachycardia
4. Accelerated junctional rhythm
Answer: 3 - Sinus tachycardia
Explanations:
A common EKG finding in patients diagnosed with pulmonary embolus is
sinus tachycardia.
The classic EKG finding is S Wave in Lead I, Q Wave in Lead III, and T
Wave Inversion in Lead III (S1 Q3 T3, seen in under 20%)

Go to the next page if you knew the correct answer, or click the link images
below to further research the concepts in this question (if desired).

Research Concepts:
Electrocardiogram:

Pulmonary Embolism:

Tap flag to report any problems with this question.


Question 261: What is the best treatment for supraventricular tachycardia?
Choices:
1. Calcium
2. Adenosine
3. Atropine
4. Metoprolol
Answer: 2 - Adenosine
Explanations:
Adenosine terminates supraventricular tachycardia

Go to the next page if you knew the correct answer, or click the link images
below to further research the concepts in this question (if desired).

Research Concepts:
Supraventricular Tachycardia:

Tap flag to report any problems with this question.


Question 262: In a patient who has had atrial flutter for 1 month, what
important study is required?

Choices:
1. Levels of thyroid hormone
2. INR
3. Echocardiogram
4. Coronary angiogram
Answer: 3 - Echocardiogram
Explanations:
When a patient has long standing atrial flutter, blood clots are common in
the ventricles.
As soon as the patient is cardioverted, these blood clots can embolize to the
brain.
Thus, all patients need an echocardiogram prior to cardioversion to ensure
that there are no blood clots.
Echo can also provide information on heart function and structural
abnormalities.

Go to the next page if you knew the correct answer, or click the link images
below to further research the concepts in this question (if desired).

Research Concepts:
Atrial Flutter:

Tap flag to report any problems with this question.


Question 263: Which of the following groups of drugs have been shown to
reduce mortality in patients with congestive heart failure?

Choices:
1. Alpha blockers
2. Calcium channel blockers
3. Diuretics
4. ACE inhibitors
Answer: 4 - ACE inhibitors
Explanations:
ACE inhibitors have been shown to reduce mortality and morbidity in
congestive heart failure.
ACE inhibitors reduce aldosterone secretion, salt and water retention, and
afterload.
ACE inhibitors are now considered first line drugs for congestive heart
failure.
Beta blockers also decrease mortality but are not used during acute
exacerbations.

Go to the next page if you knew the correct answer, or click the link images
below to further research the concepts in this question (if desired).

Research Concepts:
Heart Failure, Congestive:

Tap flag to report any problems with this question.


Question 264: A 58 year old female is referred for evaluation of a grade
IV/VI murmur without symptoms. ECG has nonspecific changes.
Echocardiogram shows aortic stenosis with a gradient of 50 mmHg and left
ventricular hypertrophy. Nuclear medicine stress test reveals no evidence of
ischemia. Select appropriate management.

Choices:
1. Aortic valve replacement now
2. Echocardiography every 6 months
3. Start a beta blocker
4. Start an ACE inhibitor
Answer: 2 - Echocardiography every 6 months
Explanations:
Medical therapy is not indicated in a patient with preserved LV function
Close follow up with echocardiography and identification of syncope,
angina, or CHF is indicated
The patient has a 40% probability of needing aortic valve replacement over
the next 5 years

Go to the next page if you knew the correct answer, or click the link images
below to further research the concepts in this question (if desired).

Research Concepts:
Aortic Stenosis:

Tap flag to report any problems with this question.


Question 265: Why would a patient experiencing shock be treated with
adrenergics?

Choices:
1. To increase cardiac output
2. To increase peripheral vascular resistance
3. To decrease hypertension
4. All of the above
Answer: 1 - To increase cardiac output
Explanations:
Adrenergics increase cardiac output while decreasing peripheral vascular
resistance

Go to the next page if you knew the correct answer, or click the link images
below to further research the concepts in this question (if desired).

Research Concepts:
Shock:

Adrenergic Drugs:

Tap flag to report any problems with this question.


Question 266: Which of the following medications used for treating
congestive heart failure does not confer a mortality benefit?

Choices:
1. Lisinopril
2. Furosemide
3. Metoprolol
4. Spironolactone
Answer: 2 - Furosemide
Explanations:
ACE inhibitors and beta blockers have well demonstrated mortality
reduction benefits
Spironolactone is unique among diuretics in having mortality benefit
whereas loop diuretics, though useful for fluid management does not have
demonstrated mortality benefit

Go to the next page if you knew the correct answer, or click the link images
below to further research the concepts in this question (if desired).

Research Concepts:
Heart Failure, Congestive:

Tap flag to report any problems with this question.


Question 267: Which of the following is not to be used to reduce risk of
deep venous thrombosis?

Choices:
1. Compression TED stockings
2. Early ambulation after surgery
3. Subcutaneous unfractionated heparin
4. Aspirin therapy
Answer: 4 - Aspirin therapy
Explanations:
Low dose subcutaneous unfractionated heparin (UFH)is commonly used for
prophylaxis of venous thromboembolism
Compression stockings are used in elderly patient who have a risk of DVT
but cannot use anticoagulants due to various reasons
Aspirin cannot prevent or reduce DVT

Go to the next page if you knew the correct answer, or click the link images
below to further research the concepts in this question (if desired).

Research Concepts:
Deep Venous Thrombosis Prophylaxis:

Tap flag to report any problems with this question.


Question 268: During exercise the CO2 that is produced is mostly
transported in the blood by which method?

Choices:
1. CO2 is transported as bicarbonate in red cells
2. CO2 is transported as carbaminohemoglobin
3. CO2 is transported as bicarbonate in serum
4. CO2 is transported as CO2 in physical solution
Answer: 3 - CO2 is transported as bicarbonate in serum
Explanations:
Intracellular conversion of CO2 to bicarbonate and hydrogen ion is
catalyzed by carbonic anhydrase, which is found in red blood cells and
other blood cells.
The bicarbonate is exchanged for chloride ion.
Only 5% of CO2 is carried in solution.

Go to the next page if you knew the correct answer, or click the link images
below to further research the concepts in this question (if desired).

Research Concepts:
Hematological CO2 Transport:

Exercise Physiology:

Tap flag to report any problems with this question.


Question 269: What is the major mechanism of class 3 anti-arrhythmic
agents?

Choices:
1. Inhibition of sodium influx
2. Inhibition of l type calcium channels
3. Reduction of the outward phase of the potassium current
4. Inhibition of Na/K ATPase
Answer: 3 - Reduction of the outward phase of the potassium current
Explanations:
The hallmark of class 3 drugs is prolongation of the action potential
duration.
The action potential prolongation by class 3 antiarrhythmic drugs is caused
by blockade of the potassium channels that are responsible for the
repolarization of the action potential.

Go to the next page if you knew the correct answer, or click the link images
below to further research the concepts in this question (if desired).

Research Concepts:
Antiarrhythmic Medication:

Tap flag to report any problems with this question.


Question 270: Which antihypertensive agent may cause hyperkalemia and
require monitoring of potassium?

Choices:
1. Furosemide
2. Diltiazem
3. Lisinopril
4. Hydralazine
Answer: 3 - Lisinopril
Explanations:
All ACE Inhibitors have a tendency to cause hyperkalemia.
Patients who take ACE inhibitors should be regularly monitored for their
potassium levels.
Their use in the presence of renal artery stenosis is contraindicated.

Go to the next page if you knew the correct answer, or click the link images
below to further research the concepts in this question (if desired).

Research Concepts:
Angiotensin Converting Enzyme Inhibitors (ACEI):

Tap flag to report any problems with this question.


Question 271: What is the drug of choice for methicillin resistant
Staphylococci?

Choices:
1. Penicillin V
2. Ceftriaxone
3. Vancomycin
4. Gentamycin
Answer: 3 - Vancomycin
Explanations:
Vancomycin is a glycopeptide that is a bactericidal by inhibiting proper
synthesis of cell wall
Vancomycin is effective against MRSA and antibiotic induced colitis
It requires dose reduction with renal impairment.Toxicity includes
ototoxicity and hypersensitivity reactions

Go to the next page if you knew the correct answer, or click the link images
below to further research the concepts in this question (if desired).

Research Concepts:
Methicillin Resistant Staphylococcus Aureus (MRSA):

Vancomycin:

Tap flag to report any problems with this question.


Question 272: What of the following clinical feature is not associated with
digoxin?

Choices:
1. Increased cardiac contractility
2. Tachycardia
3. Inhibition of the sodium pump
4. Nausea and vomiting
Answer: 2 - Tachycardia
Explanations:
Digoxin has inotropic properties but also blocks atrioventricular node
conduction
The treatment of toxicity involves the use of antibodies or dialysis

Go to the next page if you knew the correct answer, or click the link images
below to further research the concepts in this question (if desired).

Research Concepts:
Digoxin:

Tap flag to report any problems with this question.


Question 273: What effect does aging have on volume of distribution?
Choices:
1. Increased
2. Decreased
3. Variable
4. Unchanged
Answer: 1 - Increased
Explanations:
Volume of distribution is a function of lipid solubility
There is an increased lipid content with the aging

Go to the next page if you knew the correct answer, or click the link images
below to further research the concepts in this question (if desired).

Research Concepts:
Volume of Distribution:

Tap flag to report any problems with this question.


Question 274: A patient with congestive heart failure is started on
furosemide. He should also be placed on what other drug?

Choices:
1. Potassium
2. Calcium channel blocker
3. Nitroglycerin
4. Lisinopril
Answer: 4 - Lisinopril
Explanations:
CHF is treated with diuretics, ACE inhibitors, beta-blockers, and sometimes
digoxin.
The most common diuretics used to treat CHF are loop diuretics.
ACE inhibitors not only reduce symptoms but also prolong life. These
drugs reduce levels of angiotensin 2 and aldosterone.
Diuretics and digoxin are the oldest treatments for CHF.

Go to the next page if you knew the correct answer, or click the link images
below to further research the concepts in this question (if desired).

Research Concepts:
Heart Failure, Congestive:

Tap flag to report any problems with this question.


Question 275: Which of the following congenital heart diseases is
inherited in an autosomal dominant fashion?

Choices:
1. Hypertrophic cardiomyopathy
2. Prolonged QT syndrome
3. Dilated cardiomyopathy
4. Ventricular septal defect
Answer: 1 - Hypertrophic cardiomyopathy
Explanations:
Hypertrophic cardiomyopathy is commonly inherited as a autosomal
dominant fashion
Prolonger QT syndrome is transmitted both as autosomal dominant or
recessive patterns
Dilated cardiomyopathy is usually caused by infections

Go to the next page if you knew the correct answer, or click the link images
below to further research the concepts in this question (if desired).

Research Concepts:
Cardiomyopathy, Hypertrophic:

Tap flag to report any problems with this question.


Question 276: Which special precaution is necessary when dealing with a
diabetic patient undergoing surgery?

Choices:
1. Calculate calorie intake necessary following surgery
2. Switch to parenteral short acting insulin before surgery
3. Measure blood sugar level daily
4. Keep NPO
Answer: 2 - Switch to parenteral short acting insulin before surgery
Explanations:
Surgery is stressful and leads to insulin resistance and high glucose levels
Diabetic patients need parenteral insulin perioperatively.
Blood sugar should be measured perioperatively.
All patients should be kept NPO before major surgery

Go to the next page if you knew the correct answer, or click the link images
below to further research the concepts in this question (if desired).

Research Concepts:
Diabetes Mellitus:

Preoperative Evaluation And Management:

Tap flag to report any problems with this question.


Question 277: What is the best route of administration of epinephrine for
anaphylaxis?

Choices:
1. Intramuscular
2. Oral
3. Subcutaneous
4. Rectal
Answer: 1 - Intramuscular
Explanations:
Intramuscular epinephrine is recommended for anaphylaxis
Absorption of subcutaneous epinephrine is variable in patients with
anaphylaxis
It can be give intravenously but administration should not wait for starting
an IV

Go to the next page if you knew the correct answer, or click the link images
below to further research the concepts in this question (if desired).

Research Concepts:
Anaphylaxis:

Tap flag to report any problems with this question.


Question 278: What is the best treatment for an infected pacemaker?
Choices:
1. Fluid
2. Antibiotics
3. Surgery
4. Observation
Answer: 3 - Surgery
Explanations:
Infected pacemakers must be removed
Antibiotic is indicated with systemic infection
New pacemaker can implant at a new site

Go to the next page if you knew the correct answer, or click the link images
below to further research the concepts in this question (if desired).

Research Concepts:
Pacemaker, Cardiac:

Postoperative Infection:

Tap flag to report any problems with this question.


Question 279: What is a feature of the murmur in a patient with tricuspid
stenosis?

Choices:
1. It is crescendo-decrescendo in nature
2. Is best heard in the axilla
3. Intensity varies with inspiration
4. Is always systolic in nature
Answer: 3 - Intensity varies with inspiration
Explanations:
Tricuspid stenosis can cause an opening snap.
It is a diastolic murmur that varies in intensity with inspiration.
If there is tricuspid regurgitation, as well, the murmur is holosystolic.

Go to the next page if you knew the correct answer, or click the link images
below to further research the concepts in this question (if desired).

Research Concepts:
Tricuspid Stenosis:

Tap flag to report any problems with this question.


Question 280: What is the initial therapy for a patient with ventricular
fibrillation?

Choices:
1. Intrapericardial epinephrine
2. Synchronized cardioversion
3. Unsynchronized defibrillation
4. Insertion of an automatic implanted cardioverter-defibrillator (AICD).
Answer: 3 - Unsynchronized defibrillation
Explanations:
Defibrillation differs in that the shock is not synchronized.
Defibrillation is essential when the rhythm is chaotic and is widely used for
pulseless ventricular tachycardia.
In ventricular fibrillations, patients are generally unconscious and sedation
is not required.

Go to the next page if you knew the correct answer, or click the link images
below to further research the concepts in this question (if desired).

Research Concepts:
Ventricular Fibrillation:

Tap flag to report any problems with this question.


Question 281: An 80-year-old man has a mean arterial pressure of 120 mm
Hg, heart rate of 60 beats per minute, stroke volume of 50 ml, cardiac output of
3000 ml/min, and a right atrial pressure of 0 mm Hg. What is the total peripheral
resistance (TPR)?

Choices:
1. 0.01 mm Hg/mL/min
2. 0.02 mm Hg/mL/min
3. 0.04 mm Hg/mL/min
4. 0.08 mm Hg/mL/min
Answer: 3 - 0.04 mm Hg/mL/min
Explanations:
TPR = 120/3000 = 0.04 mm Hg/mL/min.
Pressure = (flow) x (resistance).

Go to the next page if you knew the correct answer, or click the link images
below to further research the concepts in this question (if desired).

Research Concepts:
Cardiac Physiology:

Tap flag to report any problems with this question.


Question 282: Which of the following is more likely to occur with
neuraxial narcotics, compared to intravenous opioids?

Choices:
1. Respiratory compromise
2. Sedation
3. Nausea and/or vomiting
4. Itching
Answer: 4 - Itching
Explanations:
Itching occurs much more frequently with neuraxial opioids. Itching occurs
in up to 90% of patients with neuraxial opioids. Itching occurs in 15-20% of
patients with intravenous opioids.
The proposed mechanism includes stimulation of an "itch center" in the
medulla or stimulation of opioid receptors in the trigeminal nucleus.
Sedation, nausea and/or vomiting, and respiratory compromise have
approximately the same incidences for both neuraxial and intravenous
opioids.

Go to the next page if you knew the correct answer, or click the link images
below to further research the concepts in this question (if desired).

Research Concepts:
Opioids:

Tap flag to report any problems with this question.


Question 283: In multivessel disease, PTCA versus CABG:
Choices:
1. Incomplete revascularization is predictive of survival
2. Angioplasty has more post procedure complications
3. There is no significant difference in survival
4. PTCA is contraindicated when 2 or more vessels are occluded
Answer: 3 - There is no significant difference in survival
Explanations:
Several studies have shown survival is not significantly different between
these modalities
CABG has a higher rate of postoperative MI and incomplete
revascularization is predictive of repeat procedure not survival

Go to the next page if you knew the correct answer, or click the link images
below to further research the concepts in this question (if desired).

Research Concepts:
Multivessel Disease:

Percutaneous Transluminal Coronary Angioplasty:

Coronary Artery Bypass Graft:


Tap flag to report any problems with this question.
Question 284: Which of the following is an indication for angiography
among those suffering from chronic stable angina?

Choices:
1. Signs and symptoms of congestive heart failure
2. Low likelihood of death from MI
3. Noninvasive testing reveals intermediate-risk
4. An occupational requirement for a definitive diagnosis
Answer: 1 - Signs and symptoms of congestive heart failure
Explanations:
Disabling chronic stable angina despite medical therapy is an indication for
angiography
High-risk criteria on noninvasive testing is another
Patients who have survived sudden cardiac death or have sustained (>30 s)
monomorphic or nonsustained (<30 s) polymorphic ventricular tachycardia
Occupational requirements for a definitive diagnosis are not a compelling
reason

Go to the next page if you knew the correct answer, or click the link images
below to further research the concepts in this question (if desired).

Research Concepts:
Angina, Stable:

Heart Failure, Congestive:

Tap flag to report any problems with this question.


Question 285: Increasing chest pain without exertion is called:
Choices:
1. Variant angina
2. Unstable angina
3. Stable angina
4. Ischemic angina
Answer: 2 - Unstable angina
Explanations:
Unstable angina is characterized by pain without exertion

Go to the next page if you knew the correct answer, or click the link images
below to further research the concepts in this question (if desired).

Research Concepts:
Angina, Unstable:

Tap flag to report any problems with this question.


Question 286: What is the first sign that a patient is developing malignant
hyperthermia after administration of succinylcholine?

Choices:
1. Neuromuscular relaxation
2. Muscle rigidity
3. Metabolic acidosis
4. Jaw clenching
Answer: 4 - Jaw clenching
Explanations:
Trismus is usually the first sign of malignant hyperthermia
A contracture of the chewing musculature of the jaw is a possible early
sign.
Malignant hyperthermia is myopathy that allows large quantities of calcium
to be released from the skeletal muscle sarcoplasmic reticulum.

Go to the next page if you knew the correct answer, or click the link images
below to further research the concepts in this question (if desired).

Research Concepts:
Malignant Hyperthermia:

Tap flag to report any problems with this question.


Question 287: Which test should be done first in a patient with angina?
Choices:
1. Cardiac enzymes
2. Electrocardiogram
3. Chest radiograph
4. Stress test
Answer: 2 - Electrocardiogram
Explanations:
An ECG is usually the first test as it can help narrow the differential
diagnosis of chest pain. ECG helps knowing the base line cardiac status.
If there is still suspicion of coronary disease, then a stress test is usually the
next step.
Stress echo can assess LV function, cardiac size, and valvular heart disease.
Today coronary angiography is the definitive diagnostic test for coronary
artery disease but it is not the first test of choice.

Go to the next page if you knew the correct answer, or click the link images
below to further research the concepts in this question (if desired).

Research Concepts:
Angina, Stable:

Tap flag to report any problems with this question.


Question 288: What is the major difference between a keloid and a
hypertrophic scar?

Choices:
1. Color
2. Keloids do not remain confined to the boundary of the wound
3. Hypertrophic scars are painful
4. Only keloids can be treated with surgery
Answer: 2 - Keloids do not remain confined to the boundary of the wound
Explanations:
Keloids are scars that tend to overgrow.
Unlike a hypertrophic scar, keloids do grow beyond the boundaries of the
original wound.
Both keloids and hypertrophic scars are a result of trauma or surgery.
Both keloids and hypertrophic scars are benign fibrotic tumors that are very
difficult to treat. African Americans and Asians tend to have a propensity to
form keloids.

Go to the next page if you knew the correct answer, or click the link images
below to further research the concepts in this question (if desired).

Research Concepts:
Hypertrophic Scarring:

Keloids:

Tap flag to report any problems with this question.


Question 289: Transesophageal echocardiography reveals an unruptured
sinus of valsalva aneurysm in a patient evaluated for widened mediastinum on
chest x-ray. What is the next best step in managment?

Choices:
1. Chest MRI for confirmation of the aneurysm, and, if confirmed, immediate
surgical consultation
2. Follow up echocardiogram in 6 weeks
3. Holter monitor
4. Referral for outpatient cardiothoracic surgery consultation
Answer: 1 - Chest MRI for confirmation of the aneurysm, and, if confirmed,
immediate surgical consultation

Explanations:
Sinus of valsalva aneurysms can cause compression of adjacent coronary
arteries resulting in myocardial ischemia and infarction.
Aneurysms should be confirmed with cardiac MRI.
Urgent surgical intervention is generally warranted if the aneurysm is
confirmed.

Go to the next page if you knew the correct answer, or click the link images
below to further research the concepts in this question (if desired).

Research Concepts:
Sinus Of Valsalva Aneurysm:

Tap flag to report any problems with this question.


Question 290: An alcoholic undergoes surgery. Which nutritional substrate
is depleted the earliest?

Choices:
1. Branched amino acids
2. Ketones
3. Glycogen
4. Fats
Answer: 3 - Glycogen
Explanations:
Glycogen is primarily made in the liver, muscles, and to some extent, in the
brain.
Glycogen forms as an energy reserve and is quickly mobilized to meet the
sudden need for glucose.

Go to the next page if you knew the correct answer, or click the link images
below to further research the concepts in this question (if desired).

Research Concepts:
Postoperative Care:

Alcoholism:

Tap flag to report any problems with this question.


Question 291: The current recommendations for daily sodium intake is:
Choices:
1. 100 mg
2. 1000 mg
3. 2400 mg
4. 5000 mg
Answer: 3 - 2400 mg
Explanations:
The current recommendations is to consume less than 2400 mg of sodium
per day
This is about one teaspoon of salt per day
There have been studies showing increased mortality in some groups with
more severe restriction.

Go to the next page if you knew the correct answer, or click the link images
below to further research the concepts in this question (if desired).

Research Concepts:
Dietary Sodium:

Tap flag to report any problems with this question.


Question 292: An adult presents with paroxysmal supraventricular
tachycardia and a blood pressure of 80/60. What is the next best step in
management?

Choices:
1. Unsynchronized cardioversion with 150 joules
2. Synchronized cardioversion with 100 joules
3. Unsynchronized cardioversion with 25 joules
4. Synchronized cardioversion with 50 joules
Answer: 4 - Synchronized cardioversion with 50 joules
Explanations:
The patient is suffering from unstable supraventricular tachycardia
The first step in management would be synchronized cardioversion with 50
joules
In stable younger patients, vagal maneuvers can be helpful

Go to the next page if you knew the correct answer, or click the link images
below to further research the concepts in this question (if desired).

Research Concepts:
Paroxysmal Supraventricular Tachycardia:

Tap flag to report any problems with this question.


Question 293: In a 65 year old with a diagnosis of coronary artery disease,
which of the following risk factor interventions has the greatest benefit?

Choices:
1. Lowering of blood pressure
2. Decreasing lipids
3. Controlling blood sugar
4. Decreasing weight
Answer: 2 - Decreasing lipids
Explanations:
Decreasing LDL is said to be of the greatest benefit when it comes to
lowering risk of heart attacks.
All the other factors do help, but lipid reduction is the key.
Thus, patients should be encouraged to change diet and enter an exercise
program.
An HMG CoA reductase inhibitor can help reduce cardiac events and
mortality.

Go to the next page if you knew the correct answer, or click the link images
below to further research the concepts in this question (if desired).

Research Concepts:
Coronary Artery Disease:

Tap flag to report any problems with this question.


Question 294: Which of the following statements is false about prosthetic
valves?

Choices:
1. The incidence of prosthetic valve endocarditis (PVE) is greater in the aortic
position than in the mitral position
2. Native valve endocarditis is more common in the mitral position than in the
aortic position
3. Ring abscess in PVE render antibiotic sterilization difficult
4. Fungal PVE can be treated with aggressive antibiotics
Answer: 4 - Fungal PVE can be treated with aggressive antibiotics
Explanations:
There appears to be no difference in risk of developing PVE between the
use of heterograft or mechanical valves
A significant finding that is universally reported is a greater risk of
developing PVE at the aortic site than at the mitral site, which is opposite to
the situation in native valve endocarditis
The pathologic focus of PVE is most often at the valve seat. Abscess
formation and destruction of tissue are hallmark and often involve the entire
circumference of the valve seat
Valve replacement in addition to antibiotic therapy is recommended for
both early and late PVE with any of the following conditions: by a non-
streptococcal organism, paravalvular leak, CHF, systemic emboli or
recurrence of medically treated infection

Go to the next page if you knew the correct answer, or click the link images
below to further research the concepts in this question (if desired).

Research Concepts:
Prosthetic Heart Valves:

Tap flag to report any problems with this question.


Question 295: Angiotensin-converting enzyme inhibitors (ACE-I) are used
in heart failure because they:

Choices:
1. Decrease afterload (reduce systolic blood pressure)
2. Thin the blood
3. Reduce the heart rate
4. Increase myocardial contractility
Answer: 1 - Decrease afterload (reduce systolic blood pressure)
Explanations:
ACE-Inhibitors reduce systolic blood pressure and after load

Go to the next page if you knew the correct answer, or click the link images
below to further research the concepts in this question (if desired).

Research Concepts:
Heart Failure:

Angiotensin Converting Enzyme Inhibitors (ACEI):

Tap flag to report any problems with this question.


Question 296: Sudden death is often seen in patients with which diagnosis?
Choices:
1. Alport syndrome
2. Hypertrophic cardiomyopathy
3. Restrictive cardiomyopathy
4. Acute pericarditis
Answer: 2 - Hypertrophic cardiomyopathy
Explanations:
Sudden death is the most common devastating presentation of hypertrophic
cardiomyopathy and may be the first clinical manifestation
Sudden death is often most common in children and adolescent children,
typically occurring during sporting activities
Hypertrophic cardiomyopathy is also associated with ventricular fibrillation
and sudden death

Go to the next page if you knew the correct answer, or click the link images
below to further research the concepts in this question (if desired).

Research Concepts:
Hyertrophic Cardiomyopathy:

Tap flag to report any problems with this question.


Question 297: Which of the following congenital cardiac diseases puts
patients at risk for developing a brain abscess?

Choices:
1. VSD
2. ASD
3. Tetralogy of Fallot
4. Patent ductus arteriosus
Answer: 3 - Tetralogy of Fallot
Explanations:
Brain abscess is more common in congenital heart diseases with right to left
shunts
Of the above choices TOF is the only one with right to left shunting

Go to the next page if you knew the correct answer, or click the link images
below to further research the concepts in this question (if desired).

Research Concepts:
Tetralogy Of Fallot:

Brain Abscess:

Tap flag to report any problems with this question.


Question 298: What is the gold standard for diagnosis of deep venous
thrombosis?

Choices:
1. Duplex ultrasound
2. Impedance plethysmography
3. Venogram
4. CT
Answer: 3 - Venogram
Explanations:
Impedance plethysmography and duplex ultrasound are most commonly
used for diagnosis of deep venous thrombosis but venogram is still the gold
standard (as of 2012 analysis).
This is an area that is evolving. In particular keep in mind that contrast
venography is operator-dependent. As the exam is performed less and less
frequently, exam quality may suffer.
Keep in mind when analyzing the literature that by definition no study can
be better than the declared gold standard, simply because when any study
disagrees with the gold standard it is viewed as a false positive rather than
viewing the gold standard result as a false negative.
While a decade ago many centers felt venous LE compression Doppler US
need only study, or was technically only capable of studying, the popliteal
and thigh regions, the current US standard, capable of achievement in most
labs, is to study the entire lower extremity including all calf veins.

Go to the next page if you knew the correct answer, or click the link images
below to further research the concepts in this question (if desired).

Research Concepts:
Deep Vein Thrombosis:

Tap flag to report any problems with this question.


Question 299: What is the classic finding on EKG in pulmonary
embolism?

Choices:
1. Large Q wave in lead III
2. Inverted T wave in lead III
3. Right axis deviation
4. S wave in lead I, Q wave in lead III, and T wave inversion in lead III
Answer: 4 - S wave in lead I, Q wave in lead III, and T wave inversion in
lead III

Explanations:
All of the choices can be seen in pulmonary embolism
The classic S1, Q3, T3 is only seen in less than 20% of cases
Other findings are right bundle branch block and sinus tachycardia

Go to the next page if you knew the correct answer, or click the link images
below to further research the concepts in this question (if desired).

Research Concepts:
Pulmonary Embolism:

Tap flag to report any problems with this question.


Question 300: The intra-aortic balloon pump (IABP) can be utilized for all
except:

Choices:
1. ECMO
2. Supraventricular arrhythmias
3. Bridge to transplant
4. Septic shock
Answer: 2 - Supraventricular arrhythmias
Explanations:
IABP can be used for all except supraventricular arrhythmias
It can, however, be used to treat ventricular arrhythmias
Contraindications include aortic insufficiency, aortic dissection and severe
peripheral vascular disease
If severe PVD is encountered, the IABP can be inserted in to the ascending
aorta

Go to the next page if you knew the correct answer, or click the link images
below to further research the concepts in this question (if desired).

Research Concepts:
Intra-Aortic Balloon Pump:

Tap flag to report any problems with this question.


Question 301: Which drug has both alpha and beta sympathetic blocking
ability and is used to treat hypertension?

Choices:
1. Metoprolol
2. Reserpine
3. Labetalol
4. Prazosin
Answer: 3 - Labetalol
Explanations:
Labetalol is a reversible beta-blocker with concurrent alpha blocking
actions that produces vasodilatation and decreases blood pressure.
Labetalol is an excellent agent for treating the elderly or black hypertensive
patients.
Labetalol does not alter serum lipid or blood glucose levels.
Labetalol is used as an alternative to hydralazine in the treatment of pre-
eclampsia (pregnancy-induced hypertension).

Go to the next page if you knew the correct answer, or click the link images
below to further research the concepts in this question (if desired).

Research Concepts:
Labetalol:

Tap flag to report any problems with this question.


Question 302: Which part of the conducting system of the heart do
electrical unipolar pacemakers simulate?

Choices:
1. The AV node
2. The SA node
3. The bundle of His
4. The atria
Answer: 2 - The SA node
Explanations:
The unipolar pacemaker initiates the heartbeat that is the normal function of
the sino-atrial node
A dual chamber pacemaker first stimulates the atria then the ventricles and
may be more appropriate for those with sick sinus syndrome and AV blocks

Go to the next page if you knew the correct answer, or click the link images
below to further research the concepts in this question (if desired).

Research Concepts:
SA Node:

Pacemaker, Cardiac:

Tap flag to report any problems with this question.


Question 303: Epinephrine causes which of the following?
Choices:
1. Bradycardia
2. Hypotension
3. Decreased urine output
4. Hypertension
Answer: 4 - Hypertension
Explanations:
Epinephrine is a sympathomimetic.
It increases both heart rate and blood pressure.

Go to the next page if you knew the correct answer, or click the link images
below to further research the concepts in this question (if desired).

Research Concepts:
Epinephrine:

Tap flag to report any problems with this question.


Question 304: Select the most common ECG finding in pulmonary
embolism.

Choices:
1. Peaked, tall P waves in II
2. S1 Q3 T3 pattern
3. Atrial fibrillation
4. Sinus tachycardia
Answer: 4 - Sinus tachycardia
Explanations:
There are many findings seen with pulmonary embolism unfortunately the
classic findings are only seen in about 20% of patients
Right hear strain, right axis deviation, right bundle branch block, and the
above findings can all occur

Go to the next page if you knew the correct answer, or click the link images
below to further research the concepts in this question (if desired).

Research Concepts:
Pulmonary Embolism:

Electrocardiogram:

Tap flag to report any problems with this question.


Question 305: A homeless man is found on the street after being knocked
unconscious by vandals. He is unresponsive to verbal commands and is
immediately brought into the emergency room. His vital signs appear stable but
he has a temperature of 29 degrees Celsius. He is rapidly resuscitated to avoid
which major complication of hypothermia?

Choices:
1. Stroke
2. Arrhythmias
3. Renal failure
4. Coagulopathy
Answer: 2 - Arrhythmias
Explanations:
Hypothermia treatment should begin during transport to the hospital. Wet
clothes should be removed and the patient should be placed in a warm
environment with passive rewarming.
Ventricular fibrillation or tachycardia should be treated with defibrillation
one time. If this is unsuccessful, the patient should be rewarmed before
repeat attempts. Automated external defibrillator (AED) defibrillation is
appropriate.
Rapid fluid resuscitation is often needed, as cardiovascular efficiency
benefits from crystalloid administration.
Survival is not improved by barbiturates, steroids, or antibiotics.

Go to the next page if you knew the correct answer, or click the link images
below to further research the concepts in this question (if desired).

Research Concepts:
Hypothermia:

Tap flag to report any problems with this question.


Question 306: What is the diuretic of choice in patients with congestive
heart failure and pulmonary edema?

Choices:
1. Hydrochlorothiazide
2. Metolazone
3. Acetazolamide
4. Furosemide
Answer: 4 - Furosemide
Explanations:
Furosemide is a potent diuretic that is preferred for use in pulmonary
edema.
Hydrochlorothiazide is preferred as a diuretic for use in mild hypertension.
Acetazolamide is used in glaucoma.

Go to the next page if you knew the correct answer, or click the link images
below to further research the concepts in this question (if desired).

Research Concepts:
Congestive Heart Failure And Pulmonary Edema:

Tap flag to report any problems with this question.


Question 307: Obstructive shock can develop from:
Choices:
1. Deep venous thrombosis
2. Aortic dissection
3. Pneumonitis
4. Pericardial effusion
Answer: 4 - Pericardial effusion
Explanations:
Obstructive shock develops from the blockage of great vessels of the
circulatory system

Go to the next page if you knew the correct answer, or click the link images
below to further research the concepts in this question (if desired).

Research Concepts:
Shock, Distributive:

Tap flag to report any problems with this question.


Question 308: Which of the following congenital heart diseases causes
polycythemia?

Choices:
1. VSD
2. ASD
3. Tetralogy of Fallot
4. PDA
Answer: 3 - Tetralogy of Fallot
Explanations:
Tetralogy of fallot is the most common congenital cyanotic heart disease
The chronic hypoxia often causes very high polycythemia and leukocytosis
It is often necessary to perform a therapeutic phlebotomy in such children
to reduce the hematocrit level

Go to the next page if you knew the correct answer, or click the link images
below to further research the concepts in this question (if desired).

Research Concepts:
Polycythemia:

Tetralogy Of Fallot:

Tap flag to report any problems with this question.


Question 309: Where are atherosclerotic aneurysms are most often
located?

Choices:
1. Coronary arteries
2. Ascending aorta
3. Thoracic aorta
4. Abdominal aorta
Answer: 4 - Abdominal aorta
Explanations:
The abdominal aorta is the most common site for atherosclerotic
aneurysms.
Of these, 90 to 95% are below the renal arteries. A large percentage extends
into the common iliac arteries.
The USPSTF recommends a single screening for AAA by ultrasound for
male non-smokers, 65 to75 years of age.
AAA is commonly noted incidentally on CT/US/MR abdominal studies
performed for other reasons. Any such incidental finding should be reported
and measurements provided.

Go to the next page if you knew the correct answer, or click the link images
below to further research the concepts in this question (if desired).

Research Concepts:
Abdominal Aortic Aneurysm:

Tap flag to report any problems with this question.


Question 310: Which is NOT a factor in myocardial oxygen demand?
Choices:
1. Myocardial wall tension (Aortic pressure)
2. Heart rate (Stroke volume)
3. Anemia
4. Inotropy (Contractility)
Answer: 3 - Anemia
Explanations:
The major determinants of myocardial oxygen demand are myocardial wall
tension, heart rate and contractility (aka inotropy)
The heart has a high demand for oxygen
Anemia would affect oxygen supply, not demand

Go to the next page if you knew the correct answer, or click the link images
below to further research the concepts in this question (if desired).

Research Concepts:
Myocardial Oxygen Demand:

Tap flag to report any problems with this question.


Question 311: What is the end result when there is injury to an artery?
Choices:
1. Adventitial fibrosis
2. Medial smooth muscle hyperplasia
3. Intimal hyperplasia
4. Fibromuscular dysplasia
Answer: 2 - Medial smooth muscle hyperplasia
Explanations:
Injury to any blood vessel, results in remodeling.
The remodeling involves smooth muscle hyperplasia.
The mucosal regeneration is a short process to prevent formation of
thrombus.

Go to the next page if you knew the correct answer, or click the link images
below to further research the concepts in this question (if desired).

Research Concepts:
Coronary Artery Disease:

Tap flag to report any problems with this question.


Question 312: Which of the following should lead to intubation of a
patient?

Choices:
1. Respiratory rate 30/min
2. PaCo2= 50 mm Hg
3. Vd/Vt=0.6
4. Vital capacity = 8 ml/kg
Answer: 2 - PaCo2= 50 mm Hg
Explanations:
All the parameters are borderline for intubation but CO2 retention is the
most serious

Go to the next page if you knew the correct answer, or click the link images
below to further research the concepts in this question (if desired).

Research Concepts:
Intubation, Indications:

Tap flag to report any problems with this question.


Question 313: A patient had abdominal surgery and receives six units of
blood. Post-surgery, she complains of numbness around her mouth and
carpopedal spasm. ECG reveals a prolonged QT interval. What is the best
treatment?

Choices:
1. Sodium bicarbonate
2. Potassium
3. Calcium
4. Citrate
Answer: 3 - Calcium
Explanations:
Non-immune related complications of blood transfusions include
hypocalcemia, hyperkalemia, and hemoglobinuria.
The hypocalcemia is due to the citrate. Hypocalcemia is most commonly
seen in trauma patients who require blood transfusions.
Only symptomatic patients need to be treated with intravenous calcium.

Go to the next page if you knew the correct answer, or click the link images
below to further research the concepts in this question (if desired).

Research Concepts:
Blood Transfusion:

Hypocalcemia:

Tap flag to report any problems with this question.


Question 314: Which of the following reduce LDL by decreasing
cholesterol synthesis?

Choices:
1. Niacin
2. Simvastatin
3. Clofibrate
4. Acarbose
Answer: 2 - Simvastatin
Explanations:
HMG-CoA reductase inhibitors can decrease levels of LDL by inhibiting
cholesterol synthesis
This leads to feedback up regulation of LDL receptors in the liver
The increased LDL receptors help to remove LDL from the circulation
Niacin mainly acts on liver synthesis of VLDL and LDL; it does not up
regulate LDL receptors

Go to the next page if you knew the correct answer, or click the link images
below to further research the concepts in this question (if desired).

Research Concepts:
Statin Medication:

Tap flag to report any problems with this question.


Question 315: A patient has had severe asthma for several years. Which of
the following medications could cause severe bronchospasm?

Choices:
1. Acebutolol
2. Terazosin
3. Propranolol
4. Phenoxybenzamine
Answer: 3 - Propranolol
Explanations:
Non-selective beta-blockers such as propranolol can cause bronchospasm
that can be life threatening to an asthmatic
Terazosin is used to treat hypertension and BPH due to the fact that it is an
alpha-adrenergic receptor antagonist
Acebutolol is a beta agonist with intrinsic sympathomimetics activity that
can be used in asthma
Phenoxybenzamine is used to treat pheochromocytoma, as it is a
nonselective alpha-blocker

Go to the next page if you knew the correct answer, or click the link images
below to further research the concepts in this question (if desired).

Research Concepts:
Asthma:

Beta-Blockers:

Tap flag to report any problems with this question.


Question 316: What is the earliest symptom of local anesthetic toxicity?
Choices:
1. Convulsions
2. Respiratory depression
3. Loss of pain
4. Tongue and circumoral numbness
Answer: 4 - Tongue and circumoral numbness
Explanations:
Adverse reactions may occur following administration of local anesthetics
and usually result from administration of too much drug.
Adverse effects usually occur when local anesthetics are injected close to a
vessel. Local anesthetics usually cause toxicity to the central nervous
system and cardiac system.
At high doses, local anesthetics are negatively inotropic and can cause
bradycardia, ventricular fibrillation, hypotension, and asystole.
Cranial nervous system side-effects include light-headedness, tinnitus,
circumoral numbness, a metallic taste, and double vision. Most patients
start to complain of tongue and circumoral numbness as the first signs of
toxicity.

Go to the next page if you knew the correct answer, or click the link images
below to further research the concepts in this question (if desired).

Research Concepts:
Local Anesthetics:

Tap flag to report any problems with this question.


Question 317: Compared to the standard CPR, two-handed heart massage
during an emergency room thoracotomy can increase improve cardiac
performance by what percentage:

Choices:
1. 10%
2. 25%
3. 55%
4. 100%
Answer: 3 - 55%
Explanations:
The two handed technique of CPR can markedly improve heart function
It is nearly 3 times more effective than the one handed CPR technique

Go to the next page if you knew the correct answer, or click the link images
below to further research the concepts in this question (if desired).

Research Concepts:
Open Heart Massage:

Emergency Room Thoracotomy:

Tap flag to report any problems with this question.


Question 318: What is a drug that inhibits the activity of HMG-CoA
reductase used to treat?

Choices:
1. Gout
2. Type 2 Diabetes mellitus
3. Hyperlipidemia
4. Chronic pancreatitis
Answer: 3 - Hyperlipidemia
Explanations:
HMG-CoA reductase inhibitors are termed statins and are used to treat
hypercholesterolemia.
The statins available for use in the United States are simvastatin,
pravastatin, lovastatin, atorvastatin, fluvastatin, rosuvastatin, and
pitavastatin.

Go to the next page if you knew the correct answer, or click the link images
below to further research the concepts in this question (if desired).

Research Concepts:
Statin Medication:

Hyperlipidemia:

Tap flag to report any problems with this question.


Question 319: Which is not true regarding spironolactone?
Choices:
1. Can be used to treat primary hyperaldosteronism
2. Can be used to treat edema and ascites from CHF, liver cirrhosis, or
nephrotic syndrome
3. May cause life-threatening hypokalemia, impotence, gynecomastia
4. Is an aldosterone antagonist
Answer: 3 - May cause life-threatening hypokalemia, impotence,
gynecomastia

Explanations:
May cause life-threatening hyperkalemia due to potassium-sparing effects,
impotence, gynecomastia

Go to the next page if you knew the correct answer, or click the link images
below to further research the concepts in this question (if desired).

Research Concepts:
Spironolactone:

Tap flag to report any problems with this question.


Question 320: Which is not a common cause of a pulseless electrical
activity?

Choices:
1. Pneumothorax
2. Cardiac tamponade
3. Hypervolemia
4. Myocardial infarction
Answer: 3 - Hypervolemia
Explanations:
The common causes of PEA is summarized in the 7 H's and 6 T's
Hypovolemia, hyperkalemia, hypokalemia, hypothermia, hypoglycemia,
hypoxia, and hydrogen ions (acidosis)
Tamponade, toxins, tension pneumothorax, trauma (causing hypovolemia),
tachycardia, thrombosis (PE or MI),

Go to the next page if you knew the correct answer, or click the link images
below to further research the concepts in this question (if desired).

Research Concepts:
Pulseless Electrical Activity:

Tap flag to report any problems with this question.


Question 321: What is meant by right-sided coronary artery dominance?
Choices:
1. The right coronary artery supplies the posterior descending artery, which in
turn supplies the atrioventricular (AV) node
2. The right coronary artery supplies the posterior septum
3. The right coronary artery supplies the sinoatrial (SA) node
4. The right coronary artery supplies > 75% of the right ventricle
Answer: 1 - The right coronary artery supplies the posterior descending
artery, which in turn supplies the atrioventricular (AV) node

Explanations:
If any vessel arises after the posterior descending artery takes off and
supplies the AV nodal artery, the system is right-dominant.
It is important to understand the branches of the right coronary artery. The
conus is the first branch of the right coronary artery. It may come off a
separate coronary ostia in the right sinus, cross the right ventricular outflow
tract anteriorly, and run to the pulmonic valve.
It is an important collateral to the left anterior descending artery. The sinus
node artery runs the opposite way to the conus and supplies the SA node in
about 60% of patients.
In the rest, the sinus node artery may come off the distal circumflex. The
next artery to come off the right coronary is the acute marginal artery,
which is visible anteriorly.

Go to the next page if you knew the correct answer, or click the link images
below to further research the concepts in this question (if desired).

Research Concepts:
Coronary Artery Imaging:

Tap flag to report any problems with this question.


Question 322: Which of the following finding is suggestive of a small
ventricular septal defect, rather than a large one?

Choices:
1. Loud murmur
2. Murmur that radiates to the axilla
3. Presence of thrill
4. Presence of apical heave
Answer: 1 - Loud murmur
Explanations:
Small VSDs usually cause a louder blowing type of murmur than large
VSDs
The other features may be present with any kind of VSDs

Go to the next page if you knew the correct answer, or click the link images
below to further research the concepts in this question (if desired).

Research Concepts:
Ventricular Septal Defect:

Tap flag to report any problems with this question.


Question 323: What is the most common side effect associated with
niacin?

Choices:
1. Edema
2. Hypoglycemia
3. Flushing
4. Tinnitus
Answer: 3 - Flushing
Explanations:
Flushing is the most common side effect associated with niacin and leads to
noncompliance with the medication amongst patients.
This can be reduced by pretreating with aspirin.

Go to the next page if you knew the correct answer, or click the link images
below to further research the concepts in this question (if desired).

Research Concepts:
Niacin:

Niacin:

Tap flag to report any problems with this question.


Question 324: What is the term for atrial fibrillation that spontaneously
terminates?

Choices:
1. Persistent atrial fibrillation
2. Paroxysmal
3. Sustained
4. Recurrent
Answer: 2 - Paroxysmal
Explanations:
When atrial fibrillation recurs 2 or more times, it is termed recurrent.
When atrial fibrillation spontaneously stops, this is paroxysmal.
If the fibrillation is sustained, this is persistent.

Go to the next page if you knew the correct answer, or click the link images
below to further research the concepts in this question (if desired).

Research Concepts:
Atrial Fibrillation:

Tap flag to report any problems with this question.


Question 325: Which option is not part of Beck triad is seen with cardiac
tamponade.

Choices:
1. Jugular venous distention
2. Hypotension
3. Decreased heart sounds
4. Pulmonary hypertension
Answer: 4 - Pulmonary hypertension
Explanations:
Beck triad of cardiac tamponade includes jugular venous distention,
hypotension, and muffled heart sounds

Go to the next page if you knew the correct answer, or click the link images
below to further research the concepts in this question (if desired).

Research Concepts:
Cardiac Tamponade:

Tap flag to report any problems with this question.


Question 326: Which one of the following drug enhances the effect of
warfarin?

Choices:
1. Rifampin
2. Omeprazole
3. Famotidine
4. Vitamin K
Answer: 2 - Omeprazole
Explanations:
Omeprazole inhibits the cytochrome P-450 system there by increasing
warfarin levels
Warfarin levels might be dangerously elevated with potent liver enzyme
inhibitors so INR must be monitored or alternate medication used
Vitamin K reverses the effect of warfarin
Rifampin is a potent liver enzyme inducer

Go to the next page if you knew the correct answer, or click the link images
below to further research the concepts in this question (if desired).

Research Concepts:
Warfarin, Drug Interactions:

Omeprazole:

Tap flag to report any problems with this question.


Question 327: A Swan Ganz, or pulmonary artery, catheter is most likely to
cause hemoptysis in a patient with which condition?

Choices:
1. Tuberculosis
2. Acute respiratory distress syndrome
3. Pulmonary hypertension
4. Congestive heart failure
Answer: 3 - Pulmonary hypertension
Explanations:
Patients who have pulmonary hypertension are prone to pulmonary artery
rupture.
A pulmonary artery catheter should usually not be used in patients with
sustained pulmonary artery hypertension.
When a pulmonary artery rupture occurs, death is inevitable.
If the diagnoses is suspected, one may be able to tamponade the bleeding
with the balloon catheter until the surgeon can safely take the patient to the
operating room.

Go to the next page if you knew the correct answer, or click the link images
below to further research the concepts in this question (if desired).

Research Concepts:
Hemoptysis:

Swan-Ganz Catheterization:

Tap flag to report any problems with this question.


Question 328: Tissue transplant from another individual is termed:
Choices:
1. Allograft
2. Autograft
3. Xenograft
4. Isograft
Answer: 1 - Allograft
Explanations:
Autograft is a self tissue transplant
Allograft is tissue transplant from another person
Xenograft is tissue transplant between species
Isograft is between identical twins

Go to the next page if you knew the correct answer, or click the link images
below to further research the concepts in this question (if desired).

Research Concepts:
Allograft:

Tap flag to report any problems with this question.


Question 329: To protect against deep vein thrombosis, heparin
prophylaxis should be started at what dose?

Choices:
1. 5000 U q8h
2. 5000 U q4h
3. 5000 U q24h
4. 500 U q6h
Answer: 1 - 5000 U q8h
Explanations:
Deep vein thromboses are very prevalent after major surgery, especially
pelvic and lower limb surgeries.
Heparin is often administered at a dose of 5000 U q8h before surgery and
continued until the patient is ambulatory.

Go to the next page if you knew the correct answer, or click the link images
below to further research the concepts in this question (if desired).

Research Concepts:
Deep Venous Thrombosis Prophylaxis:

Tap flag to report any problems with this question.


Question 330: Which of the following is most increased with ventricular
tachycardia (VT)?

Choices:
1. Myocardial infarction (MI)
2. Pulmonary embolism (PE)
3. Sudden death
4. COPD
Answer: 3 - Sudden death
Explanations:
VT is associated with an increased risk of sudden death.
Stable VT should be treated with intravenous procainamide or sotalol.
Lidocaine is often used especially if there is known impairment of left
ventricular function.
VT is usually a consequence of ischemic or structural heart disease or
electrolyte deficiencies.
Unstable patients with monomorphic VT should be treated with
synchronized direct current cardioversion.

Go to the next page if you knew the correct answer, or click the link images
below to further research the concepts in this question (if desired).

Research Concepts:
Ventricular Tachycardia:

Tap flag to report any problems with this question.


Question 331: In the elderly individual, the most common cause of aortic
stenosis is:

Choices:
1. Bicuspid valve
2. Rheumatic fever
3. Degenerative valve disease
4. Endocarditis
Answer: 3 - Degenerative valve disease
Explanations:
Degenerative valve disease is the most common cause of aortic stenosis in
the elderly
Bicuspid aortic valve is a more common cause of AS in middle-aged
individuals
Both rheumatic valve disease and endocarditis produce aortic regurgitation

Go to the next page if you knew the correct answer, or click the link images
below to further research the concepts in this question (if desired).

Research Concepts:
Aortic Stenosis:

Aortic Stenosis:

Tap flag to report any problems with this question.


Question 332: What is the primary therapy for Prinzmetal angina?
Choices:
1. Metoprolol
2. Nifedipine
3. Surgery
4. Stenting
Answer: 2 - Nifedipine
Explanations:
Nitrates and calcium channel blockers are the mainstay of treatment for
coronary spasm.
All calcium channel blockers are effective for coronary spasms.
Beta blockers are preferred in patients with atherosclerotic coronary disease
and angina.
Nitrates produce a direct endothelium vasodilatation and reduce preload.

Go to the next page if you knew the correct answer, or click the link images
below to further research the concepts in this question (if desired).

Research Concepts:
Angina, Prinzmetal:

Tap flag to report any problems with this question.


Question 333: A 56-year-old male is presents to the clinic and is found to
have coronary artery disease. In the majority of cases, what is the most common
presentation of patients with CAD?

Choices:
1. Acute heart failure
2. Syncope
3. Chest pain
4. Discovered incidentally
Answer: 4 - Discovered incidentally
Explanations:
In the majority of patients, coronary artery disease is discovered while
being worked up for another medical disorder.
Many individuals do have CAD but have no idea because they are
asymptomatic.
The process of atherosclerosis begins in the second or third decade of life
and progresses slowly.
Only 10-20% of patients with CAD presents acutely with chest pain and
other symptoms.

Go to the next page if you knew the correct answer, or click the link images
below to further research the concepts in this question (if desired).

Research Concepts:
Coronary Artery Disease:

Tap flag to report any problems with this question.


Question 334: A 59-year old female is placed on digoxin for atrial
fibrillation. She is seen in the medical clinic and you decide to add another drug
to her therapy. Which of the following drugs can increase the toxicity of
digoxin?

Choices:
1. Potassium supplements
2. Vasotec
3. Vitamin D
4. Quinidine
Answer: 4 - Quinidine
Explanations:
Quinidine is known to enhance the toxicity of digoxin by displacing
digoxin from the protein binding sites.
Quinidine is rarely used today primarily because of the availability of safer
drugs.
Quinidine is a class 1 anti-arrhythmic agent and works by blocking the fast
sodium current.
Quinidine toxicity can lead to thrombocytopenia, tinnitus, and torsades.

Go to the next page if you knew the correct answer, or click the link images
below to further research the concepts in this question (if desired).

Research Concepts:
Antiarrhythmic Medication:

Digoxin:

Tap flag to report any problems with this question.


Question 335: With which of the following is purulent pericarditis a
possible complication?

Choices:
1. Coxsackie virus infection
2. Sepsis
3. Myocardial infarction
4. Tuberculosis
Answer: 2 - Sepsis
Explanations:
Sepsis can occasionally be a cause of purulent pericarditis.
Bacterial pericarditis usually results from pulmonary extension,
hematogenous spread, myocardial abscess, or endocarditis.
The organisms typically responsible for purulent pericarditis include strep,
staph, and gram-negative organisms.
Even when treated, many individuals develop constrictive pericarditis.
Surgery may be required to remove the thickened, fibrosed pericardium.

Go to the next page if you knew the correct answer, or click the link images
below to further research the concepts in this question (if desired).

Research Concepts:
Pericarditis, Bacterial:

Tap flag to report any problems with this question.


Question 336: Where are the majority of deep venous thrombi located?
Choices:
1. In the superior vena cava
2. In the popliteal veins
3. In the inferior vena cava
4. In the tibial/peroneal veins
Answer: 4 - In the tibial/peroneal veins
Explanations:
The majority of deep venous clots occur in the lower extremity and pelvis.
The clinical diagnosis of deep venous thrombosis is difficult and diagnostic
tests are required.
A thrombus that occurs at the iliac vein bifurcation or in the pelvic veins
can produce bilateral edema and swelling. This is often mistaken for heart
failure.
Venous ultrasound is the best initial test and rapidly assesses venous flow,
compressibility, and the presence of luminal echoes.

Go to the next page if you knew the correct answer, or click the link images
below to further research the concepts in this question (if desired).

Research Concepts:
Deep Venous Thrombosis, Lower Extremity:

Tap flag to report any problems with this question.


Question 337: Acute pericarditis is not complicated by:
Choices:
1. Cardiac tamponade
2. Constrictive pericarditis
3. Coronary artery disease
4. Recurrent pericarditis
Answer: 3 - Coronary artery disease
Explanations:
Acute pericarditis does not cause coronary artery disease but extrinsic
compression can limit coronary flow

Go to the next page if you knew the correct answer, or click the link images
below to further research the concepts in this question (if desired).

Research Concepts:
Pericarditis, Acute:

Tap flag to report any problems with this question.


Question 338: Which of the following can depress the chemoreceptor
trigger zone?

Choices:
1. Morphine
2. Prochlorperazine
3. Imipramine
4. Apomorphine
Answer: 2 - Prochlorperazine
Explanations:
Prochlorperazine blocks dopamine receptors and is indicated to treat nausea
and vomiting associated with surgery, chemotherapy, radiation, and other
conditions.
It is also indicated to treat psychosis.
Off label, it is used to treat migraines.
At low doses, about 2% of patients experience extrapyramidal side effects
but at high doses, it can be as much as 40%.

Go to the next page if you knew the correct answer, or click the link images
below to further research the concepts in this question (if desired).

Research Concepts:
Prochlorperazine:

Tap flag to report any problems with this question.


Question 339: What agent is added to increase the half-life of imipenem?
Choices:
1. Cilastatin
2. Probenecid
3. Allopurinol
4. Sodium bicarbonate
Answer: 1 - Cilastatin
Explanations:
Cilastatin inhibits of renal dehydropeptidase I.
This enzyme causes the hydrolysis of imipenem compounds with no
antibacterial activity that are nephrotoxic.
It has no antimicrobial activity itself but maintains serum and urine levels
of the intact medication.

Go to the next page if you knew the correct answer, or click the link images
below to further research the concepts in this question (if desired).

Research Concepts:
Imipenem Cilastatin Combination:

Tap flag to report any problems with this question.


Question 340: An patient with recent cardiac surgery should avoid what
activity for at least 4 weeks?

Choices:
1. Showering
2. Driving
3. Sleeping on their chest
4. Lifting anything greater than 10 pounds
Answer: 4 - Lifting anything greater than 10 pounds
Explanations:
Most cardiac surgical patients have had median sternotomies, making
avoidance of heavy lifting imperative in the post operative period

Go to the next page if you knew the correct answer, or click the link images
below to further research the concepts in this question (if desired).

Research Concepts:
Cardiac Surgery:

Tap flag to report any problems with this question.


Question 341: In addition to ventricular septal defect and pulmonary
stenosis, which of the following are also included inTetralogy of Fallot?

Choices:
1. Right ventricular hypertrophy and dextroposition of the aorta
2. Left ventricular hypertrophy and dextroposition of the aorta
3. Right ventricular hypertrophy and anomalous right coronary artery origin
4. Left ventricular hypertrophy and anomalous right coronary artery origin
Answer: 1 - Right ventricular hypertrophy and dextroposition of the aorta
Explanations:
The other components of the TOF include aortic overriding and right
ventricular hypertrophy.
In rare cases an atrial septal defect is present. This is called the pentad of
tetralogy.

Go to the next page if you knew the correct answer, or click the link images
below to further research the concepts in this question (if desired).

Research Concepts:
Tetralogy Of Fallot:

Tap flag to report any problems with this question.


Question 342: Only patients that have an overdose of benzodiazepines
should be given:

Choices:
1. Flumazenil
2. Naloxone
3. Activated charcoal
4. Dopamine
Answer: 1 - Flumazenil
Explanations:
Flumazenil (Romazicon) is used to reverse effects of benzodiazepines
Flumazenil used with other medications may induce seizure
Naloxone is used for opioid overdose.

Go to the next page if you knew the correct answer, or click the link images
below to further research the concepts in this question (if desired).

Research Concepts:
Toxic, Benzodiazepine:

Flumazenil:

Tap flag to report any problems with this question.


Question 343: What kind of shock is seen with acute myocardial
infarction?

Choices:
1. Neurogenic
2. Septic
3. Cardiogenic
4. Hypovolemic
Answer: 3 - Cardiogenic
Explanations:
Acute myocardial infarction can result in cardiogenic shock
Prolonged diarrhea or vomiting can result in hypovolemic shock
Systemic infections can result in septic shock
Vasovagal reactions and general anesthesia can result in neurogenic shock

Go to the next page if you knew the correct answer, or click the link images
below to further research the concepts in this question (if desired).

Research Concepts:
Cardiogenic Shock:

Acute Myocardial Infarction:

Tap flag to report any problems with this question.


Question 344: Which of the following studies is NOT used for a patient
with a widened mediastinum after injury?

Choices:
1. CT scan
2. Upper endoscopy
3. Transesophageal ultrasound
4. Nerve conduction studies
Answer: 4 - Nerve conduction studies
Explanations:
In an injured patient with a widened mediastinum, several structures of
great importance may be injured.
Possible causes of a widened mediastinum include aortic transection, aortic
pseudoaneurysm, esophageal rupture, mediastinal mass, cardiac tamponade,
pericardial hemorrhage, and thoracic vertebral fractures.
CT scans (including CT angiography), cardiac ultrasound, endoscopy, or
bronchoscopy may be required to assess the patient.
Nerve conduction studies are not used to assess patients with a widened
mediastinum. Nerve conduction studies may be used in patients who are
undergoing thoracic aortic aneurysm repair to assess spinal cord viability.

Go to the next page if you knew the correct answer, or click the link images
below to further research the concepts in this question (if desired).

Research Concepts:
Mediastinal Injury:

Tap flag to report any problems with this question.


Question 345: Which of the following medication may cause
thrombocytopenia?

Choices:
1. Warfarin
2. Heparin
3. Ampicillin
4. Doxycycline
Answer: 2 - Heparin
Explanations:
Heparin is known to cause severe thrombocytopenia
Thrombocytopenia is a well-recognized complication of heparin therapy
Warfarin is associated with bleeding and skin necrosis
Doxycycline is associated with bone staining

Go to the next page if you knew the correct answer, or click the link images
below to further research the concepts in this question (if desired).

Research Concepts:
Thrombocytopenia, Heparin Induced:

Tap flag to report any problems with this question.


Question 346: Which drug is most likely to interact with lisinopril?
Choices:
1. Furosemide
2. Spironolactone
3. Acetazolamide
4. Indapamide
Answer: 2 - Spironolactone
Explanations:
ACE inhibitors can result in hyperkalemia due to decreased aldosterone
production.
Aldosterone normally acts to decrease potassium levels.
Spironolactone is a potassium sparing diuretic.
Administration of lisinopril and spironolactone may lead to hyperkalemia.

Go to the next page if you knew the correct answer, or click the link images
below to further research the concepts in this question (if desired).

Research Concepts:
Spironolactone:

ACE Inhibitors:

Tap flag to report any problems with this question.


Question 347: In what position should a patient be placed after a left lung
collapse secondary to aspiration?

Choices:
1. Left lateral decubitus
2. Right lateral decubitus
3. Trendelenburg
4. Lithotomy
Answer: 2 - Right lateral decubitus
Explanations:
The proper patient positioning is healthy side down.
Right lateral decubitus is the best position for this patient.
This position will maximize perfusion to the right lung.
Supine is the next best position.

Go to the next page if you knew the correct answer, or click the link images
below to further research the concepts in this question (if desired).

Research Concepts:
Pneumonia, Aspiration:

Tap flag to report any problems with this question.


Question 348: Which of the following can result from excessive use of low
tidal volume treatment?

Choices:
1. Hypercapnic respiratory acidosis
2. Hypocapnic respiratory acidosis
3. Hypercapnic respiratory alkalosis
4. Hypocapnic respiratory alkalosis
Answer: 1 - Hypercapnic respiratory acidosis
Explanations:
Excess use of low tidal volume can result in relative hypoventilation.
Hypoventilation can result in hypercapnia with development of respiratory
acidosis.
pH levels of 7.20 are tolerated but lower levels need treatment with
bicarbonate.
Sedation is required with hypercapnia.

Go to the next page if you knew the correct answer, or click the link images
below to further research the concepts in this question (if desired).

Research Concepts:
Mechanical Ventilation:

Tidal Volume:

Tap flag to report any problems with this question.


Question 349: An autopsy of an elderly person reveals that there is brown
atrophy of the heart. Which of the following explains this finding?

Choices:
1. Patients with myocardial infarction
2. Hemochromatosis
3. Normal part of aging
4. Coarctation of aorta
Answer: 3 - Normal part of aging
Explanations:
Brown atrophy of the heart is seen in elderly individuals.
The disorder is due to deposition of pigmented lipofuscin in the cell nuclei.
This finding is normal and part of aging.

Go to the next page if you knew the correct answer, or click the link images
below to further research the concepts in this question (if desired).

Research Concepts:
Cardiac Aging:

Tap flag to report any problems with this question.


Question 350: What is the probably of a child affected by Carney complex
with positive family history?

Choices:
1. 0 percent
2. 25 percent
3. 50 percent
4. 100 present
Answer: 3 - 50 percent
Explanations:
Carney complex is an autosomal dominant syndrome so a child with an
affected parent has 50% of chance of the disease

Go to the next page if you knew the correct answer, or click the link images
below to further research the concepts in this question (if desired).

Research Concepts:
Carney Complex:

Tap flag to report any problems with this question.


Question 351: What is the most common cause of hemolytic transfusion
reaction?

Choices:
1. ABO incompatibility
2. Rh factor incompatibility
3. Clerical error
4. Expired blood
Answer: 3 - Clerical error
Explanations:
The number one cause of hemolytic transfusion reaction is clerical error

Go to the next page if you knew the correct answer, or click the link images
below to further research the concepts in this question (if desired).

Research Concepts:
Hemolytic Transfusion Reaction:

Tap flag to report any problems with this question.


Question 352: When does the typical wound infection occur after surgery?
Choices:
1. 1 day
2. 5 days
3. 15 days
4. 20 days
Answer: 2 - 5 days
Explanations:
Typical wound infections occurs five days after surgery
Most common organism is staphylococcus

Go to the next page if you knew the correct answer, or click the link images
below to further research the concepts in this question (if desired).

Research Concepts:
Post-op Wound Infection:

Tap flag to report any problems with this question.


Question 353: Which of the following medication can cause dry cough as
an adverse effect?

Choices:
1. Digoxin
2. Quinidine
3. Lisinopril
4. Nitroglycerin
Answer: 3 - Lisinopril
Explanations:
Angiotensin-converting enzyme (ACE) inhibitors commonly cause cough
Digoxin can cause arrhythmias
Nitroglycerin can cause headaches
Quinidine is known to cause torsades de pointes

Go to the next page if you knew the correct answer, or click the link images
below to further research the concepts in this question (if desired).

Research Concepts:
ACE Inhibitors:

Cough:

Tap flag to report any problems with this question.


Question 354: What is the highest energy setting for defibrillation during
sudden cardiac arrest?

Choices:
1. 200 joules
2. 250 joules
3. 360 joules
4. 500 joules
Answer: 3 - 360 joules
Explanations:
Current protocols call for no more than 360 joules with defibrillation of
sudden cardiac arrest patients
The initial energy setting should be 200 joules
Protocols gradually increase the energy to a maximum of 360 joules
500 joules would be inappropriate in this setting

Go to the next page if you knew the correct answer, or click the link images
below to further research the concepts in this question (if desired).

Research Concepts:
Cardiac Arrest:

Tap flag to report any problems with this question.


Question 355: A 75 year old female with a history of cardiac arrhythmias
has been started on a new medication and develops a facial rash, anemia, fever,
and joint pain. Which of the following is the most likely medication?

Choices:
1. Sotalol
2. Quinidine
3. Amiodarone
4. Procainamide
Answer: 4 - Procainamide
Explanations:
Procainamide causes drug induced lupus with positive anti histone antibody
in 95%
Symptoms include facial rash, hemolytic anemia, fever, and joint pain
Antinuclear antibodies occur in 50% of patients treated long term

Go to the next page if you knew the correct answer, or click the link images
below to further research the concepts in this question (if desired).

Research Concepts:
Lupus Erythematosus, Drug-Induced:

Tap flag to report any problems with this question.


Question 356: Most patients experience some digoxin toxicity at serum
levels above:

Choices:
1. 0.5 ng/mL
2. 1 ng/mL
3. 1.5 ng/mL
4. 2 ng/mL
Answer: 4 - 2 ng/mL
Explanations:
Serum digoxin levels above 2 ng/mL may cause digoxin toxicity in most
patients
Acute toxicity may lead to GI disturbances and potentially dangerous
arrhythmias
It is also important to investigate serum electrolyte levels if toxicity is
suspected

Go to the next page if you knew the correct answer, or click the link images
below to further research the concepts in this question (if desired).

Research Concepts:
Digoxin Toxicity:

Tap flag to report any problems with this question.


Question 357: Which of the following is the most common physical exam
finding with pulmonary embolus (PE)?

Choices:
1. Lower extremity edema
2. Tachypnea
3. Diaphoresis
4. None of the above
Answer: 2 - Tachypnea
Explanations:
Tachypnea is present in 96% of patients diagnosed with PE
Diaphoresis is present in 36%
Lower extremity edema is present in 24%
Tachycardia is present in 44%

Go to the next page if you knew the correct answer, or click the link images
below to further research the concepts in this question (if desired).

Research Concepts:
Pulmonary Embolism:

Tap flag to report any problems with this question.


Question 358: Patients with large size pleural effusions often present with:
Choices:
1. Dizziness
2. Dyspnea
3. Tachycardia
4. Palpitations
Answer: 2 - Dyspnea
Explanations:
Other pleural effusion sign and symptoms include neck vein distention,
ventricular gallop (S3), tachypnea, and egophony on exam.

Go to the next page if you knew the correct answer, or click the link images
below to further research the concepts in this question (if desired).

Research Concepts:
Pleural Effusion:

Tap flag to report any problems with this question.


Question 359: Which of the following is an action of morphine?
Choices:
1. Analgesia
2. Gastroparesis
3. Itching
4. All of the above
Answer: 4 - All of the above
Explanations:
Morphine is a narcotic.
Morphine acts by binding opioid receptors.
Most morphine receptors are located in the posterior amygdala, thalamus,
and hypothalamus.

Go to the next page if you knew the correct answer, or click the link images
below to further research the concepts in this question (if desired).

Research Concepts:
Morphine:

Tap flag to report any problems with this question.


Question 360: What is the treatment of choice for ventricular fibrillation?
Choices:
1. Basic life support
2. Cardioversion
3. Immediate defibrillation
4. Lidocaine
Answer: 3 - Immediate defibrillation
Explanations:
Immediate defibrillation is the treatment of choice for ventricular
defibrillation
Ventricular fibrillation is the most commonly identified arrhythmia in
cardiac arrest patients
It is most often associated with coronary artery disease
Patients with a history of ventricular fibrillation require implantation of an
ICD

Go to the next page if you knew the correct answer, or click the link images
below to further research the concepts in this question (if desired).

Research Concepts:
Ventricular Fibrillation:

Tap flag to report any problems with this question.


Question 361: Which group of drugs has been shown to reduce mortality
in patients with congestive heart failure?

Choices:
1. Beta blockers
2. Calcium channel blockers
3. Diuretics
4. ACE inhibitors
Answer: 4 - ACE inhibitors
Explanations:
ACE Inhibitors have been shown to reduce mortality and morbidity in CHF.
These agents reduce aldosterone secretion, salt and water retention and
decrease afterload.
ACE inhibitors are now considered first line drugs for CHF.

Go to the next page if you knew the correct answer, or click the link images
below to further research the concepts in this question (if desired).

Research Concepts:
Angiotensin Converting Enzyme Inhibitors (ACEI):

Heart Failure, Congestive:

Tap flag to report any problems with this question.


Question 362: All of the following can be a complication of pulmonary
embolus (PE) EXCEPT:

Choices:
1. Pulmonary infarction
2. Pleural effusion
3. Lung carcinoma
4. Pulmonary hypertension
Answer: 3 - Lung carcinoma
Explanations:
Pulmonary embolus is a common condition and usually a result of DVT or
venous thromboembolic disease
It can lead to pulmonary infarction, pleural effusion or pulmonary
hypertension
The most common symptom of pulmonary infarction is hemoptysis
Lung carcinoma is not a complication of PE

Go to the next page if you knew the correct answer, or click the link images
below to further research the concepts in this question (if desired).

Research Concepts:
Pulmonary Embolism:

Tap flag to report any problems with this question.


Question 363: With which of the following is Stevens-Johnson syndrome
associated?

Choices:
1. Erythema nodosum
2. Erythema multiforme
3. Erythema chronicum migrans
4. Psoriasis
Answer: 2 - Erythema multiforme
Explanations:
Stevens-Johnson syndrome is potentially life-threatening and may present
with the same target lesions of erythema multiforme

Go to the next page if you knew the correct answer, or click the link images
below to further research the concepts in this question (if desired).

Research Concepts:
Erythema Multiforme (Stevens-Johnson Syndrome):

Tap flag to report any problems with this question.


Question 364: What cardiac change is associated with primary pulmonary
hypertension?

Choices:
1. Tricuspid stenosis
2. Right ventricular hypertrophy
3. Left ventricular hypertrophy
4. Aortic regurgitation
Answer: 2 - Right ventricular hypertrophy
Explanations:
Primary pulmonary hypertension carries an unknown etiology
Associated with right ventricular hypertrophy
The mean survival less than four years from the time of diagnosis

Go to the next page if you knew the correct answer, or click the link images
below to further research the concepts in this question (if desired).

Research Concepts:
Pulmonary Hypertension, Idiopathic:

Tap flag to report any problems with this question.


Question 365: What drug intervention is used in the management of aortic
dissection?

Choices:
1. Alpha methyldopa
2. Diltiazem
3. Nitroprusside
4. Furosemide
Answer: 3 - Nitroprusside
Explanations:
Aortic dissection is a surgical emergency. It requires immediate control of
blood pressure.
Various drug combinations have been used in the past. The combination of
nitroprusside and a short acting beta-blocker (esmolol) are the most useful.
Trimethaphan is a ganglionic blocker that can also be used in the treatment
of blood pressure.
In the ICU setting, either a nitroglycerin or nitroprusside drip can be used to
lower blood pressure.

Go to the next page if you knew the correct answer, or click the link images
below to further research the concepts in this question (if desired).

Research Concepts:
Nitroprusside:

Dissection, Aortic:

Tap flag to report any problems with this question.


Question 366: Which medication should not be withheld prior to vascular
surgery?

Choices:
1. Oral hypoglycemics
2. Plavix
3. Beta blockers
4. Garlic tablets
Answer: 3 - Beta blockers
Explanations:
Beta blockers, if not started in the immediate preoperative period, have
been shown to be beneficial in terms of morbidity and mortality in
cardiovascular surgery.
Oral hypoglycemic medications may cause intraoperative hypoglycemia.
Certain OTC medications may cause hypocoagulation.

Go to the next page if you knew the correct answer, or click the link images
below to further research the concepts in this question (if desired).

Research Concepts:
Preoperative Evaluation And Management:

Vascular Surgery:

Tap flag to report any problems with this question.


Question 367: All of the following are causes of embolic stroke EXCEPT:
Choices:
1. Cardiac arrhythmia
2. Atheromatous plaques of carotid artery
3. Lambl excrescences
4. Septic emboli from endocarditis
Answer: 1 - Cardiac arrhythmia
Explanations:
Additional sources are atrial fibrillation, arterial dissection and atrial
myxoma
Arterial dissection can knock off pieces of plaque that become emboli.
Plaque is made up of fat, cholesterol and calcium. Arterial dissection
sometime results in a partial to complete blockage without any emboli
Lambl excrescences are small filamentous tumor-like growths on the heart
valves that start as small thrombi

Go to the next page if you knew the correct answer, or click the link images
below to further research the concepts in this question (if desired).

Research Concepts:
Embolic Stroke:

Tap flag to report any problems with this question.


Question 368: In a patient with hypotension, which factor will not be
released?

Choices:
1. BNP
2. Aldosterone
3. ADH
4. Angiotensin I
Answer: 1 - BNP
Explanations:
BNP is a marker for CHF.

Go to the next page if you knew the correct answer, or click the link images
below to further research the concepts in this question (if desired).

Research Concepts:
Hypotension:

Tap flag to report any problems with this question.


Question 369: Of the following signs or symptoms, which of the following
would be most commonly found in a patient with mitral regurgitation?

Choices:
1. Substernal chest pain
2. Recurrent syncope
3. Dyspnea on exertion
4. Pulsus paradoxus
Answer: 3 - Dyspnea on exertion
Explanations:
Weight gain due to fluid retention is common during heart failure
Patients often develop exertional dyspnea

Go to the next page if you knew the correct answer, or click the link images
below to further research the concepts in this question (if desired).

Research Concepts:
Mitral Regurgitation:

Dyspnea on Exertion (DOE):

Tap flag to report any problems with this question.


Question 370: For what condition is digoxin NOT used?
Choices:
1. Atrial flutter
2. Atrial fibrillation
3. CHF
4. Ventricular tachycardia
Answer: 4 - Ventricular tachycardia
Explanations:
Digoxin is the classic drug for CHF.
However, it is now being replaced with better and safer drugs.
The drug has a tendency to cause toxicity which can be aggravated by the
use of diuretics and quinidine.
Digoxin has also been used in the treatment of atrial fibrillation and flutter.

Go to the next page if you knew the correct answer, or click the link images
below to further research the concepts in this question (if desired).

Research Concepts:
Digoxin:

Tap flag to report any problems with this question.


Question 371: Which of the following is a common cardiac presentation of
digitalis intoxication?

Choices:
1. Atrial flutter
2. Atrial fibrillation
3. Mobitz type II second degree AV block
4. Ventricular arrhythmias
Answer: 4 - Ventricular arrhythmias
Explanations:
The earliest sign of toxicity is ventricular ectopy, usually manifest by
premature ventricular contractions.
Increased automaticity may produce atrial tachyarrhythmias and AV nodal
depression, which frequently produces a high-degree block.
Ventricular arrhythmias (both ventricular tachycardia and ventricular
fibrillation) may be seen in digoxin toxicity.
Atrial flutter, atrial fibrillation, and Mobitz type II second degree AV block
are unlikely to be caused by digitalis toxicity.

Go to the next page if you knew the correct answer, or click the link images
below to further research the concepts in this question (if desired).

Research Concepts:
Digoxin Toxicity:

Tap flag to report any problems with this question.


Question 372: A patient develops a reaction to his heart medication
including fever, joint pain, anemia, and a facial rash. He has a history of
ventricular arrhythmias. Which of the following is the most likely the
medication?

Choices:
1. Quinidine
2. Amiodarone
3. Digoxin
4. Procainamide
Answer: 4 - Procainamide
Explanations:
Procainamide can cause a "lupus-like" reaction including fever, arthralgias,
hemolytic anemia, and rash.
Quinidine toxicity causes torsades-de-pointe and cinchonism or a syndrome
of headache, tinnitus, psychosis, and headache.
Amiodarone can cause hypotension, bradycardia, AV block, pulmonary or
hepatic toxicity, and optic neuritis.

Go to the next page if you knew the correct answer, or click the link images
below to further research the concepts in this question (if desired).

Research Concepts:
Procainamide:

Lupus Erythematosus, Drug-Induced:

Tap flag to report any problems with this question.


Question 373: Which of the following is most effective in a patient with
ischemic Prinzmetal angina?

Choices:
1. Propranolol
2. NTG
3. ACE Inhibitor
4. Nifedipine
Answer: 4 - Nifedipine
Explanations:
Prinzmetal angina is also known as variant angina and is usually caused by
vasospasm of the coronary vessels.
Prinzmetal tends to be more frequent in young females and often occurs at
rest.
The majority of patients already have some degree of atherosclerosis in the
coronary vessels but the disease is mild in nature.
Prinzmetal responds to calcium channel blockers, particularly nifedipine. In
the rare patient it may respond to nitroglycerin but the response is very
short lived.

Go to the next page if you knew the correct answer, or click the link images
below to further research the concepts in this question (if desired).

Research Concepts:
Angina, Prinzmetal:

Tap flag to report any problems with this question.


Question 374: Hemodynamic findings associated with hypovolemic shock
include:

Choices:
1. Tachycardia and hypotension
2. Bradycardia and increased cardiac index
3. Tachycardia and elevated PCWP
4. Elevated central venous pressure and hypotension
Answer: 1 - Tachycardia and hypotension
Explanations:
Hemodynamic findings associated with hypovolemic shock include
tachycardia and hypotension

Go to the next page if you knew the correct answer, or click the link images
below to further research the concepts in this question (if desired).

Research Concepts:
Hypovolemic Shock:

Tap flag to report any problems with this question.


Question 375: What is the most appropriate treatment in the case of a
elderly male with history of myocardial infarction three weeks ago who presents
complaining of pleuritic chest pain who is found to have a mild fever, a normal
physical exam, and normal ECG?

Choices:
1. Anxiolytics
2. Proton pump inhibitors
3. Nitrates PRN
4. Anti-inflammatory drugs
Answer: 4 - Anti-inflammatory drugs
Explanations:
This case is likely due to Dressler syndrome, also known as post
myocardial infarction syndrome, which is a benign autoimmune pleuritis or
pericarditis that occurs up to 2-3 weeks after MI
It occurs most commonly following open heart surgery, ST elevation MI, or
if there was pericardial bleeding
Therapy is rest and NSAIDs or sometimes glucocorticoids

Go to the next page if you knew the correct answer, or click the link images
below to further research the concepts in this question (if desired).

Research Concepts:
Myocardial Infarction:

Dressler Syndrome:

Tap flag to report any problems with this question.


Question 376: In general, acute transplant rejection occurs in what time
period?

Choices:
1. Minutes
2. Days
3. Months
4. Years
Answer: 3 - Months
Explanations:
Hyperacute transplant rejection occurs within minutes.
Acute rejection occurs within weeks to several months.
Chronic rejection usually occurs after several years.

Go to the next page if you knew the correct answer, or click the link images
below to further research the concepts in this question (if desired).

Research Concepts:
Acute Transplant Rejection:

Tap flag to report any problems with this question.


Question 377: Sodium nitroprusside is used for which of the following
illnesses?

Choices:
1. Hypertensive emergency
2. High output failure
3. Arteriovenous shunting
4. Aortic coarctation
Answer: 1 - Hypertensive emergency
Explanations:
All of the above conditions may be associated with hypertension
Hypertensive emergency is characterized by severe hypertension and organ
damage
Blood pressure should be reduced by nitroprusside
Nitroprusside is contraindicated for treatment of compensatory
hypertension

Go to the next page if you knew the correct answer, or click the link images
below to further research the concepts in this question (if desired).

Research Concepts:
Sodium Nitroprusside:

Hypertension, Malignant:

Tap flag to report any problems with this question.


Question 378: Which of the following structures is not in the
mediastinum?

Choices:
1. Phrenic nerve
2. Cardiac nerve
3. Lungs
4. Thymus
Answer: 3 - Lungs
Explanations:
The mediastinum is the central compartment of the thoracic cavity.
In it are the heart, great vessels, esophagus, trachea, phrenic nerve, thoracic
duct, cardiac nerve, thymus, and lymph nodes.
These are surrounded by loose connective tissue.
The lungs are not in the mediastinum.

Go to the next page if you knew the correct answer, or click the link images
below to further research the concepts in this question (if desired).

Research Concepts:
Mediastinum Anatomy:

Tap flag to report any problems with this question.


Question 379: Which of following medications is a class III antiarrhythmic
drug?

Choices:
1. Disopyramide
2. Timolol
3. Sotalol
4. Verapamil
Answer: 3 - Sotalol
Explanations:
Sotalol is a class III antiarrhythmic drug
Sotalol is a beta blocker, but it also acts on potassium channels

Go to the next page if you knew the correct answer, or click the link images
below to further research the concepts in this question (if desired).

Research Concepts:
Antiarrhythmic Medication:

Sotalol:

Tap flag to report any problems with this question.


Question 380: Which of the following is not a risk factors for
atherosclerosis?

Choices:
1. Advanced age
2. Heredity
3. Diabetes
4. Use of estrogens
Answer: 4 - Use of estrogens
Explanations:
Estrogens are not a risk factor for atherosclerosis.

Go to the next page if you knew the correct answer, or click the link images
below to further research the concepts in this question (if desired).

Research Concepts:
Atherosclerosis:

Tap flag to report any problems with this question.


Question 381: Which of the following patients is most likely to test
positive for Epstein-Barr virus (EBV) infection?

Choices:
1. A patient with T-cell lymphoma of the nasal cavity
2. A patient with B-cell lymphoma of the nasal cavity
3. A patient diagnosed with posttransplant immunoproliferative disorders
4. 1 and 3
Answer: 4 - 1 and 3
Explanations:
T-cell lymphoma is frequently of natural killer (NK) cell phenotype. NK
cells perform cell-mediated cytotoxicity without prior sensitization or major
histocompatibility complex.
Epstein Barr virus infection causes NK cell activation and migrate to the
nasal mucosa.
It is theorized that a single clone of NK cells with other molecular accidents
proliferate without regulation and become an NK cell lymphoma. This has
been observed with EBV and NK cell proliferation in post-transplant
immunoproliferative disorders.
EBV infection has not been shown to be linked to NK cell and T cell
lymphomas.

Go to the next page if you knew the correct answer, or click the link images
below to further research the concepts in this question (if desired).

Research Concepts:
Posttransplantation Lymphoproliferative Disorders:

T-Cell Lymphoma:
Tap flag to report any problems with this question.
Question 382: Which of the following is not a clinical feature of
hypocalcemia?

Choices:
1. Laryngospasm
2. Prolonged QT interval on EKG
3. Positive Trousseau sign
4. Short PR interval on EKG
Answer: 4 - Short PR interval on EKG
Explanations:
Laryngospasm, prolonged QT interval on EKG, and a positive Trousseau
sign are common findings in hypocalcemia
Short PR interval is not a typical sign on EKG
The signs and symptoms of acute hypocalcemia typically result from
increased neuromuscular irritability
Other signs and symptoms include seizures, irritability, impaired school
performance, and behavioral changes.

Go to the next page if you knew the correct answer, or click the link images
below to further research the concepts in this question (if desired).

Research Concepts:
Hypocalcemia:

Tap flag to report any problems with this question.


Question 383: Apolipoproteins are proteins that when combined with fats
form lipoproteins such as HDL, LDL, VLDL and chylomicrons. Which of the
following apolipoproteins is involved in forming LDL and VLDL?

Choices:
1. Apo A
2. Apo B
3. Apo C
4. Apo E
Answer: 2 - Apo B
Explanations:
Apolipoprotein B (Apo B) is the major protein involved in forming LDL
and VLDL.
Apolipoprotein A (Apo A) is the major protein in HDL.
Apolipoprotein C (Apo C) is involved in activating lipoprotein lipase.

Go to the next page if you knew the correct answer, or click the link images
below to further research the concepts in this question (if desired).

Research Concepts:
Apolipoprotein-B:

Tap flag to report any problems with this question.


Question 384: A patient with a history of palpitations, a regular cardiac
rate and rhythm, and a holosystolic murmur along the left sternal border that
increases with Valsalva maneuver has as the most likely cause of symptoms:

Choices:
1. Hypertrophic obstructive cardiomyopathy
2. Mitral valve prolapse
3. Dilated cardiomyopathy
4. Atrial fibrillation
Answer: 1 - Hypertrophic obstructive cardiomyopathy
Explanations:
Hypertrophic cardiomyopathy may be associated with atrial fibrillation and
ventricular tachycardia.
The murmur of hypertrophic obstructive cardiomyopathy is a systolic
ejection murmur that worsens with the Valsalva maneuver.

Go to the next page if you knew the correct answer, or click the link images
below to further research the concepts in this question (if desired).

Research Concepts:
Hypertrophic Obstructive Cardiomyopathy:

Tap flag to report any problems with this question.


Question 385: Which of the following is not an effect of dobutamine on
the cardiovascular system?

Choices:
1. Increases cardiac contractility
2. Increases cardiac output
3. Decreases peripheral resistance
4. Increases peripheral resistance
Answer: 4 - Increases peripheral resistance
Explanations:
Dobutamine is beta 1-adrenergic agonist, with weak beta 2 activity, and
alpha 1 selective activity
Dobutamine can be used in CHF to increase cardiac output
Its primary mechanism is direct stimulation of ?1 receptors
It does not cause increased peripheral resistance

Go to the next page if you knew the correct answer, or click the link images
below to further research the concepts in this question (if desired).

Research Concepts:
Dobutamine:

Tap flag to report any problems with this question.


Question 386: The middle cardiac vein which ascends in the posterior
interventricular groove is accompanied by which artery?

Choices:
1. Left coronary artery
2. Right coronary artery
3. Thoracic artery
4. Internal carotid
Answer: 2 - Right coronary artery
Explanations:
The middle cardiac vein commences at the apex of the heart.
It ascends in the posterior longitudinal sulcus, and ends in the coronary
sinus near its right extremity.
It is accompanied by the right coronary artery in the posterior
interventricular groove.

Go to the next page if you knew the correct answer, or click the link images
below to further research the concepts in this question (if desired).

Research Concepts:
Heart Anatomy:

Tap flag to report any problems with this question.


Question 387: Vancomycin-resistant Enterococcus faecium (VRE)
infection is treated with:

Choices:
1. Imipenem-Cilastatin
2. Ciprofloxacin
3. Linezolid
4. Clarithromycin
Answer: 3 - Linezolid
Explanations:
Linezolid can be used to treat Staphylococcus aureus including MRSA or
Streptococcus pneumonia
It is also used for treatment of vancomycin-resistant Enterococcus faecium
(VRE) infection
The other medications would not be effective for a high profile resistant
bacteria such as VREs

Go to the next page if you knew the correct answer, or click the link images
below to further research the concepts in this question (if desired).

Research Concepts:
Vancomycin Resistant Enterococcus (VRE):

Tap flag to report any problems with this question.


Question 388: An intra-aortic balloon pump will decrease which of the
following?

Choices:
1. Cardiac output
2. Myocardial oxygen demand
3. Heart rate
4. Coronary perfusion
Answer: 2 - Myocardial oxygen demand
Explanations:
An intra-aortic pump improves cardiac output and reduces oxygen demand.

Go to the next page if you knew the correct answer, or click the link images
below to further research the concepts in this question (if desired).

Research Concepts:
Intra-Aortic Balloon Pump:

Tap flag to report any problems with this question.


Question 389: Which statement is most accurate regarding the use of
patient restraints?

Choices:
1. They are the preferred way to manage agitated patients
2. They are used instead of sedative medications
3. They are the best way to preserve a patient's autonomy
4. They are indicated when a patient's behavior poses a significant risk of
damage to equipment
Answer: 4 - They are indicated when a patient's behavior poses a significant
risk of damage to equipment

Explanations:
Patient restraints are indicated when a patient's behavior poses a significant
risk of damage to equipment or injury to himself or others
They violate a patient's autonomy and should be used only when other
methods of behavior control have failed
Sedatives should generally accompany the use of restraints

Go to the next page if you knew the correct answer, or click the link images
below to further research the concepts in this question (if desired).

Research Concepts:
Patient Restraint:

Tap flag to report any problems with this question.


Question 390: A patient undergoes surgery on his nose. He is found to
have subcutaneous emphysema in the right chest. Which of the following is not
appropriate?

Choices:
1. Right thoracotomy
2. Chest x-ray
3. Chest tube
4. Endoscopy
Answer: 1 - Right thoracotomy
Explanations:
Subcutaneous emphysema feels like crunchy skin with crackles. It may
result from air escaping from the respiratory tract or the gastrointestinal
tract.
Most cases of subcutaneous emphysema occur in the intensive care unit in
ventilated patients who develop pneumothorax. In such cases, the air starts
to track up underneath the skin.
Symptoms may include difficulty swallowing, neck pain, and sore throat. If
there is subcutaneous emphysema in the presence of a chest tube, this
indicates that the tube is not functioning or is blocked.
Most cases of subcutaneous emphysema are benign but one should obtain a
chest x-ray to ensure there is no pneumothorax. The chest tube should be
repositioned. Breathing oxygen helps the body absorb subcutaneous air.

Go to the next page if you knew the correct answer, or click the link images
below to further research the concepts in this question (if desired).

Research Concepts:
Subcutaneous Emphysema:

Tap flag to report any problems with this question.


Question 391: Which of the following does NOT lead to the occurrence of
Eisenmenger syndrome?

Choices:
1. Atrial septal defect
2. Ventricular septal defect
3. Patent ductus arteriosus
4. Tetralogy of Fallot
Answer: 4 - Tetralogy of Fallot
Explanations:
Tetralogy of Fallot is a right to left shunt and does not lead to Eisenmenger
syndrome.

Go to the next page if you knew the correct answer, or click the link images
below to further research the concepts in this question (if desired).

Research Concepts:
Eisenmenger Syndrome:

Tap flag to report any problems with this question.


Question 392: Independent predictors of late mortality in patients with
acute MI and prior CABG receiving PTCA, include all of the following
EXCEPT:

Choices:
1. Post-PTCA TIMI flow grade <3
2. Advanced age
3. Triple vessel disease
4. Atrial fibrillation
Answer: 4 - Atrial fibrillation
Explanations:
Stone et al analyzed patient characteristics and the efficacy of primary
PTCA in patients with prior CABG and AMI
Independent determinants of late mortality in this group included advanced
age, triple vessel disease, Killip class and post-PTCA TIMI flow grade <3
Reperfusion success of primary PTCA in those with prior CABG was
reduced as compared in those without prior CABG

Go to the next page if you knew the correct answer, or click the link images
below to further research the concepts in this question (if desired).

Research Concepts:
Coronary Artery Bypass Graft:

Acute Myocardial Infarction:

Percutaneous Transluminal Coronary Angioplasty:


Tap flag to report any problems with this question.
Question 393: Which of the following is not characteristic of early
hemorrhagic shock?

Choices:
1. Bradycardia
2. Cold skin
3. Slow capillary refill
4. Confusion
Answer: 1 - Bradycardia
Explanations:
Hemorrhagic shock is result of blood loss.
It results in end organ hypoperfusion.
It usually causes tachycardia initially.
Bradycardia may result in the latter stages if untreated.

Go to the next page if you knew the correct answer, or click the link images
below to further research the concepts in this question (if desired).

Research Concepts:
Shock, Hemorrhagic:

Tap flag to report any problems with this question.


Question 394: The 3rd aortic arch gives rise to which of the following
structures?

Choices:
1. Descending aorta
2. Internal carotid artery
3. Pulmonary artery
4. None of the above
Answer: 2 - Internal carotid artery
Explanations:
The third aortic arch gives rise to the internal carotid arteries.

Go to the next page if you knew the correct answer, or click the link images
below to further research the concepts in this question (if desired).

Research Concepts:
Embryology:

Tap flag to report any problems with this question.


Question 395: In the presence of a femoral artery blood clot and heparin-
induced thrombocytopenia, which drug is most appropriate for treatment?

Choices:
1. Ticlopidine
2. Warfarin
3. Low molecular weight heparin
4. Lepirudin
Answer: 4 - Lepirudin
Explanations:
Lepirudin is a direct acting thrombin inhibitor. It is a synthetic product
based on hirudin, a peptide found in leech saliva.
It is indicated for patients with heparin-induced thrombocytopenia with or
without clot.
Lepirudin can cause an anaphylactic reaction in patients with prior
exposure.
Warfarin should not be started until thrombocytopenia has resolved.

Go to the next page if you knew the correct answer, or click the link images
below to further research the concepts in this question (if desired).

Research Concepts:
Thrombocytopenia, Heparin Induced:

Tap flag to report any problems with this question.


Question 396: A neonate is noted to have a flat face, simian crease, and
prominent epicanthal folds. The patient vomits after each feeding. Plain
radiograph shows a double-bubble sign. Which cardiac abnormality is most
common in patients with these findings?

Choices:
1. Ventricular septal defect
2. Transposition of the great vessels
3. Tetralogy of Fallot
4. Endocardial cushion defect
Answer: 4 - Endocardial cushion defect
Explanations:
The patient has Down syndrome with duodenal atresia
The most common cardiac defect in patients with Down syndrome is
endocardial cushion defect
ASD and VSD are also seen but not as often

Go to the next page if you knew the correct answer, or click the link images
below to further research the concepts in this question (if desired).

Research Concepts:
Down Syndrome:

Duodenal Atresia:

Endocardial Cushion Defect:


Tap flag to report any problems with this question.
Question 397: How many doses of antibiotics are recommended for a clean
surgical case to reduce the risk of surgical infection?

Choices:
1. 0
2. 1
3. 2
4. 3
Answer: 2 - 1
Explanations:
For a clean surgical case like a mastectomy, usually one dose of antibiotic
given preoperatively is sufficient.
The antibiotic should be administered within 1 hour before incision to
provide adequate blood levels.
The dressing should be covered from 24-48 hours with a non adherent
sterile dressing.

Go to the next page if you knew the correct answer, or click the link images
below to further research the concepts in this question (if desired).

Research Concepts:
Antibiotics:

Tap flag to report any problems with this question.


Question 398: What is the mechanism of action of aspirin for pain?
Choices:
1. Inhibition of phospholipase A2
2. Inhibition of phospholipase C
3. Inhibition of cyclooxygenase
4. Inhibition of lipoxygenase
Answer: 3 - Inhibition of cyclooxygenase
Explanations:
NSAIDs inhibit cyclooxygenase but only aspirin is an irreversible inhibitor

Go to the next page if you knew the correct answer, or click the link images
below to further research the concepts in this question (if desired).

Research Concepts:
Aspirin:

Tap flag to report any problems with this question.


Question 399: With which of the following conditions is pulsus alternans
not seen?

Choices:
1. Heart failure
2. Aortic regurgitation
3. Dilated cardiomyopathy
4. Coronary atherosclerosis disease
Answer: 2 - Aortic regurgitation
Explanations:
Pulsus alternans is an alternating weak and strong pulse without a change in
cycle length.
Pulsus alternans is commonly seen in heart failure but can be seen in any
disorder where there is increased resistance to left ventricular ejection.
Other disorders which show pulsus alternans include hypertension, aortic
stenosis, and dilated cardiomyopathy.
Severe coronary artery disease can also be associated with pulsus alternans.

Go to the next page if you knew the correct answer, or click the link images
below to further research the concepts in this question (if desired).

Research Concepts:
Pulsus Alternans:

Tap flag to report any problems with this question.


Question 400: What is the most proximal branch of the RCA?
Choices:
1. PDA
2. Acute marginal
3. Av nodal artery
4. Conal
Answer: 4 - Conal
Explanations:
The conus is the most proximal branch of the RCA.
It is an important collateral to the LAD. It courses along the anterior surface
towards the right ventricle.

Go to the next page if you knew the correct answer, or click the link images
below to further research the concepts in this question (if desired).

Research Concepts:
Coronary Artery Anatomy:

Tap flag to report any problems with this question.


Section 3
Question 401: A 67 year old patient is on a mechanical ventilator and has
an anatomic dead space of 175 ml. The ventilator has a dead space of 325 ml and
a rate of 20/min. In order to provide alveolar ventilation of 6 liters per minute,
what should be the tidal volume?

Choices:
1. 900 ml
2. 800 ml
3. 1200 ml
4. 600 ml
Answer: 2 - 800 ml
Explanations:
Alveolar ventilation plus dead space ventilation equals total ventilation
(175+325=500)
The alveolar ventilation is 6,000 ml per minute which needs to be divided
into 20 respirations, or 300 ml per respiration
300+500=800

Go to the next page if you knew the correct answer, or click the link images
below to further research the concepts in this question (if desired).

Research Concepts:
Ventilator Management:

Tap flag to report any problems with this question.


Question 402: In a 56-year-old obese male who has not been feeling well,
what complaint would lead you to think that he is suffering from angina?

Choices:
1. Dyspnea
2. Bloated feeling with nausea
3. Squeezing chest pain that is relieved with rest
4. Right arm pain radiating to the jaw
Answer: 3 - Squeezing chest pain that is relieved with rest
Explanations:
The chest pain from angina is often described as crushing or squeezing in
nature.
The pain may be associated with diaphoresis, dizziness, or dyspnea.
Some patients may also complain of left arm pain radiating to the jaw. This
feature of anginal pain occurs in at least 5-15% of patients.
Pain from angina can present with a variety of symptoms. It is important to
have a high degree of suspicion in order to make the correct diagnosis.

Go to the next page if you knew the correct answer, or click the link images
below to further research the concepts in this question (if desired).

Research Concepts:
Coronary Artery Disease:

Tap flag to report any problems with this question.


Question 403: An unconscious trauma patient is a Jehovah's Witness and
according to the family should not receive a blood transfusion. The patient might
die without blood. Which of the following is the best choice?

Choices:
1. Ask the next of kin for a firm decision
2. Use blood volume expanders and hope for the best
3. Call the hospital attorney to authorize blood transfusion
4. Transfuse blood
Answer: 1 - Ask the next of kin for a firm decision
Explanations:
The first obligation is to follow the wishes of the patient, in this case, the
spouse should have a clear knowledge of the patient's wishes and as a
result, those wishes should be followed
The next of kin has a right to refuse treatment
Volume expanders may be tried

Go to the next page if you knew the correct answer, or click the link images
below to further research the concepts in this question (if desired).

Research Concepts:
Autonomy:

Tap flag to report any problems with this question.


Question 404: Which is least likely to be found in a patient with
cardiogenic shock?

Choices:
1. Irregular pulse
2. Increased pulse pressure
3. Ankle edema
4. Third heart sound (S3)
Answer: 2 - Increased pulse pressure
Explanations:
The pulse pressure is decreased in cardiogenic shock and most other forms
of shock
It is often increased in septic shock, especially during the early stages

Go to the next page if you knew the correct answer, or click the link images
below to further research the concepts in this question (if desired).

Research Concepts:
Cardiogenic Shock:

Tap flag to report any problems with this question.


Question 405: Which of the following prevent the cusps from being
everted into the atrium during ventricular contraction?

Choices:
1. Papillary muscles and chordae tendineae
2. Pectinate muscles
3. AV and SA nodes
4. Myometrium
Answer: 1 - Papillary muscles and chordae tendineae
Explanations:
The flaps on the bicuspid valve and tricuspid valve are connected to the
chordae tendineae which is connected to the papillary muscle.
They prevent the cusps from being everted into the atrium during
ventricular contraction.

Go to the next page if you knew the correct answer, or click the link images
below to further research the concepts in this question (if desired).

Research Concepts:
Cardiac Physiology:

Heart Anatomy:

Tap flag to report any problems with this question.


Question 406: Which of the following antihypertensive medications can
cause acute exacerbation of asthma?

Choices:
1. Metolazone
2. Labetalol
3. Enalapril
4. Nifedipine
Answer: 2 - Labetalol
Explanations:
Beta-blockers like labetalol are contraindicated in asthmatic patients
Selective beta-blockers like metoprolol can be used with relative safety in
asthmatic patients

Go to the next page if you knew the correct answer, or click the link images
below to further research the concepts in this question (if desired).

Research Concepts:
Antihypertensive Medications:

Asthma:

Tap flag to report any problems with this question.


Question 407: Furosemide (Lasix) 40 mg is to be administered
intravenously to a patient with congestive heart failure and excessive weight
gain. While the patient is receiving furosemide, the caregiver will closely
observe for which of the following?

Choices:
1. Elevated blood pressure
2. Presence of S3 on auscultation of the heart
3. Decrease in serum potassium level
4. Nausea and vomiting
Answer: 3 - Decrease in serum potassium level
Explanations:
The caregiver will observe for a decrease in serum potassium levels when
patients are taking furosemide (Lasix). Normal potassium levels are 3.5 to
5.5 mEq/l. The most significant side effects of furosemide include depletion
of potassium (K) and sodium (Na), and hypotension. Patients should be
specifically monitored for hypokalemia.
Furosemide (Lasix) is a loop diuretic that increases excretion of water and
electrolytes (e.g., potassium and sodium). Diuretics tend to lower blood
pressure, not elevate it.
Therefore, clients receiving loop diuretics should be assessed for
hypotension. An S3 heart sound is a sign of congestive heart failure, which
should improve with the administration of a diuretic like furosemide
(Lasix).
Furosemide may cause nausea and vomiting, but these are mild side effects
compared to a decrease in serum potassium levels, which may be life
threatening. Client Need: Physiological Integrity

Go to the next page if you knew the correct answer, or click the link images
below to further research the concepts in this question (if desired).

Research Concepts:
Loop Diuretics:

Heart Failure, Congestive:


Tap flag to report any problems with this question.
Question 408: Which of the following patients do not need prophylactic
antibiotics before a dental visit?

Choices:
1. Patients with artificial heart valves
2. Patients with a history of infectious endocarditis
3. Patients with a history of uncorrected congenital cyanotic heart disease
4. Patients with cardiac transplant and no cardiac abnormalities
Answer: 4 - Patients with cardiac transplant and no cardiac abnormalities
Explanations:
In 2009, the American Heart association narrowed the recommendations for
SBE prophylaxis
It was found that the risks of antibiotics outweighed the benefits in most
patients
Cardiac transplant patients with valvular lesions should receive antibiotics
Patients with cyanotic congenital heart disease with prosthetic materials for
6 months only

Go to the next page if you knew the correct answer, or click the link images
below to further research the concepts in this question (if desired).

Research Concepts:
SBE Prophylaxis:

Tap flag to report any problems with this question.


Question 409: Where is the point of maximal impulse on the chest of an
infant?

Choices:
1. Between the third and fourth intercostal space
2. Between the fourth and fifth intercostal space
3. Between the third and second intercostal space
4. Between the first and second intercostal space
Answer: 1 - Between the third and fourth intercostal space
Explanations:
The point of maximum impulse in a young infant can be located between
the third and fourth intercostal space

Go to the next page if you knew the correct answer, or click the link images
below to further research the concepts in this question (if desired).

Research Concepts:
Cardiac Exam:

Tap flag to report any problems with this question.


Question 410: Which of the following statements correctly describes the
physiological action of ACE inhibitor?

Choices:
1. It increases preload and afterload
2. It increases preload and reduces afterload
3. It increases afterload and reduces preload
4. It reduces both preload and afterload
Answer: 4 - It reduces both preload and afterload
Explanations:
ACE inhibitors block the conversion of angiotensin I to the more potent
angiotensin II
This reduces the afterload by preventing vasoconstriction
Aldosterone production is also inhibited by ACE inhibitors
This would result in reduced intravascular volume and reduction in preload

Go to the next page if you knew the correct answer, or click the link images
below to further research the concepts in this question (if desired).

Research Concepts:
ACE Inhibitors:

Tap flag to report any problems with this question.


Question 411: Which of the following drugs is most likely to cause
pulmonary fibrosis?

Choices:
1. Procainamide
2. Quinidine
3. Lidocaine
4. Amiodarone
Answer: 4 - Amiodarone
Explanations:
Amiodarone can cause pulmonary fibrosis with long-term use
Routine lung function testing is indicated
It can also cause thyroid and liver problems
Procainamide may cause torsades

Go to the next page if you knew the correct answer, or click the link images
below to further research the concepts in this question (if desired).

Research Concepts:
Amiodarone:

Tap flag to report any problems with this question.


Question 412: Which patient is most likey to have a Staphylococcal
induced liver abscess?

Choices:
1. Cellulitis of the right chest
2. Infectious endocarditis
3. Lymphangitis of the right groin
4. Presence of crohn disease
Answer: 2 - Infectious endocarditis
Explanations:
Staphylococcus induced liver abscess usually occurs by hematogenous
spread.
One disorder where this commonly occurs is infectious endocarditis.

Go to the next page if you knew the correct answer, or click the link images
below to further research the concepts in this question (if desired).

Research Concepts:
Endocarditis, Infectious:

Liver Abscess:

Tap flag to report any problems with this question.


Question 413: First pass metabolism maximally reduces the bioavailability
of which of the following drugs?

Choices:
1. Ampicillin
2. Nitroglycerin
3. Warfarin
4. Metformin
Answer: 2 - Nitroglycerin
Explanations:
The bioavailability of nitroglycerin is markedly reduced after oral
administration
This is because nitroglycerin undergoes extensive first pass metabolism
Sublingual nitroglycerin is administered for rapid effect

Go to the next page if you knew the correct answer, or click the link images
below to further research the concepts in this question (if desired).

Research Concepts:
Nitroglycerin:

Tap flag to report any problems with this question.


Question 414: Which of the following findings on a routine stress test is
strongest evidence for ischemic heart disease?

Choices:
1. Ectopic ventricular beats
2. ST segment depression continuing into recovery
3. Failure to achieve 90% of maximal predicted heart rate
4. Exercise induced, 2 mm up-sloping ST segments during exercise
Answer: 2 - ST segment depression continuing into recovery
Explanations:
ST segment depression lasting for more than 6 minutes in recovery is of
greatest concern
Target heart rate is 85% of maximal predicted heart rate
Ectopy alone should be noted but is not clear evidence of ischemia
Up-sloping ST elevation is not diagnostic

Go to the next page if you knew the correct answer, or click the link images
below to further research the concepts in this question (if desired).

Research Concepts:
Treadmill Stress Testing:

Coronary Artery Disease:

Tap flag to report any problems with this question.


Question 415: When a patient is taking nitroglycerin, he should avoid
exposure of the medication to?

Choices:
1. Television
2. Excess humidity
3. Direct light
4. Snow
Answer: 3 - Direct light
Explanations:
Nitroglycerin is supplied in sublingual, transdermal, and intravenous form
Must keep away from direct light

Go to the next page if you knew the correct answer, or click the link images
below to further research the concepts in this question (if desired).

Research Concepts:
Nitroglycerin:

Tap flag to report any problems with this question.


Question 416: Which of the following echocardiogram findings is not
considered as a major criteria for infective endocarditis?

Choices:
1. Mitral stenosis
2. Acute mitral regurgitation
3. Myocardial abscess
4. Oscillating mass
Answer: 1 - Mitral stenosis
Explanations:
Echocardiogram findings are second major criteria after blood culture
results
Acute mitral regurgitation, myocardial abscess, and oscillating masses are
considered major evidences for infective endocarditis
An echocardiogram finding of stenosed mitral valve is not considered as an
evidence of infective endocarditis

Go to the next page if you knew the correct answer, or click the link images
below to further research the concepts in this question (if desired).

Research Concepts:
Endocarditis, Infectious:

Tap flag to report any problems with this question.


Question 417: Which of the following is NOT a frequent complication of
Gram positive bacterial endocarditis in a 38 year old male?

Choices:
1. Fever
2. Septic arthritis
3. Hematuria
4. Heberden nodes
Answer: 4 - Heberden nodes
Explanations:
Clinical features of endocarditis are highly variable and non specific. Fever
is common and the patient may appear toxic.
Besides heart failure, other features may include hematuria, petechiae,
septic arthritis, pneumonia, malaise, splenomegaly, or conjunctival
hemorrhage.
Physical findings include Osler nodes, Roth spots, splinter hemorrhages, or
Janeway lesions.
Heberden nodes are hard bony swellings seen in the distal IP joints in
osteoarthritis.

Go to the next page if you knew the correct answer, or click the link images
below to further research the concepts in this question (if desired).

Research Concepts:
Endocarditis, Bacterial:

Tap flag to report any problems with this question.


Question 418: Which of following medications is a class IV antiarrhythmic
drug?

Choices:
1. Disopyramide
2. Timolol
3. Sotalol
4. Verapamil
Answer: 4 - Verapamil
Explanations:
Verapamil is a class IV antiarrhythmic drug
Blocks calcium channel

Go to the next page if you knew the correct answer, or click the link images
below to further research the concepts in this question (if desired).

Research Concepts:
Verapamil:

Antiarrhythmic Medication:

Tap flag to report any problems with this question.


Question 419: Which of the following is attached to the surface of the
lung?

Choices:
1. Delto-pectoral fascia
2. Visceral pleura
3. Parietal pleura
4. Serous pericardium
Answer: 2 - Visceral pleura
Explanations:
The parietal pleura lines the chest wall.
The visceral pleura lines the lung and surrounding structures.
The parietal pleura is sensitive to pain but the visceral pleura has no
innervation.
The visceral pleura develops from the visceral mesoderm layer.

Go to the next page if you knew the correct answer, or click the link images
below to further research the concepts in this question (if desired).

Research Concepts:
Thorax Anatomy:

Tap flag to report any problems with this question.


Question 420: What is the most common type of ventricular septal defect?
Choices:
1. Ostium primum defect
2. Perimembranous
3. Coronary sinus defect
4. AV canal type
Answer: 2 - Perimembranous
Explanations:
Ventricular septal defect is the second most common congenital cardiac
malformation and accounts for about one fifth of cardiac anomalies
VSD usually is diagnosed during childhood
In the adults, it is diagnosed less often in adults as large defects are
surgically corrects and because there is spontaneous closure of the smaller
defects
The majority perimembranous VSD close spontaneously by the age of 1
year

Go to the next page if you knew the correct answer, or click the link images
below to further research the concepts in this question (if desired).

Research Concepts:
Ventricular Septal Defect:

Tap flag to report any problems with this question.


Question 421: Which of the following is a colloidal fluid?
Choices:
1. Normal saline
2. Ringers lactate
3. Albumin
4. Cryoprecipitate
Answer: 3 - Albumin
Explanations:
Isotonic saline and ringer's lactate are examples of crystalloid fluids.
Colloidal solutions include starch, albumin, dextran, and mannitol. A
colloid, by definition, cannot pass through a semi-permeable membrane.

Go to the next page if you knew the correct answer, or click the link images
below to further research the concepts in this question (if desired).

Research Concepts:
Colloid Solutions:

Tap flag to report any problems with this question.


Question 422: Which of the following findings on ECG is not indicative of
left ventricular hypertrophy?

Choices:
1. S in V1 + R in V5 or V6 > 35 mm
2. R or S in a limb lead >20 mm
3. S in V1 or V2 >30 mm
4. R in aVR >9 mm
Answer: 4 - R in aVR >9 mm
Explanations:
aVR is a limb lead with negative deflection as its primary vector
In LVH the S wave would be larger without R wave effects

Go to the next page if you knew the correct answer, or click the link images
below to further research the concepts in this question (if desired).

Research Concepts:
Electrocardiogram:

Left Ventricular Hypertrophy:

Tap flag to report any problems with this question.


Question 423: Which of the following symptoms is not common in
patients with acute myocardial infarction?

Choices:
1. Sweating
2. Dyspnea
3. Heartburn
4. Somnolence
Answer: 4 - Somnolence
Explanations:
Sympathetic stimulation is very common in patients with acute MI
Chest pain during acute MI is often mistaken for heartburn
High degree of suspicion is needed to diagnose acute MI
Agitation is also common with the other sympathetic symptoms

Go to the next page if you knew the correct answer, or click the link images
below to further research the concepts in this question (if desired).

Research Concepts:
Acute Myocardial Infarction:

Tap flag to report any problems with this question.


Question 424: Which of the following is not a risk for perioperative deep
venous thrombosis?

Choices:
1. Varicose veins
2. Obesity
3. Infection
4. Male sex
Answer: 4 - Male sex
Explanations:
Other risk factors include malignancy, oral contraceptives, long surgical
duration, and genetic hypercoagulability.

Go to the next page if you knew the correct answer, or click the link images
below to further research the concepts in this question (if desired).

Research Concepts:
Deep Venous Thrombosis, Lower Extremity:

Tap flag to report any problems with this question.


Question 425: Which of the following is NOT true about use of IMA in
coronary artery bypass?

Choices:
1. Too short a length of mammary may prevent usage
2. Subclavian artery stenosis may decrease flow in the IMA
3. Failure to clip proximal branches may lead to steal
4. In situ IMA grafts have same patency as free mammary grafts
Answer: 4 - In situ IMA grafts have same patency as free mammary grafts
Explanations:
Studies to date show that free IMA grafts have a lesser patency than in situ
grafts.
When the IMA is short, grafts using the radial artery can be done.

Go to the next page if you knew the correct answer, or click the link images
below to further research the concepts in this question (if desired).

Research Concepts:
Internal Mammary Artery Bypass:

Tap flag to report any problems with this question.


Question 426: A mural thrombus from the left ventricle may embolize to
all of the following organs, except which one?

Choices:
1. Brain
2. Kidney
3. Lung
4. Spleen
Answer: 3 - Lung
Explanations:
Emboli formed in the left ventricle usually embolize in the arterial
circulation.
The most common site of embolization is the brain, followed by the
extremities.
Mural thrombi from the left ventricle can only go to the lung if there is a
ventricular septal defect present.

Go to the next page if you knew the correct answer, or click the link images
below to further research the concepts in this question (if desired).

Research Concepts:
Mural Thrombi:

Tap flag to report any problems with this question.


Question 427: What is the most common cause of isolated aortic stenosis
in North America?

Choices:
1. Endocarditis
2. Rheumatic fever
3. Calcification of bicuspid valve
4. Degeneration of the valve
Answer: 3 - Calcification of bicuspid valve
Explanations:
The most common cause of aortic stenosis in North America is calcification
of a bicuspid aortic valve.
In those over 70 years of age, it is due to degeneration.

Go to the next page if you knew the correct answer, or click the link images
below to further research the concepts in this question (if desired).

Research Concepts:
Aortic Stenosis:

Tap flag to report any problems with this question.


Question 428: For reconstruction of the chest wall, one can use either
Marlex mesh or Gortex. Which of the following is false?

Choices:
1. The diaphragm is always reconstructed with a Gortex patch
2. Gortex patch becomes lax with time
3. Marlex mesh gets incorporated in the tissues and maintains shape
4. Gortex gets incorporated into tissues and is expensive
Answer: 4 - Gortex gets incorporated into tissues and is expensive
Explanations:
Gortex does not incorporate into tissues.
It is more expensive than Marlex.

Go to the next page if you knew the correct answer, or click the link images
below to further research the concepts in this question (if desired).

Research Concepts:
Chest Wall Reconstruction Surgery:

Tap flag to report any problems with this question.


Question 429: When are giant A waves more likely to be seen?
Choices:
1. Mitral regurgitation
2. Pulmonary stenosis
3. Tricuspid stenosis
4. Hypertrophic cardiomyopathy
Answer: 3 - Tricuspid stenosis
Explanations:
Jugular venous pulses are increased with tricuspid stenosis if the patient is
in sinus rhythm.
This is referred to as giant A waves.
The major cause of tricuspid stenosis is rheumatic fever.

Go to the next page if you knew the correct answer, or click the link images
below to further research the concepts in this question (if desired).

Research Concepts:
Tricuspid Stenosis:

Tap flag to report any problems with this question.


Question 430: Life threatening pulmonary emboli are most commonly
derived from the:

Choices:
1. Renal veins
2. Deep leg veins
3. Inferior mesenteric veins
4. Splenic veins
Answer: 2 - Deep leg veins
Explanations:
The majority of deep vein clots arise from the large pelvic and thigh veins.

Go to the next page if you knew the correct answer, or click the link images
below to further research the concepts in this question (if desired).

Research Concepts:
Pulmonary Embolism:

Deep Venous Thrombosis, Lower Extremity:

Tap flag to report any problems with this question.


Question 431: Which is not a feature of mitral stenosis on a chest
radiograph?

Choices:
1. Left atrial enlargement
2. Depressed left main stem bronchus
3. Double cardiac opacity
4. Prominent left heart border
Answer: 2 - Depressed left main stem bronchus
Explanations:
Chest radiographic signs of mitral stenosis include an enlarged left atrium,
prominent left heart border, and double cardiac opacity (enlarged LA seen
through LV).
The enlarged left atrium usually pushes the left mainstem bronchus up.
Complications include pulmonary edema and pleural effusions.
The imaging study of choice to assess mitral stenosis is cardiac US.

Go to the next page if you knew the correct answer, or click the link images
below to further research the concepts in this question (if desired).

Research Concepts:
Mitral Stenosis:

Tap flag to report any problems with this question.


Question 432: What is not true about body fluids?
Choices:
1. Average urine output is about 1.5-2.0 liters/day
2. Insensible losses amount to 200-300 cc per day
3. An adult requires about 40-60 mmol of KCL/day
4. Protein requirements are increased during burns
Answer: 2 - Insensible losses amount to 200-300 cc per day
Explanations:
On average, an adult makes about 1.5 -2.0 liters of urine per day. This may
be slightly less in the temperate climates.
The insensible losses via the skin and lungs average 800-1200 cc per day
and may be higher in warm, dry weather.
The average requirements for KCL per day are anywhere from 40-60
mmol/day.
An adult requires about 70-90 grams of protein per day. This number is a
lot higher in the presence of hyper catabolic states that occur with burns.

Go to the next page if you knew the correct answer, or click the link images
below to further research the concepts in this question (if desired).

Research Concepts:
Body Fluids:

Tap flag to report any problems with this question.


Question 433: Which of the following signs or symptoms is not found with
pulmonary embolus (PE)?

Choices:
1. Dyspnea
2. Unilateral lower extremity edema
3. Pleuritic chest pain
4. Erythema multiforme
Answer: 4 - Erythema multiforme
Explanations:
Erythema multiforme is not associated with PE
Some common symptoms of PE are dyspnea, pleuritic chest pain, and
cough
Unilateral lower extremity edema is suggestive of DVT, which is
commonly associated with PE
Most of the symptoms of PE are nonspecific and health care professionals
need to have a high index of suspicion

Go to the next page if you knew the correct answer, or click the link images
below to further research the concepts in this question (if desired).

Research Concepts:
Pulmonary Embolism:

Tap flag to report any problems with this question.


Question 434: Which is the most effective way to prevent atelectasis after
surgery?

Choices:
1. Supplemental oxygen
2. Antibiotics
3. Bed rest
4. Incentive spirometry
Answer: 4 - Incentive spirometry
Explanations:
Post-operative patients should have an incentive spirometer at their bedside
so it is available for frequent use (eg, every 1 to 2 hours).
Incentive spirometry is one of the most effective noninvasive methods to
prevent atelectasis.

Go to the next page if you knew the correct answer, or click the link images
below to further research the concepts in this question (if desired).

Research Concepts:
Atelectasis:

Tap flag to report any problems with this question.


Question 435: Which is a class 1 antiarrhythmic agent?
Choices:
1. Digoxin
2. Hydralazine
3. Propranolol
4. Quinidine
Answer: 4 - Quinidine
Explanations:
Classes 1 anti arrhythmics comprise the sodium channel blockers. These
drugs alter sodium flux into the cell.
The class 1 drugs are also know as membrane stabilizing agents and are
divided in three subgroups.
Quinidine, procainamide lidocaine, phenytoin, flecainide, propafenone, and
moricizine are included in the class one category.
Class 1 drugs can be used to treat ventricular arrhythmia and Wolff
Parkinson White syndrome

Go to the next page if you knew the correct answer, or click the link images
below to further research the concepts in this question (if desired).

Research Concepts:
Quinidine:

Tap flag to report any problems with this question.


Question 436: What is the effect of beta-1-blockers on respiratory system?
Choices:
1. Bronchodilation
2. Bronchoconstriction
3. No effect
4. Decreased secretion
Answer: 2 - Bronchoconstriction
Explanations:
Beta-1-receptors are found in smooth muscle of the bronchioles
Blocking these receptors leads to constriction of these smooth muscles
Stimulating these receptors by beta-1-agonists leads to bronchodilation
Bronchial secretion is mainly influenced by cholinergic muscarinic
receptors

Go to the next page if you knew the correct answer, or click the link images
below to further research the concepts in this question (if desired).

Research Concepts:
Beta-Blockers:

Bronchoconstriction:

Tap flag to report any problems with this question.


Question 437: Antibiotic prophylaxis for dental procedures is required for
all of the following individuals except?

Choices:
1. A 39-year-old with porcine valve implanted less than 2 years ago
2. A 29-year-old HIV male
3. A 71-year-old with hip prosthesis
4. A 65-year-old with Parkinson disease
Answer: 4 - A 65-year-old with Parkinson disease
Explanations:
Prophylaxis for dental procedures is warranted in certain patients who are at
risk of infections.
The presence of prostheses, immunosuppression, and valvular heart disease
needs antibiotic prophylaxis before any dental procedure.
Antibiotic prophylaxis is not recommended for patients with Parkinson
disease.

Go to the next page if you knew the correct answer, or click the link images
below to further research the concepts in this question (if desired).

Research Concepts:
SBE Prophylaxis:

Tap flag to report any problems with this question.


Question 438: Which is true regarding pulmonary stenosis?
Choices:
1. Diagnosis is made by chest radiograph
2. Even mild cases need surgical treatment
3. Echocardiogram determines the diagnosis
4. Rheumatic disease is the most common etiology
Answer: 3 - Echocardiogram determines the diagnosis
Explanations:
Diagnosis is made by echocardiogram
Mild cases do not require treatment
Surgery or valvuloplasty is curative for isolated pulmonary stenosis
Pulmonary stenosis is usually congenita

Go to the next page if you knew the correct answer, or click the link images
below to further research the concepts in this question (if desired).

Research Concepts:
Pulmonic Stenosis:

Tap flag to report any problems with this question.


Question 439: Which is not a potential complication of mechanical
ventilation?

Choices:
1. Barotrauma
2. Decreased venous return
3. Increased renal flow
4. Reduced cardiac output
Answer: 3 - Increased renal flow
Explanations:
Ventilation reduces venous return and cardiac output
Also reduces renal blood flow

Go to the next page if you knew the correct answer, or click the link images
below to further research the concepts in this question (if desired).

Research Concepts:
Ventilator Complications:

Tap flag to report any problems with this question.


Question 440: What is the most effective as a thrombolytic agent for
dissolution of blood clots?

Choices:
1. Heparin
2. Tissue plasminogen activator
3. Streptokinase
4. Warfarin
Answer: 2 - Tissue plasminogen activator
Explanations:
TPA is more effective than streptokinase as a thrombolytic agent.
Heparin and warfarin are blood thinners.
Streptokinase is no longer available in the United States.

Go to the next page if you knew the correct answer, or click the link images
below to further research the concepts in this question (if desired).

Research Concepts:
Thrombolytic Therapy:

Tap flag to report any problems with this question.


Question 441: Select the best treatment for pericarditis secondary to acute
myocardial infarction.

Choices:
1. Ibuprofen
2. Aspirin
3. Colchicine
4. Corticosteroids
Answer: 2 - Aspirin
Explanations:
Asprin is usually used for Dressler syndrome.
Other nonsteroidal anti-inflammatory drugs and corticosteroids are
deleterious to ventricular remodeling during acute MI and can result in
ventricular rupture.
Rarely methotrexate or intravenous immunoglobulin are used.

Go to the next page if you knew the correct answer, or click the link images
below to further research the concepts in this question (if desired).

Research Concepts:
Pericarditis, Acute:

Acute Myocardial Infarction:

Tap flag to report any problems with this question.


Question 442: What percent of the population has latex allergy?
Choices:
1. 1-5
2. 7-12
3. 20-30
4. > 50
Answer: 1 - 1-5
Explanations:
Latex allergy is present in 1-5 percent of the population
The incidence is high in atopic individuals

Go to the next page if you knew the correct answer, or click the link images
below to further research the concepts in this question (if desired).

Research Concepts:
Latex Allergy:

Tap flag to report any problems with this question.


Question 443: After starting anticoagulation therapy, when will 50% of
pulmonary emboli disappear?

Choices:
1. 7 days
2. 3 days
3. 14 days
4. 3 months
Answer: 3 - 14 days
Explanations:
Pulmonary emboli do resolve with anticoagulation therapy.
In most cases, 36% of lung defects resolve in 5 days and 50% resolve in 3
months.

Go to the next page if you knew the correct answer, or click the link images
below to further research the concepts in this question (if desired).

Research Concepts:
Pulmonary Embolism:

Tap flag to report any problems with this question.


Question 444: Which of the following lacks activity against Pseudomonas?
Choices:
1. Cefepime
2. Carbenicillin
3. Ticarcillin
4. Ceftriaxone
Answer: 4 - Ceftriaxone
Explanations:
Resistance to Pseudomonas is at an all time high.
Pseudomonas is extremely difficult to treat and resistance to almost every
antibiotic is common.
Among the cephalosporins, only the 4th generation drugs work.
Ceftriaxone and cefotaxime have no activity against Pseudomonas.

Go to the next page if you knew the correct answer, or click the link images
below to further research the concepts in this question (if desired).

Research Concepts:
Pseudomonas:

Tap flag to report any problems with this question.


Question 445: An asymptomatic 5-year-old has a III/VI holosystolic
murmur in the pulmonic area. There is fixed splitting of the second heart sound.
What is the most appropriate management?

Choices:
1. Endocarditis prophylaxis for dental work
2. Observation
3. ECG and echocardiogram
4. Cardiology referral
Answer: 3 - ECG and echocardiogram
Explanations:
The murmur is consistent with atrial septal defect.
There is increased flow across the pulmonic valve secondary to left-to-right
shunting.
The ECG may show right ventricular hypertrophy and the echocardiogram
may show cardiomegaly.
Endocarditis prophylaxis is not needed and if the ASD does not close,
elective repair may be needed.

Go to the next page if you knew the correct answer, or click the link images
below to further research the concepts in this question (if desired).

Research Concepts:
Atrial Septal Defect:

Tap flag to report any problems with this question.


Question 446: What clinical finding is characteristic of pulmonary
hypertension?

Choices:
1. Loud S1
2. Soft A2
3. Loud P2
4. S2 splits
Answer: 3 - Loud P2
Explanations:
Pulmonary hypertension is characterized by a loud P2
There can be the murmur of pulmonary regurgitation.
Some patients have a right ventricular heave or signs of right heart failure.
Late findings are hepatomegaly with palpable pulsations of the liver and
abdominal-jugular reflex.

Go to the next page if you knew the correct answer, or click the link images
below to further research the concepts in this question (if desired).

Research Concepts:
Pulmonary Hypertension:

Tap flag to report any problems with this question.


Question 447: Which of the following has been used to treat migraine,
arrhythmias, tremor, hyperthyroidism, and angina?

Choices:
1. Nifedipine
2. Clonidine
3. Propranolol
4. Methyldopa
Answer: 3 - Propranolol
Explanations:
Propranolol is a non-selective beta-blocker
Propranolol also has some local anesthetic activity but is not a partial
agonist
Propranolol is often used to treat hypertension, angina, arrhythmias,
migraine, hyperthyroidism and essential tremor
Propranolol can worsen asthma, induce AV block and worsen heart failure

Go to the next page if you knew the correct answer, or click the link images
below to further research the concepts in this question (if desired).

Research Concepts:
Propranolol:

Tap flag to report any problems with this question.


Question 448: Cardiac myxomas is most commonly found in:
Choices:
1. Right atrium
2. Right ventricle
3. Left atrium
4. Left ventricle
Answer: 3 - Left atrium
Explanations:
75% percent of atrial myxomas are founding left atrium
Most are pedunculated
18% are in the right atrium, 3% in the RV, and and 3% in the LV

Go to the next page if you knew the correct answer, or click the link images
below to further research the concepts in this question (if desired).

Research Concepts:
Atrial Myxoma:

Tap flag to report any problems with this question.


Question 449: Which of the following factors does NOT affect rate of
absorption of a drug?

Choices:
1. Particle size
2. Degree of ionization
3. Lipid solubility
4. Shape
Answer: 4 - Shape
Explanations:
Rate of absorption of a drug is affected by particle size
Non-ionized particles are better absorbed than more ionized particles
Lipid solubility greatly affects rate of absorption with lipid soluble ones
passing fast than others

Go to the next page if you knew the correct answer, or click the link images
below to further research the concepts in this question (if desired).

Research Concepts:
Drug Absorption:

Tap flag to report any problems with this question.


Question 450: Which of the following does not make up the femoral
triangle?

Choices:
1. Superior border made by inguinal ligament
2. Lateral border by sartorius
3. Medial border by adductor longus
4. Posterior border by psoas muscle
Answer: 4 - Posterior border by psoas muscle
Explanations:
The femoral vein lies in the femoral triangle
The superior border is made by the inguinal ligament
The lateral border is made by the sartorius
The medial border is made by the adductor longus

Go to the next page if you knew the correct answer, or click the link images
below to further research the concepts in this question (if desired).

Research Concepts:
Femoral Triangle:

Tap flag to report any problems with this question.


Question 451: Which of the following can be a side effect of beta
blockers?

Choices:
1. Hyperglycemia
2. Hypoglycemia
3. Ketoacidosis
4. Hyperosmolar coma
Answer: 2 - Hypoglycemia
Explanations:
All beta-blockers can cause hypoglycemia.
Beta blockers inhibit the sympathetic response to hypoglycemia and can
mask the onset of insulin coma.

Go to the next page if you knew the correct answer, or click the link images
below to further research the concepts in this question (if desired).

Research Concepts:
Hypoglycemia:

Beta-Blockers:

Tap flag to report any problems with this question.


Question 452: Which of the following anomalies are not associated with
VACTERL association?

Choices:
1. Vertebral defects
2. Aniridia
3. Trachea-esophageal fistula
4. Radial dysplasia
Answer: 2 - Aniridia
Explanations:
VACTERL or VATER association of disorders encompasses a wide range of
anomalies
Vertebral defects, anal atresia, cardiovascular defects, tracheoesophageal
fistula, renal defects, and limb are common with this condition
Aniridia is not part of VATER association
Radial dysplasia is another typical feature of this disease

Go to the next page if you knew the correct answer, or click the link images
below to further research the concepts in this question (if desired).

Research Concepts:
VACTERL Association:

Tap flag to report any problems with this question.


Question 453: What is the mechanism of action of ticlopidine (Ticlid)?
Choices:
1. It inhibits thromboxane
2. It inhibits the adenosine diphosphate receptor
3. It is an antagonist of the G11b/111a receptor
4. It is a sodium/potassium ATPase inhibitor
Answer: 2 - It inhibits the adenosine diphosphate receptor
Explanations:
Ticlid is an anti-platelet drug. It is an adenosine diphosphate receptor
inhibitor. It is used in patients that do not tolerate aspirin.
Ticlid inhibits platelet aggregation by altering the function of the platelet
membrane.
Ticlid prolongs bleeding time.
Ticlid is contraindicated in bleeding disorders, severe liver disease, and
hypersensitivity disorders.

Go to the next page if you knew the correct answer, or click the link images
below to further research the concepts in this question (if desired).

Research Concepts:
Antiplatelet Medications:

Tap flag to report any problems with this question.


Question 454: Harsh holosystolic murmur is heard in:
Choices:
1. Atrial septal defect
2. Ventricular septal defect
3. Pulmonary stenosis
4. Patent ductus arteriosus
Answer: 2 - Ventricular septal defect
Explanations:
Harsh systolic murmur is heard in ventricular septal defect
Loud S1 and fixed splitting of the second heart sound is a characteristic of
atrial septal defect
PDA is characterized by to and fro murmur

Go to the next page if you knew the correct answer, or click the link images
below to further research the concepts in this question (if desired).

Research Concepts:
Ventricular Septal Defect:

Tap flag to report any problems with this question.


Question 455: Which of the following is the most likely explanation for
hyperacute graft rejection?

Choices:
1. Antibody-mediated immunity
2. Cell-mediated immunity
3. Cell-mediated immunity to minor HLA antigens
4. Preformed ABO blood group antibodies
Answer: 4 - Preformed ABO blood group antibodies
Explanations:
Hyperacute rejection occurs within minutes to a few hours of the time of
transplantation
This is often caused by preformed antibodies
Preformed antibodies can also be due to pre sensitization to a previous graft
or blood transfusion

Go to the next page if you knew the correct answer, or click the link images
below to further research the concepts in this question (if desired).

Research Concepts:
Graft Rejection:

Tap flag to report any problems with this question.


Question 456: The apex of the heart receives blood supply from which of
the coronary arteries?

Choices:
1. Anterior interventricular artery
2. Circumflex artery
3. Posterior interventricular artery
4. Right coronary artery
Answer: 1 - Anterior interventricular artery
Explanations:
The anterior interventricular artery, also known as the left anterior
descending artery, is one of the divisions of the left coronary artery.
It runs down the surface of the interventricular septum towards the apex.
It sends branches to the interventricular septum to supply most of its
myocardium and into the myocardium of the adjacent right and left
ventricle.
It is one of the most involved in coronary occlusions and is often the one
that is bypassed in cardiac bypass surgery.

Go to the next page if you knew the correct answer, or click the link images
below to further research the concepts in this question (if desired).

Research Concepts:
Left Anterior Descending Artery (LAD):

Tap flag to report any problems with this question.


Question 457: What agent is most beneficial to lower the risk of
myocardial infarction?

Choices:
1. Beta blocker
2. Alpha blocker
3. ACE inhibitor
4. Calcium channel blocker
Answer: 1 - Beta blocker
Explanations:
Beta blocker has shown to lower the risk of myocardial infarction

Go to the next page if you knew the correct answer, or click the link images
below to further research the concepts in this question (if desired).

Research Concepts:
Primary And Secondary Prevention Of Coronary Artery
Disease:

Tap flag to report any problems with this question.


Question 458: What is the first is a patient with suspected aortic arch
aneurysm?

Choices:
1. Echocardiogram
2. CT scan
3. Hydralazine
4. Observation
Answer: 1 - Echocardiogram
Explanations:
Echocardiogram is indicated to diagnose aortic arch aneurysm

Go to the next page if you knew the correct answer, or click the link images
below to further research the concepts in this question (if desired).

Research Concepts:
Aortic Arch Aneurysm:

Tap flag to report any problems with this question.


Question 459: Elevation of which component would identify a specimen as
plasma rather than serum?

Choices:
1. Fibrinogen
2. Serotonin
3. Albumin
4. Erythrocytes
Answer: 1 - Fibrinogen
Explanations:
The extraction of fibrinogen and coagulation factors II, V. and VIII converts
plasma to serum.
Whole blood, allowed to clot and followed by the removal of the clot,
establishes the conversion of plasma to serum.
Albumen is present in both plasma and serum.
Erythrocytes are absent in both blood products.

Go to the next page if you knew the correct answer, or click the link images
below to further research the concepts in this question (if desired).

Research Concepts:
Plasma:

Tap flag to report any problems with this question.


Question 460: The pathologist reports the presence of McCallum plaques.
This means that the patient may have which disease?

Choices:
1. Lupus
2. Hemosiderosis
3. Rheumatic fever
4. Pneumonia
Answer: 3 - Rheumatic fever
Explanations:
Rheumatic fever is an inflammatory disorder that develops several weeks
after a Group A streptococcal infection. The infection can affect the brain,
heart, skin, and joints.
The pathological hallmarks of rheumatic carditis are McCallum patch and
the Aschoff nodule.
McCallum patches are defined as warty protuberances arising from the
bacterial deposits, which results in irregular thickening of the valves.
The major Jones criteria for diagnosis include migratory polyarthritis,
carditis, a new heart murmur, subcutaneous nodules, erythema marginatum,
and Sydenham chorea.

Go to the next page if you knew the correct answer, or click the link images
below to further research the concepts in this question (if desired).

Research Concepts:
Rheumatic Fever, Pathology:

Tap flag to report any problems with this question.


Question 461: A 40 year old female is diagnosed with New York Heart
Association class II congestive heart failure secondary to mitral stenosis. Which
of the following is the most likely complication?

Choices:
1. Bundle branch block
2. Atrial fibrillation
3. Myocardial infarction
4. Multifocal atrial tachycardia
Answer: 2 - Atrial fibrillation
Explanations:
The most probable etiology of this scenario is rheumatic heart disease
Atrial fibrillation is the most probable complication
Bundle branch block or myocardial infarction is unlikely in this patient

Go to the next page if you knew the correct answer, or click the link images
below to further research the concepts in this question (if desired).

Research Concepts:
Heart Failure, Congestive:

Mitral Stenosis:

Tap flag to report any problems with this question.


Question 462: Which of the following is a voice transmission test to elicit
lung consolidation on physical examination?

Choices:
1. Tactile fremitus
2. Bronchophony
3. Egophony
4. All of the above
Answer: 4 - All of the above
Explanations:
All of the voice transmission tests have been largely supplanted by the CXR
In egophony, there are "E to A" changes upon auscultation
In bronchophony, "99" is heard upon auscultation instead of muffled,
indistinct sounds
Voice transmission tests are employed when there is a high suspicion for
lung consolidation in disease such as pneumonia

Go to the next page if you knew the correct answer, or click the link images
below to further research the concepts in this question (if desired).

Research Concepts:
Lung Exam:

Tap flag to report any problems with this question.


Question 463: In a patient with mild chest discomfort, there is an audible
diastolic murmur heard over the femoral artery when it is compressed. Which of
the following may be present?

Choices:
1. Atherosclerosis of the femoral artery
2. Coarctation of aorta
3. Aortic regurgitation
4. Aortic aneurysm
Answer: 3 - Aortic regurgitation
Explanations:
Duroziez sign is a feature of aortic insufficiency.
When the femoral artery is compressed, you will hear an audible diastolic
murmur.
Duroziez sign is just one feature of aortic aneurysm.

Go to the next page if you knew the correct answer, or click the link images
below to further research the concepts in this question (if desired).

Research Concepts:
Aortic Insufficiency:

Duroziez Sign:

Tap flag to report any problems with this question.


Question 464: Which is true with respect to atrial septal defects?
Choices:
1. ASDs account for 25% of congenital heart disease.
2. All of them require surgery soon after birth
3. May present with a fixed split of the second heart sound
4. A continuous murmur is a common feature in presence of large defects
Answer: 3 - May present with a fixed split of the second heart sound
Explanations:
Atrial septal defects allow blood flow between the two atria.
Left to right shunt usually occurs during late ventricular systole into early
diastole.
Most patients do not require surgery. Patients may have fixed splitting of
the second heart sound.
ASDs account for 10% of congenital heart disease.

Go to the next page if you knew the correct answer, or click the link images
below to further research the concepts in this question (if desired).

Research Concepts:
Atrial Septal Defect:

Tap flag to report any problems with this question.


Question 465: Which is the most common cause of hypertension?
Choices:
1. Essential, or unknown or primary hypertension
2. Secondary due to hyperaldosteronism
3. Secondary, due to hyperthyroidism
4. Anatomical, due to renal artery stenosis
Answer: 1 - Essential, or unknown or primary hypertension
Explanations:
Essential or primary hypertension is due to an unknown cause and is most
common

Go to the next page if you knew the correct answer, or click the link images
below to further research the concepts in this question (if desired).

Research Concepts:
Essential Hypertension:

Tap flag to report any problems with this question.


Question 466: A hyperacute graft rejection is mediated by which of the
following?

Choices:
1. Preformed antibody
2. T cell
3. Neutrophil
4. Macrophage
Answer: 1 - Preformed antibody
Explanations:
Hyperacute graft rejection due to preformed antibody.
Characterized by fever, rash, and diarrhea.

Go to the next page if you knew the correct answer, or click the link images
below to further research the concepts in this question (if desired).

Research Concepts:
Hyperacute Transplant Rejection:

Tap flag to report any problems with this question.


Question 467: Which is a correct definition of oliguria in adults?
Choices:
1. 15 mL per hour
2. 20 mL per hour
3. 30 mL per hour
4. 40 mL per hour
Answer: 1 - 15 mL per hour
Explanations:
Oliguria is defined as less than to fifteen mL of urine per hour.
This would be less than 400 mL per day.

Go to the next page if you knew the correct answer, or click the link images
below to further research the concepts in this question (if desired).

Research Concepts:
Oliguria:

Tap flag to report any problems with this question.


Question 468: A patient is experiencing an evolving acute myocardial
infarction and is to receive alteplase (t-PA). What is the primary action of this
medication?

Choices:
1. Increase heart rate
2. Elevate blood pressure
3. Lyse blood clots
4. Promote diuresis
Answer: 3 - Lyse blood clots
Explanations:
Alteplase (t-PA) lyses blood clots. Thrombolytic medications (agents that
dissolve thrombi), such as alteplase (t-PA), cause fibrinolysis.
Thrombolytics activate plasminogen by converting it to plasmin. The
plasmin in turn lyses fibrin clots.
Any change in heart rate, blood pressure, and diuresis would be a result of
changes in cardiac output as a consequence of the success of fibrinolysis.
Patient Need: Physiological Integrity

Go to the next page if you knew the correct answer, or click the link images
below to further research the concepts in this question (if desired).

Research Concepts:
Thrombolytic Therapy:

Myocardial Infarction, Acute:

Tap flag to report any problems with this question.


Question 469: Which of the following lab results is a contraindication for
general anesthesia?

Choices:
1. Serum calcium level of 9
2. Serum potassium level of 6
3. Hematocrit of 35%
4. Absolute neutrophil count of 750
Answer: 2 - Serum potassium level of 6
Explanations:
Hyperkalemia can cause arrhythmias when under general anesthesia.
Preoperative screening and electrolyte determination is essential.
Serum calcium level of 9 is normal.
Low hematocrit and neutrophil count is not a contraindication.

Go to the next page if you knew the correct answer, or click the link images
below to further research the concepts in this question (if desired).

Research Concepts:
Preoperative Testing:

Hyperkalemia:

Tap flag to report any problems with this question.


Question 470: What should be done prior to performing phlebotomy in
patients with Eisenmenger syndrome?

Choices:
1. Place a central venous line
2. Attach a pulse oximeter
3. Infuse normal saline
4. Have fresh frozen plasma
Answer: 3 - Infuse normal saline
Explanations:
Erythrocytosis in patients with Eisenmenger can present with many
symptoms including fatigue, paresthesia, headache, and vision problems.
Phlebotomy is recommended but can lead to iron deficiency anemia.
Hydration is recommended in all individuals with a hematocrit above 65
with normal saline being the fluid of choice.

Go to the next page if you knew the correct answer, or click the link images
below to further research the concepts in this question (if desired).

Research Concepts:
Eisenmenger Syndrome:

Tap flag to report any problems with this question.


Question 471: Which is not a sign of cardiac cirrhosis?
Choices:
1. Peripheral edema
2. Hypertension
3. Elevated JVD
4. Liver cirrhosis
Answer: 2 - Hypertension
Explanations:
Cardiac cirrhosis is presents with signs of right sided heart failure
Blood pressure is mostly determined by the left side

Go to the next page if you knew the correct answer, or click the link images
below to further research the concepts in this question (if desired).

Research Concepts:
Cardiac Cirrhosis:

Tap flag to report any problems with this question.


Question 472: Dressler syndrome describes post myocardial infarction
associated:

Choices:
1. Hypertension
2. Hypertrophy
3. Pericarditis
4. Effusion
Answer: 3 - Pericarditis
Explanations:
Dressler syndrome describes pericarditis after an myocardial infarction
It occurs 2 to 3 weeks after myocardial infarction
Presenting symptoms are chest pain and fever

Go to the next page if you knew the correct answer, or click the link images
below to further research the concepts in this question (if desired).

Research Concepts:
Dressler Syndrome:

Tap flag to report any problems with this question.


Question 473: What is false about heart-lung transplantation?
Choices:
1. The lung is usually protected from rejection compared to the heart
2. Eisenmenger's syndrome from a VSD can be treated by repairing the heart
defect and a single lung transplant
3. Rejection may be difficult to exclude from infection
4. A close size match between donor and recipients is required
Answer: 1 - The lung is usually protected from rejection compared to the
heart

Explanations:
The heart (not lung) is protected from rejection in this setting
Routine endomyocardial biopsy is not done
FEV1 is routinely followed in these patients and any suspicion of lung
rejection should be followed by a bronchoscopy possible lavage/biopsy

Go to the next page if you knew the correct answer, or click the link images
below to further research the concepts in this question (if desired).

Research Concepts:
Heart-Lung Transplantation:

Tap flag to report any problems with this question.


Question 474: A PCA machine is most commonly used to administer
which of the following types of medications?

Choices:
1. Sedatives
2. Analgesics
3. Antihypertensives
4. Any medication
Answer: 2 - Analgesics
Explanations:
PCA is the abbreviation for patient-controlled analgesia.
With a PCA machine, the patient can control when they receive doses of
analgesic.

Go to the next page if you knew the correct answer, or click the link images
below to further research the concepts in this question (if desired).

Research Concepts:
Patient Controlled Anesthesia (PCA):

Tap flag to report any problems with this question.


Question 475: Which is not a use of preoperative pulmonary function
testing?

Choices:
1. Reject borderline surgical candidates
2. Optimize patients with COPD
3. Plan postoperative medications
4. Plan intraoperative care
Answer: 1 - Reject borderline surgical candidates
Explanations:
PFTs should be used to devise a cohesive plan for pulmonary care.
Unless considering a lung resection, they should not be used to discourage a
patient from an operative intervention.

Go to the next page if you knew the correct answer, or click the link images
below to further research the concepts in this question (if desired).

Research Concepts:
Pulmonary Function Tests:

Preoperative Evaluation And Management:

Tap flag to report any problems with this question.


Question 476: What is the medication of choice to provide the best
outcomes with hyperlipidemia?

Choices:
1. HMG-CoA reductase inhibitors
2. Omega-3 Acids
3. Nicotinic acid
4. Fibrates
Answer: 1 - HMG-CoA reductase inhibitors
Explanations:
Nicotinic acid (Niacin) stimulates lipid metabolism, increases HDL while
lowering triglyceride levels but has not been shown to reduce myocardial
infarctions
Statin medications also lower triglycerides and LDL while raising HDL
levels and have been proven in both primary and secondary prevention

Go to the next page if you knew the correct answer, or click the link images
below to further research the concepts in this question (if desired).

Research Concepts:
Hyperlipidemia:

Tap flag to report any problems with this question.


Question 477: All of the following behave as emboli. Which is the most
common embolus?

Choices:
1. Air
2. Nitrogen bubbles
3. Thrombus
4. Amniotic fluid
Answer: 3 - Thrombus
Explanations:
Thrombi are the most common emboli. Thrombi can form in veins or in the
heart.
Atrial fibrillation and deep vein thrombi are common causes of emboli.

Go to the next page if you knew the correct answer, or click the link images
below to further research the concepts in this question (if desired).

Research Concepts:
Thromboembolism:

Tap flag to report any problems with this question.


Question 478: An acyanotic newborn infant is noted to have a holosystolic
murmur. Which of the following congenital abnormalities is most likely present?

Choices:
1. Tetralogy of Fallot
2. Complete transposition of the great vessels
3. Ventricular septal defect (VSD)
4. PDA
Answer: 3 - Ventricular septal defect (VSD)
Explanations:
VSD presents with a holosystolic murmur, but without cyanosis.
The most common heart defect in childhood is VSD. The majority of cases
are asymptomatic soon after birth.
The majority of small muscular VSDs close by 12 months. In asymptomatic
patients, observation is recommended.
A VSD occuring after an acute MI is often lethal. Surgery is the only life
saving treatment.

Go to the next page if you knew the correct answer, or click the link images
below to further research the concepts in this question (if desired).

Research Concepts:
Ventricular Septal Defect:

Tap flag to report any problems with this question.


Question 479: In pediatric bradycardia, what should be done if no response
to epinephrine?

Choices:
1. Repeat epinephrine for total of 3 rounds
2. Administration of atropine
3. Administration of dobutamine
4. All of the above
Answer: 2 - Administration of atropine
Explanations:
In pediatric bradycardia, atropine should be administered if no response to
epinephrine
Atropine may be repeated only once in pediatric bradycardia
Epinephrine (1:10,000) may be administered every 3 to 5 minutes

Go to the next page if you knew the correct answer, or click the link images
below to further research the concepts in this question (if desired).

Research Concepts:
Bradycardia:

Tap flag to report any problems with this question.


Question 480: A patient with an Oliver sign will have which of the
following?

Choices:
1. Hypertension
2. Pulmonary fibrosis
3. Aortic aneurysm
4. Peripheral vascular disease
Answer: 3 - Aortic aneurysm
Explanations:
The Oliver sign is a downward tug of the trachea during systole.
It is a positive sign for an aortic arch aneurysm.
Echocardiogram is indicated.

Go to the next page if you knew the correct answer, or click the link images
below to further research the concepts in this question (if desired).

Research Concepts:
Aortic Aneurysm:

Oliver Sign:

Tap flag to report any problems with this question.


Question 481: When cannulating the femoral artery for hemodynamic
monitoring, one should insert the cannula:

Choices:
1. Right at the point of maximal impulse
2. 1 cm above the inguinal ligament
3. 1 cm below the inguinal ligament
4. Lower in the mid thigh
Answer: 3 - 1 cm below the inguinal ligament
Explanations:
It is important to insert the cannula 1-2 cm below the inguinal ligament.
The reason for this is that if there is bleeding the artery can be compressed
Above the inguinal ligament, the femoral artery cannot be compressed and
bleeding can occur in the retroperitoneal space

Go to the next page if you knew the correct answer, or click the link images
below to further research the concepts in this question (if desired).

Research Concepts:
Arterial Cannulation:

Tap flag to report any problems with this question.


Question 482: Aspirin does not exhibit which of the following effects?
Choices:
1. Antipyretic
2. Antihypertensive
3. Antiplatelet
4. Inhibition of cyclooxygenase
Answer: 2 - Antihypertensive
Explanations:
Aspirin inhibits the enzyme cyclooxygenase (COX), which leads to
antipyretic and antiplatelet effects.

Go to the next page if you knew the correct answer, or click the link images
below to further research the concepts in this question (if desired).

Research Concepts:
Aspirin:

Tap flag to report any problems with this question.


Question 483: Which of the following is not a normal function of
platelets?

Choices:
1. Adherence
2. Degranulation
3. Apoptosis
4. Aggregation
Answer: 3 - Apoptosis
Explanations:
The normal functions of platelets are to adhere, aggregate, and degranulate
They can be activated directly through glycoprotein VI of collagen or
indirectly by von Willebrand factor binding to glycoprotein 1b
Platelet granules have alpha granules containing multiple vasoactive
factors, delta granules containing ATP, calcium, and serotonin, and lambda
granules containing hydrolytic enzymes

Go to the next page if you knew the correct answer, or click the link images
below to further research the concepts in this question (if desired).

Research Concepts:
Platelets:

Tap flag to report any problems with this question.


Question 484: In patients with aortic stenosis who develop heart failure,
what percent will be dead by 2 years?

Choices:
1. 5%
2. 10%
3. 25%
4. 50%
Answer: 4 - 50%
Explanations:
Once patients with AS has develop congestive heart failure, 50% will die
within 2 years

Go to the next page if you knew the correct answer, or click the link images
below to further research the concepts in this question (if desired).

Research Concepts:
Aortic Stenosis:

Aortic Stenosis:

Tap flag to report any problems with this question.


Question 485: After starting a patient on a heparin drip for a pulmonary
embolus, when should the first PTT be monitored?

Choices:
1. 4-6 hrs
2. 12 hrs
3. 24 hrs
4. 1 hr
Answer: 1 - 4-6 hrs
Explanations:
After starting heparin, the PTT is monitored every 4-6 hrs for the first 24
hours.
When the PTT is therapeutic between 55-70 seconds, the PTT can me
monitored on a daily basis.

Go to the next page if you knew the correct answer, or click the link images
below to further research the concepts in this question (if desired).

Research Concepts:
Anticoagulation:

Tap flag to report any problems with this question.


Question 486: Your patient is to begin a regimen of spironolactone
(Aldactone), a potassium-sparing diuretic. Which of the following effects do you
anticipate?

Choices:
1. A sudden drop in serum potassium levels
2. Sodium and potassium retention
3. Potential lethal cardiac dysrhythmias due to hypokalemia
4. Sodium and water excretion and potassium retention
Answer: 4 - Sodium and water excretion and potassium retention
Explanations:
You will anticipate sodium and water excretion and potassium retention in
patients taking spironolactone (Aldactone). Spironolactone (Aldactone)
does not cause potassium excretion, as do loop diuretics.
It is referred to as a potassium-sparing diuretic. Hypokalemia or a drop in
serum potassium levels are not expected with potassium-sparing diuretics
such as Aldactone.
Sodium and water are excreted, not retained. Cardiac dysrhythmia should
not occur because of hypokalemia, since potassium is not excreted.
Patient Need: Physiological Integrity

Go to the next page if you knew the correct answer, or click the link images
below to further research the concepts in this question (if desired).

Research Concepts:
Potassium Sparing Diuretics:

Tap flag to report any problems with this question.


Question 487: In the femoral triangle, the order of the neurovascular
bundles, from LATERAL TO MEDIAL, runs:

Choices:
1. Nerve, artery, vein
2. Nerve, vein, artery
3. Vein, artery, nerve
4. Vein, nerve, artery
Answer: 1 - Nerve, artery, vein
Explanations:
The neurovascular bundle is important in the femoral triangle because
femoral punctures are commonly performed in the hospital
The femoral nerve is always lateral to the artery
Femoral venipunctures are done by palpating the pulse and placing the
needle about 1 cm medially

Go to the next page if you knew the correct answer, or click the link images
below to further research the concepts in this question (if desired).

Research Concepts:
Femoral Triangle:

Tap flag to report any problems with this question.


Question 488: Which drug is known to cause hyperpolarization of smooth
muscle through increased potassium permeability, hypertrichosis, salt and water
retention, and reflex tachycardia?

Choices:
1. Clonidine
2. Methyl dopa
3. Hydralazine
4. Minoxidil
Answer: 4 - Minoxidil
Explanations:
Minoxidil is a vasodilator which works by opening the potassium channels
The drug causes hyperpolarization of the cell membranes
It is widely used for alopecia but does not work. The maximum growth of
hair is probably a few hair strands after a few years
Common side effects include itching, redness or irritation at site of
application

Go to the next page if you knew the correct answer, or click the link images
below to further research the concepts in this question (if desired).

Research Concepts:
Minoxidil:

Tap flag to report any problems with this question.


Question 489: What is the next step in a 65-year-old with an inconclusive
V/Q perfusion scan for PE?

Choices:
1. Doppler ultrasound
2. Measure levels of D-dimer
3. Start warfarin
4. Angiogram
Answer: 4 - Angiogram
Explanations:
Spiral CT is an initial study.
Arteriogram is indicated when initial studies are inconclusive.

Go to the next page if you knew the correct answer, or click the link images
below to further research the concepts in this question (if desired).

Research Concepts:
Pulmonary Embolism:

Tap flag to report any problems with this question.


Question 490: On opening the chest for an elective coronary artery bypass
graft (CABG), a hard thickened aorta is palpated. The next step is all of the
following except:

Choices:
1. Cannulate the groins
2. Accomplish ventricular fibrillation and hypothermia
3. Give only retrograde cardioplegia
4. Perform endarterectomy of aorta
Answer: 3 - Give only retrograde cardioplegia
Explanations:
Before cardioplegia is administered, the aorta must be cross clamped
If the aorta is calcified it should never be cross clamped
If the aorta is calcified the options are off pump CABG, using the internal
mammary arteries as sites of proximal anastomosis, fibrillatory arrest,
femoral cannulation, circulatory arrest, or either replacement or
endarterectomy of the aorta

Go to the next page if you knew the correct answer, or click the link images
below to further research the concepts in this question (if desired).

Research Concepts:
Coronary Artery Bypass Graft:

Aortic Calcification:

Tap flag to report any problems with this question.


Question 491: Right axis deviation (RAD) on an ECG is defined as:
Choices:
1. 0 degrees
2. <60 degrees
3. >90 degrees
4. None of the above
Answer: 3 - >90 degrees
Explanations:
RAD is defined as >900 on an ECG
RAD can be seen with left posterior fascicular block
It can be seen in some cases of right bundle branch block
It can also be seen in some cases of Wolff-Parkinson-White syndrome

Go to the next page if you knew the correct answer, or click the link images
below to further research the concepts in this question (if desired).

Research Concepts:
Right Axis Deviation:

Tap flag to report any problems with this question.


Question 492: Which of the following is not true of the recurrent laryngeal
nerve?

Choices:
1. It is the major motor nerve of the larynx
2. It innervates the cricothyroid muscles
3. It is the major sensory nerve of the larynx and below
4. It is a branch of the vagus nerve
Answer: 2 - It innervates the cricothyroid muscles
Explanations:
The recurrent laryngeal nerve supplies the sensory innervation to the
laryngeal mucosa and below.
It also is the motor nerve of the larynx, but it does not innervate the
cricothyroid muscles. They are the only laryngeal muscles supplied by the
external branch of the superior laryngeal nerve, which is a branch of the
vagus nerve.
The recurrent laryngeal nerves are branches of the vagus nerve.
They are called recurrent because they travel in a reverse direction to the
vagus nerve.

Go to the next page if you knew the correct answer, or click the link images
below to further research the concepts in this question (if desired).

Research Concepts:
Recurrent Laryngeal Nerve:

Tap flag to report any problems with this question.


Question 493: Which of the following agents has cardiac toxicity as its
limiting factor?

Choices:
1. Bleomycin
2. Taxol
3. Asparaginase
4. Doxorubicin
Answer: 4 - Doxorubicin
Explanations:
Doxorubicin is an anthracycline antibiotic used to treat a variety of cancers.
Doxorubicin works by intercalating DNA.
The drug is administered intravenously and has several side effects.
Cumulative doses can cause cardiac side effects which can lead to CHF and
dilated cardiomyopathy. The damage to the heart is due to free radical.

Go to the next page if you knew the correct answer, or click the link images
below to further research the concepts in this question (if desired).

Research Concepts:
Doxorubicin:

Tap flag to report any problems with this question.


Question 494: To prevent development of post phlebitis syndrome after a
deep vein thrombosis, how long should compression stockings be worn?

Choices:
1. 1 week
2. 1 month
3. 3-6 months
4. 24 months
Answer: 4 - 24 months
Explanations:
Many physicians forget the importance of compression stockings in patients
with a DVT.
Compression stockings can help prevent the post phlebitic syndrome.
The compressions stocking should be worn for at least two years to be of
benefit.

Go to the next page if you knew the correct answer, or click the link images
below to further research the concepts in this question (if desired).

Research Concepts:
Deep Vein Thrombosis:

Tap flag to report any problems with this question.


Question 495: Which of the following is not a complication of a significant
atrial septal defect?

Choices:
1. Paradoxical embolism
2. Pulmonary hypertension
3. Atrial arrhythmias
4. Calcific aortic stenosis
Answer: 4 - Calcific aortic stenosis
Explanations:
A significant atrial septal defect could result in paradoxical systemic
emboli, pulmonary hypertension, congestive heart failure, atrial
enlargement, and atrial arrhythmias
There is no association with calcific aortic stenosis

Go to the next page if you knew the correct answer, or click the link images
below to further research the concepts in this question (if desired).

Research Concepts:
Atrial Septal Defect:

Tap flag to report any problems with this question.


Question 496: Rebound hypertension is often seen with which
antihypertensive?

Choices:
1. Clonidine
2. Prazosin
3. Labetalol
4. Vasotec
Answer: 1 - Clonidine
Explanations:
Rebound hypertension does occur if there is abrupt discontinuation of
clonidine.
Clonidine should be tapered slowly.

Go to the next page if you knew the correct answer, or click the link images
below to further research the concepts in this question (if desired).

Research Concepts:
Clonidine:

Tap flag to report any problems with this question.


Question 497: Right ventricular infarction is characterized by ST segment
elevation in:

Choices:
1. Lead V1
2. Lead V4
3. AVL
4. AVF
Answer: 2 - Lead V4
Explanations:
Right ventricular infarction will show ECG changes in lead V3 and V4

Go to the next page if you knew the correct answer, or click the link images
below to further research the concepts in this question (if desired).

Research Concepts:
Right Ventricular Infarction:

Tap flag to report any problems with this question.


Question 498: Which course of action is an appropriate treatment?
Choices:
1. Administering metoprolol to a patient with history of asthma for an acute
MI
2. Starting IV morphine for an acute MI for a patient with airway reaction to
codeine
3. Administering IV nitroglycerin on a patient with acute MI 5 hours after
onset
4. Bolus of IV digoxin for a patient with CHF 5 hours status post an MI
Answer: 3 - Administering IV nitroglycerin on a patient with acute MI 5
hours after onset

Explanations:
Nitroglycerin may be repeated at 5 minute intervals for as long as BP is
maintained
Starting dose for nitroglycerin is 10 ug/min

Go to the next page if you knew the correct answer, or click the link images
below to further research the concepts in this question (if desired).

Research Concepts:
Acute Myocardial Infarction:

Tap flag to report any problems with this question.


Question 499: Which of the following is not a physical finding with
pleural effusions?

Choices:
1. Asymmetrical chest expansion
2. Decreased air entry
3. Increased tactile fremitus
4. Dull percussion
Answer: 3 - Increased tactile fremitus
Explanations:
Chest expansion can be asymmetrical and chest lag can be seen.
Air entry is decreased and the percussion tone becomes dull.
Tactile fremitus in increased when there is pneumothorax and decreased
with a pleural effusion.

Go to the next page if you knew the correct answer, or click the link images
below to further research the concepts in this question (if desired).

Research Concepts:
Effusion, Pleural:

Tap flag to report any problems with this question.


Question 500: Which cardiac condition is most likely to have ankle
edema?

Choices:
1. Left heart failure
2. Right heart failure
3. Aortic stenosis
4. Mitral stenosis
Answer: 2 - Right heart failure
Explanations:
Peripheral edema is a sign of right heart failure
Left heart failure can cause pulmonary edema

Go to the next page if you knew the correct answer, or click the link images
below to further research the concepts in this question (if desired).

Research Concepts:
Heart Failure, Congestive:

Tap flag to report any problems with this question.


Question 501: What intervention can be helpful to a patient with
worsening dilated cardiomyopathy while on the waiting list for a heart
transplant?

Choices:
1. Regular medication schedule
2. Use of cardiac rehabilitation
3. Use of ventricular assist device
4. External pacemaker
Answer: 3 - Use of ventricular assist device
Explanations:
Patients who's hearts are getting worse before there is an available heart can
be offered a ventricular assistive device
Cardiac rehabilitation and continuing regular medications would not be
helpful with a deteriorating heart condition

Go to the next page if you knew the correct answer, or click the link images
below to further research the concepts in this question (if desired).

Research Concepts:
Cardiomyopathy, Dilated:

Left Ventricular Assist Device, Function:

Tap flag to report any problems with this question.


Question 502: Which of the following is a contraindication to image-
guided surgery (IGS)?

Choices:
1. The patient has a history of allergy to contrast products
2. The patient has a pacemaker
3. The procedure is revision endoscopic sinus surgery with distorted anatomy
anticipated
4. None of the above
Answer: 4 - None of the above
Explanations:
IGS has virtually no contraindication.
No contrast injection is required. No interference is to be feared with
implantable devices or prostheses, even by electromagnetic IGS systems.
An up-to-date preoperative CT scan is always a prerequisite of IGS,
regardless of the extent of pre-existing anatomical distortion.
The issue of head movements during surgery is also eliminated by the use
of a specific headset during image acquisition for automated registration.

Go to the next page if you knew the correct answer, or click the link images
below to further research the concepts in this question (if desired).

Research Concepts:
Image-Guided Surgery:

Tap flag to report any problems with this question.


Question 503: Select the factor that does not increase the chance of
infection after a clean surgery?

Choices:
1. Longer duration
2. Bleeding
3. Hematoma
4. Preoperative antibiotics
Answer: 4 - Preoperative antibiotics
Explanations:
Preoperative antibiotics can decrease the risk of infection after surgery
Hematoma, longer surgery, bleeding, and use of foreign material are all risk
factors for infection

Go to the next page if you knew the correct answer, or click the link images
below to further research the concepts in this question (if desired).

Research Concepts:
Post-op Wound Infection:

Tap flag to report any problems with this question.


Question 504: What is the hemodynamic effect of nesiritide
administration?

Choices:
1. Increase preload and decrease afterload
2. Decrease preload and increase afterload
3. Decrease in both preload and afterload
4. Increase in both preload and afterload
Answer: 3 - Decrease in both preload and afterload
Explanations:
Nesiritide is a recombinant human brain natriuretic peptide
It causes both arterial and venous vasodilation
It is used in treatment of acute decompensated heart failure

Go to the next page if you knew the correct answer, or click the link images
below to further research the concepts in this question (if desired).

Research Concepts:
Nesiritide:

Tap flag to report any problems with this question.


Question 505: Which substance released from endothelium causes
vasodilatation?

Choices:
1. Bradykinin
2. Thromboxane
3. Nitric oxide
4. Angiotensin
Answer: 3 - Nitric oxide
Explanations:
The blood vessels endothelium utilize nitric oxide as a signal to relax
surrounding smooth muscle, dilating the artery resulting in increased blood
flow.
The actions of nitrates including nitroglycerin involve this mechanism.
These compounds are converted to nitric oxide.
This is the pathophysiology in treatment of coronary artery disease.

Go to the next page if you knew the correct answer, or click the link images
below to further research the concepts in this question (if desired).

Research Concepts:
Vasodilation:

Tap flag to report any problems with this question.


Question 506: Which of the following cancers is most common in patients
who undergo transplants or receive chemotherapy?

Choices:
1. Colon cancer
2. Lung cancer
3. Skin cancer
4. Blood cancer
Answer: 3 - Skin cancer
Explanations:
Transplant recipients are at high risk for cancer because of
immunosuppression, chronic antigenemia, susceptibility to oncogenic
viruses, and use of potent drugs.
Cancers of the colon, lung, bladder, and prostate do occur.
The risk of cancer is highest for the skin, lip, and kidney, and for
lymphomas.
Surveillance is necessary for all patients who undergo transplant. All
suspicious lesions must be biopsied.

Go to the next page if you knew the correct answer, or click the link images
below to further research the concepts in this question (if desired).

Research Concepts:
Posttransplant Cancer:

Tap flag to report any problems with this question.


Question 507: Which is not true about dextran?
Choices:
1. Increases bleeding time
2. Is only available synthetically
3. Impairs platelet aggregation
4. Causes hyperglycemia
Answer: 2 - Is only available synthetically
Explanations:
Dextran can occur naturally or synthesized in the GI mucosa

Go to the next page if you knew the correct answer, or click the link images
below to further research the concepts in this question (if desired).

Research Concepts:
Dextran:

Tap flag to report any problems with this question.


Question 508: What drug has the highest blocking potency for sodium
channel?

Choices:
1. Quinidine
2. Mexiletine
3. Flecainide
4. Phenytoin
Answer: 3 - Flecainide
Explanations:
Flecainide is a class IC drug
It has the highest potency

Go to the next page if you knew the correct answer, or click the link images
below to further research the concepts in this question (if desired).

Research Concepts:
Flecainide:

Tap flag to report any problems with this question.


Question 509: A patient has just been diagnosed with a presumed MI.
Which is the most appropriate initial route of administration for nitroglycerin?

Choices:
1. Intravenous
2. Oral
3. Sublingual
4. Transdermal
Answer: 3 - Sublingual
Explanations:
Sublingual nitroglycerin has a faster onset of action than oral and
transdermal forms.
Sublingual and intravenous nitroglycerin have a similar onset of action.
However, it takes longer to administer intravenous nitroglycerin because it
requires an intravenous infusion to be set up.

Go to the next page if you knew the correct answer, or click the link images
below to further research the concepts in this question (if desired).

Research Concepts:
Myocardial Infarction, Acute:

Nitroglycerin:

Tap flag to report any problems with this question.


Question 510: Which of following recommendations is not true in regard
to a one week post heart transplantation patient presenting to ER with vomiting,
diarrhea, and chest pain?

Choices:
1. Start methylprednisolone IV
2. Evaluate patient for cytomegalovirus infection
3. Possible acute rejection
4. Patient may need liver biopsy
Answer: 1 - Start methylprednisolone IV
Explanations:
Endomyocardial biopsy must be performed to rule out acute rejection
Although methylprednisolone is first line for acute rejection, it should be
delayed until confirmation from biopsy

Go to the next page if you knew the correct answer, or click the link images
below to further research the concepts in this question (if desired).

Research Concepts:
Heart Transplantation:

Tap flag to report any problems with this question.


Question 511: Which of following abbreviations is acceptable?
Choices:
1. U
2. MS
3. QHS
4. QHR
Answer: 4 - QHR
Explanations:
Abbreviations to avoid are U, MS, and QHS
They often lead to confusion resulting in medical errors

Go to the next page if you knew the correct answer, or click the link images
below to further research the concepts in this question (if desired).

Research Concepts:
Prescriptions:

Tap flag to report any problems with this question.


Question 512: Which of the following physical findings would be expected
in an individual with long standing chronic obstructive pulmonary disease?

Choices:
1. Normal diaphragm movement
2. Barrel chest deformity
3. Increased tactile fremitus
4. Normal lung sounds
Answer: 2 - Barrel chest deformity
Explanations:
Barrel chest deformity refers to increased A-P diameters in individuals with
COPD
A-P to transverse diameter is close to 1:1 with barrel chest deformity
Other physical findings could include distant lung sounds, decreased
diaphragm movement and decreased tactile fremitus

Go to the next page if you knew the correct answer, or click the link images
below to further research the concepts in this question (if desired).

Research Concepts:
Obstructive Pulmonary Disease, Chronic (COPD):

Barrel Chest:

Tap flag to report any problems with this question.


Question 513: Which is true regarding pulmonic stenosis?
Choices:
1. Frequently caused by rheumatic heart disease
2. Usually caused by congenital and hereditary cause
3. Medical management is indicated in severe cases
4. Not seen in Tetralogy of Fallot
Answer: 2 - Usually caused by congenital and hereditary cause
Explanations:
Pulmonary stenosis is usually congenital, rarely from rheumatic heart
disease
May be associated with other congenital cardiac defects, such as Tetralogy
of Fallot, or hypoplastic RV, PFO, or PDA
Surgical management, balloon valvuloplasty, or other techniques to open or
replace the valve is curative

Go to the next page if you knew the correct answer, or click the link images
below to further research the concepts in this question (if desired).

Research Concepts:
Pulmonic Stenosis:

Tap flag to report any problems with this question.


Question 514: Which of the following will not delay the healing of a
wound?

Choices:
1. Vitamin deficiency
2. Ischemia
3. Infection
4. Blood transfusion
Answer: 4 - Blood transfusion
Explanations:
Blood transfusion may aid healing of a wound if there was anemia
Deficiency of vitamins C and E would slow wound healing
Infections and ischemia will delay or prevent wound healing

Go to the next page if you knew the correct answer, or click the link images
below to further research the concepts in this question (if desired).

Research Concepts:
Wound Healing:

Tap flag to report any problems with this question.


Question 515: A patient has a stage 2 decubitus ulcer over the sacrum
measuring 1.5 cm. There is no discharge, odor, or surrounding erythema. Select
the most important treatment to promote healing.

Choices:
1. Hydrocolloid dressing
2. Oral antibiotics
3. Hydrotherapy
4. Avoiding pressure
Answer: 4 - Avoiding pressure
Explanations:
Decubitus ulcers are caused by pressure leading to ischemia
Frequent turning of immobilized patients is needed for prevention
Once an ulcer has formed, avoidance of pressure over the area is needed for
healing
The area should be kept clean and moist, so hydrocolloid dressings can be
helpful

Go to the next page if you knew the correct answer, or click the link images
below to further research the concepts in this question (if desired).

Research Concepts:
Decubitus Ulcers:

Tap flag to report any problems with this question.


Question 516: Select the statement that best describes pulsus paradoxus.
Choices:
1. It is seen during cardiac tamponade resulting in a larger than normal drop
during inspiration of the systolic blood pressure
2. It is seen during severe asthma when there is larger than normal increase
during expiration of the diastolic blood pressure
3. It is not seen during superior vena cava syndrome
4. It is seen during bigeminy when there is an alternating decrease in pulse
Answer: 1 - It is seen during cardiac tamponade resulting in a larger than
normal drop during inspiration of the systolic blood pressure

Explanations:
Acute asthma, COPD, cardiac tamponade, superior vena cava syndrome can
all cause pulsus paradoxus
The physical finding consists of a greater than 10 mm Hg drop in systolic
blood pressure during inspiration

Go to the next page if you knew the correct answer, or click the link images
below to further research the concepts in this question (if desired).

Research Concepts:
Pulsus Paradoxus:

Tap flag to report any problems with this question.


Question 517: A 14-year-old male with hemophilia A is undergoing
surgery to remove his tonsils. Which of the following is the best option for
preventing bleeding complications?

Choices:
1. Fresh-frozen plasma and cryoprecipitate
2. Desmopressin
3. Erythropoietin
4. Synthetic factor VIII
Answer: 4 - Synthetic factor VIII
Explanations:
Hemophilia A is a coagulation disorder resulting from a deficiency or
abnormality of factor VIII. Although FFP contains factor VIII, the levels
are too low to prevent or control bleeding in hemophilia.
Desmopressin (DDAVP) is a synthetic analogue of anti-diuretic hormone
that increases levels of factor VIII and von Willebrand factor. DDAVP can
be used alone for mild hemophilia A, but it is ineffective in severe forms of
the disease.
Erythropoietin does not prevent bleeding complications in hemophilia.
Pooled concentrated or recombinant factor VIII is given before surgery

Go to the next page if you knew the correct answer, or click the link images
below to further research the concepts in this question (if desired).

Research Concepts:
Hemophilia A:

Tap flag to report any problems with this question.


Question 518: Which of the following conditions causes cyanosis?
Choices:
1. Ventricular septal defect
2. Tetralogy of Fallot
3. Atrial septal defect
4. Patent ductus arteriosus
Answer: 2 - Tetralogy of Fallot
Explanations:
Tetralogy of Fallot is a congenital heart lesion associated with cyanosis
It consists of right ventricular outflow obstruction, ventricular septal defect,
right ventricular hypertrophy, and dextroposition of the aorta with
overriding of the aorta over the septum
The right ventricular outflow obstruction helps create a right to left shunt
causing venous blood to be shunted to the overriding aorta and into the
systemic circulation causing cyanosis
Patent ductus arteriosus, atrial septal defect, and ventricular septal defect all
cause a left to right shunts

Go to the next page if you knew the correct answer, or click the link images
below to further research the concepts in this question (if desired).

Research Concepts:
Tetralogy Of Fallot:

Tap flag to report any problems with this question.


Question 519: What is the longest acting local anesthetic?
Choices:
1. Lidocaine
2. Cocaine
3. Bupivacaine
4. Procaine
Answer: 3 - Bupivacaine
Explanations:
Bupivacaine is indicated for local anesthesia and is used for epidural
anesthesia. Sometimes bupivacaine is combined with epinephrine to
prolong the action.
Bupivacaine binds to sodium channels and blocks sodium entry into nerve
cells, preventing depolarization.
Bupivacaine is one of the longest acting local anesthetics and can often
provide pain relief for up to 20 hours.
Compared to other local anesthetics, the drug is markedly cardiotoxic.
Cardiac effects include hypotension, bradycardia, arrhythmias, and cardiac
arrest.

Go to the next page if you knew the correct answer, or click the link images
below to further research the concepts in this question (if desired).

Research Concepts:
Local Anesthetics:

Tap flag to report any problems with this question.


Question 520: Which of the following drugs is the best initial choice for
controlling ventricular tachycardia in patients with acute myocardial infarction?

Choices:
1. Dobutamine
2. Digitalis
3. Quinidine
4. Lidocaine
Answer: 4 - Lidocaine
Explanations:
Lidocaine is a class IB antiarrhythmic with low incidence of toxicity
It has high degree of effectiveness in arrhythmias associated with acute
myocardial infarction
Lidocaine acts by blocking activated and inactivated sodium channels

Go to the next page if you knew the correct answer, or click the link images
below to further research the concepts in this question (if desired).

Research Concepts:
Lidocaine:

Acute Myocardial Infarction:

Ventricular Tachycardia:
Tap flag to report any problems with this question.
Question 521: Which of the following side effects is seen with the use of
ticarcillin?

Choices:
1. Fulminant liver failure
2. Drug-induced lupus
3. Hematologic abnormalities
4. Nephrotoxicity
Answer: 3 - Hematologic abnormalities
Explanations:
Ticarcillin works just like other penicillin by inhibiting bacterial cell wall
synthesis by binding to one or more of the penicillin binding proteins
Hematologic abnormalities such as bleeding or eosinophilia occur with the
use of this drug
Positive Coombs' reaction and abnormal coagulation profiles may also
occur

Go to the next page if you knew the correct answer, or click the link images
below to further research the concepts in this question (if desired).

Research Concepts:
Ticarcillin/Clavulanate:

Tap flag to report any problems with this question.


Question 522: What is the drug of choice in patients post myocardial
infarction?

Choices:
1. Metoprolol
2. Clonidine
3. Furosemide
4. Captopril
Answer: 1 - Metoprolol
Explanations:
Post MI, beta blockers should be initiated.
Moreover, one should not forget to add aspirin to the regimen.
If there is left ventricular dysfunction, an ACE inhibitor should be started.

Go to the next page if you knew the correct answer, or click the link images
below to further research the concepts in this question (if desired).

Research Concepts:
Myocardial Infarction, Acute:

Tap flag to report any problems with this question.


Question 523: Which of the following will trigger the low-pressure alarm
in a ventilated patient?

Choices:
1. Increased secretions
2. Endotracheal tube cuff leak
3. Uncontrolled pain
4. Coughing
Answer: 2 - Endotracheal tube cuff leak
Explanations:
The low-pressure alarm is an indicator that the patient is disconnected from
the ventilator.
He or she is not receiving the benefit of the ventilator.

Go to the next page if you knew the correct answer, or click the link images
below to further research the concepts in this question (if desired).

Research Concepts:
Mechanical Ventilation:

Tap flag to report any problems with this question.


Question 524: When is thrombolytic therapy for patients with a PE
indicated?

Choices:
1. Sepsis
2. Recent surgery
3. Right heart strain
4. Patient is allergic to heparin
Answer: 3 - Right heart strain
Explanations:
To date, thrombolytic therapy has not proven to improve morbidity or
mortality in patients with PE.
Thrombolytic therapy is only indicated in patients with hemodynamic
instability or right ventricular dysfunction.
The therapy must be started within the first 24 hours to resolve the
thrombus.

Go to the next page if you knew the correct answer, or click the link images
below to further research the concepts in this question (if desired).

Research Concepts:
Pulmonary Embolism:

Tap flag to report any problems with this question.


Question 525: What percentage of the elderly do not complain of chest
pain with an acute myocardial infarction (AMI)?

Choices:
1. 10%
2. 30%
3. 40%
4. >50%
Answer: 4 - >50%
Explanations:
>50% of all elderly do not complain of chest pain while an AMI is evolving
Neuropathy prevents them from sensing the pain of AMI
The phenomenon is especially true of the elderly with diabetes mellitus

Go to the next page if you knew the correct answer, or click the link images
below to further research the concepts in this question (if desired).

Research Concepts:
Acute Myocardial Infarction:

Silent Myocardial Ischemia:

Tap flag to report any problems with this question.


Question 526: Myocardial fiber length is mainly determined by which of
the following?

Choices:
1. Preload
2. Afterload
3. Resistance
4. Contractility
Answer: 1 - Preload
Explanations:
Preload is the predominant factor for myocardial fiber length.

Go to the next page if you knew the correct answer, or click the link images
below to further research the concepts in this question (if desired).

Research Concepts:
Cardiac Physiology:

Tap flag to report any problems with this question.


Question 527: What is the best way to monitor a patient with a cardiac
contusion?

Choices:
1. CT of chest
2. ECHO
3. ECG monitoring x 24 hrs
4. Cardiac enzymes
Answer: 3 - ECG monitoring x 24 hrs
Explanations:
There is a poor correlation between cardiac enzymes and contusion. Thus
the enzymes are not part of an investigation.
Patients with cardiac contusion are best monitored by 24 hour ECG
monitoring. A normal ECG at admission makes cardiac contusion very
unlikely.
Echo has been used to assess cardiac wall abnormalities to exclude lesions
that may require surgery.
Sometimes sternal fractures are associated with cardiac contusion.

Go to the next page if you knew the correct answer, or click the link images
below to further research the concepts in this question (if desired).

Research Concepts:
Cardiac Contusion:

Tap flag to report any problems with this question.


Question 528: Which of the following is least likely in patients with beta-
blocker overdose?

Choices:
1. Hypotension and bradycardia
2. PR prolongation
3. QRS prolongation
4. Hyperglycemia
Answer: 4 - Hyperglycemia
Explanations:
EKG will show PR and QRS prolongation
Hypotension and bradycardia is a common finding in beta-blocker overdose
Hypoglycemia, rather than hyperglycemia, is commonly seen in such cases

Go to the next page if you knew the correct answer, or click the link images
below to further research the concepts in this question (if desired).

Research Concepts:
Toxic, Beta-Blocker:

Tap flag to report any problems with this question.


Question 529: Which of the following is true about Wolff-Parkinson-White
(WPW) syndrome?

Choices:
1. PR prolongation is a common finding
2. Only half of patients will have symptoms
3. Seventy percent of these cases are associated with underlying cardiac
disease
4. Wide complex tachycardias are the most common presentation
Answer: 2 - Only half of patients will have symptoms
Explanations:
WPW syndrome is the most common accessory pathway syndrome
Classically the PR interval is shortened and a slurring of the upstroke of the
QRS complex occurs, called a delta wave
Only half of WPW patients ever experience symptoms
The most common symptoms are related to supraventricular tachycardia
due to reentry

Go to the next page if you knew the correct answer, or click the link images
below to further research the concepts in this question (if desired).

Research Concepts:
Wolff-Parkinson-White Syndrome:

Tap flag to report any problems with this question.


Question 530: Which non-steroidal anti-inflammatory drug is used
frequently to occlude the patent ductus arteriosus?

Choices:
1. Aspirin
2. Indomethacin
3. Celebrex
4. Ibuprofen
Answer: 2 - Indomethacin
Explanations:
Indomethacin is the drug often used to close the patent ductus arteriosus.

Go to the next page if you knew the correct answer, or click the link images
below to further research the concepts in this question (if desired).

Research Concepts:
Patent Ductus Arteriosus:

Tap flag to report any problems with this question.


Question 531: Which is least likely to prevent infection in patients with
central venous catheter?

Choices:
1. Good sterile barrier
2. Use of povidone iodine over chlorhexidine
3. Use of antibiotic impregnated catheters
4. Proper dressing at entry site
Answer: 2 - Use of povidone iodine over chlorhexidine
Explanations:
Infection rate is lower with chlorhexidine than povidone iodine solution.

Go to the next page if you knew the correct answer, or click the link images
below to further research the concepts in this question (if desired).

Research Concepts:
Central Venous Catheter:

Tap flag to report any problems with this question.


Question 532: What is the next best step in a patient with a mid-systolic
ejection murmur heard best at the 2nd intercostal space and radiating to the
neck?

Choices:
1. Coronary artery bypass graft (CABG)
2. Antibiotics prior to dental procedures
3. Doppler echocardiography
4. None of the above
Answer: 3 - Doppler echocardiography
Explanations:
The murmur of aortic stenosis is classically mid-systolic heard best at 2nd
intercostal space and radiating to the neck
Doppler echocardiography is the imaging modality of choice to diagnose
and estimate severity of disease
Antibiotics prior to dental procedure are usually recommended to those
with mitral valve prolapse with valvular regurgitation
CABG is not an appropriate procedure in this patient

Go to the next page if you knew the correct answer, or click the link images
below to further research the concepts in this question (if desired).

Research Concepts:
Aortic Stenosis:

Tap flag to report any problems with this question.


Question 533: What medication is most rarely used for acute angina?
Choices:
1. Oxygen
2. Calcium channel blockers
3. Morphine
4. Nitroglycerin
Answer: 2 - Calcium channel blockers
Explanations:
Morphine is also a venodilator and analgesic
All patients with acute angina should be given oxygen, nitroglycerin,
oxygen, and aspirin
Other forms of anticoagulants are often given
Calcium channel blockers are only used if the patient has Prinzmetal angina

Go to the next page if you knew the correct answer, or click the link images
below to further research the concepts in this question (if desired).

Research Concepts:
Angina:

Tap flag to report any problems with this question.


Question 534: Which of the following statements is true with regards to
aortic stenosis?

Choices:
1. Rheumatic heart disease is associated with 5 percent of the patients
2. Congenital bicuspid valve is the most common cause
3. ECG may show ST segment depression
4. Patient may experience exertional angina
Answer: 2 - Congenital bicuspid valve is the most common cause
Explanations:
Congenital bicuspid valve is the number one cause of AS
Second most common cause is rheumatic heart disease, which is associated
with 2 percent of the population

Go to the next page if you knew the correct answer, or click the link images
below to further research the concepts in this question (if desired).

Research Concepts:
Aortic Stenosis:

Tap flag to report any problems with this question.


Question 535: Pulmonary arterial hypertension can be treated with:
Choices:
1. Treprostinil
2. Indomethacin
3. Prazosin
4. Hydralazine
Answer: 1 - Treprostinil
Explanations:
Treprostinil is a prostaglandin analog used for treatment of pulmonary
arterial hypertension.
It decrease exercise-associated symptoms in patients with NYHA Class II-
IV.
Treprostinil is a direct vasodilator of both pulmonary and systemic arterial
vascular beds.
It also inhibits platelet aggregation.

Go to the next page if you knew the correct answer, or click the link images
below to further research the concepts in this question (if desired).

Research Concepts:
Treprostinil:

Pulmonary Hypertension:

Tap flag to report any problems with this question.


Question 536: Beta blockers are contraindicated in patients with acute
myocardial infarction and:

Choices:
1. Asthma
2. Left ventricular dysfunction
3. Second degree AV block
4. Pulse less than 60 per minute
Answer: 4 - Pulse less than 60 per minute
Explanations:
Due to its mortality reduction, beta blockers are indicated in most patients
with AMI
Contraindicated of BB are second-degree AV block, COPD, and heart rate
less than 60 per minute

Go to the next page if you knew the correct answer, or click the link images
below to further research the concepts in this question (if desired).

Research Concepts:
Beta-Blockers:

Tap flag to report any problems with this question.


Question 537: After coronary artery bypass grafting a patient develops
pericarditis. Which of the following would be contraindicated?

Choices:
1. Active range of motion of the upper extremities
2. High-level exercise
3. Ambulation in the halls
4. Bed positioning to avoid skin breakdown
Answer: 2 - High-level exercise
Explanations:
Pericarditis is a contraindication for high-level exercise

Go to the next page if you knew the correct answer, or click the link images
below to further research the concepts in this question (if desired).

Research Concepts:
Pericarditis:

Tap flag to report any problems with this question.


Question 538: What is the best treatment regimen for a postoperative deep
venous thrombosis?

Choices:
1. Heparin GTT and Plavix 75 mg qd
2. Aspirin 325 mg PO qd with warfarin titrated to INR of >3.0
3. Plavix 75 mg qd PO with warfarin titrated to INR of >2.0
4. Enoxaparin at 1 mg/kg SC with warfarin titrated to INR of 2.0 to 2.5
Answer: 4 - Enoxaparin at 1 mg/kg SC with warfarin titrated to INR of 2.0
to 2.5

Explanations:
The patient needs to become therapeutic with a heparin gtt or enoxaparin
followed by warfarin with an INR of 2.0 to 2.5 for 3-6 months.
DVT prophylaxis is Heparin 5000 U SQ BID or TID, or Enoxaparin 30 to
40 mg SQ daily or BID.

Go to the next page if you knew the correct answer, or click the link images
below to further research the concepts in this question (if desired).

Research Concepts:
Deep Venous Thrombosis, Lower Extremity:

Tap flag to report any problems with this question.


Question 539: How does the radiation dose of a PA and lateral CXR
compare to average North American background radiation?

Choices:
1. It is 20 times less
2. It is 2 times more
3. It is equal to
4. It is 200 times less
Answer: 1 - It is 20 times less
Explanations:
Background radiation varies by elevation from sea level. On average in
North America it is 3 mSv (.3 Rem)
A PA/Lat CXR exposes the patient to 0.15 mSv (15 mRem)
The CXR is exposes patients to less ionizing radiation than virtually any
other imaging study using ionizing radiation.
It takes less ionizing radiation to penetrate lung than soft tissue or bone.

Go to the next page if you knew the correct answer, or click the link images
below to further research the concepts in this question (if desired).

Research Concepts:
Chest Radiographs:

Ionizing Radiation Exposure, Medical Imaging:

Tap flag to report any problems with this question.


Question 540: Which of the following is incorrect about mitral stenosis?
Choices:
1. Patients may present with cough, hemoptysis, and dyspnea on exertion
2. Patients usually present during the third decade
3. Complications include pulmonary hypertension and right ventricular failure
4. Other complications include chronic atrial fibrillation and embolization
Answer: 2 - Patients usually present during the third decade
Explanations:
Mitral stenosis usually becomes symptomatic in the fourth decade but some
patients are affected at a younger age

Go to the next page if you knew the correct answer, or click the link images
below to further research the concepts in this question (if desired).

Research Concepts:
Mitral Stenosis:

Tap flag to report any problems with this question.


Question 541: A patient receiving intravenous morphine is particularly
closely monitored for which one of the following?

Choices:
1. Heart rate
2. Mental status
3. Constipation
4. Respiratory status
Answer: 4 - Respiratory status
Explanations:
Patients who receive IV morphine are usually monitored with regard to
their respiratory status utilizing pulse oximetry

Go to the next page if you knew the correct answer, or click the link images
below to further research the concepts in this question (if desired).

Research Concepts:
Morphine:

Tap flag to report any problems with this question.


Question 542: Which of the following medications is contraindicated in
pregnant women?

Choices:
1. Alphamethyldopa
2. Labetalol
3. Verapamil
4. Captopril
Answer: 4 - Captopril
Explanations:
Captopril and other ACE inhibitors are considered teratogenic
Their use is contraindicated in pregnancy
Methyldopa is a drug frequently used to manage hypertension in pregnant
women

Go to the next page if you knew the correct answer, or click the link images
below to further research the concepts in this question (if desired).

Research Concepts:
ACE Inhibitors:

Teratogens:

Tap flag to report any problems with this question.


Question 543: During cardiopulmonary bypass, the blood flow is adjusted
to what rate?

Choices:
1. 25 mL per kg per minute
2. 35 mL per kg per minute
3. 45 mL per kg per minute
4. 55 mL per kg per minute
Answer: 4 - 55 mL per kg per minute
Explanations:
A normal flow rate is 55 mL per kg per minute.
The rate should be adjusted to peripheral resistance.

Go to the next page if you knew the correct answer, or click the link images
below to further research the concepts in this question (if desired).

Research Concepts:
Cardiopulmonary Bypass:

Tap flag to report any problems with this question.


Question 544: What is the most common cause of myocarditis in
adolescents?

Choices:
1. Drugs
2. Coxsackie virus
3. Tuberculosis
4. Post myocardial infarction
Answer: 2 - Coxsackie virus
Explanations:
Group B coxsackie viruses are a major cause of myopericarditis
The manifestations of enterovirus caused myocarditis are variable. The
disease is usually very severe and acute in neonates with other general
pathology.
Myocarditis in adults and adolescents is usually benign and follows a
chronic and subacute course. Only a few individuals develop a severe
myocarditis that requires a transplant.
It is thought that the virus may persist after the initial infection and may
cause dilated cardiomyopathy.

Go to the next page if you knew the correct answer, or click the link images
below to further research the concepts in this question (if desired).

Research Concepts:
Myocarditis, Viral:

Tap flag to report any problems with this question.


Question 545: Which of the following drugs needs special dose adjustment
in patients with decreased renal function?

Choices:
1. Gentamicin
2. Cefoxitin
3. Cholestyramine
4. Terbinafine
Answer: 1 - Gentamicin
Explanations:
Gentamicin is usually given in the following dose: 1.5 mg/kg q 8 h or 3
mg/kg q 24 hours
Gentamicin dose calculation should occur in patients with renal dysfunction
Amikacin is not absorbed from the gastrointestinal tract
Terbinafine is a topical antifungal that is not indicated for systemic use

Go to the next page if you knew the correct answer, or click the link images
below to further research the concepts in this question (if desired).

Research Concepts:
Gentamicin:

Renal Insufficiency:

Tap flag to report any problems with this question.


Question 546: What is not a common clinical finding with aortic stenosis?
Choices:
1. Split S1
2. Loud S2
3. Paradoxical split
4. Soft S4
Answer: 2 - Loud S2
Explanations:
Aortic stenosis is characterized by paradoxical splitting and soft S2

Go to the next page if you knew the correct answer, or click the link images
below to further research the concepts in this question (if desired).

Research Concepts:
Aortic Stenosis:

Tap flag to report any problems with this question.


Question 547: Carotid artery occlusion would result in:
Choices:
1. Bradycardia
2. Arrhythmia
3. Tachycardia
4. Bradyarrhythmia
Answer: 3 - Tachycardia
Explanations:
Carotid artery occlusion raises the heart rate by stimulating baroreceptors
The baroreceptors in the carotid body are sensitive to blood flow
The tachycardia, caused by the baroreceptors, can be reversed by beta
blockers

Go to the next page if you knew the correct answer, or click the link images
below to further research the concepts in this question (if desired).

Research Concepts:
Baroreceptors:

Tap flag to report any problems with this question.


Question 548: Hypokalemia can worsen the toxicity of which of the
following drugs?

Choices:
1. Metoprolol
2. Quinidine
3. Digoxin
4. Lidocaine
Answer: 3 - Digoxin
Explanations:
Adverse effects of digoxin are more common in the presence of
hypokalemia.
Additionally hypercalcemia may also increase arrhythmias including
bigeminy and ventricular tachycardia.
Other adverse effects of digoxin include nausea, vomiting, and diarrhea.
At higher doses, digoxin can cause confusion, amnesia, dizziness, and
yellowish green halos.

Go to the next page if you knew the correct answer, or click the link images
below to further research the concepts in this question (if desired).

Research Concepts:
Hypokalemia:

Digoxin:

Tap flag to report any problems with this question.


Question 549: What are advanced directives?
Choices:
1. Documents that state the patients medical wishes in the event they become
incompetent
2. Documents specifying who has legal authority to make health care
decisions on their behalf
3. Documents which establish the patient's do not resuscitate status
4. Documents that establish CPR and intubation protocols
Answer: 1 - Documents that state the patients medical wishes in the event
they become incompetent

Explanations:
Advanced directives may take several forms; they may designate who may
make health care decisions, establish a living will, and DNR status
They are recognized most but not all states

Go to the next page if you knew the correct answer, or click the link images
below to further research the concepts in this question (if desired).

Research Concepts:
Advanced Directives:

Tap flag to report any problems with this question.


Question 550: What LDL-C goal did the NCEP ATP III guidelines
recommend for patients with a prior history of myocardial infarction?

Choices:
1. <200 mg/dL
2. <100 mg/dL
3. <130 mg/dL
4. <50 mg/dL
Answer: 2 - <100 mg/dL
Explanations:
LDL-C lowering therapy greatly reduces risk for major coronary events and
strokes in those with CAD and CAD risk equivalents
The recommended LDL-C goal in high-risk patients is <100 mg/dL
An LDL-C lowering drug will be required to achieve levels <100 mg/dL in
addition to lifestyle changes
Recent clinical evidence advocates an LDL-C<70 mg/dL as a therapeutic
option for very high-risk patients

Go to the next page if you knew the correct answer, or click the link images
below to further research the concepts in this question (if desired).

Research Concepts:
Myocardial Infarction:

Low Density Lipoprotein:

Primary And Secondary Prevention Of Coronary Artery


Disease:
Tap flag to report any problems with this question.
Question 551: A 55-year-old man is concerned about his risk of having a
heart attack. Among other things, his physician explains the importance of
maintaining normal blood cholesterol levels and orders lab studies to evaluate
his blood lipids. Which of the following lipoproteins has the greatest cholesterol
component?

Choices:
1. HDL
2. IDL
3. VLDL
4. LDL
Answer: 4 - LDL
Explanations:
Of all the lipoproteins, LDL (also referred to as "bad cholesterol") has the
greatest cholesterol component and serves in transporting cholesterol from
the liver to other body tissues.
HDL (also referred to as "good cholesterol") carries cholesterol from other
body parts back to the liver.

Go to the next page if you knew the correct answer, or click the link images
below to further research the concepts in this question (if desired).

Research Concepts:
Low Density Lipoprotein:

Tap flag to report any problems with this question.


Question 552: The best time to repair an elective asymptomatic tetralogy
of Fallot (TOF) is:

Choices:
1. At the time of diagnosis
2. About 1 year of age
3. After placement of a BT shunt
4. Between the ages of 3-6
Answer: 2 - About 1 year of age
Explanations:
The timing of surgery for TOF is slightly controversial. Good results can be
achieved in selected patients in all ages
The current trend is to repair most asymptomatic TOF when a child is about
1 year of age
Complete repair in symptomatic infants is carried out earlier if the infant is
over 6 kg, has adequate size pulmonary arteries, absence of multiple VSD,
AV septal defect, or pulmonary atresia
The presence of an anomalous coronary artery crossing the right ventricular
outflow tract might lead to deferral of surgery to about 2 years

Go to the next page if you knew the correct answer, or click the link images
below to further research the concepts in this question (if desired).

Research Concepts:
Tetralogy Of Fallot:

Tap flag to report any problems with this question.


Question 553: Postoperative DVT prevention is best accomplished by
which answer?

Choices:
1. Sequential Compression Devices
2. Aspirin
3. Low molecular weight heparin
4. Warfarin
Answer: 3 - Low molecular weight heparin
Explanations:
Low molecular weight heparin has been shown to be effective in DVT
prevention
Warfarin has more side effects and has a higher risk of intracranial bleeding
SCDs and aspirin are not sufficient for DVT prevention

Go to the next page if you knew the correct answer, or click the link images
below to further research the concepts in this question (if desired).

Research Concepts:
Deep Venous Thrombosis Prophylaxis:

Tap flag to report any problems with this question.


Question 554: What temperature should red blood cells be warmed prior to
transfusion?

Choices:
1. 37 degree F
2. 60 degree F
3. 100 degree F
4. 120 degree F
Answer: 3 - 100 degree F
Explanations:
Blood product should be warmed to 100 degree F prior administration to
prevent hypothermia and acidosis.

Go to the next page if you knew the correct answer, or click the link images
below to further research the concepts in this question (if desired).

Research Concepts:
Blood Transfusion:

Tap flag to report any problems with this question.


Question 555: Which diuretic does not deplete potassium?
Choices:
1. Bumetanide
2. Triamterene
3. Metolazone
4. Hydrochlorothiazide
Answer: 2 - Triamterene
Explanations:
Triamterene, spironolactone, and amiloride are potassium-sparing diuretics.
Triamterene and amiloride directly block the sodium channels in the
epithelium of the collecting tubules.
Spironolactone is a competitive aldosterone antagonist.

Go to the next page if you knew the correct answer, or click the link images
below to further research the concepts in this question (if desired).

Research Concepts:
Potassium Sparing Diuretics:

Tap flag to report any problems with this question.


Question 556: What is the most likely cause of a systolic ejection-type
murmur in a newborn best heard at the upper left sternal border that progresses
to a continuous murmur?

Choices:
1. Ventricular septal defect
2. Atrial septal defect
3. Patent ductus arteriosus
4. Coarctation of the aorta
Answer: 3 - Patent ductus arteriosus
Explanations:
The most likely cause is patent ductus arteriosus
Atrial septal defects generally do not cause murmurs in the newborn
Coarctation of the aorta presents as a continuous and/or late systolic
murmur that is best heard posteriorly over the thoracic spine
Stress management is mobilizing resources to meet or exceed the demands
of events and circumstances.

Go to the next page if you knew the correct answer, or click the link images
below to further research the concepts in this question (if desired).

Research Concepts:
Patent Ductus Arteriosus:

Tap flag to report any problems with this question.


Question 557: The point of maximal impulse in an infant is located?
Choices:
1. Between the third and fourth intercostal space
2. Between the fourth and fifth intercostal space
3. Between the third and second intercostal space
4. Between the first and second intercostal space
Answer: 1 - Between the third and fourth intercostal space
Explanations:
The point of maximum impulse in a young infant can be located between
the third and fourth intercostal space

Go to the next page if you knew the correct answer, or click the link images
below to further research the concepts in this question (if desired).

Research Concepts:
Cardiac Exam:

Tap flag to report any problems with this question.


Question 558: Which is not a sign of intravenous (IV) line infiltration?
Choices:
1. Slowed rate of infusion
2. Warmth
3. Pain
4. Swelling
Answer: 2 - Warmth
Explanations:
Signs of IV line infiltration include coolness, pain, and swelling at the
insertion site.
Warmth would be evidence of superimposed infection/inflammation, which
may occur later as a complication.

Go to the next page if you knew the correct answer, or click the link images
below to further research the concepts in this question (if desired).

Research Concepts:
Intravenous Infiltration:

Tap flag to report any problems with this question.


Question 559: Which statement regarding the diagnosis of aortic dissection
is NOT true?

Choices:
1. Up to 30% of patients may have focal or general neurological signs
2. Acute aortic dissection most commonly presents with an abrupt onset of
"tearing," interscapular or anterior chest pain
3. The incidence of acute myocardial infarction is 1000 times that of aortic
dissection
4. 30% of patients are men under the age of 50
Answer: 4 - 30% of patients are men under the age of 50
Explanations:
20% of patients are males less than 50 years old
Aortic dissection must be considered in the differential diagnosis of acute
chest pain, along with pulmonary embolism, myocardial infarction,
esophageal rupture, and pneumothorax.

Go to the next page if you knew the correct answer, or click the link images
below to further research the concepts in this question (if desired).

Research Concepts:
Dissection, Aortic:

Tap flag to report any problems with this question.


Question 560: Which drug is an antidote for heparin toxicity?
Choices:
1. Vitamin K
2. Coumadin
3. Protamine sulfate
4. Lepirudin
Answer: 3 - Protamine sulfate
Explanations:
Heparin is anticoagulant
Its toxicity can be reversed by protamine sulfate
Lepirudin are a direct thrombin inhibitor, an alternative to heparin

Go to the next page if you knew the correct answer, or click the link images
below to further research the concepts in this question (if desired).

Research Concepts:
Heparin Reversal:

Protamine:

Tap flag to report any problems with this question.


Question 561: On examination of a patient the point of maximal impulse
(PMI) is palpated below the 5th intercostal space. This physical exam finding
suggests:

Choices:
1. Right atrial enlargement
2. Right lung collapse
3. Left ventricular enlargement
4. Left atrial enlargement
Answer: 3 - Left ventricular enlargement
Explanations:
A normal apical impulse is felt over the apex of the heart
This apical impulse is usually located over the left fifth intercostal space in
the midclavicular line
An apical impulse may be dislocated below the fifth intercostal space in the
presence of left ventricular enlargement

Go to the next page if you knew the correct answer, or click the link images
below to further research the concepts in this question (if desired).

Research Concepts:
Left Ventricular Enlargement:

Tap flag to report any problems with this question.


Question 562: Which of the following groups with pulmonary arterial
hypertension (PAH) has a contraindication to calcium channel blocker (CCB)
therapy?

Choices:
1. Right sided heart failure
2. Diabetic
3. Oxygen dependent
4. None of the above
Answer: 1 - Right sided heart failure
Explanations:
Right sided heart failure is a contraindication to CCB therapy in PAH
CCBs are widely used in the treatment of PAH
Vasodilator therapy is used where CCBs are contraindicated, ineffective, or
poorly tolerated
Sildenafil, iloprost, and epoprostenol are some vasodilator agents

Go to the next page if you knew the correct answer, or click the link images
below to further research the concepts in this question (if desired).

Research Concepts:
Pulmonary Hypertension:

Calcium Channel blockers:

Tap flag to report any problems with this question.


Question 563: A 48-year-old male presents to his doctor for a physical
exam. He is obese, smokes and has an elevated sugar level. He is told to start
exercising, control his sugar and quit smoking. The doctor tells him that he is
likely to develop atherosclerosis. What is the most common site for development
atherosclerosis?

Choices:
1. Popliteal artery
2. Abdominal aorta
3. Coronary
4. Iliac artery
Answer: 2 - Abdominal aorta
Explanations:
The most common location of atherosclerosis is the abdominal aorta.
Other sites are the coronaries, popliteal and carotid artery bifurcation.
Location of the pain is determined by the anatomical location of the arterial
lesion.
PVD is most common in the distal superficial femoral artery which
corresponds to claudication in the calf muscle area.

Go to the next page if you knew the correct answer, or click the link images
below to further research the concepts in this question (if desired).

Research Concepts:
Atherosclerosis:

Tap flag to report any problems with this question.


Question 564: Which of the following is the most important intervention in
therapy of a pulmonary embolus (PE)?

Choices:
1. Administration of oxygen
2. Chest x-ray (CXR)
3. Anticoagulation
4. Pain medication
Answer: 3 - Anticoagulation
Explanations:
Anticoagulation to prevent further emboli is the most important
intervention when treating PE
CXR is typically not useful in the definitive diagnosis of PE
Administration of O2 has no effect on the hypoxia associated with PE
Doppler ultrasound of lower extremities can establish diagnosis of DVT,
which is often associated with PE

Go to the next page if you knew the correct answer, or click the link images
below to further research the concepts in this question (if desired).

Research Concepts:
Pulmonary Embolism:

Tap flag to report any problems with this question.


Question 565: What is the initial treatment for a patient with ventricular
fibrillation?

Choices:
1. Intrapericardial epinephrine
2. Synchronized cardioversion
3. Unsynchronized defibrillation
4. Insertion of an AICD
Answer: 3 - Unsynchronized defibrillation
Explanations:
Defibrillation differs in that the shock is not synchronized.
Defibrillation is essential when the rhythm is chaotic and is widely used for
pulse less ventricular tachycardia.
In ventricular fibrillation, patients are generally unconscious and sedation is
not required.

Go to the next page if you knew the correct answer, or click the link images
below to further research the concepts in this question (if desired).

Research Concepts:
Ventricular Fibrillation:

Tap flag to report any problems with this question.


Question 566: What valve area is considered critical mitral valve stenosis?
Choices:
1. 0.5 cm2
2. 1 cm2
3. 2 cm2
4. 4 cm2
Answer: 2 - 1 cm2
Explanations:
The normal mitral valve area is between 4-6 cm2.
When the valve area decreases to about 2 cm2, symptoms start to develop.
It is considered critical when the valve area is l cm2 or less.
With critical mitral stenosis, there is usually a 20 mmHg gradient between
the left atrium and left ventricle.

Go to the next page if you knew the correct answer, or click the link images
below to further research the concepts in this question (if desired).

Research Concepts:
Mitral Stenosis:

Tap flag to report any problems with this question.


Question 567: What should patients allergic to shellfish not receive?
Choices:
1. Iodine skin preparations
2. Iodinated radiocontrast
3. Glucosamine
4. None of the above
Answer: 4 - None of the above
Explanations:
People allergic to shellfish may have been told they are allergic to iodine.
People with a contact reaction to povidone iodine are not allergic to the
iodine but to the provodone. People truly allergic to shellfish can receive
both the topic iodine skin preps and IV iodinated contrast without concern.
Iodine is an essential nutrient and not involved in shellfish allergy. Shellfish
allergy is to a protein.
Glucosamine is commonly manufactured from shellfish shells. There are no
known heightened allergy among those with shellfish allergy.
Shellfish allergy is not a contraindication for administration iodinated
contrast.

Go to the next page if you knew the correct answer, or click the link images
below to further research the concepts in this question (if desired).

Research Concepts:
Allergy, Shellfish:

Tap flag to report any problems with this question.


Question 568: Select the medication that has not been shown to reduce risk
of events in known coronary artery disease.

Choices:
1. Aspirin
2. Statins
3. Beta blockers
4. Calcium channel blockers
Answer: 4 - Calcium channel blockers
Explanations:
Risk reduction for patients with known coronary artery disease includes life
style modification and medications
Treatment of hyperlipidemia with statins is important but some patients
required additional treatment for hypertriglyceridemia
Hypertension should be treated with beta blockers and ACE inhibitors
initially
There is no evidence that calcium channel blockers reduce the risk of
cardiac events

Go to the next page if you knew the correct answer, or click the link images
below to further research the concepts in this question (if desired).

Research Concepts:
Primary And Secondary Prevention Of Coronary Artery
Disease:

Tap flag to report any problems with this question.


Question 569: Exercise has shown to increase:
Choices:
1. HDL
2. LDL
3. VLDL
4. None
Answer: 1 - HDL
Explanations:
Other than medication, exercise also increases HDL

Go to the next page if you knew the correct answer, or click the link images
below to further research the concepts in this question (if desired).

Research Concepts:
High-density Lipoprotein:

Tap flag to report any problems with this question.


Question 570: What are ECG features of right ventricular hypertrophy?
Choices:
1. Right axis deviation
2. QR pattern in leads V1
3. R waves in V1
4. All of the above
Answer: 4 - All of the above
Explanations:
There are many features on an ECG which indicate right ventricular
hypertrophy.
There is right axis deviation, the R/S amplitude ratio of V1 > 1, and
pulmonale pattern
S1Q3T3 and a right bundle branch block are seen if there is a pulmonary
embolus.
Others may have low voltage QRS because of underlying COPD with
hyperinflation.

Go to the next page if you knew the correct answer, or click the link images
below to further research the concepts in this question (if desired).

Research Concepts:
Right Ventricular Hypertrophy:

Tap flag to report any problems with this question.


Question 571: With regard to the posterior descending artery, what does
"left dominant" refer to?

Choices:
1. PDA arising from left main coronary artery
2. PDA arising from left circumflex (LCX) artery
3. PDA arising from right coronary artery (RCA)
4. PDA arising from left anterior descending (LAD) artery
Answer: 2 - PDA arising from left circumflex (LCX) artery
Explanations:
"Left dominant" refers to PDA arising from LCX artery
"Right dominant" refers to PDA arising from RCA
Most often PDA is "right dominant"
LAD and LCX coronary arteries arise at the left main coronary artery
bifurcation

Go to the next page if you knew the correct answer, or click the link images
below to further research the concepts in this question (if desired).

Research Concepts:
Cardiac Dominance:

Tap flag to report any problems with this question.


Question 572: When a patient on ibutilide (Corvert) develops blurred
vision, nausea and vomiting, they may be experiencing:

Choices:
1. Antiarrhythmic toxicity
2. Gastrointestinal side effects
3. Cataracts
4. Needing a higher dose of medication
Answer: 1 - Antiarrhythmic toxicity
Explanations:
These are side effects of antiarrhythmics that can indicate toxic levels of the
drug

Go to the next page if you knew the correct answer, or click the link images
below to further research the concepts in this question (if desired).

Research Concepts:
Ibutilide:

Tap flag to report any problems with this question.


Question 573: What is the best test to confirm cor pulmonale?
Choices:
1. Echo
2. Chest x-ray
3. Chest CT
4. Right heart catheterization
Answer: 4 - Right heart catheterization
Explanations:
Right heart catheterization is invasive but the most accurate test to confirm
cor pulmonale.
Echo can demonstrate signs of right ventricular pressure overload.
Echo may show right ventricular dilatation and the septum bulging into the
left ventricle.

Go to the next page if you knew the correct answer, or click the link images
below to further research the concepts in this question (if desired).

Research Concepts:
Cor Pulmonale:

Tap flag to report any problems with this question.


Question 574: Which is false about the neonatal heart?
Choices:
1. Left and right ventricles are the same size
2. Left and right ventricles have the same wall thickness
3. The resting potential is less negative
4. The cells repolarize slower
Answer: 4 - The cells repolarize slower
Explanations:
The resting potential of cardiac cells in a neonate is less negative and
repolarizes at a faster rate.

Go to the next page if you knew the correct answer, or click the link images
below to further research the concepts in this question (if desired).

Research Concepts:
Neonatal:

Heart Anatomy:

Cardiac Physiology:

Tap flag to report any problems with this question.


Question 575: Select the congenital heart disease not commonly seen in
neonates with Down syndrome.

Choices:
1. Endocardial cushion defect
2. Hypoplastic left heart syndrome
3. Tetralogy of Fallot
4. Atrial septal defect
Answer: 2 - Hypoplastic left heart syndrome
Explanations:
Hypoplastic left heart syndrome is not commonly seen in patients with
Down syndrome
The most common congenital heart malformations seen in Down syndrome
are ASD, VSD, endocardial cushion defect, tetralogy of Fallot, and
persistent PDA

Go to the next page if you knew the correct answer, or click the link images
below to further research the concepts in this question (if desired).

Research Concepts:
Congenital Heart Disease:

Down Syndrome:

Tap flag to report any problems with this question.


Question 576: What is the most common complaint by patients after
general anesthesia?

Choices:
1. Bleeding
2. Sedation
3. Vomiting
4. DVT
Answer: 3 - Vomiting
Explanations:
Nausea and vomiting are the leading reasons for admission post outpatient
surgery.

Go to the next page if you knew the correct answer, or click the link images
below to further research the concepts in this question (if desired).

Research Concepts:
Vomiting:

General Anesthesia:

Tap flag to report any problems with this question.


Question 577: Neuropathy that occurs after cardiopulmonary bypass is
attributed to which of the following?

Choices:
1. Hypertension
2. Stroke
3. Hypoperfusion
4. Silent seizure
Answer: 3 - Hypoperfusion
Explanations:
Neuropathy after bypass is usually due to hypoperfusion.

Go to the next page if you knew the correct answer, or click the link images
below to further research the concepts in this question (if desired).

Research Concepts:
Neuropathy:

Cardiopulmonary Bypass:

Tap flag to report any problems with this question.


Question 578: Which of the following drugs is associated with
hypotension and tachycardia?

Choices:
1. Droperidol
2. Ondansetron
3. Morphine
4. Cisplatin
Answer: 1 - Droperidol
Explanations:
Droperidol is an antiemetic that can cause hypotension and tachycardia.
Other potential side effects include restlessness, anxiety, EPS, dystonia,
seizures, QT prolongation, and sedation
Morphine overdose causes bradycardia and hypotension. Ondansetron is a
potent antiemetic.
Cisplatin causes severe nausea and vomiting.

Go to the next page if you knew the correct answer, or click the link images
below to further research the concepts in this question (if desired).

Research Concepts:
Droperidol:

Tap flag to report any problems with this question.


Question 579: Which arrhythmia is NOT treated with cardioversion?
Choices:
1. Complete heart block
2. Ventricular tachycardia
3. Ventricular fibrillation
4. Supraventricular tachycardia
Answer: 1 - Complete heart block
Explanations:
Pacemaker is indicated for complete heart block.

Go to the next page if you knew the correct answer, or click the link images
below to further research the concepts in this question (if desired).

Research Concepts:
Defibrillation And Cardioversion:

Heart Block, Third Degree:

Arrhythmias:

Tap flag to report any problems with this question.


Question 580: In an ascending aortic dissection, where is the false lumen
generally located?

Choices:
1. Left posterior half of the aorta
2. Right anterior half of the aorta
3. Left anterior half of the aorta
4. Right posterior half of the aorta
Answer: 2 - Right anterior half of the aorta
Explanations:
In the ascending aorta, the false lumen is located in the outer third of the
media and usually involves the right anterior part of the aorta.
The medial third of the ascending aorta is usually normal.

Go to the next page if you knew the correct answer, or click the link images
below to further research the concepts in this question (if desired).

Research Concepts:
Dissection, Aortic:

Tap flag to report any problems with this question.


Question 581: Which of the following is FALSE about blood groups?
Choices:
1. ABO compatibility is a must for renal transplant
2. Rh antigens must be matched prior to transfusions
3. Febrile reactions may be due to bacterial contamination
4. Citrate toxicity can cause hypocalcemia
Answer: 2 - Rh antigens must be matched prior to transfusions
Explanations:
There must be compatibility testing between donor and recipient blood.
Typing determines the ABO and Rh status. ABO must be matched but Rh
antigen usually not an issue.
Rh sensitization only occurs during pregnancy or prior transfusion.
Citrate is used as preservative for red blood cells and can cause
hypocalcemia with large transfusions.

Go to the next page if you knew the correct answer, or click the link images
below to further research the concepts in this question (if desired).

Research Concepts:
Blood Transfusion:

Tap flag to report any problems with this question.


Question 582: Which of the following heart rhythms would need
immediate defibrillation?

Choices:
1. Ventricular tachycardia
2. Atrial tachycardia
3. Ventricular fibrillation
4. Atrial fibrillation
Answer: 3 - Ventricular fibrillation
Explanations:
Ventricular fibrillation requires immediate defibrillation
Ventricular tachycardia may require cardioversion
Atrial fibrillation may also require cardioversion

Go to the next page if you knew the correct answer, or click the link images
below to further research the concepts in this question (if desired).

Research Concepts:
Atrial Fibrillation:

Tap flag to report any problems with this question.


Question 583: Which of the following is NOT a complication of
myocardial infarction?

Choices:
1. Cor pulmonale
2. CNS emboli
3. Dissecting aortic aneurysm
4. Arrhythmia
Answer: 3 - Dissecting aortic aneurysm
Explanations:
There are many complications of myocardial infarction, and they typically
occur in the first 2-4 days.
Conduction defects are the most common complication, and are observed in
patients that have an inferior wall MI.
Cardiogenic shock occurs when at least 40% of the heart muscle becomes
necrotic. Cardiogenic shock carries a 90% mortality rate.
Other complications include CHF and embolism. Aortic dissection is not a
complication of MI. It is a complication of hypertension and connective
tissue disorders.

Go to the next page if you knew the correct answer, or click the link images
below to further research the concepts in this question (if desired).

Research Concepts:
Myocardial Infarction:

Tap flag to report any problems with this question.


Question 584: A 59-year-old diabetic develops a myocardial infarction. He
also has a history of hypertension and premature ventricular contractions. What
is the drug of choice for this patient?

Choices:
1. Lidocaine
2. Metoprolol
3. Amiodarone
4. Clonidine
Answer: 2 - Metoprolol
Explanations:
Metoprolol is a beta-blocker that is often used to treat people with
hypertension.
Metoprolol is often used to treat people post-myocardial infarction.
Metoprolol is also helpful for the treatment of premature ventricular
contractions.

Go to the next page if you knew the correct answer, or click the link images
below to further research the concepts in this question (if desired).

Research Concepts:
Premature Ventricular Contraction:

Myocardial Infarction, Acute:

Tap flag to report any problems with this question.


Question 585: Which of the following best determines the tone or systemic
vascular resistance?

Choices:
1. Ventricular wall thickness
2. Ability of the small arteries and arterioles to constrict
3. Resistance to blood flow in the arterioles
4. The difference in pulse pressure
Answer: 2 - Ability of the small arteries and arterioles to constrict
Explanations:
Systemic vascular resistance = (mean arterial pressure - central venous
pressure) x 80 / cardiac output.
Systemic vascular resistance is a function of small arteriolar, or resistance
arteriolar, tone.

Go to the next page if you knew the correct answer, or click the link images
below to further research the concepts in this question (if desired).

Research Concepts:
Cardiac Physiology:

Tap flag to report any problems with this question.


Question 586: Which is a distinctive feature of a cardioembolic stroke?
Choices:
1. Onset of seizures right away
2. Presence of atrial fibrillation at onset of stroke
3. Decreased level of consciousness at onset of stroke
4. Usually strokes are permanent
Answer: 3 - Decreased level of consciousness at onset of stroke
Explanations:
Several features distinguish cardioembolic stroke from other causes of
stroke.
One of them is reduced level of consciousness at onset of stroke.
Other features of cardioembolic stroke include rapid onset with maximal
severity at onset.
Cardioembolic strokes also tend to recover quickly within a few hours due
to reperfusion.

Go to the next page if you knew the correct answer, or click the link images
below to further research the concepts in this question (if desired).

Research Concepts:
Cardioembolic Stroke:

Tap flag to report any problems with this question.


Question 587: Which of the following is not a complication of right
internal jugular puncture?

Choices:
1. Arrhythmias
2. Hemomediastinum
3. Thoracic duct injury
4. Tracheal puncture
Answer: 3 - Thoracic duct injury
Explanations:
Numerous complications can result from central line placement, so benefit
must be greater than the risk.
Arrhythmias can occur if the guide wire contacts the endocardium.
Hemomediastinum and pneumothorax can be minimized by using
ultrasound guidance.
The risk of these is higher with catheterization of the subclavian vein than
with right jugular lines.
Injury to the thoracic duct is not seen with right-sided procedures.

Go to the next page if you knew the correct answer, or click the link images
below to further research the concepts in this question (if desired).

Research Concepts:
Central Venous Access, Internal Jugular Vein, Anterior:

Tap flag to report any problems with this question.


Question 588: Which is an unlikely finding in tension pneumothorax?
Choices:
1. Decreased breath sounds
2. Hyperresonance
3. Respiratory distress
4. Hypertension
Answer: 4 - Hypertension
Explanations:
Tension pneumothorax is not characterized by hypertension

Go to the next page if you knew the correct answer, or click the link images
below to further research the concepts in this question (if desired).

Research Concepts:
Tension Pneumothorax:

Tap flag to report any problems with this question.


Question 589: Which of the following is increased in iron deficiency
anemia?

Choices:
1. Mean corpuscular volume (MCV)
2. Total iron binding capacity (TIBC)
3. Ferritin
4. Haptoglobin
Answer: 2 - Total iron binding capacity (TIBC)
Explanations:
Iron deficiency anemia is a microcytic hypochromic anemia
Ferritin and serum iron will be decreased while TIBC will be elevated
Ferritin is the most reliable indicator of iron stores short of bone marrow
aspiration
Haptoglobin is decreased with hemolytic anemias

Go to the next page if you knew the correct answer, or click the link images
below to further research the concepts in this question (if desired).

Research Concepts:
Iron Deficiency Anemia:

Tap flag to report any problems with this question.


Question 590: What additional medication should be initiated in a 66 year
old male recently hospitalized for dyspnea and diagnosed with congestive heart
failure who is on aspirin and atorvastatin?

Choices:
1. Digoxin
2. ACE inhibitor
3. Thiazide
4. Nebulizer
Answer: 2 - ACE inhibitor
Explanations:
ACE inhibitors are important in management of congestive heart failure as
they decrease afterload and have been shown to increase survival.
Diuretics may be needed if fluid accumulation becomes a problem.
Spironolactone is indicated in advanced heart failure.

Go to the next page if you knew the correct answer, or click the link images
below to further research the concepts in this question (if desired).

Research Concepts:
Heart Failure, Congestive:

Tap flag to report any problems with this question.


Question 591: Which of the following diseases is suggested by positive
antihistone antibodies?

Choices:
1. CREST syndrome
2. Drug-induced lupus
3. Sjogren's syndrome
4. Systemic lupus erythematosus
Answer: 2 - Drug-induced lupus
Explanations:
Autoantibodies to histones alone suggest drug-induced lupus.
It develops in over half of patients on procainamide.
It also occurs with isoniazid and hydralazine.
Drug induced lupus is usually milder than systemic lupus erythematosus.

Go to the next page if you knew the correct answer, or click the link images
below to further research the concepts in this question (if desired).

Research Concepts:
Lupus Erythematosus, Drug-Induced:

Tap flag to report any problems with this question.


Question 592: What is the next step in myocardial ischemia that is
refractory to medical therapy?

Choices:
1. Observation
2. Continue medical treatment
3. Stress test
4. Angioplasty
Answer: 4 - Angioplasty
Explanations:
Angioplasty is indicated with medical refractory ischemia

Go to the next page if you knew the correct answer, or click the link images
below to further research the concepts in this question (if desired).

Research Concepts:
Revascularization Procedures In Coronary Artery
Disease:

Tap flag to report any problems with this question.


Question 593: Which complication is least likely with amiodarone?
Choices:
1. Hypothyroidism
2. Hepatic toxicity
3. Pulmonary toxicity
4. Bone marrow suppression
Answer: 4 - Bone marrow suppression
Explanations:
Amiodarone can cause serious complications
Thyroid abnormalities are common with long term amiodarone use
Long term amiodarone use can lead to lung fibrosis
Hepatic toxicity is also seen with amiodarone use, Amiodarone is not
associated with bone marrow suppression

Go to the next page if you knew the correct answer, or click the link images
below to further research the concepts in this question (if desired).

Research Concepts:
Amiodarone:

Tap flag to report any problems with this question.


Question 594: A 65 year old with sudden onset of chest pain, tachypnea,
cyanosis and low BP may have:

Choices:
1. Chronic obstructive pulmonary disease
2. Exercise-induced asthma
3. Subcutaneous emphysema
4. Pulmonary embolism
Answer: 4 - Pulmonary embolism
Explanations:
Sign and symptoms of pulmonary embolism include sudden onset of
dyspnea, chest pain, tachypnea, cyanosis, low blood pressure and wheezing.

Go to the next page if you knew the correct answer, or click the link images
below to further research the concepts in this question (if desired).

Research Concepts:
Pulmonary Embolism:

Tap flag to report any problems with this question.


Question 595: A 65-year-old male is seen because of fatigue. Auscultation
reveals dullness to percussion, increased egophony and bronchial breath sounds
which are best heard at the tip of the left scapula. This suggests that he may
have?

Choices:
1. Pneumothorax
2. Asthma
3. Pericardial effusion
4. Emphysema
Answer: 3 - Pericardial effusion
Explanations:
Ewart sign is presence of physical findings in patients with fluid collection
around the heart
Dullness to percussion, bronchial breath sounds, and egophony may be best
heard at the tip of left scapula
These finding indicate a large pericardial effusion

Go to the next page if you knew the correct answer, or click the link images
below to further research the concepts in this question (if desired).

Research Concepts:
Pericardial Effusion:

Tap flag to report any problems with this question.


Question 596: Signs and symptoms of delirium tremens can safely be
treated with:

Choices:
1. Meperidine (Demerol)
2. Metaproterenol (Alupent)
3. Midazolam (Versed)
4. Lorazepam (Ativan)
Answer: 4 - Lorazepam (Ativan)
Explanations:
Long acting benzodiazepines are best in treating DT's, especially patients
with coinciding liver disease

Go to the next page if you knew the correct answer, or click the link images
below to further research the concepts in this question (if desired).

Research Concepts:
Delirium Tremens:

Tap flag to report any problems with this question.


Question 597: A 72 year old female is dying of colon cancer and the
family is at the bedside. One of them asks how much longer the patient will live.
Select the best response.

Choices:
1. "Things will be over soon, don't worry."
2. "This must be very hard for you."
3. "We can't know for sure. Do you have any worries that she is in pain?"
4. "We can never tell."
Answer: 3 - "We can't know for sure. Do you have any worries that she is in
pain?"

Explanations:
The important consideration is why the family member is asking the
question at this point
If there are concerns the patient is uncomfortable or in pain these should be
addressed
Telling the family not to worry is poor advice
It expresses sympathy to recognize that watching a family member die but
does not address their concerns

Go to the next page if you knew the correct answer, or click the link images
below to further research the concepts in this question (if desired).

Research Concepts:
Colon Cancer:

Hospice Care:

Tap flag to report any problems with this question.


Question 598: Which of following is associated with dobutamine?
Choices:
1. Racemic mixture
2. Beta 2 agonist
3. Increases cardiac contractility
4. All of the above
Answer: 4 - All of the above
Explanations:
Dobutamine is a racemic mixture of the beta one agonist.

Go to the next page if you knew the correct answer, or click the link images
below to further research the concepts in this question (if desired).

Research Concepts:
Dobutamine:

Tap flag to report any problems with this question.


Question 599: A medication that inhibits the activity of HMG-CoA
reductase would be prescribed for which condition?

Choices:
1. Hyperuricemia
2. Diabetes mellitus
3. Hyperlipidemia
4. Hypertension
Answer: 3 - Hyperlipidemia
Explanations:
HMG-CoA reductase inhibitors are termed statins and are used to treat
hypercholesterolemia.
The statins available for use in the United States are simvastatin (Zocor),
pravastatin (Pravachol), lovastatin (Mevacor), atorvastatin (Lipitor),
fluvastatin (Lescol), and rosuvastatin (Crestor).

Go to the next page if you knew the correct answer, or click the link images
below to further research the concepts in this question (if desired).

Research Concepts:
Statin Medication:

Tap flag to report any problems with this question.


Question 600: Which condition is the most common cause of multiple
brain abscesses in children?

Choices:
1. Trauma
2. Chronic steroid use
3. Cyanotic heart disease
4. Diabetes
Answer: 3 - Cyanotic heart disease
Explanations:
Cyanotic heart disease is a common cause of multiple brain abscesses in
children.

Go to the next page if you knew the correct answer, or click the link images
below to further research the concepts in this question (if desired).

Research Concepts:
Cyanotic Heart Disease:

Brain Abscess:

Tap flag to report any problems with this question.


Section 4
Question 601: What is the drug of choice for the treatment of anaphylaxis?
Choices:
1. Corticosteroids
2. Antihistamines
3. Epinephrine
4. Bronchodilators
Answer: 3 - Epinephrine
Explanations:
Epinephrine is the drug of choice for the treatment of anaphylaxis
Epinephrine should be injected intramuscularly into the thigh
Antihistamines, corticosteroids and bronchodilators are all adjuncts to
epinephrine

Go to the next page if you knew the correct answer, or click the link images
below to further research the concepts in this question (if desired).

Research Concepts:
Anaphylaxis:

Tap flag to report any problems with this question.


Question 602: The pathologist reports that Aschoff nodules were found.
What was the most likely organ examined?

Choices:
1. Lungs
2. Skin
3. Heart
4. Brain
Answer: 3 - Heart
Explanations:
Aschoff nodules are seen in hearts of patients with rheumatic fever
The nodules result from inflammation and consist of fibrinoid reaction and
inflammatory cells.
Multinucleated giant cells may be seen

Go to the next page if you knew the correct answer, or click the link images
below to further research the concepts in this question (if desired).

Research Concepts:
Rheumatic Fever, Pathology:

Tap flag to report any problems with this question.


Question 603: Which of the following is a cause of high output cardiac
failure?

Choices:
1. Hypothyroidism
2. Hemochromatosis
3. Pregnancy
4. Essential hypertension
Answer: 3 - Pregnancy
Explanations:
Other causes of high output failure include hyperthyroidism, anemia, and
Paget disease

Go to the next page if you knew the correct answer, or click the link images
below to further research the concepts in this question (if desired).

Research Concepts:
High Output Cardiac Failure:

Tap flag to report any problems with this question.


Question 604: The left atrium and ventricle receive blood from which of
the following arteries?

Choices:
1. Left anterior descending
2. Posterior descending
3. Circumflex artery
4. Marginal artery
Answer: 3 - Circumflex artery
Explanations:
The circumflex coronary artery is a branch of the left main coronary artery
It runs in the left coronary sulcus, a groove that separates the left atrium and
left ventricle
It supplies the left atrium and the left ventricle

Go to the next page if you knew the correct answer, or click the link images
below to further research the concepts in this question (if desired).

Research Concepts:
Heart Anatomy:

Tap flag to report any problems with this question.


Question 605: Which of the following does NOT increase the infection
rates in a clean operative procedure?

Choices:
1. Preoperative antibiotics
2. Long surgery
3. Complicated surgery
4. Use of steroids
Answer: 1 - Preoperative antibiotics
Explanations:
Preoperative antibiotics do not increase the infection rate. All the other
listed factors do.
Systemic factors that increase risk of infection include malnutrition, poor
tissue perfusion, obesity, diabetes, use of steroids, and immunosuppressants
Wound characteristics that increase infection include hematoma, foreign
material, dead space, poor skin preparation, shaving, and use of drains.
Surgery factors which increase infection include poor surgery technique,
long surgery, intraoperative contamination, and inadequate theatre
ventilation.

Go to the next page if you knew the correct answer, or click the link images
below to further research the concepts in this question (if desired).

Research Concepts:
Postoperative Infection:

Tap flag to report any problems with this question.


Question 606: Paroxysmal atrial fibrillation (AF) is defined as lasting:
Choices:
1. <7 days
2. >3 months
3. >6 months
4. >1 year
Answer: 1 - <7 days
Explanations:
During AF, the atria contract chaotically and irregularly when compared to
the ventricles
Paroxysmal AF is defined as lasting <7 days
Most episodes of AF last <24 hours
Persistent AF is defined as lasting >7 days

Go to the next page if you knew the correct answer, or click the link images
below to further research the concepts in this question (if desired).

Research Concepts:
Paroxysmal Atrial Fibrillation:

Tap flag to report any problems with this question.


Question 607: How is fentanyl overdose usually treated?
Choices:
1. Flumazenil
2. Midazolam
3. Naloxone
4. Protamine
Answer: 3 - Naloxone
Explanations:
Naloxone is a short acting opioid antagonist

Go to the next page if you knew the correct answer, or click the link images
below to further research the concepts in this question (if desired).

Research Concepts:
Opiate Overdose:

Naloxone:

Tap flag to report any problems with this question.


Question 608: Which of the following medications does not reduce the risk
of stroke in predisposed patients?

Choices:
1. Ticlopidine
2. Warfarin
3. Aspirin
4. Pentoxifylline
Answer: 4 - Pentoxifylline
Explanations:
Daily aspirin therapy is known to reduce the risk of stroke in predisposed
patients.
Patients who have transient ischemic attack are advised to take daily aspirin
to reduce future strokes.
In cases where aspirin is not tolerated ,clopidogrel or ticlopidine are a good
alternatives.
Pentoxifylline does not prevent strokes.

Go to the next page if you knew the correct answer, or click the link images
below to further research the concepts in this question (if desired).

Research Concepts:
Stroke Prevention:

Tap flag to report any problems with this question.


Question 609: Hospital acquired pneumonia:
Choices:
1. Is associated with an increase in hospital stay by one month
2. Is associated with a 1%-3% daily increase risk of infection in patients on
mechanical ventilation
3. Is the most common nosocomial infection
4. In the immunocompromised host, is most often associated with Respiratory
syncytial virus.
Answer: 2 - Is associated with a 1%-3% daily increase risk of infection in
patients on mechanical ventilation

Explanations:
Hospital acquired pneumonia is associated with an average increased
hospital stay of 7-9 days.
Mechanical ventilation is associated with a 1%-3% daily increase of
infection and is often due aspiration.
Urinary tract infections are the most common nosocomial infection; HAP is
the third most common.
In the USA, the most frequent type of hospital acquired infections are
urinary tract infection (36%), followed by surgical site infection (20%), and
bloodstream infection and pneumonia (both 11%).

Go to the next page if you knew the correct answer, or click the link images
below to further research the concepts in this question (if desired).

Research Concepts:
Hospital Acquired Pneumonia:

Tap flag to report any problems with this question.


Question 610: What risk factor is most strongly associated with an
abdominal aortic aneurysm (AAA)?

Choices:
1. Family history
2. Sedentary lifestyle
3. Smoking
4. Hypertension
Answer: 3 - Smoking
Explanations:
The risk factor with the strongest association with an abdominal aortic
aneurysm is smoking.
Other risk factors include family history, hypertension, coronary artery
disease, previous aneurysm repair or peripheral aneurysm
Sedentary lifestyle is not a risk factor for AAA

Go to the next page if you knew the correct answer, or click the link images
below to further research the concepts in this question (if desired).

Research Concepts:
Abdominal Aortic Aneurysm:

Tap flag to report any problems with this question.


Question 611: A typical boot shaped heart is seen in:
Choices:
1. Tricuspid atresia
2. Tetralogy of Fallot (TOF)
3. Transposition
4. Total anomalous pulmonary venous return (TAPVR)
Answer: 2 - Tetralogy of Fallot (TOF)
Explanations:
The chest x-ray in TOF may give the cardiac silhouette resembling a Dutch
clog (Coeur en sabot) often referred to as a "boot shaped" heart
The x-ray silhouette reflects RV hypertrophy and a concave upper left heart
border due to a small or absent main PA

Go to the next page if you knew the correct answer, or click the link images
below to further research the concepts in this question (if desired).

Research Concepts:
Tetralogy Of Fallot:

Tap flag to report any problems with this question.


Question 612: Streptococcus is a primary cause of:
Choices:
1. Endocarditis
2. Thrush
3. Influenza
4. Infectious mononucleosis
Answer: 1 - Endocarditis
Explanations:
Streptococcus is a primary cause of native valve endocarditis

Go to the next page if you knew the correct answer, or click the link images
below to further research the concepts in this question (if desired).

Research Concepts:
Streptococcus:

Endocarditis:

Tap flag to report any problems with this question.


Question 613: What is a normal pulse pressure?
Choices:
1. 20 mmHg
2. 40 mmHg
3. 60 mmHg
4. 80 mmHg
Answer: 2 - 40 mmHg
Explanations:
A normal pulse pressure is 40 mmHg
Calculated by the difference of systolic to diastolic blood pressure

Go to the next page if you knew the correct answer, or click the link images
below to further research the concepts in this question (if desired).

Research Concepts:
Pulse Pressure:

Tap flag to report any problems with this question.


Question 614: Which of the following would be expected in a patient who
suffers a myocardial infarction?

Choices:
1. Liquefactive necrosis
2. Coagulative necrosis
3. Fibrinoid necrosis
4. Suppurative collection
Answer: 2 - Coagulative necrosis
Explanations:
Coagulative necrosis is caused by hypoxic conditions, such as occur in
coronary artery disease.
The hypoxia leads to localized injury to the area supplied by the blood
vessel.

Go to the next page if you knew the correct answer, or click the link images
below to further research the concepts in this question (if desired).

Research Concepts:
Myocardial Infarction, Acute:

Coagulation Necrosis:

Tap flag to report any problems with this question.


Question 615: Which of the following is false about hypernatremia?
Choices:
1. Hypernatremia may indicate a deficit of total body water.
2. Hypernatremia may be caused by high protein intake.
3. Hypernatremia can be caused by the replacement of lost water with
hypertonic saline.
4. Hypernatremia can be caused by hyperglycemia.
Answer: 4 - Hypernatremia can be caused by hyperglycemia.
Explanations:
Hypernatremia is associated with a high mortality rate when sodium levels
rise above 150 mEq/l.
Hypernatremia may occur in infants and psychiatric patients because of
problems associated with renal concentrating mechanisms.
Hypernatremia is either too little water or too much salt. The most common
cause is lack of free water.
Other causes of hypernatremia include renal losses, sodium bicarbonate
administration, mineral corticoid excess, and hyperventilation.

Go to the next page if you knew the correct answer, or click the link images
below to further research the concepts in this question (if desired).

Research Concepts:
Hypernatremia:

Tap flag to report any problems with this question.


Question 616: Which of the following electrolytes can be used in the
management of digitalis-induced arrhythmias?

Choices:
1. Calcium
2. Magnesium
3. Phosphorous
4. Sodium
Answer: 2 - Magnesium
Explanations:
Magnesium sulfate can be used for digitalis toxicity.
Fab fragments of digitalis inactivate digitalis in the serum.
Potassium is used to correct hypokalemia associated with digitalis induced
arrhythmias

Go to the next page if you knew the correct answer, or click the link images
below to further research the concepts in this question (if desired).

Research Concepts:
Digoxin Toxicity:

Tap flag to report any problems with this question.


Question 617: What should be the first intervention if a patient loses
consciousness and is diagnosed with pulseless ventricular tachycardia?

Choices:
1. Defibrillation with 200J
2. Epinephrine 1 mg IV
3. Amiodarone 300 mg IV
4. Atropine 1 mg IV
Answer: 1 - Defibrillation with 200J
Explanations:
Ventricular fibrillation or pulseless ventricular tachycardia should be treated
with defibrillation starting at 200J then increasing to 300J and then 360J if a
monophasic device is used
Epinephrine is also a treatment but first line is immediate defibrillation

Go to the next page if you knew the correct answer, or click the link images
below to further research the concepts in this question (if desired).

Research Concepts:
Defibrillation And Cardioversion:

Ventricular Tachycardia:

Tap flag to report any problems with this question.


Question 618: An accurate statement about the diagnosis of "shock" is:
Choices:
1. The body cannot metabolize glucose
2. The body is in a state of multi-system organ failure
3. The body cannot perfuse tissues properly
4. The body is resistant to antibiotics
Answer: 3 - The body cannot perfuse tissues properly
Explanations:
Perfusion of tissues is impaired in all types of shock

Go to the next page if you knew the correct answer, or click the link images
below to further research the concepts in this question (if desired).

Research Concepts:
Shock:

Tap flag to report any problems with this question.


Question 619: Which of the following is not true of aortic regurgitation?
Choices:
1. The first heart sound is normal
2. A high pitched diastolic murmur is often heard
3. The murmur is loudest in 3rd or 4th left intercostal space
4. The murmur is best heard with patient supine
Answer: 4 - The murmur is best heard with patient supine
Explanations:
Auscultatory findings of aortic regurgitation (AR) include a normal first
heart sound with a loud, non-split, sharp second heart sound. This is
secondary to increased elastic recoil of the aorta.
The murmur is high pitched, diastolic, and blowing. It starts soon after the
second heart sound and is best heard at the left third or fourth intercostal
space.
It is loudest if the patient is leaning forward and holds his or her breath after
expiration.
It increases with squatting or isometric handgrips that increases afterload.

Go to the next page if you knew the correct answer, or click the link images
below to further research the concepts in this question (if desired).

Research Concepts:
Aortic Regurgitation:

Tap flag to report any problems with this question.


Question 620: A 66-year-old patient completes a nuclear exercise stress
test with the following results: failure to achieve stage II of the Bruce protocol
with ST-segment depression of 1.0 mm in one lead, lasting 1 minute into
recovery. Which result has the worst prognostic sign for this patient?

Choices:
1. Failure to achieve stage II of the Bruce protocol
2. ST-segment depression of 1.0 mm
3. ST depression in 1 lead
4. ST depression lasting 1 minute into recovery
Answer: 1 - Failure to achieve stage II of the Bruce protocol
Explanations:
Failure to achieve stage II of the Bruce protocol is a poor prognostic sign.
ST depression more than 2.0 mm, ST depression in more than one lead and
ST depression lasting more than 6 minutes into recovery are all poor
prognostic signs.
Stress testing does not detect significant disease in some cases.

Go to the next page if you knew the correct answer, or click the link images
below to further research the concepts in this question (if desired).

Research Concepts:
Nuclear Medicine Stress Test:

Tap flag to report any problems with this question.


Question 621: Which of the following drugs is most likely to induce
thrombocytopenia?

Choices:
1. Warfarin
2. Aspirin
3. Heparin
4. Hydralazine
Answer: 3 - Heparin
Explanations:
Heparin can induce thrombocytopenia
Lepirudin and argatroban are direct thrombin inhibitors used to treat HIT
It is important to discontinue heparin and heparin flushes.

Go to the next page if you knew the correct answer, or click the link images
below to further research the concepts in this question (if desired).

Research Concepts:
Thrombocytopenia, Heparin Induced:

Thrombocytopenia:

Tap flag to report any problems with this question.


Question 622: Which of the following medications will prolong the
bleeding time?

Choices:
1. Aspirin
2. Warfarin
3. Heparin
4. Alteplase
Answer: 1 - Aspirin
Explanations:
Bleeding time is a function of platelet number and function.
From the above choices, only aspirin can compromise the function of
platelets.
Inhibition of thromboxane-2 by aspirin prevents effective aggregation.

Go to the next page if you knew the correct answer, or click the link images
below to further research the concepts in this question (if desired).

Research Concepts:
Aspirin:

Bleeding Time:

Tap flag to report any problems with this question.


Question 623: A 66-year-old male presents with a fainting episode. His
blood pressure is 83/55 mmHg. ECG reveals tachycardia with absent P waves.
What is the most appropriate next step in the management of this patient?

Choices:
1. Digoxin infusion
2. Heparin anticoagulation
3. Synchronized cardioversion
4. Lidocaine
Answer: 3 - Synchronized cardioversion
Explanations:
The patient most likely has atrial fibrillation with severe hypotension.
In the presence of hemodynamic instability, the best management of atrial
fibrillation is synchronized cardioversion.
Anticoagulation can be started after electric defibrillation.
If it is a chronic atrial fibrillation in a stable patient, anticoagulation should
be started first.

Go to the next page if you knew the correct answer, or click the link images
below to further research the concepts in this question (if desired).

Research Concepts:
Hypotension:

Atrial Fibrillation:

Tap flag to report any problems with this question.


Question 624: Select the correct statement about duplex scanning for
thrombosis of the inferior vena cava (IVC).

Choices:
1. The proximity of the IVC to the abdominal wall posteriorly makes the
procedure technically easy
2. Respiratory variation and compressibility of the IVC are not as useful as in
duplex scanning of the femoral system
3. The large caliber of the IVC makes duplex scanning here more reliable than
at the femoral level
4. Only the IVC and iliac systems are visualized with duplex in the abdomen
Answer: 2 - Respiratory variation and compressibility of the IVC are not as
useful as in duplex scanning of the femoral system

Explanations:
The cardiac cycle has more effect than respiratory variation in the IVC
above the renal veins
The IVC is not compressible as the femoral system is
Visualization of dilated collaterals may be of value in diagnosis of IVC
thrombosis
The disadvantages of duplex in this situation include anatomic limitations
and operator dependence

Go to the next page if you knew the correct answer, or click the link images
below to further research the concepts in this question (if desired).

Research Concepts:
Inferior Vena Caval Thrombosis:

Tap flag to report any problems with this question.


Question 625: A 72 year old female is recovering after angioplasty with
stent for an inferior myocardial infarction. She develops orthopnea and shortness
of breath. Vital signs are stable. There is jugular venous distension, a new III/VI
systolic murmur at the heart base. It has a crescendo-decrescendo musical
quality. There is no change with respiration. There are crackles at the lung bases.
Select the most probable finding on echocardiogram.

Choices:
1. Anterior mitral leaflet fluttering
2. A mitral regurgitant jet that is eccentric
3. Changes with respirations of the velocity of flow across the mitral valve
4. Ventricular septal defect
Answer: 2 - A mitral regurgitant jet that is eccentric
Explanations:
The patient most likely has chordae rupture of the posterior leaflet mitral
valve
This leaflet is more commonly affected because of its singular blood supply
A flail leaflet produces the musical quality of the murmur
Acute septal perforation causes hypotension and cardiogenic shock

Go to the next page if you knew the correct answer, or click the link images
below to further research the concepts in this question (if desired).

Research Concepts:
Inferior Wall Myocardial Infarction:

Mitral Regurgitation:

Tap flag to report any problems with this question.


Question 626: What is the most common valvular disease associated with
sudden death in young athletes?

Choices:
1. Idiopathic hypertrophic subaortic stenosis
2. Pulmonic valve stenosis
3. Mitral valve stenosis
4. Tricuspid valve stenosis
Answer: 1 - Idiopathic hypertrophic subaortic stenosis
Explanations:
Idiopathic hypertrophic subaortic stenosis is often asymmetric and is a
common cause of sudden death in young athletes

Go to the next page if you knew the correct answer, or click the link images
below to further research the concepts in this question (if desired).

Research Concepts:
Hypertrophic Obstructive Cardiomyopathy:

Tap flag to report any problems with this question.


Question 627: Which of the following electrolyte disturbances is not seen
with the drug spironolactone?

Choices:
1. Hyponatremia
2. Hypokalemia
3. Hypomagnesemia
4. Hypocalcemia
Answer: 2 - Hypokalemia
Explanations:
Spironolactone is a potassium sparing diuretic
Hyponatremia and hypocalcemia can occur with excess use of diuretics

Go to the next page if you knew the correct answer, or click the link images
below to further research the concepts in this question (if desired).

Research Concepts:
Spironolactone:

Tap flag to report any problems with this question.


Question 628: The weight loss medication combination phentermine-
fenfluramine has been associated with which of the following adverse effects?

Choices:
1. Dilated cardiomyopathy
2. Mitral regurgitation
3. Supraventricular tachycardia
4. Malignant hypertension
Answer: 2 - Mitral regurgitation
Explanations:
The appetite suppressant combination, known as Phen-Fen, was banned
because of its propensity to produce inflammatory mitral valve changes
resulting in severe regurgitation and pulmonary hypertension

Go to the next page if you knew the correct answer, or click the link images
below to further research the concepts in this question (if desired).

Research Concepts:
Phentermine:

Fenfluramine:

Tap flag to report any problems with this question.


Question 629: In the operating room, which of the following is the best
method for controlling a bleeding vessel?

Choices:
1. Ligation of the vessel
2. Extra-anatomic bypass
3. Use of an intraluminal shunt
4. Embolization with fibrin glue
Answer: 3 - Use of an intraluminal shunt
Explanations:
There are many types and sizes of intraluminal shunts available today.
While not widely used, shunts can be very helpful in managing troublesome
bleeding. These shunts can even be left in place for several days.
Packing may help tamponade the bleeding but with active vessel bleeding
more aggressive maneuvers are needed.
Ligation alone always risks the possibility of developing ischemia.

Go to the next page if you knew the correct answer, or click the link images
below to further research the concepts in this question (if desired).

Research Concepts:
Intraluminal Shunt:

Tap flag to report any problems with this question.


Question 630: Which of the following medication can cause epinephrine
reversal?

Choices:
1. Carvedilol
2. Terazosin
3. Dobutamine
4. Phentolamine
Answer: 2 - Terazosin
Explanations:
Epinephrine has a stronger action on alpha receptors than beta receptors
Blockage of alpha effect by alpha-blockers leads to unopposed stimulation
of beta receptors
Stimulation of beta receptors by epinephrine can result in vasodilatation and
hypotension
A decrease in blood pressure when epinephrine is administered with alpha
blockers is often called epinephrine reversal

Go to the next page if you knew the correct answer, or click the link images
below to further research the concepts in this question (if desired).

Research Concepts:
Epinephrine:

Alpha-Blockers:

Tap flag to report any problems with this question.


Question 631: Which of the following statements is TRUE regarding
exercise testing?

Choices:
1. In patients with 2 or more vessel disease, exercise testing is contraindicated
2. It should not be done within 5 days of a myocardial infarction
3. Relative contraindications for exercise stress testing can be superseded if
the benefits of exercise testing outweigh the risks
4. Exercise testing in a patient with an ejection fraction less than 25% is
contraindicated
Answer: 3 - Relative contraindications for exercise stress testing can be
superseded if the benefits of exercise testing outweigh the risks

Explanations:
Absolute contraindications for exercise testing include MI in the previous
48 hours, uncontrolled arrhythmias resulting in symptoms or hemodynamic
compromise, severe symptomatic stenosis, uncontrolled angina or failure,
acute PE, and acute aortic dissection.
Relative contraindications include left main stenosis, electrolyte imbalance,
severe hypertension, hypertrophic cardiomyopathy, and high degree
atrioventricular block.
The American Heart Association and the American College of
Cardiologists state that "Relative contraindications for exercise stress
testing can be superseded if the benefits of exercise testing outweigh the
risks".

Go to the next page if you knew the correct answer, or click the link images
below to further research the concepts in this question (if desired).

Research Concepts:
Treadmill Stress Testing:

Tap flag to report any problems with this question.


Question 632: The gold standard for making a diagnosis of pulmonary
embolism is:

Choices:
1. Arterial blood gas
2. Pulmonary angiography
3. Magnetic resonance imaging
4. V/Q scan
Answer: 2 - Pulmonary angiography
Explanations:
Pulmonary arteriogram is the definitive study for pulmonary embolism.

Go to the next page if you knew the correct answer, or click the link images
below to further research the concepts in this question (if desired).

Research Concepts:
Pulmonary Angiography:

Pulmonary Embolism:

Tap flag to report any problems with this question.


Question 633: How does cholestyramine effect other drugs?
Choices:
1. Increases metabolism of drugs
2. Increases levels of drugs
3. Decreases absorption of drugs
4. Increase urinary excretion of drugs
Answer: 3 - Decreases absorption of drugs
Explanations:
Cholestyramine can interfere with absorption of many drugs and impair
absorption of fat-soluble vitamins like A, D, E, and K.
Cholestyramine can interact with digoxin, estrogens, penicillin
spironolactone, tetracycline, thiazides, thyroid hormone, warfarin, and
phenobarbital.
The most common side effects of cholestyramine is constipation.
There is an increased risk for gallstones due to increased cholesterol
concentration of bile.

Go to the next page if you knew the correct answer, or click the link images
below to further research the concepts in this question (if desired).

Research Concepts:
Bile Acid Sequestrant:

Tap flag to report any problems with this question.


Question 634: Bleeding from a peptic ulcer could present in all of the
following ways except:

Choices:
1. Coffee-ground emesis
2. Black tarry stools
3. Hematemesis
4. Epistaxis
Answer: 4 - Epistaxis
Explanations:
Peptic ulcers are prone to bleeding and perforation
Ulcers on the lesser curvature of the stomach are prone to severe bleeding
Ulcers on greater curvature are prone to perforation
Epistaxis is not a feature of a GI bleeding

Go to the next page if you knew the correct answer, or click the link images
below to further research the concepts in this question (if desired).

Research Concepts:
Peptic Ulcer Disease:

Tap flag to report any problems with this question.


Question 635: What is the most common disease associated with sudden
cardiac death?

Choices:
1. Hypertension
2. Aortic stenosis
3. Coronary artery disease
4. Peripheral vascular disease
Answer: 3 - Coronary artery disease
Explanations:
CAD accounts for over 80 percent of sudden cardiac death

Go to the next page if you knew the correct answer, or click the link images
below to further research the concepts in this question (if desired).

Research Concepts:
Sudden Cardiac Death:

Coronary Artery Disease:

Tap flag to report any problems with this question.


Question 636: What is the most likely explanation for an arterial blood gas
sample that has clotted before analysis?

Choices:
1. It wasn't put on ice
2. It was not drawn in a heparinized syringe
3. It was not heated properly
4. None of the above
Answer: 2 - It was not drawn in a heparinized syringe
Explanations:
The ABG sample clotted before analysis because it was not drawn with a
heparinized syringe
It should be placed on ice if it can't be analyzed immediately
Ice retards conversion of O2 to CO2, which should prevent falsely elevated
PCO2 levels causing falsely low pH readings

Go to the next page if you knew the correct answer, or click the link images
below to further research the concepts in this question (if desired).

Research Concepts:
Arterial Blood Gas:

Clotting Mechanism:

Tap flag to report any problems with this question.


Question 637: By which of the following mechanisms does hydralazine
vasodilate blood vessels?

Choices:
1. Hydralazine blocks alpha receptors.
2. Hydralazine increases cyclic AMP.
3. Hydralazine increases nitric oxide levels.
4. Hydralazine blocks beta receptors.
Answer: 3 - Hydralazine increases nitric oxide levels.
Explanations:
Hydralazine acts via direct relaxation of smooth muscle, most likely
involving the nitric oxide system.
Hydralazine can cause a lupus-like effect at high doses.
Hydralazine is acetylated and the dose must be individualized due to
variations in activity of this enzyme.

Go to the next page if you knew the correct answer, or click the link images
below to further research the concepts in this question (if desired).

Research Concepts:
Antihypertensive Medications:

Tap flag to report any problems with this question.


Question 638: Which of the following is a class 1C anti-arrhythmic agent?
Choices:
1. Procainamide
2. Amiodarone
3. Quinidine
4. Flecainide
Answer: 4 - Flecainide
Explanations:
Flecainide is a group 1C anti-arrhythmic prototype drug.
Flecainide is used to treat ventricular tachycardia and rapid atrial
arrhythmias, such as in Wolff-Parkinson-White syndrome.
The drug is effective, but it is toxic. It can cause a worsening of arrhythmias
as well as central nervous system excitation.

Go to the next page if you knew the correct answer, or click the link images
below to further research the concepts in this question (if desired).

Research Concepts:
Antiarrhythmic Medication:

Tap flag to report any problems with this question.


Question 639: Which medication has no effect on a transplanted heart?
Choices:
1. Atropine
2. Epinephrine
3. Dopamine
4. Phenylephrine
Answer: 1 - Atropine
Explanations:
A transplanted heart has no autonomic innervations

Go to the next page if you knew the correct answer, or click the link images
below to further research the concepts in this question (if desired).

Research Concepts:
Atropine:

Heart Transplantation:

Tap flag to report any problems with this question.


Question 640: Your patient had an acute myocardial infarction and is to be
given atenolol (Tenormin) 5 mg intravenously stat. This medication will do
which of the following?

Choices:
1. Increase the heart rate
2. Reduce myocardial oxygen consumption
3. Increase the blood pressure
4. Lyse thrombus formation
Answer: 2 - Reduce myocardial oxygen consumption
Explanations:
Atenolol (Tenormin) reduces myocardial oxygen demand. It is a beta-
adrenergic blocker and affects the sympathetic nervous system by
decreasing heart rate, blood pressure, and myocardial oxygen demand.
Atenolol will decrease heart rate, not increase it. Atenolol will decrease
blood pressure, not increase it.
Atenolol is a beta-adrenergic blocker, not a thrombolytic.
Pregnancy Category: C Patient Need: Physiological Integrity

Go to the next page if you knew the correct answer, or click the link images
below to further research the concepts in this question (if desired).

Research Concepts:
Beta-Blockers:

Myocardial Infarction, Acute:

Tap flag to report any problems with this question.


Question 641: Which of the following is a serious side effect of heparin?
Choices:
1. Bone marrow depression
2. Anemia
3. Thrombocytopenia
4. Neutropenia
Answer: 3 - Thrombocytopenia
Explanations:
Heparin induced thrombocytopenia is a common complication
It usually occurs within 5 to 10 days after heparin treatment has started
It occurs due to formation of antibodies against the heparin-platelet factor 4
complex
One known risk factor is the use of unfractionated heparin

Go to the next page if you knew the correct answer, or click the link images
below to further research the concepts in this question (if desired).

Research Concepts:
Thrombocytopenia, Heparin Induced:

Tap flag to report any problems with this question.


Question 642: A patient has deep venous thrombosis in the femoral veins
and develops an embolic stroke. Select the most likely explanation.

Choices:
1. Aortic valve endocarditis
2. Patent foramen ovale
3. Pulmonary embolism resulting in hypotension
4. Cerebrovascular and peripheral arterial disease
Answer: 2 - Patent foramen ovale
Explanations:
The patient has deep venous thrombosis and paradoxical emboli
The thromboembolism traveled from the venous system through the patent
foramen ovale to the arterial system
This caused the stroke

Go to the next page if you knew the correct answer, or click the link images
below to further research the concepts in this question (if desired).

Research Concepts:
Paradoxical Embolism:

Tap flag to report any problems with this question.


Question 643: Which of the followings is not a clinical feature of acute
myocardial infarction?

Choices:
1. Mild fever
2. Mild chest discomfort
3. Elderly patients are less likely to present with confusion
4. Increase of heart rate
Answer: 3 - Elderly patients are less likely to present with confusion
Explanations:
Elderly patients may present with nonspecific symptoms of low cardiac
output
Acute MI is commonly associated with increased heart rate and mild fever

Go to the next page if you knew the correct answer, or click the link images
below to further research the concepts in this question (if desired).

Research Concepts:
Acute Myocardial Infarction:

Tap flag to report any problems with this question.


Question 644: Which of the following is rarely added to stored packed red
blood cells?

Choices:
1. Calcium
2. Citrate
3. Phosphate
4. Adenine
Answer: 1 - Calcium
Explanations:
PRBC is usually stored for no longer than 30 days
Additive to PRBC includes citrate, phosphate, and adenine

Go to the next page if you knew the correct answer, or click the link images
below to further research the concepts in this question (if desired).

Research Concepts:
Blood Transfusion:

Tap flag to report any problems with this question.


Question 645: The lesser saphenous vein typically empties into the:
Choices:
1. Greater saphenous vein
2. Femoral vein
3. Anterior tibial vein
4. Popliteal vein
Answer: 4 - Popliteal vein
Explanations:
The lesser saphenous vein runs on the posterolateral lower leg and empties
into the popliteal vein at the popliteal fossa

Go to the next page if you knew the correct answer, or click the link images
below to further research the concepts in this question (if desired).

Research Concepts:
Saphenous Vein:

Tap flag to report any problems with this question.


Question 646: What pressure gradient across the aortic valve in an aortic
valve replacement recommended?

Choices:
1. 25 mmHg
2. 35 mmHg
3. 40 mmHg
4. 55 mmHg
Answer: 4 - 55 mmHg
Explanations:
Aortic valve replacement is recommended with pressure gradient above 50
mmHg

Go to the next page if you knew the correct answer, or click the link images
below to further research the concepts in this question (if desired).

Research Concepts:
Aortic Stenosis:

Tap flag to report any problems with this question.


Question 647: Which is FALSE about skin antiseptic?
Choices:
1. Ethanol 70% is an effective skin antiseptic
2. Formaldehyde is preferable to ethanol because it is less irritating
3. Acetic acid can be used to treat gram-negative skin infection
4. Salicylic acid is used to treat certain skin yeast infections
Answer: 2 - Formaldehyde is preferable to ethanol because it is less irritating
Explanations:
Formaldehyde works like ethanol but is too irritating for topical use and is
only used as a disinfectant.
Alcohol acts by denaturing bacterial proteins.
Acetic acid is sometimes used in surgical dressings to treat certain gram-
negative skin infections.

Go to the next page if you knew the correct answer, or click the link images
below to further research the concepts in this question (if desired).

Research Concepts:
Skin Antiseptics:

Tap flag to report any problems with this question.


Question 648: What is the beta blocker that is classified as a class 3 anti-
arrhythmic agent?

Choices:
1. Metoprolol
2. Atenolol
3. Esmolol
4. Sotalol
Answer: 4 - Sotalol
Explanations:
Sotalol is in the same anti arrhythmic class drug as Amiodarone.
Sotalol is used in the treatment of ventricular arrhythmias.

Go to the next page if you knew the correct answer, or click the link images
below to further research the concepts in this question (if desired).

Research Concepts:
Antiarrhythmic Medication:

Tap flag to report any problems with this question.


Question 649: Which of the following is not true about extracorporeal
membrane oxygenation (ECMO)?

Choices:
1. It is used to treat congenital diaphragmatic hernias
2. It can be used to treat meconium aspiration
3. It can be used to treat persistent pulmonary hypertension
4. It can be used to treat congenital adenomatoid malformation
Answer: 4 - It can be used to treat congenital adenomatoid malformation
Explanations:
ECMO is not used for congenital adenomatoid malformation.
When used in the arterial-venous form, the carotid artery is ligated at the
end of the procedure.
Heparinization is used and, in neonates, central nervous system hemorrhage
is a major complication.

Go to the next page if you knew the correct answer, or click the link images
below to further research the concepts in this question (if desired).

Research Concepts:
Extracorporeal Membrane Oxygenation:

Tap flag to report any problems with this question.


Question 650: Which hematological side effect can be seen with nitrous
oxide?

Choices:
1. Megaloblastic anemia
2. Aplastic anemia
3. Hypochromic anemia
4. Microcytic anemia
Answer: 1 - Megaloblastic anemia
Explanations:
Nitrous oxide interacts with vitamin B12. B12 is a coenzyme that works
with methionine synthetase.
It has a monovalent cobalt with tetrapyrrole rings.
Nitrous oxide changes the cobalt to bivalent so the activity of methionine
synthetase is decreased.
This causes a megaloblastic anemia.

Go to the next page if you knew the correct answer, or click the link images
below to further research the concepts in this question (if desired).

Research Concepts:
Megaloblastic Anemia:

Nitrous Oxide:

Tap flag to report any problems with this question.


Question 651: Which agents have shown the most benefit in treating
patients with poor ejection fraction?

Choices:
1. ACE inhibitors
2. Calcium channel blockers
3. Diuretics
4. Beta blockers
Answer: 1 - ACE inhibitors
Explanations:
Congestive heart failure was traditionally treated with a positive inotrope,
digoxin, and diuretics
ACE inhibitors have been found to reduce afterload and preload. They can
be used as monotherapy in some patients.
They are also used for patients with recent myocardial infarction.
Patients with the lowest ejection fraction have the best response.

Go to the next page if you knew the correct answer, or click the link images
below to further research the concepts in this question (if desired).

Research Concepts:
Heart Failure, Congestive:

Tap flag to report any problems with this question.


Question 652: Your patient is admitted with the complaint of "fast,
skipping heartbeats." The patient is to receive disopyramide (Norpace) 200 mg
every 6 hours by mouth. Given that disopyramide is available only in oral form,
which of the following is true?

Choices:
1. It is not used in acute situations
2. It must be taken with milk
3. The onset of action is between 1 to 2 hours
4. It must be given 30 minutes before or 1 hour after meals
Answer: 1 - It is not used in acute situations
Explanations:
Disopyramide (Norpace) is available only in oral form and is therefore not
used in acute situations. Disopyramide (Norpace) blocks sodium channels
(local anesthetic effect) and slows conduction of electrical impulses in the
heart.
It is used to treat ectopic ventricular beats. Side effects include hypotension,
congestive heart failure, worsened or new arrhythmias, nausea, and
vomiting.
Disopyramide (Norpace) should not be taken with food because food may
affect absorption. The onset of action of disopyramide (Norpace) is 0.5 to
3.5 hours.
Plasma levels peak within 2 to 2.5 hours and the effects persist for 1.5 to
8.5 hours after the last dose. Disopyramide (Norpace) should be given on an
empty stomach 1 hour before meals or 2 hours after meals.

Go to the next page if you knew the correct answer, or click the link images
below to further research the concepts in this question (if desired).

Research Concepts:
Antiarrhythmic Medication:

Tap flag to report any problems with this question.


Question 653: A 66-year-old presents with a history of coronary artery
bypass 3 years ago. He is on furosemide and allopurinol. Which of the following
statements concerning antiplatelet therapy is correct?

Choices:
1. Aspirin has been shown to be an effective antiplatelet agent
2. All antiplatelet agents work by inhibiting prostaglandin synthesis
3. Antiplatelet agents must be started 48 hours after surgery for maximal
benefit
4. Aspirin is effective for treatment of deep venous thrombosis
Answer: 1 - Aspirin has been shown to be an effective antiplatelet agent
Explanations:
Thrombotic and embolic events can be prevented by the use of antiplatelet
agents. Studies have shown that strokes in men with carotid artery disease
can be reduced by the use of antiplatelet agents.
Coronary artery bypass grafts are more likely to remain patent if antiplatelet
therapy is started preoperatively and continued long-term.
Aspirin irreversibly inhibits cyclo-oxygenase, but ticlopidine alters the
platelet membrane.
Deep venous thrombosis is not treated with aspirin.

Go to the next page if you knew the correct answer, or click the link images
below to further research the concepts in this question (if desired).

Research Concepts:
Antiplatelet Medications:

Coronary Artery Bypass Graft:

Tap flag to report any problems with this question.


Question 654: Which of the following is not a theorized cause of unstable
angina?

Choices:
1. Plaque rupture
2. Vasospasm
3. Mismatch of supply and demand
4. Occlusive thrombus
Answer: 4 - Occlusive thrombus
Explanations:
The cause of unstable angina is a mismatch of oxygen supply and demand
This can be secondary to plaque rupture, partial occlusion by thrombus, or
vasospasm
Complete occlusion by a thrombus results in infarction, not unstable angina
Stable angina is caused by coronary atherosclerosis

Go to the next page if you knew the correct answer, or click the link images
below to further research the concepts in this question (if desired).

Research Concepts:
Angina, Unstable:

Tap flag to report any problems with this question.


Question 655: How is the ED50 of a drug defined?
Choices:
1. No response
2. Ten percent of maximal response
3. Half of maximal response
4. Effect seen in half of patients
Answer: 4 - Effect seen in half of patients
Explanations:
ED50 is the drug dosage that is effective in 50% of patients.

Go to the next page if you knew the correct answer, or click the link images
below to further research the concepts in this question (if desired).

Research Concepts:
Effective Dose:

ED50:

Tap flag to report any problems with this question.


Question 656: A 69 year old with syncope is found to have complete heart
block and a wide complex escape rhythm on ECG. Several weeks ago, ECG and
all vital signs were normal. Which of the following is the ideal treatment?

Choices:
1. Vagal maneuvers
2. Transvenous pacing
3. Oral beta blocker therapy
4. Holter monitor
Answer: 2 - Transvenous pacing
Explanations:
The wide complex rhythm suggests block below the AV node which is a life
threatening condition.
Immediate pacing is indicated.

Go to the next page if you knew the correct answer, or click the link images
below to further research the concepts in this question (if desired).

Research Concepts:
Heart Block, Third Degree:

Tap flag to report any problems with this question.


Question 657: Which of the following clotting factors are not dependent on
vitamin K?

Choices:
1. Factor II
2. Factor VI
3. Factor IX
4. Protein C
Answer: 2 - Factor VI
Explanations:
Clotting factors II, VII, IX, and X are vitamin K dependent
Protein C and S are anticoagulants also dependent on Vitamin K
Protein C has the shortest half-life than other clotting factors
Protein C deficiency is implicated in warfarin induced skin necrosis

Go to the next page if you knew the correct answer, or click the link images
below to further research the concepts in this question (if desired).

Research Concepts:
Vitamin K:

Clotting Factors:

Tap flag to report any problems with this question.


Question 658: In a 66 year old intubated, comatose patient, what is one of
the most important information that one needs to obtain?

Choices:
1. Organ donation status
2. Power of attorney
3. Lawyer
4. Driver's license
Answer: 2 - Power of attorney
Explanations:
A power of attorney is important to obtain.
It dictates the further management of this patient.

Go to the next page if you knew the correct answer, or click the link images
below to further research the concepts in this question (if desired).

Research Concepts:
Power of Attorney:

Tap flag to report any problems with this question.


Question 659: How does the potency of hydromorphone compare to
morphine?

Choices:
1. Hydromorphone and morphine are equipotent
2. Hydromorphone is approximately 2 times more potent than morphine
3. Hydromorphone is 5 to 8 times more potent than morphine
4. Hydromorphone is 15 to 20 times more potent that morphine
Answer: 3 - Hydromorphone is 5 to 8 times more potent than morphine
Explanations:
Hydromorphone (Dilaudid) is a semi-synthetic derivative of morphine.
It approximately 5 to 8 times more potent than morphine, therefore it must
be given in smaller doses.

Go to the next page if you knew the correct answer, or click the link images
below to further research the concepts in this question (if desired).

Research Concepts:
Hydromorphone:

Tap flag to report any problems with this question.


Question 660: A 67-year-old undergoes an elective catheterization for an
evaluation of chest pain. During the procedure the cardiologist notices that the
guide wire from the femoral artery has caused a small 0.2 cm flap just distal to
the left subclavian artery. The patient is stable and has no symptoms. Which of
the following is most appropriate for the continued management of this patient?

Choices:
1. Continue with catheterization to determine coronary artery disease
2. Set the patient for a left thoracotomy
3. Obtain a CT scan to determine the exact dimension of the tear
4. Observe the patient
Answer: 4 - Observe the patient
Explanations:
The catheterization should be cancelled and the patient shoule be observed.
The dissection is small and the patient is asymptomatic. Since the flow of
blood is antegrade this dissected flap will just fold over and close.
However, the patient will need follow-up in the future. For the present, no
interventions are required and catheterization should be repeated at a later
date.
Most iatrogenic dissections in the catheterization lab will close.

Go to the next page if you knew the correct answer, or click the link images
below to further research the concepts in this question (if desired).

Research Concepts:
Dissection, Aortic:

Coronary Artery Disease:

Tap flag to report any problems with this question.


Question 661: 5-year survival after angioplasty for nondiabetics is
beneficially effected by all of the following EXCEPT:

Choices:
1. Normotension
2. Younger age
3. Normal ejection fraction
4. No CHF
Answer: 1 - Normotension
Explanations:
Blood pressure does not correlate with post PTCA survival

Go to the next page if you knew the correct answer, or click the link images
below to further research the concepts in this question (if desired).

Research Concepts:
Percutaneous Transluminal Coronary Angioplasty:

Tap flag to report any problems with this question.


Question 662: Which of the following is LEAST likely with use of
metoprolol?

Choices:
1. Slow heart rate
2. Bronchoconstriction
3. Impotence
4. Hyperglycemia
Answer: 4 - Hyperglycemia
Explanations:
Beta blockers, such as metoprolol, can worsen pulmonary status in patients
with emphysema, asthma, or chronic bronchitis.
They can also cause symptomatic bradycardia or atrioventricular blocks.
Beta blockers improve quality of life and survival in CHF but acutely can
precipitate the condition.
Sudden discontinuation of beta blockers in patients with CAD can result in
ischemia.

Go to the next page if you knew the correct answer, or click the link images
below to further research the concepts in this question (if desired).

Research Concepts:
Beta-Blockers:

Tap flag to report any problems with this question.


Question 663: Xanthelasma are usually located where on the face?
Choices:
1. Medial aspect of nose
2. Medial aspect of upper eyelids
3. Lateral to the ear
4. On the bridge of the nose
Answer: 2 - Medial aspect of upper eyelids
Explanations:
Xanthelasma are usually located on the medial side of the upper eye lids.
They usually appear yellowish, soft, and present as small plaques.
Xanthelasma do not affect eye function.

Go to the next page if you knew the correct answer, or click the link images
below to further research the concepts in this question (if desired).

Research Concepts:
Xanthelasma:

Tap flag to report any problems with this question.


Question 664: Bag mask ventilation is least difficult in a patient with:
Choices:
1. Facial hair
2. Lack of teeth
3. History of sleep apnea
4. BMI of less than 17
Answer: 4 - BMI of less than 17
Explanations:
Bag mask ventilation requires a good seal and a patent airway
Certain factors predict difficult bag mask ventilation
These factors include age > 60, history of sleep apnea, obesity, BMI > 26,
lack of teeth and facial hair

Go to the next page if you knew the correct answer, or click the link images
below to further research the concepts in this question (if desired).

Research Concepts:
Bag Mask Ventilation:

Tap flag to report any problems with this question.


Question 665: How many types of P wave are in multifocal atrial
tachycardia?

Choices:
1. 1
2. 2
3. 3
4. 4
Answer: 3 - 3
Explanations:
MAT is characterized by atrial rate greater than 100 per minute and the
presence of three morphologies of P waves
and the presence of three morphologies of P waves

Go to the next page if you knew the correct answer, or click the link images
below to further research the concepts in this question (if desired).

Research Concepts:
Multifocal Atrial Tachycardia:

Tap flag to report any problems with this question.


Question 666: In a patient with suspected atherosclerosis, what is the first
test of choice?

Choices:
1. Coronary angiogram
2. Stress test
3. ECG plus lipid profile
4. Blood work to rule out thrombotic disorders like protein C and S deficiency
Answer: 3 - ECG plus lipid profile
Explanations:
As in all medical disorders, the basic blood work needs to be ordered.
An ECG will be obtained as a baseline. A lipid profile is essential.
Other blood work may include chemistry and thyroid function tests.
A chest x-ray is performed to look at the heart.

Go to the next page if you knew the correct answer, or click the link images
below to further research the concepts in this question (if desired).

Research Concepts:
Atherosclerosis:

Tap flag to report any problems with this question.


Question 667: Which of the following receptors is not affected by
dopamine?

Choices:
1. Alpha
2. Beta
3. Dopamine
4. Delta
Answer: 4 - Delta
Explanations:
Dopamine at low doses affects the dopamine receptors. This can increase
renal blood flow and improve urine output.
At high doses, dopamine stimulates the beta-receptor.
At very high doses, the alpha-receptors are stimulated.
The renal dopamine dose is 2-5mcq/min/kg.

Go to the next page if you knew the correct answer, or click the link images
below to further research the concepts in this question (if desired).

Research Concepts:
Inotropes And Vasopressors:

Tap flag to report any problems with this question.


Question 668: In which of the following is a prophylactic antibiotic
indicated?

Choices:
1. Prior to a foley catheter placement
2. In mechanically ventilated patients
3. Prior to insertion of a central line
4. To decrease the incidence of surgical site infections prior to elective surgery
Answer: 4 - To decrease the incidence of surgical site infections prior to
elective surgery

Explanations:
Prophylactic antibiotics do not prevent many types of infections such as
urinary tract infections, ventilator-associated pneumonia, or intravenous
line sepsis.
Prophylactic antibiotics are able to decrease the risk of infection in certain
elective surgery procedures.
Prophylactic antibiotics are not recommended for all surgeries and misuse
can lead to the development of resistance.

Go to the next page if you knew the correct answer, or click the link images
below to further research the concepts in this question (if desired).

Research Concepts:
Preoperative Antibiotics:

Tap flag to report any problems with this question.


Question 669: What is the most common reason for patients operated with
tetralogy of Fallot (TOF) in childhood to return with symptoms in adult life?

Choices:
1. Recurrent ventricular septal defect
2. Pulmonary valve stenosis
3. Pulmonary valve regurgitation
4. Aortic regurgitation
Answer: 3 - Pulmonary valve regurgitation
Explanations:
After TOF repair, any recurrent ventricular septal defects have to be
repaired early, since these patients do not tolerate any residual shunting.
The pulmonary valve stenosis is also repaired early with a patch and/or
valvoplasty. These patients will do poorly if there is any type of right
ventricular outflow tract obstruction.
The most common reason for patients to return as adults after TOF repair is
for pulmonary valve regurgitation. They usually tolerate pulmonary
insufficiency for years, but after a decade or two, patients will return with
symptoms.
These patients now require a homograft to replace the pulmonary valve.

Go to the next page if you knew the correct answer, or click the link images
below to further research the concepts in this question (if desired).

Research Concepts:
Tetralogy Of Fallot:

Tap flag to report any problems with this question.


Question 670: What is the mechanism of digoxin?
Choices:
1. Sodium channel blocker
2. Potassium channel blocker
3. Na+/K+-ATPase inhibitor
4. Diuretic
Answer: 3 - Na+/K+-ATPase inhibitor
Explanations:
Digoxin is a cardiac glycoside that partially inhibits the Na+/K+-ATPase
This results in increased levels of calcium in cardiac cells.
This increases contractility slightly and decreases heart rate

Go to the next page if you knew the correct answer, or click the link images
below to further research the concepts in this question (if desired).

Research Concepts:
Digoxin:

Tap flag to report any problems with this question.


Question 671: What is the single greatest preventable cause of death in the
United States?

Choices:
1. Alcohol
2. Unprotected anal intercourse
3. Smoking
4. Obesity
Answer: 3 - Smoking
Explanations:
Smoking is closely linked not only with cancer, but also with
cardiovascular, pulmonary, and other systemic diseases.
Numerous cutaneous manifestations result from smoking.

Go to the next page if you knew the correct answer, or click the link images
below to further research the concepts in this question (if desired).

Research Concepts:
Smoking:

Tap flag to report any problems with this question.


Question 672: A midsystolic click is indicative of:
Choices:
1. Mitral valve prolapse
2. Mitral stenosis
3. Aortic stenosis
4. Tricuspid regurgitation
Answer: 1 - Mitral valve prolapse
Explanations:
Midsystolic click is presented with MVP

Go to the next page if you knew the correct answer, or click the link images
below to further research the concepts in this question (if desired).

Research Concepts:
Mitral Valve Prolapse:

Tap flag to report any problems with this question.


Question 673: What is the term for the potential maximum therapeutic
response that a drug can produce?

Choices:
1. Affinity
2. Bioavailability
3. Efficacy
4. Potency
Answer: 3 - Efficacy
Explanations:
Efficacy refers to the potential maximum therapeutic response that a drug
can produce
Potency refers to the amount of drug needed to produce an effect
Affinity refers to the ability of a drug to bind to receptors and form drug
receptor complex

Go to the next page if you knew the correct answer, or click the link images
below to further research the concepts in this question (if desired).

Research Concepts:
Efficacy:

Tap flag to report any problems with this question.


Question 674: In the PIOPED trial, which of the following was the most
commonly presenting feature of a pulmonary embolism?

Choices:
1. Dyspnea
2. Chest pain
3. Cough
4. Hemoptysis
Answer: 1 - Dyspnea
Explanations:
The PIOPED trial showed that 3 consistent features of PE are dyspnea,
chest pain, and cough.
Less common features of PE include fever, wheezing, abdominal pain, and
syncope.

Go to the next page if you knew the correct answer, or click the link images
below to further research the concepts in this question (if desired).

Research Concepts:
Pulmonary Embolism:

Tap flag to report any problems with this question.


Question 675: Which of the following muscle relaxants causes muscle
fasciculations prior to muscle relaxation?

Choices:
1. Atracurium
2. Succinylcholine
3. Pancuronium
4. Rocuronium
Answer: 2 - Succinylcholine
Explanations:
Succinylcholine, a depolarizing neuromuscular blocker
It initially causes fasciculations because it stimulates the nicotinic
acetylcholine receptors on skeletal muscle
Muscle fasciculations can be observed before the muscle goes into
depolarization block

Go to the next page if you knew the correct answer, or click the link images
below to further research the concepts in this question (if desired).

Research Concepts:
Succinylcholine:

Tap flag to report any problems with this question.


Question 676: With a VVI pacemaker, the I stands for which of the
following?

Choices:
1. Index
2. Inhibition
3. Inflex
4. Intelligent
Answer: 2 - Inhibition
Explanations:
VVI is a ventricular demand inhibited pacemaker.

Go to the next page if you knew the correct answer, or click the link images
below to further research the concepts in this question (if desired).

Research Concepts:
Pacemaker:

Tap flag to report any problems with this question.


Question 677: What is not an effect of calcium channel blockers?
Choices:
1. Decreased heart rate
2. Decreased blood pressure
3. Vasodilation
4. Prevent myocardial infarction
Answer: 4 - Prevent myocardial infarction
Explanations:
Calcium channel blockers do not prevent myocardial infarction

Go to the next page if you knew the correct answer, or click the link images
below to further research the concepts in this question (if desired).

Research Concepts:
Calcium Channel blockers:

Tap flag to report any problems with this question.


Question 678: Which of the following does not transmit viral hepatitis?
Choices:
1. Albumin
2. Immunoglobulin
3. Cryoprecipitate
4. Packed red blood cells
Answer: 1 - Albumin
Explanations:
Donor blood is routinely screened for hepatitis B, hepatitis C, HIV, syphilis,
and West Nile virus, but not all donors are screened for cytomegalovirus.
Transmission of infectious organisms can occur with all blood products
except albumin.
Albumin is a highly processed plasma derivative and transfusion reactions
are very rare.
The risk of acquiring HIV via a blood transfusion is about 1 in 2 million
units transfused.

Go to the next page if you knew the correct answer, or click the link images
below to further research the concepts in this question (if desired).

Research Concepts:
Viral Hepatitis:

Tap flag to report any problems with this question.


Question 679: What is false about quinidine?
Choices:
1. Effective against atrial arrhythmia
2. Effective against ventricular arrhythmia
3. No protein binding
4. Orally effective
Answer: 3 - No protein binding
Explanations:
Quinidine is highly protein bound

Go to the next page if you knew the correct answer, or click the link images
below to further research the concepts in this question (if desired).

Research Concepts:
Quinidine:

Tap flag to report any problems with this question.


Question 680: Which of the following is not a beneficial effect associated
with HMG-CoA reductase inhibitors?

Choices:
1. Revascularization of occluded vessels
2. Improved vasodilatory tone
3. Stabilization of preexisting coronary lesions
4. Reduction of LDL
Answer: 1 - Revascularization of occluded vessels
Explanations:
HMG-Co A reductase inhibitors lower LDL and stabilize preexisting
coronary lesions
They improve vasodilatory tone by modulation of vasodilators from the
endothelium
They probably cause regression of fixed lesions
They are unlikely to open a completely occluded vessel

Go to the next page if you knew the correct answer, or click the link images
below to further research the concepts in this question (if desired).

Research Concepts:
Statin Medication:

Tap flag to report any problems with this question.


Question 681: What is the largest cutaneous branch of the femoral nerve?
Choices:
1. Saphenous nerve
2. Nerve to vastus lateralis
3. Nerve to gracilis
4. Sural
Answer: 1 - Saphenous nerve
Explanations:
The largest cutaneous branch of the femoral nerve is the saphenous nerve.

Go to the next page if you knew the correct answer, or click the link images
below to further research the concepts in this question (if desired).

Research Concepts:
Femoral Nerve:

Tap flag to report any problems with this question.


Question 682: What is the most common cardiac lesion associated with
Marfan syndrome?

Choices:
1. Aortic stenosis
2. Mitral valve prolapse
3. Coarctation of the aorta
4. Left ventricular outflow tract obstruction
Answer: 2 - Mitral valve prolapse
Explanations:
The most common heart lesions with Marfan syndrome include mitral valve
prolapse and dilatation of the aortic root
They are caused by abnormalities in connective tissue as a result of genetic
mutation

Go to the next page if you knew the correct answer, or click the link images
below to further research the concepts in this question (if desired).

Research Concepts:
Marfan Syndrome:

Tap flag to report any problems with this question.


Question 683: Which of the following is the most appropriate first step in
management of pseudomembranous colitis?

Choices:
1. Complete the intended antibiotic course
2. NPO and intravenous fluids
3. Prescribe oral metronidazole
4. Prescribe oral vancomycin
Answer: 3 - Prescribe oral metronidazole
Explanations:
C. difficile infection causes pseudomembranous colitis
This usually follows patients who are taking broad-spectrum antibiotics that
should be discontinued if possible
Diagnosis is made by C. difficile toxin assay or rarely endoscopy
Metronidazole is the treatment of choice for C. difficile infection

Go to the next page if you knew the correct answer, or click the link images
below to further research the concepts in this question (if desired).

Research Concepts:
Clostridium Difficile Colitis:

Tap flag to report any problems with this question.


Question 684: Virchow triad explains the factors predisposing individuals
to:

Choices:
1. Pneumonia
2. Myocardial infarction (MI)
3. Pulmonary embolus (PE)
4. Pleural effusions
Answer: 3 - Pulmonary embolus (PE)
Explanations:
Virchow triad explains the factors predisposing individuals to PE
Venous stasis can occur with bed rest and immobilization
Hypercoagulability can be inherited or acquired and can occur post-surgery
or post-pregnancy
Trauma to vascular walls is the last of the triad

Go to the next page if you knew the correct answer, or click the link images
below to further research the concepts in this question (if desired).

Research Concepts:
Pulmonary Embolism:

Tap flag to report any problems with this question.


Question 685: Physiological fluid inside the lumen of blood vessels is
considered:

Choices:
1. Interstitial
2. Intravascular
3. Intracellular
4. Extracellular
Answer: 2 - Intravascular
Explanations:
The intracellular space describes the compartment inside of a cell
Interstitial space describes the compartment between cells
Extracellular describes the compartment outside of a cell
Extracellular is the sum of interstitial plus intravascular

Go to the next page if you knew the correct answer, or click the link images
below to further research the concepts in this question (if desired).

Research Concepts:
Body Fluids:

Tap flag to report any problems with this question.


Question 686: What is the mechanism of action of phenoxybenzamine?
Choices:
1. It is a ganglionic blocker
2. It is a non-selective alpha and beta adrenergic blocker
3. It is a non-selective alpha adrenergic blocker
4. It is a non-selective alpha agonist
Answer: 3 - It is a non-selective alpha adrenergic blocker
Explanations:
Phenoxybenzamine is a non-selective alpha adrenergic blocker.
The drug can be used in cases of catecholamine excess such as
pheochromocytoma.
An example of non-selective alpha agonist is phenylephrine.
An example of ganglionic blocker is hexamethonium.

Go to the next page if you knew the correct answer, or click the link images
below to further research the concepts in this question (if desired).

Research Concepts:
Phenoxybenzamine:

Tap flag to report any problems with this question.


Question 687: In the ROSS procedure for aortic valve replacement, what is
the alternate valve?

Choices:
1. Mechanical ST Jude valve
2. Pulmonary valve
3. Mitral valve
4. Porcine valve
Answer: 2 - Pulmonary valve
Explanations:
In the Ross procedure, the pulmonary valve is transplanted into the aortic
position
The Ross procedure is done in young people but is technically demanding
The pulmonary valve is replaced by a homograft

Go to the next page if you knew the correct answer, or click the link images
below to further research the concepts in this question (if desired).

Research Concepts:
Aortic Valve:

Tap flag to report any problems with this question.


Question 688: Which of the following is not a sign of irreversible cell
injury?

Choices:
1. Apoptosis
2. Pyknosis
3. Karyorrhexis
4. Vacuolar degeneration
Answer: 4 - Vacuolar degeneration
Explanations:
Apoptosis, pyknosis, and karyorrhexis are signs of irreversible cell injury.
Vacuolar degeneration is the formation of cytoplasmic nonlipid vacuoles
and is not a sign of irreversible cell injury.
Apoptosis is programmed cell death and shows blebbing, nuclear
fragmentation, cell shrinkage, chromosomal DNA fragmentation, and
chromatin condensation.
Pyknosis is condensation of nuclear chromatin.

Go to the next page if you knew the correct answer, or click the link images
below to further research the concepts in this question (if desired).

Research Concepts:
Cell Death:

Tap flag to report any problems with this question.


Question 689: Which of the following is not true of an aortic dissection?
Choices:
1. 2-4% of patients have no visible intimal tear
2. Intimal dissections without intimal tear are usually thrombosed
3. The intimal tear rarely exceeds more than one half the circumference of the
aorta
4. The dissection, when antegrade, can involve the right kidney
Answer: 4 - The dissection, when antegrade, can involve the right kidney
Explanations:
Intimal tear is a consistent feature of aortic dissection. 2-4% have no
intimal tear and most are confined to the descending aorta. In the majority
of dissections without tears, the false lumen is totally thrombosed
(intramural hematoma).
The majority of tears are in the ascending aorta, followed by the descending
aorta. Tears are least common in the arch and are very rare in the abdominal
aorta. The tears are usually transverse and are rarely more than half the
circumference of the aorta.
In the arch, the false lumen extends along the greater curve and extends in
to the head vessels. In the descending aorta, the false lumen follows the
anterior lateral wall of the aorta and usually involves the left kidney.

Go to the next page if you knew the correct answer, or click the link images
below to further research the concepts in this question (if desired).

Research Concepts:
Dissection, Aortic:

Tap flag to report any problems with this question.


Question 690: What is the best treatment for a patient with sick sinus
syndrome?

Choices:
1. Digoxin
2. Amiodarone
3. Pacemaker
4. Lidocaine
Answer: 3 - Pacemaker
Explanations:
Sick sinus syndrome is commonly seen in the elderly due to degenerative
and inflammatory changes in the conduction system of the heart.
The condition manifests with bradycardia-tachycardia syndrome.
Thus, beta blockers help prevent the tachycardia while the pacemaker
prevents bradycardia.

Go to the next page if you knew the correct answer, or click the link images
below to further research the concepts in this question (if desired).

Research Concepts:
Sick Sinus Syndrome:

Tap flag to report any problems with this question.


Question 691: Which of the following is not true of a mucormycosis
infection?

Choices:
1. Extensive infection occurs in diabetics
2. Usually the organism invades blood vessels, causing thrombosis and
infarction
3. Cultures are usually negative
4. Immediate treatment with amphotericin prevents the high morbidity
Answer: 4 - Immediate treatment with amphotericin prevents the high
morbidity

Explanations:
Mucormycosis affects rhinocerebral sinuses and pulmonary tissues.
Mucormycosis is seen in diabetics, the immunosuppressed, leukemics, and
neutropenic patients. The fungi appear as right angle-shaped hyphae and
cause thrombosis of blood, leading to infarction and gangrene.
The best treatment for mucormycosis is radical surgical debridement of
infected tissue.
Amphotericin coverage must be used at all times. It carries a high mortality
despite surgery.

Go to the next page if you knew the correct answer, or click the link images
below to further research the concepts in this question (if desired).

Research Concepts:
Mucormycosis:

Tap flag to report any problems with this question.


Question 692: Which medication is given to a ductal-dependent cyanotic
infant?

Choices:
1. Indomethacin
2. Prostaglandin E infusion
3. Nitric oxide
4. Viagra
Answer: 2 - Prostaglandin E infusion
Explanations:
Some congenital heart diseases are dependent on the patency of the ductus.
Without a patent ductus, some of the cyanotic congenital heart diseases are
not compatible with life.
A prostaglandin E infusion is used to keep the ductus open after birth.
Otherwise, the ductus closes spontaneously within 24 hours after birth.

Go to the next page if you knew the correct answer, or click the link images
below to further research the concepts in this question (if desired).

Research Concepts:
Congenital Heart Disease:

Tap flag to report any problems with this question.


Question 693: Costophrenic angle blunting on a plain film is often
representative of:

Choices:
1. Pneumothorax
2. Pneumomediastinum
3. Pleural effusion
4. Perforated organ
Answer: 3 - Pleural effusion
Explanations:
A pleural effusion is often represented by costophrenic angle blunting on a
x-ray

Go to the next page if you knew the correct answer, or click the link images
below to further research the concepts in this question (if desired).

Research Concepts:
Effusion, Pleural:

Tap flag to report any problems with this question.


Question 694: A 71-year-old male is admitted to the intensive care unit for
hemodynamic monitoring. He is placed on a number of inotropes and has a left
radial line for blood pressure monitoring. Two days later he is found have a cold
left hand. How could this development have been avoided?

Choices:
1. By starting heparin at the time of cannulation
2. By placing a small gauge cannula
3. By doppler assessment of the wrist arteries
4. By placing the line in the brachial artery
Answer: 3 - By doppler assessment of the wrist arteries
Explanations:
In some patients cannulation of the radial artery can lead to ischemia of the
hand.
Therefore, it is essential to perform a Doppler assessment in all individuals
who are going to need a radial artery line.
The palmar arch is not patent, and placement of a radial line can lead to
complications.

Go to the next page if you knew the correct answer, or click the link images
below to further research the concepts in this question (if desired).

Research Concepts:
Arterial Cannulization:

Tap flag to report any problems with this question.


Question 695: Which of the following is NOT a feature of progressive
systemic sclerosis (scleroderma)?

Choices:
1. Interstitial fibrosis of the lung
2. Myocardial fibrosis
3. Esophagus dysfunction secondary to fibrosis
4. Meningitis
Answer: 4 - Meningitis
Explanations:
Systemic sclerosis is a systemic connective tissue disorder which can affect
almost every organ in the body.
The disorder results in excess collagen deposition in the organs and skin.
Organs affected in systemic sclerosis include the skin, lungs, heart, gut,
kidneys, muscles, joints. and those of the central nervous system.
Renal and lung changes are causes of death in most patients.

Go to the next page if you knew the correct answer, or click the link images
below to further research the concepts in this question (if desired).

Research Concepts:
Scleroderma:

Tap flag to report any problems with this question.


Question 696: Which antihypertensive agent may decrease HDL and
increase triglycerides?

Choices:
1. Thiazide
2. Clonidine
3. Propranolol
4. Captopril
Answer: 3 - Propranolol
Explanations:
Beta blockers can cause numerous side effects including CNS effects.
Side effects of beta blockers include fatigue, insomnia, and lethargy. Beta
blockers may also decrease libido and cause impotence.
Use of beta blockers has also been known to disturb lipid metabolism by
decreasing HDL levels and increasing triacylglycerols levels.
Abrupt withdrawal may cause rebound hypertension. Patients should be
tapered off beta blockers slowly in order to avoid precipitation of
arrhythmias.

Go to the next page if you knew the correct answer, or click the link images
below to further research the concepts in this question (if desired).

Research Concepts:
Beta-Blockers:

Tap flag to report any problems with this question.


Question 697: The purpose of using digitalis for patients with heart failure
is to:

Choices:
1. Decrease paroxysmal atrial tachycardia
2. Increase diastolic filling time
3. Improve sympathetic response
4. Increase heart rate
Answer: 2 - Increase diastolic filling time
Explanations:
Digitalis enhances contractility by controlling ventricular rate and
increasing diastolic filling time
Paroxysmal atrial tachycardia is a common arrhythmia in digitalis toxicity

Go to the next page if you knew the correct answer, or click the link images
below to further research the concepts in this question (if desired).

Research Concepts:
Heart Failure, Congestive:

Digoxin:

Tap flag to report any problems with this question.


Question 698: The incidence of heparin induced thrombocytopenia for
unfractionated heparin is 2.6%. Low molecular weight heparins have an
incidence for this of which of the following?

Choices:
1. 0%
2. 0.5%
3. 1 to 2%
4. 5%
Answer: 3 - 1 to 2%
Explanations:
The incidence of heparin induced thrombocytopenia with low molecular
weight heparin is 1 to 2%

Go to the next page if you knew the correct answer, or click the link images
below to further research the concepts in this question (if desired).

Research Concepts:
Thrombocytopenia, Heparin Induced:

Low Molecular Weight Heparin (LMWH):

Tap flag to report any problems with this question.


Question 699: Which of the following is the MOST common serious
complication of deep vein thrombosis?

Choices:
1. Post phlebitis pain
2. Pulmonary embolism
3. Mesenteric insufficiency
4. Myocardial infarction
Answer: 2 - Pulmonary embolism
Explanations:
Pulmonary embolism is the most common serious complication that can
occur with DVT.
Another possibly more serious but uncommon consequence is a paradoxical
embolism, where the embolus crosses through a cardiac defect to the
arterial system. This can result in stroke.
Mesenteric insufficiency may be due to mesenteric arterial or venous
thrombi, but not deep vein (extremity) venous thrombi.
Patients with PE and elevated plasma lactate levels have increased risk of
other complications including mortality independent of other factors.

Go to the next page if you knew the correct answer, or click the link images
below to further research the concepts in this question (if desired).

Research Concepts:
Pulmonary Embolism:

Tap flag to report any problems with this question.


Question 700: Which of the following is not true of positive end-expiratory
pressure (PEEP)?

Choices:
1. It decreases end diastolic volume
2. It decreases cardiac output
3. It decreases mixed venous oxygen
4. It decreases compliance of the left ventricle
Answer: 4 - It decreases compliance of the left ventricle
Explanations:
PEEP increases alveolar surface area for gas exchange, increases functional
residual capacity, and therefore increases arteriolar oxygen tension.
PEEP can decrease cardiac output and ventricular filling pressure.
PEEP does not affect compliance of the left ventricle.
Hemodynamic effects of PEEP can be reversed with fluids.

Go to the next page if you knew the correct answer, or click the link images
below to further research the concepts in this question (if desired).

Research Concepts:
Positive End-Expiratory Pressure (PEEP):

Tap flag to report any problems with this question.


Question 701: In a patient with stable angina, what is the emphasis of
treatment?

Choices:
1. Start vasodilators
2. Adjust blood pressure medications
3. Treat all modifiable risk factors
4. Obtain a stress test or an angiogram
Answer: 3 - Treat all modifiable risk factors
Explanations:
Coronary artery disease has many risk factors some of which can be
modified.
Once a diagnosis of CAD is made, it is vital to treat all modifiable risk
factors.
Modifiable risk factors include discontinuing smoking, losing weight,
changing diet, and controlling blood sugar.
Unmodifiable risk factors include age, genes, and gender.

Go to the next page if you knew the correct answer, or click the link images
below to further research the concepts in this question (if desired).

Research Concepts:
Angina, Stable:

Tap flag to report any problems with this question.


Question 702: Orthotopic transplant refers to placement of the organ at
what location?

Choices:
1. In a different anatomical site
2. In its anatomical position
3. On top of the previous organ
4. Anywhere where it can function
Answer: 2 - In its anatomical position
Explanations:
Orthotopic transplantation relates to grafting tissue back in the natural
anatomical position.Heterotopic transplantation moves tissue into a
different anatomic location (e.g., transplanting beta pancreatic islet cells
into the forearm.)
Autografts refer to the transplantion of tissue to the same person
Isografts refer to a subset of autografts representing transplants from a
donor to a genetically identical recipient
Xenografts refer to transplants from one species to another.

Go to the next page if you knew the correct answer, or click the link images
below to further research the concepts in this question (if desired).

Research Concepts:
Transplantation:

Tap flag to report any problems with this question.


Question 703: How long before elective surgery should patients stop
aspirin?

Choices:
1. 12 hours
2. 2 days
3. 5 days
4. 7 days
Answer: 4 - 7 days
Explanations:
The effects of aspirin last about 5-7 days in the body.
Most surgeons wait about a week before undertaking any elective surgery.

Go to the next page if you knew the correct answer, or click the link images
below to further research the concepts in this question (if desired).

Research Concepts:
Aspirin:

Preoperative Evaluation And Management:

Tap flag to report any problems with this question.


Question 704: Lidocaine exerts antiarrhythmic effect by acting on which of
the following?

Choices:
1. Calcium channels
2. Sodium channels
3. Potassium channels
4. Magnesium channels
Answer: 2 - Sodium channels
Explanations:
Lidocaine is a class I antiarrhythmic drug.

Go to the next page if you knew the correct answer, or click the link images
below to further research the concepts in this question (if desired).

Research Concepts:
Lidocaine:

Tap flag to report any problems with this question.


Question 705: pH 7.35, PO2 63 mm Hg, PCO2 48 mm Hg, and HCO3 36
mEq/L. What is the interpretation of the ABG?

Choices:
1. Compensated metabolic alkalosis
2. Uncompensated metabolic acidosis
3. Compensated respiratory acidosis
4. Uncompensated respiratory alkalosis
Answer: 3 - Compensated respiratory acidosis
Explanations:
The ABG is consistent with compensated respiratory acidosis
PCO2 is elevated, which is seen in primary respiratory acidosis and as a
compensatory response to metabolic alkalosis
Elevation of bicarbonate is seen in primary metabolic alkalosis and in a
compensatory response to respiratory acidosis
pH is normal, but leaning toward acidosis and tells us primary disturbance
is acidosis which has been compensated

Go to the next page if you knew the correct answer, or click the link images
below to further research the concepts in this question (if desired).

Research Concepts:
Respiratory Acidosis:

Tap flag to report any problems with this question.


Question 706: Which group of drugs has been shown to reduce mortality
in patients with congestive heart failure?

Choices:
1. Alpha blockers
2. Calcium channel blockers
3. Diuretics
4. Angiotensin converting enzyme (ACE) inhibitors
Answer: 4 - Angiotensin converting enzyme (ACE) inhibitors
Explanations:
ACE Inhibitors have been shown to reduce mortality and morbidity in
congestive heart failure.
ACE inhibitors reduce aldosterone secretion, salt and water retention, and
afterload.
ACE inhibitors are now considered first line drugs for congestive heart
failure.
Beta blockers reduce mortality but are not for use during acute
exacerbations.

Go to the next page if you knew the correct answer, or click the link images
below to further research the concepts in this question (if desired).

Research Concepts:
Heart Failure, Congestive:

Tap flag to report any problems with this question.


Question 707: Phentolamine mesylate is administered for the treatment of
hypertension associated with pheochromocytoma. Because of its vasodilatory
effect, it is also administered via the subcutaneous route to treat which of the
following?

Choices:
1. Second-degree burns
2. Norepinephrine extravasations
3. Fungal infections of the oral mucosa
4. Subcutaneous emphysema
Answer: 2 - Norepinephrine extravasations
Explanations:
Phentolamine mesylate (Regitine) may be prescribed to treat
norepinephrine extravasations (escape of norepinephrine into the tissues).
Administration of phentolamine is the recommended treatment for damage
of surrounding tissue associated with norepinephrine extravasation.
Because norepinephrine (Levophed) has potent vasoconstrictive effects,
tissue ischemia and necrosis may occur if infiltration occurs. The
vasodilatory effect of phentolamine helps prevent such tissue damage.
Second-degree burns are usually treated with anti-infectives. Fungal
infections of the oral mucosa are treated with anti-fungals.
Subcutaneous emphysema is often treated without pharmaceuticals.
Pregnancy Category: C Patient Need: Physiological Integrity

Go to the next page if you knew the correct answer, or click the link images
below to further research the concepts in this question (if desired).

Research Concepts:
Toxic, Sympathomimetics:

Tap flag to report any problems with this question.


Question 708: Which of the following drugs is NOT used in the treatment
of congestive heart failure?

Choices:
1. Captopril
2. Furosemide
3. Metoprolol
4. Minoxidil
Answer: 4 - Minoxidil
Explanations:
Minoxidil is an alpha antagonist and has no role in the treatment of
congestive heart failure.
Minoxidil is used in the treatment of hypertension and has an odd side-
effect of excess hair growth.

Go to the next page if you knew the correct answer, or click the link images
below to further research the concepts in this question (if desired).

Research Concepts:
Heart Failure, Congestive:

Tap flag to report any problems with this question.


Question 709: Cardiogenic pulmonary edema may be treated with which
of the following EXCEPT?

Choices:
1. Nitroglycerin for preload reduction
2. Morphine sulfate
3. Captopril for afterload reduction
4. Loop diuretics (furosemide) for preload reduction (diuresis)
Answer: 2 - Morphine sulfate
Explanations:
Although previously used for cardiogenic pulmonary edema, there is no
good evidence to support the use of morphine sulfate as a preload reducing
agent.
It may, in fact, result in decreased respiratory effort and sedation .

Go to the next page if you knew the correct answer, or click the link images
below to further research the concepts in this question (if desired).

Research Concepts:
Pulmonary Edema, Cardiogenic:

Tap flag to report any problems with this question.


Question 710: Which of the following has no affect on torsades de pointes?
Choices:
1. Lidocaine
2. Magnesium
3. Mexiletine
4. Potassium
Answer: 1 - Lidocaine
Explanations:
Lidocaine has no effect on torsades. Torsades usually recurs after a
temporary stop.
Magnesium is the drug of first choice followed by potassium.
Mexiletine may be helpful in stopping torsades.
Sometimes overdrive pacing is used to treat torsades.

Go to the next page if you knew the correct answer, or click the link images
below to further research the concepts in this question (if desired).

Research Concepts:
Torsade de Pointes:

Tap flag to report any problems with this question.


Question 711: Which of the following measures should be taken when
creatinine clearance reaches 50 ml/min?

Choices:
1. Dialysis
2. Fluid restriction
3. Diuresis
4. Dose adjustment
Answer: 4 - Dose adjustment
Explanations:
Many drugs are excreted through the kidneys, either as a metabolite or
unchanged
Dose adjustment is important when creatinine clearance reaches at or below
50 ml/min
End stage renal disease is reached when creatinine clearance is below
25ml/min
Fluid restriction, dialysis and diuresis are indicated in patients with renal
failure

Go to the next page if you knew the correct answer, or click the link images
below to further research the concepts in this question (if desired).

Research Concepts:
Renal Insufficiency:

Tap flag to report any problems with this question.


Question 712: In a patient with acute myocardial infarction (AMI),
thrombolytic door-to-drug administration should occur in no less than:

Choices:
1. 20 minutes
2. 30 minutes
3. 45 minutes
4. 1 hour
Answer: 2 - 30 minutes
Explanations:
AMI is a leading cause of morbidity and mortality in the United States
For AMI, door-to-drug time should be no longer than 30 minutes
Thrombolytic therapy improves survival in ST-segment elevation MI
It is not indicated in the treatment of non-ST-segment elevation MI

Go to the next page if you knew the correct answer, or click the link images
below to further research the concepts in this question (if desired).

Research Concepts:
Acute Myocardial Infarction:

Thrombolysis:

Tap flag to report any problems with this question.


Question 713: Which is appropriate for patient on metoprolol prior to
elective surgery?

Choices:
1. Stop preoperatively
2. Stop intraoperatively
3. Stop postoperatively
4. Continue medication
Answer: 4 - Continue medication
Explanations:
Beta-blockers should be continued throughout the procedure

Go to the next page if you knew the correct answer, or click the link images
below to further research the concepts in this question (if desired).

Research Concepts:
Preoperative Evaluation And Management:

Beta-Blockers:

Tap flag to report any problems with this question.


Question 714: What is the approximate 10 year patency of LIMA graft?
Choices:
1. 90%
2. 75%
3. 50%
4. <40%
Answer: 1 - 90%
Explanations:
The patency of the IMA at 10 years is about 90% or greater.
Use of the IMA confers a survival benefit, decreased episodes of MI,
decreased risk of hospitalization and decrease risk of reoperation.
The IMA is generally free of intimal thickening at 10 years and it is
believed that this is because the vaso vasorum is preserved.

Go to the next page if you knew the correct answer, or click the link images
below to further research the concepts in this question (if desired).

Research Concepts:
Internal Mammary Artery Bypass:

Tap flag to report any problems with this question.


Question 715: In a 60-year-old African American patient with persistently
elevated blood pressure, which of the following drug is recommended in
addition to hydrochlorothiazide?

Choices:
1. Amlodipine (Norvasc)
2. Lisinopril (Prinivil)
3. Metoprolol (Lopressor)
4. Losartan (Cozaar)
Answer: 1 - Amlodipine (Norvasc)
Explanations:
In African American patients, calcium channel blockers are more effective
at treating refractory hypertension in combination with hydrochlorothiazide.
Beta-blockers, ACE inhibitors and ARBs also treat hypertension and can be
combined with hydrochlorothiazide.

Go to the next page if you knew the correct answer, or click the link images
below to further research the concepts in this question (if desired).

Research Concepts:
Essential Hypertension:

Tap flag to report any problems with this question.


Question 716: What is the current standard of care for suspected
pulmonary embolus (PE)?

Choices:
1. Venography
2. Computed tomography angiography (CTA)
3. CXR
4. Ventilation-perfusion (V/Q) scanning
Answer: 2 - Computed tomography angiography (CTA)
Explanations:
The American College of Radiology considers chest CTA the current
standard of care for suspected PE
Venography is utilized in the diagnosis of DVT
A V/Q scan becomes more important when CT scanning is not available or
when there is a contraindication to IV contrast
CXR findings are nonspecific in the diagnosis of PE

Go to the next page if you knew the correct answer, or click the link images
below to further research the concepts in this question (if desired).

Research Concepts:
Pulmonary Embolism:

Tap flag to report any problems with this question.


Question 717: For aspiration of pericardial fluid the needle should be
inserted through which of the following intercostal spaces?

Choices:
1. Second intercostal space
2. Third intercostal space
3. Fifth intercostal space
4. Seventh intercostal space
Answer: 3 - Fifth intercostal space
Explanations:
Pericardial effusion is accumulation of fluid in the pericardial cavity.
Percutaneous drainage is most commonly done with ultrasound guidance.
The interspace between the fifth and sixth ribs is used as point for insertion.
More common sites are the subxiphoid or the left sternocostal margin
approach.

Go to the next page if you knew the correct answer, or click the link images
below to further research the concepts in this question (if desired).

Research Concepts:
Pericardiocentesis:

Tap flag to report any problems with this question.


Question 718: Mural thrombi are most likely seen in which of the
following conditions?

Choices:
1. Pericarditis
2. Acute myocardial infarction
3. Atrial fibrillation
4. Myxoma
Answer: 2 - Acute myocardial infarction
Explanations:
A mural thrombus is the formation of a thrombus that occurs in contact with
the endocardial lining of the cardiac chamber.
Mural thrombi commonly occur with an acute myocardial infarction.
Mural thrombi may resolve spontaneously, but they can embolize.
A mural thrombus can be confirmed with an echocardiogram.

Go to the next page if you knew the correct answer, or click the link images
below to further research the concepts in this question (if desired).

Research Concepts:
Mural Thrombi:

Tap flag to report any problems with this question.


Question 719: Which of the following is the most important risk factor for
atherosclerosis in people over 45?

Choices:
1. Smoking
2. Hypertension
3. Hyperlipidemia
4. Obesity
Answer: 2 - Hypertension
Explanations:
Over 45, hypertension is the most important risk factor while under 45;
hyperlipidemia is the most important risk factor
Other risk factors include smoking, diabetes mellitus, obesity, and sedentary
lifestyle

Go to the next page if you knew the correct answer, or click the link images
below to further research the concepts in this question (if desired).

Research Concepts:
Atherosclerosis:

Hypertension:

Tap flag to report any problems with this question.


Question 720: Which antihypertensive agent is associated with hemolytic
anemia?

Choices:
1. Alpha methyldopa
2. Clonidine
3. Propranolol
4. Hydralazine
Answer: 1 - Alpha methyldopa
Explanations:
Alpha methyldopa is known to cause hemolytic anemia.

Go to the next page if you knew the correct answer, or click the link images
below to further research the concepts in this question (if desired).

Research Concepts:
Hemolytic Anemia:

Antihypertensive Medications:

Tap flag to report any problems with this question.


Question 721: Which of the following is true regarding warfarin?
Choices:
1. In inhibits clotting factors II, VII and XII
2. The effectiveness of the dose is measured by the partial thromboplastin time
3. The effectiveness of the dose is measured by the INR
4. Protamine is used to reverse the effects of warfarin
Answer: 3 - The effectiveness of the dose is measured by the INR
Explanations:
Dosing of warfarin and the dependent on the INR level

Go to the next page if you knew the correct answer, or click the link images
below to further research the concepts in this question (if desired).

Research Concepts:
Warfarin:

Tap flag to report any problems with this question.


Question 722: What is the average intrinsic rate of atrioventricular node?
Choices:
1. 20 beats per minute
2. 40 beats per minute
3. 80 beats per minute
4. 160 beats per minute
Answer: 2 - 40 beats per minute
Explanations:
The intrinsic rate of the atrioventricular node is about 40 beats per minute

Go to the next page if you knew the correct answer, or click the link images
below to further research the concepts in this question (if desired).

Research Concepts:
Atrioventricular Node:

Tap flag to report any problems with this question.


Question 723: Examination of a patient shows visible pulsations of the
retinal arteries. This phenomenon is often seen in patients with which condition?

Choices:
1. Diabetic retinopathy
2. Melanoma of the eye
3. Aortic insufficiency
4. Von economo disease
Answer: 3 - Aortic insufficiency
Explanations:
Becker sign is observation of visible pulsations of retinal arteries.
This sign may be seen in patients with aortic regurgitation and Graves
disease.

Go to the next page if you knew the correct answer, or click the link images
below to further research the concepts in this question (if desired).

Research Concepts:
Aortic Insufficiency:

Tap flag to report any problems with this question.


Question 724: Excess use of sodium bicarbonate during CPR can result in
which of the following?

Choices:
1. Hyperosmolarity
2. Hypernatremia
3. Cerebral vasoconstriction
4. All of the above
Answer: 4 - All of the above
Explanations:
Currently, the use of sodium bicarbonate during CPR is not routine.
Evidence suggests that sodium bicarbonate can cause hyperosmolarity
hypernatremia, alkalosis, and hyperkalemia.
The alkalosis can result in central vasoconstriction, arrhythmias and left
shift of the oxygen/hemoglobin dissociation curve.
It is used to treat preexisting hyperkalemia or acidosis.

Go to the next page if you knew the correct answer, or click the link images
below to further research the concepts in this question (if desired).

Research Concepts:
Sodium Bicarbonate:

Tap flag to report any problems with this question.


Question 725: What medication has no interactions with warfarin?
Choices:
1. Captopril
2. Phenobarbital
3. Cimetidine
4. Ciprofloxacin
Answer: 1 - Captopril
Explanations:
Broad-spectrum antibiotics potentiate warfarin
ACE inhibitors do not interfere with warfarin
Phenobarbital decreases the effect of warfarin
Cimetidine increases the effect of warfarin

Go to the next page if you knew the correct answer, or click the link images
below to further research the concepts in this question (if desired).

Research Concepts:
Warfarin, Drug Interactions:

Tap flag to report any problems with this question.


Question 726: Which centrally acting antihypertensive diminishes
adrenergic outflow?

Choices:
1. Diltiazem
2. Hydralazine
3. Clonidine
4. Nitroprusside
Answer: 3 - Clonidine
Explanations:
Clonidine and alpha methyldopa are two centrally acting agonists.

Go to the next page if you knew the correct answer, or click the link images
below to further research the concepts in this question (if desired).

Research Concepts:
Clonidine:

Tap flag to report any problems with this question.


Question 727: ACE inhibitors can cause which of the following electrolyte
imbalances?

Choices:
1. Hypernatremia
2. Hyponatremia
3. Hyperkalemia
4. Hypokalemia
Answer: 3 - Hyperkalemia
Explanations:
Hyperkalemia is potential side effect ACE inhibitors
Inhibition of ACE and reduction in aldosterone production leads to
hyperkalemia
Most diuretics are associated with hypokalemia
Aldosterone antagonist medications also cause hyperkalemia

Go to the next page if you knew the correct answer, or click the link images
below to further research the concepts in this question (if desired).

Research Concepts:
ACE Inhibitors:

Hyperkalemia:

Tap flag to report any problems with this question.


Question 728: What is the preferred therapy for paroxysmal atrial
tachycardia which is unaffected by carotid massage?

Choices:
1. Immediate electrical cardioversion
2. IV digoxin
3. IV labetalol
4. IV adenosine
Answer: 4 - IV adenosine
Explanations:
Treatment of choice is adenosine, 6 mg IV, supraventricular tachycardia
followed by a 12 mg dose after a few minutes if needed
IV verapamil is an alternative at 2.5 to 5 mg followed by up to two
additional doses 10 minutes apart if needed
Cardioversion would be indicated if the patient was hemodynamically
unstable

Go to the next page if you knew the correct answer, or click the link images
below to further research the concepts in this question (if desired).

Research Concepts:
Paroxysmal Atrial Tachycardia:

Tap flag to report any problems with this question.


Question 729: Drug induced lupus is common in patients taking which of
the following medications?

Choices:
1. Pravastatin
2. Carvedilol
3. Nitroglycerin
4. Hydralazine
Answer: 4 - Hydralazine
Explanations:
Hydralazine can cause drug-induced lupus
Drug induced lupus is uniquely characterized by production of anti-histone
antibodies
Procainamide and isoniazid can also cause similar problem

Go to the next page if you knew the correct answer, or click the link images
below to further research the concepts in this question (if desired).

Research Concepts:
Hydralazine:

Lupus Erythematosus, Drug-Induced:

Tap flag to report any problems with this question.


Question 730: Which of the following is a frequent finding with patients
on synchronized intermittent mandatory ventilation?

Choices:
1. Pneumothorax
2. Hypercarbia
3. Increased work of breathing
4. Hypoxia
Answer: 3 - Increased work of breathing
Explanations:
Patients on SIMV often attempt spontaneous breathing despite a high
resistance circuit that increases the work of breathing
This can be prevented by setting a respiratory rate to decrease hypercarbia

Go to the next page if you knew the correct answer, or click the link images
below to further research the concepts in this question (if desired).

Research Concepts:
Synchronized Intermittent Mandatory Ventilation:

Tap flag to report any problems with this question.


Question 731: What causes high fever, tachycardia, and muscle rigidity
during halothane general anesthesia?

Choices:
1. Development of de novo Parkinson disease
2. Malignant hyperthermia syndrome
3. Sepsis
4. Seizure
Answer: 2 - Malignant hyperthermia syndrome
Explanations:
Malignant hyperthermia is an inherited predisposition triggered by
depolarizing muscle relaxants and some anesthetic agents including
halothane.
The condition can occur during anesthesia or a few hours after recovery.
Some patients develop malignant hypertension even though they have
received triggering agents in the past.
The pathophysiology involves reduction in the reuptake of calcium by the
sarcoplasmic reticulum. Termination of muscle contraction is thus impaired.

Go to the next page if you knew the correct answer, or click the link images
below to further research the concepts in this question (if desired).

Research Concepts:
Malignant Hyperthermia:

Tap flag to report any problems with this question.


Question 732: Increased resorption of salt and water by aldosterone occurs
at what portion of the nephron?

Choices:
1. proximal convoluted tubule
2. distal convoluted tubule
3. collecting duct
4. ascending straight tubule
Answer: 2 - distal convoluted tubule
Explanations:
Aldosterone binds to the cells of the collecting tubules inducing proteins
that increase the conductance of the luminal membrane at the sodium
channels
Aldosterone increased the number of Na+/K+ pump sites on the basolateral
membrane and increased formation of ATP by mitochondrial enzymes

Go to the next page if you knew the correct answer, or click the link images
below to further research the concepts in this question (if desired).

Research Concepts:
Aldosterone:

Renal Physiology:

Tap flag to report any problems with this question.


Question 733: What is the most appropriate test to confirm the diagnosis of
a patent ductus arteriosus?

Choices:
1. Chest x-ray
2. Echocardiogram
3. Quantitative serum prostaglandin assay
4. ECG
Answer: 2 - Echocardiogram
Explanations:
The most appropriate confirmatory test is the echocardiogram
It will also assess for associated cardiac pathology, which is essential in
planning operative care

Go to the next page if you knew the correct answer, or click the link images
below to further research the concepts in this question (if desired).

Research Concepts:
Patent Ductus Arteriosus:

Tap flag to report any problems with this question.


Question 734: All of the following are causes of pulseless electrical
activity (PEA) EXCEPT:

Choices:
1. Pulmonary embolus (PE)
2. Hyperglycemia
3. Hypovolemia
4. Acidosis
Answer: 2 - Hyperglycemia
Explanations:
Hypoglycemia, not hyperglycemia, can cause PEA.
PE, hypovolemia, and acidosis can cause PEA.
Other causes of PEA include hypoxia, tension pneumothorax, cardiac
tamponade, hypothermia, hyperkalemia, acute myocardial infarction, and
drug overdose.

Go to the next page if you knew the correct answer, or click the link images
below to further research the concepts in this question (if desired).

Research Concepts:
Pulseless Electrical Activity:

Tap flag to report any problems with this question.


Question 735: What is the term for ventricular tachycardia of varying
polarity with a wide QRS?

Choices:
1. Paroxysmal atrial tachycardia
2. Wolff-Parkinson-White
3. Torsade de pointes
4. Ventricular tachycardia
Answer: 3 - Torsade de pointes
Explanations:
Torsade is a polymorphic ventricular tachycardia characterized by gradual
changes in amplitude and twisting of the QRS complex.
Torsade is associated with a prolonged QT interval that may be acquired or
congenital.
Torsade can terminate spontaneously, or it can deteriorate into ventricular
fibrillation.

Go to the next page if you knew the correct answer, or click the link images
below to further research the concepts in this question (if desired).

Research Concepts:
Torsade de Pointes:

Tap flag to report any problems with this question.


Question 736: Emboli to the right coronary artery may be seen in what
ECG lead?

Choices:
1. I
2. V2
3. aVL
4. III
Answer: 4 - III
Explanations:
Right coronary artery supplies flow to the inferior portion of the heart.
Inferior leads II, III, and aVF are most likely to be affected.

Go to the next page if you knew the correct answer, or click the link images
below to further research the concepts in this question (if desired).

Research Concepts:
Inferior Wall Myocardial Infarction:

Tap flag to report any problems with this question.


Question 737: Which is true about Hamman's sign noted on palpation of
the chest?

Choices:
1. Holosystolic murmur
2. A sound heard after systole
3. Presence of air in the mediastinum
4. Crepitus at the lower lung bases
Answer: 3 - Presence of air in the mediastinum
Explanations:
Hamman's sign is heard over the mediastinum due to mediastinal
emphysema or pneumomediastinum.

Go to the next page if you knew the correct answer, or click the link images
below to further research the concepts in this question (if desired).

Research Concepts:
Pneumomediastinum:

Lung Exam:

Tap flag to report any problems with this question.


Question 738: The antibiotic associated with cartilage damage in children
is:

Choices:
1. Ampicillin
2. Levofloxacin
3. Trimethoprim sulfamethoxazole
4. Erythromycin
Answer: 2 - Levofloxacin
Explanations:
Quinolones can damage cartilage in children and are not recommended for
use in children less than 17

Go to the next page if you knew the correct answer, or click the link images
below to further research the concepts in this question (if desired).

Research Concepts:
Quinolones:

Tap flag to report any problems with this question.


Question 739: Which of the following is the best initial treatment for beta
blocker overdose?

Choices:
1. Phenylephrine
2. Ephedrine
3. Glucagon
4. Isoproterenol
Answer: 3 - Glucagon
Explanations:
Glucagon is first line in beta-blocker overdose along with IV fluids
Glucagon increases cAMP and this improves cardiac contractility, AV
conduction, and heart rate
IV calcium salts and vasopressors are alternate options

Go to the next page if you knew the correct answer, or click the link images
below to further research the concepts in this question (if desired).

Research Concepts:
Toxic, Beta-Blocker:

Tap flag to report any problems with this question.


Question 740: The pulmonary artery is derived from which structure?
Choices:
1. Aortic arch 3
2. Aortic arch 4
3. Aortic arch 5
4. Aortic arch 6
Answer: 4 - Aortic arch 6
Explanations:
Outgrowths of the sixth aortic arches become the proximal portions of the
pulmonary arteries.
The arteries of the right sixth aortic arch disappear.
The ductus arteriosus is derived from the left sixth aortic arch.

Go to the next page if you knew the correct answer, or click the link images
below to further research the concepts in this question (if desired).

Research Concepts:
Great Vessel Embryology:

Tap flag to report any problems with this question.


Question 741: What is the most common presentation of an infant with
tetralogy of Fallot?

Choices:
1. Dyspnea
2. Cyanosis
3. Flushed skin
4. Delayed radial-femoral pulse
Answer: 2 - Cyanosis
Explanations:
Tetralogy of Fallot is the most common cyanotic congenital heart disorder.
The majority of infants present with cyanosis in the first few months after
birth.
TOF consists of four anatomical defects including overriding aorta, VSD,
septal deviation anteriorl,y and right outflow tract obstruction. Today the
condition is readily diagnosed with ECHO at birth or during pregnancy.
The diagnosis is made by ECHO and surgery is performed on an elective
basis. Most surgeons prefer to do the surgery within the first year of life.

Go to the next page if you knew the correct answer, or click the link images
below to further research the concepts in this question (if desired).

Research Concepts:
Tetralogy Of Fallot:

Tap flag to report any problems with this question.


Question 742: The human body can easily develop tolerance to one of the
following medications:

Choices:
1. Prazosin
2. Methyldopa
3. Lisinopril
4. Nitroglycerin
Answer: 4 - Nitroglycerin
Explanations:
Nitroglycerin works by relaxing vascular smooth muscles
This occurs due to utilization of nitric oxide
Nitric oxide can be easily depleted from vascular smooth muscles, if
nitroglycerin is used continuously

Go to the next page if you knew the correct answer, or click the link images
below to further research the concepts in this question (if desired).

Research Concepts:
Nitroglycerin:

Tap flag to report any problems with this question.


Question 743: Which antihypertensive drug can lead to the development of
serious cardiac arrhythmias?

Choices:
1. Metoprolol
2. Clonidine
3. Captopril
4. Thiazide
Answer: 4 - Thiazide
Explanations:
Thiazide diuretics can decrease blood pressure and are often used in
combination therapy with beta-blockers or ACE inhibitors.
Thiazide diuretics are particularly useful in African American and elderly
patients.
Thiazide drugs can induce hypokalemia, hyperuricemia, and
hyperglycemia. Serum potassium levels should be closely monitored in
patients who are predisposed to cardiac arrhythmias (individuals with left
ventricular hypertrophy, coronary artery disease, and CHF) and who are
concurrently being treated with thiazide diuretics and digitalis.
Thiazide diuretics should be used with caution in the treatment of
hypertensive diabetics or patients with hyperglycemia.

Go to the next page if you knew the correct answer, or click the link images
below to further research the concepts in this question (if desired).

Research Concepts:
Diuretics:

Tap flag to report any problems with this question.


Question 744: Which is the best initial treatment of hypercalcemia?
Choices:
1. Magnesium
2. Calcium
3. Vitamin D
4. Fluid
Answer: 4 - Fluid
Explanations:
Hypocalcemia is characterized by muscle weakness, twitching,
bronchospasm, and possible seizure
Treatment involves calcium replacement
Hypercalcemia is initially treated with fluid replacement
Nausea and vomiting Loss of appetite Excessive thirst Frequent urination
Constipation Signs and symptoms of hypercalcemia include nausea,
vomiting, excessive thirst, constipation, frequent urination, abdominal pain,
muscle weakness, confusion, muscle and joint aches, confusion, lethargy,
fatigue

Go to the next page if you knew the correct answer, or click the link images
below to further research the concepts in this question (if desired).

Research Concepts:
Hypercalcemia:

Tap flag to report any problems with this question.


Question 745: A 71-year-old with congestive heart failure is admitted to
the intensive care unit. He is treated with diuretics, oxygen, and a nitroglycerin
drip. Over the next few days, he improves and is transferred to the floor. You
decide to place him on a drug class that has been shown to reduce mortality in
patients with congestive heart failure. What is the drug you use?

Choices:
1. Alpha blockers
2. Calcium channel blockers
3. Diuretics
4. ACE inhibitors
Answer: 4 - ACE inhibitors
Explanations:
ACE Inhibitors have been shown to reduce mortality and morbidity in
congestive heart failure.
ACE inhibitors reduce aldosterone secretion, salt and water retention, and
afterload.
ACE inhibitors are now considered first line drugs for congestive heart
failure.
ACE inhibitors lower arterial resistance and increase venous capacitance,
increase cardiac output, and lower renovascular resistance.

Go to the next page if you knew the correct answer, or click the link images
below to further research the concepts in this question (if desired).

Research Concepts:
Heart Failure, Congestive:

Tap flag to report any problems with this question.


Question 746: Which of the following will help reduce FI02 requirements
in a patient where it is high?

Choices:
1. Positive end expiratory pressure
2. Synchronized intermittent mandatory ventilation
3. Assist-control ventilation
4. Pressure-control ventilation
Answer: 1 - Positive end expiratory pressure
Explanations:
Alveolar recruitment causes decreased intrapulmonary shunting, increased
lung compliance and improved gas exchange
This can be accomplished by positive end expiratory pressure (PEEP)

Go to the next page if you knew the correct answer, or click the link images
below to further research the concepts in this question (if desired).

Research Concepts:
Positive End-Expiratory Pressure (PEEP):

Tap flag to report any problems with this question.


Question 747: Which effect is not seen with labetalol?
Choices:
1. Alpha 1 antagonist
2. Alpha 2 antagonist
3. Beta 1 antagonist
4. Beta 2 antagonist
Answer: 2 - Alpha 2 antagonist
Explanations:
Labetalol is an alpha 1, nonselective beta antagonist.

Go to the next page if you knew the correct answer, or click the link images
below to further research the concepts in this question (if desired).

Research Concepts:
Labetalol:

Tap flag to report any problems with this question.


Question 748: Constipation is associated with which of the following
medications?

Choices:
1. Castor oil
2. Bethanechol
3. Ferrous sulfate
4. Amikacin
Answer: 3 - Ferrous sulfate
Explanations:
Oral ferrous sulfate can cause constipation
Castor oil is an irritant laxative
Bethanechol also increases bowel motion
Amikacin cannot be absorbed from the gastrointestinal system

Go to the next page if you knew the correct answer, or click the link images
below to further research the concepts in this question (if desired).

Research Concepts:
Iron Supplementation:

Constipation:

Tap flag to report any problems with this question.


Question 749: What type of acute renal failure is associated with
hypovolemia?

Choices:
1. Prerenal
2. Intra-renal
3. Post renal
4. ESRD
Answer: 1 - Prerenal
Explanations:
Prerenal failure is associated with decrease blood supply to the kidneys
Intra renal failure is associated with nephrotoxicity
Post renal failure is common in obstructive disorder

Go to the next page if you knew the correct answer, or click the link images
below to further research the concepts in this question (if desired).

Research Concepts:
Acute Renal Failure:

Tap flag to report any problems with this question.


Question 750: A 67-year-old with multiple comorbidities undergoes
surgery. Which of the following is the best predictor of a poor outcome after
surgery?

Choices:
1. Age
2. Albumin levels less than 2.5 g/l
3. Renal failure
4. Weight greater than 120 kg
Answer: 2 - Albumin levels less than 2.5 g/l
Explanations:
Albumin has a half-life of 20 days and is often used as an inflammatory
marker.
Levels of albumin < 2.5 g/l are associated with high morbidity.
Low levels of albumin are usually seen in liver disease, nephritic syndrome,
burns, and starvation.

Go to the next page if you knew the correct answer, or click the link images
below to further research the concepts in this question (if desired).

Research Concepts:
Preoperative Evaluation And Management:

Tap flag to report any problems with this question.


Question 751: What is the most common cause of febrile non-hemolytic
reaction following blood transfusion?

Choices:
1. ABO antibody
2. ABO antigen
3. Leukocyte antigens
4. Blood contaminants
Answer: 3 - Leukocyte antigens
Explanations:
Febrile non-hemolytic reaction is caused by preformed leukocyte antigen

Go to the next page if you knew the correct answer, or click the link images
below to further research the concepts in this question (if desired).

Research Concepts:
Blood Transfusion:

Tap flag to report any problems with this question.


Question 752: A 55 year old man is started on digoxin. A common
complication of digoxin toxicity includes which of the following?

Choices:
1. Abdominal pain
2. Arrhythmia
3. Myalgia
4. Seizures
Answer: 2 - Arrhythmia
Explanations:
Digoxin toxicity has been associated with visual changes, bradycardia,
nausea, and arrhythmia.
Cardiac glycoside toxicity can be problematic due to its narrow therapeutic
window.
Digoxin is among the 50 most prescribed drugs in the United States.
Despite visual changes, pupils are spared.

Go to the next page if you knew the correct answer, or click the link images
below to further research the concepts in this question (if desired).

Research Concepts:
Digoxin Toxicity:

Tap flag to report any problems with this question.


Question 753: Which of the following leads to the greatest reduction in
myocardial oxygen during cardiopulmonary bypass?

Choices:
1. Hyperkalemic cardiac arrest
2. Hypothermic arrest
3. Fibrillating heart
4. Empty beating heart
Answer: 1 - Hyperkalemic cardiac arrest
Explanations:
The hyperkalemic arrested heart has the least oxygen requirement. The
highest requirements for oxygen are in the empty beating heart, followed by
the fibrillating heart.
Even though hypothermia decreases oxygen requirements, it is still not as
efficient as the potassium arrested heart in decreasing oxygen requirements.
Doing open heart surgery, inadequate oxygen to the heart results in
subendocardial damage with subsequent depression in function. The left
ventricular subendocardium is the most susceptible region of the left
ventricle because it can receive its blood supply only during diastole.
Conversely, the outer shell of the left ventricle and normal right ventricle
have lower compressive forces and can be perfused throughout the cardiac
cycle.

Go to the next page if you knew the correct answer, or click the link images
below to further research the concepts in this question (if desired).

Research Concepts:
Cardiopulmonary Bypass:

Tap flag to report any problems with this question.


Question 754: A complication of mitral valve surgery can be:
Choices:
1. Right sided heart failure
2. Clicking of the mechanical valve
3. Post-operative CVA
4. Lower exercise tolerance
Answer: 3 - Post-operative CVA
Explanations:
Mitral valve replacement can cause pieces of a calcified valve to move into
the brain causing stroke

Go to the next page if you knew the correct answer, or click the link images
below to further research the concepts in this question (if desired).

Research Concepts:
Mitral Valve Surgery, Complications:

Embolic Stroke:

Tap flag to report any problems with this question.


Question 755: You are likely to see digoxin toxicity in a patient with which
of the following?

Choices:
1. Hypercalcemia
2. Hypocalcemia
3. Hyperkalemia
4. Hypokalemia
Answer: 4 - Hypokalemia
Explanations:
Hypokalemia enhances digoxin toxicity.

Go to the next page if you knew the correct answer, or click the link images
below to further research the concepts in this question (if desired).

Research Concepts:
Hypokalemia:

Tap flag to report any problems with this question.


Question 756: When should tissue plasminogen activator (t-PA) be
administered after an acute MI?

Choices:
1. Within 1 hour
2. Within 3 hours
3. Within 5 hours
4. Within 6 hours
Answer: 4 - Within 6 hours
Explanations:
It is recommended that t-PA be administered within 6 hours of an acute MI.

Go to the next page if you knew the correct answer, or click the link images
below to further research the concepts in this question (if desired).

Research Concepts:
Myocardial Infarction, Acute:

Tissue Plasminogen Activator:

Tap flag to report any problems with this question.


Question 757: Symptoms of aortic stenosis include all of the following
except:

Choices:
1. Pulsus parvus
2. Malar flush
3. Pulsus tardus
4. Increased murmur with squatting
Answer: 2 - Malar flush
Explanations:
Other findings to in aortic stenosis include a decreased carotid pulse.
Malar flush occurs with mitral stenosis.
Syncope, angina, and dyspnea on exertion are also prevalent symptoms.

Go to the next page if you knew the correct answer, or click the link images
below to further research the concepts in this question (if desired).

Research Concepts:
Aortic Stenosis:

Tap flag to report any problems with this question.


Question 758: Which of the following statements regarding propofol is not
true?

Choices:
1. It is a hypnotic agent
2. It is commonly used for induction of anesthesia
3. Recovery from propofol is fast
4. It has analgesic properties
Answer: 4 - It has analgesic properties
Explanations:
Propofol is a common hypnotic agent used for induction of general
anesthesia
Opioids such as fentanyl may be combined with propofol to alleviate pain
Propofol is not considered an analgesic

Go to the next page if you knew the correct answer, or click the link images
below to further research the concepts in this question (if desired).

Research Concepts:
Propofol:

Tap flag to report any problems with this question.


Question 759: A 60 year old man experienced substernal chest pain 3 days
ago while camping. Select the test that would best detect myocardial damage.

Choices:
1. Myoglobin
2. Creatine kinase
3. Troponin I
4. Creatine kinase, MB
Answer: 3 - Troponin I
Explanations:
Troponin I will remain positive for up to a week after myocardial damage
CK(MB) remains positive for 2 to 3 days
Myoglobin is only elevated for 1 to 2 days and is very nonspecific

Go to the next page if you knew the correct answer, or click the link images
below to further research the concepts in this question (if desired).

Research Concepts:
Chest Pain:

Tap flag to report any problems with this question.


Question 760: During inspiration, there is a usually a decrease in what
variable?

Choices:
1. Right ventricular end diastolic volume
2. Alveolar pressure
3. Right atrium volume
4. Left ventricular pressure
Answer: 4 - Left ventricular pressure
Explanations:
Left ventricular pressure decrease with inspiration

Go to the next page if you knew the correct answer, or click the link images
below to further research the concepts in this question (if desired).

Research Concepts:
Cardiac Physiology:

Tap flag to report any problems with this question.


Question 761: What percentage of people who stop smoking will relapse
within 3 months?

Choices:
1. 25%
2. 45%
3. 65%
4. 85%
Answer: 3 - 65%
Explanations:
Sixty-five percent of people who stop smoking will relapse within 3 months

Go to the next page if you knew the correct answer, or click the link images
below to further research the concepts in this question (if desired).

Research Concepts:
Smoking Cessation:

Tap flag to report any problems with this question.


Question 762: What is the treatment for respiratory depression caused by
midazolam overdose?

Choices:
1. Naloxone
2. Naltrexone
3. Flumazenil
4. Fentanyl
Answer: 3 - Flumazenil
Explanations:
Overdose of benzodiazepines is treated with flumazenil.
Naloxone is an antidote for opiate overdose.
It is important to continue with resuscitation until there is full recovery.
Fentanyl can worsen respiratory depression if administered with
benzodiazepines.

Go to the next page if you knew the correct answer, or click the link images
below to further research the concepts in this question (if desired).

Research Concepts:
Flumazenil:

Toxic, Benzodiazepine:

Tap flag to report any problems with this question.


Question 763: Which anticoagulants can cause heparin induced
thrombocytopenia (HIT)?

Choices:
1. Lepirudin
2. Argatroban
3. Low molecular weight heparin (LMWH)
4. Fondaparinux
Answer: 3 - Low molecular weight heparin (LMWH)
Explanations:
LMWH has heparin, can contribute to immune mediated HIT by forming
immune complexes

Go to the next page if you knew the correct answer, or click the link images
below to further research the concepts in this question (if desired).

Research Concepts:
Thrombocytopenia, Heparin Induced:

Tap flag to report any problems with this question.


Question 764: What is not an anatomical defect in tetralogy of Fallot?
Choices:
1. Ventricular septal defect
2. Overriding aorta
3. Pulmonary hypertension
4. Ventricular hypertrophy
Answer: 3 - Pulmonary hypertension
Explanations:
The tetralogy of Fallot is pulmonary artery obstruction, overriding of the
aorta, right ventricular hypertrophy, and VSD

Go to the next page if you knew the correct answer, or click the link images
below to further research the concepts in this question (if desired).

Research Concepts:
Tetralogy Of Fallot:

Tap flag to report any problems with this question.


Question 765: The best approach to repair of postinfarct ventricular septal
rupture is:

Choices:
1. Longitudinal right ventriculotomy
2. Transventricular repair via infarct
3. Repair ventricular septal repair via right atrium
4. Longitudinal left ventriculotomy
Answer: 2 - Transventricular repair via infarct
Explanations:
The VSR is always approached via a transventricular approach through the
infarct zone
The defect in the septum is usually found immediately beneath this
The infarct zone is excised and a patch is used to close the defect
When the VSR is located in the apex, it consists of amputation of the apex
of the ventricle, including the involved portion of the ventricular septum

Go to the next page if you knew the correct answer, or click the link images
below to further research the concepts in this question (if desired).

Research Concepts:
Post Infarct Ventricular Septal Rupture:

Tap flag to report any problems with this question.


Question 766: A 78 year old female with a history of coronary artery
disease and is found to have an ST segment myocardial infarction with classic
symptoms. Within 3 hours the patient is stable and pain free. Which
complication would be of greatest concern over the next 24 hours?

Choices:
1. Septal rupture
2. Ventricular arrhythmia
3. Cardiogenic shock
4. Atrial fibrillation
Answer: 2 - Ventricular arrhythmia
Explanations:
Ventricular arrhythmias occur in up to 75% of patients within the first 72
hours after myocardial infarction, mostly in the first day
Septal rupture is possible but much less common
Atrial fibrillation is also a possibility but is not the major concern
An extension of the infarction or a second MI could cause cardiogenic
shock but this is only seen in 10% of patients

Go to the next page if you knew the correct answer, or click the link images
below to further research the concepts in this question (if desired).

Research Concepts:
Acute Myocardial Infarction:

Tap flag to report any problems with this question.


Question 767: Which of the following is the most likely complication of
hyperkalemia?

Choices:
1. Pulmonary edema
2. Stroke
3. Arrhythmias
4. Hemolysis
Answer: 3 - Arrhythmias
Explanations:
Hyperkalemia is defined as a serum potassium > 5.5 mEq/L and may cause
cardiac dysrhythmias and sudden death.

Go to the next page if you knew the correct answer, or click the link images
below to further research the concepts in this question (if desired).

Research Concepts:
Hyperkalemia:

Tap flag to report any problems with this question.


Question 768: Which of the following does not act as beta-lactamase
inhibitors?

Choices:
1. Tazobactam
2. Sulbactam
3. Clavulanic acid
4. Ticarcillin
Answer: 4 - Ticarcillin
Explanations:
A beta-lactamase inhibitors are given with a beta-lactam antibiotics.
The three in common use are clavulanic acid, tazobactam, and sulbactam.
The inhibitors do not usually have significant antibiotic activity. They
inhibit beta-lactamase activity prevent inactivation of beta lactam
antibiotics.
Ticarcillin is a beta-lactam antibiotics.

Go to the next page if you knew the correct answer, or click the link images
below to further research the concepts in this question (if desired).

Research Concepts:
Beta Lactamase Inhibitors:

Tap flag to report any problems with this question.


Question 769: Which of the following conditions is not associated with an
elevated pulmonary capillary wedge pressure?

Choices:
1. Mitral stenosis
2. Mitral regurgitation
3. Endotracheal intubation
4. Tricuspid stenosis
Answer: 4 - Tricuspid stenosis
Explanations:
Pulmonary pressure measures left sided diseases.

Go to the next page if you knew the correct answer, or click the link images
below to further research the concepts in this question (if desired).

Research Concepts:
Mitral Stenosis:

Swan-Ganz Catheterization:

Tricuspid Stenosis:

Tap flag to report any problems with this question.


Question 770: Which is not a feature of cardiac tamponade?
Choices:
1. Hypotension
2. Wide pulse pressure
3. Distant heart sound
4. Low voltage QRS
Answer: 2 - Wide pulse pressure
Explanations:
Cardiac tamponade is a surgical emergency.
The disorder often presents with hypotension, narrow pulse pressure and
elevated JVP.
Other findings include low voltage QRS and distant heart sounds.
Pulsus paradoxus may be present.

Go to the next page if you knew the correct answer, or click the link images
below to further research the concepts in this question (if desired).

Research Concepts:
Cardiac Tamponade:

Tap flag to report any problems with this question.


Question 771: Common symptoms of aortic regurgitation may include all
of the following except?

Choices:
1. Fatigue
2. Orthopnea
3. Dyspnea
4. Ataxia
Answer: 4 - Ataxia
Explanations:
The common symptoms of aortic regurgitation are caused by left-sided
failure.
Many patients may initially have no symptoms.
Deterioration may occur before symptoms develop, so serial
echocardiograms on these patients may be indicated.
The prevention of permanent cardiac damage may include surgery before
left ventricular dysfunction deteriorates.

Go to the next page if you knew the correct answer, or click the link images
below to further research the concepts in this question (if desired).

Research Concepts:
Aortic Regurgitation:

Tap flag to report any problems with this question.


Question 772: Nosocomial pneumonia in patient on mechanical ventilation
is most likely caused by:

Choices:
1. Gram positive cocci
2. Gram negative cocci
3. Gram positive rods
4. Gram negative rods
Answer: 4 - Gram negative rods
Explanations:
Gram negative rods are the most common cause of nosocomial pneumonia
in patient on mechanical ventilation

Go to the next page if you knew the correct answer, or click the link images
below to further research the concepts in this question (if desired).

Research Concepts:
Nosocomial Pneumonia:

Mechanical Ventilation:

Tap flag to report any problems with this question.


Question 773: Beta blocker therapy is indicated for angina because:
Choices:
1. It reduces the preload
2. It reduces the afterload
3. It reduces cardiac output
4. It reduces heart rate
Answer: 4 - It reduces heart rate
Explanations:
Beta-blockers reduce heart rate and indirectly improve coronary circulation
Coronary circulation mainly occurs in diastole
Beta-blockers do not directly affect preload and afterload
Nitroglycerin reduces preload and alleviates the workload on the heart

Go to the next page if you knew the correct answer, or click the link images
below to further research the concepts in this question (if desired).

Research Concepts:
Beta-Blockers:

Angina, Stable:

Angina, Unstable:
Tap flag to report any problems with this question.
Question 774: The first heart sound usually correlates with what wave on
the jugular pressure curve?

Choices:
1. V wave
2. C wave
3. Y descent
4. X descent
Answer: 2 - C wave
Explanations:
The first heart sound correlates with the C wave.
The C wave usually signifies closure of the tricuspid valve.

Go to the next page if you knew the correct answer, or click the link images
below to further research the concepts in this question (if desired).

Research Concepts:
Heart Sounds:

Jugular Venous Pulsation:

Tap flag to report any problems with this question.


Question 775: What is a normal body pH?
Choices:
1. 7
2. 7.4
3. 8
4. 8.5
Answer: 2 - 7.4
Explanations:
Normal pH ranges from 7.35 - 7.45.

Go to the next page if you knew the correct answer, or click the link images
below to further research the concepts in this question (if desired).

Research Concepts:
Acid-base Balance:

Tap flag to report any problems with this question.


Question 776: Bilateral transection of which of the following nerves can
result in death of a patient?

Choices:
1. Ulnar nerve
2. Trigeminal nerve
3. Facial nerve
4. Phrenic nerve
Answer: 4 - Phrenic nerve
Explanations:
Bilateral phrenic nerve transection would cause paralysis of the diaphragm
and lead to death
Unilateral phrenic nerve transection is usually tolerated
Prognosis is dependent on underlying pulmonary status

Go to the next page if you knew the correct answer, or click the link images
below to further research the concepts in this question (if desired).

Research Concepts:
Phrenic Nerve Transection:

Tap flag to report any problems with this question.


Question 777: A 58-year-old is found to have a prostate mass and
ultrasound biopsy reveals an adenocarcinoma. The urologist feels that the best
treatment is a radical prostatectomy. During surgery, there is significant bleeding
and immediate blood work by the anesthesiologist reveals that the patient has a
hemoglobin of 9, platelet count of 156, prothrombin time of 11.2, partial
thromboplastin time of 41, and an international normalized ratio of 1.4. He is
also found to have excessive activation of plasminogen to plasmin. Which of the
following is the best therapy to counter the bleeding?

Choices:
1. Aminocaproic acid
2. Fresh frozen plasma
3. Vitamin K
4. Platelet transfusion
Answer: 1 - Aminocaproic acid
Explanations:
Anti-plasmin agents are useful in open-heart surgery where excessive
plasmin is generated.
Drugs such as aminocaproic acid or tranexamic acid are useful and inhibit
the fibrinolysis by inhibiting plasminogen activation.

Go to the next page if you knew the correct answer, or click the link images
below to further research the concepts in this question (if desired).

Research Concepts:
Fibrinolysis:

Anti-Plasmin Agents:

Tap flag to report any problems with this question.


Question 778: Which is not true regarding adrenal crisis?
Choices:
1. May present with illness or surgery
2. Has a risk of cardiovascular collapse and death
3. Hypertension is a principal feature
4. May occur with abrupt withdrawal of corticosteroids
Answer: 3 - Hypertension is a principal feature
Explanations:
Hypotension is a principal feature

Go to the next page if you knew the correct answer, or click the link images
below to further research the concepts in this question (if desired).

Research Concepts:
Adrenal Crisis:

Tap flag to report any problems with this question.


Question 779: Which of the following is a true statement about blood
cultures in a patient with endocarditis?

Choices:
1. They are always positive
2. They should be obtained 3-4 times in the first 24 hrs
3. They should be obtained after antibiotic therapy
4. They are usually negative
Answer: 2 - They should be obtained 3-4 times in the first 24 hrs
Explanations:
Dukes' criteria for establishing infectious endocarditis include two major
criteria and three minor criteria. Positive blood cultures from 2 separate
occasions should be drawn at least 12 hours apart.
An echocardiogram should reveal an oscillating mass on the valve in the
path of a regurgitant jet. In the absence of an alternate anatomic
explanation, the echo should reveal implanted material.
Minor Dukes' criteria include fever, evidence of embolism, pulmonary
infarcts, Janeway lesions, and conjunctival hemorrhage.
Other features of endocarditis include glomerulonephritis, Osler nodes, and
a positive echocardiogram.

Go to the next page if you knew the correct answer, or click the link images
below to further research the concepts in this question (if desired).

Research Concepts:
Endocarditis, Infectious:

Tap flag to report any problems with this question.


Question 780: Which of the following is the correct heparin loading dose
in acute MI?

Choices:
1. 100 U/hr
2. 55 U/kg
3. 75 U/kg
4. 13 U/kg
Answer: 3 - 75 U/kg
Explanations:
The correct heparin loading dose is 75 U/kg follow by maintenance of 13
U/kg/hr

Go to the next page if you knew the correct answer, or click the link images
below to further research the concepts in this question (if desired).

Research Concepts:
Heparin:

Acute Myocardial Infarction:

Tap flag to report any problems with this question.


Question 781: Which one of the following is used for treatment of
pulmonary hypertension?

Choices:
1. Bosentan
2. Prostaglandin E
3. Alprostadil
4. Latanoprost
Answer: 4 - Latanoprost
Explanations:
Latanoprost is a prostaglandin analog used for glaucoma
Nifedipine, diltiazem, iloprost, and treprostinil are also used
Prostaglandin E suppositories are used to induce abortion
Bosentan is used in pulmonary hypertension

Go to the next page if you knew the correct answer, or click the link images
below to further research the concepts in this question (if desired).

Research Concepts:
Bosentan:

Pulmonary Hypertension, Primary:

Tap flag to report any problems with this question.


Question 782: Fever in the first 24 hours after surgery is usually due to
which of the following?

Choices:
1. Infection
2. Atelectasis
3. DVT
4. UTI
Answer: 2 - Atelectasis
Explanations:
Fever in post operative day one is most likely to be pulmonary related
Poor lung expansion mainly due to pain and bed rest

Go to the next page if you knew the correct answer, or click the link images
below to further research the concepts in this question (if desired).

Research Concepts:
Postoperative Management:

Atelectasis:

Tap flag to report any problems with this question.


Question 783: Which of the following features is most helpful in
distinguishing ileus from bowel obstruction?

Choices:
1. Pain
2. Vomiting
3. Nausea
4. Bowel sounds
Answer: 4 - Bowel sounds
Explanations:
Hyperactive bowel sounds are consistent with obstruction, whereas with
ileus bowel sounds are generally absent.

Go to the next page if you knew the correct answer, or click the link images
below to further research the concepts in this question (if desired).

Research Concepts:
Ileus:

Tap flag to report any problems with this question.


Question 784: What is the most appropriate management of anaphylactoid
reaction following penicillin administration?

Choices:
1. Antihistamines
2. Topical steroids
3. Epinephrine
4. Topical antipruritics
Answer: 3 - Epinephrine
Explanations:
The primary therapy is to discontinue the offending agent
The remaining therapies all can be helpful depending on the extent and
severity of the reaction

Go to the next page if you knew the correct answer, or click the link images
below to further research the concepts in this question (if desired).

Research Concepts:
Anaphylaxis:

Tap flag to report any problems with this question.


Question 785: Which is false about aldosterone?
Choices:
1. Is secreted when the blood pressure decreases
2. Is a polypeptide hormone
3. Can increase concentration of angiotensin II
4. Aldosterone stimulates uptake of potassium into cells
Answer: 2 - Is a polypeptide hormone
Explanations:
Aldosterone is a steroid hormone produced in the zona glomerulosa of the
adrenal gland. Its functions to case conservation of sodium and secretion of
water
Levels of aldosterone are increased when the blood volume or blood
pressure is low
The hormone can increase plasma concentration of angiotensin II, causes
potassium uptake into cells and stimulated sodium reabsorption from the
gut and salivary glands
Aldosterone secretion is stimulated by the stretch receptors of the atrium
and plasma acidosis. Levels of aldosterone are low in Addison's disease and
increased in Conn syndrome

Go to the next page if you knew the correct answer, or click the link images
below to further research the concepts in this question (if desired).

Research Concepts:
Aldosterone:

Tap flag to report any problems with this question.


Question 786: What is the most common presentation of an infant with
tricuspid atresia?

Choices:
1. Congestive heart failure
2. Cyanosis
3. Decreased peripheral perfusion
4. Absent lower extremity pulses
Answer: 2 - Cyanosis
Explanations:
Cyanosis is the most common presentation of tricuspid atresia.
The degree of cyanosis depends on the amount of blood flow to the
pulmonary circulation.
Digital clubbing is also a common feature of the disorder.
Other findings include JVP pulsations, decreased peripheral pulses, and
hyperdynamic apical impulse.

Go to the next page if you knew the correct answer, or click the link images
below to further research the concepts in this question (if desired).

Research Concepts:
Tricuspid Atresia:

Tap flag to report any problems with this question.


Question 787: Which of the following is a risk factor for aortic dissection?
Choices:
1. Connective tissue disease
2. Pregnancy
3. Trauma
4. All of the above
Answer: 4 - All of the above
Explanations:
Connective tissue disease, such as Marfan's, pregnancy, and trauma,
especially acceleration injuries, are risk factors for aortic dissection

Go to the next page if you knew the correct answer, or click the link images
below to further research the concepts in this question (if desired).

Research Concepts:
Aortic Dissection:

Tap flag to report any problems with this question.


Question 788: In a symptomatic patient with aortic stenosis (AS), what is
the treatment of choice?

Choices:
1. Furosemide
2. Balloon dilatation
3. Emergent surgery
4. Stress test
Answer: 3 - Emergent surgery
Explanations:
In all symptomatic patients with AS, surgery is indicated.
If the symptoms persist, there is a 50% chance of death within 2-3 years.
Aortic stenosis is best treated with a valve replacement.
Valve repair techniques do not work for the aortic valve.

Go to the next page if you knew the correct answer, or click the link images
below to further research the concepts in this question (if desired).

Research Concepts:
Aortic Stenosis:

Tap flag to report any problems with this question.


Question 789: Which of the following cephalosporin is not associated with
defective coagulation?

Choices:
1. Cefoperazone
2. Cefotetan
3. Cefazolin
4. Cefoxitin
Answer: 4 - Cefoxitin
Explanations:
Cefoperazone, cefazolin, and cefotetan may cause hypoprothrombinemia
and disulfiram like reactions.

Go to the next page if you knew the correct answer, or click the link images
below to further research the concepts in this question (if desired).

Research Concepts:
Cephalosporins:

Hypoprothrombinemia:

Tap flag to report any problems with this question.


Question 790: A stable postoperative patient with hypertension is treated
with a vasodilator. The patient appears cyanotic despite adequate blood gas. The
chest x-ray is unremarkable. What is the most likely cause of this patient's
presentation?

Choices:
1. Severe vasoconstriction
2. Methemoglobinemia
3. Pulmonary edema
4. Bleeding
Answer: 2 - Methemoglobinemia
Explanations:
Excess nitroglycerin can cause methemoglobinemia, which can be fatal if
left undiagnosed.
Treatment involves administration of methylene blue or N acetyl cysteine.

Go to the next page if you knew the correct answer, or click the link images
below to further research the concepts in this question (if desired).

Research Concepts:
Methemoglobinemia:

Tap flag to report any problems with this question.


Question 791: Pharmacologic stress testing with adenosine or
dipyridamole is contraindicated in with which of the following conditions?

Choices:
1. Morbid obesity
2. Valvular cardiac disease
3. Hypertension
4. Bronchospasm
Answer: 4 - Bronchospasm
Explanations:
The administration of adenosine or dipyridamole may precipitate acute
bronchospasm and is contraindicated with asthma or other bronchospastic
diseases.
They cause dilation and increased blood flow in normal coronary arteries,
but not in diseased, stenotic coronary arteries.
They are used for pharmacologic stress testing in patients who cannot
tolerate physical stress, such as individuals with severe emphysema or
morbid obesity.

Go to the next page if you knew the correct answer, or click the link images
below to further research the concepts in this question (if desired).

Research Concepts:
Pharmacologic Stress Testing:

Bronchospasm:

Tap flag to report any problems with this question.


Question 792: What protective measure should be undertaken when
performing tracheal suctioning on a patient requiring ventilation?

Choices:
1. Face mask
2. Gloves
3. Eye protection
4. All of the above
Answer: 4 - All of the above
Explanations:
A facemask should be worn for protection when performing tracheal
suctioning
Gloves should also be worn for protection
Eye protection should also be worn
Sterile technique should be observed when performing tracheal suctioning

Go to the next page if you knew the correct answer, or click the link images
below to further research the concepts in this question (if desired).

Research Concepts:
Mechanical Ventilation:

Nasotracheal Suctioning:

Tap flag to report any problems with this question.


Question 793: Nitroglycerin is not available as?
Choices:
1. Sublingual
2. Transdermal
3. Rectal suppository
4. Intravenous
Answer: 3 - Rectal suppository
Explanations:
Nitroglycerin is supplied in sublingual, transdermal, oral spray, and
intravenous form
Must keep away from direct light

Go to the next page if you knew the correct answer, or click the link images
below to further research the concepts in this question (if desired).

Research Concepts:
Nitroglycerin:

Tap flag to report any problems with this question.


Question 794: Which of the following is not true of pericardial effusions?
Choices:
1. Pericardial volume and pressure are linearly related
2. Uremia may be associated with both pericarditis and effusion
3. Tuberculous pericarditis may be calcific
4. In cardiac tamponade, the filling of the heart is limited during diastole
Answer: 1 - Pericardial volume and pressure are linearly related
Explanations:
Acute tamponade may be caused by a very small blood clot and is not
directly related to the volume.
In chronic cases, the pericardium can be stretched and an increase in
volume is tolerated. As the pressure in the pericardial sac is increased, the
diastolic filling pressure also increases.
Compensatory mechanisms include increased sympathetic activity,
increased vasoconstriction, and increased heart rate.
The venous pressure rise and coronary flow is decreased with a decrease in
cardiac output.

Go to the next page if you knew the correct answer, or click the link images
below to further research the concepts in this question (if desired).

Research Concepts:
Pericardial Effusion:

Tap flag to report any problems with this question.


Question 795: Coronary artery fistulas arise most commonly from which
of the following?

Choices:
1. Right coronary artery
2. Left coronary artery
3. Circumflex
4. Left main
Answer: 1 - Right coronary artery
Explanations:
The majority of coronary artery fistulas arise from the RCA.

Go to the next page if you knew the correct answer, or click the link images
below to further research the concepts in this question (if desired).

Research Concepts:
Coronary Artery Fistula:

Tap flag to report any problems with this question.


Question 796: A 50 year old patient is evaluated for a mitral stenosis and
echocardiogram shows estimation of the valve area to be1.7 cm2. This is
consistent with mild disease. Select the test that would be indicated to make the
decision to recommend surgical correction.

Choices:
1. Nuclear medicine stress test
2. Right heart catheterization
3. Left hear catheterization with coronary angiogram
4. Holter monitor
Answer: 2 - Right heart catheterization
Explanations:
Indications for surgical repair for mitral stenosis include pulmonary
hypertension and systemic embolization
A young patient would not need stress testing or coronary artery
angiography
Holter monitoring might detect atrial fibrillation

Go to the next page if you knew the correct answer, or click the link images
below to further research the concepts in this question (if desired).

Research Concepts:
Mitral Stenosis:

Tap flag to report any problems with this question.


Question 797: Which of the following is the most important aspect of
compliance for individuals who are about to undergo mechanical valve
replacement?

Choices:
1. Diet
2. Dental care
3. Medications
4. Keeping appointments
Answer: 3 - Medications
Explanations:
Postoperatively, all patients with mechanical valves and some patients with
bioprostheses are maintained indefinitely on anticoagulant therapy

Go to the next page if you knew the correct answer, or click the link images
below to further research the concepts in this question (if desired).

Research Concepts:
Aortic Valve:

Mitral Valve Replacement:

Tap flag to report any problems with this question.


Question 798: Select the true statement about antibiotic prophylaxis
regimen for patients prosthetic heart valves.

Choices:
1. Antibiotic should be administered only before a procedure
2. Penicillin is the choice for dental procedures
3. Dermatologic procedure requires clindamycin
4. Erythromycin is an option for TURP
Answer: 1 - Antibiotic should be administered only before a procedure
Explanations:
Antibiotic prophylaxis is administered before a procedure to prevent
endocarditis
Procedures on infected skin should be treated with an antistaphylococcal
antibiotic or cephalosporin
TURP does not require prophylaxis
Clindamycin can be used for patients with allergy to penicillin

Go to the next page if you knew the correct answer, or click the link images
below to further research the concepts in this question (if desired).

Research Concepts:
Prosthetic Heart Valve:

Subacute Bacterial Endocarditis (SBE) Prophylaxis:

Tap flag to report any problems with this question.


Question 799: What is the one major benefit of using tacrolimus over
cyclosporine?

Choices:
1. Tacrolimus causes less neurotoxicity
2. Tacrolimus does not induce seizures
3. Tacrolimus does not cause hirsutism
4. Dose levels of tacrolimus do not require monitoring
Answer: 3 - Tacrolimus does not cause hirsutism
Explanations:
Tacrolimus is more toxic than cyclosporine in certain cases.
Tacrolimus is more nephrotoxic and neurotoxic than cyclosporine.
Tacrolimus should not be used in individuals who have a history of
seizures.
The one benefit of tacrolimus is that it does not cause gingival hyperplasia
or hirsutism.

Go to the next page if you knew the correct answer, or click the link images
below to further research the concepts in this question (if desired).

Research Concepts:
Liver Transplantation:

Kidney Transplantation:

Heart Transplantation:
Tap flag to report any problems with this question.
Question 800: How much does blood pressure drop with postprandial
hypotension?

Choices:
1. 5 mmHg
2. 10 mmHg
3. 15 mmHg
4. 20 mmHg
Answer: 4 - 20 mmHg
Explanations:
Blood pressure can drop 20 mm Hg
Avoid large meal

Go to the next page if you knew the correct answer, or click the link images
below to further research the concepts in this question (if desired).

Research Concepts:
Postprandial Hypotension:

Tap flag to report any problems with this question.


Section 5
Question 801: What is the most common cause of death during a blood
transfusion?

Choices:
1. Hyperkalemia
2. Transfusion-related acute lung injury
3. Hemolytic reactions
4. Metabolic acidosis
Answer: 1 - Hyperkalemia
Explanations:
Transfusion-associated metabolic reactions can occur but are most common
during massive transfusion.
The most common cause of death in the US from transfusions is secondary
to transfusion-related acute lung injury and occurs in 0.04-0.16% of patients
transfused.
Fatal hemolytic reactions occur in 1 case per 250,000-600,000 .
Hyperkalemia is due to potassium leakage in stored blood. Massive blood
transfusion can result in death if the patient is acidotic.

Go to the next page if you knew the correct answer, or click the link images
below to further research the concepts in this question (if desired).

Research Concepts:
Transfusion-related Acute Lung Injury:

Blood Transfusion:

Tap flag to report any problems with this question.


Question 802: Which of the following is not a typical side effect of I.V.
lidocaine?

Choices:
1. Tinnitus
2. Dizziness
3. Numbness of the extremities
4. Palpitations
Answer: 4 - Palpitations
Explanations:
Common side effects of lidocaine include tinnitus, dizziness, tremors, and
blurred vision
The initial symptoms of toxicity may include circumoral numbness, tingling
of extremities, diaphoresis and hypotension
At high doses, seizures and coma can occur

Go to the next page if you knew the correct answer, or click the link images
below to further research the concepts in this question (if desired).

Research Concepts:
Lidocaine:

Side Effects:

Tap flag to report any problems with this question.


Question 803: In a surgical incision, epithelium usually forms within:
Choices:
1. 24 hours
2. 36 hours
3. 48 hours
4. 72 hours
Answer: 1 - 24 hours
Explanations:
Epithelium is formed within 24 hours after surgery

Go to the next page if you knew the correct answer, or click the link images
below to further research the concepts in this question (if desired).

Research Concepts:
Wound Healing:

Epithelialization:

Tap flag to report any problems with this question.


Question 804: What is a normal PvO2 level?
Choices:
1. 20 mmHg
2. 30 mmHg
3. 40 mmHg
4. 50 mmHg
Answer: 3 - 40 mmHg
Explanations:
PvO2 is a sample from the pulmonary artery
It is an indicator of mixed venous oxygen level
Normal values are between 35 to 40 mmHg

Go to the next page if you knew the correct answer, or click the link images
below to further research the concepts in this question (if desired).

Research Concepts:
Pulmonary Artery Catheterization:

Tap flag to report any problems with this question.


Question 805: A patient involved in a motor vehicle accident has a
traumatic injury to his proximal descending aorta. Which of the following is
currently not an acceptable technique in a stable patient?

Choices:
1. Clamp and sew
2. Femoral-femoral bypass
3. Partial left heart bypass
4. Cardiopulmonary bypass, hypothermia, and circulatory arrest
Answer: 1 - Clamp and sew
Explanations:
The clamp and sew technique is acceptable if the patient is unstable.
However, if used in a stable patient, and the patient develops post-operative
paraplegia, the physician may face serious litigation.
Today it is recommended that some form of bypass/shunt be used to reduce
the incidence of paraplegia.

Go to the next page if you knew the correct answer, or click the link images
below to further research the concepts in this question (if desired).

Research Concepts:
Chest Trauma:

Tap flag to report any problems with this question.


Question 806: The best indicator for adequate alveoli ventilation is:
Choices:
1. PaCO2
2. PaO2
3. Saturation
4. pH
Answer: 1 - PaCO2
Explanations:
Adequate alveoli ventilation is best determined by paCO2 level
Normal PaCO2 is 35 to 45 mmHg
It may be altered by adjusting respiratory rate, tidal volume, or dead space

Go to the next page if you knew the correct answer, or click the link images
below to further research the concepts in this question (if desired).

Research Concepts:
Pulmonary Physiology:

Partial Pressure Of Oxygen (PO2):

Tap flag to report any problems with this question.


Question 807: A 63-year-old male was admitted to the hospital with chest
pain. An ECG revealed ST segment elevations and q waves in the anterior wall
leads. Cardiac enzymes were elevated and he was treated in the intensive care
unit with heparin and nitroglycerin. His recovery was uneventful and he was
discharged home to be readmitted in 4 weeks for a cardiac catheterization. Three
weeks after his myocardial infarction, he returns to the hospital complaining of a
low-grade fever and chest pain on deep breathing. He says that the pain is not
crushing and does not radiate, but is more centered around his mid chest. He has
had some associated neck pain and has general malaise. An ECG reveals non-
specific ST elevations in the anterior wall leads. Which of the following should
be performed next?

Choices:
1. Admit to rule out pulmonary embolism
2. Chest x-ray to rule out pneumothorax
3. Nitroglycerin drip to treat angina
4. Start non-steroidal anti-inflammatory drugs (NSAIDs)
Answer: 4 - Start non-steroidal anti-inflammatory drugs (NSAIDs)
Explanations:
The above patient has a classic presentation of Dressler syndrome. Dressler
is a post-myocardial infarction pericarditis, which presents about 2-4 weeks
after the acute myocardial infarction.
Patients will generally present with a low-grade fever, neck and pleuritic
chest pain, general malaise, and fatigue. Blood work is normal and an ECG
will show non-specific ST elevations.
A few patients will develop a pericardial effusion. All studies to investigate
this disorder will yield normal results and the diagnosis is one of exclusion.
The treatment involves non-steroidal anti-inflammatory drugs (NSAIDs).
When NSAIDs fail, one may elect to give a short course of steroids.

Go to the next page if you knew the correct answer, or click the link images
below to further research the concepts in this question (if desired).

Research Concepts:
Dressler Syndrome:

Tap flag to report any problems with this question.


Question 808: Select the treatment of choice for a hemodynamically stable
patient with paroxysmal supraventricular tachycardia.

Choices:
1. Beta blockers
2. Adenosine
3. Calcium channel blocker
4. Synchronized cardioversion
Answer: 2 - Adenosine
Explanations:
Supraventricular tachycardia can be treated with maneuvers that increase
vagal tone such as valsalva, carotid massage, or immersing the face in cold
water
Adenosine is the drug of choice as it slows AV node conduction and dilates
coronary arteries
Diltiazem and verapamil can both be effective
Unstable patients should be treated with synchronized cardioversion

Go to the next page if you knew the correct answer, or click the link images
below to further research the concepts in this question (if desired).

Research Concepts:
Paroxysmal Supraventricular Tachycardia:

Adenosine:

Tap flag to report any problems with this question.


Question 809: What condition is most often associated with pulsus
alternans?

Choices:
1. Aortic stenosis
2. Patent ductus arteriosus
3. Atrial septal defect
4. Congestive heart failure
Answer: 4 - Congestive heart failure
Explanations:
Pulsus alternans is a waveform with alternating strong and weak beats.
Pulsus alternans is always an indication of a failing heart.
Patients with congestive heart failure and weak left ventricle usually
develop pulsus alternans.
Pulsus alternans is associated with a poor prognosis.

Go to the next page if you knew the correct answer, or click the link images
below to further research the concepts in this question (if desired).

Research Concepts:
Heart Failure, Congestive:

Tap flag to report any problems with this question.


Question 810: Which of the following is not an effective method to prevent
deep vein thrombosis?

Choices:
1. Early ambulation after surgery
2. Use of LMWH
3. Use of aspirin
4. Compression stockings
Answer: 3 - Use of aspirin
Explanations:
Deep vein thrombosis is the formation of a thrombus in a deep vein.
Anticoagulation with heparin/LMWH, compression stockings, and early
ambulation after surgery are the effective methods for prevention of a DVT.
Aspirin is not effective

Go to the next page if you knew the correct answer, or click the link images
below to further research the concepts in this question (if desired).

Research Concepts:
Deep Venous Thrombosis Prophylaxis:

Tap flag to report any problems with this question.


Question 811: Which is not a feature of disseminated intravascular
coagulation?

Choices:
1. Decreased fibrinogen and elevated FDP
2. Diffuse bleeding
3. Consumption of coagulation factors
4. Multiple clots in large vessels
Answer: 4 - Multiple clots in large vessels
Explanations:
Large vessel thrombosis is not a characteristic of disseminated intravascular
coagulation (DIC)

Go to the next page if you knew the correct answer, or click the link images
below to further research the concepts in this question (if desired).

Research Concepts:
Disseminated Intravascular Coagulation:

Tap flag to report any problems with this question.


Question 812: When do symptoms of a surgical site infection usually
become obvious?

Choices:
1. Within 48 hours after surgery
2. Five to six days after the surgery
3. After a high fever
4. When the surgery site is shaved in the operating room
Answer: 2 - Five to six days after the surgery
Explanations:
Most surgical site infections become obvious five or more days after the
operation. More than 50 percent are seen just prior to discharge from the
hospital.
Fevers after surgery are usually not due to an infection, but rather to
atelectasis.
Surgical site infections that occur within 48 hours after surgery may be
caused by Group B Streptococci or Clostridia. However, these infections
are very rare.
Infections can present late after surgery, but this is rare and often due to
poor skincare at home.

Go to the next page if you knew the correct answer, or click the link images
below to further research the concepts in this question (if desired).

Research Concepts:
Wound Infection:

Tap flag to report any problems with this question.


Question 813: Which population has the highest risk for minor surgery?
Choices:
1. Male
2. Diabetics
3. Young
4. Elderly
Answer: 2 - Diabetics
Explanations:
Diabetic patients are at increased risk for any surgery.

Go to the next page if you knew the correct answer, or click the link images
below to further research the concepts in this question (if desired).

Research Concepts:
Diabetes Mellitus, Type 2:

Diabetes Mellitus, Type 1:

Surgical Complications:

Tap flag to report any problems with this question.


Question 814: Which of the following would not be appropriate steps in
managing hypokalemia?

Choices:
1. Cardiorespiratory monitoring
2. Administration of oral potassium
3. Administration of insulin and glucose
4. IV fluids with potassium
Answer: 3 - Administration of insulin and glucose
Explanations:
Hypokalemia can be worsened with administration of insulin and glucose
Administration of insulin and glucose pushes potassium into cells

Go to the next page if you knew the correct answer, or click the link images
below to further research the concepts in this question (if desired).

Research Concepts:
Hypokalemia:

Tap flag to report any problems with this question.


Question 815: Which treatment would NOT be indicated for pulseless
electrical activity (PEA) during cardiac arrest?

Choices:
1. Epinephrine
2. Transvenous pacemaker
3. Fluid challenge
4. Needle thoracostomy
Answer: 2 - Transvenous pacemaker
Explanations:
Possible causes of pulseless electrical activity (PEA) include hypoxia,
hypovolemia, cardiac tamponade, acidosis, tension pneumothorax, and
pulmonary embolus.
Pacing and calcium are not effective treatments.

Go to the next page if you knew the correct answer, or click the link images
below to further research the concepts in this question (if desired).

Research Concepts:
Pulseless Electrical Activity:

Tap flag to report any problems with this question.


Question 816: Lowering of very high blood pressure in the ICU is best
done with which drug?

Choices:
1. Nitroprusside
2. Metoprolol
3. Hydralazine
4. Nitroglycerin
Answer: 1 - Nitroprusside
Explanations:
To lower blood pressure, a short acting agent such as nitroprusside is the
choice.

Go to the next page if you knew the correct answer, or click the link images
below to further research the concepts in this question (if desired).

Research Concepts:
Hypertension:

Tap flag to report any problems with this question.


Question 817: Which of the following about vancomycin is FALSE?
Choices:
1. It is bactericidal
2. Has a broad spectrum of activity including gram negatives
3. Is effective against C difficile
4. Rapid infusion causes the red man syndrome
Answer: 2 - Has a broad spectrum of activity including gram negatives
Explanations:
Vancomycin is a bactericidal glycoprotein and has a narrow spectrum of
activity.
Vancomycin is used as an agonist against severe infections caused by gram-
positive infections, including MRSA and c difficile.
Vancomycin is not absorbed from the GI tract and is administered
parenterally. Dose modification is necessary in the presence of renal failure.
Toxic effects include chills, fever, phlebitis, ototoxicity, and nephrotoxicity.
Rapid infusion causes the red man syndrome.

Go to the next page if you knew the correct answer, or click the link images
below to further research the concepts in this question (if desired).

Research Concepts:
Vancomycin:

Tap flag to report any problems with this question.


Question 818: Which of the categories of drug use in pregnancy states "no
harm to fetus shown with animal studies but no adequate and well controlled
studies in pregnant women"?

Choices:
1. C
2. A
3. B
4. X
Answer: 3 - B
Explanations:
Category A indicates well controlled studies in pregnant women have
shown no fetal abnormalities. Category C indicates that animal studies have
shown an adverse effect and there are no well controlled studies in pregnant
women
Category D indicates adequate well controlled and observational studies in
pregnant women have demonstrated fetal abnormalities. The benefits may
outweigh the risks
X category indicates adequate well controlled and observational studies
with animals and pregnant women have shown fetal defects. The drug is
contraindicated.
Acetaminophen is the drug of choice for OTC pain relievers in pregnant
patients.

Go to the next page if you knew the correct answer, or click the link images
below to further research the concepts in this question (if desired).

Research Concepts:
Medication, Pregnancy:

Tap flag to report any problems with this question.


Question 819: Select the incorrect statement about the cardiopulmonary
system.

Choices:
1. A normal pressure in the right ventricle is 25/0 mmHg
2. A normal pressure in the pulmonary artery is 24/8 mmHg
3. A normal pressure in the left ventricle is 120/0 mmHg
4. A red blood cell needs one second to equilibrate the exchange between O2
and CO2 in the lungs
Answer: 4 - A red blood cell needs one second to equilibrate the exchange
between O2 and CO2 in the lungs

Explanations:
The pressures described are all in the normal range for the location
Only 0.25 seconds is required for equilibration of O2 and CO2 in the lungs
by RBCs

Go to the next page if you knew the correct answer, or click the link images
below to further research the concepts in this question (if desired).

Research Concepts:
Cardiac Physiology:

Pulmonary Physiology:

Tap flag to report any problems with this question.


Question 820: Which is not a contributing factor to macrovascular disease
in diabetes?

Choices:
1. Hypotension
2. Hypercholesterolemia
3. Coagulation
4. Platelet function
Answer: 1 - Hypotension
Explanations:
Risk factor for diabetes is hypertension

Go to the next page if you knew the correct answer, or click the link images
below to further research the concepts in this question (if desired).

Research Concepts:
Macrovascular Complications:

Diabetes Mellitus:

Tap flag to report any problems with this question.


Question 821: Which of the following is not a modifiable risk factor in a
patient with coronary artery disease?

Choices:
1. Smoking
2. Hypertension
3. Family history
4. Sedentary lifestyle
Answer: 3 - Family history
Explanations:
Modifiable risk factors include high lipids, smoking, hypertension, diabetes,
and sedentary lifestyle.
Non-modifiable risk factors include age, gender, and family history.
Other risk factors include heavy alcohol intake and elevated plasma
homocysteine levels.
Of the modifiable risk factors, decreases in lipid levels have the most
profound effect on coronary artery disease.

Go to the next page if you knew the correct answer, or click the link images
below to further research the concepts in this question (if desired).

Research Concepts:
Coronary Artery Disease:

Tap flag to report any problems with this question.


Question 822: What will biopsy of a yellowish nodule on the surface of the
eyelid most likely show?

Choices:
1. Accumulated keratin
2. An overproduction of sebaceous glands
3. Meibomian gland occlusion
4. An abundance of lipid cells
Answer: 4 - An abundance of lipid cells
Explanations:
Xanthelasmas are formed on the eyelid and represent cholesterol deposits
They are most common in those of Asian and Mediterranean backgrounds.
They may be seen more commonly but not exclusively in those with
hyperlipidemia.
They are often hereditary.

Go to the next page if you knew the correct answer, or click the link images
below to further research the concepts in this question (if desired).

Research Concepts:
Xanthelasma:

Tap flag to report any problems with this question.


Question 823: An 18 year old had undergone tetralogy of fallot repair in
childhood. Now he has an elevated ratio of right to left ventricular pressure.
Which of the following pathologies is NOT responsible for this finding?

Choices:
1. Persistent pulmonary stenosis
2. Atrial septal defect
3. Persistent ventricular septal defect
4. Residual pulmonary atresia
Answer: 2 - Atrial septal defect
Explanations:
An elevated RV/LV pressure level is a risk factor for premature death both
early and late after repair.

Go to the next page if you knew the correct answer, or click the link images
below to further research the concepts in this question (if desired).

Research Concepts:
Tetralogy Of Fallot:

Tap flag to report any problems with this question.


Question 824: A 62-year-old is admitted with chest pain. ECG reveals that
he is having an acute myocardial infarction. There is no cardiac catheterization
facility nearby and you decide to start him on a drug that is an antibody to the
platelet receptor. Which of the following is the drug you use?

Choices:
1. Clopidogrel
2. Ticlopidine
3. Heparin
4. Abciximab
Answer: 4 - Abciximab
Explanations:
Abciximab is a monoclonal antibody that reversibly inhibits the binding of
fibrin and other ligands to the platelet glycoprotein receptor.

Go to the next page if you knew the correct answer, or click the link images
below to further research the concepts in this question (if desired).

Research Concepts:
Antiplatelet Medications:

Tap flag to report any problems with this question.


Question 825: Which of the following is not a commonly used drug in the
treatment of congestive heart failure?

Choices:
1. Captopril
2. Furosemide
3. Digoxin
4. Minoxidil
Answer: 4 - Minoxidil
Explanations:
Minoxidil is an alpha antagonist and has no role in the treatment of
congestive heart failure.
Minoxidil is used in the treatment of hypertension and has an odd side
effect of excess hair growth.

Go to the next page if you knew the correct answer, or click the link images
below to further research the concepts in this question (if desired).

Research Concepts:
Heart Failure, Congestive:

Tap flag to report any problems with this question.


Question 826: To prevent bedsores, a bedridden patient should be turned
how often?

Choices:
1. Every one hour
2. Every two hours
3. Every four hours
4. Every six hours
Answer: 2 - Every two hours
Explanations:
A bedridden patient should be turned every two hours
This is the best way to prevent bedsores

Go to the next page if you knew the correct answer, or click the link images
below to further research the concepts in this question (if desired).

Research Concepts:
Decubitus Ulcers:

Tap flag to report any problems with this question.


Question 827: What is the most common complication of a bicuspid aortic
valve?

Choices:
1. Aortic stenosis
2. Endocarditis
3. Aortic root dissection
4. Right ventricular hypertrophy
Answer: 1 - Aortic stenosis
Explanations:
Calcification of the bicuspid aortic valve begins in the second decade of
life.
Almost 50% of patients with severe aortic stenosis have a congenital
bicuspid valve.
Bicuspid aortic valve is also a common cause of aortic regurgitation.
The risk of developing endocarditis with a bicuspid aortic valve is about
20% over a lifetime.

Go to the next page if you knew the correct answer, or click the link images
below to further research the concepts in this question (if desired).

Research Concepts:
Aortic Stenosis:

Aortic Valve:

Tap flag to report any problems with this question.


Question 828: Which of the following is an indication for consideration of
coronary artery bypass grafting (CABG)?

Choices:
1. Two-vessel disease with severe proximal LAD involvement
2. Complex three-vessel disease
3. Left main disease
4. All of the above
Answer: 4 - All of the above
Explanations:
ACC/AHA guidelines recommend considering CABG for people in all of
the above groups
Another group for consideration of CABG are high-risk patients with LV
systolic dysfunction
Low-risk patients receive negligible or modestly increased survival with
CABG

Go to the next page if you knew the correct answer, or click the link images
below to further research the concepts in this question (if desired).

Research Concepts:
Coronary Artery Bypass Graft:

Tap flag to report any problems with this question.


Question 829: Nitroglycerin does NOT generate which of the following?
Choices:
1. Nitric oxide
2. cAMP
3. cGMP
4. Phosphodiesterase
Answer: 2 - cAMP
Explanations:
Nitroglycerin metabolizes into nitric oxide, which leads to an increase of
cGMP.

Go to the next page if you knew the correct answer, or click the link images
below to further research the concepts in this question (if desired).

Research Concepts:
Nitroglycerin:

Tap flag to report any problems with this question.


Question 830: Which of the following statements is false about acute
pericarditis?

Choices:
1. Patients present with sharp chest pain
2. Pain improve with position
3. Most patients will have normal ECG
4. Hospital admission is not advised for viral etiology
Answer: 3 - Most patients will have normal ECG
Explanations:
Acute pericarditis is associated with ST elevation in the precordial leads
and PR segment depression in lead II and V4 to V6

Go to the next page if you knew the correct answer, or click the link images
below to further research the concepts in this question (if desired).

Research Concepts:
Pericarditis:

Tap flag to report any problems with this question.


Question 831: What is the leading cause of death in the U.S.?
Choices:
1. Cardiovascular disease
2. Accidents
3. Cancer
4. Diabetes
Answer: 1 - Cardiovascular disease
Explanations:
The leading cause of death in the United States is cardiovascular disease.
Cancer is the second most common cause of death in the U.S.
Accidents are the fifth leading cause of death
Diabetes is the seventh leading cause of death

Go to the next page if you knew the correct answer, or click the link images
below to further research the concepts in this question (if desired).

Research Concepts:
Epidemiology, Morbidity And Mortality Measurement:

Cardiovascular Disease:

Tap flag to report any problems with this question.


Question 832: A patient involved in a motor vehicle accident is found to
have a ruptured spleen and undergoes an emergent exploratory laparotomy.
Forty-eight hours later, the patient appears to be lethargic and has tachypnea and
tachycardia. The respiratory efforts are minimal and he has a low-grade fever. In
the above patient, the arterial oxygen is less than 55, the C02 is >48 and the pH
is 7.1. What is the most appropriate therapy for this patient?

Choices:
1. Mechanical ventilation
2. BiPAP
3. 100% non-rebreather oxygen
4. Theophylline to stimulate the respiratory center
Answer: 1 - Mechanical ventilation
Explanations:
There are many indications for mechanical ventilation. In general, with
ineffective gas exchange and labored breathing, the individual should be
intubated.
Other criteria for ventilation include tachypnea, low vital capacity, retention
of CO2, clinical deterioration, and respiratory muscle fatigue.
Obtundation, coma, and hypotension are other indications to ventilate the
patient.

Go to the next page if you knew the correct answer, or click the link images
below to further research the concepts in this question (if desired).

Research Concepts:
Intubation, Indications:

Tap flag to report any problems with this question.


Question 833: How long should air travel be postponed after a patient
experiences an uncomplicated myocardial infarction?

Choices:
1. 1 week
2. 2 weeks
3. 1 month
4. 6 months
Answer: 2 - 2 weeks
Explanations:
Based on consensus opinion of the Aerospace Medical Association
(ASMA), the American College of Cardiology (ACC), and the American
Heart Association (AHA), patients can fly at 2 weeks after an
uncomplicated myocardial infarction or simple percutaneous intervention.
The patient should be able to tolerate his usual activities. Following a
myocardial infarction, a stress test is done in almost all low-risk patients.
This test can assist in determining the safety of flying.
Myocardial infarction, with associated severe depression of cardiac
function, should delay flying until 2 weeks after the patient is medically
stable. Patients with unstable angina should not fly. Stable angina is not a
contraindication.
Patients with advanced congestive heart failure may benefit from
supplemental oxygen.

Go to the next page if you knew the correct answer, or click the link images
below to further research the concepts in this question (if desired).

Research Concepts:
Myocardial Infarction, Acute:

Tap flag to report any problems with this question.


Question 834: Complications of idiopathic pulmonary fibrosis (IPF) may
include all of the following EXCEPT:

Choices:
1. Cor pulmonale
2. Polycythemia
3. Pneumothorax
4. Hepatitis
Answer: 4 - Hepatitis
Explanations:
IPF causes scarring of the lungs with no apparent cause
Complications of IPF may include cor pulmonale, pneumothorax,
polycythemia, chronic hypoxemia, pulmonary hypertension, and respiratory
failure
Therapy for IPF may include corticosteroids, immunosuppressive agents,
supplemental oxygen, and lung rehabilitation
Hepatitis is not a complication of the disease

Go to the next page if you knew the correct answer, or click the link images
below to further research the concepts in this question (if desired).

Research Concepts:
Pulmonary Fibrosis, Idiopathic:

Tap flag to report any problems with this question.


Question 835: Which of the following is NOT needed to determine the
half-life of a medication?

Choices:
1. Elimination rate constant
2. Clearance (Cl)
3. Volume of distribution (Vd)
4. Loading dose
Answer: 4 - Loading dose
Explanations:
Half-life is a function of both Cl and Vd
Elimination rate constant= Cl/Vd
Half-life= 0.693/Elimination rate constant
As Cl and Vd changes, so does the half-life of a drug

Go to the next page if you knew the correct answer, or click the link images
below to further research the concepts in this question (if desired).

Research Concepts:
Half Life:

Tap flag to report any problems with this question.


Question 836: Which of the following drugs is likely to cause hemolytic
anemia?

Choices:
1. Digoxin
2. Hydralazine
3. Propranolol
4. Quinidine
Answer: 4 - Quinidine
Explanations:
Quinidine can cause hemolytic anemia
The mechanism of such a side effect is most likely oxidative stress to red
blood cells
Hydralazine is implicated in drug induced SLE
Propranolol can cause severe bradycardia

Go to the next page if you knew the correct answer, or click the link images
below to further research the concepts in this question (if desired).

Research Concepts:
Quinidine:

Hemolytic Anemia:

Tap flag to report any problems with this question.


Question 837: Which of the following is a class IB antiarrhythmic?
Choices:
1. Atenolol
2. Amiodarone
3. Lidocaine
4. Amlodipine
Answer: 3 - Lidocaine
Explanations:
Lidocaine is a class IB antiarrhythmic
Atenolol is a class II antiarrhythmic
Amiodarone is a class III antiarrhythmic
Amlodipine is a class IV antiarrhythmic

Go to the next page if you knew the correct answer, or click the link images
below to further research the concepts in this question (if desired).

Research Concepts:
Antiarrhythmic Medication:

Lidocaine:

Tap flag to report any problems with this question.


Question 838: One is most likely to see a water hammer pulse in a patient
with:

Choices:
1. Aortic stenosis
2. Aortic regurgitation
3. Coarctation
4. Congestive heart failure
Answer: 2 - Aortic regurgitation
Explanations:
Watson water hammer pulse describe a pulse that is very forceful or
bounding.
The pulse is often rapidly increasing and suddenly collapsing.
Water hammer pulse is due to increased stroke volume of the left ventricle
and a decrease in systemic vascular resistance.
Water hammer pulse is often seen in patients with aortic regurgitation.

Go to the next page if you knew the correct answer, or click the link images
below to further research the concepts in this question (if desired).

Research Concepts:
Aortic Regurgitation:

Tap flag to report any problems with this question.


Question 839: What is the most common cardiac malformation?
Choices:
1. Atrial septal defect
2. Endocardial cushion defect
3. Ventricular septal defect
4. Aortic stenosis
Answer: 3 - Ventricular septal defect
Explanations:
Collectively ventricular septal defects (VSDs) are the most common
congenital cardiac defects.
The perimembranous variety is the most common type of VSD.
Only the small perimembranous VSD close within the first year. Any other
type of VSD is unlikely to close.
Closure of all VSD is recommended because it can lead to pulmonary
hypertension.

Go to the next page if you knew the correct answer, or click the link images
below to further research the concepts in this question (if desired).

Research Concepts:
Ventricular Septal Defect:

Tap flag to report any problems with this question.


Question 840: In which electrolye abnormality are peaked T waves
commonly seen?

Choices:
1. Hypocalcemia
2. Hypercalcemia
3. Hyperkalemia
4. Hypomagnesemia
Answer: 3 - Hyperkalemia
Explanations:
The presence of peaked T waves on an ECG should suggest hyperkalemia.
Severe hyperkalemia may also be associated with wide QRS complexes and
reduction in the size of P waves.
Hyperkalemia causes an overall membrane depolarization that inactivates
many sodium channels.

Go to the next page if you knew the correct answer, or click the link images
below to further research the concepts in this question (if desired).

Research Concepts:
Electrocardiogram:

Hyperkalemia:

Tap flag to report any problems with this question.


Question 841: Which of the following is NOT part of the clinical triad of
cardiac tamponade?

Choices:
1. Muffled heart sounds
2. Bradycardia
3. Hypotension
4. Jugular venous distension
Answer: 2 - Bradycardia
Explanations:
Cardiac tamponade is characterized by muffled heart sounds, hypotension,
and jugular venous distention.
There is often tachycardia, not bradycardia.

Go to the next page if you knew the correct answer, or click the link images
below to further research the concepts in this question (if desired).

Research Concepts:
Cardiac Tamponade:

Tap flag to report any problems with this question.


Question 842: After a routine bypass, the patient takes a long time to warm
up. The nasopharyngeal temperature is 8 degrees higher than the bladder
temperature. Despite changing the bladder probe and warming for 45 minutes,
the bladder temperature fails to rise but the temperature difference gets wider.
Which of the following is the most appropriate response?

Choices:
1. Ignore temperature of bladder and come off bypass
2. Place a rectal temperature probe
3. TEE
4. Check warmer in circuit and replace oxygenator
Answer: 3 - TEE
Explanations:
When the temperature is different in 2 probes and the patient fails to warm
up, please think dissection.
TEE will be required to search for dissection.

Go to the next page if you knew the correct answer, or click the link images
below to further research the concepts in this question (if desired).

Research Concepts:
Dissection, Aortic:

Tap flag to report any problems with this question.


Question 843: Which of the following is a correct statement about
warfarin?

Choices:
1. It is structurally similar to vitamin K
2. It works immediately after administration
3. It cannot be administered simultaneously with heparin
4. It affects factors II, VII, IX, and X
Answer: 4 - It affects factors II, VII, IX, and X
Explanations:
Warfarin inhibits epoxide reductase, reducing the amount of available
vitamin K.
It is is not structurally similar to vitamin K. It reduces production of factors
II, VII, IX, and X, as well as proteins C, S, and Z.
Warfarin can initially increase coagulability as it reduces levels of protein S
and C. Therefore, it is often given after heparin (or low molecular weight
heparin) is started.
The coagulation factors must be depleted before warfarin is effective.

Go to the next page if you knew the correct answer, or click the link images
below to further research the concepts in this question (if desired).

Research Concepts:
Anticoagulation:

Warfarin:

Tap flag to report any problems with this question.


Question 844: What is used to stain amyloid protein?
Choices:
1. Congo Red
2. Acetic acid
3. Methylene Blue
4. Iodine
Answer: 1 - Congo Red
Explanations:
Amyloid is a proteinaceous material, which is easily identified by Congo
red
The protein can appear apple green and birefringent under polarized light

Go to the next page if you knew the correct answer, or click the link images
below to further research the concepts in this question (if desired).

Research Concepts:
Amyloid Protein:

Tap flag to report any problems with this question.


Question 845: In which of these patients with aortic stenosis is surgery not
indicated?

Choices:
1. Patients with frequent episodic angina
2. Patients with syncope
3. Valvular gradient of 40 mmHg
4. Aortic valve area of < 0.5 cm2
Answer: 3 - Valvular gradient of 40 mmHg
Explanations:
Asymptomatic patients with AS may do well for many years, but the onset
of symptoms is generally followed by rapid deterioration
Surgery is indicated in symptomatic patients. Indications for surgery in
asymptomatic patients include progressive cardiac enlargement and
findings of elevated LVEDP at rest
The gradient across the valve has to be about 50 mmHg to satisfy the
criteria for surgery
An aortic valve area of 0.5 cm2 is the smallest area and almost all patients
will be symptomatic

Go to the next page if you knew the correct answer, or click the link images
below to further research the concepts in this question (if desired).

Research Concepts:
Aortic Stenosis:

Tap flag to report any problems with this question.


Question 846: Which is least likely to be a risk factor for wound
infections?

Choices:
1. Diabetes
2. Corticosteroid therapy
3. Obesity
4. Hypertension
Answer: 4 - Hypertension
Explanations:
Risk factors for wound infections include obesity, diabetes, corticosteroid
therapy, anemia, immunosuppression, and poor hemostasis during surgery.

Go to the next page if you knew the correct answer, or click the link images
below to further research the concepts in this question (if desired).

Research Concepts:
Wound Infection:

Tap flag to report any problems with this question.


Question 847: Which is FALSE about acute respiratory distress syndrome
(ARDS)?

Choices:
1. Hypoxia is major pathology
2. Pulmonary wedge pressure is high
3. Infection often present
4. PEEP is beneficial
Answer: 2 - Pulmonary wedge pressure is high
Explanations:
ARDS has a mortality of at least 30 to 40%.
There is inflammation of alveoli with collapse resulting in hypoxia and
mechanical ventilatory support.
ARDS can be caused by blood borne infection, major trauma, aspiration, or
be idiopathic.
The condition is of sudden onset commonly 24 to 48 hours after the insult.

Go to the next page if you knew the correct answer, or click the link images
below to further research the concepts in this question (if desired).

Research Concepts:
Acute Respiratory Distress Syndrome:

Tap flag to report any problems with this question.


Question 848: The underlying pathophysiology of a decrease in urine
output can be:

Choices:
1. Hypertension
2. Increased cardiac output
3. Compromised organ profusion
4. Bradypnea
Answer: 3 - Compromised organ profusion
Explanations:
A decrease in urine output can result from altered organ profusion that can
occur with the development of shock

Go to the next page if you knew the correct answer, or click the link images
below to further research the concepts in this question (if desired).

Research Concepts:
Oliguria:

Tap flag to report any problems with this question.


Question 849: What is the main extracellular ion in humans?
Choices:
1. Cl
2. Na
3. K
4. Ca
Answer: 2 - Na
Explanations:
The main extracellular ion is sodium.

Go to the next page if you knew the correct answer, or click the link images
below to further research the concepts in this question (if desired).

Research Concepts:
Electrolytes:

Tap flag to report any problems with this question.


Question 850: Moving a patient from the supine to Trendelenburg position
is least likely to increase which of the following?

Choices:
1. Systemic arterial blood pressure
2. Central venous pressure
3. Intracranial pressure
4. Heart rate
Answer: 4 - Heart rate
Explanations:
The Trendelenburg position increases venous return.
This often increases the cardiac output, central venous pressure, and
systemic arterial blood pressure.
The heart rate generally does not increase.
If the heart rate does change, it tends to decrease.

Go to the next page if you knew the correct answer, or click the link images
below to further research the concepts in this question (if desired).

Research Concepts:
Trendelenburg Position:

Tap flag to report any problems with this question.


Question 851: The half-life of lidocaine is approximately:
Choices:
1. 60 minutes
2. 100 minutes
3. 140 minutes
4. 160 minutes
Answer: 2 - 100 minutes
Explanations:
Lidocaine's elimination half-life is approximately 90–120 minutes

Go to the next page if you knew the correct answer, or click the link images
below to further research the concepts in this question (if desired).

Research Concepts:
Half Life:

Lidocaine:

Tap flag to report any problems with this question.


Question 852: Which Medicare program is responsible for covering
financial costs of inpatient hospital care in people over the age of 65?

Choices:
1. Medicare part A
2. Medicare part C
3. Medicare part D
4. Medicare part B
Answer: 1 - Medicare part A
Explanations:
Medicare part A covers for inpatient hospital care.
It is funded by social security tax.
Medicare part A is automatically provided to all elderly patients.
Medicare part B covers for physician services and durable medical
equipment.

Go to the next page if you knew the correct answer, or click the link images
below to further research the concepts in this question (if desired).

Research Concepts:
Medicare Coverage:

Tap flag to report any problems with this question.


Question 853: Which of the following is not a clinical feature of air
embolism?

Choices:
1. Hypertension
2. Murmur
3. Petechiae
4. Desaturation
Answer: 1 - Hypertension
Explanations:
Air embolism is associated with hypotension.

Go to the next page if you knew the correct answer, or click the link images
below to further research the concepts in this question (if desired).

Research Concepts:
Venous Air Embolism:

Tap flag to report any problems with this question.


Question 854: What type of allergic reactions is implicated in
anaphylactoid reaction following penicillin administration?

Choices:
1. Type I allergic reaction
2. Type II allergic reaction
3. Type III allergic reaction
4. Type IV reaction
Answer: 1 - Type I allergic reaction
Explanations:
Type 1 hypersensitivity reaction is divided into immediate and delayed
types
The immediate type is mediated by IgE and is responsible for anaphylactoid
reactions
The delayed type occurs days after drug exposure and not mediated by IgE

Go to the next page if you knew the correct answer, or click the link images
below to further research the concepts in this question (if desired).

Research Concepts:
Anaphylaxis:

Tap flag to report any problems with this question.


Question 855: Atrioventricular dissociation is least often associated with:
Choices:
1. Hyperkalemia
2. Inotropes
3. Digoxin toxicity
4. Hyponatremia
Answer: 4 - Hyponatremia
Explanations:
AV dissociation can be secondary to anesthesia, intubation, inotropes,
medications that block catecholamines, hyperkalemia, sinus node disease,
ventricular pacing, vagal activation, myocardial infarction, and structural
heart disease
It is very rarely associated with hyponatremia

Go to the next page if you knew the correct answer, or click the link images
below to further research the concepts in this question (if desired).

Research Concepts:
Atrioventricular Dissociation:

Tap flag to report any problems with this question.


Question 856: The artery that supplies the left ventricle is which of the
following?

Choices:
1. Right descending artery
2. Posterior descending artery
3. Circumflex artery
4. Great cardiac artery
Answer: 3 - Circumflex artery
Explanations:
The circumflex artery is a branch of the left main coronary artery.
It travels in the left atrioventricular groove.
It supplies posterolateral left ventricle and anterolateral papillary muscle.
It ends at the point where it joins to form posterior interventricular artery in
the 10% of cases where the posterior interventricular artery does not arise
from the right coronary artery.

Go to the next page if you knew the correct answer, or click the link images
below to further research the concepts in this question (if desired).

Research Concepts:
Heart Anatomy:

Tap flag to report any problems with this question.


Question 857: Which arterial blood gas finding is most suggestive of acute
respiratory failure?

Choices:
1. PCO2= 50, PO2= 80
2. PCO2 =40, PO2=60
3. PCO2=60, PO2= 50
4. PCO2= 30, PO2=70
Answer: 3 - PCO2=60, PO2= 50
Explanations:
In a patient with normal lung function, acute respiratory failure is a PO2
<60 mm Hg and PCO2 >50 mm Hg.

Go to the next page if you knew the correct answer, or click the link images
below to further research the concepts in this question (if desired).

Research Concepts:
Respiratory Failure:

Tap flag to report any problems with this question.


Question 858: All of the following are potential complications of
nasotracheal suctioning EXCEPT:

Choices:
1. Hypoxia
2. Atelectasis
3. Pulmonary embolus (PE)
4. Hypotension
Answer: 3 - Pulmonary embolus (PE)
Explanations:
PE is not a potential complication of nasotracheal suctioning
Potential complications include hypoxia, atelectasis, hypotension, cardiac
arrhythmias, infection, and pulmonary hemorrhage
Correct position for the procedure is sitting up at 45-degree angle with the
head turned away from the nostril entered
The position straightens the trachea and airway easing entry into bronchus

Go to the next page if you knew the correct answer, or click the link images
below to further research the concepts in this question (if desired).

Research Concepts:
Nasotracheal Suctioning:

Tap flag to report any problems with this question.


Question 859: Which of the following antihypertensives may cause lipid
abnormalities?

Choices:
1. Metoprolol
2. Amlodipine
3. Nitroprusside
4. Terazosin
Answer: 1 - Metoprolol
Explanations:
Beta blockers can cause dyslipidemia
Amlodipine is a calcium channel blocker
Nitroprusside can cause cyanide toxicity
Terazosin can cause orthostatic hypotension

Go to the next page if you knew the correct answer, or click the link images
below to further research the concepts in this question (if desired).

Research Concepts:
Beta-Blockers:

Dyslipidemia:

Tap flag to report any problems with this question.


Question 860: Bilateral facial paralysis accompanied by fever, rash, and
arthralgias is suggestive of which of the following types of infection?

Choices:
1. HIV
2. Mycobacterium avium
3. Treponema pallidum
4. Borrelia burgdorferi (Lyme disease)
Answer: 4 - Borrelia burgdorferi (Lyme disease)
Explanations:
Erythema migrans is the characteristic rash associated with Lyme disease.
The initial infections is secondary to a deer tick (Ixodes) bite. Borrelia
spreads hematogenously and can result in fever, headache, migratory
arthralgias, and adenopathy.
Late manifestations of the disease can involve the heart, joints, or nervous
system.
The most common neurologic manifestation is facial nerve paralysis that
does not respond to corticosteroids. This is bilateral in up to 25% of
patients.

Go to the next page if you knew the correct answer, or click the link images
below to further research the concepts in this question (if desired).

Research Concepts:
Lyme Disease:

Tap flag to report any problems with this question.


Question 861: Which of the following is true of coronary stenting for
multivessel coronary artery disease when compared with CABG?

Choices:
1. Increased protection against death, stroke, and myocardial infarction
2. Same degree of protection against death, stroke, and MI
3. Decreased protection against death, stroke, and MI
4. Stenting is more expensive than CABG
Answer: 2 - Same degree of protection against death, stroke, and MI
Explanations:
The Arterial Revascularization Study (ARTS) concluded stenting offered
the same degree of protection against death, stroke and MI as a CABG
Stenting for multivessel CAD is less expensive than CABG
Stenting was associated with an increased need for repeat revascularization
The Arterial Revascularization Study Semin Intern Cardiol. 1999
Dec;4(4):209-19.

Go to the next page if you knew the correct answer, or click the link images
below to further research the concepts in this question (if desired).

Research Concepts:
Intracoronary Stents:

Coronary Artery Bypass Graft:

Multivessel Disease:
Tap flag to report any problems with this question.
Question 862: Transfusion of red blood cells in a trauma patient results in
immediate hypotension, hematuria, and fever. Which of the following is not part
of the management of this patient?

Choices:
1. Intravenous fluids
2. Stopping the blood transfusion
3. Sodium bicarbonate
4. Dobutamine
Answer: 4 - Dobutamine
Explanations:
The first step in any transfusion reaction is to stop the blood transfusion.
One should anticipate hypotension, renal failure, and disseminated
intravascular coagulation.
Aggressive hydration, with crystalloid and osmotic diuresis with mannitol,
should follow.
If disseminated intravascular coagulation is documented, one may require
fresh frozen plasma, pooled cryoprecipitate, and platelets.
If there is any evidence of an allergic or anaphylactic reaction, epinephrine
and diphenhydramine should be administered. The role of corticosteroids is
controversial.

Go to the next page if you knew the correct answer, or click the link images
below to further research the concepts in this question (if desired).

Research Concepts:
Transfusion Reactions:

Tap flag to report any problems with this question.


Question 863: Which of the following often increases the threshold for the
activation of pacemakers?

Choices:
1. Hyperkalemia
2. Hyperventilation
3. Hypoxia
4. Anxiety
Answer: 3 - Hypoxia
Explanations:
Hypoxia increases the threshold of pacemakers for intervention.

Go to the next page if you knew the correct answer, or click the link images
below to further research the concepts in this question (if desired).

Research Concepts:
Pacemaker:

Tap flag to report any problems with this question.


Question 864: All of the following antibiotics can be used for treatment of
Gram-negative infections, except:

Choices:
1. Ciprofloxacin
2. Imipenem
3. Aztreonam
4. Clindamycin
Answer: 4 - Clindamycin
Explanations:
Gram-negative infections are often treated with combination of drugs
Aminoglycosides, are often first choice, in patients with normal renal
function
Ciprofloxacin, imipenem and aztreonam are other choices to treat Gram-
negative infections
Clindamycin has good coverage of Gram positives and anaerobes, but not
Gram-negative organisms

Go to the next page if you knew the correct answer, or click the link images
below to further research the concepts in this question (if desired).

Research Concepts:
Gram-Negative Bacteria:

Antibiotics:

Tap flag to report any problems with this question.


Question 865: Which of the following statements is false about multifocal
atrial tachycardia (MAT)?

Choices:
1. Atrial rate is over 100 bpm
2. P-waves are all inverted
3. P-waves are separated by iso-electric intervals
4. MAT is seen in patients with significant lung disease
Answer: 2 - P-waves are all inverted
Explanations:
About 60 percent of patients with MAT have significant lung disease.
P-waves in MAT must have at least 3 morphologies including one normal
form.
This is best observed in leads II, III, and, V1.

Go to the next page if you knew the correct answer, or click the link images
below to further research the concepts in this question (if desired).

Research Concepts:
Multifocal Atrial Tachycardia:

Tap flag to report any problems with this question.


Question 866: Which the following statement is true regarding mitral valve
prolapse?

Choices:
1. The incidence in general population 15 percent
2. Commonly associated with sudden death in infants
3. Most presentations are asymptomatic
4. S4 sound is a classic on auscultation
Answer: 3 - Most presentations are asymptomatic
Explanations:
Sudden death is rare in population with MVP
Mid systolic click is classic heart sound
The incidence of MVP in the general population is under 10 percent

Go to the next page if you knew the correct answer, or click the link images
below to further research the concepts in this question (if desired).

Research Concepts:
Mitral Valve Prolapse:

Tap flag to report any problems with this question.


Question 867: What is the most common cause of non-ischemic mitral
regurgitation?

Choices:
1. Rheumatic fever
2. Aortic Dissection
3. Mitral valve prolapse
4. Carcinoid syndrome
Answer: 3 - Mitral valve prolapse
Explanations:
MVP is the most common cause of non-ischemic mitral regurgitation.
Patients with MVP may present with palpitations, atrial fibrillation, or
syncope.
Both anxiety and panic attacks tend to be more frequent in patients with
MVP.
MVP is more frequent in patients with Marfan syndrome polycystic kidney
disease and in Ehlers danlos syndrome.

Go to the next page if you knew the correct answer, or click the link images
below to further research the concepts in this question (if desired).

Research Concepts:
Mitral Valve Prolapse:

Tap flag to report any problems with this question.


Question 868: During cardiopulmonary bypass as the temperature
decreases, the concentration of oxygen dissolved in blood does which of the
following?

Choices:
1. Decreases
2. Increases
3. Remains the same
4. Increases then decreases
Answer: 2 - Increases
Explanations:
All gases dissolve in plasma as the temperature falls.
During hypothermia, the oxygen-Hb curve is shifted to the left and thus
oxygen bound to the Hg is not released.

Go to the next page if you knew the correct answer, or click the link images
below to further research the concepts in this question (if desired).

Research Concepts:
Oxyhemoglobin Dissociation Curve:

Tap flag to report any problems with this question.


Question 869: What is the best initial step in management in a patient with
oliguria?

Choices:
1. IV fluids
2. Renal ultrasound
3. Furosemide
4. Bladder pressure
Answer: 1 - IV fluids
Explanations:
Fluid challenge is the initial treatment for patients with oliguria.

Go to the next page if you knew the correct answer, or click the link images
below to further research the concepts in this question (if desired).

Research Concepts:
Oliguria:

Tap flag to report any problems with this question.


Question 870: Coronary perfusion occurs during which part of the cardiac
cycle?

Choices:
1. Systole
2. Mid systolic
3. Diastole
4. End diastolic
Answer: 3 - Diastole
Explanations:
Coronary perfusion occurs at diastole

Go to the next page if you knew the correct answer, or click the link images
below to further research the concepts in this question (if desired).

Research Concepts:
Coronary Perfusion Pressure:

Tap flag to report any problems with this question.


Question 871: What usually causes post-transplant lymphomas?
Choices:
1. CMV
2. EBV
3. Herpes
4. HIV
Answer: 2 - EBV
Explanations:
The single most important risk factor for post-transplant lymphoma is lack
of previous exposure to EBV.
Studies indicate that seronegativity to EBV increases post-transplant
lymphomas.

Go to the next page if you knew the correct answer, or click the link images
below to further research the concepts in this question (if desired).

Research Concepts:
Posttransplantation Lymphoproliferative Disorders:

Tap flag to report any problems with this question.


Question 872: Which of the followings is false in regards to patient with
dilated atrium and left ventricular hypertrophy?

Choices:
1. This patient is likely to have mitral valve stenosis
2. Administration of diuretic is beneficial
3. Angina and dizziness are likely associated symptoms
4. Beta blockers are appropriate treatment option
Answer: 2 - Administration of diuretic is beneficial
Explanations:
Rate control is critical in patients with left ventricular hypertrophy
Because ventricular filling pressure must be maintained, depletion of
vascular volume is generally not recommended

Go to the next page if you knew the correct answer, or click the link images
below to further research the concepts in this question (if desired).

Research Concepts:
Left Ventricular Hypertrophy:

Tap flag to report any problems with this question.


Question 873: Which cofactor is required in collagen synthesis?
Choices:
1. Vitamin A
2. Vitamin D
3. Calcium
4. Thiamine
Answer: 4 - Thiamine
Explanations:
Thiamine is required for the synthesis of collagen

Go to the next page if you knew the correct answer, or click the link images
below to further research the concepts in this question (if desired).

Research Concepts:
Collagen Synthesis:

Thiamine:

Tap flag to report any problems with this question.


Question 874: What drug is used to reverse heparin?
Choices:
1. Warfarin
2. Fresh frozen plasma
3. Protamine
4. Vitamin K
Answer: 3 - Protamine
Explanations:
Protamine antagonizes the anticoagulant effects of heparin.
Protamine is derived from fish sperm and is high in arginine content.
The positively charged protein particles in protamine interact with the
negatively charged heparin.
Adverse effects of protamine include hypersensitivity, bronchoconstriction,
flushing, and hypotension.

Go to the next page if you knew the correct answer, or click the link images
below to further research the concepts in this question (if desired).

Research Concepts:
Heparin Reversal:

Tap flag to report any problems with this question.


Question 875: Which of the following is the most common presentation of
heparin induced thrombocytopenia?

Choices:
1. Bleeding
2. Thrombosis
3. Acute renal failure
4. Hemolysis
Answer: 2 - Thrombosis
Explanations:
Heparin induced thrombocytopenia is an immune mediated destruction of
platelets that is commonly seen after a few days of heparin therapy
The most common presentation is paradoxical thrombosis

Go to the next page if you knew the correct answer, or click the link images
below to further research the concepts in this question (if desired).

Research Concepts:
Thrombocytopenia, Heparin Induced:

Tap flag to report any problems with this question.


Question 876: Chemotherapy associated cardiac toxicity is a common side
effect of which of the following drugs?

Choices:
1. Daunorubicin
2. Cisplatin
3. Cyclophosphamide
4. Vinblastine
Answer: 1 - Daunorubicin
Explanations:
For daunorubicin, myocardial toxicity, is potentially fatal congestive heart
failure
This may occur when a total cumulative dosage exceeds 400 to 550 mg/m
in adults
The risk of developing congestive heart failure increases with increasing
total cumulative doses of doxorubicin in excess of 450 mg/m

Go to the next page if you knew the correct answer, or click the link images
below to further research the concepts in this question (if desired).

Research Concepts:
Daunorubicin:

Heart Failure, Congestive:

Tap flag to report any problems with this question.


Question 877: Which of the following medications cannot be given orally?
Choices:
1. Chlorpromazine
2. Metformin
3. Amiodarone
4. Lidocaine
Answer: 4 - Lidocaine
Explanations:
Lidocaine has almost zero bioavailability when given orally but can be used
topically in the oropharynx
This is because it undergoes significant first pass metabolism before being
absorbed
Aminoglycosides are not given orally for systemic uses because of
significant first pass effect

Go to the next page if you knew the correct answer, or click the link images
below to further research the concepts in this question (if desired).

Research Concepts:
Lidocaine:

Tap flag to report any problems with this question.


Question 878: Which of the following drugs can result in malignant
hyperthermia?

Choices:
1. Atracurium
2. Benztropine
3. Succinylcholine
4. Dantrolene
Answer: 3 - Succinylcholine
Explanations:
Succinylcholine is a depolarizing agent used for rapid sequence intubation
The disorder is estimated to occur in 0.0004 to 0.00625 percent of patients
who receive this medication
Succinylcholine is contraindicated in patients with a personal or family
history of malignant hyperthermia
Sevoflurane, desflurane and isoflurane can also cause similar problem

Go to the next page if you knew the correct answer, or click the link images
below to further research the concepts in this question (if desired).

Research Concepts:
Malignant Hyperthermia:

Succinylcholine:

Tap flag to report any problems with this question.


Question 879: Which of the following types of sutures is not absorbable?
Choices:
1. Catgut
2. Vicryl
3. Polypropylene
4. Dexon
Answer: 3 - Polypropylene
Explanations:
The goal of suturing is to approximate the skin and eliminate unnecessary
dead space. Tension at the wound site should be minimized.
To achieve a maximal cosmetic result, a suture is chosen based on the
clinical situation. Monofilament sutures have significantly lowered the
incidence of infection compared with multifilament sutures that can harbor
bacteria.
Nonabsorbable sutures (i.e., nylon, silk, polypropylene, braided polyester,
and polybutester) are usually used to close the superficial layer of skin.
Absorbable sutures (i.e., catgut, chromic catgut, Dexon, Maxon, and Vicryl)
are used to close deep layers of skin.
Silk is often used on oral lacerations because of its tolerability in the mouth.

Go to the next page if you knew the correct answer, or click the link images
below to further research the concepts in this question (if desired).

Research Concepts:
Sutures:

Tap flag to report any problems with this question.


Question 880: After suffering a myocardial infarction, a 63-year-old
develops heparin-induced thrombocytopenia. After being placed on aspirin for a
week, the patient develops hives and a rash. What is the next best prophylactic
drug for post-myocardial infarction?

Choices:
1. Ticlopidine
2. Dipyridamole
3. Enteric coated Aspirin
4. Warfarin
Answer: 1 - Ticlopidine
Explanations:
Ticlopidine is a platelet inhibitor, which is different from aspirin.
Ticlopidine inhibits ADP-induced platelet aggregation and has been shown
to decrease the incidence of thrombotic strokes.
Ticlopidine can cause prolonged bleeding and its most serious side effect is
neutropenia.
Ticlopidine is generally reserved for patients who cannot tolerate aspirin.

Go to the next page if you knew the correct answer, or click the link images
below to further research the concepts in this question (if desired).

Research Concepts:
Antiplatelet Medications:

Tap flag to report any problems with this question.


Question 881: Which condition is treated with norepinephrine?
Choices:
1. Cardiac arrhythmias
2. Stroke
3. Sepsis
4. Arterial occlusion
Answer: 3 - Sepsis
Explanations:
Norepinephrine is a potent alpha one agonist
Norepinephrine acts on both alpha and beta adrenergic receptors and causes
vasoconstriction
Norepinephrine is widely used to treat hypotension in sepsis
Side effects of norepinephrine include ischemia, arrhythmias and
hypertension

Go to the next page if you knew the correct answer, or click the link images
below to further research the concepts in this question (if desired).

Research Concepts:
Norepinephrine:

Tap flag to report any problems with this question.


Question 882: Which of the following statements with respect to an
atrioventricular block is false?

Choices:
1. A third degree block generally requires pacing
2. The risk of atrioventricular block depends on location of the infarction
3. First degree block is an indication for defibrillation
4. Mobitz II second degree is an indication for pacing
Answer: 3 - First degree block is an indication for defibrillation
Explanations:
Patients with first-degree or Mobitz I second degree atrioventricular block
usually requires no cardioversion

Go to the next page if you knew the correct answer, or click the link images
below to further research the concepts in this question (if desired).

Research Concepts:
Heart Block, First Degree:

Tap flag to report any problems with this question.


Question 883: Catheterization findings of pulmonary embolism include all
except which of the following?

Choices:
1. Increased JVP
2. Increased pulmonary artery pressure
3. Increased right ventricular pressure
4. Increased left atrial pressure
Answer: 4 - Increased left atrial pressure
Explanations:
PE causes a variety of hemodynamic alterations that result in elevated
pulmonary vascular resistance. The increased pulmonary vascular
resistance leads to right ventricular overload
Over time the right ventricular overload, lead to right ventricular failure
Other signs of PE may include a loud P2, S3 gallop, elevated JVP, tricuspid
regurgitation and pleural friction rub
The most common physical signs include tachypnea, rales, tachycardia, 4th
heart sound and loud P2

Go to the next page if you knew the correct answer, or click the link images
below to further research the concepts in this question (if desired).

Research Concepts:
Swan-Ganz Catheterization:

Pulmonary Embolism:

Tap flag to report any problems with this question.


Question 884: Which of the following antiarrhythmics can work as a local
anesthetic?

Choices:
1. Amiodarone
2. Lidocaine
3. Procainamide
4. Cocaine
Answer: 2 - Lidocaine
Explanations:
Lidocaine is a class I antiarrhythmic that can be used as a local anesthetic
Amiodarone is a class III antiarrhythmic and it is never used as a local
anesthetic
Cocaine can serve as a local anesthetic

Go to the next page if you knew the correct answer, or click the link images
below to further research the concepts in this question (if desired).

Research Concepts:
Lidocaine:

Local Anesthetics:

Antiarrhythmic Medication:
Tap flag to report any problems with this question.
Question 885: Palliative surgery in a patient with tetralogy of Fallot
requires which of the following?

Choices:
1. Placement of a shunt
2. Enlargement of the ASD
3. Closure of the VSD
4. Dilation of the pulmonary valve
Answer: 1 - Placement of a shunt
Explanations:
The definitive procedure is best done at age 12 months.
The palliative procedure involves the placement of a shunt to increase
pulmonary blood flow.

Go to the next page if you knew the correct answer, or click the link images
below to further research the concepts in this question (if desired).

Research Concepts:
Tetralogy Of Fallot:

Tap flag to report any problems with this question.


Question 886: Which agent can only be given intravenously for congestive
heart failure?

Choices:
1. Digoxin
2. Captopril
3. Quinidine
4. Dobutamine
Answer: 4 - Dobutamine
Explanations:
Dobutamine is an inotropic agent with beta 1 receptor agonist activity.
Besides increasing the force of contraction of the heart, it can also be a
vasodilator to the pulmonary arteries.
Dobutamine does not stimulate dopamine receptors or stimulate the release
of norepinephrine.

Go to the next page if you knew the correct answer, or click the link images
below to further research the concepts in this question (if desired).

Research Concepts:
Heart Failure, Congestive:

Tap flag to report any problems with this question.


Question 887: How long after an thromboembolic event should elective
surgery ideally be postponed?

Choices:
1. 1 week
2. 1 month
3. 3 months
4. 6 months
Answer: 3 - 3 months
Explanations:
Elective surgery should be postponed at least one month.
However, ideally it should be delayed three months.

Go to the next page if you knew the correct answer, or click the link images
below to further research the concepts in this question (if desired).

Research Concepts:
Elective Surgery:

Thromboembolic Event:

Tap flag to report any problems with this question.


Question 888: Which of the following is not released from platelets?
Choices:
1. Dopamine
2. Serotonin
3. ATP
4. Fibrinogen
Answer: 1 - Dopamine
Explanations:
Platelets contain lambda, alpha, and dense granules which contain a number
of chemicals. When platelets activate, these chemicals are released.
Delta granules contain ATP, calcium, and serotonin.
Alpha granules contain platelet factor 4, transforming growth factor,
platelet-derived growth factor, fibrinogen, and factors 5 and 13.
Lambda granules contain lysosomal and several proteolytic enzymes.

Go to the next page if you knew the correct answer, or click the link images
below to further research the concepts in this question (if desired).

Research Concepts:
Platelets:

Tap flag to report any problems with this question.


Question 889: Which of the following is false about verapamil?
Choices:
1. Used to treat hypertension
2. Used to treat supraventricular arrhythmia
3. Used to treat coronary spasm
4. Used to treat dilated cardiomyopathy
Answer: 4 - Used to treat dilated cardiomyopathy
Explanations:
All non dihydropyridine calcium channel blockers are considered to have
negative chronotropic and inotropic effects
These agents are not used to treat dilated cardiomyopathy as they can
worsen the ejection fraction
Calcium channel blockers are effective for hypertension, supraventricular
arrhythmias, and coronary spasm.

Go to the next page if you knew the correct answer, or click the link images
below to further research the concepts in this question (if desired).

Research Concepts:
Calcium Channel blockers:

Tap flag to report any problems with this question.


Question 890: Which of the following properties is not true concerning
lidocaine?

Choices:
1. Lidocaine has minimal inotropic effects
2. CNS toxicity is usually the first manifested sign
3. Lidocaine has arrhythmic effect
4. Lidocaine is metabolized by the liver
Answer: 3 - Lidocaine has arrhythmic effect
Explanations:
Lidocaine has minimal inotropic and no arrhythmic effects

Go to the next page if you knew the correct answer, or click the link images
below to further research the concepts in this question (if desired).

Research Concepts:
Lidocaine:

Tap flag to report any problems with this question.


Question 891: A 60 year old patient has chronic stable angina. The patient
was a heavy smoker but has quit and is exercising. Cardiac catheterization
showed nonobstructing coronary artery disease. The angina occurs after
moderate exercise and is relieved by sublingual nitroglycerin. Select the factor
that would be lease important in causing his angina.

Choices:
1. Heart rate
2. Hematocrit
3. Resistance in epicardial coronary arteries
4. Lung disease
Answer: 3 - Resistance in epicardial coronary arteries
Explanations:
Coronary blood flow can be limited in 3 regions: arteriolar and
intramyocardial capillaries, prearteriolar vessels, and epicardial vessels
The cardiac catheterization indicated that there is not significant flow
resistance in the epicardial arteries, thus the resistance is secondary to the
arteriolar and intramyocardial capillaries and the prearteriolar vessels
Heart rate, lung function, and hematocrit are factors that affect myocardial
demand and supply

Go to the next page if you knew the correct answer, or click the link images
below to further research the concepts in this question (if desired).

Research Concepts:
Angina, Stable:

Tap flag to report any problems with this question.


Question 892: Which of the following is the correct order for weaning a
patient from mechanical ventilation?

Choices:
1. Decrease FIO2, then PEEP, then rate, then ABG
2. Decrease PEEP, then FIO2, then rate, then ABG
3. Decrease FIO2, then PEEP, then ABG, then rate
4. Decrease rate, then FIO2, then PEEP, then ABG
Answer: 1 - Decrease FIO2, then PEEP, then rate, then ABG
Explanations:
Always decrease FI02 first, then decrease PEEP and ventilator rate,
followed by a repeat ABG.

Go to the next page if you knew the correct answer, or click the link images
below to further research the concepts in this question (if desired).

Research Concepts:
Ventilator Management:

Tap flag to report any problems with this question.


Question 893: Which of the following most commonly predisposes patient
to pulmonary embolism?

Choices:
1. Diabetes
2. Varicose veins
3. Thoracic surgery
4. Hypertension
Answer: 3 - Thoracic surgery
Explanations:
Hypercoagulability, oral contraceptive use, and deep venous stasis are
predisposing factors for PE.
Any surgery is a risk factor for PE.
Varicose veins on the legs are superficial, not deep, veins.
Neither diabetes nor hypertension are viewed as significant risk factors for
pulmonary embolism (good thing, as that would make PE even more
common than it is!).

Go to the next page if you knew the correct answer, or click the link images
below to further research the concepts in this question (if desired).

Research Concepts:
Pulmonary Embolism:

Tap flag to report any problems with this question.


Question 894: Which is least likely to be found as a normal consequence
of aging?

Choices:
1. Elevated mean arterial blood pressure
2. Elevated systolic blood pressure
3. Elevated diastolic blood pressure
4. Elevated peripheral vascular resistance
Answer: 3 - Elevated diastolic blood pressure
Explanations:
Mean arterial blood pressure is normally elevated with aging.
This is primarily caused by an elevated systolic blood pressure.
Diastolic blood pressure is usually not increased in the elderly.

Go to the next page if you knew the correct answer, or click the link images
below to further research the concepts in this question (if desired).

Research Concepts:
Blood Pressure, Age-related Changes:

Tap flag to report any problems with this question.


Question 895: Performing CPR with chest compressions over the xiphoid
process may result in laceration of which of the following organs?

Choices:
1. Small bowel
2. Stomach
3. Lung
4. Liver
Answer: 4 - Liver
Explanations:
Because of its location just underneath the xiphoid process, the liver is
likely to be damaged from physical pressure exerted over the xiphoid
process during CPR

Go to the next page if you knew the correct answer, or click the link images
below to further research the concepts in this question (if desired).

Research Concepts:
Cardiopulmonary Resuscitation:

Complications:

Tap flag to report any problems with this question.


Question 896: Patients taking furosemide should also be given:
Choices:
1. Spironolactone
2. Indapamide
3. Magnesium
4. Potassium
Answer: 4 - Potassium
Explanations:
Potassium supplementation is needed in patients taking loop diuretics
Loop diuretics commonly result in electrolyte disturbances
This is particularly dangerous for patients who are also taking digoxin
KCl oral tablets are used to supplement the potassium lost during diuretic
therapy

Go to the next page if you knew the correct answer, or click the link images
below to further research the concepts in this question (if desired).

Research Concepts:
Loop Diuretics:

Hypokalemia:

Tap flag to report any problems with this question.


Question 897: Which of the following would suggest that a murmur found
in a 5-year-old boy is not benign?

Choices:
1. It is systolic
2. It is grade I or II/VI
3. It does not radiate
4. It is holosystolic
Answer: 4 - It is holosystolic
Explanations:
Further evaluation should be prompted by a grade IV through VI murmur,
radiation, a diastolic murmur, a holosystolic murmur, or the presence of
other cardiac symptoms.

Go to the next page if you knew the correct answer, or click the link images
below to further research the concepts in this question (if desired).

Research Concepts:
Murmur:

Tap flag to report any problems with this question.


Question 898: Which of the following would not be elevated in acute
myocardial infarction?

Choices:
1. Troponin
2. Creatine kinase
3. Alkaline phosphatase
4. Aspartate aminotransferase
Answer: 3 - Alkaline phosphatase
Explanations:
Alkaline phosphatase is found in the biliary tree and in bone, not the
myocardium

Go to the next page if you knew the correct answer, or click the link images
below to further research the concepts in this question (if desired).

Research Concepts:
Myocardial Infarction, Serum Markers:

Tap flag to report any problems with this question.


Question 899: Which of the following congenital heart diseases is most
likely to be associated with Ehlers-Danlos syndrome?

Choices:
1. Coarctation of the aorta
2. ASD
3. Mitral valve prolapse
4. Pulmonic stenosis
Answer: 3 - Mitral valve prolapse
Explanations:
Ehler-Danlos syndrome is a hereditary connective tissue disorder
Mitral valve prolapse is a common manifestation of this disease
The skin is unusually very elastic due to defect in collagen synthesis

Go to the next page if you knew the correct answer, or click the link images
below to further research the concepts in this question (if desired).

Research Concepts:
Ehlers-Danlos Syndrome:

Tap flag to report any problems with this question.


Question 900: Cardioplegia solution functions to do which of the
following?

Choices:
1. Increase oxygen demand
2. Increase heart rate
3. Decrease contractility
4. Decrease peripheral resistance
Answer: 3 - Decrease contractility
Explanations:
Cardioplegia solution decreases cardiac oxygen consumption, thereby
allowing for a greater length of time for surgical intervention.

Go to the next page if you knew the correct answer, or click the link images
below to further research the concepts in this question (if desired).

Research Concepts:
Cardioplegia:

Tap flag to report any problems with this question.


Question 901: Which of the following medications should be given for a
ductal-dependent cyanotic congenital heart lesion?

Choices:
1. Indomethacin infusion
2. Prostaglandin E infusion
3. Adenosine infusion
4. Digoxin infusion
Answer: 2 - Prostaglandin E infusion
Explanations:
Some congenital heart diseases are dependent on patency of the ductus
Without a patent ductus some of cyanotic congenital heart diseases are not
compatible with life
Prostaglandin E infusion is used to keep the ductus open after birth
Otherwise the ductus closes spontaneously within 24 hours after birth

Go to the next page if you knew the correct answer, or click the link images
below to further research the concepts in this question (if desired).

Research Concepts:
Cyanotic Heart Disease:

Tap flag to report any problems with this question.


Question 902: Which of the following is associated with cannon A waves?
Choices:
1. Atrial fibrillation
2. Heart block
3. Ventricular fibrillation
4. Mitral regurgitation
Answer: 2 - Heart block
Explanations:
Cannon A waves are seen in the jugular vein in heart block.
The atria are unable to empty the blood in the ventricles because of the
heart block. This is reflected as cannon A waves.
Cannon A waves can also cause pulsations in the neck and jaw pain.

Go to the next page if you knew the correct answer, or click the link images
below to further research the concepts in this question (if desired).

Research Concepts:
Heart Block, Third Degree:

Cannon A Waves:

Tap flag to report any problems with this question.


Question 903: In adults, what is the most common cause of chronic
unilateral lower extremity edema?

Choices:
1. Lymphedema
2. Deep vein thrombosis
3. Superficial venous insufficiency
4. Cirrhosis
Answer: 3 - Superficial venous insufficiency
Explanations:
Venous insufficiency is the most common cause of unilateral leg edema,
whereas lymphedema is rare.

Go to the next page if you knew the correct answer, or click the link images
below to further research the concepts in this question (if desired).

Research Concepts:
Edema:

Venous Insufficiency:

Tap flag to report any problems with this question.


Question 904: Which of the following findings is most significant in a
newborn with ectopic cordis?

Choices:
1. Intestinal contents coming out of the abdomen
2. Diaphragmatic hernia
3. Heart protruding out of the sternum
4. Large head
Answer: 3 - Heart protruding out of the sternum
Explanations:
Ectopic is a rare birth defect which is characterized by the heart protruding
outside the sternum.
The disorder is also associated with other anomalies and is usually fatal.
Ectopic cordis is a rare lesion often seen in patients with Turner syndrome
and trisomy 18.

Go to the next page if you knew the correct answer, or click the link images
below to further research the concepts in this question (if desired).

Research Concepts:
Ectopic Cordis:

Tap flag to report any problems with this question.


Question 905: A CO2 level of one percent indicates the endotrachial tube
is located in the:

Choices:
1. Trachea
2. Esophagus
3. Right bronchi
4. Left bronchi
Answer: 2 - Esophagus
Explanations:
A normal CO2 detector level is five percent at the end of an ET tube

Go to the next page if you knew the correct answer, or click the link images
below to further research the concepts in this question (if desired).

Research Concepts:
Carbon Dioxide Detector:

Tracheal Intubation:

Tap flag to report any problems with this question.


Question 906: In a patient with a high likelihood of a pulmonary embolus,
which of the following would be seen in the blood work?

Choices:
1. Decreased D-dimer
2. Hypocarbia
3. Increased platelets
4. Depressed prothrombin time
Answer: 2 - Hypocarbia
Explanations:
Patients with a pulmonary embolus will usually have tachypnea that will
manifest as hypocarbia. Initially, hypoxia may not be that pronounced.
The chest x-ray is often normal.
The D-dimer is increased.

Go to the next page if you knew the correct answer, or click the link images
below to further research the concepts in this question (if desired).

Research Concepts:
Pulmonary Embolism:

Tap flag to report any problems with this question.


Question 907: Which of the following associations between atherosclerosis
and diabetes mellitus is NOT true?

Choices:
1. Diabetics have more extensive coronary artery disease
2. Diabetics have more unstable lesions
3. Diabetics have less favorable long-term outcomes with coronary
revascularization (e.g. CABG, PTCA)
4. Diabetics are less likely to have peripheral vascular disease
Answer: 4 - Diabetics are less likely to have peripheral vascular disease
Explanations:
Medical treatment decisions for diabetics with UA/NSTEMI are very
similar to nondiabetics
Diabetes is a comorbidity in ~20% of patients with UA/NSTEMI
It is an independent predictor of adverse outcomes.
Coronary revascularization in diabetics has less favorable outcomes when
compared to nondiabetics

Go to the next page if you knew the correct answer, or click the link images
below to further research the concepts in this question (if desired).

Research Concepts:
Angina, Unstable:

Diabetes Mellitus:

Tap flag to report any problems with this question.


Question 908: Which of the following groups is not an absolute
contraindication for use of a thrombolytic agent?

Choices:
1. Patients with MI 30 minutes prior
2. Patients with CVA 5 months ago
3. Pregnant patients
4. Patients with a known AV malformation
Answer: 1 - Patients with MI 30 minutes prior
Explanations:
Thrombolic therapy of acute MI is most beneficial if initiated within the
first hour of onset
Common absolute contraindications include CVA less than 6 months, high
risk for internal bleeding, recent surgery, pregnancy, and known bleeding
disorder

Go to the next page if you knew the correct answer, or click the link images
below to further research the concepts in this question (if desired).

Research Concepts:
Thrombolytic Therapy:

Tap flag to report any problems with this question.


Question 909: Which treatment modality should be considered first for
pulmonary embolism?

Choices:
1. Thrombectomy
2. Heparin
3. Mechanical ventilation
4. Clopidogrel
Answer: 2 - Heparin
Explanations:
Treatment of pulmonary embolus include administration of thrombolytics
and heparin, and rarely insertion of an IVC filter.

Go to the next page if you knew the correct answer, or click the link images
below to further research the concepts in this question (if desired).

Research Concepts:
Heparin:

Pulmonary Embolism:

Tap flag to report any problems with this question.


Question 910: A 68-year-old female is admitted with a broken hip after a
fall. She undergoes a left hip replacement that was uneventful. After spending 2
days in bed, she undergoes partial non-weight bearing physiotherapy. Five days
later, she has a swollen leg and a deep vein thrombosis is discovered. An arterial
blood gas evaluation is done for suspected pulmonary embolism. What is the
most consistent finding of a pulmonary embolism?

Choices:
1. Elevated PaCO2
2. Decreased A-a gradient
3. Right axis deviation
4. Decreased PO2
Answer: 4 - Decreased PO2
Explanations:
There is not a blood or serum test that can help make a diagnosis of PE.
Hypoxemia is a constant feature but is not very specific for PE.
The WBC count may be normal or elevated. Clotting parameters have no
prognostic value.
The D-dimer screen test is only useful if used in conjunction with clinical
assessment of patient. D-dimer may also be positive in infection, trauma,
and malignancies.
The initial chest x-ray in PE is almost always normal but sometimes may
show the Westermarck sign, dilated pulmonary vessels proximal to the clot
with collapse of distal vessels and a sharp cutoff.

Go to the next page if you knew the correct answer, or click the link images
below to further research the concepts in this question (if desired).

Research Concepts:
Pulmonary Embolism:

Tap flag to report any problems with this question.


Question 911: A 4-month-old infant is seen at another hospital and the x-
ray report indicates that he has Coeur en Sabot. Which of the following is the
correct diagnosis?

Choices:
1. Pes planus
2. Tetralogy of fallot
3. Sickle cell disease
4. Kartegener's syndrome
Answer: 2 - Tetralogy of fallot
Explanations:
Coeur en Sabot or a boot shaped heart is seen commonly in patients with
tetralogy of fallot.
Tetralogy of fallot is associated with a right to left shunt and cyanosis.
On x-ray, the pulmonary segment is usually small and concave suggesting
pulmonary stenosis or atresia.

Go to the next page if you knew the correct answer, or click the link images
below to further research the concepts in this question (if desired).

Research Concepts:
Tetralogy Of Fallot:

Tap flag to report any problems with this question.


Question 912: Atrial contraction is responsible for which of the following
venous pulses?

Choices:
1. V wave
2. X descent
3. C wave
4. A wave
Answer: 4 - A wave
Explanations:
The positive A wave is caused by the right atrial contraction transmitted to
the jugular veins during right atrial systole
Atrial relaxation is demonstrated by the descent of the A wave. This is
usually interrupted by the C wave
The X descent is a negative wave occurring in late systole secondary to the
delay in pulse transmission.
The v wave is the increase in right atrial and jugular venous pressure
secondary to continued inflow of blood to the venous system. It occurs in
late ventricular systole while the tricuspid valve is still closed.

Go to the next page if you knew the correct answer, or click the link images
below to further research the concepts in this question (if desired).

Research Concepts:
A Wave:

Tap flag to report any problems with this question.


Question 913: Which of the following factors most increases the incidence
of sternal wound infections most after bilateral mammary usage?

Choices:
1. Diabetes
2. Obesity
3. Prolonged ventilation
4. Rebleeding
Answer: 1 - Diabetes
Explanations:
All the above are risk factors for sternal wound infections, but the
association with diabetes is the strongest

Go to the next page if you knew the correct answer, or click the link images
below to further research the concepts in this question (if desired).

Research Concepts:
Post-op Wound Infection:

Coronary Artery Bypass Graft:

Tap flag to report any problems with this question.


Question 914: Which of the following antihypertensive is contraindicated
in patients with COPD?

Choices:
1. Atenolol
2. Esmolol
3. Metoprolol
4. Nadolol
Answer: 4 - Nadolol
Explanations:
Non-selective beta-blockers are contraindicated in COPD due to their
bronchoconstriction effects
Only nadolol is a non selective beta blocker and therefore contraindicated

Go to the next page if you knew the correct answer, or click the link images
below to further research the concepts in this question (if desired).

Research Concepts:
Obstructive Pulmonary Disease, Chronic (COPD):

Beta-Blockers:

Tap flag to report any problems with this question.


Question 915: To which side effect of morphine does tolerance not
develop?

Choices:
1. Sedation
2. Respiratory depression
3. Nausea
4. Constipation
Answer: 4 - Constipation
Explanations:
Tolerance develops to most effects of morphine except to constipation and
miosis

Go to the next page if you knew the correct answer, or click the link images
below to further research the concepts in this question (if desired).

Research Concepts:
Morphine:

Tap flag to report any problems with this question.


Question 916: Which of the following is a symptom of oxygen toxicity?
Choices:
1. Diarrhea
2. Cataracts
3. Seizures
4. None of the above
Answer: 3 - Seizures
Explanations:
Seizures can be a symptom of oxygen toxicity
Pulmonary symptoms include dyspnea and chest pain
Ocular symptoms may include detachment of the retina
Nausea but not diarrhea can be a symptom

Go to the next page if you knew the correct answer, or click the link images
below to further research the concepts in this question (if desired).

Research Concepts:
Oxygen Toxicity:

Tap flag to report any problems with this question.


Question 917: What is the major complication of fibrinolytic therapy?
Choices:
1. Renal failure
2. Hemorrhagic stroke
3. Infection
4. Liver failure
Answer: 2 - Hemorrhagic stroke
Explanations:
The most common serious adverse effects of the fibrinolytics is bleeding,
especially in the brain.
Unlike heparin, they do not induce anti-platelet antibodies.
Streptokinase is a bactericidal product and can induce an allergic reaction.

Go to the next page if you knew the correct answer, or click the link images
below to further research the concepts in this question (if desired).

Research Concepts:
Thrombolytic Therapy:

Tap flag to report any problems with this question.


Question 918: A post cardiac transplant patient develops dyspnea on
exertion 48 months after transplant. With regard to post transplant coronary
artery disease, which of the following is TRUE:

Choices:
1. Coronary disease develops at a rate of 5% per year
2. Angina is the most common presenting symptom
3. CABG in post transplant has 50% mortality
4. Cytomegalovirus is the greatest infectious cause of developing coronary
disease
Answer: 3 - CABG in post transplant has 50% mortality
Explanations:
CAD develops at 10% per year and dyspnea on exertion is the most
common presentation
Epstein Barr virus is the most common cause of CAD in transplanted hearts

Go to the next page if you knew the correct answer, or click the link images
below to further research the concepts in this question (if desired).

Research Concepts:
Heart Transplantation:

Coronary Artery Disease:

Tap flag to report any problems with this question.


Question 919: Which of the following is true regarding ventricular septal
defect?

Choices:
1. It occurs right after birth
2. It is the most common congenital cardiac anomaly
3. Membranous type VSDs can close spontaneously
4. It causes cyanosis
Answer: 2 - It is the most common congenital cardiac anomaly
Explanations:
VSD are the most common congenital cardiac anomalies in children
It usually becomes symptomatic in the first few months after birth
It rarely becomes symptomatic at birth because the pulmonary and system
pressure are almost equal so that left right shunting does not occur
Muscular type VSDs may close spontaneously, but not the membranous
types

Go to the next page if you knew the correct answer, or click the link images
below to further research the concepts in this question (if desired).

Research Concepts:
Ventricular Septal Defect:

Tap flag to report any problems with this question.


Question 920: A 7-year-old child presented with severe headache. Blood
pressure measurement revealed severe hypertension with 110/95 mm Hg. What
is the most likely cause of hypertension in children?

Choices:
1. Renal parenchymal disease
2. Essential hypertension
3. Pheochromocytoma
4. Renal vascular disease
Answer: 2 - Essential hypertension
Explanations:
Essential hypertension is the most common cause of hypertension in
children
Family history of such disease is important since it often runs in families

Go to the next page if you knew the correct answer, or click the link images
below to further research the concepts in this question (if desired).

Research Concepts:
Essential Hypertension:

Tap flag to report any problems with this question.


Question 921: Lidocaine is metabolized by:
Choices:
1. Liver
2. Kidney
3. Serum enzymes
4. GI mucosa
Answer: 1 - Liver
Explanations:
Lidocaine is metabolized in the liver

Go to the next page if you knew the correct answer, or click the link images
below to further research the concepts in this question (if desired).

Research Concepts:
Lidocaine:

Tap flag to report any problems with this question.


Question 922: Which lipid lowering medication is the most effective at
increasing high-density lipoprotein (HDL) levels?

Choices:
1. Niacin
2. Atorvastatin
3. Lisinopril
4. Colesevelam
Answer: 1 - Niacin
Explanations:
While atorvastatin, lisinopril and colesevelam have been shown to increase
HDL levels, the most effective is niacin.
Niacin can increase HDL by 30% but this has NOT been shown to reduce
cardiac mortality.
The major troublesome side effect is flushing.
This can be decreased by pretreating with aspirin.

Go to the next page if you knew the correct answer, or click the link images
below to further research the concepts in this question (if desired).

Research Concepts:
Niacin:

Tap flag to report any problems with this question.


Question 923: Which of the following is not a cause of hypochromic
microcytic anemia?

Choices:
1. Acute bleeding
2. Pregnancy
3. Aluminum toxicity
4. Thalassemia
Answer: 1 - Acute bleeding
Explanations:
Acute bleeding is associated with normochromic anemia

Go to the next page if you knew the correct answer, or click the link images
below to further research the concepts in this question (if desired).

Research Concepts:
Microcytic Hypochromic Anemia:

Tap flag to report any problems with this question.


Question 924: Which of the following is a common medication in the
treatment of congestive heart failure (CHF)?

Choices:
1. Diuretics
2. Beta blockers
3. ACE inhibitors
4. All of the above
Answer: 4 - All of the above
Explanations:
Diuretics are one of the mainstays of treating CHF
Diuretics can cause hypokalemia and as a result patients require potassium
supplementation
Beta blockers are another mainstay of treatment and decrease mortality
rates
ACE inhibitors have been show to improve quality of life and mortality

Go to the next page if you knew the correct answer, or click the link images
below to further research the concepts in this question (if desired).

Research Concepts:
Heart Failure, Congestive:

Tap flag to report any problems with this question.


Question 925: Which is not a risk factor for postoperative delirium?
Choices:
1. Elderly
2. Alzheimer disease
3. Female gender
4. Memory impairment
Answer: 3 - Female gender
Explanations:
Postoperative delirium is associated with age, male gender, and history of
dementia

Go to the next page if you knew the correct answer, or click the link images
below to further research the concepts in this question (if desired).

Research Concepts:
Postoperative Delirium:

Tap flag to report any problems with this question.


Question 926: Cholesterol levels are expected to be the lowest in which of
the following situations?

Choices:
1. Pregnancy
2. Post myocardial infarction
3. Post prandial
4. After an overnight fast
Answer: 2 - Post myocardial infarction
Explanations:
Cholesterol levels will decrease following myocardial infarction
This decrease can persist up to weeks post infarction
Cholesterol levels are higher during pregnancy and during post-prandial
period
Cholesterol levels are low after an overnight fast

Go to the next page if you knew the correct answer, or click the link images
below to further research the concepts in this question (if desired).

Research Concepts:
Acute Myocardial Infarction:

Cholesterol Levels:

Tap flag to report any problems with this question.


Question 927: After discontinuation of smoking, what will the lipid profile
of a 50-year old will show?

Choices:
1. Increased HDL
2. Decreased HDL
3. Increased LDL
4. Decreased LDL
Answer: 1 - Increased HDL
Explanations:
A 5 to 10 mg/dL increase in HDL should be expected in a patient who quits
smoking.

Go to the next page if you knew the correct answer, or click the link images
below to further research the concepts in this question (if desired).

Research Concepts:
Smoking Cessation:

High-density Lipoprotein:

Tap flag to report any problems with this question.


Question 928: Systolic reversal flow is most often seen in:
Choices:
1. Left coronary artery
2. Right subclavian artery
3. Femoral artery
4. Right renal artery
Answer: 1 - Left coronary artery
Explanations:
Due to high ventricular pressure, systolic reversal flow is normal on the left
side

Go to the next page if you knew the correct answer, or click the link images
below to further research the concepts in this question (if desired).

Research Concepts:
Cardiac Physiology:

Tap flag to report any problems with this question.


Question 929: Which one of the following drug shares properties with all
classes of antiarrhythmics?

Choices:
1. Procainamide
2. Lidocaine
3. Propranolol
4. Amiodarone
Answer: 4 - Amiodarone
Explanations:
Amiodarone shares the properties of all the major classes of antiarrhythmics
Antiarrhythmics are classified based on their main mechanism of action
Propranolol is a class two and procainamide is a class three
Lidocaine is a class one antiarrhythmic drug

Go to the next page if you knew the correct answer, or click the link images
below to further research the concepts in this question (if desired).

Research Concepts:
Amiodarone:

Tap flag to report any problems with this question.


Question 930: A central line on the right side of the neck does not usually
cause which complication?

Choices:
1. Chylothorax
2. Pneumothorax
3. Air embolism
4. Hematoma
Answer: 1 - Chylothorax
Explanations:
The thoracic duct empties in the left side only
The right lymphatic duct is less likely to be traumatized from a central line

Go to the next page if you knew the correct answer, or click the link images
below to further research the concepts in this question (if desired).

Research Concepts:
Central Line Placement:

Tap flag to report any problems with this question.


Question 931: Select the condition that would not produce similar
symptoms to esophageal spasm.

Choices:
1. Angina pectoris
2. Pulmonary embolism
3. Costochondritis
4. Gastroesophageal reflux disease
Answer: 2 - Pulmonary embolism
Explanations:
The symptoms of pulmonary embolism are least likely to resemble
esophageal spasms
Other conditions that might mimic esophageal spasms are diabetic
neuropathy, esophageal cancer, achalasia, esophageal diverticula, strictures,
webs, and rings

Go to the next page if you knew the correct answer, or click the link images
below to further research the concepts in this question (if desired).

Research Concepts:
Pulmonary Embolism:

Esophageal Spasm:

Tap flag to report any problems with this question.


Question 932: All of the following factors affect creatinine clearance,
except:

Choices:
1. Muscle mass
2. Age
3. Gender
4. Hydration status
Answer: 4 - Hydration status
Explanations:
The normal range for creatinine clearance values is 100 - 125 ml / minute /
1.73 m2
Creatinine level is higher in males and varies according to muscle mass
The creatinine clearance decreases with age
Hydration status affects levels of BUN but creatinine clearance remains
relatively constant

Go to the next page if you knew the correct answer, or click the link images
below to further research the concepts in this question (if desired).

Research Concepts:
Creatinine Clearance:

Tap flag to report any problems with this question.


Question 933: Which of the following diseases is caused by an accessory
atrioventricular conduction pathway?

Choices:
1. Paroxysmal ventricular tachycardia
2. Paroxysmal supraventricular tachycardia
3. Wolff-Parkinson-White syndrome
4. Atrial fibrillation
Answer: 3 - Wolff-Parkinson-White syndrome
Explanations:
Cardiac depolarization from the atria to the ventricles normally occurs via
the atrioventricular node-bundle of His-Purkinje system.
Patients with WPW syndrome have accessory AV conduction via a bypass
tract.
This results in preexcitation of the ventricles and tachycardia.
Radiofrequency ablation of the accessory pathway is the definitive
treatment.

Go to the next page if you knew the correct answer, or click the link images
below to further research the concepts in this question (if desired).

Research Concepts:
Wolff-Parkinson-White Syndrome:

Tap flag to report any problems with this question.


Question 934: PEEP can increase:
Choices:
1. Peripheral vascular resistance
2. Shunting
3. Pulmonary wedge pressure
4. Carbon monoxide levels
Answer: 1 - Peripheral vascular resistance
Explanations:
Hemodynamically, PEEP causes an increase in pulmonary vascular
resistance.

Go to the next page if you knew the correct answer, or click the link images
below to further research the concepts in this question (if desired).

Research Concepts:
Positive End-Expiratory Pressure (PEEP):

Tap flag to report any problems with this question.


Question 935: What is the first neurologic feature of lidocaine toxicity?
Choices:
1. Seizures
2. Weakness
3. Numbness
4. Confusion
Answer: 3 - Numbness
Explanations:
Lidocaine is a myocardial depressant.
The first sign of central toxicity is perioral numbness.

Go to the next page if you knew the correct answer, or click the link images
below to further research the concepts in this question (if desired).

Research Concepts:
Toxicity, Lidocaine:

Tap flag to report any problems with this question.


Question 936: Which is the most common organism causing cellulitis?
Choices:
1. S. pyogenes
2. Bacteroides
3. Corynebacterium
4. Clostridium
Answer: 1 - S. pyogenes
Explanations:
Staphylococci still remain the most common cause of cellulitis.
Group A Streptococci are also a common cause of cellulitis.
Wounds contaminated by fresh water grow Aeromonas hydrophila, a Gram-
negative rod.
Salt water and brackish water carry Vibrio vulnificus that can cause wound
cellulitis.

Go to the next page if you knew the correct answer, or click the link images
below to further research the concepts in this question (if desired).

Research Concepts:
Cellulitis:

Tap flag to report any problems with this question.


Question 937: All of the following are true regarding amiodarone
EXCEPT:

Choices:
1. Can increase levels of simvastatin
2. Can increase levels of lovastatin
3. Can increase levels of pravastatin
4. Can increase levels of atorvastatin
Answer: 3 - Can increase levels of pravastatin
Explanations:
Pravastatin is the only statin with minimal to no metabolism by the
cytochrome P450 system
Amiodarone inhibits the cytochrome P450 enzyme system
As a result, it can increase serum levels of simvastatin, lovastatin and
atorvastatin
It does not increase levels of pravastatin

Go to the next page if you knew the correct answer, or click the link images
below to further research the concepts in this question (if desired).

Research Concepts:
Amiodarone:

Statin Medication:

Drug Interactions:
Tap flag to report any problems with this question.
Question 938: How many hours after admission is nosocomial pneumonia
defined according to the Center for Disease Control?

Choices:
1. 24
2. 48
3. 72
4. None of the above
Answer: 3 - 72
Explanations:
A nosocomial pneumonia is one defined as occurring >72 hours post-
admission
Other names include hospital-acquired pneumonia or health care-associated
pneumonia
It is the second most common nosocomial infection
It is typically bacterial in origin

Go to the next page if you knew the correct answer, or click the link images
below to further research the concepts in this question (if desired).

Research Concepts:
Nosocomial Pneumonia:

Tap flag to report any problems with this question.


Question 939: What is the most appropriate intervention for a patient who
is hemodynamically stable and develops complete heart block following an
anterior myocardial infarction?

Choices:
1. IV lidocaine
2. IV epinephrine
3. Pacemaker placement
4. Electrical cardioversion
Answer: 3 - Pacemaker placement
Explanations:
When complete heart block occurs following an acute MI, a pacemaker is
needed.
On ECG, complete heart block is diagnosed by P waves and QRS
complexes that are dissociated, indicating that the atria and ventricles are
operating independently.
Lidocaine is contraindicated in this situation because it may cause asystole
due to its ability to suppress the ventricular pacemaker.

Go to the next page if you knew the correct answer, or click the link images
below to further research the concepts in this question (if desired).

Research Concepts:
Heart Block, Third Degree:

Myocardial Infarction, Acute:

Tap flag to report any problems with this question.


Question 940: Which is not part of the Beck triad for cardiac tamponade?
Choices:
1. Hypotension
2. Hypertension
3. Muffled heart sound
4. High JVD
Answer: 2 - Hypertension
Explanations:
Beck triad includes hypotension

Go to the next page if you knew the correct answer, or click the link images
below to further research the concepts in this question (if desired).

Research Concepts:
Cardiac Tamponade:

Tap flag to report any problems with this question.


Question 941: The most important deformity in tetralogy of Fallot is:
Choices:
1. Anterior and leftward displacement of the infundibular septum
2. Over riding aorta
3. Right ventricular hypertrophy
4. Ventricular septal defect
Answer: 1 - Anterior and leftward displacement of the infundibular septum
Explanations:
Although Fallot's original idea included a combination of right ventricular
outflow tract obstruction, overriding aorta, malalignment ventricular septal
defect, and right ventricular hypertrophy, current thinking implicates
anteriocephalad deviation of the outlet septum as the primary cause

Go to the next page if you knew the correct answer, or click the link images
below to further research the concepts in this question (if desired).

Research Concepts:
Tetralogy Of Fallot:

Tap flag to report any problems with this question.


Question 942: What cardiac condition is often present with chronic
untreated mitral stenosis?

Choices:
1. Hypertension
2. Atrial fibrillation
3. Mitral regurgitation
4. Enlarge left ventricle
Answer: 2 - Atrial fibrillation
Explanations:
Untreated mitral stenosis leads to enlargement of left atrium
Leads to risk of atrial fibrillation

Go to the next page if you knew the correct answer, or click the link images
below to further research the concepts in this question (if desired).

Research Concepts:
Mitral Stenosis:

Atrial Fibrillation:

Tap flag to report any problems with this question.


Question 943: Which choice correctly matches percent body volume loss
by hemorrhage to signs and symptoms?

Choices:
1. 10%, mild tachycardias, normal capillary refill, and clear sensorium
2. 15%, tachycardia, normotensive with widened pulse pressure, decreased
level of consciousness
3. 20%, tachycardia without palpable pulses, clear sensorium
4. 30%, tachycardia, normotensive, clear sensorium
Answer: 1 - 10%, mild tachycardias, normal capillary refill, and clear
sensorium

Explanations:
Class I shock has 15% loss of blood volume may be asymptomatic or have
mild tachycardia
Class II shock has a 15 to 30% blood loss, tachycardia, anxiety, and is
normotensive with increased pulse pressure
Class III shock has 30 to 40% blood loss, tachycardia, hypotension, and
confusion
Class IV shock has >40% blood loss, tachycardia, hypotension, and a
decreased level of consciousness

Go to the next page if you knew the correct answer, or click the link images
below to further research the concepts in this question (if desired).

Research Concepts:
Hemorrhage:

Tap flag to report any problems with this question.


Question 944: The left atrium and left ventricle receive blood from which
of the following arteries?

Choices:
1. Left anterior descending
2. Posterior descending
3. Circumflex artery
4. Marginal artery
Answer: 3 - Circumflex artery
Explanations:
The circumflex coronary artery is a branch of the left main coronary artery.
It travels in the groove that separates the left atrium from the left ventricle.
It supplies the left atrium and the left ventricle.
It can give rise to one or more diagonal or marginal arteries.

Go to the next page if you knew the correct answer, or click the link images
below to further research the concepts in this question (if desired).

Research Concepts:
Coronary Artery Anatomy:

Tap flag to report any problems with this question.


Question 945: Which of the followings is not a major determinant of
myocardial oxygen demand?

Choices:
1. Diastolic pressure
2. Pulse
3. Ventricular contractility
4. Ventricular wall tension
Answer: 1 - Diastolic pressure
Explanations:
The three major determinants of myocardial oxygen demand are heart rate,
contractility, and wall tension

Go to the next page if you knew the correct answer, or click the link images
below to further research the concepts in this question (if desired).

Research Concepts:
Myocardial Oxygen Demand:

Tap flag to report any problems with this question.


Question 946: Which is a clinical sign of cardiac tamponade?
Choices:
1. Diminished S2 sound
2. Tachycardia
3. Hypertension
4. Slow breath rate
Answer: 1 - Diminished S2 sound
Explanations:
Heart sounds are usually diminished in cardiac tamponade.

Go to the next page if you knew the correct answer, or click the link images
below to further research the concepts in this question (if desired).

Research Concepts:
Cardiac Tamponade:

Tap flag to report any problems with this question.


Question 947: The right middle lobe can be auscultated in what area of the
chest?

Choices:
1. Midsternum
2. Right posterior
3. Right midclavicular
4. Right apex
Answer: 3 - Right midclavicular
Explanations:
The right middle lobe is located on the right side and is situated
anteroinferior.
Auscultation is best heard at the right anterior midclavicular line.
It can also be auscultated around the right midaxillary line.

Go to the next page if you knew the correct answer, or click the link images
below to further research the concepts in this question (if desired).

Research Concepts:
Lung Exam:

Tap flag to report any problems with this question.


Question 948: Hyperkalemia can be treated with which medication?
Choices:
1. Magnesium
2. Albuterol
3. Atenolol
4. Fluid restriction
Answer: 2 - Albuterol
Explanations:
Hyperkalemia is treated with insulin, bicarbonate, sodium polystyrene
sulfonate, or a beta agonist.
Calcium is given first for cardiac stabilization.

Go to the next page if you knew the correct answer, or click the link images
below to further research the concepts in this question (if desired).

Research Concepts:
Hyperkalemia:

Tap flag to report any problems with this question.


Question 949: Which of the following cells are found in greatest number in
a wound 2 days post injury?

Choices:
1. Fibroblasts
2. Macrophages
3. Neutrophils
4. Mast cells
Answer: 3 - Neutrophils
Explanations:
The first phase of wound healing is the inflammatory phase.
The inflammatory phase lasts 24-48 hours and is characterized by an influx
of neutrophils.
The neutrophils remove debris and bacteria from the wound so as to allow
collagen synthesis to begin.

Go to the next page if you knew the correct answer, or click the link images
below to further research the concepts in this question (if desired).

Research Concepts:
Wound Healing Phases:

Tap flag to report any problems with this question.


Question 950: Which of the following medications act on the loop of
Henle?

Choices:
1. Furosemide
2. Hydrochlorothiazide
3. Amiloride
4. Acetazolamide
Answer: 1 - Furosemide
Explanations:
The loop of Henle is found between the proximal and distal tubules
The thick ascending limb of loop of Henle is impermeable to water and
permeable to electrolytes
Furosemide acts on this site to inhibit absorption of electrolytes and
subsequent diuresis
Loop diuretics are common causes of electrolyte imbalance

Go to the next page if you knew the correct answer, or click the link images
below to further research the concepts in this question (if desired).

Research Concepts:
Furosemide:

Tap flag to report any problems with this question.


Question 951: In the recovery room after elective surgery, how often
should vital signs be obtained?

Choices:
1. Every minute
2. Every five minutes
3. Every fifteen minutes
4. Every twenty minutes
Answer: 3 - Every fifteen minutes
Explanations:
Vitals are normally taken every fifteen minutes in the PACU.

Go to the next page if you knew the correct answer, or click the link images
below to further research the concepts in this question (if desired).

Research Concepts:
Postoperative Recovery:

Tap flag to report any problems with this question.


Question 952: What percent of the original drug is left after three half-
lives?

Choices:
1. 50 percent
2. 25 percent
3. 12 percent
4. 6 percent
Answer: 3 - 12 percent
Explanations:
For every half-life, 50 percent of the drug remains

Go to the next page if you knew the correct answer, or click the link images
below to further research the concepts in this question (if desired).

Research Concepts:
Drug Elimination:

Tap flag to report any problems with this question.


Question 953: Select the structure that is not associated with terminal
bronchioles.

Choices:
1. Hyaline cartilage
2. Smooth muscle
3. Ciliated epithelial cells
4. Clara cells
Answer: 1 - Hyaline cartilage
Explanations:
Hyaline cartilage is not present at terminal bronchioles
Clara cells are non-ciliated bronchiolar cells
Terminal bronchioles also have ciliated epithelial cells and smooth muscle
cells

Go to the next page if you knew the correct answer, or click the link images
below to further research the concepts in this question (if desired).

Research Concepts:
Lung Anatomy:

Tap flag to report any problems with this question.


Question 954: Which is false about fevers caused during a blood
transfusion?

Choices:
1. Fever is a signal that there is mismatch of blood transfusion
2. Fever is caused by antibodies towards white cells
3. Fever is more common with platelet transfusions than red cell transfusions
4. Fever does not occur if cold blood is administered
Answer: 4 - Fever does not occur if cold blood is administered
Explanations:
Febrile reactions are caused by recipient antibodies towards white cell
antigens or fragments.
Fever is usually more common with platelets than with RBC transfusions.
Fever may be the initial sign that a hemolytic reaction is occurring. If chills
and hypotension occur, the transfusion should be immediately discontinued.
In some individuals, febrile reactions can be stopped by giving leukocytes
filtered blood.

Go to the next page if you knew the correct answer, or click the link images
below to further research the concepts in this question (if desired).

Research Concepts:
Transfusion Reactions:

Tap flag to report any problems with this question.


Question 955: A patient is administered adenosine in the EP lab and
immediately becomes symptomatic and requires urgent treatment. Which drug
must be on hand when administering patients adenosine?

Choices:
1. Verapamil
2. Digoxin
3. Isoproterenol
4. Theophylline
Answer: 4 - Theophylline
Explanations:
Bothe caffeine and adenosine bind to the same receptor. The
pharmacological effect of adenosine can be terminated by methylxanthine
or theophylline.
Adenosine is contraindicated in asthmatics, atrial fibrillation, or atrial
flutter
Side effects of adenosine include facial flushing, rash on chest, diaphoresis
or lightheadedness. Metallic taste is very common with adenosine.
Adenosine is broken down by adenosine deaminase in red blood cells.

Go to the next page if you knew the correct answer, or click the link images
below to further research the concepts in this question (if desired).

Research Concepts:
Adenosine:

Tap flag to report any problems with this question.


Question 956: What is the neuromuscular-blocking agent associated with
malignant hyperthermia?

Choices:
1. Succinylcholine
2. Halothane
3. Curare
4. Atracurium
Answer: 1 - Succinylcholine
Explanations:
Succinylcholine can cause malignant hyperthermia.
Malignant hyperthermia is associated with the inability of the body to
remove carbon dioxide. It is also associated with fever, circulatory collapse,
and muscle rigidity.
Malignant hyperthermia is an autosomal dominant disorder, but there is no
simple test to determine susceptibility.
Dantrolene is often used for the treatment of malignant hyperthermia.

Go to the next page if you knew the correct answer, or click the link images
below to further research the concepts in this question (if desired).

Research Concepts:
Malignant Hyperthermia:

Tap flag to report any problems with this question.


Question 957: Which of the following would not be seen in a patient with
chronic severe aortic regurgitation?

Choices:
1. Decreased coronary perfusion
2. Decreased myocardial oxygen consumption
3. End diastolic pressure equalization between the left ventricle and the aorta
4. Left ventricular hypertrophy
Answer: 2 - Decreased myocardial oxygen consumption
Explanations:
With severe aortic regurgitation the ventricle initially hypertrophies
This leads to increased myocardial oxygen consumption
The thickened myocardium causes back pressure decreasing coronary
perfusion

Go to the next page if you knew the correct answer, or click the link images
below to further research the concepts in this question (if desired).

Research Concepts:
Aortic Regurgitation:

Tap flag to report any problems with this question.


Question 958: What is the first branch of the aorta?
Choices:
1. Right brachiocephalic artery
2. Left common carotid
3. Left subclavian artery
4. Right coronary artery
Answer: 4 - Right coronary artery
Explanations:
The innominate artery is the largest branch of the aorta and arises at the 2nd
right costal cartilage. It divides into the right common carotid and right
subclavian artery
The left common carotid artery arises from the arch of the aorta in the
superior mediastinum at the left of the left sternoclavicular joint
The left subclavian artery begins at the arch of the aorta behind the left
common carotid and at the level of the 4th thoracic vertebra
The right coronary is the first branch of the aorta

Go to the next page if you knew the correct answer, or click the link images
below to further research the concepts in this question (if desired).

Research Concepts:
Right Coronary Artery:

Tap flag to report any problems with this question.


Question 959: Which of the following is most often associated with rib
notching?

Choices:
1. Atrial septal defect
2. Ventricular septal defect
3. Coarctation of aorta
4. Tetralogy of fallot
Answer: 3 - Coarctation of aorta
Explanations:
Untreated coarctation usually leads to development of collaterals.
These collaterals involve the intercostal arteries, which run underneath the
rib and form notches over time. Tetralogy of Fallot can also cause rib
notching, but much less commonly.
Rib notching may also be seen in neurofibromatosis, a result of developing
intercostal nerve neurofibromas.
Venous causes of rib notching would include superior vena cava
obstruction.

Go to the next page if you knew the correct answer, or click the link images
below to further research the concepts in this question (if desired).

Research Concepts:
Aortic Coarctation:

Tap flag to report any problems with this question.


Question 960: Carotid massage usually leads to:
Choices:
1. Hypertension
2. Arrhythmias
3. Bradycardia
4. Tachycardia
Answer: 3 - Bradycardia
Explanations:
Carotid massage decreases heart rate and causes vasodilatation.
The effect is mediated by cranial nerve X.

Go to the next page if you knew the correct answer, or click the link images
below to further research the concepts in this question (if desired).

Research Concepts:
Carotid Massage:

Tap flag to report any problems with this question.


Question 961: What is the drug of choice during Advanced Cardiac Life
Support (ACLS)?

Choices:
1. Dopamine
2. Norepinephrine
3. Epinephrine
4. Isoprenaline
Answer: 3 - Epinephrine
Explanations:
Many alpha-adrenergic drugs have been used for ACLS.
Today, epinephrine has been proven to be the most successful.

Go to the next page if you knew the correct answer, or click the link images
below to further research the concepts in this question (if desired).

Research Concepts:
Cardiopulmonary Resuscitation:

Tap flag to report any problems with this question.


Question 962: Which of the following is most often associated with
alcoholism?

Choices:
1. Hypertrophic cardiomyopathy
2. Restrictive cardiomyopathy
3. Dilated cardiomyopathy
4. Infective endocarditis
Answer: 3 - Dilated cardiomyopathy
Explanations:
Congestive or dilated cardiomyopathy can be caused by alcohol abuse
Adriamycin can cause the same condition
Infective endocarditis is often associated with IV drug abuse
Mutations in genes encoding contractile proteins results in hypertrophic
cardiomyopathy

Go to the next page if you knew the correct answer, or click the link images
below to further research the concepts in this question (if desired).

Research Concepts:
Alcohol And Substance Abuse:

Cardiomyopathy, Dilated:

Tap flag to report any problems with this question.


Question 963: What condition is commonly associated with cardiac
fibroma?

Choices:
1. Hypertension
2. Aortic stenosis
3. Ventricular fibrillation
4. Atrial flutter
Answer: 3 - Ventricular fibrillation
Explanations:
Cardiac fibromas commonly present with ventricular arrhythmias

Go to the next page if you knew the correct answer, or click the link images
below to further research the concepts in this question (if desired).

Research Concepts:
Cardiac Fibroma:

Tap flag to report any problems with this question.


Question 964: Select the medication indicated for long term management
of congestive heart failure.

Choices:
1. Diltiazem
2. Lisinopril
3. Amlodipine
4. Amiodarone
Answer: 2 - Lisinopril
Explanations:
Ace Inhibitors are effective for CHF as they reduce preload and afterload
They affect the renin-angiotensin-renin system
Other medications used for long term are beta blockers, spironolactone, and
digoxin
In African-Americans the combination of hydralazine and isosorbide has
been proven to be of benefit

Go to the next page if you knew the correct answer, or click the link images
below to further research the concepts in this question (if desired).

Research Concepts:
Heart Failure, Congestive:

Tap flag to report any problems with this question.


Question 965: Which of the following is a Class I indication for cardiac
stress imaging in patients with chronic stable angina who are able to exercise?

Choices:
1. Right bundle branch block
2. Prior myocardial infarction
3. Prior revascularization (PTCA or CABG)
4. Congestive heart failure
Answer: 3 - Prior revascularization (PTCA or CABG)
Explanations:
Exercise myocardial perfusion imaging or exercise echocardiography is
recommended in patients with prior revascularization
The same is recommended in patients with intermediate pretest probability
of CAD with WPW syndrome or >1 mm of rest ST segment depression
Adenosine or dipyridamole myocardial perfusion imaging in patients with
electronically paced ventricular rhythm or LBBB
CHF or RBBB are not indications for cardiac stress imaging

Go to the next page if you knew the correct answer, or click the link images
below to further research the concepts in this question (if desired).

Research Concepts:
Cardiac Stress Imaging:

Tap flag to report any problems with this question.


Question 966: In a patient with postoperative hypothermia, which clinical
feature will be seen?

Choices:
1. Tachycardia
2. Sweating
3. High oxygen consumption
4. Hypertension
Answer: 3 - High oxygen consumption
Explanations:
Post-operative hypothermia leads to muscle shivering, which increases
oxygen consumption.

Go to the next page if you knew the correct answer, or click the link images
below to further research the concepts in this question (if desired).

Research Concepts:
Hypothermia:

Tap flag to report any problems with this question.


Question 967: Supravalvular aortic stenosis is commonly seen in patients
with what syndrome?

Choices:
1. Down
2. Williams
3. Turners
4. Ebstein
Answer: 2 - Williams
Explanations:
Supravalvular aortic stenosis is a rare medical disorder that is often seen in
patients with William syndrome
William syndrome consists of elfin faces, hypercalcemia and peripheral
pulmonary artery stenosis

Go to the next page if you knew the correct answer, or click the link images
below to further research the concepts in this question (if desired).

Research Concepts:
Williams Syndrome:

Tap flag to report any problems with this question.


Question 968: When assessing pain intensity, which of the following
individuals has the most validity?

Choices:
1. Patient
2. Physician
3. Nurse
4. Pain Doctor
Answer: 1 - Patient
Explanations:
Self-reporting is the most valid assessment of pain level

Go to the next page if you knew the correct answer, or click the link images
below to further research the concepts in this question (if desired).

Research Concepts:
Pain Assessment:

Tap flag to report any problems with this question.


Question 969: Which of the following is not a possible signs of chronic
digitalis toxicity?

Choices:
1. Headache
2. Fatigue
3. Muscle weakness
4. Tinnitus
Answer: 4 - Tinnitus
Explanations:
Digitalis toxicity can be acute or chronic
Acute toxicities commonly appear as new forms of arrhythmia
Chronic digoxin toxicities commonly present with muscle weakness,
fatigue and headache
Tinnitus more commonly seen with quinidine toxicity

Go to the next page if you knew the correct answer, or click the link images
below to further research the concepts in this question (if desired).

Research Concepts:
Digoxin Toxicity:

Tap flag to report any problems with this question.


Question 970: Which of the following indicates an increase in the size of
the heart in a patient with hypertension?

Choices:
1. Hyperplasia
2. Hypertrophy
3. Apoptosis
4. Metaplasia
Answer: 2 - Hypertrophy
Explanations:
An increase in the size of an organ is due to increased cell size.

Go to the next page if you knew the correct answer, or click the link images
below to further research the concepts in this question (if desired).

Research Concepts:
Cardiomyopathy, Hypertrophic:

Tap flag to report any problems with this question.


Question 971: What is the most appropriate initial management of
hypovolemic shock?

Choices:
1. Blood products
2. Crystalloid fluids
3. Inotropic medications
4. Surgery
Answer: 2 - Crystalloid fluids
Explanations:
The most appropriate initial management of hypovolemic shock is the
administration of crystalloid fluids.

Go to the next page if you knew the correct answer, or click the link images
below to further research the concepts in this question (if desired).

Research Concepts:
Shock, Hypovolemic:

Tap flag to report any problems with this question.


Question 972: After air embolism, which factor is not important for
outcome?

Choices:
1. Health status of patient
2. Amount of air
3. Pulmonary function
4. Site of air entry
Answer: 4 - Site of air entry
Explanations:
The amount of air and the status of the patient are the most important
factors in determining the outcome.

Go to the next page if you knew the correct answer, or click the link images
below to further research the concepts in this question (if desired).

Research Concepts:
Venous Air Embolism:

Tap flag to report any problems with this question.


Question 973: Which of the following can lead to apnea in a patient with
emphysema?

Choices:
1. 2 L/min O2 per nasal cannula
2. Administration of theophylline
3. 50% O2 via Venturi mask
4. None of the above
Answer: 3 - 50% O2 via Venturi mask
Explanations:
Administering high concentration O2 therapy to a COPDer can lead to
hypoventilation and eventually apnea
Due to chronically high PCO2 levels, they have little or no respiratory
response to rising levels
Paradoxically, they have to rely on low PO2 levels to stimulate respiration
Increasing PO2 with O2 therapy, especially at high concentrations, will
decrease respiratory rates and may lead to respiratory arrest

Go to the next page if you knew the correct answer, or click the link images
below to further research the concepts in this question (if desired).

Research Concepts:
Emphysema, Chronic:

Carbon Dioxide Retention:

Tap flag to report any problems with this question.


Question 974: Forcefully advancing a catheter into the right atrium can
result in?

Choices:
1. Arrhythmia
2. Asystole
3. Perforation
4. Clot
Answer: 3 - Perforation
Explanations:
Catheter in the right atrium can lead to perforation

Go to the next page if you knew the correct answer, or click the link images
below to further research the concepts in this question (if desired).

Research Concepts:
Cardiac Catheterization:

Tap flag to report any problems with this question.


Question 975: Which is not an exclusion criterion for aortic valve
replacement?

Choices:
1. Atrial septal defect
2. Ventricular septal defect
3. Patent ductus
4. Pulmonary stenosis
Answer: 4 - Pulmonary stenosis
Explanations:
Patients with pulmonary stenosis can have aortic valve replacement.
Ddirect communication between arterial and venous side are
contraindications to surgery.

Go to the next page if you knew the correct answer, or click the link images
below to further research the concepts in this question (if desired).

Research Concepts:
Aortic Valve:

Tap flag to report any problems with this question.


Question 976: An 8 year old female has a small ventricular septal defect
without symptoms. Select the best advice about sports participation.

Choices:
1. The child should avoid isometric exercises
2. The child can be involved in any sport without concern
3. The child should avoid swimming due to the risk of syncope
4. The child should avoid competitive running but other sports are acceptable
Answer: 2 - The child can be involved in any sport without concern
Explanations:
A small VSD should not impact the child and there should be no restrictions
Some would recommend an ECG to screen for pulmonary hypertension
The murmur of VSD is harsh and holosystolic best hear at the left lower
sternal border

Go to the next page if you knew the correct answer, or click the link images
below to further research the concepts in this question (if desired).

Research Concepts:
Ventricular Septal Defect:

Tap flag to report any problems with this question.


Question 977: When does wound dehiscence occur after surgery?
Choices:
1. 3 day
2. 7 days
3. 15 days
4. 30 days
Answer: 2 - 7 days
Explanations:
Wound dehiscence occurs at seven days after surgery

Go to the next page if you knew the correct answer, or click the link images
below to further research the concepts in this question (if desired).

Research Concepts:
Wound Dehiscence:

Tap flag to report any problems with this question.


Question 978: When a 60-year-old patient is prescribed a nitrate patch for
angina, what instructions should be provided?

Choices:
1. Wash skin surrounding patch with rubbing alcohol
2. Patch should be removed after 30 days
3. Smoking while wearing a nitrate patch is harmful
4. Patch should be removed for a period of time each day to avoid tolerance to
nitrates
Answer: 4 - Patch should be removed for a period of time each day to avoid
tolerance to nitrates

Explanations:
Patients who consistently rely on nitrates to manage their angina symptoms
can develop tolerance to nitrates.
Tolerance is best avoided by having the patient remove the nitrate patch for
10 to 12 hours each day.

Go to the next page if you knew the correct answer, or click the link images
below to further research the concepts in this question (if desired).

Research Concepts:
Nitroglycerin Patch:

Tap flag to report any problems with this question.


Question 979: What is the primary goal in the initial resuscitation of a
cardiac arrest?

Choices:
1. Renal perfusion
2. Limb perfusion
3. Myocardial perfusion
4. Brain perfusion
Answer: 4 - Brain perfusion
Explanations:
Even though cardiopulmonary resuscitation is done to restart the heart, the
major purpose is to get blood flowing back to the brain, which is vital for
survival.
Prompt cardiopulmonary resuscitation can prevent hypoxic injury and
prevent tissue death.
On average, only 5-10% of people who receive cardiopulmonary
resuscitation survive. Studies have shown that defibrillation within 3-5
minutes of sudden ventricular fibrillation cardiac arrest can improve
survival.
Many protocols now downplay the importance of artificial respiration and
focus more on the chest compressing activity.

Go to the next page if you knew the correct answer, or click the link images
below to further research the concepts in this question (if desired).

Research Concepts:
Cardiac Arrest:

Tap flag to report any problems with this question.


Question 980: A patient is on mechanical ventilator and his arterial blood
gas reveals a PCO2 of 38 and a pH of 7.41. What is the appropriate next step?

Choices:
1. Increase oxygen
2. Observe
3. Increase rate
4. Increase tidal volume
Answer: 2 - Observe
Explanations:
A tidal volume of 5-7 cc/kg is now recommended for patients on
mechanical ventilation. Some patients with neuromuscular disease may
benefit from high tidal volumes. The patient above has normal values and
should be observed.
The inhalation/exhalation ratio can be adjusted and is normally set at 1:3.
Patients with chronic obstructive pulmonary disease or with asthma should
have ratios of 1:4.
Fi02 is initially set at 1.0 but gradually decreased to the lowest level
necessary to maintain an oxygenation saturation of greater than 95%.
Positive end-expiratory pressure (PEEP) is set at 5 cm H2O, but in acute
respiratory distress syndrome it can be increased to 8 or more. PEEP also
increases intra-thoracic pressure and impedes venous return. This can be
countered by giving more volume to the patient.

Go to the next page if you knew the correct answer, or click the link images
below to further research the concepts in this question (if desired).

Research Concepts:
Ventilator Management:

Tap flag to report any problems with this question.


Question 981: Which of the following medications does not cause QT
interval prolongation?

Choices:
1. Quinidine
2. Disopyramide
3. Lidocaine
4. Procainamide
Answer: 3 - Lidocaine
Explanations:
Class one antiarrhythmics commonly cause QT prolongation and torsade de
pointes
Lidocaine is also a class one antiarrhythmic but does not cause QT
prolongation
Quinidine, disopyramide, and procainamide are class one antiarrhythmics

Go to the next page if you knew the correct answer, or click the link images
below to further research the concepts in this question (if desired).

Research Concepts:
Antiarrhythmic Medication:

Lidocaine:

Tap flag to report any problems with this question.


Question 982: When is a patient with a pulmonary embolism a candidate
for fibrinolytic therapy?

Choices:
1. When they are at risk for an MI
2. Patients with right ventricular dysfunction
3. When their oxygen saturation is below 90%
4. If they are not on warfarin
Answer: 2 - Patients with right ventricular dysfunction
Explanations:
Fibrinolytic therapy is suggested for patients with PE if they are
hemodynamically unstable and/or have right ventricular dysfunction

Go to the next page if you knew the correct answer, or click the link images
below to further research the concepts in this question (if desired).

Research Concepts:
Pulmonary Embolism:

Thrombolytic Therapy:

Tap flag to report any problems with this question.


Question 983: Which of the following will decrease the facial flushing that
occurs with niacin?

Choices:
1. Theophylline
2. Antihistamine
3. Prednisone
4. Aspirin
Answer: 4 - Aspirin
Explanations:
Aspirin taken with niacin decreases the side effect of facial flushing.
Caffeine, carbohydrates and water do not impact flushing.
Some studies have shown that niacin does not decrease cardiovascular risk
when added to statin therapy.

Go to the next page if you knew the correct answer, or click the link images
below to further research the concepts in this question (if desired).

Research Concepts:
Niacin:

Tap flag to report any problems with this question.


Question 984: Which of the following drugs can damage growing
cartilage?

Choices:
1. Tetracycline
2. Fluoroquinolones
3. Macrolides
4. Cephalosporins
Answer: 2 - Fluoroquinolones
Explanations:
Fluoroquinolones are contraindicated in children and pregnant mothers
Damage to growing cartilage by Fluoroquinolones can cause growth
problems
Tetracycline is also contraindicated in children and pregnant mothers
because it can stain growing bones

Go to the next page if you knew the correct answer, or click the link images
below to further research the concepts in this question (if desired).

Research Concepts:
Quinolones:

Tap flag to report any problems with this question.


Question 985: Treatment that most likely will not assist in attaining a
positive outcome is considered to be:

Choices:
1. Unethical treatment
2. Protocol
3. Futile
4. Experimental
Answer: 3 - Futile
Explanations:
Treatment unlikely to assist the patient in their care is considered futile

Go to the next page if you knew the correct answer, or click the link images
below to further research the concepts in this question (if desired).

Research Concepts:
Futile Treatment:

Tap flag to report any problems with this question.


Question 986: What is the mechanism of action of ezetimibe?
Choices:
1. Reduces liver cholesterol synthesis
2. Enhances HDL production
3. Inhibits cholesterol absorption
4. Binds bile acids
Answer: 3 - Inhibits cholesterol absorption
Explanations:
Ezetimibe works by inhibiting absorption of cholesterol at the brush border
of the small intestine via the sterol transporter
This leads to a decreased delivery of cholesterol to the liver
It also leads to reduction of hepatic cholesterol stores and an increased
clearance of cholesterol from the blood
It has no direct effect on cholesterol synthesis and degradation

Go to the next page if you knew the correct answer, or click the link images
below to further research the concepts in this question (if desired).

Research Concepts:
Ezetimibe:

Tap flag to report any problems with this question.


Question 987: What causes rib notching in people with aortic coarctation?
Choices:
1. Enlarged and tortuous intercostal arteries in the costal sulcus of the ribs
2. Collaterals from the left subclavian artery
3. Dilated thoracic aorta
4. Vessels from the thyroid arteries
Answer: 1 - Enlarged and tortuous intercostal arteries in the costal sulcus of
the ribs

Explanations:
Prolonged aortic coarctation causes rib notching via intercostal arterial
dilation.
The chronic cases result in development of collaterals usually from the
internal mammary and intercostal arteries,
These collateral from just beneath the 4-8th ribs and cause notching of the
ribs along the inferior rib margin (where the intercostal artery, vein, and
nerve are found).
Superior rib notching can be seen in patients with polio, osteogenesis
imperfecta and certain collagen vascular disorders.

Go to the next page if you knew the correct answer, or click the link images
below to further research the concepts in this question (if desired).

Research Concepts:
Aortic Coarctation:

Tap flag to report any problems with this question.


Question 988: Pulmonary venoocclusive disease:
Choices:
1. Is narrowing of the pulmonary vessels
2. Is reversible with prompt treatment
3. Is exacerbated with exercise
4. Is vasodilation of vessels entering the lungs
Answer: 1 - Is narrowing of the pulmonary vessels
Explanations:
Pulmonary vessels narrow, increasing pulmonary congestion and pressure
in the lungs

Go to the next page if you knew the correct answer, or click the link images
below to further research the concepts in this question (if desired).

Research Concepts:
Pulmonary Venoocclusive Disease:

Tap flag to report any problems with this question.


Question 989: Which is a first-line drug for rate control in the long-term
management of atrial fibrillation that is stable but symptomatic?

Choices:
1. Digoxin
2. Verapamil
3. Amiodarone
4. Quinidine
Answer: 2 - Verapamil
Explanations:
Rate control is the goal of long-term management in most patients in whom
restoration of normal sinus rhythm cannot be achieved.
Calcium channel blockers and beta-blockers are the preferred drugs.
Digoxin is a second-line therapy because of its side effects and lower
efficacy.

Go to the next page if you knew the correct answer, or click the link images
below to further research the concepts in this question (if desired).

Research Concepts:
Atrial Fibrillation:

Tap flag to report any problems with this question.


Question 990: When is the best time to give prophylactic antibiotic for
surgery?

Choices:
1. 12 hours prior
2. 12 hour after
3. Immediately before
4. After surgery
Answer: 3 - Immediately before
Explanations:
Antibiotic is best to be given right before surgery

Go to the next page if you knew the correct answer, or click the link images
below to further research the concepts in this question (if desired).

Research Concepts:
Preoperative Prophylactic Antibiotics:

Tap flag to report any problems with this question.


Question 991: The drug of choice for the treatment of ventricular
dysrhythmias is?

Choices:
1. Metoprolol
2. Digoxin
3. Lidocaine
4. Adenosine
Answer: 3 - Lidocaine
Explanations:
Lidocaine blocks sodium channels.
It is a class IB antiarrhythmic agent that shortens the action potential.
It should not be used in 2nd or 3rd degree heart block unless there is a
pacemaker.

Go to the next page if you knew the correct answer, or click the link images
below to further research the concepts in this question (if desired).

Research Concepts:
Arrhythmias:

Lidocaine:

Tap flag to report any problems with this question.


Question 992: Which of the following is not true about the sino-atrial (SA)
node?

Choices:
1. It has no phase 1 or phase 2
2. The resting potential is a constant -70mV
3. The SA node is the pacemaker of the heart
4. The SA node has an inward sodium current in phase 4
Answer: 2 - The resting potential is a constant -70mV
Explanations:
The pacemaker of the heart is the SA node
Phase 0 is caused by an increase in calcium conductance and is an upstroke
There is no phase 1 or 2
Phase 3 is the repolarization cause by an increase in potassium current
outward

Go to the next page if you knew the correct answer, or click the link images
below to further research the concepts in this question (if desired).

Research Concepts:
SA Node:

Tap flag to report any problems with this question.


Question 993: Which of the following is least likely to present as
congestive heart failure at 6 weeks of age?

Choices:
1. Patent ductus arteriosus
2. Ventricular septal defect
3. Aortopulmonary window
4. Atrial septal defect
Answer: 4 - Atrial septal defect
Explanations:
Atrial septal defects generally do not present with congestive heart failure
at this age

Go to the next page if you knew the correct answer, or click the link images
below to further research the concepts in this question (if desired).

Research Concepts:
Heart Failure, Congestive:

Congenital Heart Disease:

Tap flag to report any problems with this question.


Question 994: Which of the following is not true of pulsus bisferiens?
Choices:
1. It can present with alternating strong and weak pulses
2. It is commonly felt in aortic regurgitation
3. It is best felt in the carotid artery
4. It is affected by inspiration
Answer: 4 - It is affected by inspiration
Explanations:
Pulsus bisferiens is a biphasic pulse when palpated.
Pulsus bisferiens may alternate between a strong and a weak pulse in one
cycle.
Pulsus bisferiens is often observed in severe aortic regurgitation and
hypertrophic cardiomyopathy.
Pulsus bisferiens is best felt in the central vessels, such as the carotid and
brachial arteries.

Go to the next page if you knew the correct answer, or click the link images
below to further research the concepts in this question (if desired).

Research Concepts:
Aortic Regurgitation:

Cardiomyopathy, Hypertrophic:

Tap flag to report any problems with this question.


Question 995: A 2-year-old child has congenital cyanosis. Which of the
following fits the description?

Choices:
1. Tetralogy of Fallot
2. Coarctation of aorta
3. Atrial septal defect
4. Transposition of the great vessels
Answer: 1 - Tetralogy of Fallot
Explanations:
Most patients with congenital cyanosis after the age of 2 years have
tetralogy of Fallot.

Go to the next page if you knew the correct answer, or click the link images
below to further research the concepts in this question (if desired).

Research Concepts:
Tetralogy Of Fallot:

Tap flag to report any problems with this question.


Question 996: Which of the following antihypertensive medications should
be used with caution in people with asthma?

Choices:
1. Metoprolol
2. Captopril
3. Diltiazem
4. Alpha methyldopa
Answer: 1 - Metoprolol
Explanations:
Beta-blockers should be used with caution in asthmatics.
Side effects of beta-blockers include bronchospasm, bradycardia, and
nightmares.

Go to the next page if you knew the correct answer, or click the link images
below to further research the concepts in this question (if desired).

Research Concepts:
Beta-Blockers:

Tap flag to report any problems with this question.


Question 997: A group A recipient has mistakenly received a unit of group
B blood. Which of the following is not a transfusion-mediated reaction?

Choices:
1. Hypotension
2. Back pain
3. Bleeding tendency
4. Muscle rigidity
Answer: 4 - Muscle rigidity
Explanations:
Hemolytic transfusion reactions have many symptoms and may include
fever, chills, flushing, burning at site of IV, chest tightness, apprehension,
back pain, and hypotension.
Hypocalcaemia can occur from multiple transfusions and can present with
circumoral tingling and tremors of skeletal muscles.
All donor blood is screened for hepatitis, Human immunodeficiency virus I
and II, Human T-Lymphotropic virus I and II, Syphilis, Chagas disease,
hepatitis B and C, and West Nile virus. Not all donors are screened for
cytomegalovirus.
Lab findings of hemolytic reactions include high bilirubin, free
hemoglobin, hemoglobinuria, eosinophilia, and elevated liver enzymes.

Go to the next page if you knew the correct answer, or click the link images
below to further research the concepts in this question (if desired).

Research Concepts:
Transfusion Reactions:

Tap flag to report any problems with this question.


Question 998: Which of the following is not an independent risk factor for
coronary artery disease?

Choices:
1. Obesity
2. Atherosclerosis
3. Alcohol
4. Smoking
Answer: 1 - Obesity
Explanations:
Obesity is not an independent risk factor for CAD
It does affect the other independent risk factors for CAD

Go to the next page if you knew the correct answer, or click the link images
below to further research the concepts in this question (if desired).

Research Concepts:
Risk Factors For Coronary Artery Disease:

Tap flag to report any problems with this question.


Question 999: Which of the following has been implicated in the
pathogenesis of atherosclerosis?

Choices:
1. Regular exercise
2. Alcohol
3. Cigarette smoking
4. Statins
Answer: 3 - Cigarette smoking
Explanations:
Smoking is the number one factor implicated in the pathogenesis of
atherosclerosis.
Smoking is a modifiable risk factor.
Evidence indicates that smoking increases the risk of atherosclerosis by
200% over 2-5 years.

Go to the next page if you knew the correct answer, or click the link images
below to further research the concepts in this question (if desired).

Research Concepts:
Smoking:

Atherosclerosis:

Tap flag to report any problems with this question.


Question 1000: What is the drug of choice for the treatment of methicillin-
resistant staphylococcus aureus?

Choices:
1. Penicillin
2. Vancomycin
3. Doxycycline
4. Streptomycin
Answer: 2 - Vancomycin
Explanations:
Vancomycin is the drug of choice for methicillin-resistant staphylococcus
aureus.
Vancomycin is bactericidal.
Vancomycin is excreted in the kidney and thus levels of the antibiotic need
to be monitored.

Go to the next page if you knew the correct answer, or click the link images
below to further research the concepts in this question (if desired).

Research Concepts:
Methicillin Resistant Staphylococcus Aureus (MRSA):

Tap flag to report any problems with this question.


Section 6
Question 1001: Select the incorrect statement about CABG versus
multivessel angioplasty.

Choices:
1. Survival is the same at 5 years.
2. Survival is identical in diabetic and non-diabetic patients.
3. The efficacy of stents has yet to determined.
4. The effect of stents or minimally invasive surgery is to be determined.
Answer: 2 - Survival is identical in diabetic and non-diabetic patients.
Explanations:
CABG has a survival advantage for diabetic patients.
The outcome of invasive revascularization is depend on patient
characteristics including cardiac function, the number of diseased vessels,
and coronary disease distribution.

Go to the next page if you knew the correct answer, or click the link images
below to further research the concepts in this question (if desired).

Research Concepts:
Coronary Artery Bypass Graft:

Tap flag to report any problems with this question.


Question 1002: A 3 day old neonate has progressive cyanosis without
respiratory distress. There is a single second heart sound. Chest x-ray shows
normal heart size and pulmonary vasculature. Electrocardiogram shows axis of
120° and prominence of the right ventricle. Select the most probable diagnosis.

Choices:
1. Patent ductus arteriosus
2. Tetralogy of Fallot
3. Transposition of the great vessels
4. Ventricular septal defect
Answer: 3 - Transposition of the great vessels
Explanations:
Early cyanosis and a normal sized heart can be seen with transposition of
the great vessels
One third of patients have the classic egg on a string x-ray
Right axis deviation and RVH are seen

Go to the next page if you knew the correct answer, or click the link images
below to further research the concepts in this question (if desired).

Research Concepts:
Transposition Of The Great Arteries:

Tap flag to report any problems with this question.


Question 1003: Which of the following would not be included in
management of hypokalemia?

Choices:
1. Oral potassium
2. Cardiac monitoring
3. Intravenous fluids with potassium
4. Intravenous glucose and insulin
Answer: 4 - Intravenous glucose and insulin
Explanations:
Intravenous glucose and insulin are treatment for hyperkalemia
Potassium can be given orally or intravenously at a slow rate (0.1-0.2
mEq/kg/h)
Cardiac monitoring for arrhythmias is appropriate

Go to the next page if you knew the correct answer, or click the link images
below to further research the concepts in this question (if desired).

Research Concepts:
Hypokalemia:

Tap flag to report any problems with this question.


Question 1004: A 35 year old female from Brazil presents to the
emergency room with congestive heart failure. She has no significant past
medical history. Exam shows heart failure and poor perfusion. ECG shows right
bundle branch block and 1st degree AV block. Echocardiogram reveals thinned
and dilated ventricles, apical aneurysm, and ventricular thrombus. Cardiac
catheterization shows no coronary artery disease. Select the correct statement.

Choices:
1. The patient needs cardiac transplantation
2. Curative therapy is nifurtimox
3. The patient will have spontaneous recovery
4. Beta blockade is necessary
Answer: 1 - The patient needs cardiac transplantation
Explanations:
Chronic Chagas disease can cause severe cardiac disease
It is caused by trypanosomiasis, which in Central or South America can
cause cardiomyopathy and conduction delays
The conduction delay is most commonly right bundle branch block
Medical therapy with nifurtimox is only beneficial in early disease

Go to the next page if you knew the correct answer, or click the link images
below to further research the concepts in this question (if desired).

Research Concepts:
Chagas Disease (American Trypanosomiasis):

Tap flag to report any problems with this question.


Question 1005: What is the initial pediatric standard defibrillating dosage
of energy?

Choices:
1. 1 joule/kg
2. 2 joules/kg
3. 4 joules/kg
4. 5 joules/kg
Answer: 2 - 2 joules/kg
Explanations:
The initial pediatric standard defibrillating dosage of energy is 2 joules/kg
Subsequent defibrillation attempts should be 4 joules/kg
Ventricular fibrillation is uncommon in infants <1 year of age

Go to the next page if you knew the correct answer, or click the link images
below to further research the concepts in this question (if desired).

Research Concepts:
Defibrillation And Cardioversion:

Tap flag to report any problems with this question.


Question 1006: After a patient receives thrombolytics, monitor the patient
for which of the following?

Choices:
1. Mental status
2. Chest pain
3. Temperature
4. Skin rashes
Answer: 2 - Chest pain
Explanations:
Observing the patient for chest pain is an assessment priority
Occlusion of an obstructed coronary artery is not unusual

Go to the next page if you knew the correct answer, or click the link images
below to further research the concepts in this question (if desired).

Research Concepts:
Thrombolytic Therapy:

Tap flag to report any problems with this question.


Question 1007: In an athlete, which arrhythmia is considered normal?
Choices:
1. Atrial fibrillation
2. Ventricular fibrillation
3. Sinus bradycardia
4. Second degree AV block
Answer: 3 - Sinus bradycardia
Explanations:
Sinus bradycardia is considered normal in an athlete
The phenomena gives rise to the term "runner's heart"
Atrial fibrillation, ventricular fibrillation and second degree AV block
would be considered abnormal in any individual

Go to the next page if you knew the correct answer, or click the link images
below to further research the concepts in this question (if desired).

Research Concepts:
Sinus Bradycardia:

Tap flag to report any problems with this question.


Question 1008: The INR goal for anticoagulation in a patient with atrial
fibrillation (AF) is:

Choices:
1. 1-2
2. 2-3
3. 3-4
4. None of the above
Answer: 2 - 2-3
Explanations:
During AF, the atria contract chaotically and irregularly when compared to
the ventricles
The INR goal for anticoagulation in a patient with AF is 2-3
The goal of long-term anticoagulation in a patient with AF is to reduce
thromboembolism risk
Patients with AF have a 5X higher risk for stroke than those in sinus rhythm

Go to the next page if you knew the correct answer, or click the link images
below to further research the concepts in this question (if desired).

Research Concepts:
Atrial Fibrillation:

International Normalized Ratio (INR):

Tap flag to report any problems with this question.


Question 1009: Which of the following would cause an ABG with pH =
7.58, PO2 = 81 mm Hg, PCO2 = 27 mm Hg, and HCO3 = 21 mEq/liter?

Choices:
1. Bradypnea
2. Somnolence
3. Hyperventilation
4. Hypoventilation
Answer: 3 - Hyperventilation
Explanations:
Hyperventilation causes respiratory alkalosis.
This can result from anxiety, pulmonary embolism, or mechanical
ventilation at too high a rate.
The other conditions listed are more likely to cause respiratory acidosis.

Go to the next page if you knew the correct answer, or click the link images
below to further research the concepts in this question (if desired).

Research Concepts:
Hyperventilation:

Respiratory Alkalosis:

Tap flag to report any problems with this question.


Question 1010: A 67 year patient on a ventilator with a PEEP of 10,
suddenly develops SOB, hypotension and becomes diaphoretic. What is the first
step in treatment?

Choices:
1. Stat portable chest x-ray
2. Fluid bolus
3. Needle decompression
4. Bedside ultrasound
Answer: 3 - Needle decompression
Explanations:
The patient most likely has iatrogenic tension pneumothorax
Decompression is urgent. Radiologic procedures are too time consuming
There is debate as to whether the decompression catheter should be placed
in the second rib interspace at the midclavicular line or in the fifth
intercostal space at the anterior axillary line

Go to the next page if you knew the correct answer, or click the link images
below to further research the concepts in this question (if desired).

Research Concepts:
Iatrogenic Pneumothorax:

Tap flag to report any problems with this question.


Question 1011: Which of the following is not a feature of Beck's triad?
Choices:
1. Jugular venous distension
2. Hypotension
3. Distant heart sounds
4. Wide pulse pressure
Answer: 4 - Wide pulse pressure
Explanations:
Beck's triad is a triad of three physiological features seen in patients with
cardiac tamponade
The triad of Beck includes hypotension, elevated JVP, and distant or
muffled heart sounds
Narrow pulse pressure is commonly seen in patients with tamponade
In severe hypovolemia, the neck veins may not be distended

Go to the next page if you knew the correct answer, or click the link images
below to further research the concepts in this question (if desired).

Research Concepts:
Cardiac Tamponade:

Tap flag to report any problems with this question.


Question 1012: Sympathetic stimulation decreases which of the
following?

Choices:
1. Heart rate
2. Contractility
3. Parasympathetic input
4. Oxygen consumption
Answer: 3 - Parasympathetic input
Explanations:
The sympathetic nervous system opposes the parasympathetic nervous
system.

Go to the next page if you knew the correct answer, or click the link images
below to further research the concepts in this question (if desired).

Research Concepts:
Sympathetic Nervous System:

Tap flag to report any problems with this question.


Question 1013: What is the most common type of IV iodinated contrast
today?

Choices:
1. Ionic agents for all patients
2. Low osmolar, nonionic agents in all patients
3. Low osmolar contrast in patients with CHF, renal failure, or documented
allergy
4. Ionic agents for all elective cases after pre-medication with
diphenhydramine and steroids
Answer: 2 - Low osmolar, nonionic agents in all patients
Explanations:
Soon after non-ionic, and then low-osmolar contrast agents came on the
market, many selective-use protocols were attempted at numerous
institutions.
They generally failed because too many exceptions were made for
individuals who did not meet strict criteria, putting the institution at legal
risk if others had a reaction.
Over time, prices fell as supply and competition increased, so it is now
routine for all patients to receive low osmolar non-ionic contrast agents.
While diphenhydramine (Benadryl) has a clear role in treatment of minor
contrast reactions, there is no clear evidence it is helpful prophylactically,
given as a preparation agent prior to contrast administration.

Go to the next page if you knew the correct answer, or click the link images
below to further research the concepts in this question (if desired).

Research Concepts:
Nonionic Contrast Dye:

Tap flag to report any problems with this question.


Question 1014: Which of the following is not a risk for perioperative
DVT?

Choices:
1. Varicose veins
2. Obesity
3. Infection
4. Male sex
Answer: 4 - Male sex
Explanations:
Other risk factors include malignancy, oral contraceptives, long surgical
duration, genetic hypercoagulability

Go to the next page if you knew the correct answer, or click the link images
below to further research the concepts in this question (if desired).

Research Concepts:
Deep Venous Thrombosis Risk Factors:

Deep Vein Thrombosis:

Tap flag to report any problems with this question.


Question 1015: Which of the following is not true of the bicuspid aortic
valve?

Choices:
1. It occurs in 1-2% of the population
2. All valves become calcific over time
3. Bacterial endocarditis can occur
4. It is associated with dissection of the descending thoracic aorta
Answer: 2 - All valves become calcific over time
Explanations:
Bicuspid aortic valves are the most common cardiac anomaly and occur in
1-2% of the population.
Fibrocalcific changes occur by the age of 30.
Stenotic valves in childhood rarely occur, but may necessitate a valvotomy
and a possible valve replacement later on. These valves are prone to
endocarditis.
Bicuspid valves are associated with dissection of the ascending aorta during
valve replacement. The incidence is unknown.

Go to the next page if you knew the correct answer, or click the link images
below to further research the concepts in this question (if desired).

Research Concepts:
Aortic Valve:

Tap flag to report any problems with this question.


Question 1016: What factor is NOT required to determine systemic
vascular resistance (SVR)?

Choices:
1. Mean arterial pressure (MAP)
2. Central venous pressure (CVP)
3. Pulse pressure (PP)
4. Cardiac output (CO)
Answer: 3 - Pulse pressure (PP)
Explanations:
SVR is MAP minus CVP divided by CO.

Go to the next page if you knew the correct answer, or click the link images
below to further research the concepts in this question (if desired).

Research Concepts:
Systemic Vascular Resistance:

Tap flag to report any problems with this question.


Question 1017: What is the most common cause of reduction in urine
output in a postoperative patient?

Choices:
1. Adverse effect of opioid drugs
2. Hypovolemia
3. Urinary tract obstruction
4. Bowel obstruction
Answer: 2 - Hypovolemia
Explanations:
Normal urine output is about 1-2 ml/kg/hr
Signs of hypovolemia should be checked in all patients

Go to the next page if you knew the correct answer, or click the link images
below to further research the concepts in this question (if desired).

Research Concepts:
Hypovolemia:

Postoperative Management:

Tap flag to report any problems with this question.


Question 1018: What is the best route of drug administration during
ACLS and PALS?

Choices:
1. Large IV
2. Subclavian central line
3. Femoral
4. Arterial
Answer: 2 - Subclavian central line
Explanations:
Central line close to the heart is most favorable during emergency
Second choice is intravenous.
If no other route is available an endotracheal tube can be used for naloxone,
atropine, valium (diazepam), epinephrine, and lidocaine (NAVEL).

Go to the next page if you knew the correct answer, or click the link images
below to further research the concepts in this question (if desired).

Research Concepts:
Advanced Cardiac Life Support:

Tap flag to report any problems with this question.


Question 1019: How much radiation can an adult working in healthcare
receive in 12 months without violating regulatory guidelines?

Choices:
1. 1 mSv
2. 33 mSv
3. 50 mSv
4. 170 mSv
Answer: 3 - 50 mSv
Explanations:
The total maximum permissible radiation dose exposure is 50 mSv per year.
Sievert is the new unit of measurement for radiation biological effect,
replacing the Rem.
100 rem = 1 Sievert.
In the older terminology, the maximum is therefore 5000 mRem, or 5 Rem
per year.

Go to the next page if you knew the correct answer, or click the link images
below to further research the concepts in this question (if desired).

Research Concepts:
Ionizing Radiation Exposure:

Tap flag to report any problems with this question.


Question 1020: Paroxysmal supraventricular tachycardia is best treated by
which of the following?

Choices:
1. Adenosine
2. Beta blockers
3. Calcium channel blockers
4. Digoxin
Answer: 1 - Adenosine
Explanations:
The positive inotropic effects of catecholamines is blocked by adenosine
slowing conduction at the AV node
Side effects include dizziness and chest pain
Adenosine is contraindicated in patients on theophylline
Calcium channel blockers are second line

Go to the next page if you knew the correct answer, or click the link images
below to further research the concepts in this question (if desired).

Research Concepts:
Paroxysmal Supraventricular Tachycardia:

Tap flag to report any problems with this question.


Question 1021: A patient presents with progressive shortness of breath and
repeated bouts of hemoptysis. Both chest x ray and CT do not reveal any
significant lesions and bronchoscopy is negative. His diagnosis is made by:

Choices:
1. Angiography
2. Echocardiogram
3. MRI
4. Nuclear scan
Answer: 2 - Echocardiogram
Explanations:
The above is caused by mitral stenosis

Go to the next page if you knew the correct answer, or click the link images
below to further research the concepts in this question (if desired).

Research Concepts:
Mitral Stenosis:

Tap flag to report any problems with this question.


Question 1022: What is the most common form of arterial hypertension?
Choices:
1. Essential hypertension
2. Renal hypertension
3. Pheochromocytoma
4. Atherosclerotic hypertension
Answer: 1 - Essential hypertension
Explanations:
Essential hypertension accounts for 90% of all cases of hypertension.
Essential hypertension tends to run in families and may have a contributing
environmental factor.
The other risk factors for hypertension include certain foods containing
high fructose.
Essential hypertension is more common in African Americans and also is
associated with a high mortality.

Go to the next page if you knew the correct answer, or click the link images
below to further research the concepts in this question (if desired).

Research Concepts:
Hypertension:

Tap flag to report any problems with this question.


Question 1023: Which of the following does not generally cause fibrinous
pericarditis?

Choices:
1. Autoimmunity
2. Myocardial infarction
3. Viral infection
4. Uremia
Answer: 1 - Autoimmunity
Explanations:
Autoimmunity is less likely to cause fibrinous pericarditis than serous
pericarditis
Viral infection, uremia, myocardial infarction, and rheumatic fever all cause
fibrinous pericarditis

Go to the next page if you knew the correct answer, or click the link images
below to further research the concepts in this question (if desired).

Research Concepts:
Fibrinous Pericarditis:

Tap flag to report any problems with this question.


Question 1024: Which of the following physiological functional
alterations in the elderly can precipitate adverse drug reactions?

Choices:
1. Increased liver microsomal P450 enzyme activity
2. Decreased body fat
3. Depressed renal function
4. Gastroparesis
Answer: 3 - Depressed renal function
Explanations:
Renal function is expected to decrease with age.
Renal and liver blood flow decreases with age. There is a reduction in total
body water.
This all contributes to drug toxicities in the elderly.
Drug doses should be adjusted according to renal functions.

Go to the next page if you knew the correct answer, or click the link images
below to further research the concepts in this question (if desired).

Research Concepts:
Renal Insufficiency:

Adverse Drug Reactions:

Normal Aging:
Tap flag to report any problems with this question.
Question 1025: To avoid damage to the sciatic nerve, intramuscular
injections should be given at which gluteal region?

Choices:
1. Lower medial quadrant
2. Lower lateral quadrant
3. Upper lateral quadrant
4. Upper medial quadrant
Answer: 3 - Upper lateral quadrant
Explanations:
The sciatic nerve begins in the lower back and runs through the buttock and
down the lower limb.
To avoid damage to this nerve, the intramuscular injections should be given
at upper lateral quadrant.
Sciatica is pain in the distribution of the sciatic nerve and is caused by
compression or irritation.

Go to the next page if you knew the correct answer, or click the link images
below to further research the concepts in this question (if desired).

Research Concepts:
Intramuscular Injection:

Tap flag to report any problems with this question.


Question 1026: Which of the following vessels is accompanied by the
phrenic nerve and descends between the mediastinal pleura and pericardium?

Choices:
1. Pericardiophrenic vessels
2. Internal mammary
3. Coronary arteries
4. Musculophrenic arteries
Answer: 1 - Pericardiophrenic vessels
Explanations:
The phrenic nerve is mainly derived from the fourth cervical nerve with
contributions from the third and fifth.
It has sympathetic, sensory, and motor nerve fibers.
The pericardiophrenic vessels run with the phrenic nerve to the diaphragm.
The pericardiophrenic artery arises from internal thoracic artery.

Go to the next page if you knew the correct answer, or click the link images
below to further research the concepts in this question (if desired).

Research Concepts:
Thoracic Anatomy:

Tap flag to report any problems with this question.


Question 1027: What is the half-life of vecuronium?
Choices:
1. 10 minutes
2. 30 minutes
3. 1 hour
4. 5 hours
Answer: 2 - 30 minutes
Explanations:
Vecuronium is a short-acting non-depolarizing paralytic agent

Go to the next page if you knew the correct answer, or click the link images
below to further research the concepts in this question (if desired).

Research Concepts:
Vecuronium:

Tap flag to report any problems with this question.


Question 1028: What antihypertensive medication does not alter renal
blood flow and is often used to treat hypertension complicated by renal disease?

Choices:
1. Metoprolol
2. Clonidine
3. Prazosin
4. Labetalol
Answer: 2 - Clonidine
Explanations:
Clonidine is an alpha 2 agonist which diminishes central sympathetic
outflow.
Clonidine does not decrease renal blood low and is often useful in treating
hypertension complicated by renal disease.

Go to the next page if you knew the correct answer, or click the link images
below to further research the concepts in this question (if desired).

Research Concepts:
Dialysis Complications:

Renal Insufficiency:

Tap flag to report any problems with this question.


Question 1029: Multifocal atrial tachycardia is diagnosed with an atrial
rate of:

Choices:
1. 40 per minute
2. 60 per minute
3. 100 per minute
4. 120 per minutes
Answer: 3 - 100 per minute
Explanations:
MAT is characterized by an atrial rate greater than 100 per minute and the
presence of three morphologies of P waves
It is most often seen with COPD

Go to the next page if you knew the correct answer, or click the link images
below to further research the concepts in this question (if desired).

Research Concepts:
Multifocal Atrial Tachycardia:

Tap flag to report any problems with this question.


Question 1030: What is the most likely cause of decreased urine output in
a postoperative patient?

Choices:
1. Prerenal
2. Renal
3. Post Renal
4. Sepsis
Answer: 1 - Prerenal
Explanations:
Hypovolemia from insensible losses, blood loss and third spacing are the
most common cause of postoperative oliguria in a patient with normal
kidneys
Sepsis in a clean case would be extremely rare

Go to the next page if you knew the correct answer, or click the link images
below to further research the concepts in this question (if desired).

Research Concepts:
Prerenal Kidney Failure:

Hypovolemia:

Postoperative Care:
Tap flag to report any problems with this question.
Question 1031: Which of the following is not a classic clinical
presentation of coronary heart disease?

Choices:
1. Non-pleuritic chest pain
2. Pain that does not change with position
3. Crushing-type chest pain
4. Tenderness of the chest wall
Answer: 4 - Tenderness of the chest wall
Explanations:
Chest wall tenderness is not seen in coronary heart disease.
Chest wall tenderness suggests costochondritis.

Go to the next page if you knew the correct answer, or click the link images
below to further research the concepts in this question (if desired).

Research Concepts:
Coronary Artery Disease:

Tap flag to report any problems with this question.


Question 1032: Which of the following should be avoided when treating
acute myocarditis?

Choices:
1. ACE inhibitors
2. Nitrates
3. IV digoxin
4. Diuretics
Answer: 3 - IV digoxin
Explanations:
The acute management of clinically significant disease includes cardiac
monitoring, supplemental oxygen, and the management of fluid status
Immunosuppression has not been demonstrated to be beneficial
The use of IV digoxin during the acute phase of myocarditis may be
associated with the increased occurrence of dysrhythmias

Go to the next page if you knew the correct answer, or click the link images
below to further research the concepts in this question (if desired).

Research Concepts:
Acute Myocarditis:

Tap flag to report any problems with this question.


Question 1033: Why is a 67 year old with atrial fibrillation prescribed
warfarin?

Choices:
1. Slows the heart rate
2. Empirically treat blood clots
3. Pharmacologically convert patients to sinus rhythm
4. Prevents formation of intracardiac thrombus
Answer: 4 - Prevents formation of intracardiac thrombus
Explanations:
Warfarin is an anticoagulant, which is used in the treatment of atrial
fibrillation to prevent thrombus formation and release of emboli into the
circulation.

Go to the next page if you knew the correct answer, or click the link images
below to further research the concepts in this question (if desired).

Research Concepts:
Atrial Fibrillation:

Tap flag to report any problems with this question.


Question 1034: In diabetic patients who undergo coronary artery bypass,
what is the safest way to administer protamine?

Choices:
1. Administer protamine rapidly via the jugular vein
2. Administer protamine as a test dose
3. Administer protamine by the surgeon
4. Give protamine with steroids
Answer: 2 - Administer protamine as a test dose
Explanations:
Protamine has a numbers of side effects, especially in diabetics.
It can cause vasodilatation, has negative inotropic effects, anaphylaxis and
pulmonary vasoconstriction.
Caution should be used when administering protamine to a diabetic. A test
dose should be given first to look for signs of allergy or sensitivity.
The dose of protamine should always be given slowly.

Go to the next page if you knew the correct answer, or click the link images
below to further research the concepts in this question (if desired).

Research Concepts:
Diabetes Mellitus, Type 2:

Protamine:

Coronary Artery Bypass Graft:


Tap flag to report any problems with this question.
Question 1035: Which of the following markers is the fastest to reveal
acute myocardial infarctions?

Choices:
1. LDH
2. WBC
3. Troponin
4. CK-MB
Answer: 3 - Troponin
Explanations:
Troponin levels increase as fast as 4 hours after infarction
It usually takes a few more hours for CK-MB to raise after infarction
LDH also rises after infarctions but it is not commonly used for diagnostic
purposes
Leukocytosis also occurs after acute myocardial infarction but it is not used
for diagnostic purposes

Go to the next page if you knew the correct answer, or click the link images
below to further research the concepts in this question (if desired).

Research Concepts:
Acute Myocardial Infarction:

Troponin:

Tap flag to report any problems with this question.


Question 1036: What is the best treatment for a patient on the ventilator
with acute respiratory acidosis?

Choices:
1. Increased oxygenation
2. Decreased oxygenation
3. Increased ventilation
4. Decreased ventilation
Answer: 3 - Increased ventilation
Explanations:
The most important priority is to treat the underlying cause of acute
respiratory acidosis.
Acute respiratory acidosis is best treated by increasing respiratory rate or
tidal volume.

Go to the next page if you knew the correct answer, or click the link images
below to further research the concepts in this question (if desired).

Research Concepts:
Ventilator Management:

Respiratory Acidosis:

Tap flag to report any problems with this question.


Question 1037: Which medication is most likely to cause orthostatic
hypotension?

Choices:
1. Furosemide
2. Spironolactone
3. Triamterene
4. Hydrochlorothiazide
Answer: 1 - Furosemide
Explanations:
Furosemide is a potent diuretic that can cause orthostatic hypotension.
Its effect may worsen when used with other antihypertensive medication.
Thiazide diuretics and spironolactone are weaker diuretics when compared
with furosemide.

Go to the next page if you knew the correct answer, or click the link images
below to further research the concepts in this question (if desired).

Research Concepts:
Orthostatic Hypotension:

Tap flag to report any problems with this question.


Question 1038: A patient with patent venous grafts and a left internal
mammary artery graft (LIMA) needs a mitral valve replacement. The lateral
chest x-ray shows virtual symphysis between the right atrium and the sternum.
In such a case, the mitral valve may be repaired or replaced using ventricular
fibrillation and hypothermia. One contraindication to this approach would be:

Choices:
1. Patent LIMA
2. Incompetent aortic valve
3. Patent vein grafts
4. Aortic stenosis
Answer: 2 - Incompetent aortic valve
Explanations:
For procedures under fibrillatory arrest, a competent aortic valve is a must
The heart will distend from the ventricular fibrillation and an incompetent
aortic valve
Many surgeons will also insert a left atrial vent as an adjunct

Go to the next page if you knew the correct answer, or click the link images
below to further research the concepts in this question (if desired).

Research Concepts:
Mitral Valve Replacement:

Cardiac Valvular Surgery:

Tap flag to report any problems with this question.


Question 1039: Which of following medications is a class II
antiarrhythmic drug?

Choices:
1. Disopyramide
2. Timolol
3. Sotalol
4. Verapamil
Answer: 2 - Timolol
Explanations:
Timolol is a class II antiarrhythmic drug.
It is a non-selective beta-adrenergic receptor antagonist.
It is most commonly used for glaucoma but is used orally for hypertension
and acute myocardial infarction.
Care must be taken when withdrawing the medication as it can precipitate
acute angina and MI.

Go to the next page if you knew the correct answer, or click the link images
below to further research the concepts in this question (if desired).

Research Concepts:
Timolol:

Antiarrhythmic Medication:

Tap flag to report any problems with this question.


Question 1040: Select the incorrect statement about atherosclerosis.
Choices:
1. Symptoms are secondary to obstruction of blood flow
2. Lipid lowering treatment cannot cause regression of fixed lesions
3. Acute coronary syndromes most often occur with stenosis of >50% of a
coronary artery
4. Pathophysiology involves inflammation
Answer: 2 - Lipid lowering treatment cannot cause regression of fixed
lesions

Explanations:
Aggressive lipid management can cause regression of fixed plaques
This was not thought to be the case despite decreased cardiac events with
statins

Go to the next page if you knew the correct answer, or click the link images
below to further research the concepts in this question (if desired).

Research Concepts:
Atherosclerosis:

Tap flag to report any problems with this question.


Question 1041: What is the best initial treatment for paroxysmal
supraventricular tachycardia?

Choices:
1. Adenosine
2. Procainamide
3. Diltiazem
4. Electric cardioversion
Answer: 1 - Adenosine
Explanations:
Adenosine and vagal maneuvers are the first line treatment for PSVT
Procainamide is used in medical cardioversion of ventricular tachycardia
Diltiazem is reserved for persistent cases of PSVT
Electric cardioversion is a last resort in persistent PSVT but is used initially
for unstable patients

Go to the next page if you knew the correct answer, or click the link images
below to further research the concepts in this question (if desired).

Research Concepts:
Paroxysmal Supraventricular Tachycardia:

Adenosine:

Tap flag to report any problems with this question.


Question 1042: Which one of the following electrolytes must be
supplemented when using furosemide?

Choices:
1. Sodium
2. Potassium
3. Magnesium
4. Phosphorus
Answer: 2 - Potassium
Explanations:
Potassium supplementation is vital when using furosemide
Hypokalemia is manifest with muscle weakness and arrhythmias

Go to the next page if you knew the correct answer, or click the link images
below to further research the concepts in this question (if desired).

Research Concepts:
Furosemide:

Hypokalemia:

Tap flag to report any problems with this question.


Question 1043: Which antibiotic administered daily is now used to treat
gram-negative organisms and has a post antibiotic effect?

Choices:
1. Sulfonamides
2. Cefazolin
3. Imipenem
4. Gentamicin
Answer: 4 - Gentamicin
Explanations:
Gentamicin is an aminoglycoside requiring dose reduction with renal
impairment
Gentamicin is as a bactericidal inhibitor of protein synthesis (30S) and
active against many aerobic gram negative bacteria
Because it can readily cause renal dysfunction and ototoxicity, gentamicin
is now given once a day maintaining potency because of the post antibiotic
effect which is persistent suppression of bacterial growth within 1-2 of
administration even with immunosuppression

Go to the next page if you knew the correct answer, or click the link images
below to further research the concepts in this question (if desired).

Research Concepts:
Gentamicin:

Tap flag to report any problems with this question.


Question 1044: A 45-year-old with a history of hypertrophic
cardiomyopathy presents with orthopnea, shortness of breath at rest, and a 10 lb
weight gain. He has been managed previously on verapamil. An echocardiogram
reveals an ejection fraction of 28% and left ventricular chamber enlargement.
Which management strategy is ideal for this patient?

Choices:
1. Insertion of a defibrillator
2. Alcohol-induced septal ablation
3. Add nitroglycerin
4. Discontinue verapamil and add furosemide and an ACE inhibitor
Answer: 4 - Discontinue verapamil and add furosemide and an ACE
inhibitor

Explanations:
A small percentage of patients with hypertrophic cardiomyopathy (HCM)
develop systolic dysfunction as well as a clinical picture consistent with
dilated cardiomyopathy.
Verapamil should be discontinued as it can reduce output. More
conventional heart failure medical optimization should be implemented.
Cardioversion is appropriate for managing atrial fibrillation as a
complication of HCM.
Septal ablation is considered in cases of severe, symptomatic obstructive
HCM.

Go to the next page if you knew the correct answer, or click the link images
below to further research the concepts in this question (if desired).

Research Concepts:
Cardiomyopathy, Hypertrophic:

Tap flag to report any problems with this question.


Question 1045: Which of the following cell types is NOT integral to
wound healing?

Choices:
1. Epithelial cells
2. Inflammatory cells
3. Fibroblasts
4. Transitional cells
Answer: 4 - Transitional cells
Explanations:
Wound healing depends on the recruitment of inflammatory cells and their
mediators into the wound bed.
The cells, in turn, through the actions of various cytokines, encourage the
activity of epithelial cells and fibroblasts.

Go to the next page if you knew the correct answer, or click the link images
below to further research the concepts in this question (if desired).

Research Concepts:
Wound Healing:

Tap flag to report any problems with this question.


Question 1046: Which is least often a disadvantage of patient-controlled
analgesia (PCA)?

Choices:
1. Overdose
2. Expensive cost
3. Malfunction
4. Patients may be physically or mentally unable to use this method
Answer: 1 - Overdose
Explanations:
The regimen and amount of drug is preadjusted my ordering physician.
There is a lock out that is to prevent overdose.
The machine is expensive and can malfunction.
Patients must be aware and able to activate the device.

Go to the next page if you knew the correct answer, or click the link images
below to further research the concepts in this question (if desired).

Research Concepts:
Patient Controlled Anesthesia (PCA):

Tap flag to report any problems with this question.


Question 1047: Which of the following pulmonary artery catheter data
suggest left sided heart failure?

Choices:
1. Cardiac index greater than 3L/min/m2
2. Pulmonary capillary wedge pressure less than 15 mmHg
3. Central venous pressure of 10 mmHg
4. Pulmonary capillary wedge pressure greater than 18 mmHg
Answer: 4 - Pulmonary capillary wedge pressure greater than 18 mmHg
Explanations:
Left sided heart failure is shown by an increase in preload.

Go to the next page if you knew the correct answer, or click the link images
below to further research the concepts in this question (if desired).

Research Concepts:
Congestive Heart Failure And Pulmonary Edema:

Tap flag to report any problems with this question.


Question 1048: Which of the following findings best correlates with the
diagnosis of Takayasu arteritis?

Choices:
1. Large vessel vasculitis
2. Vasculitis of small and medium-sized vessels
3. Non-caseating granulomas
4. Endarteritis obliterans
Answer: 1 - Large vessel vasculitis
Explanations:
Takayasu arteritis is most common in Asian and Indian populations
It is characterized by large vessel vasculitis
Aortic lesions may lead to aneurysm and its sequelae

Go to the next page if you knew the correct answer, or click the link images
below to further research the concepts in this question (if desired).

Research Concepts:
Arteritis, Takayasu:

Tap flag to report any problems with this question.


Question 1049: Which of the following is not recommended in a patient
with hypercholesterolemia?

Choices:
1. Niacin
2. Fish oil
3. Increase intake of processed foods
4. Weight loss and exercise
Answer: 3 - Increase intake of processed foods
Explanations:
Processed foods typically contain high levels of saturated fats and salt.
Niacin, fish oil, weight loss, and exercise are all appropriate treatments for
patients with high cholesterol.
Niacin is effective at reducing LDL, while fish oil, weight loss, and exercise
are effective at increasing HDL.

Go to the next page if you knew the correct answer, or click the link images
below to further research the concepts in this question (if desired).

Research Concepts:
Hypercholesterolemia:

Tap flag to report any problems with this question.


Question 1050: Select the correct statement about blood flow in coronary
arteries.

Choices:
1. Blood flow is maximum during diastole
2. It is maximum during systole
3. Blood flow distal to a thrombus increases
4. Blood flow proximal to a thrombus decreases
Answer: 1 - Blood flow is maximum during diastole
Explanations:
Coronary circulation is the circulation of blood in the blood vessels of the
heart muscle.
Blood flow in these arteries is maximal during diastole.
Blood flow distal to a thrombus decreases.
Blood flow proximal to the thrombus it increases.

Go to the next page if you knew the correct answer, or click the link images
below to further research the concepts in this question (if desired).

Research Concepts:
Cardiac Physiology:

Tap flag to report any problems with this question.


Question 1051: Which of the following is the inotropic agent that inhibits
phosphodiesterase?

Choices:
1. Digoxin
2. Quinidine
3. Amrinone
4. Dobutamine
Answer: 3 - Amrinone
Explanations:
Amrinone and milrinone are inotropic agents which act by inhibiting
phosphodiesterase.
Both amrinone and milrinone increase the levels of cyclic AMP.

Go to the next page if you knew the correct answer, or click the link images
below to further research the concepts in this question (if desired).

Research Concepts:
Inotropes And Vasopressors:

Tap flag to report any problems with this question.


Question 1052: The leading cause of right heart failure is:
Choices:
1. Hypertension
2. Lung disease
3. Renal failure
4. Left heart failure
Answer: 4 - Left heart failure
Explanations:
The most common cause of right heart failure is left heart failure.
The most common cause of left heart failure is ischemic disease.

Go to the next page if you knew the correct answer, or click the link images
below to further research the concepts in this question (if desired).

Research Concepts:
Right Heart Failure:

Tap flag to report any problems with this question.


Question 1053: The ductus arteriosus eventually becomes what structure?
Choices:
1. Ligamentum teres
2. Ligamentum arteriosum
3. Ligamentum venosum
4. Falciform ligament
Answer: 2 - Ligamentum arteriosum
Explanations:
The ductus arteriosus connects the aorta to the pulmonary artery in utero.
Once the ductus closes after birth, it becomes the ligamentum arteriosum.
The recurrent laryngeal nerve gets trapped under the ligamentum
arteriosum and can be injured during surgery.

Go to the next page if you knew the correct answer, or click the link images
below to further research the concepts in this question (if desired).

Research Concepts:
Fetal Circulation:

Tap flag to report any problems with this question.


Question 1054: Which is not associated with pulmonary hypertension?
Choices:
1. Primary pulmonary hypertension is seen more in males than females
2. Connective tissue disease such as lupus (SLE) or CREST syndrome
(scleroderma variant)
3. Cocaine and amphetamine abuse
4. HIV
Answer: 1 - Primary pulmonary hypertension is seen more in males than
females

Explanations:
Pulmonary hypertension is seen more in females than males
Associated with connective tissue disease or autoimmune disease, cocaine
abuse, HIV, and thyroid disease

Go to the next page if you knew the correct answer, or click the link images
below to further research the concepts in this question (if desired).

Research Concepts:
Pulmonary Hypertension:

Tap flag to report any problems with this question.


Question 1055: What is one of the most common causes of decreased lung
compliance in the ICU?

Choices:
1. Pneumothorax
2. Pulmonary edema
3. Wheezing
4. Flail chest
Answer: 2 - Pulmonary edema
Explanations:
A decrease in static compliance is generally associated with a
pneumothorax.
In the ICU, pulmonary edema is by far more common than pneumothorax.

Go to the next page if you knew the correct answer, or click the link images
below to further research the concepts in this question (if desired).

Research Concepts:
Pulmonary Edema:

Tap flag to report any problems with this question.


Question 1056: For a patient who has anaphylaxis from penicillin, what is
the best choice to cover Gram-positive organisms?

Choices:
1. Sulfonamide
2. Cephalexin
3. Erythromycin
4. Gentamycin
Answer: 3 - Erythromycin
Explanations:
Macrolide antibiotics are good substitutes for patients allergic to penicillin

Go to the next page if you knew the correct answer, or click the link images
below to further research the concepts in this question (if desired).

Research Concepts:
Gram Positive Bacteria:

Erythromycin:

Tap flag to report any problems with this question.


Question 1057: The circumflex branch of the left coronary artery supplies
which of the following areas of the heart?

Choices:
1. Anterior portion of the left atrium
2. Posterior portion of the left ventricle
3. Anterior portion of the left ventricle
4. Posterior portion of the right atrium
Answer: 2 - Posterior portion of the left ventricle
Explanations:
The circumflex artery is a branch of the left main coronary artery.
It travels in the left atrioventricular groove.
It supplies posterolateral left ventricle and anterolateral papillary muscle.
It ends at the point where it joins to form the posterior interventricular
artery in the 10% of cases where the posterior interventricular artery does
not arise from the right coronary artery.

Go to the next page if you knew the correct answer, or click the link images
below to further research the concepts in this question (if desired).

Research Concepts:
Heart Anatomy:

Tap flag to report any problems with this question.


Question 1058: Which of the following diagnostic test have the highest
specificity in detecting myocardial ischemia in a 61 year old female?

Choices:
1. Dipyridamole echocardiography
2. CT helical
3. Stress test
4. Thallium scintigraphy
Answer: 1 - Dipyridamole echocardiography
Explanations:
Dipyridamole echocardiography has a high specificity, near to 93%

Go to the next page if you knew the correct answer, or click the link images
below to further research the concepts in this question (if desired).

Research Concepts:
Coronary Artery Disease:

Tap flag to report any problems with this question.


Question 1059: Which antihypertensive is associated with excessive hair
growth?

Choices:
1. Minoxidil
2. Clonidine
3. Hydralazine
4. Alpha methyldopa
Answer: 1 - Minoxidil
Explanations:
Minoxidil is reserved for severe hypertension.
Minoxidil is a prodrug and a potassium channel opener that hyperpolarizes
and relaxes smooth muscle.
Minoxidil can cause hirsutism.

Go to the next page if you knew the correct answer, or click the link images
below to further research the concepts in this question (if desired).

Research Concepts:
Antihypertensive Medications:

Tap flag to report any problems with this question.


Question 1060: The 5th aortic arch gives rise to what structure?
Choices:
1. Descending aorta
2. Left carotid artery
3. Pulmonary artery
4. None of the above
Answer: 4 - None of the above
Explanations:
The fifth arch disappears; it does not give rise to any structures.

Go to the next page if you knew the correct answer, or click the link images
below to further research the concepts in this question (if desired).

Research Concepts:
Embryology:

Tap flag to report any problems with this question.


Question 1061: Which of the following is a contraindication to receiving
tPA in a 66 year old patient?

Choices:
1. Age greater than 65
2. Diabetes mellitus
3. Current aspirin therapy
4. Recent cerebral hemorrhage
Answer: 4 - Recent cerebral hemorrhage
Explanations:
A history of cerebral hemorrhage is a contraindication to administration of
thrombolytics because the risk of hemorrhage may be further increased.

Go to the next page if you knew the correct answer, or click the link images
below to further research the concepts in this question (if desired).

Research Concepts:
Tissue Plasminogen Activator:

Tap flag to report any problems with this question.


Question 1062: What is the mortality for cardiogenic shock?
Choices:
1. 20 to 30%
2. 30 to 50%
3. 50 to 75%
4. 75 to 99%
Answer: 3 - 50 to 75%
Explanations:
Most recent studies show mortality rates of 50 to 75%
Mortality is as high as 90% in the absence of experienced technical support

Go to the next page if you knew the correct answer, or click the link images
below to further research the concepts in this question (if desired).

Research Concepts:
Cardiogenic Shock:

Tap flag to report any problems with this question.


Question 1063: Which is not associated with hypertension?
Choices:
1. Left ventricular hypertrophy (LVH)
2. Subarachnoid hemorrhage (SAH)
3. Cerebrovascular accident (CVA)
4. Subdural hemorrhage (SDH)
Answer: 4 - Subdural hemorrhage (SDH)
Explanations:
SDH is not associated with hypertension.

Go to the next page if you knew the correct answer, or click the link images
below to further research the concepts in this question (if desired).

Research Concepts:
Hypertension:

Complications:

Tap flag to report any problems with this question.


Question 1064: Which of the following antibiotics is not metabolized by
the P-450 system?

Choices:
1. Clarithromycin
2. Azithromycin
3. Rifampin
4. Erythromycin
Answer: 2 - Azithromycin
Explanations:
Azithromycin is not metabolized by the P-450 system.
Therefore is usually does not have adverse drug-drug interactions caused by
an effect on the cytochrome P-450 system.

Go to the next page if you knew the correct answer, or click the link images
below to further research the concepts in this question (if desired).

Research Concepts:
Cytochrome P450:

Antibiotics:

Tap flag to report any problems with this question.


Question 1065: Which of the following would be expected in a patient
with thyrotoxicosis?

Choices:
1. High TSH
2. Low TSH
3. Normal TSH
4. Low T4
Answer: 2 - Low TSH
Explanations:
Thyrotoxicosis is associated with high thyroid hormone
And low TSH level

Go to the next page if you knew the correct answer, or click the link images
below to further research the concepts in this question (if desired).

Research Concepts:
Thyrotoxicosis:

Tap flag to report any problems with this question.


Question 1066: What is term for a designated individual who can make
healthcare decisions for someone other than themselves?

Choices:
1. An elder
2. A member of an ethics board
3. A health-care proxy
4. Power of Attorney
Answer: 3 - A health-care proxy
Explanations:
A designated health-care proxy is a person authorized to make health care
decisions for another person in the event they are unable to make their own
due to health or emergent circumstances

Go to the next page if you knew the correct answer, or click the link images
below to further research the concepts in this question (if desired).

Research Concepts:
Proxy:

Tap flag to report any problems with this question.


Question 1067: What effect does aspirin have on pain control when given
with morphine?

Choices:
1. Additive
2. Agonist
3. Antagonist
4. Synergistic
Answer: 4 - Synergistic
Explanations:
NSAIDS and opioid have synergistic effect on each other

Go to the next page if you knew the correct answer, or click the link images
below to further research the concepts in this question (if desired).

Research Concepts:
Nonsteroidal Anti-inflammatory Drugs (NSAIDs):

Opioids:

Pain Management:

Tap flag to report any problems with this question.


Question 1068: Of all the improvements in myocardial protection during
open heart surgery, which is the most important?

Choices:
1. Delivery of cardioplegia
2. Potassium induced cardiac arrest
3. Hypothermia
4. Use of transesophageal echocardiography
Answer: 3 - Hypothermia
Explanations:
Hypothermia is the core foundation of open heart surgery.
It decreases metabolism and oxygen requirements, allows cell viability at
lower temperatures and has a protective effect on cells.

Go to the next page if you knew the correct answer, or click the link images
below to further research the concepts in this question (if desired).

Research Concepts:
Hypothermia, Circulatory Arrest And Cardiopulmonary
Bypass:

Tap flag to report any problems with this question.


Question 1069: Tertiary syphilis commonly affects what part of the
cardiovascular system?

Choices:
1. Mitral valve
2. Ascending aorta
3. Abdominal aorta
4. Patent ductus arteriosus
Answer: 2 - Ascending aorta
Explanations:
Tertiary syphilis occurs in 33% of people who are untreated and the
manifestation may take decades. Lesions are generally due to the cardiac or
nervous systems.
The cardiac manifestations include dilatation of the ascending aorta, aortic
valve insufficiency, and narrowing of the coronary ostia.
Symptoms of the aortic aneurysm may include pressure on the trachea,
vocal cord paralysis, and painful erosion into the sternum or spine.
The neural manifestations can vary from paresis to dementia. Tabes dorsalis
is a slow progressive degeneration of the posterior columns and nerve roots.
Patients often complain of sharp stabbing pains in the back of the legs.

Go to the next page if you knew the correct answer, or click the link images
below to further research the concepts in this question (if desired).

Research Concepts:
Syphilis:

Tap flag to report any problems with this question.


Question 1070: Which medication can help relieve symptoms of aortic
regurgitation?

Choices:
1. Epinephrine
2. Dobutamine
3. Nitroglycerin
4. Metoprolol
Answer: 2 - Dobutamine
Explanations:
An increase in heart rate without affecting afterload will reduce aortic
regurgitation.

Go to the next page if you knew the correct answer, or click the link images
below to further research the concepts in this question (if desired).

Research Concepts:
Aortic Regurgitation:

Tap flag to report any problems with this question.


Question 1071: By which mechanism do ACE inhibitors cause
vasodilatation?

Choices:
1. Raising levels of bradykinin
2. Increasing levels of nitric oxide
3. Blocking aldosterone receptors
4. Increasing production of cyclic AMP
Answer: 1 - Raising levels of bradykinin
Explanations:
ACE inhibitors raise levels of bradykinin, a vasodilator that consequently
causes a drop in blood pressure.

Go to the next page if you knew the correct answer, or click the link images
below to further research the concepts in this question (if desired).

Research Concepts:
Angiotensin Converting Enzyme Inhibitors (ACEI):

Tap flag to report any problems with this question.


Question 1072: An 18 year old is seen in the cardiology clinic because his
chest x-ray revealed an enlarged cardiac silhouette. The individual has no major
complaints except that in the past he had some type of a tube placed in his chest.
Examination reveals that he is tall, thin individual and auscultation reveals that
he has a systolic click with a possible murmur. The gene responsible for these
findings is?

Choices:
1. KRas
2. P52
3. Fibrillin
4. BCl
Answer: 3 - Fibrillin
Explanations:
Fibrillin is a glycoprotein that is required for synthesis of elastin fibers
Mutations in fibrillin 1 gene cause Marfan syndrome
Mutations of fibrillin 2 gene are linked to Beal's syndrome
Fibrillin appear to lay a vital role in the structure of blood vessels

Go to the next page if you knew the correct answer, or click the link images
below to further research the concepts in this question (if desired).

Research Concepts:
Marfan Syndrome:

Tap flag to report any problems with this question.


Question 1073: A patient with discrepancy of left and right arm blood
pressure should prompt a suspicion of which of the following conditions?

Choices:
1. Aortic coarctation
2. Aortic stenosis
3. Aortic dissection
4. Aortic regurgitation
Answer: 1 - Aortic coarctation
Explanations:
Coarctation of the aorta can cause a difference in pressure readings of the
upper extremities.

Go to the next page if you knew the correct answer, or click the link images
below to further research the concepts in this question (if desired).

Research Concepts:
Aortic Coarctation:

Tap flag to report any problems with this question.


Question 1074: At what interval is cardioversion most effective in patients
with atrial fibrillation (AF)?

Choices:
1. >14 days after onset of AF
2. >21 days after onset of AF
3. <7 days after onset of AF
4. >7 days after onset of AF
Answer: 3 - <7 days after onset of AF
Explanations:
During AF, the atria contract chaotically and irregularly when compared to
the ventricles
Cardioversion is most successful if attempted <7 days after onset of AF
It should be done emergently for those with symptomatic (hypotension,
heart failure or angina) AF

Go to the next page if you knew the correct answer, or click the link images
below to further research the concepts in this question (if desired).

Research Concepts:
Atrial Fibrillation:

Synchronized Electrical Cardioversion:

Tap flag to report any problems with this question.


Question 1075: Which is not appropriate after a suspected air embolism?
Choices:
1. Apply pressure on neck veins
2. Stop nitrous oxide gas
3. Pack surgical field with dry gauze
4. Remove air with central catheter
Answer: 3 - Pack surgical field with dry gauze
Explanations:
The surgical site should be flooded and packed with wet gauze to prevent
further air uptake.

Go to the next page if you knew the correct answer, or click the link images
below to further research the concepts in this question (if desired).

Research Concepts:
Venous Air Embolism:

Tap flag to report any problems with this question.


Question 1076: Which of the following is associated with cardiac muscle
atrophy?

Choices:
1. Old age
2. Infancy
3. Exercise
4. Pregnancy
Answer: 1 - Old age
Explanations:
Lack of exercise and sedentary life style will lead to cardiac atrophy.
Old age is a risk factor for muscle atrophy.

Go to the next page if you knew the correct answer, or click the link images
below to further research the concepts in this question (if desired).

Research Concepts:
Cardiac Atrophy:

Tap flag to report any problems with this question.


Question 1077: What is the main cause of tricuspid stenosis?
Choices:
1. Endocarditis
2. Degenerative changes
3. Rheumatic fever
4. Congenital abnormality
Answer: 3 - Rheumatic fever
Explanations:
Mitral and aortic valve disease are the most commonly seen complications
of rheumatic fever.
The most common etiology of tricuspid stenosis is rheumatic fever.
It is never seen without involvement of mitral and aortic valves.
Tricuspid stenosis is rarely congenital.

Go to the next page if you knew the correct answer, or click the link images
below to further research the concepts in this question (if desired).

Research Concepts:
Tricuspid Stenosis:

Tap flag to report any problems with this question.


Question 1078: Which of the following should be checked before
delivering bad news to a dying patient?

Choices:
1. Make sure how much the patient wants to hear about the illness
2. Call friends and family to provide social support
3. Inquire the education level of the patient
4. Inquire if the patient wants hospice care to be arranged
Answer: 1 - Make sure how much the patient wants to hear about the illness
Explanations:
Readiness to hear a terminal diagnosis should be assessed before disclosure.
Some patients may choose not to hear about a terminal diagnosis.
Family and friends are not notified without the patient's permission.
Discussion about end of life care comes later.

Go to the next page if you knew the correct answer, or click the link images
below to further research the concepts in this question (if desired).

Research Concepts:
Terminal Illness:

Tap flag to report any problems with this question.


Question 1079: How can excessive activation of plasminogen to plasmin
be inhibited?

Choices:
1. Urokinase
2. Aminocaproic acid
3. Fresh frozen plasma
4. Vitamin K
Answer: 2 - Aminocaproic acid
Explanations:
Antiplasmin agents are useful in open-heart surgery where excessive
plasmin is generated.
Drugs such as aminocaproic acid or tranexamic acid are useful in open
heart surgery.
These drugs inhibit the fibrinolysis by inhibiting plasminogen activation.

Go to the next page if you knew the correct answer, or click the link images
below to further research the concepts in this question (if desired).

Research Concepts:
Thrombolysis:

Tap flag to report any problems with this question.


Question 1080: In a patient where you want to minimize HLA
sensitization, which of the following platelet transfusions should be done?

Choices:
1. Pooled platelets
2. Single donor
3. Radiated platelets
4. Washed platelets
Answer: 2 - Single donor
Explanations:
Single donor platelet transfusions can help minimize the risk of
desensitization when there are HLA incompatibility issues.
However, apheresed platelets are more expensive than pooled platelet
concentrate.
Prophylactic transfusions should be done when the platelet counts are less
than 10,000/ml.
For routine use 5-6 units of platelets is adequate for most patients and can
increase platelet count by 30,000.

Go to the next page if you knew the correct answer, or click the link images
below to further research the concepts in this question (if desired).

Research Concepts:
Platelet Transfusion:

Tap flag to report any problems with this question.


Question 1081: Cephalic vein cut down is contraindicated in patients with
which of the following?

Choices:
1. Ipsilateral mastectomy
2. Patients with pacemakers
3. Patients with pancoast tumor
4. Patients with carpal tunnel syndrome
Answer: 1 - Ipsilateral mastectomy
Explanations:
Cephalic vein cut down offers quick access to ensure chemotherapy
delivery.
Most physicians do not recommend cephalic vein cut down in patients with
ipsilateral mastectomy.
Other contraindications to cephalic vein cut down include chronic
venostasis, lymphedema, and excess scarring.

Go to the next page if you knew the correct answer, or click the link images
below to further research the concepts in this question (if desired).

Research Concepts:
Cephalic Vein Cut Down:

Tap flag to report any problems with this question.


Question 1082: A patient with a mechanical aortic valve presents with a
massive hemarthrosis of his right knee. His INR is 5.6. What is the best drug to
reverse his anticoagulation?

Choices:
1. Heparin
2. Protamine
3. Vitamin K
4. Aspirin
Answer: 3 - Vitamin K
Explanations:
Vitamin K administration can decrease bleeding problems due to oral anti-
coagulants such as warfarin.
The response to vitamin K is slow and requires about 24 hours.
If immediate homeostasis is required, fresh frozen plasma should be
infused.

Go to the next page if you knew the correct answer, or click the link images
below to further research the concepts in this question (if desired).

Research Concepts:
Anticoagulation:

Tap flag to report any problems with this question.


Question 1083: After open-heart surgery, the patient develops a claw
hand. The most likely nerve segment damaged is:

Choices:
1. C5, 6
2. C8, T1
3. T11-12
4. C3,4,5
Answer: 2 - C8, T1
Explanations:
Sometimes during anesthesia, the ulnar nerve is compressed at the elbow
known as the cubital tunnel syndrome. The syndrome present with
numbness in the small and ring fingers
Over time, the numbness becomes long standing and there is weakness in
the hand. The little and ring fingers curl up and appear like a claw
The claw hand caused by compression of the ulnar nerve at the wrist is
worse (Guyon canal compression)
The cubital tunnel syndrome occurs because the ulnar nerve passing along
the edge of the outer elbow is not protected. Supportive measures can help
prevent the numbness or tingling

Go to the next page if you knew the correct answer, or click the link images
below to further research the concepts in this question (if desired).

Research Concepts:
Cubital Tunnel Syndrome:

Claw Hand:

Tap flag to report any problems with this question.


Question 1084: Which of the following is not true of scimitar syndrome?
Choices:
1. It is associated with anomalous venous drainage to the inferior vena cava
2. It may be associated with a diaphragmatic defect
3. The right lung is hypoplastic
4. There is always an associated atrial septal defect
Answer: 4 - There is always an associated atrial septal defect
Explanations:
In the scimitar syndrome, the right pulmonary vein connects to the inferior
vena cava.
The vein is visible on an x-ray as a vertically oriented, crescent-shaped
density.
The disorder is accompanied by hypoplasia of the lung and an anomalous
arterial supply. In most cases there is no atrial septal defect.
If the lung is severely hypoplastic, the patient may have recurrent
pneumonia and a lobectomy/pneumonectomy may be required.

Go to the next page if you knew the correct answer, or click the link images
below to further research the concepts in this question (if desired).

Research Concepts:
Congenital Heart Disease:

Tap flag to report any problems with this question.


Question 1085: A 44-year-old African-American male complains of a
headache and general malaise. He says he has not been feeling well for the past
48 hours. He has a history of hypertension and takes a diuretic. He appears pale
with a BP measuring 220/110. Soon after, he develops chest pain and expires.
Which of the following findings would be associated with this disorder?

Choices:
1. Red cell casts in urine
2. Flea bitten kidney
3. Polyuria
4. Right ventricular hypertrophy
Answer: 2 - Flea bitten kidney
Explanations:
Individuals with high blood pressure often develop severe left ventricular
hypertrophy.
Uncontrolled high blood pressure can result in a heart attack, stroke, renal
failure, retinopathy, and early death.
The kidneys usually appear flea bitten with petechial hemorrhage and
arteriosclerosis.
Malignant hypertension is most common in young black males who are
non- compliant with their medications.

Go to the next page if you knew the correct answer, or click the link images
below to further research the concepts in this question (if desired).

Research Concepts:
Hypertension, Malignant:

Tap flag to report any problems with this question.


Question 1086: A patient is found to have a rim of calcification around the
pericardium. He does not give any significant past medical history of any
medical problems. Which of the following is the LEAST likely cause of
pericardial calcification?

Choices:
1. Prior trauma
2. Lupus
3. Histoplasmosis
4. Myxoma
Answer: 4 - Myxoma
Explanations:
Calcification of the pericardium can occur after an episode of pericarditis or
trauma
Infectious agents known to cause calcified pericardium include coxsackle,
TB, influenza and histoplasmosis.
With lupus, rheumatic heart disease, uremia, and post open-heart surgery
one can also develop pericardial calcification.
Rarely, teratomas, or pericardial cysts may become calcified.

Go to the next page if you knew the correct answer, or click the link images
below to further research the concepts in this question (if desired).

Research Concepts:
Pericardial Calcification:

Tap flag to report any problems with this question.


Question 1087: Bronchial arteries originate from which location?
Choices:
1. Pulmonary artery
2. Ascending aorta
3. Subclavian
4. Descending aorta
Answer: 4 - Descending aorta
Explanations:
Bronchial arteries come off the descending aorta.

Go to the next page if you knew the correct answer, or click the link images
below to further research the concepts in this question (if desired).

Research Concepts:
Vascular Anatomy:

Tap flag to report any problems with this question.


Question 1088: Which is least likely to be found in a patient with a tension
pneumothorax?

Choices:
1. Hypertension
2. Hypoxemia
3. Tachycardia
4. Respiratory acidosis
Answer: 1 - Hypertension
Explanations:
Tension pneumothorax causes a decreased cardiac output, resulting in
hypotension and a compensatory tachycardia.
Respiratory impairment also occurs, causing hypoxemia, hypercarbia, and
respiratory acidosis.

Go to the next page if you knew the correct answer, or click the link images
below to further research the concepts in this question (if desired).

Research Concepts:
Tension Pneumothorax:

Tap flag to report any problems with this question.


Question 1089: What is the drug of choice for treatment of torsade de
pointes?

Choices:
1. Lidocaine
2. Magnesium sulfate
3. Bretylium
4. Adenosine
Answer: 2 - Magnesium sulfate
Explanations:
Magnesium sulfate is the drug of choice for treatment of torsade de pointes
Torsade de pointes is an unusual and distinctive form of polymorphic
ventricular tachycardia
It can degenerate into ventricular fibrillation
Lidocaine usually has no effect on torsade

Go to the next page if you knew the correct answer, or click the link images
below to further research the concepts in this question (if desired).

Research Concepts:
Torsade de Pointes:

Tap flag to report any problems with this question.


Question 1090: A 42-year-old male is evaluated for the presence of a
holosystolic murmur with a diastolic rumble. Echocardiography shows mitral
regurgitation with a dilated left ventricle and left atrium as well as an ejection
fraction of 40%. He says he feels fine. What should be recommended?

Choices:
1. Warfarin
2. Repeat echo in 12 months
3. Surgery consult
4. Daily aspirin
Answer: 3 - Surgery consult
Explanations:
Patients with severe, gradual onset mitral valve regurgitation with ejection
fraction <50% should undergo a more definitive transesophageal
echocardiogram in preparation for possible valve replacement surgery.
Endocarditis prophylaxis is advisable, but surgical consult should not be
delayed.
As there is no indication of atrial fibrillation, there is no need for
anticoagulation.

Go to the next page if you knew the correct answer, or click the link images
below to further research the concepts in this question (if desired).

Research Concepts:
Mitral Regurgitation:

Tap flag to report any problems with this question.


Question 1091: Which of the following is least likely to cause drug
induced lupus?

Choices:
1. Furosemide
2. Hydralazine
3. Procainamide
4. Quinidine
Answer: 1 - Furosemide
Explanations:
Drug-induced lupus is rare and reversible with withdrawal of the offending
drug.
Implicated medications include hydralazine, procainamide, quinidine, and
phenytoin.
The symptoms of lupus generally start 3 to 6 months after initiation of the
drug and resolve within a few weeks or months after discontinuation.
As these medications are more commonly used in males, drug induced
lupus is more common in males.

Go to the next page if you knew the correct answer, or click the link images
below to further research the concepts in this question (if desired).

Research Concepts:
Lupus Erythematosus, Drug-Induced:

Tap flag to report any problems with this question.


Question 1092: How should respiratory depression caused by morphine
overdose be treated?

Choices:
1. Flumazenil
2. Phenobarbital
3. Naltrexone
4. Naloxone
Answer: 4 - Naloxone
Explanations:
Naloxone can reverse the effects of morphine and other narcotics
Flumazenil reverses the effects of benzodiazepines
Phenobarbital can worsen respiratory depression
Naltrexone is used for alcohol addiction

Go to the next page if you knew the correct answer, or click the link images
below to further research the concepts in this question (if desired).

Research Concepts:
Opioid Toxicity:

Tap flag to report any problems with this question.


Question 1093: A 71-year-old has uncontrolled atrial fibrillation. He is
cardioverted and then started on oral digoxin therapy. He is later seen in the
emergency room with complaints of diarrhea, yellow vision, and first-degree
heart block. His condition can easily be worsened by which of the following?

Choices:
1. Hyperkalemia
2. Hyponatremia
3. Hypercalcemia
4. Hypermagnesemia
Answer: 3 - Hypercalcemia
Explanations:
Digoxin toxicity is worsened by hypokalemia, hypomagnesemia, and
hypercalcemia.
Digoxin toxicity is also worsened by quinidine.
Digoxin toxicity can be treated by anti-digoxin antibodies.
Digoxin is not removed by hemodialysis or peritoneal dialysis.

Go to the next page if you knew the correct answer, or click the link images
below to further research the concepts in this question (if desired).

Research Concepts:
Hypercalcemia:

Digoxin:

Tap flag to report any problems with this question.


Question 1094: What is the current AHA recommendation level for HDL
in high-risk patients?

Choices:
1. Above10
2. Above 30
3. Above 40
4. Above 50
Answer: 4 - Above 50
Explanations:
The American Heart Association recommends above 50 for HDL and less
than 150 for triglycerides for high risk patients

Go to the next page if you knew the correct answer, or click the link images
below to further research the concepts in this question (if desired).

Research Concepts:
Hyperlipidemia:

Tap flag to report any problems with this question.


Question 1095: What is the normal left atrial pressure during diastole?
Choices:
1. 3 mmHg
2. 10 mmHg
3. 50 mmHg
4. 120 mmHg
Answer: 1 - 3 mmHg
Explanations:
Left atrium pressure is close to 1-5 mmHg

Go to the next page if you knew the correct answer, or click the link images
below to further research the concepts in this question (if desired).

Research Concepts:
Cardiac Physiology:

Tap flag to report any problems with this question.


Question 1096: The antiadrenergic agent with both alpha and beta
blocking activity is:

Choices:
1. Pindolol
2. Propranolol
3. Labetalol
4. Acebutolol
Answer: 3 - Labetalol
Explanations:
Labetalol has both alpha and beta blocking ability. It has a short half-life.

Go to the next page if you knew the correct answer, or click the link images
below to further research the concepts in this question (if desired).

Research Concepts:
Labetalol:

Tap flag to report any problems with this question.


Question 1097: Select the reason that a person experiences chest pain
during a myocardial infarction.

Choices:
1. Deficiency of oxygen reaching the heart muscle
2. Poor peripheral circulation
3. Increased left ventricular filling pressures
4. Increased need of oxygen for salvage of necrotic myocardium
Answer: 1 - Deficiency of oxygen reaching the heart muscle
Explanations:
Diminished oxygenation or lack of oxygen to the cardiac muscle results in
ischemic pain or angina.

Go to the next page if you knew the correct answer, or click the link images
below to further research the concepts in this question (if desired).

Research Concepts:
Myocardial Infarction, Acute:

Tap flag to report any problems with this question.


Question 1098: A patient with a graft versus host disease will not have
which of the following?

Choices:
1. Low alkaline phosphatase
2. High bilirubin
3. Fever
4. Rash
Answer: 1 - Low alkaline phosphatase
Explanations:
Graft versus host disease is mediated by T cells attacking the host cells.
Characterized by fever, rash, high alkaline phosphatase, and elevated
bilirubin.

Go to the next page if you knew the correct answer, or click the link images
below to further research the concepts in this question (if desired).

Research Concepts:
Graft Versus Host Disease:

Tap flag to report any problems with this question.


Question 1099: A 73-year-old has not passed urine for 8 hours after
undergoing a hernia repair. Examination reveals that there is suprapubic
tenderness and a feeling of discomfort on palpation. The patient has the urge to
urinate but is unable to void. What is the next step in this patient's management?

Choices:
1. Suprapubic cystotomy
2. Foley catheter
3. Fluids
4. Urology consult
Answer: 2 - Foley catheter
Explanations:
Obstructive uropathy has many causes and may occur after surgery in an
elderly individual with benign prostatic hyperplasia.
Pain is common when the bladder is acutely distended. Most patients will
complain of suprapubic tenderness and will be in distress.
The history may suggest symptoms of benign prostatic hyperplasia.
Because the disorder is very distressing, treatment should be rapid.
Treatment consists of inserting a Foley catheter. If this is not possible, a
urology consult for a suprapubic catheter may be required.

Go to the next page if you knew the correct answer, or click the link images
below to further research the concepts in this question (if desired).

Research Concepts:
Post-op Urinary Retention:

Tap flag to report any problems with this question.


Question 1100: Which is NOT a sign of IV contrast allergy?
Choices:
1. Urticaria
2. Edema
3. Hypertension
4. Wheezing
Answer: 3 - Hypertension
Explanations:
Allergy is associated with local or general edema
Results in wheezing, urticaria, and hypotension due to vasodilatation

Go to the next page if you knew the correct answer, or click the link images
below to further research the concepts in this question (if desired).

Research Concepts:
Contrast Medium Reactions:

Tap flag to report any problems with this question.


Question 1101: Mitral valve transplant from a pig is termed:
Choices:
1. Allograft
2. Autograft
3. Xenograft
4. Isograft
Answer: 3 - Xenograft
Explanations:
Autograft is a self tissue transplant
Allograft is tissue transplant from another person
Xenograft is tissue transplant between species

Go to the next page if you knew the correct answer, or click the link images
below to further research the concepts in this question (if desired).

Research Concepts:
Xenograft:

Tap flag to report any problems with this question.


Question 1102: The right neck vein is preferred for central line over the
left because of what structure?

Choices:
1. Subclavian
2. Lung apex
3. Thoracic duct
4. Heart
Answer: 3 - Thoracic duct
Explanations:
The thoracic duct is located on the left side

Go to the next page if you knew the correct answer, or click the link images
below to further research the concepts in this question (if desired).

Research Concepts:
Central Line Placement:

Tap flag to report any problems with this question.


Question 1103: Which medication has both alpha and beta blocking
activity?

Choices:
1. Labetalol
2. Sotalol
3. Propranolol
4. Atenolol
Answer: 1 - Labetalol
Explanations:
Labetalol is a short acting reversible beta-blocker with alpha blocking
activities.
Labetalol is very useful in the ICU for immediate control of blood pressure.
Labetalol is often used in treatment of malignant hypertension.
Relative contraindications for labetalol include asthma, congestive heart
failure, heart block, bradycardia, hypotension, and cardiogenic shock.

Go to the next page if you knew the correct answer, or click the link images
below to further research the concepts in this question (if desired).

Research Concepts:
Labetalol:

Tap flag to report any problems with this question.


Question 1104: Select the least likely complication of cor triatriatum
sinistrum.

Choices:
1. Right heart failure
2. Left ventricular systolic dysfunction
3. Pulmonary hypertension
4. Hemoptysis
Answer: 2 - Left ventricular systolic dysfunction
Explanations:
Cor triatriatum sinistrum is a condition where there are 3 atria, with the left
atria divided by an accessory membrane
This can lead to pulmonary hypertension, pulmonary edema, hemoptysis,
and right sided heart failure
Left ventricular function is usually not affected

Go to the next page if you knew the correct answer, or click the link images
below to further research the concepts in this question (if desired).

Research Concepts:
Cor Triatriatum:

Tap flag to report any problems with this question.


Question 1105: Myocardial oxygen demand is reduced by which of the
following?

Choices:
1. Exercise
2. Sympathetic
3. Parasympathetic
4. Epinephrine
Answer: 3 - Parasympathetic
Explanations:
Parasympathetic stimulation decreases heart rate

Go to the next page if you knew the correct answer, or click the link images
below to further research the concepts in this question (if desired).

Research Concepts:
Myocardial Oxygen Demand:

Tap flag to report any problems with this question.


Question 1106: Which is NOT a signs or symptoms of developing septic
shock?

Choices:
1. Temperature 103.2 F
2. Respiratory rate 26
3. Heart rate 130 bpm
4. Blood pressure 122/80 mmHg
Answer: 4 - Blood pressure 122/80 mmHg
Explanations:
Patients developing septic shock will have increased temperature, chills,
tachypnea, tachycardia and hypotension

Go to the next page if you knew the correct answer, or click the link images
below to further research the concepts in this question (if desired).

Research Concepts:
Shock, Septic:

Tap flag to report any problems with this question.


Question 1107: All of the following statements about vecuronium are true
except:

Choices:
1. It is a non-depolarizing skeletal muscle paralytic agent
2. It has more cardiovascular effects than pancuronium and atracurium
3. It causes less histamine release than pancuronium and atracurium
4. Excretion is 30% unchanged in urine and 45% unchanged in the bile
Answer: 2 - It has more cardiovascular effects than pancuronium and
atracurium

Explanations:
Vecuronium has less cardiovascular effects than pancuronium and
atracurium

Go to the next page if you knew the correct answer, or click the link images
below to further research the concepts in this question (if desired).

Research Concepts:
Vecuronium:

Paralytics, Non-depolarizing:

Tap flag to report any problems with this question.


Question 1108: Percutaneous intervention 3 to 28 days after acute
myocardial infarction had:

Choices:
1. No clinical benefit
2. Significant clinical benefit
3. Excess mortality
4. None of the above
Answer: 1 - No clinical benefit
Explanations:
The Occluded Artery Trial (OAT) concluded no clinical benefit with PCI 3
to 28 days after acute MI with respect to death, reinfarction or heart failure
The trial had high rates of PCI procedural success and sustained patency
The trial had an average follow-up of 3 years. A trend toward excess
nonfatal reinfarction was seen during follow-up
N Engl J Med 2006; 355:2395-2407 December 7, 2006

Go to the next page if you knew the correct answer, or click the link images
below to further research the concepts in this question (if desired).

Research Concepts:
Percutaneous Coronary Intervention:

Myocardial Infarction:

Tap flag to report any problems with this question.


Question 1109: A 77 year old female with congestive heart failure due to
mitral regurgitation cannot be weaned from mechanical ventilation due to
persistent pulmonary edema despite aggressive diuresis. What is the next step?

Choices:
1. Dialysis
2. Start IV enalapril
3. Tracheostomy
4. Mitral valve replacement
Answer: 2 - Start IV enalapril
Explanations:
The patient has persistent pulmonary edema so the goal should be to
decrease afterload in order to increase the cardiac output and reduce the
retrograde flow across the incompetent mitral valve.
This is accomplished using ACE inhibitors, nitrates, and hydralazine.
Mitral valve replacement surgery would correct the problem but the patient
needs to be stabilized first.

Go to the next page if you knew the correct answer, or click the link images
below to further research the concepts in this question (if desired).

Research Concepts:
Mitral Regurgitation:

Tap flag to report any problems with this question.


Question 1110: What is the biggest disadvantage of femoral vein
cannulation?

Choices:
1. Difficult access
2. Poorly marked borders
3. Risk of contamination
4. Risk of thrombosis
Answer: 3 - Risk of contamination
Explanations:
The major disadvantage of the femoral site for venous cannulation is
contamination.

Go to the next page if you knew the correct answer, or click the link images
below to further research the concepts in this question (if desired).

Research Concepts:
Central Venous Access, Femoral Vein:

Tap flag to report any problems with this question.


Question 1111: What is the best treatment regimen for a postoperative
deep vein thrombosis (DVT)

Choices:
1. Heparin GTT and Plavix 75 mg qd
2. Aspirin 325 mg PO qd with Coumadin titrated to INR of >3.0
3. Plavix 75 mg qd PO with Coumadin titrated to INR of >2.0
4. Enoxaparin at 1 mg/kg SC with warfarin titrated to INR of 2.0 to 2.5
Answer: 4 - Enoxaparin at 1 mg/kg SC with warfarin titrated to INR of 2.0
to 2.5

Explanations:
The patient needs to become therapeutic with a heparin drip or enoxaparin.
They are started on warfarin with an INR of 2.0 to 2.5 for 3-6 months.
The treatments must overlap for several days.
DVT prophylaxis is unfractionated heparin 5000 U SQ BID or TID, or
enoxaparin 30 to 40 mg Qday or BID

Go to the next page if you knew the correct answer, or click the link images
below to further research the concepts in this question (if desired).

Research Concepts:
Deep Vein Thrombosis:

Tap flag to report any problems with this question.


Question 1112: Which is not a sign of a secondary hypertension?
Choices:
1. Abdominal bruit
2. Enlarged thyroid gland
3. Peripheral edema
4. Diaphoresis and tachycardia
Answer: 3 - Peripheral edema
Explanations:
Abdominal bruits could indicate renal artery stenosis as a cause of
hypertension.
Enlarged thyroid gland could indicate hyperthyroidism as a secondary cause
for hypertension.
Tachycardia combine with elevated blood pressure could indicate
pheochromocytoma as a cause of secondary hypertension.
Peripheral edema is a sign associated with chronic heart failure, rather than
hypertension.

Go to the next page if you knew the correct answer, or click the link images
below to further research the concepts in this question (if desired).

Research Concepts:
Secondary Hypertension:

Tap flag to report any problems with this question.


Question 1113: Pulmonary embolism is most accurately evaluated by:
Choices:
1. Plain radiograph
2. CT
3. MRI
4. Ventilation perfusion scan
Answer: 2 - CT
Explanations:
Pulmonary embolism is most accurately evaluated by CT pulmonary
angiography
The gold standard is no longer conventional pulmonary angiography
Ventilation/perfusion scan is used less now than in the past

Go to the next page if you knew the correct answer, or click the link images
below to further research the concepts in this question (if desired).

Research Concepts:
Pulmonary Embolism:

Tap flag to report any problems with this question.


Question 1114: Abdominal aortic aneurysms (AAAs) predominantly
affect:

Choices:
1. Elderly black men
2. Elderly white women
3. Elderly black women
4. Elderly white men
Answer: 4 - Elderly white men
Explanations:
AAAs predominantly affect elderly white men
Smoking is the risk factor most strongly associated with AAAs
An aneurysm is a focal dilatation in an artery with at least a 50% increase in
normal diameter
Ultrasound is the standard imaging tool for AAAs

Go to the next page if you knew the correct answer, or click the link images
below to further research the concepts in this question (if desired).

Research Concepts:
Abdominal Aortic Aneurysm:

Tap flag to report any problems with this question.


Question 1115: Which antihypertensive agent can cause severe
constipation?

Choices:
1. Acebutolol
2. Hydralazine
3. Methyldopa
4. Diltiazem
Answer: 4 - Diltiazem
Explanations:
Diltiazem is a calcium channel blocker used to treat atrial arrhythmias,
angina and hypertension
It is a useful drug but can cause constipation and headaches

Go to the next page if you knew the correct answer, or click the link images
below to further research the concepts in this question (if desired).

Research Concepts:
Diltiazem:

Tap flag to report any problems with this question.


Question 1116: In what percent of patients is probe patency of the foramen
ovale present?

Choices:
1. 1%
2. 25%
3. 50%
4. 70%
Answer: 2 - 25%
Explanations:
Patent foramen ovale is present in 10%-15% of the population when
demonstrated by contrast transthoracic echocardiography
There is a 25% prevalence of probe-patent foramen ovale seen in autopsy
studies
This difference in observed patency of the foramen ovale at autopsy and
during echocardiogram is most likely due to direct visualization of the
patent foramen ovale at autopsy. One may miss a small opening on contrast
echocardiography,
In most people, probe patent foramen ovale is of no consequence

Go to the next page if you knew the correct answer, or click the link images
below to further research the concepts in this question (if desired).

Research Concepts:
Patent Foramen Ovale:

Tap flag to report any problems with this question.


Question 1117: After administering a certain drug, how many half lives
does it take to reach steady state concentration?

Choices:
1. One half life
2. Two half lives
3. Three half lives
4. Four half lives
Answer: 4 - Four half lives
Explanations:
It takes an approximate of 4-5 half lives to reach steady states
Maintenance doses can be scheduled depending on the half life of a drug

Go to the next page if you knew the correct answer, or click the link images
below to further research the concepts in this question (if desired).

Research Concepts:
Steady State Concentration:

Tap flag to report any problems with this question.


Question 1118: Besides hypokalemia, what other factor can worsen
digoxin toxicity?

Choices:
1. Hyponatremia
2. Hypercalcemia
3. Hypomagnesemia
4. Hyperkalemia
Answer: 3 - Hypomagnesemia
Explanations:
Magnesium, like potassium, is an intracellular ion and plays a vital role in
many biochemical and metabolic reactions.
Magnesium depletion and potassium depletion usually have similar side-
effects and toxicity.
In some cases, the toxicity of digoxin due to hypokalemia cannot be
reversed until hypomagnesemia is corrected.

Go to the next page if you knew the correct answer, or click the link images
below to further research the concepts in this question (if desired).

Research Concepts:
Digoxin:

Tap flag to report any problems with this question.


Question 1119: During CPR, the highest amount of blood flow is observed
with which technique?

Choices:
1. High impulse CPR at 80 compressions per min
2. High impulse CPR at 60 compressions per min
3. High impulse CPR at 100-120 compressions per min
4. High impulse CPR at 150 compressions per min
Answer: 3 - High impulse CPR at 100-120 compressions per min
Explanations:
Ultimate method to improve blood flow is by manual compression of the
heart via thoracotomy.

Go to the next page if you knew the correct answer, or click the link images
below to further research the concepts in this question (if desired).

Research Concepts:
Cardiopulmonary Resuscitation:

Tap flag to report any problems with this question.


Question 1120: what infusion rate of sodium nitroprusside
Choices:
1. 2 mcg per kg per min
2. 5 mcg per kg per min
3. 8 mcg per kg per min
4. 10 mcg per kg per min
Answer: 1 - 2 mcg per kg per min
Explanations:
Cyanide toxicity is rare when infusion rate is less than 2 mcg per kg per
min.

Go to the next page if you knew the correct answer, or click the link images
below to further research the concepts in this question (if desired).

Research Concepts:
Sodium Nitroprusside:

Tap flag to report any problems with this question.


Question 1121: The effects of succinylcholine are prolonged when there is
abnormality in which enzyme?

Choices:
1. Glucose-6-phosphate dehydrogenase
2. Plasma cholinesterase
3. Dopamine b hydroxylase
4. Cytochrome P450 oxidase
Answer: 2 - Plasma cholinesterase
Explanations:
Plasma cholinesterase activity may be diminished in pregnancy, anemia,
myxedema, severe liver or kidney disease, infections, malignant tumors,
burns, decompensated heart disease, or peptic ulcers.
Some patients have genetic abnormalities of plasma cholinesterase.
Medications that reduce plasma cholinesterase activity include
corticosteroids, some MAO inhibitors, and oral contraceptives.
Organophosphate insecticides are irreversible inhibitors of plasma
cholinesterase.
The effects of succinylcholine are prolonged when there is abnormality in
plasma cholinesterase activity.

Go to the next page if you knew the correct answer, or click the link images
below to further research the concepts in this question (if desired).

Research Concepts:
Succinylcholine:

Tap flag to report any problems with this question.


Question 1122: What medication is most effective in the prevention of
cardiac death in patients with coronary artery disease?

Choices:
1. Nifedipine
2. Aspirin
3. Digoxin
4. Metoprolol
Answer: 4 - Metoprolol
Explanations:
Beta blockers have been shown to reduce sudden cardiac death

Go to the next page if you knew the correct answer, or click the link images
below to further research the concepts in this question (if desired).

Research Concepts:
Coronary Artery Disease:

Beta-Blockers:

Tap flag to report any problems with this question.


Question 1123: A 42 year old male experience an episode of syncope
while cutting the grass. The episode was witnessed. He has no medical care for
10 years but has no history of a heart murmur. He does not smoke, drink, or use
illicit drugs. Exam shows hypertension and a loud murmur of aortic stenosis.
ECG shows LVH with strain. Select the most likely cause of his aortic stenosis.

Choices:
1. Rheumatic heart disease
2. Age related calcification
3. Bicuspid aortic valve
4. Congenital heart disease
Answer: 3 - Bicuspid aortic valve
Explanations:
Age related calcific stenosis of the aortic valve usually presents in the 70s
or 80s
Congenital heart disease would have be noted earlier
AS secondary to rheumatic heart disease presents in the 60s or 70s
Bicuspid aortic valve is more common in males, occurs in 1 to 2% of
individuals, and most often presents in 40s or 50s

Go to the next page if you knew the correct answer, or click the link images
below to further research the concepts in this question (if desired).

Research Concepts:
Aortic Stenosis:

Tap flag to report any problems with this question.


Question 1124: Select the organism that most commonly causes infective
endocarditis.

Choices:
1. Streptococcus pneumonia
2. Streptococcus viridans
3. Streptococcus pyogenes
4. Streptococcus agalactiae
Answer: 2 - Streptococcus viridans
Explanations:
Streptococcus pneumonia often causes otitis media, sinusitis, and
pneumonia
Streptococcus agalactiae, part of the normal vaginal flora, can cause
neonatal meningitis and sepsis
Streptococcus pyogenes often causes pharyngitis that may be complicated
by nephritis or rheumatic fever
Streptococcus viridans is the most common cause of infective endocarditis

Go to the next page if you knew the correct answer, or click the link images
below to further research the concepts in this question (if desired).

Research Concepts:
Endocarditis:

Tap flag to report any problems with this question.


Question 1125: What organism is associated with rheumatic fever?
Choices:
1. Staphylococcus
2. Pseudomonas
3. Streptococcus
4. Ticks
Answer: 3 - Streptococcus
Explanations:
Rheumatic fever is associated with streptococcal infection

Go to the next page if you knew the correct answer, or click the link images
below to further research the concepts in this question (if desired).

Research Concepts:
Rheumatic Fever, Acute:

Tap flag to report any problems with this question.


Question 1126: All "amide" local anesthetics are metabolized in which of
the following?

Choices:
1. Liver
2. Kidney
3. Gastrointestinal mucosa
4. Serum
Answer: 1 - Liver
Explanations:
All amide anesthetics are metabolized in the liver.

Go to the next page if you knew the correct answer, or click the link images
below to further research the concepts in this question (if desired).

Research Concepts:
Local Anesthetics:

Tap flag to report any problems with this question.


Question 1127: Which of the following diuretics has been known to affect
lipid levels?

Choices:
1. Spironolactone
2. Triamterene
3. Acetazolamide
4. Hydrochlorothiazide
Answer: 4 - Hydrochlorothiazide
Explanations:
Thiazide diuretics are used to treat patients with hypertension and heart
failure.
Thiazides act by blocking Na+/Cl- transporters in the distal convoluted
tubule.
Prolonged use of hydrochlorothiazide can cause elevations in serum
glucose, lipids, and uric acid.
Hydrochlorothiazide can cause profound hypokalemia.

Go to the next page if you knew the correct answer, or click the link images
below to further research the concepts in this question (if desired).

Research Concepts:
Diuretics:

Tap flag to report any problems with this question.


Question 1128: Which is false about atherosclerosis?
Choices:
1. Most common lesion occur at sites of bifurcation of arteries
2. Causes aneurysms in the circle of Willis
3. Most common site affected is the abdominal aorta below the renal arteries
4. Atherosclerotic aneurysms can become infected
Answer: 2 - Causes aneurysms in the circle of Willis
Explanations:
Atherosclerotic disease typically occurs in areas of bifurcation where there
is turbulence of blood. Besides the femoral and iliac, the carotid artery is
frequently involved
In the circle of Willis, the aneurysms are congenital and not associated with
atherosclerosis
The abdominal aorta often gets atherosclerotic and may result in an
aneurysm
Mycotic aneurysms can occur in atherosclerotic vessels

Go to the next page if you knew the correct answer, or click the link images
below to further research the concepts in this question (if desired).

Research Concepts:
Atherosclerosis:

Tap flag to report any problems with this question.


Question 1129: Which of the following is contraindicated in the treatment
of aortic regurgitation?

Choices:
1. IABP (intra-aortic balloon pump)
2. Vasodilators, inotropes, diuretics
3. AVR (aortic valve replacement)
4. Medical management as a bridge to surgery
Answer: 1 - IABP (intra-aortic balloon pump)
Explanations:
IABP is contraindicated, as the pressure would increase the aortic
regurgitation

Go to the next page if you knew the correct answer, or click the link images
below to further research the concepts in this question (if desired).

Research Concepts:
Aortic Regurgitation:

Intra-Aortic Balloon Pump:

Tap flag to report any problems with this question.


Question 1130: Which is not a typical side effect of intravenous lidocaine?
Choices:
1. Tinnitus
2. Dizziness
3. Numbness of the extremities
4. Palpitations
Answer: 4 - Palpitations
Explanations:
Common adverse effects of lidocaine hydrochloride include dizziness,
tinnitus, blurred vision, tremors, numbness and tingling of extremities,
excessive perspiration, hypotension, seizures, and finally coma

Go to the next page if you knew the correct answer, or click the link images
below to further research the concepts in this question (if desired).

Research Concepts:
Toxicity, Lidocaine:

Tap flag to report any problems with this question.


Question 1131: A patient presents with a complaint of chest pain, an
abnormal ECG inferiorly and on exam has elevated jugular venous pressure
(JVP), no murmurs and clear lungs. What is the most likely diagnosis?

Choices:
1. Inferior wall myocardial infarction only
2. GERD
3. Inferior wall myocardial infarction (IWMI) with additional right ventricular
infarction (RVI)
4. Congestive heart failure
Answer: 3 - Inferior wall myocardial infarction (IWMI) with additional right
ventricular infarction (RVI)

Explanations:
Elevated JVP with clear lungs in setting of IWMI is pathognomic of
additional RVI
Isolated RVI is a very rare occurrence
CHF usually presents with elevated JVP and rales
GERD is not a diagnostic consideration in this clinical scenario

Go to the next page if you knew the correct answer, or click the link images
below to further research the concepts in this question (if desired).

Research Concepts:
Inferior Wall Myocardial Infarction:

Right Ventricular Infarction:

Tap flag to report any problems with this question.


Question 1132: A low serum potassium is least likely to cause which of
the following?

Choices:
1. Muscle weakness
2. Palpitations
3. Paresthesias
4. Diarrhea
Answer: 4 - Diarrhea
Explanations:
Hypokalemia (low serum potassium) tends to cause constipation, not
diarrhea
Conversely, diarrhea can cause hypokalemia although it is very common
with protracted emesis

Go to the next page if you knew the correct answer, or click the link images
below to further research the concepts in this question (if desired).

Research Concepts:
Hypokalemia:

Tap flag to report any problems with this question.


Question 1133: Which is not a part of the metabolic syndrome?
Choices:
1. Regular exercise
2. Obesity
3. Hypertension
4. Insulin resistance
Answer: 1 - Regular exercise
Explanations:
Regular exercise is not a part of the metabolic syndrome
Metabolic syndrome includes dyslipidemia, sedentary lifestyle

Go to the next page if you knew the correct answer, or click the link images
below to further research the concepts in this question (if desired).

Research Concepts:
Metabolic Syndrome:

Tap flag to report any problems with this question.


Question 1134: All of the following are true regarding cyanosis except:
Choices:
1. It is usually is a sign of hypoxemia
2. It is most often seen and detected on the eyelids
3. Peripheral vasoconstriction can cause cyanosis
4. It occurs when unoxygenated hemoglobin content reaches 5 g/dL
Answer: 2 - It is most often seen and detected on the eyelids
Explanations:
Cyanosis is a blue or purple tinge to skin or mucous membranes that is most
often detected on finger tips and lips
Peripheral vasoconstriction as occurs with Raynaud phenomenon can cause
peripheral cyanosis
Evaluation of a patient with cyanosis requires measuring oxygen saturation
and arterial blood gases
Cyanosis can be caused by methemoglobin, which cannot release oxygen,
producing a normal oxygen saturation and decreased PO2

Go to the next page if you knew the correct answer, or click the link images
below to further research the concepts in this question (if desired).

Research Concepts:
Cyanosis:

Tap flag to report any problems with this question.


Question 1135: A patient is in the ICU with congestive heart failure.
Which of the following cannot be used as an inotropic agent?

Choices:
1. Thyroid hormone
2. Milrinone
3. Calcium
4. Insulin
Answer: 4 - Insulin
Explanations:
Insulin has no inotropic activity.
Common inotropes used in the ICU include bipyridine derivative-like
milrinone, calcium, digoxin, and catecholamines.
Sometimes glucagon and thyroid hormone are also be used to stimulate the
heart's pumping activity.
Negative inotropes include beta-blockers, calcium channel blockers, and
class 1 anti-arrhythmic agents.

Go to the next page if you knew the correct answer, or click the link images
below to further research the concepts in this question (if desired).

Research Concepts:
Heart Failure:

Tap flag to report any problems with this question.


Question 1136: Which of the following is best to assess function of
platelets?

Choices:
1. Prothrombin time
2. Partial thromboplastin time
3. Peripheral smear
4. Functional closure time
Answer: 4 - Functional closure time
Explanations:
The platelet function time is a new method of assessing platelet function
This vital test measures time required for a platelet plug to occlude an
aperture in a membrane containing some collagen
The bleeding rime is no longer used today and is not satisfactory for use in
children
Platelet aggregation has been used to evaluate neonatal platelet function but
there is a lot of difficulty in obtaining adequate blood from neonates
without it first clotting

Go to the next page if you knew the correct answer, or click the link images
below to further research the concepts in this question (if desired).

Research Concepts:
Functional Closure Time:

Tap flag to report any problems with this question.


Question 1137: Which antihypertensive agent is associated with a lupus
like syndrome?

Choices:
1. Nifedipine
2. Hydralazine
3. Alpha methyldopa
4. Minoxidil
Answer: 2 - Hydralazine
Explanations:
Procainamide, hydralazine, and isoniazid can all cause lupus like syndrome.
Incidence of lupus features are common in slow acetylators.
The lupus symptoms disappear once the drug is stopped.

Go to the next page if you knew the correct answer, or click the link images
below to further research the concepts in this question (if desired).

Research Concepts:
Lupus Erythematosus, Drug-Induced:

Antihypertensive Medications:

Tap flag to report any problems with this question.


Question 1138: Which is true regarding Venturi mask use?
Choices:
1. It delivers 70% oxygen
2. It delivers a carbon dioxide oxygen mixture
3. It delivers an air oxygen mixture
4. It delivers 100% oxygen
Answer: 3 - It delivers an air oxygen mixture
Explanations:
The Venturi mask is an air-entrainment mask which can deliver a known
oxygen concentration to patients.
A Venturi mask is a type of high-flow oxygen therapy device.
The device is color-coded to supply different settings of FiO2.

Go to the next page if you knew the correct answer, or click the link images
below to further research the concepts in this question (if desired).

Research Concepts:
Respiratory Therapy:

Respiratory Support:

Tap flag to report any problems with this question.


Question 1139: What is the afferent nerve for the carotid baroreceptors?
Choices:
1. CN V
2. CN VII
3. CN IX
4. CN X
Answer: 3 - CN IX
Explanations:
Glossopharyngeal nerve relays signal from carotid baroreceptors

Go to the next page if you knew the correct answer, or click the link images
below to further research the concepts in this question (if desired).

Research Concepts:
Carotid Artery:

Carotid, Baroreceptors:

Tap flag to report any problems with this question.


Question 1140: Which is least likely associated with hypokalemia?
Choices:
1. ST depression
2. Vomiting
3. Ileus
4. Hyperreflexia
Answer: 4 - Hyperreflexia
Explanations:
Patients with hypokalemia usually are asymptomatic. However, symptoms
when they occur are linked to problems with the renal, GI, CNS and heart
Common symptoms include weakness, paralysis, constipation, nausea,
psychosis and depression
Findings associated with hypokalemia includes ileus, hypotension,
arrhythmias, hypoventilation, decreased reflexes or tetany
Causes of hypokalemia include losses in the kidney and GI tract.
Medications like diuretics, steroids, theophylline and aminoglycosides also
cause hypokalemia

Go to the next page if you knew the correct answer, or click the link images
below to further research the concepts in this question (if desired).

Research Concepts:
Hypokalemia:

Tap flag to report any problems with this question.


Question 1141: You have decided to do an all arterial bypass, using the
bilateral mammary and the radial artery. Which of the following is one
significant factor in perioperative and post-operative care?

Choices:
1. Maintain blood pressure above 80 mmHg
2. Use of inotropes
3. Use of calcium channel blockers
4. Use of aprotinin
Answer: 3 - Use of calcium channel blockers
Explanations:
When ever one uses the radial artery as a conduit, it is important to use a
vasodilator.
Almost all surgeons use some form of vasodilator, and the calcium channel
blockers are preferred.
The radial artery has a high incidence of spasm and thus it should be
vasodilator at all times.

Go to the next page if you knew the correct answer, or click the link images
below to further research the concepts in this question (if desired).

Research Concepts:
Radial Artery Coronary Bypass:

Coronary Artery Bypass Graft:

Tap flag to report any problems with this question.


Question 1142: An asymptomatic patient with mild mitral valve stenosis
should be treated with which of the following?

Choices:
1. Observation
2. ECG every 6 months
3. Echocardiogram every year
4. Surgery
Answer: 1 - Observation
Explanations:
Asymptomatic patient with mild mitral stenosis requires no intervention.
Requires echo yearly.

Go to the next page if you knew the correct answer, or click the link images
below to further research the concepts in this question (if desired).

Research Concepts:
Mitral Stenosis:

Tap flag to report any problems with this question.


Question 1143: For which of the following conditions is NOT a
contraindication for fibrinolytic therapy?

Choices:
1. Recent hemorrhagic brain injury
2. Recent head trauma
3. Recent acute pulmonary embolism
4. Presence of aortic dissection
Answer: 3 - Recent acute pulmonary embolism
Explanations:
Absolute contraindications for thrombolytics include any type of head
injury, intracranial bleeding, or a recent hemorrhagic stroke.
Uncontrolled high blood pressure is also a contraindication for the use of
thrombolytics.
Individuals who have been diagnosed with an aortic dissection should not
be treated with thrombolytics.
When pulmonary embolism is associated with low cardiac output,
hypotension, and shock, it can be treated with thrombolytic therapy.

Go to the next page if you knew the correct answer, or click the link images
below to further research the concepts in this question (if desired).

Research Concepts:
Thrombolysis:

Tap flag to report any problems with this question.


Question 1144: Pulsus paradoxus is a decrease in systolic blood pressure
commonly observed during:

Choices:
1. Apnea
2. Inspiration
3. Expiration
4. Stridor
Answer: 2 - Inspiration
Explanations:
Pulsus paradoxus describes a decrease of systolic pressure during
inspiration.

Go to the next page if you knew the correct answer, or click the link images
below to further research the concepts in this question (if desired).

Research Concepts:
Pulsus Paradoxus:

Tap flag to report any problems with this question.


Question 1145: For a patient who is taking warfarin for some time, INR
should be checked:

Choices:
1. Daily
2. Weekly
3. Biweekly
4. Monthly
Answer: 4 - Monthly
Explanations:
Close monitoring is needed during the first 4 weeks of warfarin initiation
Once patients are stabilized monthly INR check up would suffice
Patients can be taught to check their own INRs using portable digital
machines
As long as the INR is in therapeutic range, there is no need to adjust the
medication

Go to the next page if you knew the correct answer, or click the link images
below to further research the concepts in this question (if desired).

Research Concepts:
Warfarin:

Tap flag to report any problems with this question.


Question 1146: Which of the following disorders does NOT cause
cyanosis?

Choices:
1. Tetralogy of Fallot
2. Transposition of great vessels
3. Total anomalous pulmonary venous return
4. Type IV truncus arteriosus
Answer: 4 - Type IV truncus arteriosus
Explanations:
In truncus, part or the entire pulmonary artery comes of the aorta.
The shunt in truncus is left to right and most infants develop congestive
heart failure.
The disorder, which presents with cyanosis at birth, is transposition of the
great vessels.
Congenital heart disorders that present with cyanosis typically have a right
to left shunt.

Go to the next page if you knew the correct answer, or click the link images
below to further research the concepts in this question (if desired).

Research Concepts:
Truncus Arteriosus:

Tap flag to report any problems with this question.


Question 1147: Which of the following is a FALSE statement about the
Heart Outcomes Prevention Evaluation (HOPE) study where patients over 55
received the ACE inhibitor ramipril?

Choices:
1. Although none had congestive heart failure, all had a history of coronary
artery disease
2. Compared to placebo group, there was a 20% relative risk reduction of
myocardial infarctions in the treatment group
3. There was a 32% relative risk reduction of strokes in the treatment group
4. Ramipril reduced the risk of CHF by 23%
Answer: 1 - Although none had congestive heart failure, all had a history of
coronary artery disease

Explanations:
The HOPE study was a large, double-blind, placebo-controlled trial that
determined the risk of cardiovascular events in over 9500 patients studied
in 267 centers in 19 countries.
Subjects could not have CHF, but were at high risk of vascular death or
morbidity by virtue of age (required to be older than 55 years), or because
they had either diabetes or a prior vascular event or existing vascular
disease.
The study was stopped about one year early, after 4.5 years, since the
available data supported a more favorable outcome for patients treated with
ramipril.
"Effects of an angiotensin-converting-enzyme inhibitor, ramipril, on
cardiovascular events in high-risk patients. The Heart Outcomes Prevention
Evaluation Study Investigators." The New England Journal of Medicine
[2000, 342(3):145-153]

Go to the next page if you knew the correct answer, or click the link images
below to further research the concepts in this question (if desired).

Research Concepts:
Angiotensin Converting Enzyme Inhibitors (ACEI):

Tap flag to report any problems with this question.


Question 1148: Which of the following symptoms is not commonly seen
with atrioventricular canal defects?

Choices:
1. Poor weight gain
2. Congestive heart failure
3. Chronic upper respiratory tract infections
4. Constipation
Answer: 4 - Constipation
Explanations:
Atrioventricular canal defect symptoms can include poor weight gain,
chronic upper respiratory infections, feeding difficulties, excessive
sweating, difficulty with crying, and nasal flaring
Later patients may have congestive heart failure with easy fatigability,
edema, and dyspnea
Constipation is not seen

Go to the next page if you knew the correct answer, or click the link images
below to further research the concepts in this question (if desired).

Research Concepts:
Atrioventricular Canal Defects:

Tap flag to report any problems with this question.


Question 1149: Which is not associated with hypovolemic shock?
Choices:
1. Inadequate tissue perfusion with resultant tissue hypoxia
2. Decreased circulating blood volume and decreased venous return
3. Kidneys, heart, brain, lungs, and liver are spared
4. Blood shunting to vital organs
Answer: 3 - Kidneys, heart, brain, lungs, and liver are spared
Explanations:
Hypovolemic shock is caused by decreased circulating blood volume,
causing inadequate tissue perfusion and oxygenation
Although blood is shunted to the vital organs, multiorgan failure can ensue

Go to the next page if you knew the correct answer, or click the link images
below to further research the concepts in this question (if desired).

Research Concepts:
Hypovolemic Shock:

Tap flag to report any problems with this question.


Question 1150: Which of the following findings is a relatively consistent
finding in moderately anemic patients?

Choices:
1. Bilateral basilar crackles
2. Pale palms
3. Low hematocrit
4. Wide pulse pressure
Answer: 4 - Wide pulse pressure
Explanations:
Pulse pressure-difference between systolic and diastolic blood pressure
The hyperdynamic circulation makes the pulse pressure wider in anemia
Hematocrit could be falsely elevated in dehydrated patients
Pale palms are seen in severe anemia

Go to the next page if you knew the correct answer, or click the link images
below to further research the concepts in this question (if desired).

Research Concepts:
Anemia:

Tap flag to report any problems with this question.


Question 1151: A 72 year old male is admitted to the hospital for elective
bowel surgery. The patient claims that his wife died of a pulmonary embolus. It
is decided to treat the patient prophylactically to prevent a deep vein thrombosis.
What is the least effective way to prevent a deep vein thrombosis?

Choices:
1. Mini-dose heparin
2. Graded stockings
3. Dextran
4. ASA
Answer: 4 - ASA
Explanations:
Mechanical compression using compression stockings is widely used but
the latest evidence indicates that they must be combined with a
pharmacological method to prevent DVT.
Aspirin is not recommended and it does not help prevent DVT. Compared
to LMWH, aspirin fared worse than placebo.
Both heparin and LMWH can be used as prophylaxis against DVT. LMWH
has a long half life and a decreased tendency to induce HIT. The LWMH do
not need monitoring and are cost effective.
Graded intermittent pneumatic stockings are also widely used and effective
in preventing DVT.

Go to the next page if you knew the correct answer, or click the link images
below to further research the concepts in this question (if desired).

Research Concepts:
Deep Venous Thrombosis Prophylaxis:

Tap flag to report any problems with this question.


Question 1152: A neonate born without complications at 37 weeks of
gestation becomes progressively tachypneic and develops respiratory distress by
36 hours. The chest radiograph shows bilateral pleural effusions, a diffuse
granular pattern, and cardiomegaly. By 4 hours of age the patient becomes
hypotensive. WBC is 3,000. Select the most likely cause.

Choices:
1. Hyaline membrane disease
2. Congenital pulmonary lymphangiectasia
3. Total anomalous pulmonary venous connection
4. Group B Streptococcal pneumonia
Answer: 3 - Total anomalous pulmonary venous connection
Explanations:
In this patient the pulmonary veins do not go to the left atrium but rather
drain into systemic circulation or the right atrium.
It is dependent on the foramen ovale or atrial septal defect.
The increased pulmonary vasculature shows up as a granular pattern on
chest x-ray. The heart size in normal when the pulmonary veins are
unobstructed there in enlargement of the heart.
Medical treatment with intubation and pressors is initial treatment but
surgical repair is needed.

Go to the next page if you knew the correct answer, or click the link images
below to further research the concepts in this question (if desired).

Research Concepts:
Total Anomalous Pulmonary Venous Connection:

Tap flag to report any problems with this question.


Question 1153: A patient with sickle cell anemia is undergoing open heart
surgery. What is the most critical factor?

Choices:
1. Avoid high perfusion pressures
2. Avoid hypothermia
3. Avoid crystalloid cardioplegia
4. Avoid vein grafts
Answer: 2 - Avoid hypothermia
Explanations:
Sickle cell patients who undergo open heart surgery should be kept warm
and excessive cooling should be avoided

Go to the next page if you knew the correct answer, or click the link images
below to further research the concepts in this question (if desired).

Research Concepts:
Open-heart surgery, Sickle cell Hemoglobinopathy:

Tap flag to report any problems with this question.


Question 1154: Which is most suggestive of a postoperative wound
infection?

Choices:
1. Erythema around the incision on the second postoperative day
2. Excessive tenderness around the incision on the second postoperative day
3. Fever on the fifth postoperative day
4. Serous drainage from the incision on the first postoperative day
Answer: 3 - Fever on the fifth postoperative day
Explanations:
Wound infections are characterized by fever beginning on the fourth or fifth
postoperative day.
Erythema and excessive tenderness around the incision may also occur at
this time.

Go to the next page if you knew the correct answer, or click the link images
below to further research the concepts in this question (if desired).

Research Concepts:
Wound Infection:

Tap flag to report any problems with this question.


Question 1155: Which of the following correctly describes ejection
fraction?

Choices:
1. (End diastolic volume - end systolic volume) / (end diastolic volume)
2. End diastolic volume / end systolic volume
3. (End diastolic volume x end systolic volume) / 100
4. End diastolic volume - end systolic volume
Answer: 1 - (End diastolic volume - end systolic volume) / (end diastolic
volume)

Explanations:
Ejection fraction is the fraction of blood pumped out by the ventricles with
each heartbeat. The ejection fraction is the fraction of the end diastolic
volumes that is ejected with each beat.
A normal person has an ejection fraction of about 60-70%.
When the ejection fraction is less than 20% the prognosis is poor.
Ejection fraction is measured by echocardiography. Other methods to
measure ejection fraction include MRI, CT, and MUGA scan.

Go to the next page if you knew the correct answer, or click the link images
below to further research the concepts in this question (if desired).

Research Concepts:
Cardiac Physiology:

Tap flag to report any problems with this question.


Question 1156: Which of the following medications should be used with
caution in patients with diabetes mellitus?

Choices:
1. Hydralazine
2. Prazosin
3. Lisinopril
4. Propranolol
Answer: 4 - Propranolol
Explanations:
Adverse effects of beta blockers include bronchospasm and alteration of
glucose and lipid metabolism.
There is a lot of evidence showing that beta blockers can worsen
hyperglycemia. They should not be used as first line drugs to treat diabetics.
In diabetics blood pressure should be reduced to less than 130/80 mmHg to
reduce morbidity and mortality.
ACE inhibitors are the drugs of choice in the management of hypertension
in diabetic patients. These drugs have been found to decrease vascular
smooth muscle proliferation and increase fibrinolysis, leading to improved
renal function in diabetics.

Go to the next page if you knew the correct answer, or click the link images
below to further research the concepts in this question (if desired).

Research Concepts:
Beta-Blockers:

Diabetes Mellitus, Type 2:

Diabetes Mellitus, Type 1:


Tap flag to report any problems with this question.
Question 1157: What cardiac abnormally is associated with Marfan
syndrome?

Choices:
1. Aortic regurgitation
2. Mitral stenosis
3. Aortic stenosis
4. Hypertrophy
Answer: 1 - Aortic regurgitation
Explanations:
Marfan's syndrome is associated with aortic regurgitation
It is a connective tissue disorder

Go to the next page if you knew the correct answer, or click the link images
below to further research the concepts in this question (if desired).

Research Concepts:
Marfan Syndrome:

Aortic Regurgitation:

Tap flag to report any problems with this question.


Question 1158: A patient received a neuromuscular blocker and had
neuromuscular paralysis for more than 7 hours. Which of the following is false?

Choices:
1. The most likely agent is succinylcholine
2. The most common inherited cause is a mutation of E1 locus on the long
arm of chromosome 3
3. Succinylcholine has a half-life of over an hour.
4. The dibucaine number is high in these patients
Answer: 3 - Succinylcholine has a half-life of over an hour.
Explanations:
An ultra short-acting depolarizing skeletal muscle relaxant, succinylcholine
bonds with motor endplate cholinergic receptors to produce depolarization.
These cause fasciculations.
The neuromuscular block remains as long as sufficient quantities of
succinylcholine remain, and is characterized by a flaccid paralysis. The
elimination half-life of succinylcholine is estimated to be 47 seconds.
Inadvertent overdose, or patients deficient in pseudocholinesterase may
result in prolonged apnea.
Mechanical ventilation with oxygen should be used until recovery.

Go to the next page if you knew the correct answer, or click the link images
below to further research the concepts in this question (if desired).

Research Concepts:
Neuromuscular Blocking Agents:

Tap flag to report any problems with this question.


Question 1159: Which of the following is not caused by lidocaine?
Choices:
1. Seizures
2. Myocardial excitation
3. Numbness
4. Confusion
Answer: 2 - Myocardial excitation
Explanations:
Lidocaine is a myocardial depressant used to treat arrhythmias and as a
local anesthetic.
The first sign of central toxicity is perioral numbness.
It is an amino amide–type local anesthetic with rapid onset of action and
duration that is intermediate.
Adverse effects include hypotension, lightheadedness, headache, visual
disturbances, sedation, tinnitus, dysarthria, and muscle tremors.

Go to the next page if you knew the correct answer, or click the link images
below to further research the concepts in this question (if desired).

Research Concepts:
Lidocaine:

Tap flag to report any problems with this question.


Question 1160: What is the cardinal symptom of left ventricular (LV)
failure?

Choices:
1. Edema
2. Fatigue
3. Shortness of breath (SOB)
4. None of the above
Answer: 3 - Shortness of breath (SOB)
Explanations:
SOB is the cardinal symptom of LV failure
Other symptoms include tachycardia, fatigue and edema
New York Heart Association classifies heart failure and comprises 4
classes-I, II, III and IV

Go to the next page if you knew the correct answer, or click the link images
below to further research the concepts in this question (if desired).

Research Concepts:
Left Ventricular Failure:

Tap flag to report any problems with this question.


Question 1161: Which of the following would not be a contraindication to
the use of electrical stimulation for wound healing?

Choices:
1. Use on the thorax in a patient with a cardiac arrhythmia
2. Use on a patient with a pacemaker
3. Use in an area with decreased blood flow
4. Use in the area of the carotid sinus (anterior neck)
Answer: 3 - Use in an area with decreased blood flow
Explanations:
Electrical stimulation increases blood flow, encourages wound contraction,
and decreases edema
It should not be used near metal staples, through malignant tissue, over the
carotid sinus, on the thorax of patients with cardiac problems, or in patients
with pacemakers

Go to the next page if you knew the correct answer, or click the link images
below to further research the concepts in this question (if desired).

Research Concepts:
Wound Healing:

Electrical Stimulation:

Tap flag to report any problems with this question.


Question 1162: Which of the following conditions is least likely to
precipitate diabetic ketoacidosis?

Choices:
1. Pregnancy
2. Infection
3. Stress
4. Starvation
Answer: 4 - Starvation
Explanations:
Any stress condition can precipitate diabetic ketoacidosis
Among common conditions, pregnancy, infection and non-compliance to
medication and diet usually trigger DKA
Starvation is also a stressful factor but it often leads to hypoglycemia
especially in patients who are on insulin therapy

Go to the next page if you knew the correct answer, or click the link images
below to further research the concepts in this question (if desired).

Research Concepts:
Diabetic Ketoacidosis:

Tap flag to report any problems with this question.


Question 1163: Which of the following disease process is characterized by
a right to left blood shunt?

Choices:
1. Patent ductus arteriosus
2. Tetralogy of Fallot
3. Ventricular septal defect
4. Atrial septal defect
Answer: 2 - Tetralogy of Fallot
Explanations:
Only TOF is characterized by right to left shunt

Go to the next page if you knew the correct answer, or click the link images
below to further research the concepts in this question (if desired).

Research Concepts:
Tetralogy Of Fallot:

Tap flag to report any problems with this question.


Question 1164: A 65 year old female is brought to the emergency
department after a motor vehicle accident. She was hit head on while traveling
on a highway. She is in severe pain and holding her chest. There is a large
contusion over the anterior chest, muffled heart sounds, pulsus paradoxus, and
jugular venous distension. Blood pressure is 85/55 mm Hg. Select initial
management.

Choices:
1. Chest radiograph
2. CT of the chest
3. Pericardiocentesis
4. Electrocardiogram
Answer: 3 - Pericardiocentesis
Explanations:
The patient has pericardial tamponade and requires emergent
pericardiocentesis
There is no time for investigative studies but transesophageal
echocardiogram is the procedure of choice
Beck's triad includes hypotension, muffled heart sounds, and JVD

Go to the next page if you knew the correct answer, or click the link images
below to further research the concepts in this question (if desired).

Research Concepts:
Cardiac Tamponade:

Tap flag to report any problems with this question.


Question 1165: What specific class of medication was involved with the
Survival and Ventricular Enlargement study?

Choices:
1. Beta blocker
2. ACE inhibitor
3. Calcium channel blocker
4. Analgesic
Answer: 2 - ACE inhibitor
Explanations:
The Survival and Ventricular Enlargement trial demonstrated that ACE
inhibitors can lower the mortality of left ventricular dysfunction

Go to the next page if you knew the correct answer, or click the link images
below to further research the concepts in this question (if desired).

Research Concepts:
Angiotensin Converting Enzyme Inhibitors (ACEI):

Left Ventricular Dysfunction:

Tap flag to report any problems with this question.


Question 1166: Preload does not affect:
Choices:
1. Peripheral resistance
2. Stroke volume
3. Ventricular contractility
4. Oxygen demand
Answer: 1 - Peripheral resistance
Explanations:
Vascular resistance is not a function of preload

Go to the next page if you knew the correct answer, or click the link images
below to further research the concepts in this question (if desired).

Research Concepts:
Preload:

Tap flag to report any problems with this question.


Question 1167: An 18-year-old female has hyperextensible skin,
hypermobile joints, and dies of abdominal aortic aneurism. What defect is
present in this patient?

Choices:
1. Lysine hydroxylase deficiency
2. Decreased translation of collagen III
3. Procollagen peptidase deficiency
4. A collagen I mutation
Answer: 2 - Decreased translation of collagen III
Explanations:
Hyperextensible skin and hypermobile joints is secondary to a collagen
defect in Ehlers-Danlos syndrome
There are many types of Ehlers-Danlos syndrome with varying severity
Type IV is characterized by either aortic or intestinal rupture secondary to a
deficiency in transcription or translation of collagen III
Type VI manifests in rupturing of the eyeball secondary to a deficiency in
lysine hydroxylase that results in a decrease in collagen glycosylation

Go to the next page if you knew the correct answer, or click the link images
below to further research the concepts in this question (if desired).

Research Concepts:
Ehlers-Danlos Syndrome:

Tap flag to report any problems with this question.


Question 1168: Which of the following is used to manage the toxicity of
class 1 anti-arrhythmic agents?

Choices:
1. Lidocaine
2. Digoxin
3. Sodium lactate
4. Diuretics
Answer: 3 - Sodium lactate
Explanations:
All class 1 anti-arrhythmic agents have the potential to cause arrhythmias.
Torsades can occur with quinidine.
Hyperkalemia can worsen the toxicity of class 1 anti-arrhythmic agents.
Treatment of overdose is with sodium lactate, which can reverse the
hypotension and arrhythmias.

Go to the next page if you knew the correct answer, or click the link images
below to further research the concepts in this question (if desired).

Research Concepts:
Antiarrhythmic Medication:

Tap flag to report any problems with this question.


Question 1169: Which is the best way to evaluate therapeutic effects of
LMWH (low molecular weight heparin?

Choices:
1. Bleeding time
2. PTT
3. PT and INR
4. Anti-Xa assay
Answer: 4 - Anti-Xa assay
Explanations:
Monitoring the levels of LMWH is usually not necessary but may be done
using the anti-Xa assay

Go to the next page if you knew the correct answer, or click the link images
below to further research the concepts in this question (if desired).

Research Concepts:
Low Molecular Weight Heparin (LMWH):

Tap flag to report any problems with this question.


Question 1170: Which of the following congenital heart disorders places
children at risk for a brain abscess?

Choices:
1. Coarctation of aorta
2. Ebstein's Anomaly
3. Tetralogy of Fallot
4. Patent ductus arteriosus
Answer: 3 - Tetralogy of Fallot
Explanations:
A brain abscess is more common in congenital heart diseases with right to
left shunts.

Go to the next page if you knew the correct answer, or click the link images
below to further research the concepts in this question (if desired).

Research Concepts:
Brain Abscess:

Tetralogy Of Fallot:

Tap flag to report any problems with this question.


Question 1171: A post-op patient develops acute confusional state with
hallucinations, marked agitation and altered sleep-wake cycle. What is the
diagnosis?

Choices:
1. Delirium
2. Paranoid personality disorder
3. Depression
4. Generalized anxiety disorder
Answer: 1 - Delirium
Explanations:
Delirium is the single most common acute mental disorder affecting adults
in the hospital
It occurs in 10-20% of hospitalized adults, 30-40% of elderly patients, and
80% of ICU patients

Go to the next page if you knew the correct answer, or click the link images
below to further research the concepts in this question (if desired).

Research Concepts:
Delirium:

Tap flag to report any problems with this question.


Question 1172: The cephalosporin not associated with a disulfiram-like
reaction is:

Choices:
1. Cephalexin
2. Cefazolin
3. Cefotetan
4. Cefoperazone
Answer: 1 - Cephalexin
Explanations:
When alcohol is consumed after ingestion of these drugs, severe nausea,
flushing, and vomiting may occur.
Cephalexin is a first-generation antibiotic, which does not cause a
disulfiram-like reaction.

Go to the next page if you knew the correct answer, or click the link images
below to further research the concepts in this question (if desired).

Research Concepts:
Cephalosporins, Antabuse effect:

Tap flag to report any problems with this question.


Question 1173: A beeping heart monitor represents:
Choices:
1. Sensory overload
2. Sleep deprivation
3. A hallucination
4. Sensory monotony
Answer: 4 - Sensory monotony
Explanations:
A beeping heart monitor is a monotonous stimuli representing sensory
monotony
Sensory overload in a hospital setting can lead to patient sleep deprivation
Sleep deprivation can lead to hallucinations, agitation, confusion, and
psychosis during hospitalization

Go to the next page if you knew the correct answer, or click the link images
below to further research the concepts in this question (if desired).

Research Concepts:
Sensory Monotony:

Tap flag to report any problems with this question.


Question 1174: A patient develops bloody diarrhea after taking 10 days
course of clindamycin. Which of the following medications can be given to treat
this problem?

Choices:
1. Erythromycin
2. Gentamicin
3. Amikacin
4. Metronidazole
Answer: 4 - Metronidazole
Explanations:
C. difficile infection can occur after prolonged antibiotic therapy
This infection is commonly termed as pseudomembranous colitis
Metronidazole is a drug of choice for this anaerobic infection
Vancomycin is also effective.

Go to the next page if you knew the correct answer, or click the link images
below to further research the concepts in this question (if desired).

Research Concepts:
Pseudomembranous Colitis:

Tap flag to report any problems with this question.


Question 1175: Which of the following is not true of beta-receptor
blocking drugs?

Choices:
1. They can be used as monotherapy for pheochromocytoma
2. They can be used to treat a migraine
3. They can be used to treat variceal bleeds
4. They can be used to treat an essential tremor
Answer: 1 - They can be used as monotherapy for pheochromocytoma
Explanations:
Beta-blockers should only be used in pheochromocytoma after alpha
blockade has taken place.
The treatment of pheochromocytoma is usually with 2 or more agents.

Go to the next page if you knew the correct answer, or click the link images
below to further research the concepts in this question (if desired).

Research Concepts:
Beta-Blockers:

Tap flag to report any problems with this question.


Question 1176: In patients with unstable angina (UA), which of the
following is recommended in those with contraindications to beta-blockade?

Choices:
1. Penicillin
2. Non-dihydropyridine calcium channel blocker
3. Dihydropyridine calcium channel blocker
4. NSAIDs
Answer: 2 - Non-dihydropyridine calcium channel blocker
Explanations:
A non-dihydropyridine calcium channel blocker (e.g. verapamil or
diltiazem) is next best choice when beta-blockade is contraindicated
Penicillin is an antibiotic and has no place in treatment of UA
NSAIDs also have no place in the treatment of UA

Go to the next page if you knew the correct answer, or click the link images
below to further research the concepts in this question (if desired).

Research Concepts:
Angina, Unstable:

Calcium Channel blockers:

Tap flag to report any problems with this question.


Question 1177: Select the congenital heart defect that can cause a right to
left shunt, often with cyanosis.

Choices:
1. Patent ductus arteriosus but no pulmonary hypertension
2. Atrial septal defect
3. Completely anomalous pulmonary venous return
4. Left coronary artery arising from the pulmonary trunk
Answer: 3 - Completely anomalous pulmonary venous return
Explanations:
Completely anomalous pulmonary venous return results in oxygenated
blood returning to the right atrium
There will be atrial septal defect with right to left shunt and probable
cyanosis
Atrial septal defect alone will result in a left to right shunt as will PDA
without hypertension
When the left coronary artery arises from the pulmonary trunk, drop in
pulmonary pressure causes reversal of flow. 20% of patients survive to
adulthood because of flow from the right coronary artery

Go to the next page if you knew the correct answer, or click the link images
below to further research the concepts in this question (if desired).

Research Concepts:
Totally Anomalous Pulmonary Venous Return:

Tap flag to report any problems with this question.


Question 1178: A patient on IV vancomycin may experience?
Choices:
1. Red man syndrome
2. Hypertension
3. Dizziness
4. Myalgia
Answer: 1 - Red man syndrome
Explanations:
Vancomycin toxicity includes hearing loss, renal toxicity, and allergic
reaction

Go to the next page if you knew the correct answer, or click the link images
below to further research the concepts in this question (if desired).

Research Concepts:
Vancomycin:

Tap flag to report any problems with this question.


Question 1179: What is the most sensitive, non-invasive test for
myocardial ischemia?

Choices:
1. Dipyridamole/Thallium
2. Treadmill
3. Exercise Echo
4. Holter monitor
Answer: 3 - Exercise Echo
Explanations:
Graded exercise stress testing can be performed alone or in combination
with echo or myocardial perfusion tests. Exercise test alone has lower
sensitivity and specificity but it is the cheapest
Stress echo can be used to assess segmental wall motion during exercise.
Ischemic segments become hypokinetic or akinetic during exercise
Thallium is widely used to assess myocardial perfusion. The presence of
increased lung uptake with thallium indicates poor prognosis and suggests
triple vessel or left main disease
In recent years, coronary artery calcium measurements by CT scan have
become popular. One should use caution with the data obtained as most
information is from elective outpatients who are asymptomatic

Go to the next page if you knew the correct answer, or click the link images
below to further research the concepts in this question (if desired).

Research Concepts:
Coronary Artery Disease:

Tap flag to report any problems with this question.


Question 1180: What is the most common presentation of a post transplant
lymphoma?

Choices:
1. Fever
2. Weight loss
3. Cervical nodes
4. Anorexia
Answer: 3 - Cervical nodes
Explanations:
As in primary lymphomas, the most common presentation is that of
adenopathy.
Extra nodal tumors are more common than nodal tumors.
Occasionally lymphoma may develop in the transplanted graft.

Go to the next page if you knew the correct answer, or click the link images
below to further research the concepts in this question (if desired).

Research Concepts:
Posttransplant Lymphoproliferative Disease:

Tap flag to report any problems with this question.


Question 1181: Which is false about symptoms of aortic stenosis?
Choices:
1. With onset of symptoms, survival is decreased
2. Asymptomatic individuals with critical stenosis have a high death rate
3. Dyspnea is the most common initial complaint
4. Triad of symptoms in aortic stenosis include angina, syncope, and
congestive heart failure
Answer: 2 - Asymptomatic individuals with critical stenosis have a high
death rate

Explanations:
The triad of aortic stenosis symptoms includes angina, syncope, and
dyspnea.
Once symptoms develop, the prognosis is worse. Congestive heart failure
carries a grave prognosis and only 60% of individuals are alive at 2 years.
When patients with aortic stenosis develop syncope, the 3-year mortality is
50%.
Angina in the presence of aortic stenosis is associated with 5 year 50%
mortality rate.

Go to the next page if you knew the correct answer, or click the link images
below to further research the concepts in this question (if desired).

Research Concepts:
Aortic Stenosis:

Tap flag to report any problems with this question.


Question 1182: Which is not an adverse effect of nitroglycerin?
Choices:
1. Headache
2. Hypotension
3. Diarrhea
4. Dizziness
Answer: 3 - Diarrhea
Explanations:
Because of its widespread vasodilatory effects, nitroglycerin often produces
such adverse effects as headache, hypotension, and dizziness.

Go to the next page if you knew the correct answer, or click the link images
below to further research the concepts in this question (if desired).

Research Concepts:
Nitroglycerin:

Tap flag to report any problems with this question.


Question 1183: A patient is 7 years post aortic valve replacement. A false
aneurysm has resulted. It measures 6 cm in size and is stuck to the posterior
sternum. What is the best approach to this patient?

Choices:
1. Stent placed via the groin
2. Femoral cannulation and circulatory arrest
3. Median sternotomy and rapid cannulation
4. Antibiotics for 6 weeks followed by surgery
Answer: 2 - Femoral cannulation and circulatory arrest
Explanations:
This patient has a false aneurysm, which is 6 cms.
The infected graft must be replaced or it will rupture. When replacing the
graft, the status of the AVR must be known, since it may also be infected.
The best approach to such a patient is to proceed with femoral cannulation,
arrest the patient prior to opening the chest.
A median sternotomy without femoral access is too dangerous and risks
lacerating the aorta.

Go to the next page if you knew the correct answer, or click the link images
below to further research the concepts in this question (if desired).

Research Concepts:
False Aneurysm:

Aortic Valve:

Tap flag to report any problems with this question.


Question 1184: From where do pulmonary emboli usually originate?
Choices:
1. The renal veins
2. The pelvic veins
3. The deep veins in the lower extremities
4. The femoral artery
Answer: 3 - The deep veins in the lower extremities
Explanations:
Nearly 90% of pulmonary emboli have their primary source as the deep
veins in the legs.
Risk factors for DVTs include immobility, recent orthopedic surgery, acute
infectious disease, age greater than 75 years, cancer, and hypercoagulable
states.
Patients with pulmonary embolism can present with chest pain, shortness of
breath, hypotension, or cardiac arrest.

Go to the next page if you knew the correct answer, or click the link images
below to further research the concepts in this question (if desired).

Research Concepts:
Pulmonary Embolism:

Pulmonary Embolism:

Tap flag to report any problems with this question.


Question 1185: What antibiotic causes histamine release leading to Red
Man Syndrome?

Choices:
1. Vancomycin
2. Sulfonamide
3. Rifampin
4. Tetracycline
Answer: 1 - Vancomycin
Explanations:
IV vancomycin can produce a histamine-mediated erythematous flushing of
the face, neck and trunk with possibility of associated hypotension.
Vancomycin causes two hypersensitivities reactions, Red Man syndrome
and anaphylaxis.
Red Man syndrome is associated with rapid infusion thus slow IV
administration will minimize adverse effects.
Stopping the IV and giving diphenhydramine will abort most reactions.

Go to the next page if you knew the correct answer, or click the link images
below to further research the concepts in this question (if desired).

Research Concepts:
Red Man Syndrome:

Vancomycin:

Tap flag to report any problems with this question.


Question 1186: Which of the following is not a feature of pericardial
cysts?

Choices:
1. Majority occur in the right cardiophrenic angle
2. They communicate with the pericardial cavity
3. They do not require excision
4. They can be aspirated
Answer: 2 - They communicate with the pericardial cavity
Explanations:
Pericardial cysts do not communicate with the pericardial space between
visceral and parietal pericardium. They are asymptomatic.
They present as a unilocular mass. An occasional patient may present with
chest heaviness.
Diagnosis is usually evident on a chest radiograph and chest CT. Majority
are located in the right cardiophrenic angle.
VATS is fast becoming an effective technique for removal of these cysts. If
the diagnosis is not certain, advanced imaging should be utilized.

Go to the next page if you knew the correct answer, or click the link images
below to further research the concepts in this question (if desired).

Research Concepts:
Pericardial Cyst:

Tap flag to report any problems with this question.


Question 1187: Which of the following is false about general anesthesia?
Choices:
1. Rapid induction
2. Low cardiovascular instability
3. Secures the airway
4. Decreases the risk of aspiration
Answer: 4 - Decreases the risk of aspiration
Explanations:
General anesthesia provides rapid induction and a secured airway.
There is an increased risk of aspiration.

Go to the next page if you knew the correct answer, or click the link images
below to further research the concepts in this question (if desired).

Research Concepts:
General Anesthesia:

Tap flag to report any problems with this question.


Question 1188: All of the following are absolute contraindications to
thrombolytic therapy use in ST-segment elevation myocardial infarction
(STEMI) EXCEPT:

Choices:
1. Prior intracranial hemorrhage (ICH)
2. Suspected aortic dissection
3. Ischemic stroke within 9 months
4. Active bleeding
Answer: 3 - Ischemic stroke within 9 months
Explanations:
AMI is a leading cause of morbidity and mortality in the United States
Ischemic stroke within 3 months is an absolute contraindication to
thrombolytic therapy for STEMI
For STEMI, door-to-drug time should be no longer than 30 minutes
Thrombolytic therapy is not indicated in the treatment of non-ST-segment
elevation MI (NSTEMI)

Go to the next page if you knew the correct answer, or click the link images
below to further research the concepts in this question (if desired).

Research Concepts:
Thrombolysis:

Acute ST-elevation Myocardial Infarction (STEMI):

Tap flag to report any problems with this question.


Question 1189: Which of the following statements about calcium
metabolism and therapy is incorrect:

Choices:
1. Calcium is approximately 40% albumin bound
2. The calcium level will fall approximately 0.8 mg/dL for each gram/dL
decrease in serum albumin
3. The goal of calcium replacement is to maintain the level between 8 and 9
mg/dL
4. Calcium is approximately 80% albumin bound
Answer: 4 - Calcium is approximately 80% albumin bound
Explanations:
Calcium is approximately 40% albumin bound.
Calcium levels must be interpreted with albumin levels taken into account.
Total and ionized calcium need to evaluated.

Go to the next page if you knew the correct answer, or click the link images
below to further research the concepts in this question (if desired).

Research Concepts:
Calcium:

Tap flag to report any problems with this question.


Question 1190: What is the most common cause of intraoperative
thrombocytopenia?

Choices:
1. Malignancy
2. Dilutional
3. Long surgery time
4. Adverse drug reaction
Answer: 2 - Dilutional
Explanations:
Dilutional thrombocytopenia is the most common cause of intraoperative
thrombocytopenia.

Go to the next page if you knew the correct answer, or click the link images
below to further research the concepts in this question (if desired).

Research Concepts:
Thrombocytopenia:

Tap flag to report any problems with this question.


Question 1191: During recovery from myocardial infarction, which of the
following explains why patients commonly develop infarcts in other organs?

Choices:
1. Hypercoagulability state
2. Embolization of mural thrombi
3. Widespread atherosclerosis
4. Generalized vasoconstriction
Answer: 2 - Embolization of mural thrombi
Explanations:
Myocardial infarction may cause dyskinesis of an area of myocardium,
leading to mural thrombi, clots by the wall.
The most common cause of mural thrombi is anterior wall myocardial
infarction.
The thrombus can embolize causing infarcts anywhere in the body.

Go to the next page if you knew the correct answer, or click the link images
below to further research the concepts in this question (if desired).

Research Concepts:
Myocardial Infarction, Acute:

Tap flag to report any problems with this question.


Question 1192: A patient is shot in the chest and is admitted to the
intensive care unite for invasive monitoring. It is decided to place a Swan Ganz
catheter and measure the pulmonary catheter wedge pressure. This will allow
one to assess the pressure in which part of the heart?

Choices:
1. Right atrium
2. Left atrium
3. Left ventricle
4. Right ventricle
Answer: 2 - Left atrium
Explanations:
Pulmonary capillary wedge pressure is approximately the same as left atrial
pressure.
Left-sided cardiac catheterization can measure pressure in the left ventricle
but not in the left atrium.

Go to the next page if you knew the correct answer, or click the link images
below to further research the concepts in this question (if desired).

Research Concepts:
Swan-Ganz Catheterization:

Tap flag to report any problems with this question.


Question 1193: A patient is instructed about the digoxin he is prescribed
for atrial fibrillation. Which teaching point is least crucial that he understands
which of the following?

Choices:
1. How to take digoxin
2. How to monitor his pulse rate
3. The common side effects of digoxin
4. The etiology of his atrial fibrillation
Answer: 4 - The etiology of his atrial fibrillation
Explanations:
Although the etiology of the patient's atrial fibrillation may be explained, it
is more important that he understands the other issues
Digoxin needs to be taken as directed to avoid toxicity
Side effects such as bradycardia, hypotension, and arrhythmias are possible

Go to the next page if you knew the correct answer, or click the link images
below to further research the concepts in this question (if desired).

Research Concepts:
Atrial Fibrillation:

Digoxin:

Tap flag to report any problems with this question.


Question 1194: Which is false of Staphylococcus infections?
Choices:
1. Can cause cellulitis
2. Can cause toxic shock syndrome
3. Usually associated with a fever
4. Often release an endotoxin
Answer: 4 - Often release an endotoxin
Explanations:
Staphylococcus is a common cause of cellulitis.
S. aureus sometimes causes the toxic shock syndrome that can be fatal if
not promptly treated.
S. aureus is a Gram-positive organism that does not release endotoxins but
rather exotoxins.
Most staphylococcus infections are associated with a fever.

Go to the next page if you knew the correct answer, or click the link images
below to further research the concepts in this question (if desired).

Research Concepts:
Staphylococcal Infections:

Tap flag to report any problems with this question.


Question 1195: Which measure reduces the risk of perioperative MI
during vascular surgery in those with cardiac ischemia?

Choices:
1. Calculation of cardiac output
2. Angiotensin converting enzyme (ACE) inhibitors
3. Sublingual nitroglycerin
4. Beta-blockers
Answer: 4 - Beta-blockers
Explanations:
Perioperative beta-blockers have been shown to decrease rates of MI and
cardiac death in patients with cardiac ischemia undergoing vascular surgery
Beta-blockers can reduce 30-day mortality by as much as 50%
They can also reduce 1-year mortality by as much as 50%

Go to the next page if you knew the correct answer, or click the link images
below to further research the concepts in this question (if desired).

Research Concepts:
Coronary Artery Disease:

Vascular Surgery:

Preoperative Evaluation And Management:


Tap flag to report any problems with this question.
Question 1196: Which of the following is NOT an emergent treatment of
acute MI?

Choices:
1. Intranasal oxygen
2. Chewable aspirin
3. Cholesterol measurement
4. Atenolol
Answer: 3 - Cholesterol measurement
Explanations:
Lipid profiles drawn during the acute event do not alter treatment
Intranasal oxygen, beta-blockers, and chewable aspirin are measures that
are needed urgently
Immediate actions are aimed to reduce the degree of ischemia

Go to the next page if you knew the correct answer, or click the link images
below to further research the concepts in this question (if desired).

Research Concepts:
Acute Myocardial Infarction:

Tap flag to report any problems with this question.


Question 1197: Which is NOT a treatment for hyperkalemia in a patient
with heart failure?

Choices:
1. Insulin
2. Albuterol
3. Calcium
4. Magnesium
Answer: 4 - Magnesium
Explanations:
Calcium gluconate is the drug of choice in hyperkalemia with ECG
changes.

Go to the next page if you knew the correct answer, or click the link images
below to further research the concepts in this question (if desired).

Research Concepts:
Heart Failure, Congestive:

Hyperkalemia:

Tap flag to report any problems with this question.


Question 1198: In a patient with chronic renal failure, which feature is
unlikely?

Choices:
1. Hypokalemia
2. Uremia
3. Platelet dysfunction
4. Metabolic acidosis
Answer: 1 - Hypokalemia
Explanations:
Decreased renal function leads hyperkalemia, metabolic acidosis.
Uremia inhibits platelet aggregation.

Go to the next page if you knew the correct answer, or click the link images
below to further research the concepts in this question (if desired).

Research Concepts:
Renal Failure, Chronic:

Tap flag to report any problems with this question.


Question 1199: A patient is seen in the clinic complaining of dyspnea and
general malaise. He says that over the past 12 months he has not been feeling
well. Examination of his fingers reveals alternating blushing and blanching of
the fingernails following compression. He most likely has:

Choices:
1. Psoriasis
2. Aortic regurgitation
3. Onychomycosis
4. Lead poisoning
Answer: 2 - Aortic regurgitation
Explanations:
Quincke sign is to detect aortic regurgitation (AR). There is alternating
blanching and blushing of the fingernail bed with light compression.
Corrigan sign is also seen in AR. It is pulsation of the carotid arteries.
Hill sign is systolic pressure greater than or equal to 60 mm Hg at the
popliteal artery.
Large volume pulse, another sign of AR, is called a water-hammer pulse.

Go to the next page if you knew the correct answer, or click the link images
below to further research the concepts in this question (if desired).

Research Concepts:
Aortic Regurgitation:

Quincke Sign:

Tap flag to report any problems with this question.


Question 1200: Which of the following increases the risk of postoperative
atelectasis?

Choices:
1. History of smoking and immobility
2. Ischemic cardiomyopathy
3. Asthenia
4. Orthopedic procedures
Answer: 1 - History of smoking and immobility
Explanations:
Risk of postoperative atelectasis is increased by immobility, history of
smoking, and procedures on the abdomen or thorax.
Other risk factors for post-operative atelectasis include obesity, muscular
weakness, and abdominal distension.
Further additional risk factors include a prior history of asthma or
respiratory infections.
Pregnancy is also a known risk factor for developing post-op atelectasis.

Go to the next page if you knew the correct answer, or click the link images
below to further research the concepts in this question (if desired).

Research Concepts:
Atelectasis:

Tap flag to report any problems with this question.


Section 7
Question 1201: Unilaterally decreased breath sounds and asymmetrical
chest movement is associated with which of the following?

Choices:
1. Cardiac tamponade
2. Pulmonary hypertension
3. Pneumothorax
4. Pulmonary fibrosis
Answer: 3 - Pneumothorax
Explanations:
Pneumothorax is associated with decreased breath sounds unilaterally and
asymmetrical chest movement.

Go to the next page if you knew the correct answer, or click the link images
below to further research the concepts in this question (if desired).

Research Concepts:
Pneumothorax:

Tap flag to report any problems with this question.


Question 1202: Who is best able to assess the need for pain medication?
Choices:
1. Nursing staff
2. Physician
3. Patient
4. Pain consultant
Answer: 3 - Patient
Explanations:
The adequacy of pain medication is based on patient self-assessment
The most efficient self-assessment is with numeric rating system

Go to the next page if you knew the correct answer, or click the link images
below to further research the concepts in this question (if desired).

Research Concepts:
Pain Assessment:

Tap flag to report any problems with this question.


Question 1203: Which of the following drugs would be inappropriate for a
patient after major surgery who takes opioids on a chronic basis?

Choices:
1. Morphine
2. Nalbuphine
3. Hydromorphone
4. Meperidine
Answer: 2 - Nalbuphine
Explanations:
Nalbuphine is a mixed opioid agonist-antagonist.
It is the only one available in the United States.
It may cause withdrawal symptoms in patients chronically receiving
opioids.
It may be effective for morphine-induced pruritus.

Go to the next page if you knew the correct answer, or click the link images
below to further research the concepts in this question (if desired).

Research Concepts:
Opioids:

Tap flag to report any problems with this question.


Question 1204: Which of the following drugs has both alpha and beta
sympathetic blocking activity?

Choices:
1. Methyldopa
2. Clonidine
3. Acebutolol
4. Labetalol
Answer: 4 - Labetalol
Explanations:
Labetalol has both alpha and beta blocking activity.
Labetalol is used to treat hypertension.
High doses can cause atrioventricular block and hypotension.

Go to the next page if you knew the correct answer, or click the link images
below to further research the concepts in this question (if desired).

Research Concepts:
Beta-Blockers:

Alpha-Blockers:

Tap flag to report any problems with this question.


Question 1205: What is the primary rationale for administering insulin via
an intravenous infusion in intensive care unit (ICU) patients?

Choices:
1. ICU patients usually have lower insulin requirements than non-ICU patients
2. ICU patients have lower pain tolerance to subcutaneous injections than non-
ICU patients
3. ICU patients often have unpredictable absorption of drugs administered
subcutaneously
4. ICU nurses are more familiar with administering drugs by intravenous
infusion than by subcutaneous injection
Answer: 3 - ICU patients often have unpredictable absorption of drugs
administered subcutaneously

Explanations:
ICU patients often have impaired peripheral perfusion and/or peripheral
edema, both of which can cause unpredictable absorption of subcutaneous
drugs
Therefore, it is generally more reliable to administer medications (including
insulin) by the intravenous than subcutaneous route

Go to the next page if you knew the correct answer, or click the link images
below to further research the concepts in this question (if desired).

Research Concepts:
Subcutaneous Insulin:

Tap flag to report any problems with this question.


Question 1206: Frank-Starling law involves:
Choices:
1. Afterload dependence of cardiac contractility
2. Preload dependence of cardiac contractility
3. Right ventricular ejection
4. Right atrial ejection
Answer: 2 - Preload dependence of cardiac contractility
Explanations:
Frank-Starling law reflects preload dependence of cardiac contractility

Go to the next page if you knew the correct answer, or click the link images
below to further research the concepts in this question (if desired).

Research Concepts:
Frank–Starling Law:

Tap flag to report any problems with this question.


Question 1207: All of the following are appropriate managements of
digoxin toxicity, except:

Choices:
1. Potassium administration
2. Calcium administration
3. Lidocaine administration
4. Fab fragment antibodies
Answer: 2 - Calcium administration
Explanations:
Hypercalcemia can trigger digitalis toxicity
Calcium administration can worsen digitalis toxicity
Potassium and magnesium supplementation are essential measures in
digitalis toxicity
Lidocaine can improve the arrhythmia and Fab fragments neutralize excess
digoxin in the serum

Go to the next page if you knew the correct answer, or click the link images
below to further research the concepts in this question (if desired).

Research Concepts:
Digoxin Toxicity:

Tap flag to report any problems with this question.


Question 1208: PEEP is known to decrease:
Choices:
1. Stroke volume
2. Contractility
3. Afterload
4. Preload
Answer: 4 - Preload
Explanations:
PEEP decreases preload.

Go to the next page if you knew the correct answer, or click the link images
below to further research the concepts in this question (if desired).

Research Concepts:
Positive End-Expiratory Pressure (PEEP):

Tap flag to report any problems with this question.


Question 1209: What medication can be used to raise blood pressure in a
patient when dopamine has been ineffective?

Choices:
1. Dobutamine
2. Nitroglycerin
3. IVIG
4. Norepinephrine
Answer: 4 - Norepinephrine
Explanations:
Vasopressor norepinephrine is used if dopamine has been ineffective

Go to the next page if you knew the correct answer, or click the link images
below to further research the concepts in this question (if desired).

Research Concepts:
Norepinephrine:

Tap flag to report any problems with this question.


Question 1210: Which of the following statements is TRUE regarding
diabetics taking metformin who require iodinated contrast?

Choices:
1. Should not receive it; iodinated contrast is absolutely contraindicated in
patients on Metformin
2. Should have Metformin withheld for 7 days after contrast is administered
3. Are at risk for hepatic failure
4. Are at risk for lactic acidosis
Answer: 4 - Are at risk for lactic acidosis
Explanations:
Glucophage should be held for 48 hours after the iodinated contrast
administration.
It need not be withheld for any significant time prior to contrast
administration.
Toxicity risk is greatest in those diabetics who also have renal insufficiency.
Each radiology department or outpatient center administering IV iodinated
contrast should have a clear protocol as to who is responsible (radiologist,
ordering physician, primary care practitioner) for informing the patient
about the 2 day hiatus in metformin administration. If the dispensing
physician is concerned about the 2 day hiatus, a discussion of risks and
benefits (CIN vs non-contrast CT vs other imaging modalities) is in order.

Go to the next page if you knew the correct answer, or click the link images
below to further research the concepts in this question (if desired).

Research Concepts:
Iodinated Intravenous Contrast:

Metformin:
Tap flag to report any problems with this question.
Question 1211: In a patient with acute respiratory distress syndrome,
which treatment is unlikely to help?

Choices:
1. Steroids
2. Diuretics
3. Antibiotics
4. Oxygen
Answer: 1 - Steroids
Explanations:
ARDS is associated with pulmonary edema
Steroid is not indicated for ARDS
Supportive care is necessary
Steroids can reduce ventilator dependent days but has not been shown to
effect long term survival

Go to the next page if you knew the correct answer, or click the link images
below to further research the concepts in this question (if desired).

Research Concepts:
Acute Respiratory Distress Syndrome:

Tap flag to report any problems with this question.


Question 1212: Which technique is used for selective debridement of a
wound?

Choices:
1. Application of elastase
2. Wound irrigation
3. Wet-to-dry dressings
4. Sharp debridement
Answer: 4 - Sharp debridement
Explanations:
Sharp debridement involves using forceps, scalpel or scissors to selectively
remove necrotic or nonviable tissue
The other options all are nonselective debridement and can remove healthy
or healing tissue

Go to the next page if you knew the correct answer, or click the link images
below to further research the concepts in this question (if desired).

Research Concepts:
Debridement:

Tap flag to report any problems with this question.


Question 1213: A patient with a seizure disorder is ambulating with
assistance in the hall and has a tonic clonic seizure. Which of the following
would NOT be appropriate acute management?

Choices:
1. Lower the patient to the floor
2. Place the patient in the lateral decubitus position
3. Place a rubber bite block in the mouth to prevent tooth breakage
4. Put a pillow under the head
Answer: 3 - Place a rubber bite block in the mouth to prevent tooth breakage
Explanations:
A patient having a generalized seizure should be placed so as not to injure
themselves.
Lowering the patient to the floor, turning them to one side to clear oral
secretions, and placing a pillow under the head are all appropriate.
Nothing should be put into the patient's mouth. Efforts to do that may lead
to bites of the healthcare worker and choking or aspiration of the seizing
patient.
Nearby objects should be moved to avoid the patient hitting them
accidentally.

Go to the next page if you knew the correct answer, or click the link images
below to further research the concepts in this question (if desired).

Research Concepts:
Seizure, Generalized Tonic-clonic:

Tap flag to report any problems with this question.


Question 1214: Which of the following is not a contraindication for
succinylcholine?

Choices:
1. Narcotic overdose
2. Hyperkalemia
3. Acute crush injuries
4. Acute renal failure
Answer: 1 - Narcotic overdose
Explanations:
Succinylcholine is often used as a paralytic agent for endotracheal
intubation
It is contraindicated in patients that may have hyperkalemia as it may cause
further increase with fatal arrhythmias
Tumor lysis syndrome, rhabdomyolysis, muscular dystrophy are all
additional contraindications

Go to the next page if you knew the correct answer, or click the link images
below to further research the concepts in this question (if desired).

Research Concepts:
Succinylcholine:

Tap flag to report any problems with this question.


Question 1215: How does digoxin help patients with heart failure?
Choices:
1. Increases contractility
2. Decreases preload
3. Decreases afterload
4. Increases preload
Answer: 1 - Increases contractility
Explanations:
Digoxin affects sodium pump and increases cardiac contractility.

Go to the next page if you knew the correct answer, or click the link images
below to further research the concepts in this question (if desired).

Research Concepts:
Heart Failure, Congestive:

Digoxin:

Tap flag to report any problems with this question.


Question 1216: A 64-year-old has been taking steroids for asthma for 5
years. She undergoes urgent surgery for a perforated ulcer. 2 days later, she
becomes hypotensive, febrile, and hyperglycemic. What is her most likely
diagnosis?

Choices:
1. Adrenal insufficiency
2. Hypovolemia
3. Steroid induced diabetes
4. Diabetic ketoacidosis
Answer: 4 - Diabetic ketoacidosis
Explanations:
Diabetic ketoacidosis is a state of insulin deficiency characterized by
hyperglycemia, dehydration, and metabolic acidosis. As the disorder
progresses it can quickly lead to hypoglycemia, hyponatremia, and
hypokalemia.
Causes of diabetic ketoacidosis include myocardial infarction, trauma,
stress, cocaine, surgery, dental abscesses, and missed insulin treatments.
The absence of insulin means that glucose is not taken up by the cells.
Other counter-regulatory hormones enhance fat breakdown into fatty acids,
causing diabetic ketoacidosis. Beta-oxidation of free fatty acids leads to the
formation of ketone bodies.
The hyperglycemic-induced osmotic diuresis depletes sodium, potassium,
phosphate, and water. The serum potassium can drop significantly and this
electrolyte must be monitored.

Go to the next page if you knew the correct answer, or click the link images
below to further research the concepts in this question (if desired).

Research Concepts:
Perforated Peptic Ulcer:

Diabetic Ketoacidosis:
Tap flag to report any problems with this question.
Question 1217: Who is most qualified to explain a surgical procedure?
Choices:
1. Ward nurse
2. Surgeon
3. Anesthesia care provider
4. Scrub nurse
Answer: 2 - Surgeon
Explanations:
The surgeon is the only person in this list qualified to explain the surgical
procedure.
At teaching institutions, attending physicians may delegate this
responsibility to a physician resident or fellow.

Go to the next page if you knew the correct answer, or click the link images
below to further research the concepts in this question (if desired).

Research Concepts:
Informed Consent:

Surgery, Explaining the Procedure:

Tap flag to report any problems with this question.


Question 1218: A 36-week term infant presents with cyanosis shortly after
birth. His arterial oxygen saturation is only 30% on room air. What is the most
likely diagnosis?

Choices:
1. Lung hypoplasia
2. Ventricular septal defect
3. Atrial septal defect
4. Transposition of the great vessels
Answer: 4 - Transposition of the great vessels
Explanations:
Transposition of the great vessels causes a right-to-left shunt.
Patients are likely cyanotic.
The other options involve shunting of oxygenated blood to the pulmonary
circulation. They therefore do not result in early cyanosis.
Tetralogy of Fallot has a VSD combined with pulmonary artery atresia.
This results in decreased blood flow in the pulmonary system which leads
to cyanosis.

Go to the next page if you knew the correct answer, or click the link images
below to further research the concepts in this question (if desired).

Research Concepts:
Transposition Of The Great Arteries:

Tap flag to report any problems with this question.


Question 1219: Select the virus that most often causes acute viral
pericarditis.

Choices:
1. Rhinovirus
2. Epstein-Barr virus
3. Coxsackie virus
4. Adenovirus
Answer: 3 - Coxsackie virus
Explanations:
Acute viral pericarditis is most often caused by coxsackievirus B.
Influenza A and B are seasonal causes.
Other viruses that can cause this condition include echovirus, adenoviruses,
enterovirus, mumps virus, Epstein-Barr virus, and human
immunodeficiency virus (HIV).

Go to the next page if you knew the correct answer, or click the link images
below to further research the concepts in this question (if desired).

Research Concepts:
Pericarditis:

Tap flag to report any problems with this question.


Question 1220: Which of the following substances is not a pulmonary
artery vasodilator?

Choices:
1. Isoproterenol
2. Nitric oxide
3. Prostaglandin
4. Nitrous oxide
Answer: 4 - Nitrous oxide
Explanations:
Nitrous oxide is an anesthetic gas and not a pulmonary artery vasodilator.

Go to the next page if you knew the correct answer, or click the link images
below to further research the concepts in this question (if desired).

Research Concepts:
Cardiac Physiology:

Pulmonary Physiology:

Tap flag to report any problems with this question.


Question 1221: At what point does the precise use of a Swan Ganz
catheter require measurement of the wedge?

Choices:
1. End expiration
2. End inspiration
3. Mid inspiration
4. Onset of inspiration
Answer: 1 - End expiration
Explanations:
Wedge pressure should be measured at end expiration in a non-ventilated
patient.

Go to the next page if you knew the correct answer, or click the link images
below to further research the concepts in this question (if desired).

Research Concepts:
Swan-Ganz Catheterization:

Tap flag to report any problems with this question.


Question 1222: Patients in respiratory distress should not undergo which
of the following?

Choices:
1. Sedation
2. Nasotracheal suctioning
3. Head elevation
4. Bronchodilator therapy
Answer: 1 - Sedation
Explanations:
Sedation is usually avoided to prevent hypoventilation

Go to the next page if you knew the correct answer, or click the link images
below to further research the concepts in this question (if desired).

Research Concepts:
Sedation:

Respiratory Distress/Failure:

Tap flag to report any problems with this question.


Question 1223: A 60-year-old obese man is admitted to the hospital with
substernal chest pain and diaphoresis. Blood work at 6 hrs will show elevation of
which of the following enzymes?

Choices:
1. Aspartate transaminase
2. Alkaline phosphatase
3. Creatine kinase
4. Lactate dehydrogenase
Answer: 3 - Creatine kinase
Explanations:
Elevation in creatine kinase is one of the criteria used in diagnosing an
myocardial infarction.
The peak elevation of creatine kinase occurs within 24 hrs and subsides
over the next 24-48 hrs.
CK(MB) levels usually revert back to normal in 2-3 days.
If a re-infarct occurs, then elevation in CK(MB) levels will again appear.

Go to the next page if you knew the correct answer, or click the link images
below to further research the concepts in this question (if desired).

Research Concepts:
Myocardial Infarction, Acute:

Tap flag to report any problems with this question.


Question 1224: What disease process does not benefit from chest
percussion therapy?

Choices:
1. Pneumonia
2. Pulmonary hypertension
3. Atelectasis
4. Bronchiectasis
Answer: 2 - Pulmonary hypertension
Explanations:
Pulmonary hypertension does not benefit from chest percussion therapy

Go to the next page if you knew the correct answer, or click the link images
below to further research the concepts in this question (if desired).

Research Concepts:
Chest Physical Therapy:

Pulmonary Hypertension:

Tap flag to report any problems with this question.


Question 1225: Constipation is a common side effect of:
Choices:
1. Pravastatin
2. Lisinopril
3. Nitroglycerin
4. Verapamil
Answer: 4 - Verapamil
Explanations:
Verapamil is a calcium channel blocker that can cause constipation
Lisinopril can cause persistent dry cough
Nitroglycerin can cause rebound tachycardia and headaches
Pravastatin may cause myopathy

Go to the next page if you knew the correct answer, or click the link images
below to further research the concepts in this question (if desired).

Research Concepts:
Verapamil:

Tap flag to report any problems with this question.


Question 1226: A 56 year old male with a history COPD on home oxygen,
hypertension, and hyperlipidemia is brought to the emergency room with several
episodes of chest pain each lasting about 30 minutes. His current medications
include hydrochlorothiazide, home oxygen, lovastatin, albuterol, and aspirin.
The pain is resolved after nitroglycerin sublingually twice and intravenous
heparin. ECG shows 1 mm of ST depression in inferior leads. Troponin is
negative. Nuclear medicine stress test shows reversible ischemia. Select the fact
that is most indicative that the patient may have an MI, urgent revascularization,
or sudden death in the next 2 weeks.

Choices:
1. Beta agonist
2. Recent aspirin
3. Hyperlipidemia
4. Abnormal stress test
Answer: 2 - Recent aspirin
Explanations:
The TIMI score identifies patient that would benefit from early intervention
Risk factors include age greater than 65, prior stenosis greater than 50%, 3
or more cardiac risk factors, angina 2 times in less than 24 hours, ST
segment deviation greater than 0.5 mm, 3 or more cardiac risk factors,
elevated cardiac enzymes, and aspirin usage within 7 days
The fact that the patient had unstable angina on aspirin make is possible
that he is aspirin resistant
This resistance can occur in 5 to 10% of the population

Go to the next page if you knew the correct answer, or click the link images
below to further research the concepts in this question (if desired).

Research Concepts:
Angina, Unstable:

Aspirin Resistance:

Tap flag to report any problems with this question.


Question 1227: What is the mechanism of action of tacrolimus?
Choices:
1. Inhibits B-lymphocyte activation
2. Inhibits T-lymphocyte activation
3. Depresses the bone marrow
4. Causes reduced chemotaxis
Answer: 2 - Inhibits T-lymphocyte activation
Explanations:
Tacrolimus works by inhibiting T-lymphocyte activation.
It is used intravenously or orally to prevent transplant rejection and
topically for atopic dermatitis.
It binds to FKBP-12, an intracellular protein and inhibits calcineurin
phosphatase activity.
It increases risk of infections and lymphoma.

Go to the next page if you knew the correct answer, or click the link images
below to further research the concepts in this question (if desired).

Research Concepts:
Tacrolimus:

Tap flag to report any problems with this question.


Question 1228: Which medication is most commonly co-administered
with tissue plasminogen activator (t-PA)?

Choices:
1. Heparin
2. Morphine
3. Propranolol
4. Hydralazine
Answer: 1 - Heparin
Explanations:
Heparin or a similar anticoagulant is generally co-administered with t-PA.
This is done to prevent re-occlusion of the artery after successful clot lysis
by t-PA.
Adjunctive anticoagulation is continued for 2 to 8 days after t-PA therapy.

Go to the next page if you knew the correct answer, or click the link images
below to further research the concepts in this question (if desired).

Research Concepts:
Tissue Plasminogen Activator:

Tap flag to report any problems with this question.


Question 1229: How should progression be followed in a patient with a
pneumothorax?

Choices:
1. CXR
2. CT
3. Ultra sound
4. Clinical
Answer: 1 - CXR
Explanations:
A simple chest radiograph can help to rule out pneumothorax.
CXR is the best test to follow the progression of pneumothorax.

Go to the next page if you knew the correct answer, or click the link images
below to further research the concepts in this question (if desired).

Research Concepts:
Pneumothorax:

Tap flag to report any problems with this question.


Question 1230: After an intramuscular fentanyl injection, pain relief
occurs in what time period?

Choices:
1. Five minutes
2. Ten minutes
3. Twenty minutes
4. Thirty minutes
Answer: 2 - Ten minutes
Explanations:
Fentanyl has a fast onset time
Roughly ten minutes after an IM injection, most people find pain relief

Go to the next page if you knew the correct answer, or click the link images
below to further research the concepts in this question (if desired).

Research Concepts:
Fentanyl:

Tap flag to report any problems with this question.


Question 1231: Furosemide works at the:
Choices:
1. Proximal tubule
2. Distal tubule
3. Loop of Henle
4. Collecting duct
Answer: 3 - Loop of Henle
Explanations:
Furosemide works at the loop of Henle
Ethacrynic acid also works at the loop of Henle
They are potent diuretics

Go to the next page if you knew the correct answer, or click the link images
below to further research the concepts in this question (if desired).

Research Concepts:
Furosemide:

Tap flag to report any problems with this question.


Question 1232: Morphine has found use in clinical medicine in all the
below conditions except:

Choices:
1. Induction of labor
2. Cough suppression
3. Treatment of diarrhea
4. Pain management
Answer: 1 - Induction of labor
Explanations:
Morphine is not used in the treatment of labor
Morphine causes a reduction in uterine tone and cause a prolongation of
labor

Go to the next page if you knew the correct answer, or click the link images
below to further research the concepts in this question (if desired).

Research Concepts:
Morphine:

Tap flag to report any problems with this question.


Question 1233: Which of the following causes post-transplant
lymphoproliferative malignancy (PTLD)?

Choices:
1. Epstein-Barr virus (EBV)
2. Cytomegalovirus (CMV)
3. Human papillomavirus (HPV)
4. Human immunodeficiency virus (HIV)
Answer: 1 - Epstein-Barr virus (EBV)
Explanations:
Post-transplant lymphoma is caused by latent Epstein-Barr virus infections.
In addition to Epstein-Barr virus, cytomegalovirus has also been implicated
in post-transplant lymphoproliferative malignancy.
The most common presentation is a lymphoid swelling, usually of the
cervical or groin lymph nodes.
Reduction in immunosuppression is the first intervention. Anti CD 20
monoclonal antibody has shown some promise.

Go to the next page if you knew the correct answer, or click the link images
below to further research the concepts in this question (if desired).

Research Concepts:
Posttransplant Lymphoproliferative Disease:

Tap flag to report any problems with this question.


Question 1234: Which of the following is not a frequent symptom/sign of
left ventricular aneurysm?

Choices:
1. Ventricular arrhythmias
2. Thromboembolism
3. Congestive heart failure
4. Mitral regurgitation
Answer: 4 - Mitral regurgitation
Explanations:
Mitral regurgitation is rare with left ventricular aneurysm

Go to the next page if you knew the correct answer, or click the link images
below to further research the concepts in this question (if desired).

Research Concepts:
Ventricular Aneurysms:

Tap flag to report any problems with this question.


Question 1235: Colestipol (Colestid), an antilipemic, acts by binding with
bile acids. In addition to lowering cholesterol, there is a risk of decreasing the
level of which of the following?

Choices:
1. B complex vitamins
2. Fat-soluble vitamins
3. Serum potassium
4. Albumin
Answer: 2 - Fat-soluble vitamins
Explanations:
The fat-soluble vitamins A, D, E, and K may also be excreted along with
bile acids during antilipemic therapy. B complex vitamins and the other
water-soluble vitamins are affected very little by colestipol (Colestid).
Serum potassium is affected little by colestipol (Colestid).
Albumin levels are affected very little by colestipol (Colestid).
Patient Need: Physiological Integrity

Go to the next page if you knew the correct answer, or click the link images
below to further research the concepts in this question (if desired).

Research Concepts:
Bile Acid Sequestrant:

Tap flag to report any problems with this question.


Question 1236: With which of the following does the fusion of the cusps
occur in a bicuspid aortic valve?

Choices:
1. Right and left cusps
2. Right and non-coronary cusps
3. Left and non-coronary cusps
4. Left coronary and right coronary arteries
Answer: 1 - Right and left cusps
Explanations:
The fusion of the bicuspid valve is along the right and left coronary cusps.

Go to the next page if you knew the correct answer, or click the link images
below to further research the concepts in this question (if desired).

Research Concepts:
Aortic Valve:

Tap flag to report any problems with this question.


Question 1237: Which of the following drugs is used to treat ischemic
ventricular arrhythmias?

Choices:
1. Captopril
2. Propranolol
3. Lidocaine
4. Labetalol
Answer: 3 - Lidocaine
Explanations:
Lidocaine is an amide local anesthetic.
Lidocaine is considered to be a highly selective, group 1B anti-arrhythmic
agent.
Lidocaine is used for nerve blocks and post-myocardial infarction ischemic
ventricular arrhythmias.
High doses of lidocaine can result in central nervous system excitation.

Go to the next page if you knew the correct answer, or click the link images
below to further research the concepts in this question (if desired).

Research Concepts:
Arrhythmias:

Tap flag to report any problems with this question.


Question 1238: Which metabolic disorder is often associated with
hypokalemia?

Choices:
1. Acidosis
2. Alkalosis
3. Hypermagnesemia
4. Hyponatremia
Answer: 2 - Alkalosis
Explanations:
Potassium ions are exchanged with hydrogen (H+) ions in the kidneys.
Therefore, hypokalemia leads to increased H+ excretion.
Hypokalemia often leads to metabolic alkalosis and vice versa.
Hypomagnesemia also commonly accompanies hypokalemia.

Go to the next page if you knew the correct answer, or click the link images
below to further research the concepts in this question (if desired).

Research Concepts:
Metabolic Alkalosis:

Hypokalemia:

Tap flag to report any problems with this question.


Question 1239: Which is a requirement for a healthcare provider acting as
an expert witness?

Choices:
1. Have held their position for at least 5 years
2. Provide their testimony free of charge
3. Discuss their testimony with the claimant
4. Review all pertinent information and records
Answer: 4 - Review all pertinent information and records
Explanations:
Expert witnesses provide opinions on information they have reviewed about
the claimants case
They do not discuss the case with the claimant and in some instances are
paid for their time

Go to the next page if you knew the correct answer, or click the link images
below to further research the concepts in this question (if desired).

Research Concepts:
Expert Witness:

Tap flag to report any problems with this question.


Question 1240: Which of the following is FALSE about beta blockers?
Choices:
1. Metoprolol is useful in the treatment of patients after an MI
2. Metoprolol can depress the cardiac output
3. Beta blockers reduce arrhythmias
4. Oral Esmolol is effective for outpatient treatment of hypertension
Answer: 4 - Oral Esmolol is effective for outpatient treatment of
hypertension

Explanations:
Esmolol is a very short acting beta blocker for intravenous administration
of atrial arrhythmias and hypertension.
The majority of beta blockers are commonly used as prophylactic agents in
patients who have had an MI.
These agents can also decrease atrial arrhythmias.
Use should be cautioned in patients with poor ejection fraction as these
agents are negatively inotropic.

Go to the next page if you knew the correct answer, or click the link images
below to further research the concepts in this question (if desired).

Research Concepts:
Beta-Blockers:

Tap flag to report any problems with this question.


Question 1241: Which of the following medications can cause histamine
release?

Choices:
1. Dantrolene
2. Vecuronium
3. Morphine
4. Baclofen
Answer: 3 - Morphine
Explanations:
Morphine can cause histamine release resulting in pruritis
Tolerance develops to this side effect
Dantrolene blocks Ca2+ at the sarcoplasmic reticulum
Baclofen is a centrally acting muscle relaxant

Go to the next page if you knew the correct answer, or click the link images
below to further research the concepts in this question (if desired).

Research Concepts:
Histamine:

Morphine:

Tap flag to report any problems with this question.


Question 1242: Where are atherosclerotic aneurysms most often located?
Choices:
1. Coronary arteries
2. Ascending aorta
3. Thoracic aorta
4. Abdominal aorta
Answer: 4 - Abdominal aorta
Explanations:
About 90-95% of aortic aneurysms involve the infrarenal abdominal aorta.
Rarely do abdominal aortic aneurysm extend above the renal arteries.
However, abdominal aortic aneurysm do extend into the common iliac
arteries.
The majority of abdominal aortic aneurysms are asymptomatic.

Go to the next page if you knew the correct answer, or click the link images
below to further research the concepts in this question (if desired).

Research Concepts:
Abdominal Aortic Aneurysm:

Tap flag to report any problems with this question.


Question 1243: Which vessel(s) are accompanied by the phrenic nerve and
descends between the mediastinal pleura and pericardium?

Choices:
1. Pericardiophrenic vessels
2. Internal mammary
3. Coronary arteries
4. Musculophrenic arteries
Answer: 1 - Pericardiophrenic vessels
Explanations:
The pericardiophrenic artery arises from the internal thoracic artery.
It travels with the phrenic nerve between the mediastinal pleura and
pericardium.
It supplies the diaphragm and anastomoses with the inferior phrenic and
musculophrenic arteries.

Go to the next page if you knew the correct answer, or click the link images
below to further research the concepts in this question (if desired).

Research Concepts:
Pericardiophrenic Vessels:

Tap flag to report any problems with this question.


Question 1244: An ECG reveals a narrow complex tachycardia with a
"saw tooth" pattern. What is your diagnosis?

Choices:
1. Atrial fibrillation (AF)
2. Supraventricular tachycardia (SVT)
3. Atrial flutter
4. Accelerated junctional tachycardia
Answer: 3 - Atrial flutter
Explanations:
Atrial flutter results in the classic "saw tooth" pattern on ECG
Pattern can be seen best in inferior ECG leads (II, III and aVF)
SVT can sometimes be aborted by vagal maneuvers
Vagal maneuvers should only be attempted in younger patients

Go to the next page if you knew the correct answer, or click the link images
below to further research the concepts in this question (if desired).

Research Concepts:
Atrial Flutter:

Tap flag to report any problems with this question.


Question 1245: Which of the following is increases the risk for anesthesia
the most?

Choices:
1. Greater than 5 premature ventricular contractions per minute
2. Emergency surgery
3. Age
4. Myocardial infarction 4 months ago
Answer: 4 - Myocardial infarction 4 months ago
Explanations:
Elderly patients have a decreased physiological reserve. There is higher
morbidity with complications. Age alone is not a contraindication to
surgery.
Emergency surgery results in increased morbidity and mortality.
Poorly controlled heart failure and unstable angina are relative
contraindications for anesthesia.
Other factors that affect risk include general medical condition, renal
failure, liver failure, and type of surgery.

Go to the next page if you knew the correct answer, or click the link images
below to further research the concepts in this question (if desired).

Research Concepts:
Preoperative Evaluation And Management:

Tap flag to report any problems with this question.


Question 1246: How should a provider respond to a patient who states that
her disease process is a punishment for misdeeds?

Choices:
1. Allow the patient to express her feelings
2. Consult a chaplain
3. Inform a family member
4. Prescribe antidepressants
Answer: 1 - Allow the patient to express her feelings
Explanations:
It is important to allow patients to express their feelings and to respond to
them in a non-threatening non-judgmental way.

Go to the next page if you knew the correct answer, or click the link images
below to further research the concepts in this question (if desired).

Research Concepts:
Counseling:

Tap flag to report any problems with this question.


Question 1247: Graft versus host reaction is best diagnosed with which
method?

Choices:
1. Complement levels
2. HLA crossmatch
3. Mixed lymphocyte culture
4. Biopsy
Answer: 4 - Biopsy
Explanations:
There is no blood test that can identify a graft versus host reaction.
Biopsy is the primary tool for evaluation suspected graft versus host
disease.
The diagnosis can be complicated from other viral exanthems and drug
eruptions and requires clinical correlation.
Overwhelming sepsis is the most common causes of death in acute graft
versus host reaction.

Go to the next page if you knew the correct answer, or click the link images
below to further research the concepts in this question (if desired).

Research Concepts:
Graft Versus Host Disease:

Tap flag to report any problems with this question.


Question 1248: A ventilator dependent neonate born at 29 weeks and 950
g still required dopamine to maintain blood pressure after a month. There is a
large patent ductus arteriosus (PDA) that has not closed after 3 trials of
indomethacin. Select appropriate treatment.

Choices:
1. Repeat indomethacin
2. Thoracoscopic ligation
3. Posterior incision for direct clipping
4. PDA division and oversewing
Answer: 3 - Posterior incision for direct clipping
Explanations:
Thoracoscopy offers no advantage
PDA division and oversewing requires a large incision and may be risky
Further usage of indomethacin is unlikely to be successful

Go to the next page if you knew the correct answer, or click the link images
below to further research the concepts in this question (if desired).

Research Concepts:
Patent Ductus Arteriosus:

Tap flag to report any problems with this question.


Question 1249: What is the most appropriate way to manage an
apprehensive patient who has many questions before a procedure or course of
treatment?

Choices:
1. Reassure her that everything will be fine
2. Have a family member try to calm her down
3. Answer all her questions and give her as much information as possible
4. Ask the chaplain to visit with the patient
Answer: 3 - Answer all her questions and give her as much information as
possible

Explanations:
Patients should given as much information as they request and all their
questions should be addressed.

Go to the next page if you knew the correct answer, or click the link images
below to further research the concepts in this question (if desired).

Research Concepts:
Informed Consent:

Tap flag to report any problems with this question.


Question 1250: Patients with atrial fibrillation are prescribed warfarin to:
Choices:
1. Slow heart rate
2. Empirically treat varicose veins
3. Pharmacologically convert patients to sinus rhythm
4. Prevent formation of intra-cardiac thrombus
Answer: 4 - Prevent formation of intra-cardiac thrombus
Explanations:
Warfarin is an anticoagulant and used in the treatment of atrial fibrillation
The drug prevents thrombus formation and release of emboli into the
systemic circulation

Go to the next page if you knew the correct answer, or click the link images
below to further research the concepts in this question (if desired).

Research Concepts:
Warfarin:

Atrial Fibrillation:

Tap flag to report any problems with this question.


Question 1251: As the ventilation rate increases, what happens to the level
of CO2?

Choices:
1. Increases
2. Decreases
3. No changes
4. Increases by four folds
Answer: 2 - Decreases
Explanations:
The concentration of carbon dioxide is inversely proportional to the
ventilation rate.

Go to the next page if you knew the correct answer, or click the link images
below to further research the concepts in this question (if desired).

Research Concepts:
Ventilator Management:

Tap flag to report any problems with this question.


Question 1252: Failure to control diabetes can lead to several
complications. What is the most common cause of death in these individuals?

Choices:
1. Renal failure
2. Stroke
3. Myocardial infarct
4. Ischemic bowel
Answer: 3 - Myocardial infarct
Explanations:
Macro-vascular disease is the leading cause of death in people with
diabetes.
At least 60 percent of people with diabetes eventually die of cardiac
complications.
Moreover, the risk of stroke is double that of non-diabetics and the risk of
peripheral vascular disease is 4 times that of the normal population.
Thus, lipid abnormalities must be treated very aggressively.

Go to the next page if you knew the correct answer, or click the link images
below to further research the concepts in this question (if desired).

Research Concepts:
Diabetes Mellitus, Type 1:

Diabetes Mellitus, Type 2:

Tap flag to report any problems with this question.


Question 1253: Antibiotic prophylaxis for subacute bacterial endocarditis
(SBE) is recommended for all of the following types of patients undergoing
dental procedures except?

Choices:
1. Prosthetic heart valves
2. Prior history of SBE
3. Congenital heart disease with prosthetic patch or prosthetic device
4. CABG
Answer: 4 - CABG
Explanations:
Antibiotic prophylaxis against SBE is recommended for the patients listed
above, except CABG patients,
Prophylaxis is not indicated in patients with an isolated ASD, CABG,
physiologic murmurs, previous Kawasaki disease without valve
dysfunction, and in those with defibrillators or pacemakers.
Other indications include unrepaired cyanotic heart disease and valvular
disease after transplant

Go to the next page if you knew the correct answer, or click the link images
below to further research the concepts in this question (if desired).

Research Concepts:
Subacute Bacterial Endocarditis (SBE) Prophylaxis:

Tap flag to report any problems with this question.


Question 1254: Which of the following medications is commonly
implicated in malignant hyperthermia?

Choices:
1. Dantrolene
2. Baclofen
3. Succinylcholine
4. Atracurium
Answer: 3 - Succinylcholine
Explanations:
Malignant hyperthermia is characterized by hyperthermia and muscular
rigidity
It is caused by excessive release of calcium from sarcoplasmic reticulum
The drug commonly implicated in this situation is succinylcholine
The treatment for this condition is dantrolene

Go to the next page if you knew the correct answer, or click the link images
below to further research the concepts in this question (if desired).

Research Concepts:
Malignant Hyperthermia:

Succinylcholine:

Tap flag to report any problems with this question.


Question 1255: A patient is suspected of having hypertrophic
cardiomyopathy. He has a grade III/VI mid systolic murmur that shows
crescendo decrescendo. Select the maneuver that would increase the volume of
the murmur.

Choices:
1. Valsalva
2. Squatting
3. Leaning forward
4. Lying supine
Answer: 1 - Valsalva
Explanations:
Asymmetric interventricular septal hypertrophy is seen with hypertrophic
cardiomyopathy
This causes dynamic outflow obstruction
If left ventricular filling is decreased the intensity of the murmur will
increase
Both valsalva and standing will achieve this

Go to the next page if you knew the correct answer, or click the link images
below to further research the concepts in this question (if desired).

Research Concepts:
Hypertrophic Obstructive Cardiomyopathy:

Tap flag to report any problems with this question.


Question 1256: Which of the following accurately describes the
bioavailability of oral digoxin?

Choices:
1. 25%
2. 50%
3. 75%
4. 100%
Answer: 3 - 75%
Explanations:
The bioavailability of IV digoxin is 100%
The bioavailability of PO digoxin is 75%
When converting from IV to PO digoxin there is a 25% decrease in
bioavailability
When converting from PO to IV digoxin there is a 25% increase in
bioavailability

Go to the next page if you knew the correct answer, or click the link images
below to further research the concepts in this question (if desired).

Research Concepts:
Digoxin:

Bioavailability:

Tap flag to report any problems with this question.


Question 1257: In a patient with heparin-induced thrombocytopenia
associated with bleeding, which of the following is the most appropriate
treatment?

Choices:
1. Fresh frozen plasma
2. Vitamin K
3. Platelets
4. Cryoprecipitate
Answer: 3 - Platelets
Explanations:
Heparin-induced thrombocytopenia occurs in about 5% of patients
receiving heparin. Auto-antibodies are directed towards the heparin/platelet
complex.
Cardiac patients routinely receive heparin and become sensitized to even
small amounts of heparin flush. Thrombocytopenia occurs, and if heparin is
stopped, the platelets recover.
Patients may harbor antibodies and the diagnosis can be confirmed by
platelet aggregation in the presence of serum. All elective cardiac surgery
should be postponed.
If a patient is bleeding from heparin-induced thrombocytopenia, the
treatment is administration of platelets.

Go to the next page if you knew the correct answer, or click the link images
below to further research the concepts in this question (if desired).

Research Concepts:
Thrombocytopenia, Heparin Induced:

Thrombocytopenia, Heparin Induced:


Tap flag to report any problems with this question.
Question 1258: Which of the following is not at common complication of
congenital heart disease seen in adults?

Choices:
1. Stroke
2. Eisenmenger syndrome
3. Cyanosis
4. Bacterial endocarditis
Answer: 1 - Stroke
Explanations:
Stroke can be seen in children less than 4 years old with congenital heart
disease (CHD)
The incidence is not increased in adults with CHD
Patients with patent ductus arteriosus and atrial septal defect can develop
left to right shunts, or Eisenmenger syndrome
This can cause pulmonary hypertension and cyanosis

Go to the next page if you knew the correct answer, or click the link images
below to further research the concepts in this question (if desired).

Research Concepts:
Congenital Heart Disease:

Tap flag to report any problems with this question.


Question 1259: What is the basic difference between a surgical infection
and a medical infection?

Choices:
1. Surgery infections are usually caused by a single group of bacteria
2. Surgery infections often have a source of contamination
3. Surgery infections are most commonly caused by anaerobes only
4. Surgery infections are less harmful
Answer: 2 - Surgery infections often have a source of contamination
Explanations:
Many surgical infections can be eliminated if the source can be identified
and treated.
Many surgical infections are polymicrobial and can be serious.
Surgical site infections can be due to anaerobes, especially when bowel
surgery is performed, but the infections are often polymicrobial.

Go to the next page if you knew the correct answer, or click the link images
below to further research the concepts in this question (if desired).

Research Concepts:
Infection:

Tap flag to report any problems with this question.


Question 1260: What bacterial pathogen is a common cause of hospital-
acquired pneumonia?

Choices:
1. Listeria
2. Pseudomonas aeruginosa
3. E. coli
4. Mycoplasma
Answer: 2 - Pseudomonas aeruginosa
Explanations:
Pseudomonas aeruginosa and Streptococcus pneumonia are the most
common pathogens for hospital-acquired pneumonia.
Treatment involves a macrolide plus a third-generation cephalosporin.

Go to the next page if you knew the correct answer, or click the link images
below to further research the concepts in this question (if desired).

Research Concepts:
Hospital Acquired Pneumonia:

Tap flag to report any problems with this question.


Question 1261: Which is false about the murmur in rheumatic mitral
stenosis?

Choices:
1. Diastolic thrill is often palpable
2. Loud S1
3. Loudness or murmur correlates with severity
4. Opening snap
Answer: 3 - Loudness or murmur correlates with severity
Explanations:
Cardiac exam in a patient with mitral stenosis will reveal a palpable
diastolic thrill and an opening snap.
The intensity of the murmur bears no correlation to severity. However, the
duration is a guide to the severity of the condition.
Most patients have a loud S1.
There is a characteristic diastolic low-pitched rumbling murmur.

Go to the next page if you knew the correct answer, or click the link images
below to further research the concepts in this question (if desired).

Research Concepts:
Mitral Stenosis:

Tap flag to report any problems with this question.


Question 1262: What di the Bypass Angioplasty Revascularization
Investigation (BARI) concluded about diabetics?

Choices:
1. CABG benefit was more robust in those on sulfonylureas
2. PTCA benefit was more robust in those on insulin
3. No difference in survival when CABG compared to PTCA
4. CABG is associated with better long-term survival than PTCA
Answer: 4 - CABG is associated with better long-term survival than PTCA
Explanations:
Amongst diabetics, CABG was associated with better long-term survival
than PTCA
CABG benefit was more robust among diabetics requiring insulin
5-year all cause mortality rate was 34.5% for those diabetics randomized to
PTCA vs. 19.4% for those randomized to CABG

Go to the next page if you knew the correct answer, or click the link images
below to further research the concepts in this question (if desired).

Research Concepts:
Diabetes Mellitus:

Coronary Artery Bypass Graft:

Percutaneous Transluminal Coronary Angioplasty:


Tap flag to report any problems with this question.
Question 1263: In a 44-year-old with primary pulmonary hypertension,
one would expect to find which of the following?

Choices:
1. Tricuspid stenosis
2. Right ventricular hypertrophy
3. Narrowing of aorta
4. Aortic stenosis
Answer: 2 - Right ventricular hypertrophy
Explanations:
Primary pulmonary hypertension carries an unknown etiology.
It is associated with right ventricular hypertrophy.
The mean survival is less than four years from the time of diagnosis.

Go to the next page if you knew the correct answer, or click the link images
below to further research the concepts in this question (if desired).

Research Concepts:
Pulmonary Hypertension:

Tap flag to report any problems with this question.


Question 1264: What effect does erythropoietin have on red blood cell
(RBC) production?

Choices:
1. It increases RBC production
2. It decreases RBC production
3. It stops RBC production
4. It has no effect on RBC production
Answer: 1 - It increases RBC production
Explanations:
Erythropoietin stimulates the bone marrow to increase RBC production.
Natural erythropoietin is a hormone that is made primarily in the kidney.

Go to the next page if you knew the correct answer, or click the link images
below to further research the concepts in this question (if desired).

Research Concepts:
Erythropoietin:

Tap flag to report any problems with this question.


Question 1265: You are most likely to observe a bicuspid aortic valve in a
patient with which of the following conditions?

Choices:
1. Patent ductus arteriosus
2. Atrial septal defect
3. Tetralogy of fallot
4. Coarctation of the aorta
Answer: 4 - Coarctation of the aorta
Explanations:
Bicuspid aortic valve is often seen in patients with other left sided
obstructive lesions.
Bicuspid aortic valve is commonly seen in patients with coarctation of aorta
and interrupted aortic arch.

Go to the next page if you knew the correct answer, or click the link images
below to further research the concepts in this question (if desired).

Research Concepts:
Aortic Coarctation:

Aortic Valve:

Tap flag to report any problems with this question.


Question 1266: Libman-Sacks endocarditis is associated with which
chronic illness?

Choices:
1. Rheumatoid arthritis
2. System lupus erythematosus
3. Polyarteritis nodosa
4. Myasthenia gravis
Answer: 2 - System lupus erythematosus
Explanations:
Libman-Sacks endocarditis is a nonbacterial vegetative endocarditis
associated with SLE, affects any valve (often mitral), can cause cerebral
emboli

Go to the next page if you knew the correct answer, or click the link images
below to further research the concepts in this question (if desired).

Research Concepts:
Libman-Sacks Endocarditis:

Systemic Lupus Erythematosus:

Tap flag to report any problems with this question.


Question 1267: If a coronary artery has a 50% reduction in diameter from
a plaque, the maximal blood flow through the artery is reduced by which value?

Choices:
1. 1/16
2. 1/2
3. 1/8
4. ¼
Answer: 1 - 1/16
Explanations:
Flow is inversely proportional to resistance
R = resistance, v = viscosity, L = length, and r = radius
The variable in question is the radius. As radius is to the 4th power the
reduction of the radius by ½ results in1/16th of the flow

Go to the next page if you knew the correct answer, or click the link images
below to further research the concepts in this question (if desired).

Research Concepts:
Coronary Artery Disease:

Tap flag to report any problems with this question.


Question 1268: Which of the following is an incorrect statement regarding
the difference between percutaneous transluminal coronary angiography (PTCA)
and coronary artery bypass grafting (CABG) for multi-vessel disease?

Choices:
1. There is no difference in death at 3 years between the two groups
2. There is an increased incidence of angina with the PTCA group up to 1 year
3. With the repeated use of angioplasty in the PTCA group, patients remain
more re-vascularized compared to CABG patients at the end of 3 years
4. CABG patients rarely require intervention for up to 10 years if the
mammary is used.
Answer: 3 - With the repeated use of angioplasty in the PTCA group,
patients remain more re-vascularized compared to CABG patients at the end of 3
years

Explanations:
Despite the use of repeat procedures in the angioplasty group, the surgery
patients remain more completely re-vascularized at the end of 3 years.
For patients with single vessel disease, both medical therapy and
angioplasty have comparable results to surgery.
For patients with simple forms of two vessel disease, improvement occurs
with angioplasty and surgery rather than medical therapy.
For those with more complex disease, surgery out-performed both medical
therapy and angioplasty treatment.

Go to the next page if you knew the correct answer, or click the link images
below to further research the concepts in this question (if desired).

Research Concepts:
Revascularization Procedures In Coronary Artery
Disease:

Tap flag to report any problems with this question.


Question 1269: What is he most serious complication of Kawasaki
disease?

Choices:
1. Coronary artery disease
2. Coronary artery fistula
3. Coronary artery aneurysms
4. Cardiomyopathy
Answer: 3 - Coronary artery aneurysms
Explanations:
Kawasaki disease affects children under the age of 5
It is a multisystem disorder and the acute illness presents with fever,
cervical lymphadenopathy, vasculitis and mucocutaneous changes
In its advanced stage, it is characterized by coronary artery aneurysms
Thrombotic occlusion may develop and cause myocardial ischemia. Death
can be caused by rupture of the coronary artery aneurysm

Go to the next page if you knew the correct answer, or click the link images
below to further research the concepts in this question (if desired).

Research Concepts:
Kawasaki Disease:

Kawasaki Disease:

Tap flag to report any problems with this question.


Question 1270: Which is not usually seen in aortic regurgitation?
Choices:
1. Dyspnea on exertion
2. Congestive heart failure
3. Re-entrant tachycardia
4. Cardiogenic shock
Answer: 3 - Re-entrant tachycardia
Explanations:
Dyspnea on exertion, CHF, and cardiogenic shock are common
presentations of aortic regurgitation
Re-entrant tachycardia is most often seen with conduction abnormalities,
such as Wolff-Parkinson-White

Go to the next page if you knew the correct answer, or click the link images
below to further research the concepts in this question (if desired).

Research Concepts:
Aortic Regurgitation:

Tap flag to report any problems with this question.


Question 1271: A patient has been taking hydrochlorothiazide for several
years and is complaining of anorexia, muscle cramps, and confusion. What is the
most likely cause of these symptoms?

Choices:
1. Drug toxicity
2. Hyponatremia
3. Hypovolemia
4. Hypokalemia
Answer: 4 - Hypokalemia
Explanations:
The patient's symptoms are characteristic of hypokalemia (potassium
depletion). Hydrochlorothiazide (Ezide) is a diuretic (thiazide) that
increases excretion of water and sodium by preventing sodium reabsorption
in the distal tubules. In addition, excretion of chloride, bicarbonate,
magnesium, and potassium takes place.
The loss of too much potassium results in hypokalemia and should be
treated. Symptoms of hypokalemia include anorexia, muscle cramps,
numbness and tingling of lower extremities, confusion, and coma.
Hypotension and tachycardia would suggest drug toxicity. Major symptoms
of hyponatremia (sodium depletion) include anxiety, drowsiness and stupor,
muscle weakness, convulsions, oliguria, and anuria.
Major symptoms of hypovolemia (abnormally decreased volume of
circulating fluid) include oliguria, hypotension, and dry skin. Patient Need:
Physiological Integrity

Go to the next page if you knew the correct answer, or click the link images
below to further research the concepts in this question (if desired).

Research Concepts:
Hypokalemia:

Thiazide Diuretics:
Tap flag to report any problems with this question.
Question 1272: A 72 year old female complains of a chronic dry cough of
6 months duration. She has a history of ischemic cardiomyopathy on
simvastatin, aspirin, lisinopril, furosemide, and metoprolol. She denies nasal
symptoms, reflux symptoms, fever, chills, or weight loss. Exam shows normal
ENT, cardiovascular, pulmonary, and abdominal exams. Chest x-ray shows
cardiomegaly but no other abnormalities. Select the most appropriate next step.

Choices:
1. Pulmonary function tests
2. CT of the sinuses
3. Change lisinopril to candesartan
4. Treat with azithromycin
Answer: 3 - Change lisinopril to candesartan
Explanations:
The patient may have an ACE inhibitor induced cough that occurs in 5 to
20% of patients.
Substitution of an angiotensin receptor blocker is the best option.
The patient has no evidence of other explanations for cough such as post
nasal drip, congestive heart failure, or gastroesophageal reflux.

Go to the next page if you knew the correct answer, or click the link images
below to further research the concepts in this question (if desired).

Research Concepts:
Angiotensin Converting Enzyme Inhibitors (ACEI):

Cough:

Tap flag to report any problems with this question.


Question 1273: Why are beta blockers given to patients with angina?
Choices:
1. It reduces the preload
2. It reduces the afterload
3. It reduces cardiac output
4. It reduces heart rate
Answer: 4 - It reduces heart rate
Explanations:
Beta blockers reduce heart rate and indirectly improve coronary circulation.
Coronary circulation mainly occurs in diastole.
Beta blockers do not directly affect preload and afterload.
Nitroglycerin reduces preload and alleviates the workload on the heart.

Go to the next page if you knew the correct answer, or click the link images
below to further research the concepts in this question (if desired).

Research Concepts:
Angina, Unstable:

Angina, Stable:

Beta-Blockers:
Tap flag to report any problems with this question.
Question 1274: A man with mitral regurgitation and stenosis that was first
diagnosed over 30 years ago most likely suffered from which of the following
illnesses as a child?

Choices:
1. Rheumatic fever
2. Scarlet fever
3. Polio
4. Smallpox
Answer: 1 - Rheumatic fever
Explanations:
Most patients with mitral stenosis have a history of rheumatic fever

Go to the next page if you knew the correct answer, or click the link images
below to further research the concepts in this question (if desired).

Research Concepts:
Rheumatic Heart Disease:

Mitral Stenosis:

Mitral Regurgitation:

Tap flag to report any problems with this question.


Question 1275: Patients who are candidates for mitral valve replacement
should demonstrate a history of compliance with which of the following medical
regimens:

Choices:
1. Diet
2. Dental care
3. Medications
4. Keeping appointments
Answer: 3 - Medications
Explanations:
Postoperatively, all patients with mechanical valves and some patients with
bioprostheses need indefinite anticoagulant therapy

Go to the next page if you knew the correct answer, or click the link images
below to further research the concepts in this question (if desired).

Research Concepts:
Mitral Valve Replacement:

Tap flag to report any problems with this question.


Question 1276: A patient is involved in a motor vehicle accident. He is
brought into the emergency room and found to have a broken femur. His blood
work reveals a hemoglobin of 8 and a hematocrit of 26. It is decided to transfuse
him with three units of blood. Soon after the blood is started, he develops a fever
and becomes hypotensive. In a patient suspected of having a hemolytic
transfusion reaction, which of the following should be done first?

Choices:
1. Insert Foley catheter
2. Restrict fluids
3. Start steroids
4. Give fluids and mannitol
Answer: 4 - Give fluids and mannitol
Explanations:
Anytime hemolytic reaction from transfusion is suspected, the transfusion
should be immediately stopped. A saline infusion should be started
immediately followed by a diuretic, such as mannitol.
Low dose dopamine may improve renal blood flow. If there is evidence of
anaphylaxis, administer epinephrine, diphenhydramine, and corticosteroids.
If there is suspected acute lung injury, the patient may require endotracheal
intubation with positive pressure ventilation.
Massive blood transfusions can often cause hypothermia, hypocalcemia,
and hyperkalemia.

Go to the next page if you knew the correct answer, or click the link images
below to further research the concepts in this question (if desired).

Research Concepts:
Hemolytic Transfusion Reaction:

Tap flag to report any problems with this question.


Question 1277: What is the name of the syndrome when a left to right
becomes a right to left shunt due to increased pulmonary pressure?

Choices:
1. Eisenmenger
2. Eisinga
3. Eisenhower
4. Munchausen
Answer: 1 - Eisenmenger
Explanations:
A number of congenital cardiac defects can result in this syndrome.
Examples are ASD, VSD, and Teratology of Fallot.

Go to the next page if you knew the correct answer, or click the link images
below to further research the concepts in this question (if desired).

Research Concepts:
Eisenmenger Syndrome:

Tap flag to report any problems with this question.


Question 1278: Which is true concerning pericardial effusion?
Choices:
1. Visualized on ultrasound, echocardiogram, or transesophageal
echocardiogram
2. Supine position is most comfortable for patients
3. Emergency surgical treatment with pericardiocentesis, pericardiotomy, or
pericardial window is essential
4. There is always and associated pleural effusion
Answer: 1 - Visualized on ultrasound, echocardiogram, or transesophageal
echocardiogram

Explanations:
Usually patients prefer to sit up and lean forward for comfort
If tamponade is present, pericardiocentesis or pericardiotomy is an
emergency procedure
If the effusion is small, or occurs very slowly over time, pericardiocentesis
may not be necessary
Rapidly occurring effusion of 150 ml may decrease cardiac output; slow
increase in accumulation may tolerate 2 L without effect

Go to the next page if you knew the correct answer, or click the link images
below to further research the concepts in this question (if desired).

Research Concepts:
Pericardial Effusion:

Tap flag to report any problems with this question.


Question 1279: A 3-year-old is seen in the clinic with short stature,
generalized swelling, and coarctation of the aorta. She appears to have normal
intellect. She most likely has which of the following conditions?

Choices:
1. Down syndrome
2. Turner syndrome
3. Cretinism
4. Klinefelter syndrome
Answer: 2 - Turner syndrome
Explanations:
Turner syndrome has a 45,X karyotype, and occurs in females.
Individuals are usually infertile secondary to streak ovaries and are also
short in stature.
Other features include a webbed neck, lymphedema of the extremities,
skeletal abnormalities, renal disease, and congenital heart defects.
Individuals can also have hypothyroidism, diabetes mellitus, autoimmune
diseases, and specific cognitive deficits.

Go to the next page if you knew the correct answer, or click the link images
below to further research the concepts in this question (if desired).

Research Concepts:
Turner Syndrome:

Tap flag to report any problems with this question.


Question 1280: Which of the following does not reduce the risk of
atherothrombotic stroke?

Choices:
1. Clopidogrel
2. Warfarin
3. Aspirin
4. Statins
Answer: 2 - Warfarin
Explanations:
Warfarin is proven to prevent cardioembolic strokes not atherothrombotic
Risk factors for atherosclerosis and therefore stroke are hypertension,
hyperlipidemia, smoking, diabetes, age, and family history
Antiplatelet therapy reduces the risk of nonfatal stroke by 25 to 30%
Statins reduce the risk of stroke, even in patients without hyperlipidemia

Go to the next page if you knew the correct answer, or click the link images
below to further research the concepts in this question (if desired).

Research Concepts:
Warfarin:

Stroke Prevention:

Tap flag to report any problems with this question.


Question 1281: Which port of the Swan-Ganz catheter is used to obtain a
mixed venous blood sample?

Choices:
1. Medial
2. Distal
3. Fluid
4. Proximal
Answer: 2 - Distal
Explanations:
A mixed venous sample is drawn from the distal (pulmonary artery) port of
a Swan-Ganz catheter.

Go to the next page if you knew the correct answer, or click the link images
below to further research the concepts in this question (if desired).

Research Concepts:
Pulmonary Artery Catheterization:

Tap flag to report any problems with this question.


Question 1282: Select the incorrect statement about mitral valve prolapse.
Choices:
1. If there is no regurgitation or valve thickening the risk of endocarditis is
minimal
2. The patients with worse prognosis are males and patients older than 50
3. The degree of mitral regurgitation causes most symptoms
4. Patients with Marfan syndrome may have this condition
Answer: 3 - The degree of mitral regurgitation causes most symptoms
Explanations:
Many patients with MVP have symptoms of dizziness, atypical chest pain,
and easy fatigability
These are thought to be secondary to autonomic dysfunction rather than the
degree of mitral regurgitation

Go to the next page if you knew the correct answer, or click the link images
below to further research the concepts in this question (if desired).

Research Concepts:
Mitral Valve Prolapse:

Tap flag to report any problems with this question.


Question 1283: Atrioventricular block is very rare after use of which
medications?

Choices:
1. Procainamide
2. Beta blockers
3. Calcium channel blockers
4. SSRIs
Answer: 4 - SSRIs
Explanations:
Other causes of AV block include Chagas disease, myotonic dystrophy,
calcium channel blockers, beta-blockers, quinidine, Lenegre disease, Lev
disease, rheumatic fever, myocarditis, and Aspergillus myocarditis.

Go to the next page if you knew the correct answer, or click the link images
below to further research the concepts in this question (if desired).

Research Concepts:
Atrioventricular Block, Third-Degree:

Atrioventricular Block, Second-Degree:

Tap flag to report any problems with this question.


Question 1284: Which of the following is NOT a side effect of
corticosteroids?

Choices:
1. Striae
2. Muscle atrophy
3. Psychosis
4. Salt wasting
Answer: 4 - Salt wasting
Explanations:
Predictable adverse effects include adrenal suppression, growth inhibition,
muscle wasting, osteoporosis, salt retention, diabetes, and psychosis.

Go to the next page if you knew the correct answer, or click the link images
below to further research the concepts in this question (if desired).

Research Concepts:
Corticosteroids:

Tap flag to report any problems with this question.


Question 1285: What is suggested by a sudden increase in blood
myoglobin levels in a 69 year old?

Choices:
1. Malignancy
2. Infection
3. Myocardial damage
4. Hemolytic anemia
Answer: 3 - Myocardial damage
Explanations:
Myoglobin is generally detected about 1 hour after a heart attack is
experienced.
It peaks within 4 to 6 hours after infarction.
It is nonspecific so it is rarely used as a cardiac marker.

Go to the next page if you knew the correct answer, or click the link images
below to further research the concepts in this question (if desired).

Research Concepts:
Myoglobin:

Tap flag to report any problems with this question.


Question 1286: Which medication is not associated with long term
survival in patients with congestive heart failure?

Choices:
1. Metoprolol
2. Digoxin
3. Lisinopril
4. Captopril
Answer: 2 - Digoxin
Explanations:
Digoxin is commonly used in congestive heart failure for its inotropic effect
But it has not been documented to increase long term survival in patients
with CHF
Beta blockers are well documented in increasing long term survival of
patients

Go to the next page if you knew the correct answer, or click the link images
below to further research the concepts in this question (if desired).

Research Concepts:
Heart Failure, Congestive:

Tap flag to report any problems with this question.


Question 1287: Which of the following is not a crystalloid fluid?
Choices:
1. Normal saline
2. Ringers lactate
3. Albumin
4. 5% dextrose in water
Answer: 3 - Albumin
Explanations:
Crystalloid solutions are composed of electrolytes and non-electrolytes.
They are able to diffuse into all body fluid compartments.
Colloid solutions cannot pass through semi-permeable membranes. These
include starch, albumin, dextran, or mannitol.

Go to the next page if you knew the correct answer, or click the link images
below to further research the concepts in this question (if desired).

Research Concepts:
Crystalloid Fluids:

Tap flag to report any problems with this question.


Question 1288: At what level is prehypertension diagnosed?
Choices:
1. Systolic blood pressure (SBP) 120-139, diastolic blood pressure (DBP) 80-
89
2. SBP 140-149, DBP 90-95
3. SBP 150-165, DBP 96-100
4. SBP 140, DBP 90
Answer: 1 - Systolic blood pressure (SBP) 120-139, diastolic blood pressure
(DBP) 80-89

Explanations:
Prehypertension is a relatively new classification.
Prehypertension is SBP between 120-139 and DBP 80-89.
Readings above or equal to 140/90 are considered hypertension.
Normal blood pressure is 120/80 or less.

Go to the next page if you knew the correct answer, or click the link images
below to further research the concepts in this question (if desired).

Research Concepts:
Prehypertension:

Tap flag to report any problems with this question.


Question 1289: A patient taking atorvastatin 80 mg daily with prior MI
has a lipid profile revealing total cholesterol 190 mg/dL, HDL 30 mg/dL, and
LDL 90. What is the most appropriate recommendation?

Choices:
1. Increase dose of statin
2. No recommendation needed
3. Addition of niacin
4. Addition of metformin
Answer: 3 - Addition of niacin
Explanations:
Addition of niacin should help increase HDL levels in this patient to the
goal of >40 mg/dL
Atorvastatin is already at maximum dose of 80 mg/day
LDL is at goal of <100 mg/dL for patient with heart disease
Total cholesterol is also at goal of <200 mg/dL

Go to the next page if you knew the correct answer, or click the link images
below to further research the concepts in this question (if desired).

Research Concepts:
Hyperlipidemia:

Coronary Artery Disease:

Tap flag to report any problems with this question.


Question 1290: Which of the following antihypertensive can cause
cyanide poisoning?

Choices:
1. Hydralazine
2. Verapamil
3. Nitroprusside
4. Prazosin
Answer: 3 - Nitroprusside
Explanations:
Thiocyanate can accumulate in nitroprusside therapy and cause poisoning
Verapamil can cause constipation
Prazosin can cause orthostatic hypotension

Go to the next page if you knew the correct answer, or click the link images
below to further research the concepts in this question (if desired).

Research Concepts:
Sodium Nitroprusside:

Cyanide Toxicity:

Tap flag to report any problems with this question.


Question 1291: What is the most common cause of valve failure after
replacement of a mitral valve with a tissue valve?

Choices:
1. Calcification
2. Thrombosis
3. Degeneration
4. Valve loosening
Answer: 3 - Degeneration
Explanations:
The major limitation of tissue valves is that they do not last very long.
By the sixth year, there is evidence of degeneration.
By the tenth year, at least 30% of valves may require replacement.

Go to the next page if you knew the correct answer, or click the link images
below to further research the concepts in this question (if desired).

Research Concepts:
Mitral Stenosis:

Tap flag to report any problems with this question.


Question 1292: Which of the following is not an innocent murmur?
Choices:
1. Ejection murmur
2. Grade 2/6 systolic murmur
3. Diastolic murmur
4. Murmur that appears after exercise
Answer: 3 - Diastolic murmur
Explanations:
Diastolic murmurs are never innocent
Innocent murmurs are always systolic and are not graded more than 2/6
Murmur appearing after exercise is most likely an innocent one

Go to the next page if you knew the correct answer, or click the link images
below to further research the concepts in this question (if desired).

Research Concepts:
Innocent Murmur:

Tap flag to report any problems with this question.


Question 1293: Which of the following drugs is NOT recommended for
use in patients with cor pulmonale?

Choices:
1. Theophylline
2. Diltiazem
3. Digoxin
4. Warfarin
Answer: 3 - Digoxin
Explanations:
The use of digoxin in patients with cor pulmonale is not well accepted.
Unlike left heart failure, the benefits of digoxin in right ventricular
dysfunction are inconsistent.
Increasing evidence shows that in the presence of hypoxia and acidosis,
digoxin may potentiate arrhythmias.

Go to the next page if you knew the correct answer, or click the link images
below to further research the concepts in this question (if desired).

Research Concepts:
Cor Pulmonale:

Tap flag to report any problems with this question.


Question 1294: Which is not a typical laboratory feature of a pleural
effusion in a 67 year old with congestive heart failure?

Choices:
1. Low LDH
2. High RBC
3. Low protein
4. High serum to pleural LDH ratio
Answer: 2 - High RBC
Explanations:
Transudate effusion is presented in congestive heart failure
This pleural effusion has low LDH and protein levels

Go to the next page if you knew the correct answer, or click the link images
below to further research the concepts in this question (if desired).

Research Concepts:
Effusion, Pleural:

Transudate:

Heart Failure, Congestive:

Tap flag to report any problems with this question.


Question 1295: What is the treatment for a new ventricular septal defect
after acute myocardial infarction?

Choices:
1. Diuretics
2. IV fluids
3. Surgery
4. Beta blockers
Answer: 3 - Surgery
Explanations:
VSD need to be surgically repaired if caused my acute MI
This is urgent

Go to the next page if you knew the correct answer, or click the link images
below to further research the concepts in this question (if desired).

Research Concepts:
Postinfarct Ventricular Septal Rupture:

Tap flag to report any problems with this question.


Question 1296: Potassium supplements are often required when there is
concomitant use of which of the following?

Choices:
1. Spironolactone
2. Hydrochlorothiazide (HCTZ)
3. Amiloride
4. Captopril
Answer: 2 - Hydrochlorothiazide (HCTZ)
Explanations:
Hydrochlorothiazide (HCTZ) is a potent diuretic associated with
hypokalemia.
Potassium supplements are usually given as concomitant therapy.
k

Go to the next page if you knew the correct answer, or click the link images
below to further research the concepts in this question (if desired).

Research Concepts:
Thiazide Diuretics:

Tap flag to report any problems with this question.


Question 1297: Which drug is most likely to precipitate congestive heart
failure?

Choices:
1. Lisinopril
2. Thiazide
3. Aspirin
4. Verapamil
Answer: 4 - Verapamil
Explanations:
Congestive heart failure can be caused by medications that are negatively
inotropic, such as calcium channel blockers and beta blockers.
Despite this, beta blockers have been shown to be beneficial in these
patients but not calcium channel blockers.
Beta blockers reduce mortality and prevent deleterious remodeling.
It can also be triggered by medications that cause fluid retention, such as
NSAIDS.

Go to the next page if you knew the correct answer, or click the link images
below to further research the concepts in this question (if desired).

Research Concepts:
Heart Failure, Congestive:

Tap flag to report any problems with this question.


Question 1298: During pulmonary vasoconstriction, pressure in which
chamber will be increased?

Choices:
1. Right ventricular
2. Pulmonary wedge
3. Left ventricle
4. Left atrial
Answer: 1 - Right ventricular
Explanations:
Right ventricular pressure will likely increase with pulmonary
vasoconstriction.

Go to the next page if you knew the correct answer, or click the link images
below to further research the concepts in this question (if desired).

Research Concepts:
Pulmonary Vasoconstriction:

Tap flag to report any problems with this question.


Question 1299: Which of the following sign or symptom is not consistent
with a pulmonary embolus?

Choices:
1. Hemoptysis
2. Tachycardia
3. Vomiting
4. Chest pain
Answer: 3 - Vomiting
Explanations:
The most common findings are tachypnea, tachycardia, rales, fourth heart
sound E2. The most common symptoms are dyspnea, pleuritic chest pain,
cough and hemoptysis

Go to the next page if you knew the correct answer, or click the link images
below to further research the concepts in this question (if desired).

Research Concepts:
Pulmonary Embolism:

Tap flag to report any problems with this question.


Question 1300: Which of the following is most appropriate for a patient
with massive hemoptysis and a normal chest x-ray?

Choices:
1. CT scan of the chest
2. Echocardiogram
3. Flexible bronchoscopy
4. Angiogram
Answer: 2 - Echocardiogram
Explanations:
Massive hemoptysis may be due to other causes, besides the lung.
If the chest x-ray is normal then one should suspect a cardiac cause.
An echocardiogram may reveal the presence of a valvular heart disorder,
congestive heart failure, or a cardiomyopathy.

Go to the next page if you knew the correct answer, or click the link images
below to further research the concepts in this question (if desired).

Research Concepts:
Hemoptysis:

Tap flag to report any problems with this question.


Question 1301: How is mean arterial pressure defined?
Choices:
1. Diastolic blood pressure + (systolic blood pressure - diastolic blood
pressure)/3
2. Systolic blood pressure + (systolic blood pressure - diastolic blood
pressure)/3
3. Diastolic blood pressure - (systolic blood pressure - diastolic blood
pressure)/3
4. Systolic blood pressure - (systolic blood pressure + diastolic blood
pressure)/3
Answer: 1 - Diastolic blood pressure + (systolic blood pressure - diastolic
blood pressure)/3

Explanations:
Mean arterial pressure is the perfusion pressure seen by the end organs of
the body.
A mean arterial pressure of 60 mm Hg or greater is adequate to sustain
viability of most organs in the body.

Go to the next page if you knew the correct answer, or click the link images
below to further research the concepts in this question (if desired).

Research Concepts:
Mean Arterial Pressure:

Tap flag to report any problems with this question.


Question 1302: All of the following can be administered via endotracheal
(ET) tube EXCEPT:

Choices:
1. Atropine
2. Atenolol
3. Epinephrine
4. Lidocaine
Answer: 2 - Atenolol
Explanations:
Atropine, epinephrine and lidocaine can be administered via ET tube
Naloxone and diazepam can also be administered via ET tube
Atenolol cannot be administered via ET tube

Go to the next page if you knew the correct answer, or click the link images
below to further research the concepts in this question (if desired).

Research Concepts:
Endotracheal Medications:

Tap flag to report any problems with this question.


Question 1303: Which is not a common cause of a pleural effusion?
Choices:
1. Congestive heart failure (CHF)
2. Pneumonia or pulmonary embolus
3. Malignancy
4. Chylothorax
Answer: 4 - Chylothorax
Explanations:
Chylothorax, or a tear or leak in the thoracic duct can cause chyle to collect
in the pleural space

Go to the next page if you knew the correct answer, or click the link images
below to further research the concepts in this question (if desired).

Research Concepts:
Pericardial Effusion:

Chylothorax:

Tap flag to report any problems with this question.


Question 1304: A patient in 1st degree heart block that is asymptomatic
should be treated with:

Choices:
1. Scheduled for pace maker insertion
2. No treatment at this time
3. Intravenous Cardizem
4. Prescription for Rythmol SR
Answer: 2 - No treatment at this time
Explanations:
Asymptomatic 1st degree heart block does not indicate a need for specific
treatment options

Go to the next page if you knew the correct answer, or click the link images
below to further research the concepts in this question (if desired).

Research Concepts:
Heart Block, First Degree:

Tap flag to report any problems with this question.


Question 1305: Which of the following procedures is contraindicated in a
patient with uncorrected severe factor IX deficiency?

Choices:
1. Peripheral intravenous line placement
2. Femoral central line placement
3. Antecubital venous blood draw
4. Brain CT scan
Answer: 2 - Femoral central line placement
Explanations:
Placement of central lines or arterial lines, lumbar punctures, intramuscular
injections, and any surgical procedures should be avoided in patients with
hemophilia (severe factor VIII or IX deficiency) until after factor
replacement therapy has been given
Peripheral venipuncture can be done, although repeated traumatic efforts
should be avoided
Hemophilia patients who have suffered head trauma should receive factor
replacement therapy before any brain imaging is performed, as any
intracranial hemorrhage that has occurred will continue bleeding until
factor replacement is given

Go to the next page if you knew the correct answer, or click the link images
below to further research the concepts in this question (if desired).

Research Concepts:
Central Line Placement:

Hemophilia B:

Tap flag to report any problems with this question.


Question 1306: Which congenital heart malformation may cause
cardiogenic shock after closure of the ductus arteriosus?

Choices:
1. Atrial septal defect
2. Ventricular septal defect
3. Mitral valve prolapse
4. Hypoplastic left heart syndrome
Answer: 4 - Hypoplastic left heart syndrome
Explanations:
Congenital heart disease may present as cardiogenic shock
Causes of neonatal cardiogenic shock include critical aortic stenosis,
hypoplastic left heart syndrome, and coarctation of the aorta

Go to the next page if you knew the correct answer, or click the link images
below to further research the concepts in this question (if desired).

Research Concepts:
Hypoplastic Left Heart Syndrome:

Tap flag to report any problems with this question.


Question 1307: How many more times potent is fentanyl than morphine?
Choices:
1. 100 times
2. 20 times
3. 80 times
4. 120 times
Answer: 1 - 100 times
Explanations:
Fentanyl is about 100 times more potent than morphine.

Go to the next page if you knew the correct answer, or click the link images
below to further research the concepts in this question (if desired).

Research Concepts:
Fentanyl:

Tap flag to report any problems with this question.


Question 1308: Sedation prior to surgery is ideal for which group of
patients?

Choices:
1. Young
2. Hypertensive
3. Older
4. Elevated ICP
Answer: 2 - Hypertensive
Explanations:
Sedation is not highly recommended for young, older, and head injury.

Go to the next page if you knew the correct answer, or click the link images
below to further research the concepts in this question (if desired).

Research Concepts:
Preoperative Evaluation And Management:

Tap flag to report any problems with this question.


Question 1309: Xanthelasma are often associated with which of the
following conditions?

Choices:
1. Trauma
2. Renal failure
3. Anemia
4. Elevated lipids
Answer: 4 - Elevated lipids
Explanations:
Xanthelasma are sharply demarcated collections of cholesterol deposits
underneath the skin, usually around the eye.
Xanthelasma are common in Asian and Mediterranean people.
In some people, they are an indication of hypercholesterolemia.

Go to the next page if you knew the correct answer, or click the link images
below to further research the concepts in this question (if desired).

Research Concepts:
Xanthelasma:

Tap flag to report any problems with this question.


Question 1310: A patient just returned from a tilt-test procedure and fell
out off the cart. The side rails were not put up. What type of liability is this?

Choices:
1. Criminal
2. Intentional tort
3. Negligence
4. Assault
Answer: 3 - Negligence
Explanations:
Negligence, a malpractice claim may be filed under tort law (unintentional
tort)

Go to the next page if you knew the correct answer, or click the link images
below to further research the concepts in this question (if desired).

Research Concepts:
Negligence:

Tap flag to report any problems with this question.


Question 1311: A medical student is learning about the mechanism of
action of several antibiotics. She learns that quinolones act by inhibiting an
enzyme that induces supercoiling in prokaryotic DNA, thereby relieving the
strain of positive supercoiling. Which of the following enzymes is inhibited by
quinolones?

Choices:
1. Helicase
2. Topoisomerase
3. DNA ligase
4. DNA polymerase
Answer: 2 - Topoisomerase
Explanations:
The quinolone antibiotics inhibit prokaryotic topoisomerase known as DNA
gyrase.
Helicase, DNA ligase and DNA polymerase are enzymes involved DNA
synthesis but not in supercoiling.

Go to the next page if you knew the correct answer, or click the link images
below to further research the concepts in this question (if desired).

Research Concepts:
Quinolones:

Topoisomerase ll:

Tap flag to report any problems with this question.


Question 1312: Which is false about aortic stenosis?
Choices:
1. It is present in 2% of elderly individuals over age 65
2. The cause of aortic stenosis is usually rheumatic disease
3. Initial symptom may be dyspnea or chest pain
4. Degenerative calcification is a common cause of aortic stenosis in elderly
Answer: 2 - The cause of aortic stenosis is usually rheumatic disease
Explanations:
The major cause of aortic stenosis include bicuspid aortic valve and
degenerative disease.
Despite what many people believe, rheumatic fever tends to cause aortic
regurgitation and minimal aortic stenosis.
Aortic calcification of a bicuspid aortic valve usually diagnosed in the 5th
decade of life and may present with symptoms in the 6/7 decade of life.
There is some evidence that the degenerative calcific stenosis that occurs in
elderly may be preventable by long-term treatment with statin drugs.

Go to the next page if you knew the correct answer, or click the link images
below to further research the concepts in this question (if desired).

Research Concepts:
Aortic Stenosis:

Tap flag to report any problems with this question.


Question 1313: A patient with stable paroxysmal supraventricular
tachycardia (PSVT) does not respond to vagal maneuvers. What is the next best
step in management?

Choices:
1. Atropine
2. Synchronized cardioversion
3. Adenosine
4. Labetalol
Answer: 3 - Adenosine
Explanations:
Adenosine can provide short-term relief of PSVT if vagal maneuvers fail to
terminate the rhythm
Calcium channel blockers can also be used for PSVT refractory to vagal
maneuvers
Synchronized cardioversion is reserved for those with unstable PSVT

Go to the next page if you knew the correct answer, or click the link images
below to further research the concepts in this question (if desired).

Research Concepts:
Paroxysmal Supraventricular Tachycardia:

Tap flag to report any problems with this question.


Question 1314: Signs and symptoms of superior vena cava syndrome:
Choices:
1. Lower extremity edema
2. Facial edema
3. Paresthesia in lower extremities
4. Increasing abdominal girth
Answer: 2 - Facial edema
Explanations:
Superior vena cava syndrome develops from a source of pressure on the
vena cava (usually tumors) impeding it's ability to return blood to the heart
from the head and arms

Go to the next page if you knew the correct answer, or click the link images
below to further research the concepts in this question (if desired).

Research Concepts:
Superior Vena Cava Syndrome:

Tap flag to report any problems with this question.


Question 1315: A new adult patient is noted to be a Jehovah's Witness.
Which is the most appropriate response?

Choices:
1. Clearly document that he is a Jehovah's Witness in his medical record
2. Encourage him to agree to accept blood
3. Refuse to accept him as a patient
4. Plan to administer blood only if an emergency situation arises
Answer: 1 - Clearly document that he is a Jehovah's Witness in his medical
record

Explanations:
Knowing that a patient is a Jehovah's Witness has major potential
implications for his care, so it must be clearly documented.
There should be a frank discussion with Jehovah's Witness patients about
their beliefs regarding blood products and the various options that are
available.
Blood should not be administered in an emergency situation unless the
patient agrees.

Go to the next page if you knew the correct answer, or click the link images
below to further research the concepts in this question (if desired).

Research Concepts:
Jehovah's Witness Patients:

Tap flag to report any problems with this question.


Question 1316: Postoperative deep vein thrombosis prevention is best
accomplished by which answer?

Choices:
1. Sequential compression devices
2. Aspirin
3. Low molecular weight heparin
4. Warfarin
Answer: 3 - Low molecular weight heparin
Explanations:
Low molecular weight heparin has been shown to be effective in DVT
prevention.
Coumadin has more side effects and has a higher risk of intracranial
bleeding.
SCDs and aspirin alone are not sufficient for DVT prevention.

Go to the next page if you knew the correct answer, or click the link images
below to further research the concepts in this question (if desired).

Research Concepts:
Deep Venous Thrombosis Prophylaxis:

Tap flag to report any problems with this question.


Question 1317: All of the following are complications of chronic cyanosis
except:

Choices:
1. Polycythemia
2. Brain abscess
3. Cerebral neoplasms
4. Stroke
Answer: 3 - Cerebral neoplasms
Explanations:
Chronic cyanosis may be complicated by polycythemia and stroke, as well
as other complications resulting from increased blood viscosity
Brain abscesses may form due to poor venous blood flow causing increased
susceptibility to infection
Impaired cognitive function may also result

Go to the next page if you knew the correct answer, or click the link images
below to further research the concepts in this question (if desired).

Research Concepts:
Cyanosis:

Tap flag to report any problems with this question.


Question 1318: Which of the following stimulates the vagus nerve to slow
the heart rate?

Choices:
1. Heimlich maneuver
2. Bretylium
3. Valsalva maneuver
4. Mexiletine
Answer: 3 - Valsalva maneuver
Explanations:
The Valsalva maneuver stimulates the vagus nerve to slow the heart rate
It is one of the vagal maneuvers
Another vagal maneuver is the carotid massage
The carotid massage should be used with caution in older patients, as they
have increased risk of embolism

Go to the next page if you knew the correct answer, or click the link images
below to further research the concepts in this question (if desired).

Research Concepts:
Valsalva Maneuver:

Tap flag to report any problems with this question.


Question 1319: Which parameter of the arterial blood gas is not affected
when the sample is not transported on ice?

Choices:
1. pH
2. PCO2
3. PO2
4. HCO3
Answer: 4 - HCO3
Explanations:
If the ABG sample is not immediately analyzed or placed on ice, the PO2
level will decrease, the PCO2 level will be falsely elevated, pH level drops,
and HCO3 is not affected.

Go to the next page if you knew the correct answer, or click the link images
below to further research the concepts in this question (if desired).

Research Concepts:
Arterial Blood Gas:

Tap flag to report any problems with this question.


Question 1320: A 65 year old has had chest pain for the past 2 months
wherever he exercises or mows the lawn. The chest pain goes away while
resting. What is the NYHA class?

Choices:
1. Class 1
2. Class 2
3. Class 3
4. Class 4
Answer: 3 - Class 3
Explanations:
NYHA class 1 is no limitation of physical activity.
NYHA class 2 is slight limitation of physical activity. Ordinary activity
does not cause symptoms.
NYHA class 3 is moderate limitation of activity but patient is comfortable
at rest.
NYHA class 4 is inability to perform any exercise and there may be
symptoms at rest.

Go to the next page if you knew the correct answer, or click the link images
below to further research the concepts in this question (if desired).

Research Concepts:
Coronary Artery Disease:

Tap flag to report any problems with this question.


Question 1321: What is the best method to confirm the placement of lead
from dual chamber pacemaker?

Choices:
1. Echocardiogram
2. X-ray
3. CT
4. ECG
Answer: 2 - X-ray
Explanations:
A two view CXR may confirm the correct position of the dual leads

Go to the next page if you knew the correct answer, or click the link images
below to further research the concepts in this question (if desired).

Research Concepts:
Dual Chamber Pacemaker:

Tap flag to report any problems with this question.


Question 1322: After reperfusion of the heart with thrombolytics, which
of the following can occur?

Choices:
1. Hypertension
2. Fever
3. Cardiac arrhythmia
4. Nausea
Answer: 3 - Cardiac arrhythmia
Explanations:
Cardiac arrhythmias may occur during the reperfusion period

Go to the next page if you knew the correct answer, or click the link images
below to further research the concepts in this question (if desired).

Research Concepts:
Thrombolytic Therapy:

Arrhythmia:

Tap flag to report any problems with this question.


Question 1323: A patient has asymmetric chest movement and unilateral
decreased breath sounds. Which is the most likely diagnosis?

Choices:
1. Pericardial tamponade
2. Pulmonary hypertension
3. Pneumothorax
4. Pulmonary fibrosis
Answer: 3 - Pneumothorax
Explanations:
Pneumothorax is associated with decreased chest movement and breath
sounds on the ipsilateral side.

Go to the next page if you knew the correct answer, or click the link images
below to further research the concepts in this question (if desired).

Research Concepts:
Pneumothorax:

Tap flag to report any problems with this question.


Question 1324: The AV nodal artery arises from which of the following?
Choices:
1. RCA
2. LAD
3. Circumflex
4. Obtuse marginal
Answer: 1 - RCA
Explanations:
In about 90% of patients, the AV node artery arises from the distal RCA.
The origin of the AV node artery is dependent on whether the right coronary
crosses the posterior interventricular groove (crux).
In 90%, the RCA dose cross the crux. In 10%, the circumflex gives rise to
the AV node artery.

Go to the next page if you knew the correct answer, or click the link images
below to further research the concepts in this question (if desired).

Research Concepts:
Coronary Artery Anatomy:

Tap flag to report any problems with this question.


Question 1325: Which statement is false about the elephant trunk?
Choices:
1. It is a two-stage procedure
2. The distal graft has to be less than 10 cms
3. The second stage does not require cardiopulmonary bypass
4. The second stage requires at least 2 anastomoses
Answer: 4 - The second stage requires at least 2 anastomoses
Explanations:
The elephant trunk is a technique for surgery on arch and descending aortic
dissections
While performing the operation on the aortic arch during profound
hypothermic total circulatory arrest, the distal anastomosis is made not to
the distal end of the Dacron graft, but to the circular reflection produced by
invaginating the distal portion of the graft back into the proximal portion
When the distal anastomosis is complete, the distal part is pushed out of the
proximal segment, to lie within the lumen of the descending aorta. The
distal segment (the elephant trunk) should not be longer than about 10 cm,
otherwise paraplegia will develop
Three to 6 months later, the descending aorta is replaced with the elephant
trunk and only a distal anastomosis is usually necessary

Go to the next page if you knew the correct answer, or click the link images
below to further research the concepts in this question (if desired).

Research Concepts:
Dissection, Aortic:

Tap flag to report any problems with this question.


Question 1326: What is the term for a healthcare worker terminating
treatment inappropriately?

Choices:
1. Malpractice
2. Abandonment
3. Recruitment
4. None of the above
Answer: 2 - Abandonment
Explanations:
Abandonment is the term for a healthcare worker terminating treatment
inappropriately
Turning care over to less qualified personnel can also be considered
abandonment
Malpractice refers to a breach of a standard of care

Go to the next page if you knew the correct answer, or click the link images
below to further research the concepts in this question (if desired).

Research Concepts:
Abandonment:

Tap flag to report any problems with this question.


Question 1327: A 65-year-old undergoes thrombolytic therapy with tissue
plasminogen activator. During the night, he continues to bleed from the groin
puncture site. Despite manual pressure, blood continues to ooze. Which of the
following offers the best method of stopping the bleeding?

Choices:
1. Fresh frozen plasma
2. Vitamin K
3. Aminocaproic acid
4. Protamine
Answer: 3 - Aminocaproic acid
Explanations:
Aminocaproic acid is a derivative of lysine and an inhibitor of certain
proteolytic enzymes.
Aminocaproic acid is used to treat excessive fibrinolysis that may result
from an overdose of streptokinase.
Side effects of aminocaproic acid include nausea, fever, and vomiting.
Aminocaproic acid can also cause inflammation of the liver and appendix.

Go to the next page if you knew the correct answer, or click the link images
below to further research the concepts in this question (if desired).

Research Concepts:
Thrombolytic Therapy:

Tap flag to report any problems with this question.


Question 1328: Which of the following describes abuse in medicine?
Choices:
1. Wrongful practices outside medical, business or fiscal standards
2. Abuse is bad treatment toward another
3. Offensive behavior toward one unable to defend their self against the
transgression
4. All of the above
Answer: 4 - All of the above
Explanations:
Abuse in medicine and healthcare includes wrong practices outside
accepted medical, business or fiscal standards.
Abuse is the bad treatment of another wherein the party is harmed or
hindered by the abuse.
Abuse is offensive behavior toward one unable to defend their self against
the transgression.

Go to the next page if you knew the correct answer, or click the link images
below to further research the concepts in this question (if desired).

Research Concepts:
Abuse:

Tap flag to report any problems with this question.


Question 1329: Which of the following is an independent predictor of
developing a clinical event or an abnormal exercise treadmill test (ETT)?

Choices:
1. ST-segment elevation
2. Younger age
3. Prior use of aspirin
4. Negative family history of CAD
Answer: 3 - Prior use of aspirin
Explanations:
Analysis of TIMI IIIB database revealed predictors of patients who fail
medical therapy
Independent predictors of a clinical event or abnormal ETT include ST-
segment depression on qualifying ECG, history of prior angina, family
history of premature CAD, prior use of ASA or heparin, and increasing age
Patients with these characteristics could benefit from expeditious invasive
therapy

Go to the next page if you knew the correct answer, or click the link images
below to further research the concepts in this question (if desired).

Research Concepts:
Treadmill Stress Testing:

Acute Myocardial Infarction:

Tap flag to report any problems with this question.


Question 1330: What is the diagnostic imaging modality of choice for
pulmonary embolus?

Choices:
1. Chest radiograph
2. V/Q scan
3. CT angiography
4. MRI
Answer: 3 - CT angiography
Explanations:
CT angiography is the best imaging modality for stable patients with
suspected PE.
If the CT angiogram is normal, then the likelihood of a PE is very small.
CT pulmonary angiography can visualize lobar, segmental, and main
pulmonary emboli with relative ease.
V/Q scintigraphy is available for those with contrast reaction history.

Go to the next page if you knew the correct answer, or click the link images
below to further research the concepts in this question (if desired).

Research Concepts:
Pulmonary Embolism:

Tap flag to report any problems with this question.


Question 1331: A patient has a tension pneumothorax that requires
decompression. Which of the following arterial blood gases would be most
likely before this procedure?

Choices:
1. pH=7.35, PO2=40 mmHg, PCO2=34 mmHg, and bicarbonate= 10 mEq/L
2. pH=7.19, PO2=45 mmHg, PCO2=68 mmHg, and bicarbonate= 3 mEq/L
3. pH=7.60, PO2=95 mmHg, PCO2=20 mmHg, and bicarbonate= 2 mEq/L
4. pH=7.21, PO2=96 mmHg, PCO2=27 mmHg, and bicarbonate=15 mEq/L
Answer: 2 - pH=7.19, PO2=45 mmHg, PCO2=68 mmHg, and bicarbonate=
3 mEq/L

Explanations:
The ABG in the second option shows hypoxia, CO2 retention, and
respiratory acidosis

Go to the next page if you knew the correct answer, or click the link images
below to further research the concepts in this question (if desired).

Research Concepts:
Tension Pneumothorax:

Arterial Blood Gas:

Tap flag to report any problems with this question.


Question 1332: A patient is suffering from end stage renal disease. His
ECG shows peaked T waves. What is the most likely cause of this finding?

Choices:
1. Ischemia
2. Hyperkalemia
3. Hypermagnesemia
4. Acidosis
Answer: 2 - Hyperkalemia
Explanations:
T waves on ECG represent ventricular repolarization process
Hyperkalemia is a common complication of end stage renal disease
ECG shows peaked T waves in chest leads

Go to the next page if you knew the correct answer, or click the link images
below to further research the concepts in this question (if desired).

Research Concepts:
End Stage Renal Disease:

Hyperkalemia:

ECG, T Wave:

Tap flag to report any problems with this question.


Question 1333: A 61 year old male presents with chest pain and altered
mental status. You measure a blood pressure of 82/60 mmHg and pulse 44/min.
EKG shows complete AV dissociation, and he continues to be symptomatic even
after given oxygen and fluid. What is the next step?

Choices:
1. Atropine 0.5 mg IV
2. Transcutaneous pacing
3. Adenosine
4. Morphine, Aspirin, Metoprolol
Answer: 2 - Transcutaneous pacing
Explanations:
In patients with symptomatic bradycardia with type II second degree or
third degree heart block, transcutaneous pacing is indicated without delay

Go to the next page if you knew the correct answer, or click the link images
below to further research the concepts in this question (if desired).

Research Concepts:
Heart Block, Third Degree:

Tap flag to report any problems with this question.


Question 1334: A patient with hypertension, diabetes mellitus type 2, and
hyperlipidemia is evaluated. Current medications include metformin,
simvastatin, metoprolol, and aspirin. Laboratories show LDL of 90 mg/dL and
triglycerides of 300 mg/dL. Select the best management.

Choices:
1. Switch simvastatin to atorvastatin
2. Start fenofibrate
3. Add extended release niacin
4. Add ezetimibe
Answer: 3 - Add extended release niacin
Explanations:
The patient's LDL is adequately treated
Adding ezetimibe is unlikely to lower the triglycerides enough and the
addition of fenofibrate has not been shown to lower mortality
Niacin may lower triglycerides but may worsen diabetic control. Another
option is omega 3 fish oils
Fenofibrate will lower triglycerides but the patient should be watched for
myopathy

Go to the next page if you knew the correct answer, or click the link images
below to further research the concepts in this question (if desired).

Research Concepts:
Hyperlipidemia:

Tap flag to report any problems with this question.


Question 1335: A patient with a long history of a congenital heart disorder
is found to have an increased Lovibond angle. This usually indicates that he may
have developed?

Choices:
1. Pulmonary hypertension
2. Clubbing
3. Endocarditis
4. Aortic regurgitation
Answer: 2 - Clubbing
Explanations:
Lovibond angle is the angle between the nail plate base and the adjacent
skin
This angle is usually less than 165 degrees
If the angle is more, it is a sign of clubbing

Go to the next page if you knew the correct answer, or click the link images
below to further research the concepts in this question (if desired).

Research Concepts:
Congenital Heart Disease:

Nails, Clubbing:

Tap flag to report any problems with this question.


Question 1336: Which antihypertensive drugs are known to cause salt and
water retention, particularly pedal edema?

Choices:
1. Beta blockers
2. ACEI
3. Calcium channel blockers
4. Vasodilators
Answer: 3 - Calcium channel blockers
Explanations:
Calcium channel blockers can cause salt and water retention, constipation
and facial flushing

Go to the next page if you knew the correct answer, or click the link images
below to further research the concepts in this question (if desired).

Research Concepts:
Calcium Channel blockers:

Tap flag to report any problems with this question.


Question 1337: A thin male is found to have an abnormal pulsation of his
trachea. When the doctor presses on the thyroid cartilage to the left, strong
pulsations are felt. What is the likely cause?

Choices:
1. Superior thyroid artery aneurysm
2. Tracheo-innominate artery fistula
3. Aneurysm of aortic arch
4. Thyroid malignancy
Answer: 3 - Aneurysm of aortic arch
Explanations:
Cardarelli sign is an abnormal pulsation of the trachea and may be found in
patients with aneurysms of the aortic arch.

Go to the next page if you knew the correct answer, or click the link images
below to further research the concepts in this question (if desired).

Research Concepts:
Thoracic Aortic Aneurysm:

Tap flag to report any problems with this question.


Question 1338: Which of the following statements regarding preoperative
evaluations is correct?

Choices:
1. A patient with a previous coronary bypass graft 2 years earlier should
undergo cardiac stress testing before clearance, regardless of the presence
of cardiac symptoms
2. Urine pregnancy testing should be done for women of childbearing age
3. Coagulation studies should be included in the laboratory assessment of all
surgical candidates
4. Patients who have had angioplasty within 6 months are not required to have
further cardiology assessment
Answer: 2 - Urine pregnancy testing should be done for women of
childbearing age

Explanations:
The most important preoperative evaluation is the history. It was
commonplace to do extensive labs, radiographs, and cardiac evolutions in
the past.
Many of these investigations have been found to have no effect on
outcomes.
The most important test is a urine pregnancy test, as all women of
childbearing years should be considered pregnant until proven otherwise.
Many patients deny sexual activity but turn out to be pregnant.
Other specific tests should be guided by patient history, age, and facility
requirements.

Go to the next page if you knew the correct answer, or click the link images
below to further research the concepts in this question (if desired).

Research Concepts:
Preoperative Testing:

Tap flag to report any problems with this question.


Question 1339: A 71-year-old undergoes surgery for an abscess that is
complicated. The surgery is difficult and the abscess is undrained. She later
develops fever, hematemesis, and melena. She also develops a purpuric skin
rash. Laboratory tests reveal thrombocytopenia, prolonged PT, and decreased
fibrinogen. What is the most important step in this patient's treatment?

Choices:
1. Administer heparin
2. Drain abscess
3. Explore patient
4. Change antibiotics
Answer: 2 - Drain abscess
Explanations:
The only definitive treatment for disseminated intravascular coagulation is
treatment of the underlying cause.
In the above patient, the risk from surgery is very high in the presence of
disseminated intravascular coagulation. Thus, the abscess should be drained
percutaneously with the help of a CT scan.
One may attempt to give platelets and fresh frozen plasma when there is
active bleeding, but these are only temporizing factors.
Disseminated intravascular coagulation results in low fibrinogen levels and
elevated levels of fibrin degradation products.

Go to the next page if you knew the correct answer, or click the link images
below to further research the concepts in this question (if desired).

Research Concepts:
Disseminated Intravascular Coagulation:

Abscess:

Tap flag to report any problems with this question.


Question 1340: Which of the following is the most common cause of
minor postoperative bleeding in a healthy patient who suffers trauma?

Choices:
1. Poor surgical technique
2. Infection
3. Platelet dysfunction
4. Renal failure
Answer: 1 - Poor surgical technique
Explanations:
In an elective case, the most common cause of postoperative bleeding is
poor surgical technique. This may include failure to tie down blood vessels
or failure to cauterize bleeders.
If the bleeding is severe, one should always check coagulation parameters
and obtain a family history of bleeding.
Patients who continue to bleed with a normal international normalized ratio
and normal partial thromboplastin time may have impaired platelet activity.
If the activated partial thromboplastin time is prolonged and the
international normalized ratio is normal, the most likely cause is heparin.
This can be treated with protamine or fresh frozen plasma.

Go to the next page if you knew the correct answer, or click the link images
below to further research the concepts in this question (if desired).

Research Concepts:
Postoperative Complications:

Tap flag to report any problems with this question.


Question 1341: Refractory ventricular fibrillation is sometimes treated
with which of the following?

Choices:
1. Adenosine
2. Vasopressin
3. Epinephrine
4. Phenylephrine
Answer: 2 - Vasopressin
Explanations:
Vasopressin is the agent of choice for refractory ventricular fibrillation.

Go to the next page if you knew the correct answer, or click the link images
below to further research the concepts in this question (if desired).

Research Concepts:
Ventricular Fibrillation:

Tap flag to report any problems with this question.


Question 1342: The cardiac structure closest to the esophagus is?
Choices:
1. Right atrium
2. Left atrium
3. Right ventricle
4. Left ventricle
Answer: 2 - Left atrium
Explanations:
Left atrium is immediately anterior to the esophagus.
In the past, left atrial enlargement was evaluated with a barium esophagram,
seen as an extrinsic compression of the anterior esophagus.
The esophagus is posterior to the heart and the left atrium is the most
posterior cardiac chamber.
The right ventricle is the most ANTERIOR cardiac chamber. The right
atrium and left ventricle are right and left border-forming cardiac chambers,
respectively.

Go to the next page if you knew the correct answer, or click the link images
below to further research the concepts in this question (if desired).

Research Concepts:
Thoracic Anatomy:

Tap flag to report any problems with this question.


Question 1343: Why are patients who are bed ridden asked to move their
legs?

Choices:
1. Good exercise for the patient
2. Takes their mind off of their medical condition
3. Prevents deep vein thrombosis of the legs
4. Prevents bed sores
Answer: 3 - Prevents deep vein thrombosis of the legs
Explanations:
This activity is taught to the patient to promote venous return.
The muscular action aids in venous return and prevents venous stasis in the
lower extremities.

Go to the next page if you knew the correct answer, or click the link images
below to further research the concepts in this question (if desired).

Research Concepts:
Deep Venous Thrombosis Prophylaxis:

Tap flag to report any problems with this question.


Question 1344: Primary pulmonary hypertension is:
Choices:
1. An indicator for a heart and lung transplant
2. A complication of hypertrophic cardiomyopathy
3. Caused by a viral infection
4. A common cause of dilated cardiomyopathy
Answer: 1 - An indicator for a heart and lung transplant
Explanations:
A heart-lung transplant is often necessary in a patient with primary
pulmonary hypertension.

Go to the next page if you knew the correct answer, or click the link images
below to further research the concepts in this question (if desired).

Research Concepts:
Pulmonary Hypertension:

Heart-Lung Transplantation:

Tap flag to report any problems with this question.


Question 1345: An elderly patient in the ICU has developed MRSA. It is
decided to treat the patient but there is no vancomycin available. The next drug
of choice is:

Choices:
1. Minocycline
2. Bacitracin
3. Amikacin
4. Cefotetan
Answer: 1 - Minocycline
Explanations:
Intravenous vancomycin is the drug of choice for MRSA.
There are sporadic cases of vancomycin-resistant MRSA.
Linezolid is an effective alternative but resistance has been seen.
Daptomycin can be used in adults and is another option.

Go to the next page if you knew the correct answer, or click the link images
below to further research the concepts in this question (if desired).

Research Concepts:
MRSA Infection:

Tap flag to report any problems with this question.


Question 1346: A male having a myocardial infarction requires emergent
endotracheal intubation. During intubation the patient has massive amount of
aspiration and develops a pneumonia. Which should be administered to ensure
proper antibiotic coverage for anaerobes?

Choices:
1. IV gentamicin
2. IV ceftriaxone
3. IV sulfonamide
4. IV clindamycin
Answer: 4 - IV clindamycin
Explanations:
Aspiration alone is not an indication for antibiotics.
If pneumonitis persists 48 hours, antibiotics should be started. In order to
cover anaerobic bacteria in aspiration pneumonia, clindamycin is used.
Often this is combined with another antibiotic, commonly a third generation
cephalosporin.
Routine use of corticosteroids has not been shown to improve survival.

Go to the next page if you knew the correct answer, or click the link images
below to further research the concepts in this question (if desired).

Research Concepts:
Pneumonia, Aspiration:

Tap flag to report any problems with this question.


Question 1347: After administration of enalapril, which substance
increases in blood?

Choices:
1. Angiotensin I
2. Angiotensin II
3. Bradykinin
4. Angiotensinogen
Answer: 3 - Bradykinin
Explanations:
Bradykinin is metabolized by ACE.

Go to the next page if you knew the correct answer, or click the link images
below to further research the concepts in this question (if desired).

Research Concepts:
Angiotensin Converting Enzyme Inhibitors (ACEI):

Tap flag to report any problems with this question.


Question 1348: Which of the following is not true of beta-blockers?
Choices:
1. Atenolol is useful in the treatment of patients after a myocardial infarction
2. Metoprolol can depress the cardiac output
3. Labetalol can be used to treat malignant hypertension
4. Esmolol is an effective agent for ventricular arrhythmias
Answer: 4 - Esmolol is an effective agent for ventricular arrhythmias
Explanations:
Esmolol is a very short-acting beta-blocker, administered intravenously for
atrial arrhythmias and hypertension.
The majority of beta-blockers are commonly used as prophylactic agents in
patients who have had a myocardial infarction.
Beta-blockers can only suppress atrial arrhythmias.
Use of beta-blockers should be cautioned in patients with poor ejection
fraction, as these agents are negative inotropes.

Go to the next page if you knew the correct answer, or click the link images
below to further research the concepts in this question (if desired).

Research Concepts:
Beta-Blockers:

Tap flag to report any problems with this question.


Question 1349: How much greater is mortality secondary to acute
myocardial infarction in patients with diabetes mellitus compared to non-
diabetics?

Choices:
1. 1.5 times greater
2. 2 times greater
3. 2.5 times greater
4. 3 times greater
Answer: 2 - 2 times greater
Explanations:
The mortality rate of diabetic MI is twice of a nondiabetic patient

Go to the next page if you knew the correct answer, or click the link images
below to further research the concepts in this question (if desired).

Research Concepts:
Diabetes Mellitus:

Tap flag to report any problems with this question.


Question 1350: Which is NOT an independent risk factor for progression
of atherosclerosis in saphenous vein grafts?

Choices:
1. Prior myocardial infarction
2. High LDL-C
3. Low ejection fraction
4. High HDL-C
Answer: 4 - High HDL-C
Explanations:
There are 12 independent prognostic factors for atherosclerosis progression
in saphenous vein grafts.
These include years post-SVG placement, maximum stenosis of the graft at
baseline angiography, small minimum graft diameter, prior myocardial
infarction, the moderate LDL-C lowering strategy, hypertriglyceridemia,
low HDL-C, high LDL-C, low ejection fraction, high mean arterial
pressure, male, and current smoking.
High HDL-C is a good prognostic indicator.

Go to the next page if you knew the correct answer, or click the link images
below to further research the concepts in this question (if desired).

Research Concepts:
Saphenous Vein Grafts:

Atherosclerosis:

Tap flag to report any problems with this question.


Question 1351: What is the standard imaging tool for abdominal aortic
aneurysms (AAAs)?

Choices:
1. Computed tomography (CT)
2. Magnetic resonance imaging (MRI)
3. X-ray
4. Ultrasound (U/S)
Answer: 4 - Ultrasound (U/S)
Explanations:
Ultrasound is the standard imaging tool for AAAs
Surgery is the treatment of choice for AAAs
The US Preventative Services Task Force recommends U/S screening for
men aged 65-75 years who have ever had a history of smoking
AAAs predominantly affect elderly white men

Go to the next page if you knew the correct answer, or click the link images
below to further research the concepts in this question (if desired).

Research Concepts:
Abdominal Aortic Aneurysm:

Tap flag to report any problems with this question.


Question 1352: Which of the following is a partial morphine agonist-
antagonist?

Choices:
1. Naloxone
2. Meperidine
3. Methanol
4. Nalbuphine
Answer: 4 - Nalbuphine
Explanations:
Nalbuphine is an opioid with mixed agonist antagonist-analgesic properties
Nalbuphine activates kappa and weakly blocks mu receptors
Nalbuphine is an effective analgesic, but with lower abuse potential and
less respiratory depressant effects

Go to the next page if you knew the correct answer, or click the link images
below to further research the concepts in this question (if desired).

Research Concepts:
Nalbuphine:

Tap flag to report any problems with this question.


Question 1353: Select the leading cause of death of the elderly in the
United States.

Choices:
1. Cancer
2. Heart disease
3. Suicide
4. Stroke
Answer: 2 - Heart disease
Explanations:
In order, the leading causes of death in the elderly in the US are heart
disease, cancer, cerebrovascular disease, chronic lower respiratory diseases,
and Alzheimer's disease

Go to the next page if you knew the correct answer, or click the link images
below to further research the concepts in this question (if desired).

Research Concepts:
Death Statistics:

Tap flag to report any problems with this question.


Question 1354: Which is not increased in a decompensated left ventricle?
Choices:
1. Wall tension
2. Ventricular contractility
3. Ejection fraction
4. End diastolic volume
Answer: 3 - Ejection fraction
Explanations:
Decompensation is characterized by suboptimal ejection fraction
Result in high EDV and ventricular wall tension

Go to the next page if you knew the correct answer, or click the link images
below to further research the concepts in this question (if desired).

Research Concepts:
Heart Failure, Congestive:

Tap flag to report any problems with this question.


Question 1355: Which of the following will be most helpful in the
differentiation of cardiac versus pulmonary cyanosis in a neonate?

Choices:
1. Chest x-ray
2. Room air ABG
3. Hyperoxia test
4. ECG
Answer: 3 - Hyperoxia test
Explanations:
The hyperoxia test is most likely to differentiate cyanotic heart disease from
pulmonary disease
The pO2 is measured during the administration of 100% oxygen
With lung disease there is usually some change from the baseline pO2
With cyanotic heart disease, due to fixed right-to-left shunting, the pO2
rarely increases above 100 mm Hg

Go to the next page if you knew the correct answer, or click the link images
below to further research the concepts in this question (if desired).

Research Concepts:
Cyanosis:

Tap flag to report any problems with this question.


Question 1356: Testing which of the following requires the addition of
tissue thromboplastin?

Choices:
1. Extrinsic pathway
2. Intrinsic pathway
3. Von Willebrand factor
4. Platelet aggravation
Answer: 1 - Extrinsic pathway
Explanations:
The extrinsic pathway is the only clotting test that requires the addition of
tissue thromboplastin.

Go to the next page if you knew the correct answer, or click the link images
below to further research the concepts in this question (if desired).

Research Concepts:
Coagulation Pathways:

Tap flag to report any problems with this question.


Question 1357: What heart sound would one expect to hear in a patient
with congestive heart failure?

Choices:
1. A systolic murmur
2. A diastolic murmur
3. S3 heart sound
4. A snap in early diastole
Answer: 3 - S3 heart sound
Explanations:
An S3 heart sound is heard during mid-diastole.
A left side S3 is best heard during expiration at the left ventricular border.
A right side S3 is best heard during inspiration at the left sternal border.

Go to the next page if you knew the correct answer, or click the link images
below to further research the concepts in this question (if desired).

Research Concepts:
Heart Failure, Congestive:

Tap flag to report any problems with this question.


Question 1358: Which of the following is an effect of digoxin, when given
in therapeutic doses?

Choices:
1. It slows the atrial rate in atrial fibrillation
2. It enhances atrioventricular conduction
3. It slows the atrial rate in sinus tachycardia
4. It slows the ventricular rate in atrial fibrillation
Answer: 4 - It slows the ventricular rate in atrial fibrillation
Explanations:
Digoxin acts at the AV node and slows the ventricular response rate in atrial
fibrillation.

Go to the next page if you knew the correct answer, or click the link images
below to further research the concepts in this question (if desired).

Research Concepts:
Atrial Fibrillation:

Digoxin:

Tap flag to report any problems with this question.


Question 1359: A dressing that adheres to the incisional site may be
removed by which of the following methods?

Choices:
1. Moistening the dressing with sterile saline
2. Pulling the dressing off quickly
3. Placing a warm compress over the dressing
4. Finding an edge and slowly working the dressing free
Answer: 1 - Moistening the dressing with sterile saline
Explanations:
Moistening the dressing softens the adhesion and aids in removal, thus
preventing damage to the incision and the surrounding tissue.

Go to the next page if you knew the correct answer, or click the link images
below to further research the concepts in this question (if desired).

Research Concepts:
Wound Care:

Tap flag to report any problems with this question.


Question 1360: Select the conditions that would cause elevated aPTT
(activated partial thromboplastin time).

Choices:
1. Heparin, thrombocytopenia, and aspirin
2. Warfarin, thrombocytopenia, and hemophilia A
3. Heparin, hemophilia A, and von Willebrand factor deficiency
4. Warfarin, aspirin, and von Willebrand factor deficiency
Answer: 3 - Heparin, hemophilia A, and von Willebrand factor deficiency
Explanations:
aPTT detects abnormalities of both the common and intrinsic coagulation
pathways
Heparin elevates aPTT by binding to antithrombin III, causing its
activation, and inactivating thrombin and factor Xa
Von Willebrand factor deficiency binds to factor VIII preventing its
deactivation
Hemophilia A is a deficiency of factor VIII

Go to the next page if you knew the correct answer, or click the link images
below to further research the concepts in this question (if desired).

Research Concepts:
Partial Thromboplastin Time:

Tap flag to report any problems with this question.


Question 1361: Which of the following best describes aminoglycoside
antibiotics?

Choices:
1. They have a wide spectrum of activity against many Gram-negative rods
2. They have a high ratio of therapeutic levels to toxic levels
3. They have exceptional coverage against Gram-positive organisms
4. Most are administered orally
Answer: 1 - They have a wide spectrum of activity against many Gram-
negative rods

Explanations:
Aminoglycoside antibiotics have very poor activity against anaerobic and
Gram-positive organisms.
These antibiotics are usually administered parenterally and have a very
narrow range for toxicity.
Levels of most aminoglycosides need to be monitored.
These antibiotics have good Gram-negative bacteria coverage.

Go to the next page if you knew the correct answer, or click the link images
below to further research the concepts in this question (if desired).

Research Concepts:
Aminoglycosides:

Antibiotics:

Tap flag to report any problems with this question.


Question 1362: Which of the following is NOT a commonly used drug in
the treatment of CHF?

Choices:
1. Captopril
2. Furosemide
3. Atenolol
4. Minoxidil
Answer: 4 - Minoxidil
Explanations:
Minoxidil is an alpha antagonist and has no role in the treatment of CHF.
Minoxidil is used in the treatment of hypertension and has an odd side
effect of excess hair growth.

Go to the next page if you knew the correct answer, or click the link images
below to further research the concepts in this question (if desired).

Research Concepts:
Heart Failure, Congestive:

Tap flag to report any problems with this question.


Question 1363: You are to administer procainamide (Pronestyl-SR) 500
mg by mouth. For which of the following conditions will you withhold the
dosage and notify the physician?

Choices:
1. Serum glucose of 115 mg/dl
2. Blood pressure (systolic) of 138 mmHg
3. Apical pulse of 42 beats per minute
4. Blood pressure (diastolic) of 72 mmHg
Answer: 3 - Apical pulse of 42 beats per minute
Explanations:
Pronestyl-SR should be withheld and the physician notified if the patient's
apical pulse is 60 bpm or less. Procainamide (Pronestyl-SR) is an anti-
arrhythmic drug used to treat both atrial and ventricular arrhythmias.
Signs of toxicity include electrocardiogram (ECG) changes, hypotension,
and bradyarrhythmias (heart rate of 60 bpm or less). The normal blood
glucose level is 60 to 115 mg /dl. Since the patient's serum glucose is 115
mg/dl, there is no reason to take any action.
A systolic blood pressure of 138 mmHg is expected in this situation. A
diastolic blood pressure of 72 mmHg is also expected in this situation.
Pregnancy Category: C Patient Need: Physiological Integrity

Go to the next page if you knew the correct answer, or click the link images
below to further research the concepts in this question (if desired).

Research Concepts:
Sinus Bradycardia:

Antiarrhythmic Medication:

Tap flag to report any problems with this question.


Question 1364: A 16 year old girl was found to have coarctation of the
aorta. With which of the following lesions is this most likely associated?

Choices:
1. Atrial septal defect
2. Bicuspid aortic valve
3. Pulmonary valve regurgitation
4. Lower extremity hypertension
Answer: 2 - Bicuspid aortic valve
Explanations:
Coarctation of the aorta is a rare heart defect accounting for 5-8% of all
congenital heart defects.
It may occur as an isolated defect or in association with other lesions, most
commonly ventricular septal defect and bicuspid aortic valve.
Coarctation of the aorta often presents with upper extremity hypertension
and radial-femoral pulse delay.
The disorder can be surgically treated but endovascular techniques are also
being applied.

Go to the next page if you knew the correct answer, or click the link images
below to further research the concepts in this question (if desired).

Research Concepts:
Aortic Coarctation:

Tap flag to report any problems with this question.


Question 1365: What is the next best step for asymptomatic premature
ventricular contractions post acute myocardial infarction?

Choices:
1. Observation
2. Metoprolol
3. Nifedipine
4. Catheterization
Answer: 1 - Observation
Explanations:
CAST studies demonstrated no treatment is necessary for asymptomatic
PVC after MI

Go to the next page if you knew the correct answer, or click the link images
below to further research the concepts in this question (if desired).

Research Concepts:
Acute Myocardial Infarction:

Premature Ventricular Contraction:

Tap flag to report any problems with this question.


Question 1366: What is the initial step in management of a patient with
newly diagnosed primary pulmonary hypertension?

Choices:
1. Prescribe inhaled corticosteroids
2. Evaluate vasoreactivity
3. Order an echocardiogram
4. Prescribe long acting calcium channel blockers
Answer: 2 - Evaluate vasoreactivity
Explanations:
Primary pulmonary hypertension, PPH, is defined as elevated pulmonary
artery pressure with no discernible cause and is most prevalent in women of
child-bearing age.
Vasoreactivity is evaluated by assessing a patient's response to an IV or
inhaled vasodilator challenge.
If the patient responds then an oral vasodilator challenge is performed, with
a catheter in the pulmonary artery to determine the dose, which causes a
20% reduction in pulmonary artery pressure.
Patients with an acute vasodilator response have a better prognosis on
calcium channel blockers only. They constitute about 20% of those with
PPH. Phosphodiesterase inhibitors are also used to treat this condition.

Go to the next page if you knew the correct answer, or click the link images
below to further research the concepts in this question (if desired).

Research Concepts:
Pulmonary Hypertension, Primary:

Tap flag to report any problems with this question.


Question 1367: Which of the following medications has not been noted to
improve long-term survival in patients with acute myocardial infarction?

Choices:
1. Captopril
2. Metoprolol
3. Spironolactone
4. Diltiazem
Answer: 4 - Diltiazem
Explanations:
Beta-blockers are known to improve long-term survival in patients with
myocardial infraction
Propranolol and metoprolol are common beta-blockers used
Spironolactone is another drug associated with good long-term survival
Calcium channel blockers have not been noted to improve survival

Go to the next page if you knew the correct answer, or click the link images
below to further research the concepts in this question (if desired).

Research Concepts:
Acute Myocardial Infarction:

Tap flag to report any problems with this question.


Question 1368: Select the patient with the clearest indication and most
probable favorable outcome for surgery for mitral regurgitation.

Choices:
1. A 70-year-old female without symptoms, ejection fraction of 45%, and left-
ventricular end-systolic dimension of 45 mm
2. A 68-year-old asymptomatic male with new atrial fibrillation, ejection
fraction of 60%, and end-systolic dimension of 40 mm
3. A 57-year-old female with an ejection fraction of 25%, NYHA class III
symptoms, and pulmonary hypertension
4. A 52-year-old man with an ejection fraction of 25%, NYHA class III
symptoms, and a left-ventricular end-systolic dimension of 60 mm
Answer: 1 - A 70-year-old female without symptoms, ejection fraction of
45%, and left-ventricular end-systolic dimension of 45 mm

Explanations:
The decision for mitral valve replacement is based on ventricular size, LV
function, and complications of chronic mitral regurgitation
Pulmonary hypertension and atrial fibrillation are such complications
Ejection fraction of <30% or end systolic dimension greater than 55 mm
can lead to incomplete recovery and reduced survival
Medical therapy is not effective so surgery should be considered if there is
greater than a 90% chance of success

Go to the next page if you knew the correct answer, or click the link images
below to further research the concepts in this question (if desired).

Research Concepts:
Mitral Regurgitation:

Tap flag to report any problems with this question.


Question 1369: Which of the following is not a common symptoms of
acute digoxin toxicity?

Choices:
1. Ringing in the ears
2. Changes in vision
3. Palpitations
4. Nausea and vomiting
Answer: 1 - Ringing in the ears
Explanations:
Nausea, vomiting, and changes in vision are common symptoms of acute
digoxin toxicity.
Ventricular and atrial arrhythmias may also manifest with palpitation.
Ringing in the ears is common symptom of aspirin toxicity.
Electrolyte disturbances frequently cause digoxin toxicity.

Go to the next page if you knew the correct answer, or click the link images
below to further research the concepts in this question (if desired).

Research Concepts:
Digoxin Toxicity:

Tap flag to report any problems with this question.


Question 1370: Which of the following is the anticoagulant that inhibits
cyclooxygenase?

Choices:
1. Prednisone
2. Warfarin
3. Heparin
4. Aspirin
Answer: 4 - Aspirin
Explanations:
The most potent mediators of inflammation are prostaglandins.
Prostaglandins are derived from arachidonic acid and require the enzyme
cyclooxygenase (COX).
COX is typically inhibited by non-steroidal anti-inflammatory drugs like
aspirin. The inhibition by aspirin is not reversible.
There are three subtypes of COX. COX-1 makes prostaglandins. COX-2 is
induced by mitogens, endotoxins, and cytokines, which produce
prostaglandins in inflammation. The function of COX-3 is not yet
understood.

Go to the next page if you knew the correct answer, or click the link images
below to further research the concepts in this question (if desired).

Research Concepts:
Antiplatelet Medications:

Tap flag to report any problems with this question.


Question 1371: In a patient with tetralogy of Fallot, which of the
following heart chambers would be expected to be hypertrophied?

Choices:
1. Left ventricle
2. Right ventricle
3. Left atrium
4. Right atrium
Answer: 2 - Right ventricle
Explanations:
In TOF there is pulmonary atresia and the right ventricle is unable to pump
the blood into the pulmonary circulation.
Over time, this leads to right ventricular hypertrophy.
Echo can be used to confirm the thickness of the right ventricle.

Go to the next page if you knew the correct answer, or click the link images
below to further research the concepts in this question (if desired).

Research Concepts:
Tetralogy Of Fallot:

Tap flag to report any problems with this question.


Question 1372: Which of the following is not a sign of morphine
overdose?

Choices:
1. Meiotic pupils
2. Shallow respiration
3. Itching
4. Seizures
Answer: 4 - Seizures
Explanations:
Morphine has many side effects but it does not cause seizures

Go to the next page if you knew the correct answer, or click the link images
below to further research the concepts in this question (if desired).

Research Concepts:
Morphine Overdose:

Tap flag to report any problems with this question.


Question 1373: What cardiac chamber pressure is best correlated with the
CVP value?

Choices:
1. Right atrium
2. Right ventricle
3. Left atrium
4. Left ventricle
Answer: 1 - Right atrium
Explanations:
A normal CVP is less than 6 mmHg
It is the a measurement of the right atrium pressure

Go to the next page if you knew the correct answer, or click the link images
below to further research the concepts in this question (if desired).

Research Concepts:
Cardiac Physiology:

Central Venous Pressure:

Tap flag to report any problems with this question.


Question 1374: What is the term for a blister larger than 0.5 cm in
diameter?

Choices:
1. Vesicle
2. Bulla
3. Nodule
4. Wheal
Answer: 2 - Bulla
Explanations:
By definition, a bulla.

Go to the next page if you knew the correct answer, or click the link images
below to further research the concepts in this question (if desired).

Research Concepts:
Bulla:

Tap flag to report any problems with this question.


Question 1375: Which is not usually heard in aortic regurgitation?
Choices:
1. Austin Flint murmur
2. High pitched diastolic blowing decrescendo murmur at the left sternal
border
3. Holosystolic murmur across precordium radiating to the left axilla
4. Mid-diastolic rumble at cardiac apex
Answer: 3 - Holosystolic murmur across precordium radiating to the left
axilla

Explanations:
An Austin Flint murmur, heard in aortic regurgitation, is a high pitched
diastolic blowing decrescendo murmur at the left sternal border
A mid diastolic rumble can also be heard at the cardiac apex in aortic
regurgitation
A holosystolic murmur across the precordium radiating to the axilla is a
mitral insufficiency murmur

Go to the next page if you knew the correct answer, or click the link images
below to further research the concepts in this question (if desired).

Research Concepts:
Aortic Regurgitation:

Tap flag to report any problems with this question.


Question 1376: Which is true regarding diabetes mellitus (DM) and
myocardial infarction (MI)?

Choices:
1. DM has no affect on risk of MI
2. Diabetics on insulin are the only diabetic subgroup at risk for MI
3. Risk of MI is elevated by DM, but DM has no affect on outcomes post-MI
4. Risk of MI is elevated by DM and equivalent to having a prior history of
MI
Answer: 4 - Risk of MI is elevated by DM and equivalent to having a prior
history of MI

Explanations:
DM elevates risk of MI and is considered a cardiovascular equivalent
DM is a major risk factor for coronary artery disease
1-year mortality in this group as high as 50%
Diabetics on insulin are no more likely at risk for MI than those not on
insulin

Go to the next page if you knew the correct answer, or click the link images
below to further research the concepts in this question (if desired).

Research Concepts:
Acute Myocardial Infarction:

Diabetes Mellitus:

Tap flag to report any problems with this question.


Question 1377: Which of the following can decrease digoxin-induced
arrhythmias?

Choices:
1. Calcium
2. Sodium
3. Magnesium
4. Chromium
Answer: 3 - Magnesium
Explanations:
Magnesium is now felt to be an important ion in the genesis of arrhythmias
induced by digoxin.
Magnesium is also used to treat Torsades.
Occasionally, potassium depletion may not be reversed until the magnesium
deficiency has first been corrected.

Go to the next page if you knew the correct answer, or click the link images
below to further research the concepts in this question (if desired).

Research Concepts:
Arrhythmias:

Digoxin:

Tap flag to report any problems with this question.


Question 1378: A 72 year old female with a long history of hypertension
on lisinopril and presents with dyspnea. She is diagnosed with congestive heart
failure (CHF). Cardiac exam is remarkable for an S4. Lungs show bibasilar
rales. Chest radiograph shows mild pulmonary congestion. ECG is remarkable
for mild LVH. Echocardiogram shows mild LVH with ejection fraction (EF) of
60%. What is the cause of this patient's CHF?

Choices:
1. Systolic dysfunction
2. Diastolic dysfunction
3. Cardiac arrhythmia
4. Valvular heart disease
Answer: 2 - Diastolic dysfunction
Explanations:
Most patients with CHF have systolic dysfunction with EF <40%
40 to 50% of patients have diastolic dysfunction and often have normal EF
Causes of CHF should investigated including ischemic heart disease,
thyroid disease, and valvular heart disease
This patient most likely has CHF secondary to hypertension but the
possibility of CAD should be considered\

Go to the next page if you knew the correct answer, or click the link images
below to further research the concepts in this question (if desired).

Research Concepts:
Diastolic Dysfunction:

Tap flag to report any problems with this question.


Question 1379: What is the pathology most often seen in benign tumors of
the heart?

Choices:
1. Sarcoma
2. Myxoma
3. Rhabdomyoma
4. Fibroma
Answer: 2 - Myxoma
Explanations:
40 to 50% of primary cardiac tumors are myxomas
Fibromas are rare and found most often in infants
Sarcoma is the most common malignancy of the heart
Cardiac rhabdomyoma is the most common tumor of the heart in children

Go to the next page if you knew the correct answer, or click the link images
below to further research the concepts in this question (if desired).

Research Concepts:
Atrial Myxoma:

Tap flag to report any problems with this question.


Question 1380: In a patient with structural heart disease and a history of
nonsustained ventricular tachycardia (VT), what is the next best step in
management?

Choices:
1. Implantable cardiac defibrillator (ICD)
2. Coronary angiogram
3. Beta-blocker
4. Pacemaker
Answer: 1 - Implantable cardiac defibrillator (ICD)
Explanations:
An ICD is indicated in a patient with structural heart disease and history of
VT
A high risk for sudden cardiac death exists in those with a history of VT and
structural heart disease
Reassurance and no intervention are recommended in a patient with no
structural heart disease and nonsustained VT
Coronary angiogram nor pacemaker are indicated

Go to the next page if you knew the correct answer, or click the link images
below to further research the concepts in this question (if desired).

Research Concepts:
Ventricular Tachycardia:

Pacemaker, Automatic Internal Cardiac Defibrillator:

Tap flag to report any problems with this question.


Question 1381: What is the mechanism of action of aspirin?
Choices:
1. Reversible COX agonist
2. Reversible COX inhibitor
3. Irreversible COX agonist
4. Irreversible COX inhibitor
Answer: 4 - Irreversible COX inhibitor
Explanations:
Aspirin is an irreversible nonselective COX inhibitor
Effective for about 7 days

Go to the next page if you knew the correct answer, or click the link images
below to further research the concepts in this question (if desired).

Research Concepts:
Aspirin:

Tap flag to report any problems with this question.


Question 1382: Which of the following is NOT a disadvantage of
retrograde cardioplegia?

Choices:
1. Potential for injury to coronary sinus
2. Inadequate right ventricular cooling
3. Inadequate capillary nutrient perfusion
4. Can be used as a sole method for cardiac protection
Answer: 4 - Can be used as a sole method for cardiac protection
Explanations:
If only retrograde cardioplegia is used, the right ventricle is not protected.
The patient will have severe right heart ischemia and failure if only
retrograde technique is used.

Go to the next page if you knew the correct answer, or click the link images
below to further research the concepts in this question (if desired).

Research Concepts:
Cardioplegia:

Tap flag to report any problems with this question.


Question 1383: Which of the following is NOT a true statement about the
diagnosis of acute myocardial infarction?

Choices:
1. Esophageal spasm can cause ST-segment depression or T-wave inversion
2. The CPK-MB rises within 1 to 2 hours after significant myocardial damage
3. New Q waves or ST-segment elevation have a predictive value of 75% and
specificity of 90%
4. CPK-MB elevation is more specific but less sensitive than total CK
elevation
Answer: 2 - The CPK-MB rises within 1 to 2 hours after significant
myocardial damage

Explanations:
Serum creatine phosphokinase (CPK) rises 4 to 8 hours after cellular
damage.

Go to the next page if you knew the correct answer, or click the link images
below to further research the concepts in this question (if desired).

Research Concepts:
Myocardial Infarction, Acute:

Tap flag to report any problems with this question.


Question 1384: Why is dopamine preferred over dobutamine for treatment
of post myocardial infarction cardiogenic shock?

Choices:
1. Dopamine has more inotropic effect than dobutamine
2. Dopamine is not associated with hypotension
3. Dopamine reduces pulmonary capillary wedge pressure
4. Dopamine decreases peripheral resistance
Answer: 2 - Dopamine is not associated with hypotension
Explanations:
High dose of dopamine has alpha agonist property
This helps to counteract the hypotension in cardiogenic shock
Dopamine actually causes increased pulmonary capillary wedge pressure
Dobutamine has vasodilator properties and can only be used if there is no
hypotension

Go to the next page if you knew the correct answer, or click the link images
below to further research the concepts in this question (if desired).

Research Concepts:
Dopamine:

Cardiogenic Shock:

Tap flag to report any problems with this question.


Question 1385: Select the choice which is FALSE concerning a co-morbid
condition.

Choices:
1. There can be only one admitting or existing diagnosis
2. A medical condition existing simultaneously with another condition
3. A co-morbid condition is a predictor of at least one day of extended LOS
4. A condition or diagnosis documented at admission aside from the admitting
diagnosis.
Answer: 1 - There can be only one admitting or existing diagnosis
Explanations:
Patients may present with multiple diagnosis and conditions.
A co-morbid is a predictor of at least one day of extended LOS.
Co-morbid conditions may include two or more disorders existing
simultaneously.
A co-morbid condition or diagnosis documented at admission aside from
the admitting diagnosis.

Go to the next page if you knew the correct answer, or click the link images
below to further research the concepts in this question (if desired).

Research Concepts:
Co-morbid Condition:

Tap flag to report any problems with this question.


Question 1386: What does polymerase chain reaction use?
Choices:
1. RNA polymerase to amplify RNA fragments
2. DNA polymerase to amplify DNA fragments
3. Reverse transcriptase to make DNA from RNA
4. Reverse transcriptase to make RNA from DNA
Answer: 2 - DNA polymerase to amplify DNA fragments
Explanations:
Polymerase chain reaction uses DNA polymerase to amplify DNA
fragments
Reverse transcription polymerase chain reaction uses reverse transcriptase
to make a DNA complement of an RNA fragment

Go to the next page if you knew the correct answer, or click the link images
below to further research the concepts in this question (if desired).

Research Concepts:
Polymerase Chain Reaction (PCR):

Tap flag to report any problems with this question.


Question 1387: Which of the following is not an ECG sign of
hypokalemia?

Choices:
1. Depressed ST segments
2. U wave
3. Flat T wave
4. Peaked T waves
Answer: 4 - Peaked T waves
Explanations:
ECG signs of hypokalemia include flattening of T waves and not peaked T
waves
Other features include a prominent U wave that appears as QT prolongation
St Segment depression and both atrial and ventricular arrhythmia may be
seen
Torsades de pointes is a common feature seen in patients with hypokalemia

Go to the next page if you knew the correct answer, or click the link images
below to further research the concepts in this question (if desired).

Research Concepts:
Electrocardiogram:

Hypokalemia:

Tap flag to report any problems with this question.


Question 1388: The most common cause of fever on postoperative day
one following surgery under general anesthesia should be managed by which of
the following actions?

Choices:
1. Obtain blood cultures
2. Instruct the patient to inhale deeply 10 times per hour or to use an incentive
spirometer
3. Change the dressing and culture the incision site
4. Obtain an abdominal CT scan
Answer: 2 - Instruct the patient to inhale deeply 10 times per hour or to use
an incentive spirometer

Explanations:
Mild temperature elevations postoperatively are most commonly due to
atelectasis from anesthesia.
Post operative respiratory infections typically present within the first 48
hours following surgery.

Go to the next page if you knew the correct answer, or click the link images
below to further research the concepts in this question (if desired).

Research Concepts:
Atelectasis:

Laparotomy:

Tap flag to report any problems with this question.


Question 1389: Which complication is least likely after peripheral IV
placement?

Choices:
1. Infiltration
2. Osteomyelitis
3. Thrombosis
4. Bleeding
Answer: 2 - Osteomyelitis
Explanations:
Bleeding, infiltration, infection, and thrombosis are common complications
of peripheral IV placement.

Go to the next page if you knew the correct answer, or click the link images
below to further research the concepts in this question (if desired).

Research Concepts:
Intravenous Infusion:

Tap flag to report any problems with this question.


Question 1390: In critical aortic stenosis, which of these medications can
be safely used?

Choices:
1. Digitalis
2. Nitroglycerin
3. Calcium channel blockers
4. Beta blockers
Answer: 1 - Digitalis
Explanations:
The best way to increase cardiac output in AS is to administer digoxin
Digoxin controls the heart rate and augments cardiac output
All the other drugs can reduce afterload and seriously harm the patient,
resulting in syncopal attacks

Go to the next page if you knew the correct answer, or click the link images
below to further research the concepts in this question (if desired).

Research Concepts:
Aortic Stenosis:

Tap flag to report any problems with this question.


Question 1391: What is an important modifiable factor in maintaining
patient body temperature in the operating room?

Choices:
1. Warm beddings
2. Warm fluids
3. Room temperature
4. Nutritional status
Answer: 3 - Room temperature
Explanations:
The temperature of the operating room should be adjusted for the comfort
of the patient.

Go to the next page if you knew the correct answer, or click the link images
below to further research the concepts in this question (if desired).

Research Concepts:
General Anesthesia:

Hypothermia:

Tap flag to report any problems with this question.


Question 1392: What is the best management option in a patient status-
post ventricular fibrillation (VF)?

Choices:
1. Automatic external defibrillator (AED)
2. Implantable cardioverter -defibrillator (ICD)
3. Pacemaker
4. None of the above
Answer: 2 - Implantable cardioverter -defibrillator (ICD)
Explanations:
ICDs improve long-term survival when compared to patients receiving only
medications S/P VF
AEDs would be used in the field and encompass prehospital care
Pacemakers are useful for those who suffer bradyarrhythmias

Go to the next page if you knew the correct answer, or click the link images
below to further research the concepts in this question (if desired).

Research Concepts:
Ventricular Fibrillation:

Cardioverter-Defibrillators, Implantable:

Tap flag to report any problems with this question.


Question 1393: Which of the following medications can reduce both
preload and afterload?

Choices:
1. Nitroprusside
2. Hydralazine
3. Minoxidil
4. Nifedipine
Answer: 1 - Nitroprusside
Explanations:
Nitroprusside is a potent vasodilator of both arterioles and venules
Nifedipine and hydralazine are potent arteriolar vasodilators

Go to the next page if you knew the correct answer, or click the link images
below to further research the concepts in this question (if desired).

Research Concepts:
Sodium Nitroprusside:

Preload:

Afterload Reduction:

Tap flag to report any problems with this question.


Question 1394: In a patient with a Bacteroides infection, what is the
antibiotic of choice?

Choices:
1. Ceftriaxone
2. Clindamycin
3. Vancomycin
4. Gentamycin
Answer: 2 - Clindamycin
Explanations:
Bacteroides bacteria are susceptible to metronidazole, carbapenems,
chloramphenicol, and other beta lactam inhibitors.
Clindamycin is very effective against anaerobes and also has good activity
against some gram-positive bacteria.
Quinolones have worked against bacteroides but most species have now
developed resistance to these antibiotics.
Cephalosporins, especially the first generation, are active against
bacteroides.

Go to the next page if you knew the correct answer, or click the link images
below to further research the concepts in this question (if desired).

Research Concepts:
Bacteroides Infection:

Tap flag to report any problems with this question.


Question 1395: According to Good Samaritan laws, which is true about a
provider who fails to stop at the scene of an accident?

Choices:
1. May be charged with a misdemeanor
2. Is within her rights to choose not to act
3. Is likely incur liability if she uses reasonable care
4. May be charged with a felony
Answer: 2 - Is within her rights to choose not to act
Explanations:
Under U.S. law there is no duty to rescue, however, if one chooses to
rescue, one must do so responsibly.
Good Samaritan laws protect those who choose to rescue from liability if
they use reasonable care.

Go to the next page if you knew the correct answer, or click the link images
below to further research the concepts in this question (if desired).

Research Concepts:
Good Samaritan Laws:

Tap flag to report any problems with this question.


Question 1396: Which is true of atelectasis?
Choices:
1. Is a rare postoperative complication
2. Is a common cause of early fever after surgery
3. Hyperinflation of the lungs is common
4. Best treatment is bed rest
Answer: 2 - Is a common cause of early fever after surgery
Explanations:
Atelectasis is a condition commonly seen after surgery. The condition
usually presents with collapse of part of the lung.
Atelectasis can present with dyspnea and low-grade fever. Diagnosis is
made by chest x-ray.
Atelectasis usually occusr when the large airways are blocked by mucus,
secretions, or foreign body.
Smokesr should be asked to discontinue smoking 6 weeks prior to elective
surgery to prevent atelectasis.

Go to the next page if you knew the correct answer, or click the link images
below to further research the concepts in this question (if desired).

Research Concepts:
Atelectasis:

Tap flag to report any problems with this question.


Question 1397: Poiseuille's law describes the flow of which of the
following?

Choices:
1. Liquid
2. Solid
3. Gas
4. Ideal gas
Answer: 1 - Liquid
Explanations:
Poiseuille's law describes the laminar flow of a fluid.
It can be used to describe blood flow through various vessels or air through
airways.
The change in pressure is proportional to the length, viscosity, and flow
rate.
It is inversely proportional to the radius to the fourth power.

Go to the next page if you knew the correct answer, or click the link images
below to further research the concepts in this question (if desired).

Research Concepts:
Poiseuille's Law:

Tap flag to report any problems with this question.


Question 1398: Which of the following causes pulmonary
vasoconstriction?

Choices:
1. Low Pa02
2. High PaC02
3. Hypothermia
4. All of them
Answer: 4 - All of them
Explanations:
Factors that worsen pulmonary vasoconstriction include a low PaO2, high
PaC02 and low pH
Hypothermia is often a cause of vasoconstriction

Go to the next page if you knew the correct answer, or click the link images
below to further research the concepts in this question (if desired).

Research Concepts:
Pulmonary Vasoconstriction:

Tap flag to report any problems with this question.


Question 1399: What is the best initial treatment for a 75 year old female
with history of hypertension on hydrochlorothiazide who presents with severe
chest pain that radiates to her back and dyspnea who is found to have a BP of
205/98, pulmonary rales, an ECG with ST changes, and a chest x-ray with a
widened mediastinum?

Choices:
1. Immediate cardiac catheterization with likely stent placement
2. IV nitroprusside and an urgent transesophageal echocardiogram
3. IV morphine and nitrates, oxygen and aspirin
4. Serial cardiac enzymes and IV beta-blockers and heparin
Answer: 2 - IV nitroprusside and an urgent transesophageal echocardiogram
Explanations:
This case is suggestive of aortic dissection that can be diagnosed with
transesophageal echo
Beta-blockers are for rate control and nitroprusside for BP control
A proximal, Type A, dissection is associated with aortic insufficiency and
hypertension and requires urgent repair while a distal dissection is
associated with arterial obstruction leading to renal failure and bowel
ischemia

Go to the next page if you knew the correct answer, or click the link images
below to further research the concepts in this question (if desired).

Research Concepts:
Dissection, Aortic:

Tap flag to report any problems with this question.


Question 1400: Which of the following can be used to treat an overdose of
tissue plasminogen activator (tPA)?

Choices:
1. Fresh frozen plasma (FFP)
2. Vitamin C
3. Aminocaproic acid
4. Factor X
Answer: 3 - Aminocaproic acid
Explanations:
Aminocaproic is an inhibitor of fibrinolysis
It can be used in a tPA overdose to combat toxic effects
FFP can be used in the treatment of deficiencies of coagulation proteins
Vitamin C and Factor X have no place in the treatment of tPA toxicity

Go to the next page if you knew the correct answer, or click the link images
below to further research the concepts in this question (if desired).

Research Concepts:
TPA Therapy:

Tap flag to report any problems with this question.


Section 8
Question 1401: Which of the following is not appropriate immediate
treatment of an intra-operative air embolism?

Choices:
1. Place the patient in reverse Trendelenburg
2. Irrigation of the surgical field
3. Aspiration of air from a multiforce central venous pressure catheter
4. Manual occlusion of the jugular veins
Answer: 1 - Place the patient in reverse Trendelenburg
Explanations:
The patient should be placed in the left lateral decubitus position with the
head lowered
Procedures done in the sitting position have a high incidence of venous air
embolism (VAE)
VAE has signs of bronchoconstriction, hypercarbia, hypotension, shock,
cardiac arrhythmias, and increased airway pressure
The most sensitive test is transesophageal echocardiography

Go to the next page if you knew the correct answer, or click the link images
below to further research the concepts in this question (if desired).

Research Concepts:
Venous Air Embolism:

Tap flag to report any problems with this question.


Question 1402: Select the group of medication that is not used for patients
with acute ST-elevation myocardial infarction (STEMI).

Choices:
1. ACE inhibitors
2. Beta blockers
3. Calcium channel blockers
4. Loop diuretics
Answer: 3 - Calcium channel blockers
Explanations:
Calcium channel blockers can increase morbidity and mortality in patients
with STEMI
Medications proven to reduce mortality are beta blockers, thrombolytics,
and ACE inhibitors
Diuretics should be used as needed

Go to the next page if you knew the correct answer, or click the link images
below to further research the concepts in this question (if desired).

Research Concepts:
Acute ST-elevation Myocardial Infarction (STEMI):

Tap flag to report any problems with this question.


Question 1403: A patient with a traumatic aortic rupture undergoes aortic
repair. The patient presents with hoarseness after surgery. What is the most likely
etiology?

Choices:
1. Postoperative viral infection
2. Damage to phrenic nerve
3. Trauma to recurrent laryngeal nerve
4. Injury to sympathetic chain
Answer: 3 - Trauma to recurrent laryngeal nerve
Explanations:
Anytime a patient undergoes surgery to repair the proximal aorta, if
hoarseness occurs postoperatively, laryngeal nerve injury must be
suspected.
Damage to the phrenic nerve presents with diaphragmatic paralysis.
The recurrent nerve runs between the left common carotid and the left
subclavian. It can easily be injured by placing a clamp in that region.
Injury to the sympathetic chain presents with Horner syndrome.

Go to the next page if you knew the correct answer, or click the link images
below to further research the concepts in this question (if desired).

Research Concepts:
Aorta, Trauma:

Tap flag to report any problems with this question.


Question 1404: What is the therapeutic level of INR for treatment of a
thrombus?

Choices:
1. 1.5 to 2.0
2. 2.0 to 2.5
3. 2.5 to 3.0
4. >3.0
Answer: 2 - 2.0 to 2.5
Explanations:
Treatment of a thrombus requires an INR between 2.0 and 2.5 and therapy
usually occurs for 3-6 months.

Go to the next page if you knew the correct answer, or click the link images
below to further research the concepts in this question (if desired).

Research Concepts:
Anticoagulation:

Tap flag to report any problems with this question.


Question 1405: All of the following evidence-based medical therapies
improve survival in acute coronary syndrome EXCEPT:

Choices:
1. Beta-blockers
2. Antiplatelet agents
3. Statins
4. Calcium channel blockers
Answer: 4 - Calcium channel blockers
Explanations:
Beta-blockers, antiplatelet agents, statins, and ACE inhibitors are
independently and strongly associated with lower 6-month mortality in
ACS
The medical therapies also had benefit across the different TIMI risk groups
Calcium channel blockers do not improve survival in ACS

Go to the next page if you knew the correct answer, or click the link images
below to further research the concepts in this question (if desired).

Research Concepts:
Coronary Syndrome, Acute:

Calcium Channel blockers:

Tap flag to report any problems with this question.


Question 1406: Sternal infections are highest in which group of patients?
Choices:
1. Diabetics
2. Elderly
3. People with arterial conduits
4. Patients with bilateral mammary
Answer: 1 - Diabetics
Explanations:
Wound infections occur in 1-3 % of patients.
Sternal infections occur more often in elderly, in diabetics and thus it is
recommended that bilateral IMA grafting be avoided in diabetics.

Go to the next page if you knew the correct answer, or click the link images
below to further research the concepts in this question (if desired).

Research Concepts:
Sternal Infection:

Tap flag to report any problems with this question.


Question 1407: What is the most common heart tumor in patients with
tuberous sclerosis?

Choices:
1. Myxoma
2. Melanoma
3. Rhabdomyoma
4. Metastatic lesions
Answer: 3 - Rhabdomyoma
Explanations:
Rhabdomyomas occur frequently in patients with tuberous sclerosis.
These benign heart tumors often cause outflow tract obstruction and
valvular dysfunction.
Most patients have a variety of arrhythmias.
The lesions do shrink or disappear with time and do not account for
increased mortality.

Go to the next page if you knew the correct answer, or click the link images
below to further research the concepts in this question (if desired).

Research Concepts:
Tuberous Sclerosis:

Tap flag to report any problems with this question.


Question 1408: What is included in informed consent?
Choices:
1. The opportunity to refuse
2. Purpose of the treatment
3. Life expectancy information
4. Side effects of treatment
Answer: 2 - Purpose of the treatment
Explanations:
Informed consent covers the purpose of treatment, personal prognosis if
treatment is declined and any alternatives to treatment available
Patients always have the opportunity to refuse treatment (autonomy)

Go to the next page if you knew the correct answer, or click the link images
below to further research the concepts in this question (if desired).

Research Concepts:
Informed Consent:

Tap flag to report any problems with this question.


Question 1409: Which of the following is false about anti-arrhythmic
drugs?

Choices:
1. Class 1 agents increase sodium influx
2. Class 2 agents antagonize the sympathomimetics
3. Amiodarone is a class 3 agent
4. Sotalol has class 2 and 3 effects
Answer: 1 - Class 1 agents increase sodium influx
Explanations:
Class I agents inhibit sodium entry
Class Ia agents affect the QRS and include quinidine, procainamide, and
disopyramide.
Class Ib agents do not affect the QRS and include lidocaine, phenytoin, and
mexiletine.
Class Ic agents include encainide, flecainide, propafenone, and moracizine.

Go to the next page if you knew the correct answer, or click the link images
below to further research the concepts in this question (if desired).

Research Concepts:
Antiarrhythmic Medication:

Tap flag to report any problems with this question.


Question 1410: A 60 year old female with hypertension and
hyperlipidemia is brought to the emergency department with acute angina.
Percutaneous angioplasty of an occluded posterior descending artery is
unsuccessful. 2 days later the patient develops a new decrescendo early systolic
murmur and becomes short of breath. Select the most likely etiology.

Choices:
1. Ventricular septal rupture
2. Acute papillary muscle rupture
3. Mural thrombus
4. Aortic stenosis
Answer: 1 - Ventricular septal rupture
Explanations:
The patient has developed acute mitral regurgitation secondary to papillary
muscle rupture
Ventricular septal rupture produces a holosystolic murmur
Aortic stenosis would not occur acutely

Go to the next page if you knew the correct answer, or click the link images
below to further research the concepts in this question (if desired).

Research Concepts:
Acute Myocardial Infarction:

Post Infarct Ventricular Septal Rupture:

Tap flag to report any problems with this question.


Question 1411: Which is not suggestive of acute urinary retention?
Choices:
1. Urine output of 30 ml on Foley catheter insertion
2. Post-void residual (PVR) urine o f>100 ml
3. Straining to void with no urine output
4. Suprapubic distention with lower abdominal discomfort
Answer: 1 - Urine output of 30 ml on Foley catheter insertion
Explanations:
PVR >100 ml, straining to void without urine output, and suprapubic
distention with lower abdominal discomfort are suggestive of acute urinary
retention
30 ml urine out by Foley catheter does not suggest acute urinary retention

Go to the next page if you knew the correct answer, or click the link images
below to further research the concepts in this question (if desired).

Research Concepts:
Urinary Retention:

Tap flag to report any problems with this question.


Question 1412: Hypersensitivity is most commonly reported after use of
which type of suture?

Choices:
1. Nylon
2. Stainless steel wire
3. Chromic catgut
4. Silk
Answer: 3 - Chromic catgut
Explanations:
Allergic reactions to sutures is rare
Allergic reactions however are most common after use of chromic catgut
Allergies to silk and nylon are also rarely reported
The least reactive material is stainless steel wire

Go to the next page if you knew the correct answer, or click the link images
below to further research the concepts in this question (if desired).

Research Concepts:
Suture Allergy:

Tap flag to report any problems with this question.


Question 1413: All of the following can affect lidocaine blood levels
EXCEPT:

Choices:
1. Ciprofloxacin
2. Metoprolol
3. Penicillin
4. Clonidine
Answer: 3 - Penicillin
Explanations:
Lidocaine is used as a local anesthetic
It is also used as an antiarrhythmic
Ciprofloxacin, metoprolol and clonidine can all increase lidocaine blood
levels and lead to toxicity
Penicillin does not affect lidocaine blood levels

Go to the next page if you knew the correct answer, or click the link images
below to further research the concepts in this question (if desired).

Research Concepts:
Lidocaine:

Drug Interactions:

Tap flag to report any problems with this question.


Question 1414: A patient has the following labs: sodium 135 meq/L,
potassium 2.7 meq/L, HCO3 9 meq/L, chloride 116 meq/L, BUN 18 mg/dL,
creatinine 1.0 mg/dL. The arterial blood gas shows PO2=80 mmHg, PCO2= 24
mmHg, pH=7.29. Select the correct description.

Choices:
1. Non-anion gap metabolic acidosis
2. Respiratory acidosis with compensatory metabolic alkalosis
3. Anion-gap metabolic acidosis
4. Metabolic alkalosis
Answer: 1 - Non-anion gap metabolic acidosis
Explanations:
The patient is acidotic with low PCO2 ruling out respiratory acidosis
The anion gap is normal {AG= [Na+] ? [Cl?] ? [HCO3?]}
There is a reduction in PCO2 showing respiratory compensation

Go to the next page if you knew the correct answer, or click the link images
below to further research the concepts in this question (if desired).

Research Concepts:
Metabolic Acidosis:

Tap flag to report any problems with this question.


Question 1415: Which of the following medications can effectively reduce
afterload?

Choices:
1. Digoxin
2. Spironolactone
3. Lisinopril
4. Chlorothiazide
Answer: 3 - Lisinopril
Explanations:
Afterload reduction is considered a very good approach in the management
of congestive heart failure
ACE inhibitors can effectively reduce afterload by reducing angiotensin II
levels
Diuretics may increase afterload by indirectly increasing sympathetic tone
while causing volume depletion

Go to the next page if you knew the correct answer, or click the link images
below to further research the concepts in this question (if desired).

Research Concepts:
Afterload Reduction:

Tap flag to report any problems with this question.


Question 1416: A patient who is tachypneic on assist control ventilation
(ACV) may develop which of the following?

Choices:
1. Decreased workload of the respiratory muscles
2. Lung hyperinflation
3. Metabolic acidosis
4. Respiratory acidosis
Answer: 2 - Lung hyperinflation
Explanations:
Regardless of the respiratory rate, ACV assists each spontaneous breath,
resulting in hyperinflation, increased work load of respiratory muscles, and
respiratory alkalosis

Go to the next page if you knew the correct answer, or click the link images
below to further research the concepts in this question (if desired).

Research Concepts:
Assist Control Ventilation:

Tap flag to report any problems with this question.


Question 1417: What is the most common malignancy causing inferior
vena cava syndrome?

Choices:
1. Melanoma
2. Renal cell adenocarcinoma
3. Colon cancer
4. Liver adenocarcinoma
Answer: 2 - Renal cell adenocarcinoma
Explanations:
Inferior vena cava syndrome is due to obstruction of the inferior vena cava.
The obstruction is usually due to tumor thrombus in the renal vein
extending into the IVC.
Renal cell cancer has a propensity of invading the renal vein and causing
intraluminal thrombosis. Caval thrombosis is also seen secondary to
hepatomas and adrenal cortical carcinomas.
Compression of the inferior vena cava can also occur by a gravid uterus and
Budd Chiari syndrome.

Go to the next page if you knew the correct answer, or click the link images
below to further research the concepts in this question (if desired).

Research Concepts:
Renal Cell Carcinoma:

Inferior Vena Cava Syndrome:

Tap flag to report any problems with this question.


Question 1418: Which of the following would best reduce the risk of
ototoxicity from gentamicin?

Choices:
1. Serum drug level 30 minutes after the 3rd dose
2. Serum drug level 30 minutes before the 3rd dose
3. Serum drug level 30 minutes after the 1st dose
4. Serum drug level 2 hours after the 2nd dose
Answer: 1 - Serum drug level 30 minutes after the 3rd dose
Explanations:
Peak levels after the 3rd dose should be drawn
Peak levels correlate with ototoxicity
Patients to be treated more than 2 weeks should be monitored with
audiometry
Trough levels are drawn just before a dose to insure efficacy

Go to the next page if you knew the correct answer, or click the link images
below to further research the concepts in this question (if desired).

Research Concepts:
Aminoglycoside Toxicity:

Tap flag to report any problems with this question.


Question 1419: Which of the following is used to reduce excessive
secretions during anesthesia?

Choices:
1. Tubocurarine
2. Atropine
3. Nitrous oxide
4. Neostigmine
Answer: 2 - Atropine
Explanations:
Atropine is widely used with anesthesia to decrease the oral secretions.
Atropine is only administered via an intravenous injections and works
immediately.
Atropine increases heart rate, dilates pupils, causes dry skin, and
anesthetizes the nerve endings in the skin.
Atropine also relaxes smooth muscle and suppresses gland and mucous
secretions. In the past, it was used to treat peptic ulcer by reducing the
production of stomach acid.

Go to the next page if you knew the correct answer, or click the link images
below to further research the concepts in this question (if desired).

Research Concepts:
Atropine:

Tap flag to report any problems with this question.


Question 1420: In an IV drug user, what does the presence of Bracht-
Wachter bodies indicate?

Choices:
1. Physical dependence to heroin
2. Infective endocarditis
3. Splenomegaly
4. Sarcoidosis
Answer: 2 - Infective endocarditis
Explanations:
Bracht-Wachter bodies are often a finding in patients with infective
endocarditis.
Bracht-Wachter bodies are yellowish-white miliary spots on the
myocardium.
Bracht-Wachter bodies are collections of inflammatory cells.
Other features of endocarditis include Janeway lesions, Roth spots, and
Osler nodes.

Go to the next page if you knew the correct answer, or click the link images
below to further research the concepts in this question (if desired).

Research Concepts:
Endocarditis, Bacterial:

Tap flag to report any problems with this question.


Question 1421: A 60 year old female with a long history of a heart
murmur presents with dyspnea on exertion and 3 pillow orthopnea. Exam and
echocardiogram show severe aortic regurgitation, left ventricular end systolic
dimension of 55 mm and ejection fraction of 40%. Select appropriate
management.

Choices:
1. ACE inhibitors and diuretics
2. Cardiac catheterization and aortic valve replacement
3. Dipyridamole stress test before further recommendation
4. Digoxin, diuretics, and ACE inhibitor
Answer: 2 - Cardiac catheterization and aortic valve replacement
Explanations:
The patient has LV dilation and dysfunction so valve replacement is needed
Coronary artery disease should be ruled out in a patient this age

Go to the next page if you knew the correct answer, or click the link images
below to further research the concepts in this question (if desired).

Research Concepts:
Aortic Regurgitation:

Heart Failure:

Tap flag to report any problems with this question.


Question 1422: Which should be assessed first in a trauma patient?
Choices:
1. Airway
2. Breathing
3. Circulation
4. Neurologic
Answer: 1 - Airway
Explanations:
The ABCs must be followed when assessing all acutely ill patients,
including trauma patients.
Airway is the first priority.

Go to the next page if you knew the correct answer, or click the link images
below to further research the concepts in this question (if desired).

Research Concepts:
Trauma Assessment:

Tap flag to report any problems with this question.


Question 1423: Which narcotic is not frequently used in the intensive care
unit for pain control?

Choices:
1. Methadone
2. Morphine
3. Fentanyl
4. Percocet
Answer: 1 - Methadone
Explanations:
Methadone is not routinely used in the ICU for pain control

Go to the next page if you knew the correct answer, or click the link images
below to further research the concepts in this question (if desired).

Research Concepts:
Acute Pain, Anesthesia:

Tap flag to report any problems with this question.


Question 1424: Which of the following tests is considered the most
appropriate for diagnosing deep venous thrombosis?

Choices:
1. D-dimers assay
2. Compression ultrasonography
3. Impedance plethysmography
4. Coagulation profile
Answer: 2 - Compression ultrasonography
Explanations:
A more direct approach to the diagnosis of DVT involves use of
compression ultrasonography
Compression ultrasonography is the noninvasive approach of choice for the
diagnosis of symptomatic patients with a first episode of suspected DVT
Venography is not recommended as an initial screening due to patient
discomfort

Go to the next page if you knew the correct answer, or click the link images
below to further research the concepts in this question (if desired).

Research Concepts:
Deep Vein Thrombosis:

Tap flag to report any problems with this question.


Question 1425: A diabetic patient has an uneventful 3-vessel bypass. After
weaning, protamine is administered and the patient's pulmonary artery pressures
are suprasystemic. The best management is all except

Choices:
1. Administer heparin
2. Administer epinephrine
3. Resume CPB
4. Insert IABP
Answer: 4 - Insert IABP
Explanations:
An occasional diabetic patient may show a severe reaction to protamine
The cardiovascular effects of protamine include hypotension, anaphylaxis
and severe pulmonary vasoconstriction
At times, just replacing volume and inotropic support may not be enough
and one has to return to CPB and rest the patient. There is no place for an
IABP here
After going back on CPB, the patient is weaned off slowly and protamine
may have to be given very slowly or not all

Go to the next page if you knew the correct answer, or click the link images
below to further research the concepts in this question (if desired).

Research Concepts:
Protamine:

Tap flag to report any problems with this question.


Question 1426: All of the following variables denote high-risk on exercise
ECG, EXCEPT:

Choices:
1. > 2.0 mm ST segment depression
2. ST segment depression in multiple leads
3. Ventricular arrhythmias
4. High maximal heart rate
Answer: 4 - High maximal heart rate
Explanations:
High-risk exercise ECG variables include ?2.0 mm ST-segment depression,
?1 mm ST-segment depression in stage I, ST segment depression in
multiple leads and ST-segment depression > 1.0 mm for > 5 minutes during
the recovery period,
Other variables include achievement of a workload of less than 4 METS or
a low maximal heart rate
Abnormal blood pressure response and ventricular arrhythmias are still
others

Go to the next page if you knew the correct answer, or click the link images
below to further research the concepts in this question (if desired).

Research Concepts:
Treadmill Stress Testing:

Tap flag to report any problems with this question.


Question 1427: Which medications has been shown to improve long term
survival in patients with congestive heart failure?

Choices:
1. Nifedipine
2. Digoxin
3. Furosemide
4. Captopril
Answer: 4 - Captopril
Explanations:
ACE-inhibitors are well known to increase long term survival in patient
with congestive heart failure
Calcium channel blockers and antiarrhythmics have not been documented
to improve long term survival

Go to the next page if you knew the correct answer, or click the link images
below to further research the concepts in this question (if desired).

Research Concepts:
Heart Failure, Congestive:

Tap flag to report any problems with this question.


Question 1428: Which of the following bodily fluids can transmit HIV?
Choices:
1. Tears
2. Sweat
3. Blood
4. Saliva
Answer: 3 - Blood
Explanations:
HIV can be spread by contact with blood, blood containing fluids, semen,
and vaginal secretions
HIV cannot be contracted from saliva, sweat, or tears

Go to the next page if you knew the correct answer, or click the link images
below to further research the concepts in this question (if desired).

Research Concepts:
Universal Precautions:

HIV Transmission:

Tap flag to report any problems with this question.


Question 1429: A 43-year-old male suddenly expires. Autopsy is
performed, which reveals infiltrates of plasma cells and lymphocytes around the
vaso vasorum of the ascending aorta. This patient may have developed which of
the following conditions?

Choices:
1. Marfan Syndrome
2. Syphilis
3. Degenerative aortic disease
4. Takayasu disease
Answer: 2 - Syphilis
Explanations:
Cardiac complications of syphilis include aortitis, aortic aneurysm, and
aortic regurgitation.
After many years of infection, the inflammation causes scarring of the
aortic wall, leading to aneurysm formation.
Histological study often reveals a mononuclear and plasma cell infiltrate
into the vaso vasorum.
The inflammatory reaction by the spirochete causes an obliterative
endarteritis.

Go to the next page if you knew the correct answer, or click the link images
below to further research the concepts in this question (if desired).

Research Concepts:
Syphilis:

Tap flag to report any problems with this question.


Question 1430: Which is the most common characteristic of
hypoglycemia?

Choices:
1. Nervousness
2. Dry skin
3. Thirst
4. Edema
Answer: 1 - Nervousness
Explanations:
Common presentations of hypoglycemia are tachycardia, sweating, and
nervousness.

Go to the next page if you knew the correct answer, or click the link images
below to further research the concepts in this question (if desired).

Research Concepts:
Hypoglycemia:

Tap flag to report any problems with this question.


Question 1431: Which of the following medications would not be
appropriate for analgesic use after major surgery in a patient who was on chronic
narcotics preoperatively?

Choices:
1. Oxycontin
2. Nalbuphine
3. Methadone
4. Hydromorphone
Answer: 2 - Nalbuphine
Explanations:
Nalbuphine is a mixed opioid agonist-antagonist.
It may cause withdrawal symptoms in patients chronically receiving
opioids.

Go to the next page if you knew the correct answer, or click the link images
below to further research the concepts in this question (if desired).

Research Concepts:
Opioids:

Tap flag to report any problems with this question.


Question 1432: Which is false about hepatitis B?
Choices:
1. Some patients with hepatitis may have no symptoms
2. Symptoms usually develop 1-3 months after acquiring the infection
3. Carriers of the disease are usually asymptomatic
4. Asymptomatic patients cannot pass the infection
Answer: 4 - Asymptomatic patients cannot pass the infection
Explanations:
Patients without symptoms can transmit hepatitis B.

Go to the next page if you knew the correct answer, or click the link images
below to further research the concepts in this question (if desired).

Research Concepts:
Hepatitis B:

Tap flag to report any problems with this question.


Question 1433: Pulsus paradoxus can be seen with all of the following
conditions EXCEPT:

Choices:
1. Cardiac tamponade
2. Asthma
3. Constrictive pericarditis
4. Pneumonia
Answer: 4 - Pneumonia
Explanations:
Pulsus paradoxus represents a decrease in pulse volume (usually >10 mm
Hg) during inspiration
it can have cardiac causes, pulmonary causes and non-cardiac/non-
pulmonary causes
Cardiac causes include cardiac tamponade, constrictive pericarditis,
pericardial effusion and shock
Pulmonary causes include asthma, COPD and tension pneumothorax

Go to the next page if you knew the correct answer, or click the link images
below to further research the concepts in this question (if desired).

Research Concepts:
Pulsus Paradoxus:

Tap flag to report any problems with this question.


Question 1434: What do Phase III drug trials involve?
Choices:
1. Small group of healthy volunteers
2. Small group of patient volunteers
3. Large group of health volunteers
4. Large group of patient volunteers
Answer: 4 - Large group of patient volunteers
Explanations:
The aim of a Phase III drug trial is to study the efficacy of the
investigational drug
Therefore, study is done on large group of patient volunteers
Small group of health volunteers are used for phase II studies

Go to the next page if you knew the correct answer, or click the link images
below to further research the concepts in this question (if desired).

Research Concepts:
Drug Trials:

Tap flag to report any problems with this question.


Question 1435: Which of the following is the earliest marker after acute
myocardial infarction?

Choices:
1. LDH
2. Troponins
3. CK-MB
4. Myoglobin
Answer: 4 - Myoglobin
Explanations:
Myoglobin is the earliest marker and rises 2-4 hours after MI
It lacks cardiospecificity and is therefore of limited diagnostic value with
regard to MI
Troponins are the most sensitive and specific marker for MI
In descending order of appearance--myoglobin, CK-MB, troponin and LDH

Go to the next page if you knew the correct answer, or click the link images
below to further research the concepts in this question (if desired).

Research Concepts:
Myoglobin:

Acute Myocardial Infarction:

Tap flag to report any problems with this question.


Question 1436: Which of the following is most likely to cause decreased
cardiac output, muscle weakness, and hemolytic anemia?

Choices:
1. Hyperkalemia
2. Hypophosphatemia
3. Hypocalcemia
4. Hypernatremia
Answer: 2 - Hypophosphatemia
Explanations:
Severe hypophosphatemia can cause muscle weakness, hemolytic anemia,
decreased cardiac output, rhabdomyolysis, and decreased oxygen
availability to tissue
It can be iatrogenic or secondary to chronic alcoholism, diabetic
ketoacidosis, refeeding, and acute respiratory acidosis

Go to the next page if you knew the correct answer, or click the link images
below to further research the concepts in this question (if desired).

Research Concepts:
Hypophosphatemia:

Tap flag to report any problems with this question.


Question 1437: Which symptoms are commonly seen in early alcohol
withdrawal?

Choices:
1. Seizures
2. Visual hallucinations
3. Tachycardia and mild tremors
4. Auditory hallucinations
Answer: 3 - Tachycardia and mild tremors
Explanations:
Seizures and visual hallucinations are seen later in alcohol withdrawal
Tachycardia, mild tremors, anxiety, restlessness, early signs and symptoms
of ETOH withdrawal
Auditory hallucinations are more commonly associated with mental illness,
such as schizophrenia

Go to the next page if you knew the correct answer, or click the link images
below to further research the concepts in this question (if desired).

Research Concepts:
Alcohol Withdrawal:

Tap flag to report any problems with this question.


Question 1438: Which of the following is not an effect of opioid
analgesics?

Choices:
1. Mydriasis
2. Constipation
3. Sedation
4. Urinary retention
Answer: 1 - Mydriasis
Explanations:
Opioid analgesics cause miosis not mydriasis
Other effects are analgesia, euphoria, depression of the cough reflex,
emesis, respiratory depression, and relaxation of the smooth muscle of the
bladder

Go to the next page if you knew the correct answer, or click the link images
below to further research the concepts in this question (if desired).

Research Concepts:
Opioid Analgesics:

Tap flag to report any problems with this question.


Question 1439: What is a potentially serious adverse reaction that has
been reported with the use of clonidine?

Choices:
1. Arrhythmias
2. Anaphylaxis
3. Renal damage
4. Rebound hypertensive crisis
Answer: 4 - Rebound hypertensive crisis
Explanations:
Patients on clonidine should not discontinue it abruptly.
Suddenly stopping clonidine can lead to rebound hypertension, anxiety, and
tremor.

Go to the next page if you knew the correct answer, or click the link images
below to further research the concepts in this question (if desired).

Research Concepts:
Clonidine:

Tap flag to report any problems with this question.


Question 1440: Patients with sulfa allergy can be given which of the
following diuretics:

Choices:
1. Hydrochlorothiazide
2. Furosemide
3. Ethacrynic acid
4. Metolazone
Answer: 3 - Ethacrynic acid
Explanations:
Furosemide and thiazide diuretics are sulfa drugs and should be avoided in
sulfa allergy
Ethacrynic acid is a loop diuretic with no sulfa component
It is safe to use in patients with sulfa allergy

Go to the next page if you knew the correct answer, or click the link images
below to further research the concepts in this question (if desired).

Research Concepts:
Sulfa Allergy:

Ethacrynic Acid:

Diuretics:
Tap flag to report any problems with this question.
Question 1441: In adults, what is the most common cause of bilateral
lower extremity edema?

Choices:
1. Varicose veins
2. Heart failure
3. Lymphedema
4. Renal failure
Answer: 2 - Heart failure
Explanations:
The most common causes of extremity edema are of systemic origin and
produce bilateral lower extremity involvement. Cardiac failure is the most
common cause followed by renal failure.
Bilateral lower extremity edema is also secondary to cirrhosis, nephrotic
syndrome, and malnutrition. Another frequent cause of bilateral leg
enlargement is lipedema.
Lipedema is not true edema. It is usually found in obese females and
involves the bilateral lower extremities.
It is non-pitting and greatest at the ankle and legs with characteristic sparing
of the feet. There are no skin changes, and the enlargement does not resolve
with elevation.

Go to the next page if you knew the correct answer, or click the link images
below to further research the concepts in this question (if desired).

Research Concepts:
Edema:

Tap flag to report any problems with this question.


Question 1442: What kind of solution is 5% dextrose in normal saline
(D5NS)?

Choices:
1. Hypotonic solution
2. Hypertonic solution
3. Isotonic solution
4. Tonic solution
Answer: 2 - Hypertonic solution
Explanations:
D5NS is a hypertonic solution
A hypertonic solution has increased solutes compared to cells
A hypotonic solution has decreased solutes compared to cells
An isotonic solution has a solute composition similar to blood plasma

Go to the next page if you knew the correct answer, or click the link images
below to further research the concepts in this question (if desired).

Research Concepts:
Hypertonic Fluids:

Tap flag to report any problems with this question.


Question 1443: What is the intrinsic rate of the atrial node?
Choices:
1. 10 to 30 bpm
2. 35 to 60 bpm
3. 60 to 100 bpm
4. 105 OT 125 bpm
Answer: 3 - 60 to 100 bpm
Explanations:
The intrinsic rate of the atrial node is 60 to 100 beats per minute.

Go to the next page if you knew the correct answer, or click the link images
below to further research the concepts in this question (if desired).

Research Concepts:
Cardiac Physiology:

Tap flag to report any problems with this question.


Question 1444: What class of drug is quinidine?
Choices:
1. Calcium channel blocker
2. Sodium channel blocker
3. Antibiotic
4. Anticoagulant
Answer: 2 - Sodium channel blocker
Explanations:
Quinidine is a sodium channel blocker

Go to the next page if you knew the correct answer, or click the link images
below to further research the concepts in this question (if desired).

Research Concepts:
Quinidine:

Tap flag to report any problems with this question.


Question 1445: ACE inhibitors can cause which of the following
electrolyte disturbances because of decreased aldosterone production and
decreased excretion?

Choices:
1. Hyperkalemia
2. Hypermagnesemia
3. Hypercalcemia
4. Hypernatremia
Answer: 1 - Hyperkalemia
Explanations:
Hyperkalemia may be caused by ACE inhibitors due to decreased
aldosterone production and decreased excretion of potassium.
ACE inhibitors are often associated with a transient increase in creatinine.
Renal failure may result in the scenario of bilateral renal artery stenosis.

Go to the next page if you knew the correct answer, or click the link images
below to further research the concepts in this question (if desired).

Research Concepts:
Angiotensin Converting Enzyme Inhibitors (ACEI):

Hyperkalemia:

Tap flag to report any problems with this question.


Question 1446: Which of the following is not a cause of torsades de
pointes?

Choices:
1. Amiodarone
2. Procainamide
3. Erythromycin
4. Hyperkalemia
Answer: 4 - Hyperkalemia
Explanations:
Torsades de pointes is a polymorphic ventricular tachycardia which literally
translates to "twisting of spikes."
Multiple cardiac medications can cause this arrhythmia, including
quinidine, disopyramide, dofetilide, ibutilide, and sotalol.
Anti-infectious agents that can cause this condition include macrolides and
pentamidine.
An anti-emetic that causes torsades de pointes is droperidol. Haloperidol,
pimozide, chlorpromazine, and thioridazine are anti-psychotics with this
complication.

Go to the next page if you knew the correct answer, or click the link images
below to further research the concepts in this question (if desired).

Research Concepts:
Torsade de Pointes:

Tap flag to report any problems with this question.


Question 1447: What is the mechanism of action of furosemide?
Choices:
1. Inhibits Na/K ATPase
2. Inhibits cyclic AMP
3. Inhibits Na/K Cl co-transporter
4. Inhibits calcium movement
Answer: 3 - Inhibits Na/K Cl co-transporter
Explanations:
Furosemide is a loop diuretic that is used to treat hypertension and
congestive heart failure.
The drug acts by inhibit the Na/K Cl symporter in the thick ascending limb
of the loop of henle. The drug reduces reabsorption of salt and stimulates
diuresis.
Prolonged use of furosemide can lead to hypokalemia and
hypomagnesemia.
Furosemide does not cause hypocalcemia.

Go to the next page if you knew the correct answer, or click the link images
below to further research the concepts in this question (if desired).

Research Concepts:
Furosemide:

Tap flag to report any problems with this question.


Question 1448: At what level of PaCO2 is cerebral ischemia seen?
Choices:
1. 20 mmHg
2. 25 mmHg
3. 30 mmHg
4. 35 mmHg
Answer: 1 - 20 mmHg
Explanations:
PaCO2 below 20 mmHg can lead to cerebral ischemia.

Go to the next page if you knew the correct answer, or click the link images
below to further research the concepts in this question (if desired).

Research Concepts:
Hypocarbia:

Tap flag to report any problems with this question.


Question 1449: Which medication can induce Parkinsonian-like features?
Choices:
1. Haloperidol
2. Cimetidine
3. Sertraline
4. Gemfibrozil
Answer: 1 - Haloperidol
Explanations:
Antidopaminergic drugs may cause drug-induced Parkinsonism.
Most antipsychotic medications block dopamine receptors and may cause
Parkinsonism.
Metoclopramide and haloperidol are antipsychotics that block dopamine
receptors.

Go to the next page if you knew the correct answer, or click the link images
below to further research the concepts in this question (if desired).

Research Concepts:
Neuroleptic Medication:

Tap flag to report any problems with this question.


Question 1450: During cardiopulmonary bypass, you suddenly notice air
in the arterial line. What should be done first?

Choices:
1. Stop CPB
2. Slow flow rate
3. Aspirate air from tubing
4. Place head down
Answer: 1 - Stop CPB
Explanations:
Whenever air is seen in the arterial line or a break in the circuit occurs, the
first thing to do is stop the pump
If air does get into the heart, the patient has to be placed in Trendelenburg
position, retrograde purging of the heart, aortic root venting and aspirating
the underside of the arch to remove as much as air possible

Go to the next page if you knew the correct answer, or click the link images
below to further research the concepts in this question (if desired).

Research Concepts:
CPB, Air Embolism:

Tap flag to report any problems with this question.


Question 1451: Which is NOT an appropriate treatment for a patient with
coronary artery disease?

Choices:
1. Minimize oxygen demand
2. Maximize coronary flow
3. Lower heart rate
4. Increase afterload
Answer: 4 - Increase afterload
Explanations:
The goal in patients with coronary artery disease is to control blood
pressure and minimize the oxygen demand.
Increasing afterload would increase work for the heart.

Go to the next page if you knew the correct answer, or click the link images
below to further research the concepts in this question (if desired).

Research Concepts:
Coronary Artery Disease:

Tap flag to report any problems with this question.


Question 1452: What is the most common type of cardiomyopathy?
Choices:
1. Dilated
2. Restrictive
3. Hypertrophic
4. Hypertensive
Answer: 1 - Dilated
Explanations:
Dilated cardiomyopathy is the most common of all myopathies and often no
cause is ever discovered.
The majority of cases present with congestive heart failure.
Both the left and right heart are affected in dilated cardiomyopathy, leading
to progressive enlargement of the heart.
The disorder occurs in men and women. Dilated cardiomyopathy has a poor
prognosis and the only viable treatment is a transplant.

Go to the next page if you knew the correct answer, or click the link images
below to further research the concepts in this question (if desired).

Research Concepts:
Cardiomyopathy, Dilated:

Tap flag to report any problems with this question.


Question 1453: Which of the following medications may impair glycemic
control in diabetic patients?

Choices:
1. Hydrochlorothiazide
2. Verapamil
3. Enalapril
4. Furosemide
Answer: 1 - Hydrochlorothiazide
Explanations:
Thiazide diuretics should be used with caution in diabetic patients
Insulin resistance and hyperglycemia may result from thiazide diuretic
therapy
ACE inhibitors are very helpful in diabetic patients

Go to the next page if you knew the correct answer, or click the link images
below to further research the concepts in this question (if desired).

Research Concepts:
Diabetes Mellitus, Type 1:

Diabetes Mellitus, Type 2:

Thiazide Diuretics:
Tap flag to report any problems with this question.
Question 1454: On which of the following do low molecular weight
heparins have their effect?

Choices:
1. Anti thrombin 3
2. Plasminogen
3. Factor X
4. Factor 7
Answer: 3 - Factor X
Explanations:
Enoxaparin is a typical low molecular weight heparin.
All low molecular weight heparins act on factor X.
Other low molecular weight heparin-like products include dalteparin and
danaparoid.
The major toxicity with these agents is bleeding.

Go to the next page if you knew the correct answer, or click the link images
below to further research the concepts in this question (if desired).

Research Concepts:
Low Molecular Weight Heparin (LMWH):

Tap flag to report any problems with this question.


Question 1455: A patient with patent venous grafts and a left internal
mammary artery graft now needs a mitral valve replacement. The lateral chest x-
ray shows virtual symphysis between the right atrium and the sternum. The best
approach is:

Choices:
1. Fem-fem bypass
2. Right thoracotomy
3. Median sternotomy
4. Minimally invasive approach
Answer: 2 - Right thoracotomy
Explanations:
A right thoracotomy can be used sometimes. With this approach, the
incision is through the left atrium just behind the interatrial groove, as in
median sternotomy
Its chief disadvantages are the limited field provided by the anterolateral
interspace and the relative inaccessibility of the ascending aorta both for
cannulation and for cardioplegia infusion
The groin may have to be cannulated for femoral artery access; the
procedure can be done under fibrillatory arrest

Go to the next page if you knew the correct answer, or click the link images
below to further research the concepts in this question (if desired).

Research Concepts:
Mitral Valve Replacement:

Cardiac Valvular Surgery:

Tap flag to report any problems with this question.


Question 1456: The middle cardiac vein that ascends in the posterior
interventricular groove is accompanied by which artery?

Choices:
1. Left coronary artery
2. Right coronary artery
3. Thoracic artery
4. Internal carotid
Answer: 2 - Right coronary artery
Explanations:
The middle cardiac vein runs from the apex of the heart, along the posterior
longitudinal sulcus, and drains into the coronary sinus.
It is accompanied by the right coronary artery middle cardiac vein in the
posterior interventricular groove.

Go to the next page if you knew the correct answer, or click the link images
below to further research the concepts in this question (if desired).

Research Concepts:
Heart Anatomy:

Tap flag to report any problems with this question.


Question 1457: A 65-year-old male is seen in the emergency room by the
triage nurse. He is exhibiting the Levine sign. He may have which of the
following conditions?

Choices:
1. Wheezing
2. Angina
3. Worst headache of his life
4. Meningitis
Answer: 2 - Angina
Explanations:
A Levine sign is making a clenched fist in front of the chest while
describing chest pain.
Patients may describe angina pain as pressing, squeezing, or strangling.

Go to the next page if you knew the correct answer, or click the link images
below to further research the concepts in this question (if desired).

Research Concepts:
Angina, Unstable:

Angina, Stable:

Tap flag to report any problems with this question.


Question 1458: Which of following abnormalities is not part of the
metabolic syndrome?

Choices:
1. Triglycerides
2. HDL
3. Blood pressure
4. GFR
Answer: 4 - GFR
Explanations:
The metabolic syndrome is characterized by obesity, glucose intolerance,
and elevated lipid levels and blood pressure

Go to the next page if you knew the correct answer, or click the link images
below to further research the concepts in this question (if desired).

Research Concepts:
Metabolic Syndrome:

Tap flag to report any problems with this question.


Question 1459: Which is the most common error made when inserting a
subclavian venous line?

Choices:
1. Inadequate landmark
2. Improper needle position
3. Aiming needle too cephalad
4. Too shallow trajectory
Answer: 2 - Improper needle position
Explanations:
There are many reasons why residents fail at cannulating the subclavian
vein.
The most common reason is improper needle position, followed by
insertion of needle through periosteum.
Other reasons for complications during subclavian line placement include
taking a shallow trajectory, aiming needle to cephalad, and failure to keep
needle in place for wire passage.

Go to the next page if you knew the correct answer, or click the link images
below to further research the concepts in this question (if desired).

Research Concepts:
Central Venous Access, Subclavian Vein, Subclavian:

Tap flag to report any problems with this question.


Question 1460: Which one of the following drugs can cause release of
histamine?

Choices:
1. Labetalol
2. Nedocromil
3. Morphine
4. Propranolol
Answer: 3 - Morphine
Explanations:
Morphine can displace histamine from granule storage sites
Therefore, morphine can cause pruritis in up to 80% of patients
Nedocromil is a mast cell stabilizer

Go to the next page if you knew the correct answer, or click the link images
below to further research the concepts in this question (if desired).

Research Concepts:
Histamine:

Morphine:

Tap flag to report any problems with this question.


Question 1461: Which of the following parameters is NOT measured in
spirometry?

Choices:
1. Vital capacity (VC)
2. Forced vital capacity (FVC)
3. Tidal volume
4. CO2 volume
Answer: 4 - CO2 volume
Explanations:
Spirometry can help diagnose lung diseases such as asthma, COPD, cystic
fibrosis, and pulmonary fibrosis
It can also be used to gauge response to treatment interventions
VC, FVC and tidal volume, residual volume, total lung capacity, expiratory
reserve volume and peak flow are some of the components measured
CO2 volume is not a part of spirometric testing

Go to the next page if you knew the correct answer, or click the link images
below to further research the concepts in this question (if desired).

Research Concepts:
Pulmonary Function Tests:

Tap flag to report any problems with this question.


Question 1462: Which of the following is generally responsible for
causing an aneurysmal dilatation of the left ventricle?

Choices:
1. Trauma
2. MI
3. Congenital
4. Aortic aneurysms
Answer: 2 - MI
Explanations:
Left ventricular aneurysms are most common after an MI.
The aneurysms generally are seen after 1 week and take about 4-6 weeks to
mature.
Unlike other aneurysms, the risk of rupture is low. These aneurysms have
fibrous tissue and do not contribute to the contractility of the ventricle.
Large aneurysms require surgery, as they have a tendency to form blood
clots.

Go to the next page if you knew the correct answer, or click the link images
below to further research the concepts in this question (if desired).

Research Concepts:
Ventricular Aneurysms:

Tap flag to report any problems with this question.


Question 1463: In a patient treated with heparin the platelet count is
50,000. A clot develops in the femoral artery. What is the best therapy?

Choices:
1. Increase dose of heparin
2. Change to low molecular weight heparin
3. Stop all heparin
4. Give a bolus of heparin
Answer: 3 - Stop all heparin
Explanations:
Heparin induced thrombocytopenia and thrombosis is a serious
complication of heparin therapy.
Even though the platelet counts are low after heparin, thrombosis can occur.
The most important treatment for heparin-induced thrombocytopenia is to
stop all heparin.
The clot in the leg may be removed by embolectomy or one may start
warfarin. Continuation of heparin therapy worsens the condition.

Go to the next page if you knew the correct answer, or click the link images
below to further research the concepts in this question (if desired).

Research Concepts:
Thrombocytopenia, Heparin Induced:

Tap flag to report any problems with this question.


Question 1464: The most important component in off pump CABG is:
Choices:
1. Anesthesia
2. Location of disease vessel
3. Stabilizer
4. Blood pressure control
Answer: 3 - Stabilizer
Explanations:
Off pump CABG is dependent on a stable heart
Initially, medications were used to reduce the heart rate and force of
contraction
Today, stabilizers are available and play a crucial role in performing
anastomosis

Go to the next page if you knew the correct answer, or click the link images
below to further research the concepts in this question (if desired).

Research Concepts:
Coronary Artery Surgery:

Tap flag to report any problems with this question.


Question 1465: Cor triatriatum sinistrum resembles:
Choices:
1. Aortic stenosis
2. Mitral stenosis
3. Tricuspid regurgitation
4. Pulmonary hypertension
Answer: 2 - Mitral stenosis
Explanations:
Cor triatriatum sinistrum is a valvular disorder
Signs resemble mitral stenosis

Go to the next page if you knew the correct answer, or click the link images
below to further research the concepts in this question (if desired).

Research Concepts:
Cor Triatriatum:

Tap flag to report any problems with this question.


Question 1466: Which one of the following drug administration routes is
likely to have significant first pass effect?

Choices:
1. Oral
2. Sublingual
3. Inhalational
4. Intravenous
Answer: 1 - Oral
Explanations:
Orally administered drugs pass through the portal circulation to the liver
Intravenous drugs do not undergo first pass metabolism
Sublingual administration does not go through first pass effect and absorbed
directly from the oral mucosa
Inhalational route is unlikely to undergo first pass effect

Go to the next page if you knew the correct answer, or click the link images
below to further research the concepts in this question (if desired).

Research Concepts:
First Pass effect:

Tap flag to report any problems with this question.


Question 1467: Which person has the ultimate power to determine
hospital transfer of a patient?

Choices:
1. Nurse
2. Charge nurse
3. Attending
4. Resident
Answer: 3 - Attending
Explanations:
The ultimate decision to transfer a patient is made by the attending
physician

Go to the next page if you knew the correct answer, or click the link images
below to further research the concepts in this question (if desired).

Research Concepts:
Hospital Transfer:

Tap flag to report any problems with this question.


Question 1468: Which of the following is not true regarding heparin
induced thrombocytopenia?

Choices:
1. It occurs months after starting therapy
2. It is caused by antibodies formed against platelet receptors
3. Surgical patients are at greater risk than medical patients
4. Hypercoagulability is a major problem
Answer: 1 - It occurs months after starting therapy
Explanations:
Thrombocytopenia is a well-recognized complication of heparin therapy
It usually occurs within 5 to 10 days after heparin treatment has started
Delayed manifestation of such a disease is rarely seen beyond two weeks
Instead of hypocoagulation, hypercoagulability is a major problem

Go to the next page if you knew the correct answer, or click the link images
below to further research the concepts in this question (if desired).

Research Concepts:
Thrombocytopenia, Heparin Induced:

Tap flag to report any problems with this question.


Question 1469: Which medication is NOT indicated initially in patients
diagnosed with heparin-induced thrombocytopenia?

Choices:
1. Argatroban
2. Coumadin
3. Fondaparinux
4. Lepirudin
Answer: 2 - Coumadin
Explanations:
Anticoagulant options with HIT are argatroban, fondaparinux, and
lepirudin.

Go to the next page if you knew the correct answer, or click the link images
below to further research the concepts in this question (if desired).

Research Concepts:
Thrombocytopenia, Heparin Induced:

Tap flag to report any problems with this question.


Question 1470: Sildenafil can have significant interaction with one of the
following medications:

Choices:
1. Captopril
2. Spironolactone
3. Methyldopa
4. Nitroglycerin
Answer: 4 - Nitroglycerin
Explanations:
Nitroglycerin increases nitric oxide levels and enhances the production of
cGMP
Sildenafil works by inhibiting phosphodiesterases to reduce cGMP
degradation
Administering both drugs together raises the cGMP level that can cause
dangerous vasodilation, shock, and syncope

Go to the next page if you knew the correct answer, or click the link images
below to further research the concepts in this question (if desired).

Research Concepts:
Sildenafil:

Nitroglycerin:

Tap flag to report any problems with this question.


Question 1471: Which one of the following patients need higher doses of
anesthesia?

Choices:
1. Obese patients
2. Pediatric patients that have had multiple procedures
3. Patients on long-term steroid therapy
4. Patients taking lopinavir/ritonavir
Answer: 1 - Obese patients
Explanations:
Most anesthetic drugs have high affinity for adipose tissue.
Increased fat stores may increase requirements for fat-soluble anesthetics.
Clearance from the body is also a problem since it is released from fat
slowly.
Ritonavir is a known liver enzyme inhibitor and delays metabolism of some
medications

Go to the next page if you knew the correct answer, or click the link images
below to further research the concepts in this question (if desired).

Research Concepts:
Obesity:

General Anesthesia:

Tap flag to report any problems with this question.


Question 1472: An otherwise healthy 18 year old female is to have an
elective surgery. Which of the following tests is indicated before general
anesthesia?

Choices:
1. Hemoglobin levels
2. WBC count
3. X-ray of the chest
4. Urine pregnancy
Answer: 4 - Urine pregnancy
Explanations:
No testing is needed other than a negative pregnancy test.
Routine preoperative testing of a healthy young adult is not recommended.
There have been no studies showing decreased operative morbidity or
mortality.
With advancing age, some laboratories or ECG may be appropriate.

Go to the next page if you knew the correct answer, or click the link images
below to further research the concepts in this question (if desired).

Research Concepts:
Preoperative Testing:

Tap flag to report any problems with this question.


Question 1473: Heart failure patients with no limitation of physical
activity are New York Heart Association (NYHA) class:

Choices:
1. I
2. II
3. III
4. IV
Answer: 1 - I
Explanations:
NYHA class I patients have no limitation of activity. NYHA class II
patients have slight limitation of activity.
NYHA class III patients have marked limitation of activity. NYHA class IV
patients have symptoms at rest and are unable to carry out any activity
without discomfort
Heart Failure Society of America. Retrieved from
www.abouthf.org/questions_stages.htm

Go to the next page if you knew the correct answer, or click the link images
below to further research the concepts in this question (if desired).

Research Concepts:
Heart Failure, Congestive:

Tap flag to report any problems with this question.


Question 1474: A 70-year-old male with congestive heart failure develops
symptomatic atrial fibrillation 2 days post surgery. His BP is 97/60 and his pulse
is > 135. His respirations are 22 times a min. What is the best treatment for this
patient?

Choices:
1. Epinephrine
2. Amiodarone
3. Digoxin
4. Lidocaine
Answer: 2 - Amiodarone
Explanations:
If the patient is stable then medications can be used to control the heart rate.
Evidence indicates that the heart rate must be under 110 but some experts
say under 80 is better.
American College of Cardiology indicates that drugs like flecainide,
propafenone, and ibutilide do work well.
Amiodarone has consistently been shown to be effective both orally and IV
and is widely used in patients. The only problem is that amiodarone does
have side effects and should not be used for long term.

Go to the next page if you knew the correct answer, or click the link images
below to further research the concepts in this question (if desired).

Research Concepts:
Atrial Fibrillation:

Heart Failure, Congestive:

Tap flag to report any problems with this question.


Question 1475: The heparin antagonist, protamine, is a:
Choices:
1. Basic protein
2. Acid protein
3. Antibody
4. Chelator
Answer: 1 - Basic protein
Explanations:
Protamine is a basic protein
Heparin is acidic

Go to the next page if you knew the correct answer, or click the link images
below to further research the concepts in this question (if desired).

Research Concepts:
Protamine:

Tap flag to report any problems with this question.


Question 1476: Which of the following statements concerning the cardiac
conduction system and EKG findings is NOT true?

Choices:
1. Some pre-excitation syndromes are not characterized by a delta wave
2. Interruption of the sinoatrial node's blood supply is the main reason for
bradyarrhythmias in acute inferior wall myocardial infarction
3. Ventricular repolarization can be affected by hypoxia, pain, myocardial
ischemia, fever, abnormal depolarization, and drugs
4. Atrial fibrillation, atrial flutter, and paroxysmal supraventricular
tachycardia are the most common tachyarrhythmias seen with Wolff-
Parkinson-White syndrome
Answer: 2 - Interruption of the sinoatrial node's blood supply is the main
reason for bradyarrhythmias in acute inferior wall myocardial infarction

Explanations:
Because 90% of humans have a blood supply to the AV node coming from
a branch off the right coronary artery, AV conduction disturbances often
occur in acute inferior wall myocardial infarction.

Go to the next page if you knew the correct answer, or click the link images
below to further research the concepts in this question (if desired).

Research Concepts:
Wolff-Parkinson-White Syndrome:

Myocardial Infarction, Acute:

Tap flag to report any problems with this question.


Question 1477: A 56 year old with multifocal tachycardia is best treated
with?

Choices:
1. Amiodarone
2. Verapamil
3. Phenytoin
4. Metoprolol
Answer: 2 - Verapamil
Explanations:
Treatment of MAT should focus on treatment of underlying problems
Calcium channel blockers such as verapamil have been documented to be
effective in MAT
Other medications that can be used in this setting are beta blockers
Beta blockers are usually not recommended in these patients since most of
them have COPD

Go to the next page if you knew the correct answer, or click the link images
below to further research the concepts in this question (if desired).

Research Concepts:
Multifocal Atrial Tachycardia:

Tap flag to report any problems with this question.


Question 1478: Which of the following is NOT a significant predictor of
mortality in patients diagnosed with severe coronary artery disease?

Choices:
1. Homocysteine
2. C-reactive protein (CRP)
3. Methylenetetrahydrofolate reductase (MTHFR) genotype
4. Diabetes mellitus (DM)
Answer: 3 - Methylenetetrahydrofolate reductase (MTHFR) genotype
Explanations:
Elevated homocysteine, increasing age, decreased LVEF, DM, elevated
CRP and hyperlipidemia were found to be significant predictors of
mortality in patients with severe CAD
Severe CAD was defined as >70% stenosis on angiogram
Homozygosity for the MTHFR genotype was weakly predictive of
homocysteine levels but not mortality

Go to the next page if you knew the correct answer, or click the link images
below to further research the concepts in this question (if desired).

Research Concepts:
Mortality:

Risk Factors For Coronary Artery Disease:

Tap flag to report any problems with this question.


Question 1479: Who CANNOT certify time of death according to The
Uniform Anatomical Gift Act?

Choices:
1. Emergency Department physician
2. Primary care physician (PCP) of patient
3. Transplant surgeon performing the procedure
4. Consulting physician
Answer: 3 - Transplant surgeon performing the procedure
Explanations:
Any physician involved in the transplant procedure is ineligible to certify
time of death.
The patient's PCP can certify time of death.
An emergency physician can certify time of death.
An organ donation may be revoked by written or oral means.

Go to the next page if you knew the correct answer, or click the link images
below to further research the concepts in this question (if desired).

Research Concepts:
Documentation:

Death Certification:

Tap flag to report any problems with this question.


Question 1480: An intensive care unit patient has developed a tracheo-
innominate artery fistula. What is the best surgical approach to this disorder?

Choices:
1. Left thoracotomy
2. Right thoracotomy
3. Median sternotomy
4. Cervical neck incision
Answer: 3 - Median sternotomy
Explanations:
A median sternotomy gives the best exposure to the innominate artery and
trachea.
During surgery, the innominate artery must be ligated and the hole in the
trachea should be closed with a muscle flap.
If a surgeon is not available, one can compress the innominate artery
against the sternum with a finger.
One may also attempt to place an endotracheal tube distal to the bleeding
site.

Go to the next page if you knew the correct answer, or click the link images
below to further research the concepts in this question (if desired).

Research Concepts:
Tracheo-innominate Artery Fistula:

Tap flag to report any problems with this question.


Question 1481: In a patient who has an impending cardiac arrest, what is
the most common complaint?

Choices:
1. Chest pain
2. Syncope
3. Hungry
4. Sleepy
Answer: 2 - Syncope
Explanations:
During a cardiac arrest, most patients will tell you that they feel lightheaded
or have a syncopal feeling.
Asystole may occur during near drowning, stroke, massive pulmonary
embolus, hyperkalemia, hypothermia, or VT.

Go to the next page if you knew the correct answer, or click the link images
below to further research the concepts in this question (if desired).

Research Concepts:
Cardiac Arrest:

Tap flag to report any problems with this question.


Question 1482: Which of the following is the agent of choice for acute
pulmonary edema?

Choices:
1. Furosemide
2. Thiazides
3. Mannitol
4. Carbonic anhydrase
Answer: 1 - Furosemide
Explanations:
The major application of loop diuretics is in the treatment of acute
pulmonary edema, where pulmonary vasodilating actions also play a role.
Loop diuretics also aid in the removal of excess body fluids.

Go to the next page if you knew the correct answer, or click the link images
below to further research the concepts in this question (if desired).

Research Concepts:
Congestive Heart Failure And Pulmonary Edema:

Tap flag to report any problems with this question.


Question 1483: Which narcotic has the shortest half-life?
Choices:
1. Methadone
2. Morphine
3. Fentanyl
4. Hydromorphone
Answer: 3 - Fentanyl
Explanations:
Fentanyl has a fast onset
It also has a short to half life

Go to the next page if you knew the correct answer, or click the link images
below to further research the concepts in this question (if desired).

Research Concepts:
Opioids:

Tap flag to report any problems with this question.


Question 1484: A patient is suspected of having aortic regurgitation
secondary to endocarditis. What is the diagnostic test of choice?

Choices:
1. ECG
2. Cardiac catheterization
3. Echocardiogram
4. CT scan
Answer: 3 - Echocardiogram
Explanations:
An echocardiogram can evaluate the severity of aortic regurgitation
Valve replacement is the next step
Antibiotic is probably indicated even if patient is not symptomatic

Go to the next page if you knew the correct answer, or click the link images
below to further research the concepts in this question (if desired).

Research Concepts:
Endocarditis, Bacterial:

Aortic Regurgitation:

Tap flag to report any problems with this question.


Question 1485: Which of the following is not an effect of narcotic
analgesics?

Choices:
1. Anti-tussive action
2. Contraction of biliary ducts
3. Sedation
4. Increased peristalsis
Answer: 4 - Increased peristalsis
Explanations:
Morphine causes constipation not increased peristalsis

Go to the next page if you knew the correct answer, or click the link images
below to further research the concepts in this question (if desired).

Research Concepts:
Opioids:

Tap flag to report any problems with this question.


Question 1486: All of the following are potential side effects of
succinylcholine EXCEPT:

Choices:
1. Hypotension
2. Bradycardia
3. Malignant hyperthermia
4. Dry mouth
Answer: 4 - Dry mouth
Explanations:
Succinylcholine is a neuromuscular blocking agent used to facilitate
endotracheal intubation and muscle relaxation
It is the shortest acting of the neuromuscular blocking agents
Side effects may include hypotension, bradycardia, and rarely malignant
hyperthermia
It can cause excessive salivation as opposed to dry mouth

Go to the next page if you knew the correct answer, or click the link images
below to further research the concepts in this question (if desired).

Research Concepts:
Succinylcholine:

Tap flag to report any problems with this question.


Question 1487: Which of the following does not affect stroke volume?
Choices:
1. Peripheral resistance
2. Heart rate
3. Preload
4. Afterload
Answer: 2 - Heart rate
Explanations:
Stroke volume is not a function of pulse

Go to the next page if you knew the correct answer, or click the link images
below to further research the concepts in this question (if desired).

Research Concepts:
Stroke Volume:

Tap flag to report any problems with this question.


Question 1488: Which of the following is not a contradiction for using
sodium nitroprusside?

Choices:
1. Liver disease
2. Vitamin B12 deficiency
3. Hypertension
4. Kidney disease
Answer: 3 - Hypertension
Explanations:
Sodium nitroprusside is indicated for elevated blood pressure.

Go to the next page if you knew the correct answer, or click the link images
below to further research the concepts in this question (if desired).

Research Concepts:
Nitroprusside:

Tap flag to report any problems with this question.


Question 1489: Which is not a characteristic of hyperkalemia?
Choices:
1. Constipation
2. Cramps
3. Nausea
4. Weakness
Answer: 1 - Constipation
Explanations:
Hyperkalemia is associated with abdominal cramps, diarrhea, and muscle
weakness

Go to the next page if you knew the correct answer, or click the link images
below to further research the concepts in this question (if desired).

Research Concepts:
Hyperkalemia:

Tap flag to report any problems with this question.


Question 1490: What medication has a more efficacious metabolite than
parental drug?

Choices:
1. Lidocaine
2. Metoprolol
3. Morphine
4. Albuterol
Answer: 3 - Morphine
Explanations:
Morphine is metabolized by liver to morphine 6 glucuronide
The metabolite is responsible for analgesic property

Go to the next page if you knew the correct answer, or click the link images
below to further research the concepts in this question (if desired).

Research Concepts:
Morphine:

Tap flag to report any problems with this question.


Question 1491: A 55 year old male is seen in the ER with an ECG
showing a repolarizing arrhythmia of varying amplitude. This arrhythmia may
be caused by what medication?

Choices:
1. Amiodarone
2. Sotalol
3. Lidocaine
4. Ticlopidine
Answer: 2 - Sotalol
Explanations:
Torsades is a ventricular arrhythmia of varying polarity.
The arrhythmia is a rapid polymorphic ventricular tachycardia with a
typical twist of the QRS complex around the isoelectric baseline.
Torsade is a side effects of certain anti arrhythmic drugs like Sotalol,
quinidine and procainamide.
Common causes of torsades include hypomagnesemia, hypokalemia and
macrolide antibiotics.

Go to the next page if you knew the correct answer, or click the link images
below to further research the concepts in this question (if desired).

Research Concepts:
Sotalol:

Tap flag to report any problems with this question.


Question 1492: Which of the following medications is contraindicated in
asthmatic patients?

Choices:
1. Esmolol
2. Atenolol
3. Nadolol
4. Metoprolol
Answer: 3 - Nadolol
Explanations:
Non selective beta blockers are contraindicated in asthmatic patients
Of the above choices, nadolol is the one with non-selective beta blocker
activity
Propranolol is another non selective beta blocker

Go to the next page if you knew the correct answer, or click the link images
below to further research the concepts in this question (if desired).

Research Concepts:
Asthma:

Beta-Blockers:

Tap flag to report any problems with this question.


Question 1493: Which medication is appropriate for a patient
experiencing alcohol withdrawal symptoms?

Choices:
1. Opioid analgesics
2. Anxiolytics
3. Benzodiazepines
4. Anticholinergic agents
Answer: 3 - Benzodiazepines
Explanations:
Benzodiazepines are used to decrease symptoms of alcohol withdrawal

Go to the next page if you knew the correct answer, or click the link images
below to further research the concepts in this question (if desired).

Research Concepts:
Alcohol Withdrawal:

Tap flag to report any problems with this question.


Question 1494: Which of the following is not among the pharmacologic
effects of morphine?

Choices:
1. Behavioral changes
2. Analgesia
3. Respiratory depression
4. Diarrhea
Answer: 4 - Diarrhea
Explanations:
Morphine can cause severe constipation and most patients require
concomitant laxative therapy
In high doses, morphine can cause apnea.
The side effects of morphine can be reversed with naloxone.

Go to the next page if you knew the correct answer, or click the link images
below to further research the concepts in this question (if desired).

Research Concepts:
Morphine:

Tap flag to report any problems with this question.


Question 1495: Which statement is false about partial atrioventricular
(AV) canal defects?

Choices:
1. The condition is associated with an ostium primum ASD
2. There is a cleft mitral valve present
3. Parachute mitral valve may be present
4. Children with partial AV canal defects develop pulmonary hypertension
early and surgery is required soon after diagnosis
Answer: 4 - Children with partial AV canal defects develop pulmonary
hypertension early and surgery is required soon after diagnosis

Explanations:
In partial AV canal defects, pulmonary hypertension is usually absent and
the optimal age of surgery is 2-4 years
In complete AV canal defect, surgery should be performed at 6 months
before pulmonary vascular disease progresses
If severe symptoms are present, surgery is indicated earlier
Cardiac catheterization will reveal an oxygen step up in both right atrium
and right ventricle in complete AV canal defect

Go to the next page if you knew the correct answer, or click the link images
below to further research the concepts in this question (if desired).

Research Concepts:
Atrioventricular Canal Defects:

Tap flag to report any problems with this question.


Question 1496: What is the main mechanism of action of nitrates?
Choices:
1. Reduce afterload
2. Reduce preload
3. Reduce pain
4. Increase SVR
Answer: 2 - Reduce preload
Explanations:
Nitrates act as vasodilators and lower oxygen demands by reducing cardiac
preload

Go to the next page if you knew the correct answer, or click the link images
below to further research the concepts in this question (if desired).

Research Concepts:
Nitrates:

Tap flag to report any problems with this question.


Question 1497: Which statement about the anomalous LAD from the PA is
NOT true?

Choices:
1. Cardiac enlargement is seen on an x ray
2. MR does not require any procedure
3. Cardiac catheterization is needed prior to surgery
4. Origin of the RCA from the PA carries has a lower morbidity than the LAD
from the PA
Answer: 3 - Cardiac catheterization is needed prior to surgery
Explanations:
The lower pressure in the RV allows for better myocardial flow and thus
origin of the RCA from the PA is better tolerated.
Cardiac catheterization is not required in all patients.
Critically ill infants can be diagnosed with an ECHO and it suffices.

Go to the next page if you knew the correct answer, or click the link images
below to further research the concepts in this question (if desired).

Research Concepts:
Anomalous Left Anterior Descending (LAD) Artery:

Tap flag to report any problems with this question.


Question 1498: Reflex tachycardia is a common feature with which
antihypertensive medications?

Choices:
1. Beta blockers
2. ACE inhibitors
3. Alpha blockers
4. Diuretics
Answer: 3 - Alpha blockers
Explanations:
All alpha-blockers have two side effects. They can cause orthostatic
hypotension and a reflex tachycardia.
The orthostatic hypotension is often so severe that many people are not able
to take high doses of alpha-blockers.
Alpha-blockers are best taken at night to avoid the hypotension.

Go to the next page if you knew the correct answer, or click the link images
below to further research the concepts in this question (if desired).

Research Concepts:
Antihypertensive Medications:

Tap flag to report any problems with this question.


Question 1499: All of the following factors are considered to be risk
factors for heparin induced thrombocytopenia (HIT) except:

Choices:
1. Age
2. Duration of therapy
3. Type of heparin used
4. Surgical patients
Answer: 1 - Age
Explanations:
There are three factors in addition to longer duration of therapy that is most
strongly associated with the development of HIT
Use of unfractionated heparin rather than low molecular weight heparin is
considered a risk factor
Surgical rather than medical patients are prone to HIT
Female rather than male patients are prone to HIT

Go to the next page if you knew the correct answer, or click the link images
below to further research the concepts in this question (if desired).

Research Concepts:
Thrombocytopenia, Heparin Induced:

Tap flag to report any problems with this question.


Question 1500: Which of the following patients in cardiogenic shock
would benefit the most from an intra-aortic balloon pump?

Choices:
1. A patient with history of cardiogenic shock who has new Q waves and
pulmonary edema
2. A trauma patient with hypotension, distended neck veins, and muffled heart
tones
3. A patient with left mainstem artery disease and poor response to IV
dopamine
4. A patient with an anteroseptal myocardial infarction with a new onset of a
cooing, systolic murmur
Answer: 4 - A patient with an anteroseptal myocardial infarction with a new
onset of a cooing, systolic murmur

Explanations:
A balloon pump is most useful to stabilize a patient with a surgically
correctable lesion that is causing heart failure, e.g., a ruptured papillary
muscle with acute mitral regurgitation.
Patients with a new myocardial infarction, in addition to severe, preexisting
cardiac disease, have a poor prognosis.
The balloon pump is not indicated for pericardial tamponade.

Go to the next page if you knew the correct answer, or click the link images
below to further research the concepts in this question (if desired).

Research Concepts:
Myocardial Infarction, Acute:

Intra-Aortic Balloon Counterpulsation:

Tap flag to report any problems with this question.


Question 1501: Which of the following would cause muscle weakness,
hemolytic anemia, and decreased cardiac output?

Choices:
1. Hypocalcemia
2. Hypophosphatemia
3. Hyponatremia
4. Hyperkalemia
Answer: 2 - Hypophosphatemia
Explanations:
Mild hypophosphatemia is asymptomatic but severe depletion can cause
muscle weakness, hemolytic anemia, and decreased cardiac output
Hypophosphatemia can be seen with chronic alcoholism or refeeding
syndrome

Go to the next page if you knew the correct answer, or click the link images
below to further research the concepts in this question (if desired).

Research Concepts:
Hypophosphatemia:

Tap flag to report any problems with this question.


Question 1502: Which of the following antiarrhythmic can cause
hemolytic anemia?

Choices:
1. Quinidine
2. Dofetilide
3. Metoprolol
4. Verapamil
Answer: 1 - Quinidine
Explanations:
Quinidine is associated with hemolytic anemia
Dofetilide may cause Torsade de pointes

Go to the next page if you knew the correct answer, or click the link images
below to further research the concepts in this question (if desired).

Research Concepts:
Quinidine:

Hemolytic Anemia:

Tap flag to report any problems with this question.


Question 1503: Which of the following is not true of cyanosis?
Choices:
1. It is usually is a sign of hypoxemia
2. It is most often detected on the eyelids
3. Peripheral vasoconstriction can cause cyanosis
4. It occurs when unoxygenated hemoglobin content reaches 5 g/dL
Answer: 2 - It is most often detected on the eyelids
Explanations:
Cyanosis is a blue or purple tinge to skin or mucous membranes that is most
often detected on finger tips and lips.
Peripheral vasoconstriction occurs with Raynaud phenomenon and can
cause peripheral cyanosis.
Evaluation of a patient with cyanosis involves determination of acuity,
measuring oxygen saturation and arterial blood gases to determine the
partial pressure of oxygen, and PO2.
Cyanosis can be caused by methemoglobin, which is oxidized hemoglobin
that cannot bind oxygen. It is associated with normal oxygen saturation but
reduced PO2.

Go to the next page if you knew the correct answer, or click the link images
below to further research the concepts in this question (if desired).

Research Concepts:
Cyanosis:

Tap flag to report any problems with this question.


Question 1504: Select the most common valve pathology in adults.
Choices:
1. Aortic regurgitation
2. Aortic stenosis
3. Mitral regurgitation
4. Mitral stenosis
Answer: 2 - Aortic stenosis
Explanations:
2 to 9% of the elderly have aortic stenosis
Aortic sclerosis is considered a precursor to aortic stenosis
By age 65, 29% of patients have aortic sclerosis and by age 75 this
increases to 37%

Go to the next page if you knew the correct answer, or click the link images
below to further research the concepts in this question (if desired).

Research Concepts:
Aortic Stenosis:

Tap flag to report any problems with this question.


Question 1505: In the treatment of myocardial infarction, low, fixed-dose
warfarin combined with low-dose aspirin (ASA):

Choices:
1. Provided significant clinical benefit
2. Provided no clinical benefit
3. Had no clinical effect
4. None of the above
Answer: 2 - Provided no clinical benefit
Explanations:
ASA and warfarin reduce cardiovascular morbidity and mortality after MI
when given alone
The Coumadin Aspirin Reinfarction Study (CARS) concluded low, fixed-
dose warfarin combined with low-dose ASA after MI provided no clinical
benefit beyond that achievable with ASA 160 mg alone
The Lancet, Volume 350, Issue 9075, Pages 389-396

Go to the next page if you knew the correct answer, or click the link images
below to further research the concepts in this question (if desired).

Research Concepts:
Acute Myocardial Infarction:

Aspirin:

Warfarin:
Tap flag to report any problems with this question.
Question 1506: In a patient with asthma and hypertension, the beta-
blocker of choice is:

Choices:
1. Pindolol
2. Propranolol
3. Isoprenaline
4. Timolol
Answer: 1 - Pindolol
Explanations:
Pindolol is a beta-blocker used to treat hypertension
Unlike the other beta blockers is slightly more specific in its actions and
does not affect the bronchi or the pancreas
It also possess intrinsic sympathomimetic activity at low doses which is of
benefit in patients with asthma and hypertension

Go to the next page if you knew the correct answer, or click the link images
below to further research the concepts in this question (if desired).

Research Concepts:
Asthma:

Hypertension:

Beta-Blockers:
Tap flag to report any problems with this question.
Question 1507: Which of these agents is associated with cinchonism?
Choices:
1. Digoxin
2. Hydralazine
3. Propranolol
4. Quinidine
Answer: 4 - Quinidine
Explanations:
Quinidine is a class 1 anti arrhythmic agent derived from the bark of
cinchona tree.
Quinidine acts by blocking the fast sodium current and lowers heart rate.
Quinidine is also an inhibitor of cytochrome P450 and can lead to high
concentration of various drugs.
Quinidine can cause cinchonism, which is described as tinnitus, hearing
loss and confusion.

Go to the next page if you knew the correct answer, or click the link images
below to further research the concepts in this question (if desired).

Research Concepts:
Cinchonism:

Tap flag to report any problems with this question.


Question 1508: A patient has an emergency room thoracotomy and once
the chest is opened, there is frank blood pouring out of a 1 cm tear on the right
ventricle. The next step is:

Choices:
1. Fire a stapler
2. Apply large nylon suture
3. Insert a foley catheter and inflate balloon
4. Place patient on heart lung machine for repair
Answer: 3 - Insert a foley catheter and inflate balloon
Explanations:
The right ventricle is thin walled and sutures can tear the heart
The safest way to stop bleeding is to place a foley and inflate the balloon
Then a pledget stitch can be placed round the tear

Go to the next page if you knew the correct answer, or click the link images
below to further research the concepts in this question (if desired).

Research Concepts:
Ventricular Repair, Surgery:

Emergency Room Thoracotomy:

Tap flag to report any problems with this question.


Question 1509: Which of the following is the nerve that loops around the
arch of aorta near ligamentum arteriosum?

Choices:
1. Trigeminal nerve
2. Right recurrent laryngeal nerve
3. Left recurrent laryngeal nerve
4. Celiac nerve
Answer: 3 - Left recurrent laryngeal nerve
Explanations:
The recurrent (inferior) laryngeal nerve is a branch of the vagus nerve.
The recurrent laryngeal nerve is responsible for laryngeal motor function
and sensation.
It is called the "recurrent" as the branches of the nerve innervate the neck
laryngeal muscles by a prolonged course.
It loops around the arch of aorta near ligamentum arteriosum.
Evolutionarily, this relates to the relation of nerves and branchial arches in
fish.

Go to the next page if you knew the correct answer, or click the link images
below to further research the concepts in this question (if desired).

Research Concepts:
Recurrent Laryngeal Nerve:

Tap flag to report any problems with this question.


Question 1510: Which of the following is an early complication of
warfarin therapy?

Choices:
1. Skin necrosis
2. Cholestatic jaundice
3. Osteoporosis
4. Anemia
Answer: 1 - Skin necrosis
Explanations:
Skin necrosis due to hypercoagulation can occur during early periods of
warfarin treatment
This is caused by rapid decline of the natural anticoagulant, Protein C
Protein C is also a vitamin K dependent factor that has a shorter half life
than other clotting factors
The other mentioned complications usually occur in long term therapy

Go to the next page if you knew the correct answer, or click the link images
below to further research the concepts in this question (if desired).

Research Concepts:
Warfarin:

Tap flag to report any problems with this question.


Question 1511: What does human insulin 70/30 contain?
Choices:
1. Regular human insulin and ultralente human insulin
2. NPH human insulin and regular human insulin
3. Insulin glargine and regular insulin
4. Lente insulin and ultra lente insulin
Answer: 2 - NPH human insulin and regular human insulin
Explanations:
Humulin 70/30 insulin is a combination of NPH and regular insulin
When compared to NPH insulin, the combination has a more rapid onset of
action and a similar duration of action

Go to the next page if you knew the correct answer, or click the link images
below to further research the concepts in this question (if desired).

Research Concepts:
Human Insulin:

Tap flag to report any problems with this question.


Question 1512: Which of the following should not be given to a patient
with respiratory acidosis?

Choices:
1. Oxygen
2. Sodium bicarbonate
3. Morphine
4. Furosemide
Answer: 2 - Sodium bicarbonate
Explanations:
In a setting of respiratory acidosis, the patient should be given oxygen,
diuretics and morphine to reduce pulmonary congestion
Bicarbonate is not indicated in respiratory acidosis

Go to the next page if you knew the correct answer, or click the link images
below to further research the concepts in this question (if desired).

Research Concepts:
Acidosis, Metabolic:

Acidosis, Respiratory:

Tap flag to report any problems with this question.


Question 1513: The highest frequency of stroke is associated with which
of the following?

Choices:
1. Ulcerated ascending aorta
2. Thickened irregular ascending aorta
3. Large mobile fragments in the ascending aorta
4. Large fragment in the arch of aorta
Answer: 3 - Large mobile fragments in the ascending aorta
Explanations:
All the above are related to stroke, but the mobile segments in the aorta are
associated with the highest incidence of stroke.
Palpation of the aorta is not sensitive technique to determine calcification or
plaques in the aorta.

Go to the next page if you knew the correct answer, or click the link images
below to further research the concepts in this question (if desired).

Research Concepts:
Aortoembolic Stroke:

Tap flag to report any problems with this question.


Question 1514: Which of the following is not a component of
endocarditis?

Choices:
1. Fever and chills
2. Amaurosis fugax
3. Janeway lesions
4. Roth spots
Answer: 2 - Amaurosis fugax
Explanations:
There are many features of infective endocarditis on physical exam. Fever
and chills are present in 90% of patients. A heart murmur is heard in about
7%, and 10% will have evidence of congestive heart failure.
Petechiae, splinter hemorrhage, Osler nodes, Janeway lesions, and Roth
spots are seen in 50% of patients.
Neurologic manifestation may include embolic stroke with focal deficit,
intracerebral hemorrhages, and multiple micro abscesses. Amaurosis fugax
is a presentation of atherosclerotic disease in the carotid artery.

Go to the next page if you knew the correct answer, or click the link images
below to further research the concepts in this question (if desired).

Research Concepts:
Endocarditis, Infectious:

Tap flag to report any problems with this question.


Question 1515: The definitive diagnosis of nosocomial pneumonia is
made by:

Choices:
1. Chest x-ray
2. Bronchoscopic lavage
3. Blood cultures
4. Tissue biopsy
Answer: 4 - Tissue biopsy
Explanations:
The definitive diagnosis of nosocomial pneumonia can only be made with
tissue biopsy, but this is rarely ever done
The diagnosis of nosocomial pneumonia is often done based on clinical
history and presentation

Go to the next page if you knew the correct answer, or click the link images
below to further research the concepts in this question (if desired).

Research Concepts:
Pneumonia, Nosocomial:

Tap flag to report any problems with this question.


Question 1516: Which of the following medications can break blood clot?
Choices:
1. Warfarin
2. TPA
3. Heparin
4. Candsartan
Answer: 2 - TPA
Explanations:
TPA a blood clot lysis agent
Can be used in MI within the first 6 hours

Go to the next page if you knew the correct answer, or click the link images
below to further research the concepts in this question (if desired).

Research Concepts:
TPA Therapy:

Tap flag to report any problems with this question.


Question 1517: When does inspiration stop with pressure support mode?
Choices:
1. Tidal volume
2. Flow below preset maximum
3. When breathing stops
4. Peak pressure
Answer: 2 - Flow below preset maximum
Explanations:
In pressure support ventilation, inspiration stops when flow falls below a
preset percentage.

Go to the next page if you knew the correct answer, or click the link images
below to further research the concepts in this question (if desired).

Research Concepts:
Pressure Support:

Mechanical Ventilation:

Tap flag to report any problems with this question.


Question 1518: What is the half-life of lidocaine 1%?
Choices:
1. 30 minutes
2. 1 hour
3. 4 hours
4. 8 hours
Answer: 2 - 1 hour
Explanations:
One percent lidocaine has duration of about one hour.

Go to the next page if you knew the correct answer, or click the link images
below to further research the concepts in this question (if desired).

Research Concepts:
Lidocaine:

Tap flag to report any problems with this question.


Question 1519: The glossopharyngeal nerve serves as an afferent to the
baroreceptors located at which of the following structures?

Choices:
1. Aortic arch
2. Carotid bifurcation
3. Brain
4. Atrium
Answer: 2 - Carotid bifurcation
Explanations:
The carotid sinus reflex sends signals to the medulla via the
glossopharyngeal nerve.

Go to the next page if you knew the correct answer, or click the link images
below to further research the concepts in this question (if desired).

Research Concepts:
Glossopharyngeal Nerve:

Tap flag to report any problems with this question.


Question 1520: When is it appropriate to manually ventilate a patient on a
mechanical ventilator?

Choices:
1. When the pressure settings are alarming
2. When the patient appears to be ready to come off the ventilator
3. To increase their oxygenation before drawing ABG's
4. If the patient has become a DNR status
Answer: 1 - When the pressure settings are alarming
Explanations:
Manually ventilating the patient enables the nurse to evaluate whether the
endotracheal tube is out of position or the patient's airway is blocked

Go to the next page if you knew the correct answer, or click the link images
below to further research the concepts in this question (if desired).

Research Concepts:
Mechanical Ventilation:

Ventilation, Management:

Tap flag to report any problems with this question.


Question 1521: What is the advantage of pressure controlled ventilation
over assist controlled ventilation?

Choices:
1. Decreased risk of hyperinflation
2. Decreased risk of respiratory acidosis
3. Better results with reduced lung compliance
4. Assists spontaneous breathing
Answer: 1 - Decreased risk of hyperinflation
Explanations:
Pressure controlled ventilation prevents over inflation
It provides lung volumes dependent on lung compliance preventing
hyperinflation and pneumothorax
It is not appropriate with intrinsic pulmonary dysfunction

Go to the next page if you knew the correct answer, or click the link images
below to further research the concepts in this question (if desired).

Research Concepts:
Pressure Controlled ventilation:

Tap flag to report any problems with this question.


Question 1522: Which of the following is a contraindication to positive
end-expiratory pressure?

Choices:
1. Alzheimer disease
2. Renal failure
3. Hypovolemia
4. All of the above
Answer: 3 - Hypovolemia
Explanations:
Hypovolemia is a decrease in circulating blood volume
It is a contraindication to the use of PEEP
PEEP compresses the heart and blood vessels decreasing venous return to
the heart resulting in low cardiac output
Alzheimer's and renal failure are not contraindications to PEEP

Go to the next page if you knew the correct answer, or click the link images
below to further research the concepts in this question (if desired).

Research Concepts:
Positive End-Expiratory Pressure (PEEP):

Hypovolemia:

Tap flag to report any problems with this question.


Question 1523: Where is aortitis most common?
Choices:
1. Descending aorta
2. Ascending aorta
3. Abdominal aorta
4. Thoracic aorta
Answer: 2 - Ascending aorta
Explanations:
Aortitis means inflammation of the aorta and is most common in the
ascending aorta.
The two most common causes of aortitis include infections and connective
tissue disorders.
Aortitis can result in aortic dilatation and aortic insufficiency.
Common infections that cause aortitis include syphilis, gonorrhea,
tuberculosis, hepatitis B, and fungus.

Go to the next page if you knew the correct answer, or click the link images
below to further research the concepts in this question (if desired).

Research Concepts:
Aortitis:

Tap flag to report any problems with this question.


Question 1524: Which is NOT a common initial sign of hemolytic
transfusion reaction?

Choices:
1. Hypotension
2. Hemoglobinuria
3. Bronchospasm
4. Jaundice
Answer: 4 - Jaundice
Explanations:
Jaundice may present as a chronic sign of hemolytic anemia.

Go to the next page if you knew the correct answer, or click the link images
below to further research the concepts in this question (if desired).

Research Concepts:
Transfusion Reactions:

Tap flag to report any problems with this question.


Question 1525: A 63 year old is seen in the ER with chest pain. After
placement of a swan ganz catheter, the following parameters are obtained. The
blood pressure is 70/40, Pulse 125, Pulmonary Capillary Wedge Pressure 28 (H,
normal is 10-14), Systemic Vascular Resistance 2000 dynes sec cm (normal is
800-900) and a central venous pressure of 12. How would the shock would be
categorized?

Choices:
1. Septic Shock
2. Hypovolemic Shock
3. Neurogenic Shock
4. Cardiogenic Shock
Answer: 4 - Cardiogenic Shock
Explanations:
In cardiogenic shock, the heart simply is not able to pump and this results in
decreased contractility and reduced stroke volume.
Besides low cardiac output, the left ventricular filling pressures are high.
The systemic vascular resistance also increases to compensate for low
blood pressure.
The heart rate is increased and the JVP will always be high because of back
congestion.
Other features of cardiogenic shock include cool peripheries, decreased
urine output, and sweating. In people who bleed, the extremity will also be
cold and there may be diaphoresis but the JVP will also be low. In septic
shock, the extremities will be warm.

Go to the next page if you knew the correct answer, or click the link images
below to further research the concepts in this question (if desired).

Research Concepts:
Cardiogenic Shock:

Tap flag to report any problems with this question.


Question 1526: Which of the following is false about antibacterial agents?
Choices:
1. An antiseptic is used to inhibit bacterial growth on living skin/tissue
2. A disinfectant is used to kill organisms on an inanimate environment
3. Sterilization is a process used to kill organisms on instruments and surgical
sheets/gowns
4. Hand sanitizers contain 20% alcohol
Answer: 4 - Hand sanitizers contain 20% alcohol
Explanations:
Hand washing is vital in preventing transmission of many infections.
There are many different antiseptics. Alcohol is an effective agent though it
dries out the skin.
Hand sanitizers contain 60-90% alcohol.

Go to the next page if you knew the correct answer, or click the link images
below to further research the concepts in this question (if desired).

Research Concepts:
Antibiotics:

Tap flag to report any problems with this question.


Question 1527: The pulmonary artery is derived from:
Choices:
1. Aortic arch 3
2. Aortic arch 4
3. Aortic arch 5
4. Aortic arch 6
Answer: 4 - Aortic arch 6
Explanations:
The proximal portions of the pulmonary arteries start as outgrowths of the
sixth aortic (pulmonary) arches
The right sixth aortic arch arteries do not persist
The left sixth aortic arch arteries distal portions persist while in the uterus
as a shunt between the aorta and the pulmonary trunk called the ductus
arteriosus

Go to the next page if you knew the correct answer, or click the link images
below to further research the concepts in this question (if desired).

Research Concepts:
Pulmonary Artery:

Tap flag to report any problems with this question.


Question 1528: An adult with rheumatic mitral stenosis will have elevated
pressure in which of the following?

Choices:
1. Left ventricle
2. Pulmonary veins
3. Aorta
4. Peripheral arteries
Answer: 2 - Pulmonary veins
Explanations:
Pulmonary venous pressure is increased with mitral valve stenosis

Go to the next page if you knew the correct answer, or click the link images
below to further research the concepts in this question (if desired).

Research Concepts:
Mitral Stenosis:

Mitral Stenosis:

Tap flag to report any problems with this question.


Question 1529: What is the most common cause of left heart failure?
Choices:
1. Ischemia
2. Hypertension
3. Cardiac valve disease
4. Atherosclerosis
Answer: 1 - Ischemia
Explanations:
Ischemia is the most common cause of left heart failure
The most common cause of right heart failure is left heart failure.

Go to the next page if you knew the correct answer, or click the link images
below to further research the concepts in this question (if desired).

Research Concepts:
Left Ventricular Failure:

Tap flag to report any problems with this question.


Question 1530: A patient who is allergy to sulfa drugs should not take
which of the following medications?

Choices:
1. Furosemide
2. Metformin
3. Glipizide
4. Erythromycin
Answer: 1 - Furosemide
Explanations:
Furosemide is a commonly used drug that can mistakenly be given to
patients with sulfa allergy
There are a number of drugs that contain sulfonamides
Another commonly used drug is cotrimoxazole

Go to the next page if you knew the correct answer, or click the link images
below to further research the concepts in this question (if desired).

Research Concepts:
Sulfa Allergy:

Furosemide:

Tap flag to report any problems with this question.


Question 1531: Which of the following is false about the P wave on an
ECG?

Choices:
1. Represents electrical impulse originating at the atrioventricular node
2. Normally has duration of 0.11 seconds or less
3. May increase in size with atrial dilation
4. Reflects atrial depolarization
Answer: 1 - Represents electrical impulse originating at the atrioventricular
node

Explanations:
The P wave on an EKG represents an electrical impulse originating from
the sinoatrial node, not the atrioventricular node.

Go to the next page if you knew the correct answer, or click the link images
below to further research the concepts in this question (if desired).

Research Concepts:
EKG, P wave:

Tap flag to report any problems with this question.


Question 1532: Which of the following is not a cause of cor pulmonale?
Choices:
1. Massive pulmonary embolism
2. Left ventricular failure
3. Acute respiratory distress syndrome
4. Pulmonary hypertension
Answer: 2 - Left ventricular failure
Explanations:
The most common causes of acute cor pulmonale are acute respiratory
distress syndrome and pulmonary embolism
Chronic cor pulmonale can be secondary to chronic obstructive pulmonary
disease, pulmonary hypertension, blood hyperviscocity, and interstitial lung
disease
Left ventricular failure can cause right ventricular failure but not cor
pulmonale

Go to the next page if you knew the correct answer, or click the link images
below to further research the concepts in this question (if desired).

Research Concepts:
Cor Pulmonale:

Tap flag to report any problems with this question.


Question 1533: What is the most likely chromosomal abnormality in a
newborn with low set ears, rocker bottom feet, microcephaly with prominent
occiput, and cardiac death at 8 months?

Choices:
1. Trisomy 13
2. Trisomy 18
3. XXY
4. Trisomy 21
Answer: 2 - Trisomy 18
Explanations:
Trisomy 18 is Edward syndrome, which also includes mental deficiency,
ventricular septal defect, and micrognathia.
Trisomy 21 is Down syndrome.
Trisomy 13 is Patau syndrome and includes mental retardation, rocker
bottom feet, polydactyly, nervous system malformations, and cleft lip and
palate.
XXY is Klinefelter syndrome.

Go to the next page if you knew the correct answer, or click the link images
below to further research the concepts in this question (if desired).

Research Concepts:
Edward Syndrome:

Tap flag to report any problems with this question.


Question 1534: Cyclosporine is a fungal polypeptide that acts by
preventing production of:

Choices:
1. IL2
2. IL4
3. IL6
4. IL12
Answer: 1 - IL2
Explanations:
Cyclosporine prevents production of IL2 via calcineurin inhibition
Calcineurin is used for induction and maintenance immunosuppression
Adverse effects of cyclosporin include nephrotoxicity, hypertension,
hypertrichosis and hyperkalemia
Hypertrichosis and hirsutism can be reduced by switching to tacrolimus

Go to the next page if you knew the correct answer, or click the link images
below to further research the concepts in this question (if desired).

Research Concepts:
Cyclosporine:

Tap flag to report any problems with this question.


Question 1535: At which area of a flow meter ball is an accurate reading
made?

Choices:
1. Top of the ball
2. Middle of the ball
3. Bottom of the ball
4. Anywhere on the ball
Answer: 2 - Middle of the ball
Explanations:
The correct flow reading is made at the middle of the ball.

Go to the next page if you knew the correct answer, or click the link images
below to further research the concepts in this question (if desired).

Research Concepts:
Flow Meter:

Tap flag to report any problems with this question.


Question 1536: Which of the following drug can cause myopathy?
Choices:
1. Simvastatin
2. Captopril
3. Labetalol
4. Furosemide
Answer: 1 - Simvastatin
Explanations:
Statins can cause myopathy by inhibiting dolichol phosphate synthesis
Lovastatin and simvastatin are known to have higher risk of causing severe
myopathy
The complication may worsen with the use of other drugs that cause
myopathy
Liver enzyme inhibitors can raise level of statins and cause myopathy

Go to the next page if you knew the correct answer, or click the link images
below to further research the concepts in this question (if desired).

Research Concepts:
Statin Medication:

Myopathy:

Tap flag to report any problems with this question.


Question 1537: Injection of lidocaine for anesthesia is unlikely to cause
which of the following?

Choices:
1. Erythema
2. Seizures
3. Periorbital numbness
4. Methemoglobinemia
Answer: 4 - Methemoglobinemia
Explanations:
Lidocaine toxicities include periorbital numbness and seizures.

Go to the next page if you knew the correct answer, or click the link images
below to further research the concepts in this question (if desired).

Research Concepts:
Lidocaine:

Tap flag to report any problems with this question.


Question 1538: What is the duration of action of bupivacaine?
Choices:
1. 1 hour
2. 2 hours
3. 3 hours
4. 4 hours
Answer: 3 - 3 hours
Explanations:
Bupivacaine has one of the longest duration of action of all local
anesthetics.

Go to the next page if you knew the correct answer, or click the link images
below to further research the concepts in this question (if desired).

Research Concepts:
Bupivacaine:

Tap flag to report any problems with this question.


Question 1539: Annular abscess from endocarditis of the aortic valve
usually tunnel:

Choices:
1. The anterior mitral valve leaflet
2. Through into the right atrium
3. Into the septum
4. The right coronary artery
Answer: 3 - Into the septum
Explanations:
These annular abscesses can tunnel through to the septum and produce
heart block

Go to the next page if you knew the correct answer, or click the link images
below to further research the concepts in this question (if desired).

Research Concepts:
Annular Abscess Formation:

Aortic Valve Endocarditis:

Tap flag to report any problems with this question.


Question 1540: In what form is the majority of carbon dioxide carried in
blood?

Choices:
1. As bicarbonate
2. As carbaminohemoglobin
3. Dissolved in plasma
4. As carbonic anhydrase
Answer: 1 - As bicarbonate
Explanations:
60% to 70% of carbon dioxide is carried as bicarbonate

Go to the next page if you knew the correct answer, or click the link images
below to further research the concepts in this question (if desired).

Research Concepts:
Carbon Dioxide Transport:

Tap flag to report any problems with this question.


Question 1541: Heart transplant candidates undergo all of the following
pre-operative tests except:

Choices:
1. Psychological consultation
2. An electroencephalogram
3. Cardiac stress testing
4. Cytomegalovirus testing (CMV)
Answer: 2 - An electroencephalogram
Explanations:
Cardiac transplant recipients are evaluated by a social worker and
psychologists prior to transplant
Physical testing is cardiac catheterization, ECG, stress test, CMV, EBV and
HIV testing

Go to the next page if you knew the correct answer, or click the link images
below to further research the concepts in this question (if desired).

Research Concepts:
Heart Transplantation:

Tap flag to report any problems with this question.


Question 1542: The biggest disadvantage of femoral vein cannulation is:
Choices:
1. Difficult access
2. Poorly marked borders
3. Risk of contamination
4. Risk of thrombosis
Answer: 3 - Risk of contamination
Explanations:
The major disadvantage of the femoral site for venous cannulation is
contamination

Go to the next page if you knew the correct answer, or click the link images
below to further research the concepts in this question (if desired).

Research Concepts:
Femoral Vein Cannulation:

Tap flag to report any problems with this question.


Question 1543: Which agent is associated with rebound hypertension if
abruptly stopped?

Choices:
1. Metoprolol
2. Vasotec
3. Hydralazine
4. Clonidine
Answer: 4 - Clonidine
Explanations:
Clonidine is a centrally acting agent, can cause salt and water retention
Side effects of clonidine include dry mouth and severe rebound
hypertension if the drug is abruptly stopped

Go to the next page if you knew the correct answer, or click the link images
below to further research the concepts in this question (if desired).

Research Concepts:
Clonidine:

Tap flag to report any problems with this question.


Question 1544: What the most appropriate investigative test in a 75-year
old with an off midline abdominal bruit and recent development of hypertension,
not easily controlled?

Choices:
1. Contrast enhanced abdominal MRI/MRA
2. Renal ultrasound
3. Captopril renal scintigraphy
4. Abdominal radiographs
Answer: 1 - Contrast enhanced abdominal MRI/MRA
Explanations:
The clinical history raises concern for renal artery stenosis (RAS) as the
etiology for the bruit and hypertension.
RAS is potentially diagnosed with US but more easily and confidentally
diagnosed with CT or MR angiography. US is much more operator
dependent.
RAS should not be confused with renovascular hypertension. The renal
artery may be narrowed without contributing to hypertension. The
radionuclide exam with an ACEI is best for making this assessment,
perhaps prior to decision to perform an intervention (plasty, stent, surgery).
In this age group, the most common cause for RAS is atherosclerosis, but in
a younger age group fibromuscular dysplasia would also be considered.
FMD classically causes a beaded appearance to the involved vessel.

Go to the next page if you knew the correct answer, or click the link images
below to further research the concepts in this question (if desired).

Research Concepts:
Renal Artery Stenosis:

Tap flag to report any problems with this question.


Question 1545: What is the most important factor in perioperative adverse
drug reactions in the elderly population?

Choices:
1. Multiple medications
2. Reduced renal function
3. Gastric alkalinity
4. Reduced total body water
Answer: 1 - Multiple medications
Explanations:
More than 50% of elderly individuals treated with multiple medications
experience a variety of adverse side effects.
Adverse side effects include short term, long term, and idiosyncratic
reactions that affect cognition, gait, and the bodily functions.
It is estimated that polypharmacy affects more than 55% of elderly patients
and causes 8-10% of all adverse drug reactions.
Elderly patients also may have renal insufficiency, poor nutrition, liver
disease, and arthritis. These are factors that can increase the severity of drug
side effects.

Go to the next page if you knew the correct answer, or click the link images
below to further research the concepts in this question (if desired).

Research Concepts:
Perioperative Adverse Drug Reaction:

Tap flag to report any problems with this question.


Question 1546: During cardiopulmonary bypass, the risk of air embolism
is greatest during:

Choices:
1. Initiation of bypass
2. Termination of bypass
3. During intracardiac procedure
4. During flushing of vein grafts with cardioplegia
Answer: 1 - Initiation of bypass
Explanations:
The highest risk of embolism is during initiation of CPB
When the cold prime solution enters the patient at the onset of CPB, cold
gases that are dissolved in the prime, air bubbles out in the warm patient
and cause embolism
That is why there should not be a gradient >10'C between the patient and
CPB circuit
In addition, the temperature should be gradually lowered

Go to the next page if you knew the correct answer, or click the link images
below to further research the concepts in this question (if desired).

Research Concepts:
Cardiopulmonary Bypass:

Tap flag to report any problems with this question.


Question 1547: Which medication is not usually given to patients with
severe penicillin allergy?

Choices:
1. Doxycycline
2. Ceftriaxone
3. Clindamycin
4. Erythromycin
Answer: 2 - Ceftriaxone
Explanations:
Cross reactivity between penicillin and cephalosporins is actually less than
7%.
Patients who had anaphylactic reaction after penicillin use should not be
given cephalosporins
Penicillin is not related to tetracycline and macrolides so they can be given
safely for penicillin allergic patients.

Go to the next page if you knew the correct answer, or click the link images
below to further research the concepts in this question (if desired).

Research Concepts:
Penicillin Allergy:

Tap flag to report any problems with this question.


Question 1548: What does cyanosis usually indicate?
Choices:
1. Presence of left to right shunt
2. Left heart failure
3. Atrial septal defect
4. Desaturated hemoglobin
Answer: 4 - Desaturated hemoglobin
Explanations:
Cyanosis basically indicates presence of desaturated hemoglobin
It is seen at 5.0 g/dL of deoxyhemoglobin or more
This can be due to pulmonary or cardiac disease

Go to the next page if you knew the correct answer, or click the link images
below to further research the concepts in this question (if desired).

Research Concepts:
Cyanosis:

Tap flag to report any problems with this question.


Question 1549: Which of the following is NOT a typical finding in a
patient with Eisenmenger syndrome?

Choices:
1. Central cyanosis
2. Loud P2
3. Right sided fourth heart sound
4. Continuous murmur
Answer: 4 - Continuous murmur
Explanations:
Classic features of Eisenmenger syndrome include central cyanosis,
clubbing, loud P2, right sided fourth heart sound, and single S2.
Other features include an elevated A wave in CVP tracing and a short soft
systolic murmur.

Go to the next page if you knew the correct answer, or click the link images
below to further research the concepts in this question (if desired).

Research Concepts:
Eisenmenger Syndrome:

Tap flag to report any problems with this question.


Question 1550: Which is not a typical sign of cardiogenic shock?
Choices:
1. Hypotension
2. Polyuria
3. Tachycardia
4. Confusion
Answer: 2 - Polyuria
Explanations:
Signs of cardiogenic shock include hypotension, tachycardia, low urine
output, weakness, and confusion.

Go to the next page if you knew the correct answer, or click the link images
below to further research the concepts in this question (if desired).

Research Concepts:
Cardiogenic Shock:

Tap flag to report any problems with this question.


Question 1551: On which of the following receptors does norepinephrine
act?

Choices:
1. Alpha receptors
2. Beta receptors
3. Cholinergic receptors
4. Nicotinic receptors
Answer: 1 - Alpha receptors
Explanations:
Norepinephrine is a catecholamine that primarily acts on alpha-receptors.
The resulting action is an increase in blood pressure and systemic vascular
resistance.
Norepinephrine is made from dopamine by the enzyme dopamine B-
hydroxylase.
Deficiency of norepinephrine has been linked to depression.
In the intensive care unit setting norepinephrine is used a pressor agent. At
very high doses, it can lead to limb ischemia and even gangrene.
Norepinephrine is frequently used to treat shock states and has a survival
benefit over dopamine.

Go to the next page if you knew the correct answer, or click the link images
below to further research the concepts in this question (if desired).

Research Concepts:
Inotropes And Vasopressors:

Tap flag to report any problems with this question.


Question 1552: Which of the following is not true of Staphylococcus
infections?

Choices:
1. They can cause cellulitis
2. They can cause toxic shock syndrome
3. They are usually associated with a fever
4. They often release an endotoxin
Answer: 4 - They often release an endotoxin
Explanations:
Staphylococcus is a common cause of cellulitis.
S. aureus sometimes causes toxic shock syndrome, which can be fatal if not
promptly treated.
S. aureus is a gram-positive organism that releases exotoxins, not
endotoxins.
Most staphylococcus infections are associated with a fever.

Go to the next page if you knew the correct answer, or click the link images
below to further research the concepts in this question (if desired).

Research Concepts:
Staphylococcus Aureus:

Tap flag to report any problems with this question.


Question 1553: When a patient is on warfarin, what foods should be
avoided?

Choices:
1. Peas and beans
2. Salmon
3. Broccoli and avocado
4. Coffee
Answer: 3 - Broccoli and avocado
Explanations:
Foods to be avoided with warfarin therapy include green leafy vegetables,
broccoli, and avocado.
These foods tend to inhibit the actions of warfarin.
One should also avoid alcohol.
There are many oriental herbs that can affect levels of warfarin.

Go to the next page if you knew the correct answer, or click the link images
below to further research the concepts in this question (if desired).

Research Concepts:
Anticoagulation:

Tap flag to report any problems with this question.


Question 1554: Which is a normal prothrombin time (PT)?
Choices:
1. 5 seconds
2. 12 seconds
3. 28 seconds
4. 50 seconds
Answer: 2 - 12 seconds
Explanations:
The normal prothrombin time is approximately 11 to 13 seconds.

Go to the next page if you knew the correct answer, or click the link images
below to further research the concepts in this question (if desired).

Research Concepts:
Prothrombin Time:

Tap flag to report any problems with this question.


Question 1555: During intubation of a patient, which of the following
does NOT occur?

Choices:
1. Increased heart rate
2. Increased systemic vascular resistance
3. Increased blood pressure
4. Decreased wedge pressure
Answer: 4 - Decreased wedge pressure
Explanations:
There is an increase in wedge pressure during intubation.

Go to the next page if you knew the correct answer, or click the link images
below to further research the concepts in this question (if desired).

Research Concepts:
Intubation, Anesthesia:

Tap flag to report any problems with this question.


Question 1556: Which of the following most commonly causes aneurysms
of the arch of the aorta?

Choices:
1. Atherosclerosis
2. Tuberculosis
3. Syphilis
4. Congenital defects
Answer: 1 - Atherosclerosis
Explanations:
Arteriosclerotic disease is the most common cause of thoracic aneurysms.

Go to the next page if you knew the correct answer, or click the link images
below to further research the concepts in this question (if desired).

Research Concepts:
Aortic Arch Aneurysm:

Tap flag to report any problems with this question.


Question 1557: A 71-year-old male is seen in the emergency room after a
motor vehicle accident with severe, sharp chest pain radiating to his back. His
blood pressure is 190/100 mmHg and his pulse is 100 beats per minute. Chest x-
ray reveals a widened mediastinum. Before more can be done, the patient dies.
At autopsy, he has a massive amount of blood in the pericardial sac. What is the
most likely diagnosis?

Choices:
1. Pneumothorax
2. Ascending aortic dissection
3. Uremic pericarditis
4. Rupture of pericardial cyst
Answer: 2 - Ascending aortic dissection
Explanations:
Trauma more commonly causes aortic rupture, but can also cause aortic
dissection in rare instances. Dissection of the ascending aorta can result in
aortic wall rupture, hemopericardium, and tamponade. Other complications
include occlusion of the coronary ostia and severe aortic insufficiency.
Patients with aortic dissection typically present with sudden onset of severe
and tearing chest pain. Thoracic aortic dissection should be considered in
all patients with chest pain that radiates to the back and migrates to the left
chest and flanks.
Neurologic deficits are also common. Syncope may result from increased
vagal tone, hypovolemia, or arrhythmias.
Cardiac manifestations of aortic dissection include tamponade, superior
vena cava syndrome, wide pulse pressure, pulse deficits, and absent pulses.

Go to the next page if you knew the correct answer, or click the link images
below to further research the concepts in this question (if desired).

Research Concepts:
Dissection, Aortic:

Dissection, Aortic:
Tap flag to report any problems with this question.
Question 1558: Tolerance develops to one of the following medications:
Choices:
1. Nitrates
2. Nimodipine
3. Diltiazem
4. Propranolol
Answer: 1 - Nitrates
Explanations:
Tolerance develops easily for nitrate drugs because of depletion of NO
Repeated use during the day results in depletion of NO
The best method to overcome this problem is to use the drug in long
intervals
There is no tolerance that develops to calcium channel blockers

Go to the next page if you knew the correct answer, or click the link images
below to further research the concepts in this question (if desired).

Research Concepts:
Nitrates:

Tap flag to report any problems with this question.


Question 1559: In a patient with a pulmonary embolus, what is the
duration of treatment?

Choices:
1. 1 month
2. 1 week
3. 3-6 months
4. For life
Answer: 3 - 3-6 months
Explanations:
Anticoagulation therapy is generally continued for 3-6 months.
Patients with factor C or S deficiency may need long-term anticoagulation.

Go to the next page if you knew the correct answer, or click the link images
below to further research the concepts in this question (if desired).

Research Concepts:
Pulmonary Embolism:

Tap flag to report any problems with this question.


Question 1560: A patient has had a heart transplant. Which of the
following will be true about the patient's heart rate?

Choices:
1. The heart rate response to exercise will be abnormal
2. The patient will have resting bradycardia
3. The patient's exercise capacity will not be reduced
4. There is sympathetic but not parasympathetic innervation of the heart
Answer: 1 - The heart rate response to exercise will be abnormal
Explanations:
After heart transplant, the heart is not innervated
Resting tachycardia is usual
Response to exercise is limited, which in turn limits exercise potential

Go to the next page if you knew the correct answer, or click the link images
below to further research the concepts in this question (if desired).

Research Concepts:
Heart Transplant:

Tap flag to report any problems with this question.


Question 1561: Which of the following patients can safely undergo major
elective surgery requiring general anesthesia?

Choices:
1. A patient who had a myocardial infarction 10 months ago.
2. A patient with a potassium of 2.4, who is on diuretic therapy.
3. A patient with an ongoing pneumonia.
4. A patient with new onset glycosuria and polyuria.
Answer: 1 - A patient who had a myocardial infarction 10 months ago.
Explanations:
Perioperative MI is associated with a 40-70 percent risk of death.
Preoperative risk factors for death include presence of an S3, JVP
distension, age greater than 70, PVCs, MI within past 6 months, and
presence of symptomatic aortic stenosis.
Other risk factors for death include history of diabetes, preoperative Cr >
2.0, and low potassium.
An exercise stress test is the standard method for detection of myocardial
ischemia.

Go to the next page if you knew the correct answer, or click the link images
below to further research the concepts in this question (if desired).

Research Concepts:
Preoperative Evaluation And Management:

Tap flag to report any problems with this question.


Question 1562: What agent can cause severe hemorrhage when ingested
and is used as a rat poison?

Choices:
1. Aspirin
2. Cyanide
3. Low molecular weight heparin
4. Warfarin
Answer: 4 - Warfarin
Explanations:
Warfarin was initially used as rat poison and later found be an effective oral
anticoagulant.
Resistance to warfarin effects is now seen in rats.
Dosage of warfarin must be monitored and adjusted using INR.
There is risk of hemorrhage even with monitoring.

Go to the next page if you knew the correct answer, or click the link images
below to further research the concepts in this question (if desired).

Research Concepts:
Warfarin:

Tap flag to report any problems with this question.


Question 1563: What is the action of low molecular weight heparins?
Choices:
1. They act on factor Xa
2. They inhibit antithrombin III
3. They inhibit platelet aggregation
4. They stimulate plasmin
Answer: 1 - They act on factor Xa
Explanations:
Low molecular weight heparins act on factor Xa and thrombin.
Heparin binds to antithrombin III.
Advantages of low molecular weight heparin include less frequent dosing,
subcutaneous administration, no monitoring of partial thromboplastin time,
and decreased (but not eliminated) risk of heparin-induced
thrombocytopenia.

Go to the next page if you knew the correct answer, or click the link images
below to further research the concepts in this question (if desired).

Research Concepts:
Anticoagulation:

Low Molecular Weight Heparin (LMWH):

Tap flag to report any problems with this question.


Question 1564: A 55-year-old is on a treadmill to rule out coronary artery
disease. On an ECG, what level of decline in the lead would suggest that the
patient does, in fact, have coronary artery disease?

Choices:
1. 1 mm
2. 5 mm
3. 10 mm
4. Any decline
Answer: 1 - 1 mm
Explanations:
Exercise testing is sometimes used to screen patients with coronary artery
disease.
During the tread mill test, if there is a decline of 1 mm or more, one should
suspect coronary artery disease.
Exercise testing is more predictable in postmenopausal females, men over
the age of 40, and those who have hypertension.
If the exercise test is positive, then the patient may be sent for an
angiogram.

Go to the next page if you knew the correct answer, or click the link images
below to further research the concepts in this question (if desired).

Research Concepts:
Treadmill Stress Testing:

Tap flag to report any problems with this question.


Question 1565: Which of the following factors does not increase risk of
wound dehiscence?

Choices:
1. Uremia
2. Malignancy
3. Liver disease
4. Diabetes
Answer: 4 - Diabetes
Explanations:
While many factors increase risk of dehiscence, diabetes on its own is not a
risk factor
Common risk factors for wound dehiscence include age, hypoalbuminemia,
obesity, uremia, malignancy and liver disease
Additionally smoking and use of blood thinners can also increase the risk
Chemotherapeutic drugs are a common cause of wound dehiscence

Go to the next page if you knew the correct answer, or click the link images
below to further research the concepts in this question (if desired).

Research Concepts:
Wound Dehiscence:

Tap flag to report any problems with this question.


Question 1566: Concerning the differential diagnosis of chest pain,
including musculoskeletal, pericardial, myocardial, and gastrointestinal origins,
which statement is NOT true?

Choices:
1. Sharp, piercing pain radiating bilaterally to the thorax or arms and typically
occuring after movement, coughing, or sneezing, may be associated with
osteoarthritis
2. Substernal pain with a positive Hamman sign may result from trauma,
asthma, or esophageal rupture
3. Short episodes of sharp precordial pain, resolving with rest and shallow
respirations, are typical of pericarditis
4. The rub of pericarditis can be intermittent and have of up to three
components
Answer: 3 - Short episodes of sharp precordial pain, resolving with rest and
shallow respirations, are typical of pericarditis

Explanations:
Precordial catch syndrome is characterized by short, sharp, anterior chest
wall pain that resolves with rest and shallow respirations.
Pericardial pain is usually substernal, severe, acute in onset, and may
radiate into the back or neck. It is lessened by leaning forward and
worsened by coughing and breathing. EKG often shows diffuse ST
elevation.

Go to the next page if you knew the correct answer, or click the link images
below to further research the concepts in this question (if desired).

Research Concepts:
Chest Pain:

Tap flag to report any problems with this question.


Question 1567: Which of the following is false about shock?
Choices:
1. Cardiogenic shock presents with warm extremities
2. Cardiac output is decreased in cardiogenic shock
3. Hypotension and tachycardia are common
4. Neurogenic shock occurs with traumatic brain injury
Answer: 1 - Cardiogenic shock presents with warm extremities
Explanations:
Cardiogenic shock is characterized by decreased cardiac output,
hypotension, and cool extremities.
Tachycardia and hypotension are common.
Septic shock more commonly shows paradoxically warm extremities.
Progression can lead to multi-organ failure and DIC.

Go to the next page if you knew the correct answer, or click the link images
below to further research the concepts in this question (if desired).

Research Concepts:
Shock:

Tap flag to report any problems with this question.


Question 1568: In a patient with chest pain of a few hours duration, which
set of cardiac enzymes will have the MOST significance on the diagnosis?

Choices:
1. Lactate dehydrogenase
2. Creatine kinase
3. Troponin
4. C-reactive protein
Answer: 3 - Troponin
Explanations:
Troponin levels start to peak in the first 24 hours after an MI.
Troponins are most sensitive for an MI.
Lactate dehydrogenase and C-reactive protein are not very specific markers
for an MI.
CPK levels usually peak between 24-48 hours.

Go to the next page if you knew the correct answer, or click the link images
below to further research the concepts in this question (if desired).

Research Concepts:
Myocardial Infarction, Acute:

Tap flag to report any problems with this question.


Question 1569: Which is the most frequent congenital heart disorder
requiring surgery in adults?

Choices:
1. Atrial septal defect
2. Ventricular septal defect
3. Tetralogy of fallot
4. Coarctation
Answer: 1 - Atrial septal defect
Explanations:
ASD is the most frequent CHD discovered in adults.
Usually, the ASD can be closed in adults but these patients have a high
incidence of arrhythmias.
Pulmonary hypertension does develop in a few patients but takes decades.
ASD patients do not develop endocarditis.

Go to the next page if you knew the correct answer, or click the link images
below to further research the concepts in this question (if desired).

Research Concepts:
Atrial Septal Defect:

Tap flag to report any problems with this question.


Question 1570: Which of the following drugs can cause constipation?
Choices:
1. Senna
2. Hydrocodone
3. Ketorolac
4. Sumatriptan
Answer: 2 - Hydrocodone
Explanations:
Opioids are known to cause constipation.
Constipation and miosis do not disappear with long-term use of opioids.
Senna is a laxative, ketorolac is an NSAID, and sumatriptan is used for
migraines.
They generally do not cause constipation.

Go to the next page if you knew the correct answer, or click the link images
below to further research the concepts in this question (if desired).

Research Concepts:
Opioids:

Constipation:

Tap flag to report any problems with this question.


Question 1571: Which of the following is a normal serum calcium level
for healthy individuals?

Choices:
1. 4 mg per dL
2. 8 mg per dL
3. 12 mg per dL
4. 15 mg per dL
Answer: 2 - 8 mg per dL
Explanations:
Normal serum calcium is 8 mg per dL.

Go to the next page if you knew the correct answer, or click the link images
below to further research the concepts in this question (if desired).

Research Concepts:
Calcium:

Tap flag to report any problems with this question.


Question 1572: Which patient has the least risk of heparin induced
thrombocytopenia (HIT)?

Choices:
1. Patient on hirudin
2. Patient receiving heparin flushes to keep an IV catheter patent
3. Patient with a heparin-coated stent
4. Patient on low molecular weight heparin for DVT
Answer: 1 - Patient on hirudin
Explanations:
Any form of heparin can cause heparin induced thrombocytopenia.
Hirudin is not a form of heparin.

Go to the next page if you knew the correct answer, or click the link images
below to further research the concepts in this question (if desired).

Research Concepts:
Thrombocytopenia, Heparin Induced:

Tap flag to report any problems with this question.


Question 1573: What effect does vitamin E have on coagulation?
Choices:
1. No effect on patients already taking anticoagulants
2. Decreases effect of warfarin
3. Decreases levels of Vitamin K
4. Increases agglutination of platelets
Answer: 3 - Decreases levels of Vitamin K
Explanations:
Vitamin E effects coagulation by decreasing levels of vitamin K, increasing
PTT, increasing effect of Coumadin absorption
Patients taking anticoagulants and having phytonadione (K) deficiency need
to be aware of effects of Vitamin E

Go to the next page if you knew the correct answer, or click the link images
below to further research the concepts in this question (if desired).

Research Concepts:
Vitamin E:

Tap flag to report any problems with this question.


Question 1574: What is the most common origin of arterial anomaly of
origin in tetralogy of Fallot?

Choices:
1. Circumflex from RCA
2. Lad from the pulmonary artery
3. Lad from non coronary sinus
4. LAD from RCA
Answer: 4 - LAD from RCA
Explanations:
LAD from the RCA is the most common anomaly seen in TOF patients.
The anterior descending arises from the RCA in 5% of TOF patients.
It courses the anterior wall of the right ventricular outflow tract towards a
variable distance towards the pulmonary valve.

Go to the next page if you knew the correct answer, or click the link images
below to further research the concepts in this question (if desired).

Research Concepts:
Tetralogy Of Fallot:

Tap flag to report any problems with this question.


Question 1575: Which of the following statements about expanded
polytetrafluoroethylene (ePTFE) is FALSE?

Choices:
1. Gore-Tex is a type of polytetrafluoroethylene (PTFE), specifically ePTFE,
implanted in humans since 1971
2. It was first used for human vascular grafts
3. There is limited vascular ingrowth or integration into surrounding tissues
4. ePTFE consists of PTFE fibrils that are not connected
Answer: 4 - ePTFE consists of PTFE fibrils that are not connected
Explanations:
Gore-Tex is a brand of polytetrafluoroethylene used in humans since 1971.
Initially it was used for vascular graft and later facial plastic surgery.
ePTFE consists of fibrils that are woven into a mesh.

Go to the next page if you knew the correct answer, or click the link images
below to further research the concepts in this question (if desired).

Research Concepts:
Expanded Polytetrafluoroethylene:

Tap flag to report any problems with this question.


Question 1576: Which of the following is not a feature of neurogenic
shock?

Choices:
1. Increased venous capacitance
2. Decreased systemic vascular resistance
3. Increased cardiac output
4. Warm dry skin
Answer: 3 - Increased cardiac output
Explanations:
Patients with neurogenic shock are usually hypotensive with warm dry skin
The loss of sympathetic tone results in increased venous capacitance and
bradycardia though tachycardia may be an initial feature.
Neurogenic shock can be present with incomplete or complete lesions of
the spinal cord

Go to the next page if you knew the correct answer, or click the link images
below to further research the concepts in this question (if desired).

Research Concepts:
Shock, Neurogenic:

Tap flag to report any problems with this question.


Question 1577: Peripheral resistance mainly determines:
Choices:
1. Systolic blood pressure
2. Diastolic pressure
3. Pulse pressure
4. Pulmonary capillary wedge pressure
Answer: 2 - Diastolic pressure
Explanations:
Peripheral resistance mainly determines the diastolic pressure
The elasticity of the aorta is the main determinant of peripheral resistance
Pulmonary wedge pressure is an estimate of preload
The pulse pressure is the difference between SBP and DBP

Go to the next page if you knew the correct answer, or click the link images
below to further research the concepts in this question (if desired).

Research Concepts:
Peripheral Vascular Resistance:

Tap flag to report any problems with this question.


Question 1578: Which is an advantage of morphine over fentanyl?
Choices:
1. Longer duration of action
2. Less vasodilatation
3. Less histamine release
4. Faster onset
Answer: 1 - Longer duration of action
Explanations:
Morphine has a longer duration of action than fentanyl.

Go to the next page if you knew the correct answer, or click the link images
below to further research the concepts in this question (if desired).

Research Concepts:
Morphine:

Fentanyl:

Tap flag to report any problems with this question.


Question 1579: In the United States, what is the leading cause of death in
men?

Choices:
1. Trauma
2. Cancer
3. Heart disease
4. Stroke
Answer: 3 - Heart disease
Explanations:
Heart disease is the leading cause of death in men in the United States.
Cancer is the second most common cause of death in men.

Go to the next page if you knew the correct answer, or click the link images
below to further research the concepts in this question (if desired).

Research Concepts:
Coronary Artery Disease:

Cause of Death:

Tap flag to report any problems with this question.


Question 1580: Which of the following is appropriate if elder neglect or
abuse is suspected?

Choices:
1. Discuss your concerns with the individual's family
2. Confirm concerns with the individual before reporting
3. No action is needed if the adult caretaker is competent
4. The concern must be reported to the appropriate state agency
Answer: 4 - The concern must be reported to the appropriate state agency
Explanations:
Most states require that suspected neglect or abuse of the elderly be
reported to the appropriate state agency.

Go to the next page if you knew the correct answer, or click the link images
below to further research the concepts in this question (if desired).

Research Concepts:
Elder Abuse:

Tap flag to report any problems with this question.


Question 1581: Select the most appropriate initial treatment for severe
hypertension in an asthmatic.

Choices:
1. Beta blockers
2. Nitroprusside
3. Sublingual nifedipine
4. Furosemide
Answer: 2 - Nitroprusside
Explanations:
Severe hypertension can be treated with beta blockers or vasodilators
Beta blockers are contraindicated in asthmatics
Nitroprusside would be the drug of choice
Sublingual nifedipine was once a favored method for lowering blood
pressure but is no longer used

Go to the next page if you knew the correct answer, or click the link images
below to further research the concepts in this question (if desired).

Research Concepts:
Hypertension:

Asthma:

Tap flag to report any problems with this question.


Question 1582: Which of the following is NOT part of management of
coronary spasm during PTCA?

Choices:
1. Nitrates
2. Calcium Channel Blockers
3. Anticholinergic therapy
4. Emergency bypass
Answer: 1 - Nitrates
Explanations:
Spasm in the distal microvascular bed does not respond to nitrates.

Go to the next page if you knew the correct answer, or click the link images
below to further research the concepts in this question (if desired).

Research Concepts:
Percutaneous Transluminal Coronary Angioplasty:

Tap flag to report any problems with this question.


Question 1583: Stenting during acute myocardial infarction is better than
percutaneous transluminal coronary angioplasty alone in terms of:

Choices:
1. Mortality
2. Reinfarction
3. Progression to coronary artery bypass grafting
4. All of the above
Answer: 4 - All of the above
Explanations:
Stenting improved 6-month cardiac free events by 15% when compared
with PTCA alone

Go to the next page if you knew the correct answer, or click the link images
below to further research the concepts in this question (if desired).

Research Concepts:
Acute Myocardial Infarction:

Percutaneous Transluminal Coronary Angioplasty:

Intracoronary Stents:

Tap flag to report any problems with this question.


Question 1584: After cardiopulmonary bypass, which compartment is
increased in fluids?

Choices:
1. Interstitial
2. Intracellular
3. Extracellular
4. Intravascular
Answer: 1 - Interstitial
Explanations:
Hemodilution dilutes plasma proteins and decreases colloid osmotic
pressure and also increases capillary permeability.
This results in accumulation of extravascular fluid. There is increased
pulmonary interstitial fluid without a rise in left atrial pressure.
There is also increased tissue and peripheral edema.

Go to the next page if you knew the correct answer, or click the link images
below to further research the concepts in this question (if desired).

Research Concepts:
Cardiopulmonary Bypass:

Tap flag to report any problems with this question.


Question 1585: From whom should consent be obtained for a procedure to
be done on a mentally ill patient?

Choices:
1. Patient
2. Patient's court-appointed guardian
3. Patient's parents
4. Patient's spouse
Answer: 1 - Patient
Explanations:
Mentally ill patients do not lose their right for making health decisions if
legally competent. Most people with a diagnosed mental illness are legally
competent, and this presumption should always be made prima facie
Remember that competence is a legal and not a medical designation
The wishes of the parents (or even of the physician are not relevant)
As a legally competent individual, the patient is unlikely to have a court-
appointed guardian

Go to the next page if you knew the correct answer, or click the link images
below to further research the concepts in this question (if desired).

Research Concepts:
Informed Consent:

Mental Illness:

Tap flag to report any problems with this question.


Question 1586: Trimethaphan will most likely cause one of the following
responses:

Choices:
1. Accommodation
2. Hypertension
3. Pupillary constriction
4. Tachycardia
Answer: 4 - Tachycardia
Explanations:
Trimethaphan is a ganglionic blocker
It is administered during surgery to control hypotension and to minimize
blood loss
Parasympathetic stimulation of the heart causes bradycardia; removal of
this would result in tachycardia

Go to the next page if you knew the correct answer, or click the link images
below to further research the concepts in this question (if desired).

Research Concepts:
Trimethaphan:

Tap flag to report any problems with this question.


Question 1587: Mexiletine is in which of the following classes?
Choices:
1. Class 1 anti-arrhythmic
2. Class 2 anti-arrhythmic
3. Class 3 anti-arrhythmic
4. Class 4 anti-arrhythmic
Answer: 1 - Class 1 anti-arrhythmic
Explanations:
Mexiletine is a Group 1B anti-arrhythmic drug.
Mexiletine is like lidocaine, but orally active.

Go to the next page if you knew the correct answer, or click the link images
below to further research the concepts in this question (if desired).

Research Concepts:
Antiarrhythmic Medication:

Tap flag to report any problems with this question.


Question 1588: Early stages of atherosclerotic plaques contain which
immune cell in greatest number?

Choices:
1. B-cells
2. T-cells
3. Macrophages
4. Eosinophils
Answer: 3 - Macrophages
Explanations:
Fatty streaks contain macrophages
B-cells manufacture immunoglobulins
T-cells are involved in cellular immunity and humeral immunity

Go to the next page if you knew the correct answer, or click the link images
below to further research the concepts in this question (if desired).

Research Concepts:
Atherosclerosis:

Tap flag to report any problems with this question.


Question 1589: In patients who suffer blunt trauma to the chest, what
percentage of patients with descending aorta injury survive?

Choices:
1. 1%
2. 8%
3. 15%
4. 50%
Answer: 3 - 15%
Explanations:
The majority of patients who suffer blunt trauma to the thoracic aorta die at
the site of trauma. Exsanguinating bleeding is common.
Among those who are brought to the emergency room, expedient surgery
salvages about 15% of patients.
Complications of the thoracic aorta include paraplegia, stroke, renal failure,
and bowel ischemia.
Many of these individuals have other organ system damage which leads to a
higher morbidity.

Go to the next page if you knew the correct answer, or click the link images
below to further research the concepts in this question (if desired).

Research Concepts:
Aorta, Trauma:

Tap flag to report any problems with this question.


Question 1590: A patient on prolonged mechanical ventilation is given
nutritional support. He has failed weaning repeatedly and his carbon dioxide
levels are markedly elevated. Which of the following may be responsible for the
elevated carbon dioxide levels?

Choices:
1. Excessive long chain fatty acids
2. Excessive nitrogenous amino acids
3. Excessive carbohydrates
4. Excessive cholesterol
Answer: 3 - Excessive carbohydrates
Explanations:
Remember, CO2 levels may increase with carbohydrate feedings.
Low carbohydrates and high fat nutrition are required to decrease CO2
production.

Go to the next page if you knew the correct answer, or click the link images
below to further research the concepts in this question (if desired).

Research Concepts:
Ventilator Weaning:

Tap flag to report any problems with this question.


Question 1591: Which beta-blocker has the most antiarrhythmic activity?
Choices:
1. Metoprolol
2. Propranolol
3. Sotalol
4. Labetalol
Answer: 3 - Sotalol
Explanations:
All beta-blockers have intrinsic antiarrhythmic properties by blocking the
arrhythmic potential of adrenergic stimulation
Sotalol is also a class III antiarrhythmic prolonging the cardiac potential
and refractory period
It is indicted for ventricular arrhythmias and for maintenance of sinus
rhythm in patients with recurrent atrial fibrillation and flutter

Go to the next page if you knew the correct answer, or click the link images
below to further research the concepts in this question (if desired).

Research Concepts:
Sotalol:

Tap flag to report any problems with this question.


Question 1592: An aneurysm just distal to the left subclavian artery in a
young male living in an urban city is most likely due to:

Choices:
1. Uncontrolled hypertension
2. Marfan
3. Penetrating trauma
4. Motor vehicle accident
Answer: 4 - Motor vehicle accident
Explanations:
When neither death nor surgery follows acute traumatic aortic transection,
the disruption in at least part of the aortic circumference at the level of the
ligamentum, allows an extravasation of blood into the periaortic area
This blood may remain in communication with the aorta and form a
pulsating hematoma contained by adventitial tissue
Because of the increased wall stress, the false aneurysm begins to enlarge
and form a typical thoracic aneurysm

Go to the next page if you knew the correct answer, or click the link images
below to further research the concepts in this question (if desired).

Research Concepts:
Thoracic Aorta Aneurysm:

Tap flag to report any problems with this question.


Question 1593: What is seen in a patient with Kussmaul sign?
Choices:
1. Falling of JVP
2. Rising of JVP
3. Pulsatile flow in the JVP
4. A flat JVP
Answer: 2 - Rising of JVP
Explanations:
Kussmaul sign is elevation of JVP with inspiration
Normally the JVP falls with inspiration due to increased pressure in
thoracic cavity
Kussmaul sign suggest impaired right ventricle filling or poorly complaint
myocardium or pericardium

Go to the next page if you knew the correct answer, or click the link images
below to further research the concepts in this question (if desired).

Research Concepts:
Kussmaul Sign:

Tap flag to report any problems with this question.


Question 1594: The metabolism of labetalol takes place in which location?
Choices:
1. Kidney
2. Serum
3. Liver
4. GI mucosa
Answer: 3 - Liver
Explanations:
Labetalol is conjugated in the liver and excreted by the kidneys.

Go to the next page if you knew the correct answer, or click the link images
below to further research the concepts in this question (if desired).

Research Concepts:
Labetalol:

Drug Metabolism:

Tap flag to report any problems with this question.


Question 1595: Which is the least likely cause of lung atelectasis?
Choices:
1. Tumors
2. Aspiration of a foreign body
3. Thick secretions
4. CPAP
Answer: 4 - CPAP
Explanations:
Atelectasis is collapse of a portion or all of a lung
Tumors, foreign bodies and thick mucus can cause complete obstruction
leading to atelectasis distally
Bronchoscopy, CXR and CT scan can delineate causes of atelectasis
Pneumonia is a frequent complication if obstruction is not relieved in a
timely manner

Go to the next page if you knew the correct answer, or click the link images
below to further research the concepts in this question (if desired).

Research Concepts:
Atelectasis:

Tap flag to report any problems with this question.


Question 1596: A patient with a coronary artery disease also has a lung
mass. What is the best technical approach to a combined procedure?

Choices:
1. Coronary artery bypass graft (CABG) first, then lung resection, then
protamine
2. Lung resection first, then CABG, then protamine
3. CABG first, then protamine followed by lung resection
4. Lung resection, then protamine followed by CABG
Answer: 3 - CABG first, then protamine followed by lung resection
Explanations:
Coronary artery bypass grafting (CABG) is generally done first, then
protamine is given and, finally, the lung mass is resected.

Go to the next page if you knew the correct answer, or click the link images
below to further research the concepts in this question (if desired).

Research Concepts:
Lung Mass:

Revascularization Procedures In Coronary Artery


Disease:

Tap flag to report any problems with this question.


Question 1597: The most common cause of constrictive pericarditis
worldwide is:

Choices:
1. Tuberculosis
2. Viral
3. Bacterial
4. Myocardial infarction
Answer: 1 - Tuberculosis
Explanations:
Tuberculosis is the most common cause of constrictive pericarditis
wordwide
In the U.S. the most common causes, in order, are idiopathic, postoperative,
and radiation

Go to the next page if you knew the correct answer, or click the link images
below to further research the concepts in this question (if desired).

Research Concepts:
Constrictive Pericarditis:

Tuberculosis:

Tap flag to report any problems with this question.


Question 1598: Which of the following is associated with the onset of
premature ventricular contractions (PVCs)?

Choices:
1. Hypomagnesemia
2. Aortic valve stenosis
3. Hyperkalemia
4. Mitral valve stenosis
Answer: 1 - Hypomagnesemia
Explanations:
Hypomagnesemia is associated with the onset of PVCs
Asymptomatic PVCs do not require treatment
The drug of choice for treatment of symptomatic PVCs is lidocaine

Go to the next page if you knew the correct answer, or click the link images
below to further research the concepts in this question (if desired).

Research Concepts:
Premature Ventricular Contractions:

Hypomagnesemia:

Tap flag to report any problems with this question.


Question 1599: Which patient is best treated with an inferior vena cava
filter?

Choices:
1. A 65 year old male with a deep venous thrombosis after knee replacement
2. A 72 year old female with a pulmonary embolism after abdominal
hysterectomy
3. A 55 year old male with a pulmonary embolism and gastrointestinal
bleeding after CABG
4. A 62 year old male with a deep venous thrombosis after radical
prostatectomy
Answer: 3 - A 55 year old male with a pulmonary embolism and
gastrointestinal bleeding after CABG

Explanations:
The patient with gastrointestinal bleeding has a contraindication to
anticoagulation and should be treated with IVC filter.
The most common indication for IVC filter placement is diagnosis of DVT
or PE in someone who has a contraindication for anticoagulation. This
accounts for 38-77% of IVC filter placements.
Anticoagulation contraindications include bleeding diathesis, active internal
bleeding, hemorrhagic stroke, major or multiple surgery or trauma, recent
neurosurgical procedures or other intracranial neoplasm, pregnancy, poor
patient compliance with medications, or a patient with unsteady gait or
tendency to fall,
Additional indications for IVC filter placement, in those without
contraindications for anticoagulation, include anticoagulation failure (PE
post heparin), and patients with free-floating iliocaval or pelvic DVT. It can
also be placed as part of PE prophylaxis in patients with high risk of DVT
or high risk of morbidity from PE (pulmonary hypertension, poor cardiac
reserve, cancer and certain trauma patients).

Go to the next page if you knew the correct answer, or click the link images
below to further research the concepts in this question (if desired).

Research Concepts:
Vascular Filter, Inferior Vena Cava:

Tap flag to report any problems with this question.


Question 1600: Which of following laboratory values is NOT measured
when sampling arterial blood gas?

Choices:
1. Lactate
2. pH
3. PaCO2
4. HCO3
Answer: 1 - Lactate
Explanations:
Standard ABG reports pH, PaCO2, PaO2, HCO3, O2 saturation, and base
excess.

Go to the next page if you knew the correct answer, or click the link images
below to further research the concepts in this question (if desired).

Research Concepts:
Arterial Blood Gas:

Tap flag to report any problems with this question.


Section 9
Question 1601: Which does not an increased risk for death during
anesthesia?

Choices:
1. S3 gallop
2. MI 2 months ago
3. Age > 70
4. Intermittent run of 2-3 PVCs
Answer: 4 - Intermittent run of 2-3 PVCs
Explanations:
PVCs are a relatively common occurrence where the heartbeat is initiated
by the ventricles rather than the SA node
A PVC can be perceived as a skipped beat as a palpitation in the chest
PVCs usually do not pose any danger and most are asymptomatic
PVCs is more frequently in elderly, after ischemia and myocarditis

Go to the next page if you knew the correct answer, or click the link images
below to further research the concepts in this question (if desired).

Research Concepts:
Premature Ventricular Contraction:

Arrythmias:

Tap flag to report any problems with this question.


Question 1602: A patient with only upper extremity hypertension may
have what condition?

Choices:
1. Aortic coarctation
2. Aortic stenosis
3. Cardiac hypertrophy
4. Pulmonary fibrosis
Answer: 1 - Aortic coarctation
Explanations:
Aortic coarctation is characterized by upper extremity hypertension

Go to the next page if you knew the correct answer, or click the link images
below to further research the concepts in this question (if desired).

Research Concepts:
Aortic Coarctation:

Tap flag to report any problems with this question.


Question 1603: Which of the folllowing properties makes methohexital a
short acting drug?

Choices:
1. Water solubility
2. Weak acidity
3. Lipid solubility
4. Weak alkalinity
Answer: 3 - Lipid solubility
Explanations:
Methohexital is lipid soluble and redistributes rapidly from the serum into
adipose tissue.
It is a barbiturate derivative used for anesthesia without analgesia.

Go to the next page if you knew the correct answer, or click the link images
below to further research the concepts in this question (if desired).

Research Concepts:
Methohexital:

Tap flag to report any problems with this question.


Question 1604: What is the most important next step in management of a
patient having an acute myocardial infarction who has a BP of 165/80, a pulse of
75, elevated troponin, T wave inversions on ECG and has been given heparin,
morphine, aspirin, and an oxygen mask?

Choices:
1. Administer benzodiazepines
2. Administer IV beta-blocker
3. Administer IV ACE inhibitor
4. Repeat cardiac enzymes
Answer: 2 - Administer IV beta-blocker
Explanations:
The patient is having a non-ST elevation myocardial infarction and a beta-
blocker should be given due to their effect of reducing myocardial oxygen
demand and ventricular wall tension and have been shown to decrease
mortality
Initial therapy for acute coronary syndrome is morphine, oxygen, aspirin
and nitrates as well as anticoagulation in NSTEMI
Cardiac enzymes are usually repeated after 6 to 8 hours

Go to the next page if you knew the correct answer, or click the link images
below to further research the concepts in this question (if desired).

Research Concepts:
Myocardial Infarction, Acute:

Tap flag to report any problems with this question.


Question 1605: Which of the following statements regarding mitral valve
stenosis is not true?

Choices:
1. Presents with diastolic murmur
2. Most common symptom is dyspnea
3. Population most at risk is age > 60
4. Rheumatic fever is the leading cause
Answer: 3 - Population most at risk is age > 60
Explanations:
Mitral valve stenosis usually occurs between age 20 to 40
The most common cause is rheumatic fever

Go to the next page if you knew the correct answer, or click the link images
below to further research the concepts in this question (if desired).

Research Concepts:
Mitral Stenosis:

Tap flag to report any problems with this question.


Question 1606: A patient on a mechanical ventilator, is fighting the
machine, and has elevated peak airway pressures. What medication should be
used?

Choices:
1. Benzodiazepines
2. Vecuronium
3. Barbiturates
4. Baclofen
Answer: 2 - Vecuronium
Explanations:
Vecuronium is a competitive antagonist for acetylcholine at the
neuromuscular junction.
It is non-depolarizing and can be reversed by cholinesterase inhibitors
including pyridostigmine, neostigmine, and edrophonium.
It is approximately 1/3 more potent than pancuronium but slightly shorter
acting but duration of action increases with increasing dose.
It is useful to prevent patients from fighting mechanical ventilation.

Go to the next page if you knew the correct answer, or click the link images
below to further research the concepts in this question (if desired).

Research Concepts:
Ventilator Management:

Vecuronium:

Tap flag to report any problems with this question.


Question 1607: How is heparin usually reversed?
Choices:
1. Blood transfusion
2. Fresh frozen plasma
3. Protamine
4. Vitamin K
Answer: 3 - Protamine
Explanations:
Vitamin K is the antidote for warfarin toxicity.
Protamine is the antidote for heparin toxicity.
EDTA is the antidote for metal toxicity.
Flumazenil is the antidote for benzodiazepine toxicity.

Go to the next page if you knew the correct answer, or click the link images
below to further research the concepts in this question (if desired).

Research Concepts:
Protamine:

Tap flag to report any problems with this question.


Question 1608: Generally, what is the first symptom in patients with aortic
stenosis?

Choices:
1. Angina
2. Dyspnea
3. Syncope
4. Arrhythmias
Answer: 1 - Angina
Explanations:
It is important to know all the symptoms and signs of mitral stenosis, mitral
regurgitation, aortic stenosis, and aortic regurgitation. The first symptom in
aortic stenosis is usually angina. Syncope is the second symptom and
dyspnea is the third symptom.
Surgery is done for patients who are symptomatic, for patients who have a
peak pressure gradient of 50 mmHg, and for patients who are asymptomatic
but have severe stenosis.
These patients need a graded exercise test, which, if positive, indicates
surgery. If surgery is not done, the right atrial pressure will increase and
hepatomegaly will result. This is the end stage of aortic stenosis.
On an x-ray, calcium may be seen in the left ventricular silhouette. Post
stenotic aortic dilatation may be visible, as well.

Go to the next page if you knew the correct answer, or click the link images
below to further research the concepts in this question (if desired).

Research Concepts:
Aortic Stenosis:

Tap flag to report any problems with this question.


Question 1609: A patient had a myocardial infarction 6 weeks ago. He is
in cardiac rehab and can walk 2 miles in an hour without pain. He is concerned
about sexual relations. What advice should be provided to the patient by the
rehab therapist?

Choices:
1. The patient should be told that he should have no problems
2. The patient should be advised that he should discuss this with his physician
3. The patient should be informed that the energy required for sexual activity
is the same as walking at 3 to 4 miles per hour
4. The patient should be referred to a psychologist
Answer: 3 - The patient should be informed that the energy required for
sexual activity is the same as walking at 3 to 4 miles per hour

Explanations:
The cardiac rehab therapist should be comfortable in discussing the cardiac
requirements for sexual activity
Sexual intercourse requires 3 to 4 METS, about the same as walking 3 to 4
miles per hour

Go to the next page if you knew the correct answer, or click the link images
below to further research the concepts in this question (if desired).

Research Concepts:
Sexual Intercourse:

Cardiac Rehabilitation:

Tap flag to report any problems with this question.


Question 1610: Histamine release is frequently seen with which narcotic?
Choices:
1. Fentanyl
2. Morphine
3. Percocet
4. Pethidine
Answer: 2 - Morphine
Explanations:
Morphine is associated with sedation, constipation, nausea, and respiratory
depression
It produces vasodilatation due to release a of histamine

Go to the next page if you knew the correct answer, or click the link images
below to further research the concepts in this question (if desired).

Research Concepts:
Morphine:

Tap flag to report any problems with this question.


Question 1611: A patient with chest pain. The ECG shows ST segment
elevation in the anterior leads. What is the most appropriate next step?

Choices:
1. Obtain a stat chest x-ray
2. Administer nitroglycerin
3. Perform a stat echocardiogram
4. Perform an emergency cardiac catheterization
Answer: 2 - Administer nitroglycerin
Explanations:
Initial treatment of patients with an acute MI includes supplemental
oxygen, nitroglycerin, aspirin, and morphine (for pain relief).

Go to the next page if you knew the correct answer, or click the link images
below to further research the concepts in this question (if desired).

Research Concepts:
Myocardial Infarction:

Tap flag to report any problems with this question.


Question 1612: What is the current recommendation level for LDL in
high-risk patients?

Choices:
1. Less than 150 mg/dL
2. Less than 100 mg/dL
3. Less than 70 mg/dL
4. Less than 50 mg/dL
Answer: 3 - Less than 70 mg/dL
Explanations:
The American Heart Association recommends LDL to be lower than 70 in
high risk patients
In average risk patients, normal is <130 mg/dL
Moderate elevation is 130 to160 mg/dL

Go to the next page if you knew the correct answer, or click the link images
below to further research the concepts in this question (if desired).

Research Concepts:
Hyperlipidemia:

Tap flag to report any problems with this question.


Question 1613: Select the primary cause of constrictive pericarditis
worldwide.

Choices:
1. Tuberculosis
2. Idiopathic
3. Post-surgical
4. Parasites
Answer: 1 - Tuberculosis
Explanations:
Tuberculosis is the most common cause of constrictive pericarditis
worldwide
Idiopathic causes are the second most common cause

Go to the next page if you knew the correct answer, or click the link images
below to further research the concepts in this question (if desired).

Research Concepts:
Constrictive Pericarditis:

Tap flag to report any problems with this question.


Question 1614: What is a normal blood pressure change with inspiration?
Choices:
1. Increases by 2 cm H2O
2. Increases by 10 cm H2O
3. Decreases by 2 cm H2O
4. Decreases by 10 cm H2O
Answer: 3 - Decreases by 2 cm H2O
Explanations:
Inspiration normally result in a drop of blood pressure
Less than 2 cm H2O is acceptable

Go to the next page if you knew the correct answer, or click the link images
below to further research the concepts in this question (if desired).

Research Concepts:
Cardiopulmonary Physiology:

Tap flag to report any problems with this question.


Question 1615: Which is a feature not associated with aortic
regurgitation?

Choices:
1. Austin-Flint murmur
2. Corrigan's pulse
3. De Musset sign
4. Delayed carotid pulse
Answer: 4 - Delayed carotid pulse
Explanations:
Aortic regurgitation has many features. The typical diastolic murmur is best
heard with the patient leaning forward.
Austin Flint murmur, Corrigan pulse Musset sign, Quincke sign, and Muller
signs are prominent some patients with chronic aortic reguritation.
Traube sign is presence of loud pistol shots over the femoral arteries.
Delayed carotid pulsations are a feature of aortic stenosis.

Go to the next page if you knew the correct answer, or click the link images
below to further research the concepts in this question (if desired).

Research Concepts:
Aortic Regurgitation:

Tap flag to report any problems with this question.


Question 1616: A 69-year-old frail male with congestive heart failure
suddenly develops a cold right leg. Examination reveals no pulses in the entire
right leg, which is cold to the touch. What is the next step in the management of
this patient?

Choices:
1. Administration of heparin
2. Embolectomy under local anesthesia
3. Endovascular stenting
4. Placement of a vena cava filter
Answer: 2 - Embolectomy under local anesthesia
Explanations:
Emboli from the heart are not uncommon in patients with congestive heart
failure.
The ideal treatment for this patient is an embolectomy under local
anesthesia.
The patient is not a candidate for general anesthesia and is at high risk for
death.
Some surgeons will attempt a Fogarty embolectomy if the patient is too
unstable for an angiogram.

Go to the next page if you knew the correct answer, or click the link images
below to further research the concepts in this question (if desired).

Research Concepts:
Thromboembolism:

Heart Failure, Congestive:

Tap flag to report any problems with this question.


Question 1617: Which of the following is not a part of Wells Criteria for
predicting a pulmonary embolism?

Choices:
1. Heart rate >100bpm
2. Recent surgery
3. Hemoptysis
4. Tachypnea
Answer: 4 - Tachypnea
Explanations:
The Wells Criteria was introduced to help identify patients at risk for a
pulmonary embolus.
The Wells Criteria include a prior history of PE, HR > 100, recent surgery,
clinical signs of DVT, hemoptysis, and a recent cancer.
Wells Criteria generally apply to younger patients without co-morbidity.

Go to the next page if you knew the correct answer, or click the link images
below to further research the concepts in this question (if desired).

Research Concepts:
Pulmonary Embolism:

Tap flag to report any problems with this question.


Question 1618: What is the normal blood volume in an adult?
Choices:
1. 50 mL per kg
2. 60 mL per kg
3. 70 mL per kg
4. 80 mL per kg
Answer: 3 - 70 mL per kg
Explanations:
The normal blood volume is about 70 mL per kg.

Go to the next page if you knew the correct answer, or click the link images
below to further research the concepts in this question (if desired).

Research Concepts:
Blood Volume:

Tap flag to report any problems with this question.


Question 1619: What is the recommendation for LDL cholesterol level for
patient with known coronary heart disease?

Choices:
1. 100 mg per dL
2. 120 mg per dL
3. 150 mg per dL
4. 200 mg per dL
Answer: 1 - 100 mg per dL
Explanations:
LDL should be lower than 100 mg per dL with coronary heart disease
Optimal goal of LDL less than 70 mg per dL in high risk
Triglycerides should be lowered to < 200 mg/dL and non HDL cholesterol
should be lowered to <130 mg/dL

Go to the next page if you knew the correct answer, or click the link images
below to further research the concepts in this question (if desired).

Research Concepts:
Coronary Artery Disease:

Low Density Lipoprotein:

Tap flag to report any problems with this question.


Question 1620: Which of the following is a common side effect of HMG
Co-A reductase inhibitors?

Choices:
1. Hypertension
2. Hyperkalemia
3. Myalgias
4. Paresthesias
Answer: 3 - Myalgias
Explanations:
HMG Co-A reductase inhibitors cause myalgias in 9 to 20% of patients
Rhabdomyolysis is rare

Go to the next page if you knew the correct answer, or click the link images
below to further research the concepts in this question (if desired).

Research Concepts:
HMG-CoA Reductase Inhibitors:

Tap flag to report any problems with this question.


Question 1621: Which of the following is not at an increased risk for the
development of infective endocarditis?

Choices:
1. Atrial septal defect
2. Aortic insufficiency
3. Aortic stenosis
4. Mitral regurgitation
Answer: 1 - Atrial septal defect
Explanations:
Valvular heart lesions are more commonly predisposed to the development
of infective endocarditis
Atrial septal defect is not a risk factor for infective endocarditis

Go to the next page if you knew the correct answer, or click the link images
below to further research the concepts in this question (if desired).

Research Concepts:
Endocarditis, Infectious:

Tap flag to report any problems with this question.


Question 1622: Select the ECG finding found in right ventricular
infarction.

Choices:
1. 2 mm ST elevation in V3R
2. 2 mm ST elevation in V3
3. 3 mm ST elevation in aVR
4. 3 mm ST elevation in V1R
Answer: 1 - 2 mm ST elevation in V3R
Explanations:
Patients with an inferior wall MI should have a right sided ECG
Right sided infarction shows greater than 1 mm of ST elevation in V3R or
V4R

Go to the next page if you knew the correct answer, or click the link images
below to further research the concepts in this question (if desired).

Research Concepts:
Right Ventricular Infarction:

Tap flag to report any problems with this question.


Question 1623: In an adult who has just had a permanent pacemaker
inserted, which of the following recommendations apply?

Choices:
1. No showers for 1 week
2. Avoid air travel for one month
3. Never use a microwave oven again
4. Avoid heavy lifting with the implantation side arm for 1 week after the
surgery
Answer: 4 - Avoid heavy lifting with the implantation side arm for 1 week
after the surgery

Explanations:
Post-operative instructions include this information.

Go to the next page if you knew the correct answer, or click the link images
below to further research the concepts in this question (if desired).

Research Concepts:
Pacemaker, Surgery:

Tap flag to report any problems with this question.


Question 1624: Which of the following is least likely a cause of embolic
strokes?

Choices:
1. Acute myocardial infarction
2. Mitral stenosis
3. Congestive heart failure
4. Deep venous thrombosis
Answer: 4 - Deep venous thrombosis
Explanations:
Embolic from the heart are the major cause of embolic strokes
Emboli usually result after an MI, mitral stenosis, congestive heart failure,
dilated cardiomyopathy and carotid artery disease.
Atrial fibrillation continues to be the most common causes of embolic
stroke
DVT will only cause an embolic stroke if there is a right to left shunt.

Go to the next page if you knew the correct answer, or click the link images
below to further research the concepts in this question (if desired).

Research Concepts:
Embolic Stroke:

Tap flag to report any problems with this question.


Question 1625: In a young patient suspected of having aortic stenosis,
what is the test of choice?

Choices:
1. Echocardiogram
2. Stress test
3. Coronary angiogram
4. ECG
Answer: 1 - Echocardiogram
Explanations:
In a young patient with aortic stenosis, there is a low chance of coronary
artery disease and thus an echo is sufficient to make the diagnosis.
In elderly patients with AS, there is always the chance that coronary disease
may be present and thus, a coronary angiogram is warranted.
Radionuclide tests are helpful for coronary patients but are risky in patients
with aortic stenosis.
Cardiac catheterization most accurately measures the gradient across the
aortic valve.

Go to the next page if you knew the correct answer, or click the link images
below to further research the concepts in this question (if desired).

Research Concepts:
Aortic Stenosis:

Tap flag to report any problems with this question.


Question 1626: Why is it important to keep MAP > 60 mmHg?
Choices:
1. Help rest the heart
2. Allows better perfusion to the extremities
3. Allows better perfusion to vital organs
4. Increases blood flow to the central nervous system
Answer: 3 - Allows better perfusion to vital organs
Explanations:
Vital organs of the cardiac and renal systems require a minimal MAP to
sustain adequate functioning of organs

Go to the next page if you knew the correct answer, or click the link images
below to further research the concepts in this question (if desired).

Research Concepts:
Hypotension:

Tap flag to report any problems with this question.


Question 1627: When performing tracheal suctioning, suctioning should
performed on:

Choices:
1. Withdrawal of the catheter
2. Insertion of the catheter
3. Upon insertion and withdrawal of the catheter
4. None of the above
Answer: 1 - Withdrawal of the catheter
Explanations:
Suction should occur on withdrawal of the catheter when performing
tracheal suctioning
Glove, face mask and eye protection should be worn during the procedure
Sterile technique should be observed when performing the procedure
If ventilated, the patient should be hyper oxygenated/hyperventilated before
and after the procedure

Go to the next page if you knew the correct answer, or click the link images
below to further research the concepts in this question (if desired).

Research Concepts:
Nasotracheal Suctioning:

Tap flag to report any problems with this question.


Question 1628: Which of the following is not a feature of opioid toxicity?
Choices:
1. Miosis
2. Respiratory depression
3. Hypertension
4. Euphoria
Answer: 3 - Hypertension
Explanations:
The classic triad of morphine toxicity included depressed level of
consciousness, depressed respiration, and miosis.
Others may present with drowsiness, conjunctival injection, and euphoria.
Hypotension is common with morphine due to peripheral vasodilatation.

Go to the next page if you knew the correct answer, or click the link images
below to further research the concepts in this question (if desired).

Research Concepts:
Opioid Toxicity:

Tap flag to report any problems with this question.


Question 1629: Headache is a common side effect of which of the
following drugs?

Choices:
1. Digoxin
2. Nitroglycerin
3. Verapamil
4. Propranolol
Answer: 2 - Nitroglycerin
Explanations:
Nitrates result in rapid vasodilatation and vascular headache
Acetaminophen might help if administered before taking nitrates
The other short coming of nitrates is tolerance with repeated use

Go to the next page if you knew the correct answer, or click the link images
below to further research the concepts in this question (if desired).

Research Concepts:
Nitroglycerin:

Tap flag to report any problems with this question.


Question 1630: Which of the following is most important when
calculating maintenance IV fluid for patients?

Choices:
1. Blood pressure and pulse rate
2. Hydration status mucus membranes
3. Urine output
4. Nutritional status
Answer: 3 - Urine output
Explanations:
Urine output is vital information used to assess hydration status.
Blood pressure and pulse rate can be misleadingly normal.
Mucus membranes could dry out due to other reasons such as medications
or NPO status.

Go to the next page if you knew the correct answer, or click the link images
below to further research the concepts in this question (if desired).

Research Concepts:
Fluid Management:

Tap flag to report any problems with this question.


Question 1631: What is the best screening test for detecting deep venous
thrombosis (DVT)?

Choices:
1. Venography
2. Duplex compression ultrasound
3. Venous plethysmography
4. Magnetic resonance angiography (MRA)
Answer: 2 - Duplex compression ultrasound
Explanations:
Compression ultrasound with color doppler is a very sensitive and specific
modality for the diagnosis of a lower extremity DVT.
The technique is portable, cheap, and readily available.
The study can be accomplished within minutes at the bedside.
The exam is more sensitive for DVT in the thigh and popliteal areas and
less sensitive in the lower leg. Fortunately, the DVTs most likely to
embolize to the lungs are those in the thigh and popliteal areas.

Go to the next page if you knew the correct answer, or click the link images
below to further research the concepts in this question (if desired).

Research Concepts:
Deep Vein Thrombosis:

Tap flag to report any problems with this question.


Question 1632: Which of the following findings on physical exam
suggests a right sided pneumothorax?

Choices:
1. Left basilar rales
2. Whispered pectoriloquy on left side
3. Tracheal deviation to the right
4. Hyperresonance to percussion on right side
Answer: 4 - Hyperresonance to percussion on right side
Explanations:
Pneumothorax is associated with hyperresonance with percussion as well as
decrease breath sounds on the affected side
Whispered pectoriloquy is increased volume of whisper heard with
stethoscope and is associated with consolidation in pneumonia
If there is a left tension pneumothorax, the trachea may be deviated toward
rightward
Rales are caused by fluid in the alveoli

Go to the next page if you knew the correct answer, or click the link images
below to further research the concepts in this question (if desired).

Research Concepts:
Pneumothorax:

Tap flag to report any problems with this question.


Question 1633: How do omega-3 fatty acids found in seafood oil tends to
lower triglyceride levels in the body>

Choices:
1. Preventing absorption
2. Increasing excretion
3. Altering them into harmless fatty acids
4. Reduce liver secretion of triglycerides
Answer: 4 - Reduce liver secretion of triglycerides
Explanations:
The omega-3 fatty acids in fish oil decrease triglyceride levels in the body
by reducing the liver secretion of triglycerides.
Adverse effects include eructation, infection, flu-like syndrome, dyspepsia,
taste perversion, back pain, pain, rash, and angina pectoris.
They can increase ALT without increasing AST.

Go to the next page if you knew the correct answer, or click the link images
below to further research the concepts in this question (if desired).

Research Concepts:
Omega-3 Fatty Acids:

Tap flag to report any problems with this question.


Question 1634: What is the best way to evaluate a cardiac contusion?
Choices:
1. CT scan of the chest
2. Echocardiogram
3. ECG monitoring x 24 hours
4. Cardiac enzymes
Answer: 3 - ECG monitoring x 24 hours
Explanations:
A cardiac contusion is a very common injury following blunt trauma. The
hallmark of the condition includes ischemic changes, arrhythmias, and wall
motion abnormalities seen on an electrocardiogram.
Very rarely do patients with cardiac contusion develop any clinical
symptoms or hypotension.
Measurements of cardiac enzymes and the echocardiogram are not helpful
tests in establishing the diagnosis or planning treatment.
An echocardiogram is indicated if there is a new murmur. Most patients can
be safely observed in the intensive care unit for 24 hours.

Go to the next page if you knew the correct answer, or click the link images
below to further research the concepts in this question (if desired).

Research Concepts:
Cardiac Contusion:

Tap flag to report any problems with this question.


Question 1635: During cardiopulmonary resuscitation, what is the goal
coronary perfusion pressure?

Choices:
1. 5 mmHg
2. 10 mmHg
3. 20 mmHg
4. 30 mmHg
Answer: 3 - 20 mmHg
Explanations:
Effective CPR should create a coronary perfusion pressure of at least 20
mmHg.

Go to the next page if you knew the correct answer, or click the link images
below to further research the concepts in this question (if desired).

Research Concepts:
Cardiopulmonary Resuscitation:

Tap flag to report any problems with this question.


Question 1636: How long should surgery be delayed in an infant who has
consumed formula?

Choices:
1. 2 hours
2. 4 hours
3. 6 hours
4. 8 hours
Answer: 1 - 2 hours
Explanations:
It is considered safe to proceed with surgery six hours after formula or milk.
Infants can breast feed 4 hours before surgery.
Clear liquids are allowed up to 2 hours before surgery.

Go to the next page if you knew the correct answer, or click the link images
below to further research the concepts in this question (if desired).

Research Concepts:
Preoperative Evaluation And Management:

Tap flag to report any problems with this question.


Question 1637: A cardiac surgeon is at highest risk for:
Choices:
1. Brucellosis
2. Hepatitis B
3. HIV
4. Hepatitis A
Answer: 2 - Hepatitis B
Explanations:
Healthcare workers are at high risk for hepatitis B and should be vaccinated
against the disease
Brucellosis occurs in populations like slaughterhouse workers,
veterinarians, farmers and hunters
Hepatitis A is transmitted primarily through fecal-oral route and surgeon's
risk is that of the general population
HIV is much less contagious than hepatitis B

Go to the next page if you knew the correct answer, or click the link images
below to further research the concepts in this question (if desired).

Research Concepts:
Hepatitis B:

Tap flag to report any problems with this question.


Question 1638: Which of the following is true of a competitive
antagonist?

Choices:
1. It binds to the receptor and produces no reaction
2. It causes an enzyme reaction
3. It binds to the receptor and mimics the agonist
4. It does not require a receptor and diffuses easily across the lipid bilayer
Answer: 1 - It binds to the receptor and produces no reaction
Explanations:
An agonist is receptor specific. It binds then creates a reaction.
An antagonist will bind, will not cause a reaction, and will prevent the
agonist from binding.

Go to the next page if you knew the correct answer, or click the link images
below to further research the concepts in this question (if desired).

Research Concepts:
Cellular Receptor:

Tap flag to report any problems with this question.


Question 1639: A ventricular septal defect is ideally diagnosed with?
Choices:
1. Echocardiogram
2. Arterial blood gas
3. Chest x-ray
4. Fluoroscopy
Answer: 1 - Echocardiogram
Explanations:
Echocardiogram is the standard diagnostic method to visualize a VSD

Go to the next page if you knew the correct answer, or click the link images
below to further research the concepts in this question (if desired).

Research Concepts:
Ventricular Septal Defect:

Tap flag to report any problems with this question.


Question 1640: A 65 year old is given a drug for his chest pain. Seven
hours later the patient is no longer responsive to the drug and needs to double the
dose. What is the drug most likely given to the patient?

Choices:
1. Labetalol
2. Aspirin
3. Norepinephrine
4. Nitroglycerin
Answer: 4 - Nitroglycerin
Explanations:
Nitroglycerin is widely used in medicine to dilate the coronary vessels to
treat angina or chronic heart failure.
Nitroglycerin paste should be removed at night to allow the body to restore
its responsiveness. This phenomenon of tachyphylaxis is quiet common
with nitroglycerin.
Nitroglycerin vasodilates blood vessels by increasing production of nitric
oxide. Nitroglycerin can decrease chest pain, decreases blood pressure,
increases heart rate and induces orthostatic hypotension.
Side effects include headache, painful urination and diarrhea. Headaches
are common after nitroglycerin therapy and induced by release of nitric
oxide.

Go to the next page if you knew the correct answer, or click the link images
below to further research the concepts in this question (if desired).

Research Concepts:
Nitroglycerin:

Tap flag to report any problems with this question.


Question 1641: In which of the following injuries is the Trendelenburg
position contraindicated?

Choices:
1. Pelvic trauma
2. Leg trauma
3. Chest trauma
4. Abdominal trauma
Answer: 3 - Chest trauma
Explanations:
Trendelenburg is contraindicated in head and chest trauma

Go to the next page if you knew the correct answer, or click the link images
below to further research the concepts in this question (if desired).

Research Concepts:
Chest Trauma:

Patient Positioning:

Tap flag to report any problems with this question.


Question 1642: Digoxin levels are highly increased with concurrent
administration of:

Choices:
1. Amiodarone
2. Lidocaine
3. Magnesium hydroxide
4. Aluminum salts
Answer: 1 - Amiodarone
Explanations:
Amiodarone is class III antiarrhythmic
It can result in highly increased serum digoxin levels if given concurrently
Lidocaine reduces the risk of arrhythmia in digitalis toxicity
Magnesium and aluminum salts decrease GI absorption of Digoxin

Go to the next page if you knew the correct answer, or click the link images
below to further research the concepts in this question (if desired).

Research Concepts:
Digoxin:

Amiodarone:

Drug Interactions:
Tap flag to report any problems with this question.
Question 1643: What is the most common work related source of hepatitis
B infections among healthcare workers?

Choices:
1. Accidental needle sticks
2. Direct contact with a patient's skin
3. Respiratory droplets
4. Oral fecal contamination
Answer: 1 - Accidental needle sticks
Explanations:
Accidental needle sticks are the most common source of work related
hepatitis B infections in healthcare workers.
They are also one of the most common source of HIV infections among
healthcare workers.
Hepatitis A not B is spread by oral fecal contamination.
Influenza is spread by respiratory droplets.

Go to the next page if you knew the correct answer, or click the link images
below to further research the concepts in this question (if desired).

Research Concepts:
Hepatitis B:

Needle Stick:

Tap flag to report any problems with this question.


Question 1644: Which medication should be avoided in a patient with
hypertension and gout?

Choices:
1. Calcium channel blockers
2. Diuretics
3. Angiotensin-converting enzyme inhibitors
4. Beta blockers
Answer: 2 - Diuretics
Explanations:
The use of diuretics should be avoided in patients with gout due to
possibility of further elevating uric acid levels.
This is worst with the thiazides but occurs with all classes.
There is rarely concern in patients with asymptomatic hyperuricemia.

Go to the next page if you knew the correct answer, or click the link images
below to further research the concepts in this question (if desired).

Research Concepts:
Gout:

Diuretics:

Tap flag to report any problems with this question.


Question 1645: Which foods might affect INR in the patient on warfarin?
Choices:
1. Foods with a high glycemic index
2. Salty foods
3. Green, leafy vegetables
4. Dairy-milk, cheese, butter
Answer: 3 - Green, leafy vegetables
Explanations:
Green, leafy vegetables are high in Vitamin K, which decreases the effect of
warfarin on INR

Go to the next page if you knew the correct answer, or click the link images
below to further research the concepts in this question (if desired).

Research Concepts:
Warfarin:

Food Interactions:

Tap flag to report any problems with this question.


Question 1646: With respect to the saphenous vein, which of the following
statements is not correct?

Choices:
1. Its course is constant
2. It drains into the femoral vein
3. It courses between the lateral malleolus and the Achilles tendon
4. It contains bicuspid instead of tricuspid valves
Answer: 3 - It courses between the lateral malleolus and the Achilles tendon
Explanations:
The great saphenous vein originates on the dorsum of the foot and passes
anterior to the anterior malleolus where it can be visualized.
At the knee, the great saphenous vein runs slightly posterior to the medial
epicondyle and on to the femur.
The great saphenos vein joins the femoral vein in the femoral triangle. The
saphenous nerve is a branch of the femoral nerve and runs with the
saphenous vein.
The saphenous vein cutdown is usually made just over the dorsum where
the vein is anterior to the medial malleolus.

Go to the next page if you knew the correct answer, or click the link images
below to further research the concepts in this question (if desired).

Research Concepts:
Saphenous Vein:

Tap flag to report any problems with this question.


Question 1647: Which of the following medication blocks aldosterone
receptors?

Choices:
1. Spironolactone
2. Amiloride
3. Triamterene
4. Eplerenone
Answer: 4 - Eplerenone
Explanations:
Eplerenone is a specific aldosterone receptor blocker that can be used as a
diuretic
Spironolactone antagonizes the action of aldosterone but does not block its
receptors
Same is true for the other potassium sparing drugs such as triamterene and
amiloride

Go to the next page if you knew the correct answer, or click the link images
below to further research the concepts in this question (if desired).

Research Concepts:
Eplerenone:

Tap flag to report any problems with this question.


Question 1648: In which of the following is preservation of the aortic
valve possible?

Choices:
1. Ehlers-Danlos with a dilated aortic root
2. Marfan syndrome
3. Annulo-aortic ectasia
4. Aortic dissection
Answer: 4 - Aortic dissection
Explanations:
When the aortic sinuses are involved, or if a patient has Marfan, Ehlers-
Danlos, or annulo-aortic ectasia, a composite conduit with a graft should be
placed.
No part of the ascending aorta must be left behind. However, in aortic
dissection, only the ascending aorta is replaced in about 70-80% of patients.
The goal in aortic dissection is to obliterate the intimal tear and close the
false lumen.

Go to the next page if you knew the correct answer, or click the link images
below to further research the concepts in this question (if desired).

Research Concepts:
Aortic Dissection:

Tap flag to report any problems with this question.


Question 1649: Which of the following is seen in a patient with aortic
stenosis?

Choices:
1. Systolic ejection murmur radiating to the neck
2. Delayed carotid upstroke
3. S4
4. All of them
Answer: 4 - All of them
Explanations:
Physical exam in a patient with AS may reveal a systolic ejection murmur
radiating to the neck, an S4 and a delayed carotid upstroke
A soft paradoxical S2 is often heard

Go to the next page if you knew the correct answer, or click the link images
below to further research the concepts in this question (if desired).

Research Concepts:
Aortic Stenosis:

Aortic Stenosis:

Tap flag to report any problems with this question.


Question 1650: What is the most common cause of mitral stenosis?
Choices:
1. Malaria
2. Rheumatic fever
3. Hepatitis B
4. Systemic lupus erythematosus (SLE)
Answer: 2 - Rheumatic fever
Explanations:
Rheumatic fever is the most common cause of mitral stenosis
It is a low-pitched, decrescendo and rumbling diastolic murmur heard best
in the left lateral decubitus position
Administration of diuretics helps in the treatment of mild disease
For disease where valve area is <1.5 cm2, balloon valvotomy is the
treatment of choice

Go to the next page if you knew the correct answer, or click the link images
below to further research the concepts in this question (if desired).

Research Concepts:
Mitral Stenosis:

Tap flag to report any problems with this question.


Question 1651: How does dopamine enhance renal perfusion?
Choices:
1. It increases cardiac output
2. It has a direct renal vessel vasodilator effects
3. It causes renal vasoconstriction in lower doses
4. It activates the renin-angiotensin-aldosterone system
Answer: 1 - It increases cardiac output
Explanations:
Improved renal perfusion with low dose of dopamine is a result of the
overall improvement in circulation
Cardiac output is improved with dopamine use and this directly increases
renal perfusion
Dopamine causes decreased renal perfusion with high doses
It has no direct effect on the renin-angiotensin-aldosterone system

Go to the next page if you knew the correct answer, or click the link images
below to further research the concepts in this question (if desired).

Research Concepts:
Dopamine:

Tap flag to report any problems with this question.


Question 1652: Which of the following diabetic medication is most likely
to cause symptomatic hypoglycemia?

Choices:
1. Acarbose
2. Metformin
3. Pioglitazone
4. Glipizide
Answer: 4 - Glipizide
Explanations:
Sulfonylurea antidiabetic drugs act through stimulation of insulin release
Excess stimulation with sulfonylureas such as glipizide, are common causes
of drug induced hypoglycemia
The other diabetic medications are not common causes of hypoglycemia

Go to the next page if you knew the correct answer, or click the link images
below to further research the concepts in this question (if desired).

Research Concepts:
Sulfonylureas:

Hypoglycemia:

Tap flag to report any problems with this question.


Question 1653: What is the correct sequence for adult basic CPR in the
field?

Choices:
1. Establish pulselessness, give two breaths, and begin chest compressions
2. Establish unresponsiveness, obtain assistance, position patient, establish
pulselessness, begin CPR protocol with 100 compressions per minute
3. Establish unresponsiveness, clear the airway, ventilate with two breaths,
and begin chest compressions at 80 per minute
4. Establish breathlessness, clear the airway, give two breaths, establish
pulselessness, begin chest compressions at 60 per minute
Answer: 2 - Establish unresponsiveness, obtain assistance, position patient,
establish pulselessness, begin CPR protocol with 100 compressions per minute

Explanations:
Establish unresponsiveness, obtain assistance, position patient, establish
pulselessness, begin CPR protocol with 100 compressions per minute.
The adult protocol is followed for children over 8 years old.
Hands-only CPR is advocated in the field. Ventilatory support is secondary.

Go to the next page if you knew the correct answer, or click the link images
below to further research the concepts in this question (if desired).

Research Concepts:
Cardiopulmonary Resuscitation:

Tap flag to report any problems with this question.


Question 1654: Which anti-lipid drug can lower cholesterol levels by
preventing absorption from the gut?

Choices:
1. Niacin
2. Ezetimibe
3. Fluvastatin
4. Lipitor
Answer: 2 - Ezetimibe
Explanations:
Ezetimibe causes cholesterol reduction by interference with gut absorption
of cholesterol.
Ezetimibe is most effective when combined with a statin.
It has not been shown to decrease cardiovascular events or decrease
atherosclerosis.

Go to the next page if you knew the correct answer, or click the link images
below to further research the concepts in this question (if desired).

Research Concepts:
Ezetimibe:

Tap flag to report any problems with this question.


Question 1655: Which of the following is not a cause of intensive care
unit psychosis?

Choices:
1. Physical restraints
2. More than three medications
3. Unfamiliar environment
4. Use of haloperidol
Answer: 4 - Use of haloperidol
Explanations:
Intensive care unit (ICU) psychosis is a disturbance of consciousness and
attention with a change in cognition. It has a rapid onset and often lasts
several days.
Factors that can precipitate ICU psychosis include use of physical
restraints, numerous medications, unfamiliar environment, and lack of
sleep.
Treatment involves the use of sedatives and antipsychotics such as
haloperidol.
Some patients may benefit from use of narcotics and benzodiazepines such
as midazolam.

Go to the next page if you knew the correct answer, or click the link images
below to further research the concepts in this question (if desired).

Research Concepts:
ICU psychosis:

Tap flag to report any problems with this question.


Question 1656: All of the following are signs and symptoms of chronic
obstructive pulmonary disease EXCEPT:

Choices:
1. Barrel chest deformity
2. Dyspnea on exertion
3. Clubbed fingers
4. Decreased respiratory rate
Answer: 4 - Decreased respiratory rate
Explanations:
A decreased respiratory rate is not a sign or symptom of COPD
Patients with COPD are usually tachypneic
Other sign and symptoms include barrel chest, clubbed fingers, and
prolonged expiratory phase

Go to the next page if you knew the correct answer, or click the link images
below to further research the concepts in this question (if desired).

Research Concepts:
Obstructive Pulmonary Disease, Chronic (COPD):

Tap flag to report any problems with this question.


Question 1657: What vitamin deficiency causes beriberi?
Choices:
1. Vitamin D
2. Thiamine
3. Riboflavin
4. Niacin
Answer: 2 - Thiamine
Explanations:
Thiamine deficiency causes beriberi
"Dry" beriberi causes neurologic symptoms of peripheral neuropathy or
Wernicke encephalopathy
"Wet" beriberi causes high output congestive heart failure secondary to
peripheral vasodilation
Thiamine deficiency either can be due to reduced intake or increased
requirements (e.g. pregnancy)

Go to the next page if you knew the correct answer, or click the link images
below to further research the concepts in this question (if desired).

Research Concepts:
Beriberi (Thiamine Deficiency):

Tap flag to report any problems with this question.


Question 1658: Which of the following agents can sometimes be used to
treat digoxin-induced arrhythmias?

Choices:
1. Quinidine
2. Amiodarone
3. Phenytoin
4. Sotalol
Answer: 3 - Phenytoin
Explanations:
Phenytoin is an anticonvulsant and is sometimes used to treat digoxin-
induced arrhythmias.

Go to the next page if you knew the correct answer, or click the link images
below to further research the concepts in this question (if desired).

Research Concepts:
Arrhythmias:

Digoxin:

Tap flag to report any problems with this question.


Question 1659: Which of the following statement is true regarding
pulmonary and embolism (PE)?

Choices:
1. An intermediate probability V/Q scan with high index of clinical suspicion
requires no further evaluation
2. PE is more common in females than males
3. The prevalence, after age of 50, is the same in both male and female
4. D dimer assay is highly diagnostic of a PE
Answer: 3 - The prevalence, after age of 50, is the same in both male and
female

Explanations:
Although the prevalence of PE is higher in males than females before 50
year of age, this trend resolves with age
An intermediate probability perfusion scan with a remain index of suspicion
would merit anticoagulation and a pulmonary arteriogram

Go to the next page if you knew the correct answer, or click the link images
below to further research the concepts in this question (if desired).

Research Concepts:
Pulmonary Embolism:

Tap flag to report any problems with this question.


Question 1660: CHARGE syndrome encompasses all of the following
abnormalities, except:

Choices:
1. Coloboma
2. Heart disease
3. Aniridia
4. Choanal atresia
Answer: 3 - Aniridia
Explanations:
CHARGE syndrome encompasses a wide spectrum of abnormalities caused
by a hereditary abnormality.
Coloboma, heart disease, atresia choanae, retarded growth and
development, and genital anomalies are consider part of CHARGE
syndrome.
Aniridia is not part of CHARGE syndrome.
No one feature is always present or adequate for the diagnosis of CHARGE
syndrome.

Go to the next page if you knew the correct answer, or click the link images
below to further research the concepts in this question (if desired).

Research Concepts:
Charge Syndrome:

Tap flag to report any problems with this question.


Question 1661: Which finding on the stress test usually indicates presence
of angina?

Choices:
1. Increase on blood pressure
2. Down sloping of ST segment by 1 mm
3. Presence of U wave on ECG
4. Peaked T waves
Answer: 2 - Down sloping of ST segment by 1 mm
Explanations:
Horizontal or down sloping ST segment depression of at least 1 mm is a
good indication for the presence of angina.

Go to the next page if you knew the correct answer, or click the link images
below to further research the concepts in this question (if desired).

Research Concepts:
Treadmill Stress Testing:

Tap flag to report any problems with this question.


Question 1662: A patient with which condition should be triaged to
receive medical attention first?

Choices:
1. Choking
2. Dizziness
3. Leg cramp
4. Vomiting
Answer: 1 - Choking
Explanations:
Airway obstruction is the first priority for treatment.
The ABCs of triage indicate one should immediately assess and treat
Airway, Breathing, and Cardiac problems.
The other choices do not represent immediately threats to life.
Choking is first treated by attempting a Heimlich maneuver.

Go to the next page if you knew the correct answer, or click the link images
below to further research the concepts in this question (if desired).

Research Concepts:
Choking:

Tap flag to report any problems with this question.


Question 1663: Which of the following foods should be avoided on a
sodium-restricted diet?

Choices:
1. Tomato juice
2. Bread
3. Chicken
4. Oranges
Answer: 1 - Tomato juice
Explanations:
Many canned foods and juices such as tomato juice are high in sodium and
should be avoided in a sodium-restricted diet

Go to the next page if you knew the correct answer, or click the link images
below to further research the concepts in this question (if desired).

Research Concepts:
Dietary Sodium:

Tap flag to report any problems with this question.


Question 1664: Which of the following is not a major criterion for
rheumatic fever?

Choices:
1. Chorea
2. Pancarditis
3. Subcutaneous nodules
4. Erythema nodosum
Answer: 4 - Erythema nodosum
Explanations:
Carditis is present in nearly 40% of patients and may include new onset
heart failure, pericarditis, and cardiomegaly.
Migratory polyarthritis occurs in most patients and usually involves the
large joints.
Subcutaneous nodules are painless and occur in less than 10% of patients.
Chorea occurs in 10% of cases. Erythema marginatum occurs in 5% of
patients; the rash is serpiginous and long-lasting.

Go to the next page if you knew the correct answer, or click the link images
below to further research the concepts in this question (if desired).

Research Concepts:
Rheumatic Fever, Acute:

Tap flag to report any problems with this question.


Question 1665: Which of the following are causes of cardiogenic shock?
Choices:
1. Exsanguinating hemorrhage, sepsis, spinal cord injury
2. Tension pneumothorax, flail chest, pericardial tamponade
3. Spontaneous pneumothorax, patent ductus arteriosus, pectus excavatum
4. Situs inversus, aortic stenosis, barrel chest
Answer: 2 - Tension pneumothorax, flail chest, pericardial tamponade
Explanations:
A decreased pumping ability of the heart causes cardiogenic shock
Tension pneumothorax and pericardial tamponade are mechanical causes
The most common cause of this is acute myocardial infarction

Go to the next page if you knew the correct answer, or click the link images
below to further research the concepts in this question (if desired).

Research Concepts:
Cardiogenic Shock:

Tap flag to report any problems with this question.


Question 1666: Which of the following is a common early sign of digoxin
toxicity?

Choices:
1. Tachycardia
2. Constipation
3. Dysrhythmia
4. Anorexia
Answer: 4 - Anorexia
Explanations:
Anorexia is a common early sign of digoxin toxicity.
Visual symptoms are also common early signs of toxicity and include
colored halos around bright or green lights and a green or yellow tint to
white objects.
Tachycardia is usually a later sign of digoxin toxicity. Constipation is not
usually a sign of digoxin toxicity.
Dysrhythmia is usually a later sign of digoxin toxicity.

Go to the next page if you knew the correct answer, or click the link images
below to further research the concepts in this question (if desired).

Research Concepts:
Digoxin Toxicity:

Tap flag to report any problems with this question.


Question 1667: A 71-year-old female is admitted unit with shortness of
breath and diaphoresis. She has been unable to lie down flat in the bed.
Auscultation reveals basilar rales and the presence of an S3 heart sound. She is
unable to eat and you decide to start her on intravenous therapy. What is the
medication that can only be administered intravenously for her condition?

Choices:
1. Digoxin
2. Captopril
3. Dobutamine
4. Amiodarone
Answer: 3 - Dobutamine
Explanations:
Dobutamine is an inotropic agent with Beta-1 receptor agonist activity.
Dobutamine increases the force of contraction of the heart and it also
vasodilates the pulmonary arteries.
Dobutamine is used to treat heart failure and cardiogenic shock.
Dobutamine has mild Beta-2 agonist activity and can only be used
intravenously.

Go to the next page if you knew the correct answer, or click the link images
below to further research the concepts in this question (if desired).

Research Concepts:
Inotropes And Vasopressors:

Heart Failure, Congestive:

Tap flag to report any problems with this question.


Question 1668: Which of the following is true of a venous air embolism?
Choices:
1. It occurs more commonly when cannulating groin veins
2. It can occur after dissection of the neck
3. Air usually enters the arterial circuit
4. It is best treated by placing the head up
Answer: 2 - It can occur after dissection of the neck
Explanations:
A venous air embolism is often an iatrogenic complication when
atmospheric gas is introduced into the systemic veins.
Recently, venous air emboli have been associated with central lines, high-
pressure mechanical ventilation, thoracentesis, and hemodialysis.
Complications can occur with as little as three mL of air. Rapid entry of air
into the right ventricle causes high pulmonary pressures and cardiovascular
collapse.
Signs of a venous air embolism include arrhythmias, "mill wheel" murmur,
hypotension, pulmonary artery hypertension, and cardiac collapse. A
transesophageal echocardiogram is the best study to use to diagnose a
venous air embolism.

Go to the next page if you knew the correct answer, or click the link images
below to further research the concepts in this question (if desired).

Research Concepts:
Venous Air Embolism:

Tap flag to report any problems with this question.


Question 1669: Which of the risk categories that drugs are assigned in
pregnancy is not correct?

Choices:
1. Category A: Adequate and well-controlled studies have failed to
demonstrate a risk to the fetus in the first trimester of pregnancy (and there
is no evidence of risk in later trimesters).
2. Category B: Animal reproduction studies have failed to demonstrate a risk
to the fetus and there are no adequate and well-controlled studies in
pregnant women.
3. Category C: Animal reproduction studies have shown an adverse effect on
the fetus and there are no adequate and well-controlled studies in humans,
but potential benefits may warrant use of the drug in pregnant women
despite potential risks.
4. Category X: There is positive evidence of human fetal risk based on
adverse reaction data from investigational or marketing experience or
studies in humans, but potential benefits may warrant use of the drug in
pregnant women despite potential risks.
Answer: 4 - Category X: There is positive evidence of human fetal risk
based on adverse reaction data from investigational or marketing experience or
studies in humans, but potential benefits may warrant use of the drug in pregnant
women despite potential risks.

Explanations:
Category X: Studies in animals or humans have demonstrated fetal
abnormalities and/or there is positive evidence of human fetal risk based on
adverse reaction data from investigational or marketing experience, and the
risks involved in use of the drug in pregnant women clearly outweigh
potential benefits.
Category B: Animal reproduction studies have failed to demonstrate a risk
to the fetus and there are no adequate and well-controlled studies in
pregnant women.
Category C: Animal reproduction studies have shown an adverse effect on
the fetus and there are no adequate and well-controlled studies in humans,
but potential benefits may warrant use of the drug in pregnant women
despite potential risks.
Category D: There is positive evidence of human fetal risk based on
adverse reaction data from investigational or marketing experience or
studies in humans, but potential benefits may warrant use of the drug in
pregnant women despite potential risks.

Go to the next page if you knew the correct answer, or click the link images
below to further research the concepts in this question (if desired).

Research Concepts:
Medications in Pregnancy:
Tap flag to report any problems with this question.
Question 1670: The most common treatment for paroxysmal
supraventricular tachycardia is:

Choices:
1. Magnesium
2. Shock
3. Calcium channel blocker
4. Digoxin
Answer: 3 - Calcium channel blocker
Explanations:
The most common treatment for paroxysmal supraventricular tachycardia is
a calcium blocker

Go to the next page if you knew the correct answer, or click the link images
below to further research the concepts in this question (if desired).

Research Concepts:
Paroxysmal Supraventricular Tachycardia:

Tap flag to report any problems with this question.


Question 1671: What is the simplest way to open the airway in an
unconscious patient?

Choices:
1. Pull out the tongue
2. Tilt head and lift chin
3. Lift neck from behind
4. Jaw thrust
Answer: 2 - Tilt head and lift chin
Explanations:
The simplest way to ensure an open airway in an unconscious patient is to
use a head tilt and chin lift technique.
This maneuver lifts the tongue from the back of the throat.

Go to the next page if you knew the correct answer, or click the link images
below to further research the concepts in this question (if desired).

Research Concepts:
Airway, Management:

Tap flag to report any problems with this question.


Question 1672: Which symptoms are not consistent with hyponatremia?
Choices:
1. Muscle cramps, nausea, vomiting
2. Fever, rash, arthralgias
3. Altered mental status, weakness, lethargy
4. Seizures, coma, permanent neurological injury, brain edema, brain stem
herniation
Answer: 2 - Fever, rash, arthralgias
Explanations:
Fever, rash and arthralgias are not usually seen with hyponatremia

Go to the next page if you knew the correct answer, or click the link images
below to further research the concepts in this question (if desired).

Research Concepts:
Hyponatremia:

Electrolyte Abnormalities:

Tap flag to report any problems with this question.


Question 1673: Which structure prevents the cusps from being everted
into the atrium during ventricular contraction?

Choices:
1. Papillary muscles and chordae tendineae
2. Pectinate muscles
3. AV and SA nodes
4. Myometrium
Answer: 1 - Papillary muscles and chordae tendineae
Explanations:
The flaps on the bicuspid valve and tricuspid valve are connected to the
chordae tendineae which are connected to the papillary muscle
They prevent the cusps from being everted into the atrium during
ventricular contraction

Go to the next page if you knew the correct answer, or click the link images
below to further research the concepts in this question (if desired).

Research Concepts:
Heart Anatomy:

Tap flag to report any problems with this question.


Question 1674: Thrombolytic agents are contraindicated in patients with?
Choices:
1. PE
2. Pneumonia
3. COPD
4. Hemothorax
Answer: 4 - Hemothorax
Explanations:
Hemothorax is a contraindication to anticoagulant use

Go to the next page if you knew the correct answer, or click the link images
below to further research the concepts in this question (if desired).

Research Concepts:
Thrombolytic Therapy:

Hemothorax:

Tap flag to report any problems with this question.


Question 1675: Congenital cardiovascular disease is seen in what
percentage of patients with Down syndrome?

Choices:
1. 1%
2. 5%
3. 20%
4. 33%
Answer: 3 - 20%
Explanations:
20% of patients with Down syndrome have congenital cardiovascular
disease
The most common findings are atrial septal defect and ventricular septal
defect

Go to the next page if you knew the correct answer, or click the link images
below to further research the concepts in this question (if desired).

Research Concepts:
Down Syndrome:

Congenital Heart Disease:

Tap flag to report any problems with this question.


Question 1676: pH 7.45, PO2 85 mm Hg, PCO2 45 mm Hg and HCO 34
mEq/L. What is the ABG interpretation?

Choices:
1. Compensated respiratory alkalosis
2. Uncompensated metabolic alkalosis
3. Compensated metabolic alkalosis
4. Uncompensated respiratory alkalosis
Answer: 3 - Compensated metabolic alkalosis
Explanations:
The ABG interpretation is a compensated metabolic alkalosis
pH is normal, indicating a compensated state but is trending toward
alkalosis
Bicarbonate is high, indicating primary event is metabolic
PCO2 is high, indicating a compensatory respiratory acidosis

Go to the next page if you knew the correct answer, or click the link images
below to further research the concepts in this question (if desired).

Research Concepts:
Metabolic Alkalosis:

Tap flag to report any problems with this question.


Question 1677: Which is not a difference between heparin and low
molecular weight heparin?

Choices:
1. LMWH are longer acting than heparin
2. LMWH do not require monitoring like heparin
3. LMWH doses are based on the weight of patient
4. LMWH is given intravenously
Answer: 4 - LMWH is given intravenously
Explanations:
LMWH is widely used in hospitals and has largely replaced unfractionated
heparin.
The agents are given subcutaneously and are just as effective as heparin.
They are never given intravenously.
Unlike heparin, they do not require monitoring.

Go to the next page if you knew the correct answer, or click the link images
below to further research the concepts in this question (if desired).

Research Concepts:
Low Molecular Weight Heparin (LMWH):

Heparin:

Tap flag to report any problems with this question.


Question 1678: What does the activated clotting time measure?
Choices:
1. Extrinsic pathway
2. Intrinsic pathway
3. Extrinsic and intrinsic pathways
4. Platelet aggravation
Answer: 2 - Intrinsic pathway
Explanations:
The normal activated clotting time is about 100 seconds.

Go to the next page if you knew the correct answer, or click the link images
below to further research the concepts in this question (if desired).

Research Concepts:
Activated Clotting Time:

Tap flag to report any problems with this question.


Question 1679: Select the feature that would not be present in a normal
heart.

Choices:
1. Centrally placed nuclei in the muscle fibers
2. Basophilic muscle fibers
3. Striated muscle
4. Intercalated discs
Answer: 2 - Basophilic muscle fibers
Explanations:
Cardiac muscle fibers are eosinophilic
Cardiac muscle has intercalated discs, striated muscle fibers, and centrally
placed nuclei in the fibers

Go to the next page if you knew the correct answer, or click the link images
below to further research the concepts in this question (if desired).

Research Concepts:
Heart Histology:

Tap flag to report any problems with this question.


Question 1680: Which of the following medications is most likely to
cause rhabdomyolysis?

Choices:
1. Heparin
2. Penicillin
3. Atorvastatin
4. Warfarin
Answer: 3 - Atorvastatin
Explanations:
The most serious side effects of statins are myositis and myopathy.
In a few individual statins can induce skeletal muscle breakdown and
rhabdomyolysis which can cause renal failure.
Risk of rhabdomyolysis is increased when statins are combined with fibrate
drugs.
Other rare side effects of statins include intestitial lung disease, cognitive
impairment, erectile dysfunction, and abdominal pain.

Go to the next page if you knew the correct answer, or click the link images
below to further research the concepts in this question (if desired).

Research Concepts:
Statin Medication:

Tap flag to report any problems with this question.


Question 1681: Which of the following is NOT a potential side effect of
prolonged use of cyclosporine?

Choices:
1. Gum hypertrophy
2. Hypertension
3. Hypokalemia
4. Hirsutism
Answer: 3 - Hypokalemia
Explanations:
Cyclosporine is used to prevent rejection of transplanted organs.
Cyclosporine is believed to act by binding to cyclophilin (immunophilin) of
immunocompetent T lymphocytes.
Cyclosporine has many side effects including gum hyperplasia, seizures,
peptic ulcers, pancreatitis, high blood pressure, hyperkalemia,
nephrotoxicity, hepatotoxicity, and an increased susceptibility to
opportunistic fungal and viral infections.

Go to the next page if you knew the correct answer, or click the link images
below to further research the concepts in this question (if desired).

Research Concepts:
Cyclosporine:

Tap flag to report any problems with this question.


Question 1682: Failure of the aorticopulmonary membrane to fuse often
leads to development of:

Choices:
1. Tetralogy of Fallot
2. Transposition of great vessels
3. Truncus arteriosus
4. Tricuspid atresia
Answer: 3 - Truncus arteriosus
Explanations:
The truncus and bulbus cordis are divided by the aorticopulmonary septum
When the aortico-pulmonary septum divides it gives rise to the aorta and
pulmonary artery
Failure of the truncus to divide leads to develop of truncus arteriosus

Go to the next page if you knew the correct answer, or click the link images
below to further research the concepts in this question (if desired).

Research Concepts:
Aorticopulmonary Membrane:

Truncus Arteriosus:

Tap flag to report any problems with this question.


Question 1683: The coronary artery is considered a:
Choices:
1. Elastic artery
2. Muscular artery
3. Large artery
4. Arteriole
Answer: 2 - Muscular artery
Explanations:
The coronary artery is a midsize artery
Also known as the muscular artery

Go to the next page if you knew the correct answer, or click the link images
below to further research the concepts in this question (if desired).

Research Concepts:
Coronary Artery Anatomy:

Tap flag to report any problems with this question.


Question 1684: Which condition produces an Austin Flint murmur?
Choices:
1. Aortic stenosis
2. Mitral stenosis
3. Aortic regurgitation
4. Tricuspid stenosis
Answer: 3 - Aortic regurgitation
Explanations:
Austin Flint is a mid-to-late diastolic rumble best heard at the apex.
Aortic regurgitation causes mitral valve vibration when atrial flow peaks.
It is similar to the murmur heard with mitral stenosis.
Patients may develop an S3 if there is LV dysfunction.

Go to the next page if you knew the correct answer, or click the link images
below to further research the concepts in this question (if desired).

Research Concepts:
Aortic Regurgitation:

Tap flag to report any problems with this question.


Question 1685: Which of the following conditions is not a sign of
rejection of a transplanted heart?

Choices:
1. Ascites
2. Myocardial infarction
3. Renal insufficiency
4. Hypotension
Answer: 3 - Renal insufficiency
Explanations:
Classic signs for heart rejection include dysrhythmia, CHF, and ascites

Go to the next page if you knew the correct answer, or click the link images
below to further research the concepts in this question (if desired).

Research Concepts:
Heart Transplantation:

Tap flag to report any problems with this question.


Question 1686: A patient presents with unstable angina (UA), a non-ST-
segment elevation myocardial infarction (NSTEMI) and an initial elevated
troponin I level. What is the most appropriate next step in his management?

Choices:
1. Echocardiogram after 24 hours
2. Percutaneous intervention (PCI) alone
3. Exercise stress test within 24 hours of admission
4. Early invasive therapy (e.g. routine catheterization within 48 hours and
revascularization as appropriate)
Answer: 4 - Early invasive therapy (e.g. routine catheterization within 48
hours and revascularization as appropriate)

Explanations:
Early invasive therapy would be most appropriate in this high-risk patient
ACC/AHA guidelines recommend ASA and clopidogrel or IV GP IIb/IIIa
inhibitor prior to intervention in medium to high-risk patients

Go to the next page if you knew the correct answer, or click the link images
below to further research the concepts in this question (if desired).

Research Concepts:
Angina, Unstable:

Acute Myocardial Infarction:

Tap flag to report any problems with this question.


Question 1687: A 69-year-old female is admitted with a diagnosis of
anasarca. This means that the patient has which of the following disorders?

Choices:
1. Lymphatic obstruction
2. Right heart failure
3. Systemic edema of the body
4. Bilateral pleural effusions
Answer: 3 - Systemic edema of the body
Explanations:
Anasarca is essentially generalized edema and is characterized by extreme
swelling of the skin.
Anasarca is due to fluid extrusion into the extracellular space.
Anasarca is usually caused by congestive heart failure or liver or renal
disease.

Go to the next page if you knew the correct answer, or click the link images
below to further research the concepts in this question (if desired).

Research Concepts:
Anasarca:

Tap flag to report any problems with this question.


Question 1688: What is an important landmark for the location of second
rib?

Choices:
1. Jugular notch
2. Body of gladiolus
3. Sternal angle
4. Clavicular notch
Answer: 3 - Sternal angle
Explanations:
The sternal angle is a slight anterior projection formed by the junction of
the manubrium and the body.
The costal cartilage of the 2nd rib attaches at this point.
The angle of Louis also marks the site of bifurcation of the trachea into left
and right main stem bronchi.
The angle of Louis also corresponds with the upper body of the right
atrium.

Go to the next page if you knew the correct answer, or click the link images
below to further research the concepts in this question (if desired).

Research Concepts:
Thorax Anatomy:

Tap flag to report any problems with this question.


Question 1689: Which class of drugs can be used to control ventricular
rate in atrial flutter?

Choices:
1. ACEI
2. Digoxin
3. Calcium channel blockers
4. Adenosine
Answer: 3 - Calcium channel blockers
Explanations:
Either calcium channel blockers or beta-blockers can be used for ventricular
control.
Most patients need to be on anticoagulation medication at the same time.

Go to the next page if you knew the correct answer, or click the link images
below to further research the concepts in this question (if desired).

Research Concepts:
Atrial Flutter:

Tap flag to report any problems with this question.


Question 1690: Which of the following is considered a working definition
of polypharmacy?

Choices:
1. The use of more than one pharmacy
2. Six or more prescribed concomitant drugs and/or the use of potentially
inappropriate medication
3. The use of multiple illegally obtained prescriptions
4. All medications prescribed to a patient
Answer: 2 - Six or more prescribed concomitant drugs and/or the use of
potentially inappropriate medication

Explanations:
The use of more than one pharmacy is ill advised, but is a consumer's
choice.
A working definition of polypharmacy is the use of six or more prescribed
concomitant drugs and/or the use of potentially inappropriate medication.
The use of multiple illegally obtained prescriptions is a crime.
All the medications prescribed to a patient is a medication inventory.

Go to the next page if you knew the correct answer, or click the link images
below to further research the concepts in this question (if desired).

Research Concepts:
Polypharmacy:

Managed Care:

Tap flag to report any problems with this question.


Question 1691: With 50 patients admitted for acute MI, how many are
expected to have diagnostic changes on ECG?

Choices:
1. 10
2. 25
3. 30
4. 35
Answer: 2 - 25
Explanations:
Only about half of all patients with acute MI will have diagnostic changes
on ECG
ECG shows ST-segment or T-wave changes

Go to the next page if you knew the correct answer, or click the link images
below to further research the concepts in this question (if desired).

Research Concepts:
Electrocardiogram:

Acute Myocardial Infarction:

Tap flag to report any problems with this question.


Question 1692: Signs and symptoms of developing tricuspid regurgitation
following an endomyocardial biopsy can include:

Choices:
1. Dizziness
2. Tachycardia
3. Jugular venous distension
4. Absence of 3rd heart sound
Answer: 3 - Jugular venous distension
Explanations:
Tricuspid regurgitation can develop following an endomyocardial biopsy to
determine the presence of rejection of the donor heart
Tricuspid regurgitation causes symptoms of right-sided heart failure

Go to the next page if you knew the correct answer, or click the link images
below to further research the concepts in this question (if desired).

Research Concepts:
Endomyocardial Biopsy:

Tricuspid Regurgitation:

Tap flag to report any problems with this question.


Question 1693: Which of the following is NOT a side effect of heparin?
Choices:
1. Thrombosis
2. Hypersensitivity reaction
3. Osteoporosis
4. Bronchoconstriction
Answer: 4 - Bronchoconstriction
Explanations:
A decrease in the number of circulating platelets may occur approximately
7-10 days after heparin therapy.
In some patients, heparin-induced platelet aggregation is followed by
formation of anti-platelet antibodies. At this point, the patients are at great
risk for thrombosis and all heparin must be stopped immediately.
Drugs that inhibit platelet aggregation and oral anticoagulants may be
instituted in place of heparin.
Heparin does not cause bronchoconstriction.

Go to the next page if you knew the correct answer, or click the link images
below to further research the concepts in this question (if desired).

Research Concepts:
Anticoagulation:

Tap flag to report any problems with this question.


Question 1694: What is the daily prophylaxis dose of aspirin for coronary
artery disease?

Choices:
1. 81 mg
2. 162 mg
3. 325 mg
4. 650 mg
Answer: 1 - 81 mg
Explanations:
Aspirin is given for prophylaxis at 81 mg per day
Aspirin is an irreversible COX inhibitor

Go to the next page if you knew the correct answer, or click the link images
below to further research the concepts in this question (if desired).

Research Concepts:
Aspirin:

Coronary Artery Disease:

Tap flag to report any problems with this question.


Question 1695: Which is not commonly seen in aortic stenosis?
Choices:
1. Exertion syncope
2. Angina
3. Aortic dissection
4. Systolic murmur from 2RICS(second right intercostals space) radiating to
neck
Answer: 3 - Aortic dissection
Explanations:
Exertional syncope, angina, CHF, and systolic murmur radiating from
2RICS to the neck are characteristic of aortic stenosis
Aortic dissection is not related to aortic stenosis

Go to the next page if you knew the correct answer, or click the link images
below to further research the concepts in this question (if desired).

Research Concepts:
Aortic Stenosis:

Tap flag to report any problems with this question.


Question 1696: A patient with a history of myocardial infarction three
weeks ago presents with pleuritic chest pain and low grade fever. Physical exam
and ECG are normal. What is the appropriate treatment?

Choices:
1. Benzodiazepine
2. Omeprazole
3. Nitroglycerin
4. NSAID
Answer: 4 - NSAID
Explanations:
This case is likely due to Dressler syndrome, also known as post
myocardial infarction syndrome.
It is a benign autoimmune pleuritis or pericarditis that occurs up to 2-3
weeks after myocardial infarction.
It occurs most commonly following open heart surgery, ST elevation MI, or
pericardial bleeding.
Therapy is rest and NSAIDs or sometimes glucocorticoids.

Go to the next page if you knew the correct answer, or click the link images
below to further research the concepts in this question (if desired).

Research Concepts:
Dressler Syndrome:

Tap flag to report any problems with this question.


Question 1697: Which is not a physicochemical property of
procainamide?

Choices:
1. Renal elimination
2. Renal metabolism
3. Class 1 antiarrhythmic agent
4. Needs a loading dose
Answer: 2 - Renal metabolism
Explanations:
Procainamide is metabolized by the liver and eliminated in the kidneys.

Go to the next page if you knew the correct answer, or click the link images
below to further research the concepts in this question (if desired).

Research Concepts:
Procainamide:

Tap flag to report any problems with this question.


Question 1698: Which of the following factors does not increase risk of
perioperative MI?

Choices:
1. History of PVCs
2. Angina on exertion
3. Angina at rest
4. History of MI 7 months ago
Answer: 4 - History of MI 7 months ago
Explanations:
Generally, a patient who waits about 6 months after an MI to have surgery
has a reduced risk of another MI.
All the rest are risk factors for MI.

Go to the next page if you knew the correct answer, or click the link images
below to further research the concepts in this question (if desired).

Research Concepts:
Perioperative Cardiac Management:

Tap flag to report any problems with this question.


Question 1699: Which vitamin is synthesized by bacteria in the intestines?
Choices:
1. Vitamin A
2. Vitamin D
3. Vitamin E
4. Vitamin K
Answer: 4 - Vitamin K
Explanations:
Vitamin K is synthesized by bacteria in the GI tract so deficiency is
relatively uncommon.
Neonates in the U.S are given an injection of vitamin K after birth since
they have sterile intestines and cannot synthesize vitamin K initially.
Vitamin K is necessary for the production of clotting factors II, VII, IX, X
and protein C and S.
Vitamin K deficiency would manifest as hemorrhage with increased PTT
and aPTT but normal bleeding time.

Go to the next page if you knew the correct answer, or click the link images
below to further research the concepts in this question (if desired).

Research Concepts:
Vitamin K:

Tap flag to report any problems with this question.


Question 1700: Which of the following medications can improve long
term survival in patients with acute myocardial infarction?

Choices:
1. Digoxin
2. Terazosin
3. Furosemide
4. Metoprolol
Answer: 4 - Metoprolol
Explanations:
Beta-blockers are known to improve long term survival in patients with
myocardial infraction
Propranolol and metoprolol are common beta-blockers used
Beta-blockers reduce sympathetic activity and cardiac contractility
Alpha-1-blockers like terazosin may result in unopposed beta stimulation

Go to the next page if you knew the correct answer, or click the link images
below to further research the concepts in this question (if desired).

Research Concepts:
Acute Myocardial Infarction:

Beta-Blockers:

Tap flag to report any problems with this question.


Question 1701: P50 on the oxyhemoglobin dissociation curve means:
Choices:
1. 50 percent saturation of oxygen
2. 50 percent saturation of carbon dioxide
3. PaO2 of 50
4. PaCO2 of 50
Answer: 1 - 50 percent saturation of oxygen
Explanations:
P50 is the point at which the oxygen saturation of hemoglobin is 50%

Go to the next page if you knew the correct answer, or click the link images
below to further research the concepts in this question (if desired).

Research Concepts:
Oxyhemoglobin Dissociation Curve:

Tap flag to report any problems with this question.


Question 1702: Which of the following statements is true regarding deep
vein thrombosis (DVT)?

Choices:
1. It is not the duration of surgery but the type of surgery that increases risk of
DVT
2. Prophylaxis against DVT is now routine in most elective surgeries
3. DVT always presents with a painful and swollen calf
4. Aspirin can prevent most cases of DVT
Answer: 2 - Prophylaxis against DVT is now routine in most elective
surgeries

Explanations:
DVT depends on the duration of surgery. The longer the procedure, the
greater the risk of DVT. While it is true that certain lower abdominal and
pelvic surgeries make a patient more prone to DVT, almost any surgery is a
risk factor for DVT.
The majority of elective surgeries done today are done under DVT
prophylaxis. Prophylaxis includes heparin, compression stockings, or use of
sequential pumps.
DVT can present in subtle ways and can be asymptomatic in many cases.
The only way to make a diagnosis is to have a suspicion and order a duplex
ultrasound.
Aspirin has never been shown to be an effective agent against DVT.

Go to the next page if you knew the correct answer, or click the link images
below to further research the concepts in this question (if desired).

Research Concepts:
DVT:

Tap flag to report any problems with this question.


Question 1703: When resuscitating a patient with hemorrhagic shock
following penetrating trauma, what is the goal?

Choices:
1. Keep blood pressure on the low side
2. Restore blood pressure to normal
3. Restore blood pressure to just 40 mmHg above normal
4. Restore blood pressure so that there is urine output
Answer: 2 - Restore blood pressure to normal
Explanations:
There has been a lot of controversy about how aggressive to be with fluid
resuscitation in patients with hemorrhage shock.
Today the consensus is that the blood pressure should be restored to normal.
The will allow organ perfusion.
Excess elevation of blood pressure with fluids can lead to more bleeding.
Controlled resuscitation strikes a balance between low blood pressure and
organ perfusion. This method is now advocated for most trauma patients.

Go to the next page if you knew the correct answer, or click the link images
below to further research the concepts in this question (if desired).

Research Concepts:
Shock, Hemorrhagic:

Tap flag to report any problems with this question.


Question 1704: Ischemia of the sino-atrial node would occur with
occlusion of which artery?

Choices:
1. Circumflex artery
2. Left coronary artery
3. Posterior interventricular artery
4. Anterior right atrial artery
Answer: 4 - Anterior right atrial artery
Explanations:
The terminal branch of the anterior right atrial artery is the SA nodal artery

Go to the next page if you knew the correct answer, or click the link images
below to further research the concepts in this question (if desired).

Research Concepts:
Anterior Right Atrial Artery:

Tap flag to report any problems with this question.


Question 1705: An anteroseptal infarction expresses an abnormal ECG in
leads:

Choices:
1. I and II
2. II, III, and aVF
3. V1 and V2
4. V5 andV6
Answer: 3 - V1 and V2
Explanations:
Elevations in leads V1 and V2 occur with anteroseptal infarctions
Elevations in leads II, III, and aVF are associated with inferior infarcts

Go to the next page if you knew the correct answer, or click the link images
below to further research the concepts in this question (if desired).

Research Concepts:
Anteroseptal Myocardial Infarction:

Tap flag to report any problems with this question.


Question 1706: A 48 year old female complains of sudden onset of pain of
2 of her toes. She was healthy until 4 months ago when she developed
progressive shortness of breath and a cough occasionally productive of thin pink
sputum. She has 3 pillow orthopnea but no chest pain. Vital signs show mild
hypertension and tachypnea. A grade III/VI mid diastolic murmur is present,
jugular venous distension, crackles in the lung bases, 1+/4 pitting pedal edema,
and dry gangrene of 2 toes on different feet. Pedal pulses are intact.
Echocardiogram shows a large atrial tumor. Select the next step in management.

Choices:
1. Intravenous antibiotics
2. Surgical referral
3. Cardiac catheterization
4. Blood cultures before antibiotics
Answer: 2 - Surgical referral
Explanations:
The patient presents with congestive heart failure secondary to an atrial
myxoma
There is evidence of peripheral embolization
Myxomas are most common in women age 20 to 50
Surgery is indicated with recurrence occurring in only 1 or 2%

Go to the next page if you knew the correct answer, or click the link images
below to further research the concepts in this question (if desired).

Research Concepts:
Atrial Myxoma:

Tap flag to report any problems with this question.


Question 1707: Which drug can reverese the actions of heparin?
Choices:
1. Vitamin K
2. Aminocaproic acid
3. Fresh frozen plasma
4. Protamine
Answer: 4 - Protamine
Explanations:
Protamine can reverse the effects of heparin. Protamine is a cationic peptide
that binds to heparin.
For every 100 units of heparin, 1 mg of protamine is administered IV.
Side effects of protamine include hypotension, bronchoconstriction, and
pulmonary hypertension.
Protamine causes side effects by releasing histamine.

Go to the next page if you knew the correct answer, or click the link images
below to further research the concepts in this question (if desired).

Research Concepts:
Protamine:

Tap flag to report any problems with this question.


Question 1708: Which of the following agents may have some usefulness
in the setting of cardiogenic shock?

Choices:
1. Norepinephrine
2. Dobutamine
3. Dopamine
4. All of the above
Answer: 4 - All of the above
Explanations:
A variety of pressor drugs are used to treat cardiogenic shock.
The classic drugs include dopamine, norepinephrine, and epinephrine.
The mean blood pressure required for adequate tissue perfusion is
60mmHg.
In patients with inadequate tissue perfusion, initiation of inotropic and
vasopressor drug therapy is necessary.

Go to the next page if you knew the correct answer, or click the link images
below to further research the concepts in this question (if desired).

Research Concepts:
Cardiogenic Shock:

Tap flag to report any problems with this question.


Question 1709: Which monoclonal antibody with activity against the
platelet G2b/3a receptor is used in angioplasty?

Choices:
1. Ticlopidine
2. Clopidogrel
3. Abciximab
4. Dipyridamole
Answer: 3 - Abciximab
Explanations:
Abciximab is a monoclonal antibody against the glycoprotein 2b/3a platelet
receptor.
It is used during angioplasty because of its very short half life and prevents
thrombus formation.
Major side effect is hemorrhage.
Thrombocytopenia can also occur several days later and transfusions of
platelets is the only effective treatment for hemorrhage.

Go to the next page if you knew the correct answer, or click the link images
below to further research the concepts in this question (if desired).

Research Concepts:
Anticoagulation:

Tap flag to report any problems with this question.


Question 1710: In a patient who has an acute myocardial infarction, when
should the first dose of aspirin be administered, assuming there are no
contraindications?

Choices:
1. Within the first 24 hrs
2. Within the first hour
3. Only after a myocardial infarction has been confirmed
4. Anytime within the first 12 hours
Answer: 2 - Within the first hour
Explanations:
Once an acute myocardial infarction has been established, emergency
angioplasty or anticoagulation are vital.
All patients should receive a baby aspirin as soon as possible.
Thrombolytic therapy should generally be started within the first 30
minutes.
The goal of angioplasty is to get the patient in the catheterization lab within
the first 60 minutes.

Go to the next page if you knew the correct answer, or click the link images
below to further research the concepts in this question (if desired).

Research Concepts:
Myocardial Infarction, Acute:

Tap flag to report any problems with this question.


Question 1711: What is a patient controlled analgesia device often used to
deliver?

Choices:
1. Sedatives
2. Pain medication
3. Anti-hypertensives
4. Antibiotics
Answer: 2 - Pain medication
Explanations:
PCA stands for patient controlled analgesic machine.
Patient has control of the administration of the drug.

Go to the next page if you knew the correct answer, or click the link images
below to further research the concepts in this question (if desired).

Research Concepts:
Patient Controlled Anesthesia (PCA):

Tap flag to report any problems with this question.


Question 1712: Select the true statement about cardiac transplantation.
Choices:
1. One-year survival is 75% and 60% survive >3 years
2. Mortality after 1 year is most often caused by infection
3. The patients given the highest priority have a pulmonary artery catheter and
need inotropic support
4. Endomyocardial biopsies are done for 2 years to detect rejection
Answer: 3 - The patients given the highest priority have a pulmonary artery
catheter and need inotropic support

Explanations:
Endomyocardial biopsies are usually done for 5 years to detect rejection
83% of patients survive for 1 year and 76% for more than 3 years
Mortality after 1 year is most often due to coronary artery disease
Within the first year, acute rejection and infection are the major causes of
mortality

Go to the next page if you knew the correct answer, or click the link images
below to further research the concepts in this question (if desired).

Research Concepts:
Heart Transplantation:

Tap flag to report any problems with this question.


Question 1713: What is not true regarding flecainide?
Choices:
1. Class II drug
2. Blocks sodium channel
3. Associated with prolong QT
4. Generic drug
Answer: 1 - Class II drug
Explanations:
Flecainide is a class I antiarrhythmic drug
Blocks sodium channel

Go to the next page if you knew the correct answer, or click the link images
below to further research the concepts in this question (if desired).

Research Concepts:
Flecainide:

Tap flag to report any problems with this question.


Question 1714: Where is the most common benign heart tumor is usually
located?

Choices:
1. Right atrium
2. Right ventricle
3. Left ventricle
4. Left atrium
Answer: 1 - Right atrium
Explanations:
The majority of benign tumors are located in the left atrium. Myxomas are
very rare in other chambers of the heart. In Carney syndrome, myxomas
may occur in other chambers of the heart.
The majority of left atrial myxomas are found near the fossa ovalis in the
atrium.
Cardiac myxomas that occur as part of the Carney syndrome are less
common in the left atrium and are often multicentric.
Cardiac myxomas can present with any number of symptoms but the most
common is a triad of embolic phenomenon, mitral stenosis like symptoms
and constitutional features. More than 50% will present with left sided heart
failure when the tumors are large.

Go to the next page if you knew the correct answer, or click the link images
below to further research the concepts in this question (if desired).

Research Concepts:
Atrial Myxoma:

Tap flag to report any problems with this question.


Question 1715: Which of the following is not a useful treatment for
hyperkalemia?

Choices:
1. Calcium chloride
2. Insulin
3. Magnesium
4. Kayexalate
Answer: 3 - Magnesium
Explanations:
The treatments of hyperkalemia involves insulin, sodium polystyrene
sulfonate, and calcium.
Beta agonists can be given.
Magnesium has no role in the treatment of hyperkalemia.

Go to the next page if you knew the correct answer, or click the link images
below to further research the concepts in this question (if desired).

Research Concepts:
Hyperkalemia:

Tap flag to report any problems with this question.


Question 1716: Which of the following is not an indication for surgery for
mitral valve pathology?

Choices:
1. Systemic embolism
2. Pulmonary hypertension
3. Tricuspid regurgitation (TR)
4. Hemoptysis
Answer: 3 - Tricuspid regurgitation (TR)
Explanations:
In about 5-10% of patients undergoing mitral valve surgery, tricuspid
annuloplasty is performed but less than 2% of patients require tricuspid
valve replacement
Patients with functional TR secondary to mitral disease who do not undergo
repair of the tricuspid valve at the time of initial surgery have a variable
course
When the TR is not repaired at the initial surgery, only 10-20% have severe
TR post operatively

Go to the next page if you knew the correct answer, or click the link images
below to further research the concepts in this question (if desired).

Research Concepts:
Mitral Valve Replacement:

Tap flag to report any problems with this question.


Question 1717: What is the first stage patients go through when diagnosed
with a terminal illness according to the Kubler-Ross model?

Choices:
1. Bargaining
2. Denial
3. Acceptance
4. Depression
Answer: 2 - Denial
Explanations:
The stages of grief: denial, anger, bargaining, depression and acceptance
The stages of grief may not necessarily occur in above sequence
There are many who are critical of this idea
It is important to understand how to deal with each stage of grief

Go to the next page if you knew the correct answer, or click the link images
below to further research the concepts in this question (if desired).

Research Concepts:
Grief Reaction:

Tap flag to report any problems with this question.


Question 1718: Which of the following is not a side effect of
cyclosporine?

Choices:
1. Gum hypertrophy
2. Hypertension
3. Hypokalemia
4. Hirsutism
Answer: 3 - Hypokalemia
Explanations:
Cyclosporine has many adverse side effects.
Most common include gingival hyperplasia, convulsions, peptic ulcers,
fever, vomiting, diarrhea, and confusion.
Cyclosporine can cause a rise in blood pressure and even leads to potassium
retention.
Burning sensation at fingertips is also widely reported in patients who use
cyclosporine.

Go to the next page if you knew the correct answer, or click the link images
below to further research the concepts in this question (if desired).

Research Concepts:
Cyclosporine:

Tap flag to report any problems with this question.


Question 1719: Which of the following statements is false with respect to
subacute infectious endocarditis?

Choices:
1. Associated with focal neurological deficits
2. Patients present with non specific symptoms
3. Fever is a common sign
4. Murmurs are often absent
Answer: 4 - Murmurs are often absent
Explanations:
While most patients present with nonspecific symptoms, murmurs are
commonly associated with subacute infectious endocarditis

Go to the next page if you knew the correct answer, or click the link images
below to further research the concepts in this question (if desired).

Research Concepts:
Endocarditis, Bacterial:

Tap flag to report any problems with this question.


Question 1720: The best indicator of an MI post CABG is:
Choices:
1. Elevated CKMB levels
2. ECG changes
3. New regional wall abnormalities on transesophageal echocardiogram
4. Decreased BP and urine output
Answer: 3 - New regional wall abnormalities on transesophageal
echocardiogram

Explanations:
Elevated levels of CKMB or troponin are not specific enough, and nor is
the ECG
Some believe that new regional wall abnormalities on TEE may be the best
indicator of a post CABG MI

Go to the next page if you knew the correct answer, or click the link images
below to further research the concepts in this question (if desired).

Research Concepts:
Acute Myocardial Infarction:

Tap flag to report any problems with this question.


Question 1721: Where are the lung hila are anatomically located?
Choices:
1. Lower bases
2. Pleural spaces
3. Mediastinal surface
4. Apical lung
Answer: 3 - Mediastinal surface
Explanations:
The mediastinal surface the lung includes the hilar area.

Go to the next page if you knew the correct answer, or click the link images
below to further research the concepts in this question (if desired).

Research Concepts:
Lung Anatomy:

Tap flag to report any problems with this question.


Question 1722: What is the third leading cause of death in the USA?
Choices:
1. Cancer
2. Trauma
3. Stroke
4. Myocardial infarction
Answer: 3 - Stroke
Explanations:
Stroke is the 3rd leading cause of death in the USA
Strokes kill more than 4 million people each year
Strokes are the leading cause of disability in USA

Go to the next page if you knew the correct answer, or click the link images
below to further research the concepts in this question (if desired).

Research Concepts:
Stroke:

Tap flag to report any problems with this question.


Question 1723: Which of the following pain medications is least likely to
cause dysphoria?

Choices:
1. Morphine
2. Meperidine
3. Fentanyl
4. Acetaminophen
Answer: 4 - Acetaminophen
Explanations:
All narcotics can cause dysphoria, irrespective of the dose.
Acetaminophen is a non-narcotic and does not produce dysphoria, even in
large doses.

Go to the next page if you knew the correct answer, or click the link images
below to further research the concepts in this question (if desired).

Research Concepts:
Non-Narcotic Pain Medicines:

Opioids:

Tap flag to report any problems with this question.


Question 1724: Which patient with myocardial infarction (MI) is a
candidate for reperfusion therapy?

Choices:
1. Non-Q wave MI (NQWMI)
2. ST-segment elevation MI (STEMI) seen within 12 hours of onset of
symptoms
3. Non-STEMI (NSTEMI)
4. None of the above
Answer: 2 - ST-segment elevation MI (STEMI) seen within 12 hours of
onset of symptoms

Explanations:
Patients with STEMI presenting within in 12 hours of onset of symptoms
are candidates for reperfusion therapy
Reperfusion therapy reduces infarct size
It also limits ventricular dysfunction and reduces serious complications
Reperfusion therapy is not appropriate for those with NQWMI and
NSTEMI

Go to the next page if you knew the correct answer, or click the link images
below to further research the concepts in this question (if desired).

Research Concepts:
Acute Myocardial Infarction:

Acute ST-elevation Myocardial Infarction (STEMI):

Tap flag to report any problems with this question.


Question 1725: Which of the following agents would be the most
efficacious in reducing symptoms of peptic ulcer disease?

Choices:
1. Bisacodyl
2. Cimetidine
3. Magnesium hydroxide
4. Omeprazole
Answer: 4 - Omeprazole
Explanations:
Peptic ulcer disease results in gastric mucosal erosion caused by acid injury
Acid production can be maximally reduced by proton pump inhibitors
Omeprazole is one of the commonly used PPIs

Go to the next page if you knew the correct answer, or click the link images
below to further research the concepts in this question (if desired).

Research Concepts:
Peptic Ulcer Disease:

Proton Pump Inhibitors (PPI):

Tap flag to report any problems with this question.


Question 1726: A buildup of lactic acid and carbon dioxide are implicated
in:

Choices:
1. Cardiac arrhythmias
2. Myocardial infarction (MI)
3. Chest pain associated with stable angina
4. None of the above
Answer: 3 - Chest pain associated with stable angina
Explanations:
A buildup of lactic acid and carbon dioxide is implicated in chest pain
associated with stable angina
Stable angina is a temporary low flow state in the coronary arteries and
does not lead to infarction (i.e. cell injury or death)
During the low flow state, lactic acid and carbon dioxide buildup causing
pain

Go to the next page if you knew the correct answer, or click the link images
below to further research the concepts in this question (if desired).

Research Concepts:
Angina, Stable:

Tap flag to report any problems with this question.


Question 1727: Which is false about rheumatic mitral stenosis?
Choices:
1. Normal area of the mitral valve is 4-6 cm2
2. Left atrial pressure in severe mitral stenosis is usually about 5 mmHg
3. Pure mitral stenosis occurs in 40 percent of individual with rheumatic heart
disease
4. Symptoms are most evident in the 3rd or 4th decade of life
Answer: 2 - Left atrial pressure in severe mitral stenosis is usually about 5
mmHg

Explanations:
Normal valve area of the mitral valve is about 4-6 cm2.
The normal left atrial pressure is 5 mm Hg and in severe cases of mitral
stenosis, it increase to 20 mmHg. This leads to increased pressure in the
pulmonary vasculature.
The majority of cases of mitral stenosis are due to rheumatic fever.
Most people who have rheumatic fever develop mitral stenosis after 1-2
decades. Once symptoms start they usually progress.

Go to the next page if you knew the correct answer, or click the link images
below to further research the concepts in this question (if desired).

Research Concepts:
Mitral Stenosis:

Tap flag to report any problems with this question.


Question 1728: Which of the following patient groups benefit most from
receiving ACE inhibitors?

Choices:
1. Ischemic cardiomyopathy
2. Mild to moderate heart failure (HF)
3. Nonischemic cardiomyopathy
4. All of the above
Answer: 4 - All of the above
Explanations:
Numerous clinical trials have concluded reduced mortality in patients with
mild to moderate HF, ischemic and nonischemic cardiomyopathy receiving
ACE inhibitors
The above is considered a class effect of ACE inhibitors
ACE inhibitors can cause potassium retention
Caution is necessary when initiating potassium in ACE inhibitor patients

Go to the next page if you knew the correct answer, or click the link images
below to further research the concepts in this question (if desired).

Research Concepts:
Angiotensin Converting Enzyme Inhibitors (ACEI):

Cardiomyopathy, Pathology:

Heart Failure, Congestive:


Tap flag to report any problems with this question.
Question 1729: Which of the following is predominantly an iatrogenic
complication?

Choices:
1. Venous air embolism
2. Deep venous thrombosis (DVT)
3. Venous thromboembolism
4. None of the above
Answer: 1 - Venous air embolism
Explanations:
Venous air embolism is predominantly an iatrogenic complication
It is mostly associated with neurosurgical procedures performed in the
sitting position
Other causes of venous air embolism include central venous catheterization,
blunt chest trauma, mechanical ventilation, thoracocentesis and
hemodialysis

Go to the next page if you knew the correct answer, or click the link images
below to further research the concepts in this question (if desired).

Research Concepts:
Venous Air Embolism:

Tap flag to report any problems with this question.


Question 1730: Which is the first step in tissue repair?
Choices:
1. Granulation tissue formation
2. Scar tissue formation
3. Inflammatory response
4. Fibrous connective tissue proliferation
Answer: 3 - Inflammatory response
Explanations:
The first step of tissue repair is the inflammatory response.
The next step is the proliferative phase where active fibroblasts form
immature granulation tissue.
Maturation involves aptosis of unnecessary blood vessels and organization
of collagen.

Go to the next page if you knew the correct answer, or click the link images
below to further research the concepts in this question (if desired).

Research Concepts:
Acute Inflammatory Response:

Wound Healing Phases:

Tap flag to report any problems with this question.


Question 1731: A patient on digoxin suddenly started to develop
premature ventricular contractions after he was started on a diuretic. What is the
most likely reason that this occurred?

Choices:
1. Increased potassium
2. Decreased potassium
3. Increased calcium
4. Dehydration
Answer: 2 - Decreased potassium
Explanations:
Digoxin has a low therapeutic index and arrhythmias are common when
potassium levels decrease.
Common symptoms include nausea, diarrhea, and vomiting. At high levels
digoxin may cause blurred vision, yellow halos, confusion, nightmares, and
depression.
When digoxin toxicity occurs, the typical feature on ECG is PR interval
prolongation.
Digoxin toxicity is treated with anti digoxin antibodies, potassium and
calcium. Digoxin levels in the blood are not affected by hemodialysis.

Go to the next page if you knew the correct answer, or click the link images
below to further research the concepts in this question (if desired).

Research Concepts:
Digoxin Toxicity:

Tap flag to report any problems with this question.


Question 1732: What is the term for the volume of air inspired and
expired with each normal breath?

Choices:
1. Vital capacity
2. Tidal capacity
3. Residual volume
4. Functional residual capacity
Answer: 2 - Tidal capacity
Explanations:
The normal tidal volume in an adult is about 500 cc

Go to the next page if you knew the correct answer, or click the link images
below to further research the concepts in this question (if desired).

Research Concepts:
Tidal Volume:

Tap flag to report any problems with this question.


Question 1733: Select the structure that is subsequently septated to form
the pulmonary artery and aorta.

Choices:
1. Bulboventricular flange
2. Right sinal horn
3. Bulbus cordis
4. Aortic sac
Answer: 3 - Bulbus cordis
Explanations:
The truncus arteriosus and the conus arteriosus are the 2 parts of the bulbus
cordis
Ridges appear as bulges in the truncus arteriosus on the left inferior walls
and right superior wall and fuse to be the septum
The septum spirals as it separates the distal pulmonary artery from the aorta

Go to the next page if you knew the correct answer, or click the link images
below to further research the concepts in this question (if desired).

Research Concepts:
Bulbus Cordis:

Aorta:

Pulmonary Artery:
Tap flag to report any problems with this question.
Question 1734: Which antihypertensive agent acts centrally?
Choices:
1. Metoprolol
2. Captopril
3. Alpha methyldopa
4. Prazosin
Answer: 3 - Alpha methyldopa
Explanations:
Alpha methyldopa is converted to methyl norepinephrine centrally to
decrease the adrenergic outflow from the CNS, leading to a decreased total
peripheral resistance, and decreased systemic blood pressure.
With alpha methyldopa, cardiac output is not affected and blood supply to
the kidney remains the same.
Because alpha methyldopa does not affect blood supply to the kidney, it is
useful in hypertensive patients with renal insufficiency.

Go to the next page if you knew the correct answer, or click the link images
below to further research the concepts in this question (if desired).

Research Concepts:
Antihypertensive Medications:

Tap flag to report any problems with this question.


Question 1735: What is a normal CO2 level in an ET detector?
Choices:
1. 1 percent
2. 5 percent
3. 9 percent
4. 14 percent
Answer: 2 - 5 percent
Explanations:
A normal CO2 detector level is five percent at the end of an ET tube

Go to the next page if you knew the correct answer, or click the link images
below to further research the concepts in this question (if desired).

Research Concepts:
Carbon Dioxide Detector:

Tap flag to report any problems with this question.


Question 1736: Which of the following statements regarding the lipid
bilayer (cell membrane) is not true?

Choices:
1. It primarily contains phospholipids
2. It contains cholesterol
3. Proteins are the most abundant based on weight
4. It contains no channels
Answer: 4 - It contains no channels
Explanations:
Phospholipids are the most abundant based on the number of molecules.
Proteins make up the channels in the lipid bilayer.

Go to the next page if you knew the correct answer, or click the link images
below to further research the concepts in this question (if desired).

Research Concepts:
Cell Membrane:

Tap flag to report any problems with this question.


Question 1737: A patient is diagnosed with a calcific valvular abnormality
at age 60. As a child, he had a severe infectious illness with arthritis. Which of
the following was the most likely cause?

Choices:
1. Staphylococcus aureus
2. Lyme disease
3. Group A Streptococcus
4. Streptococcus viridans
Answer: 3 - Group A Streptococcus
Explanations:
The patient most likely had rheumatic fever
Lyme disease can cause cardiomyopathy and cardiac conduction
abnormalities but not valvular disease
Strep viridans and Staph aureus can cause infectious endocarditis

Go to the next page if you knew the correct answer, or click the link images
below to further research the concepts in this question (if desired).

Research Concepts:
Rheumatic Fever, Pathology:

Tap flag to report any problems with this question.


Question 1738: Overdose with which medication can cause bradycardia
and hypotension?

Choices:
1. Flumazenil
2. Naloxone
3. Naltrexone
4. Morphine sulfate
Answer: 4 - Morphine sulfate
Explanations:
Morphine overdose causes respiratory depression, bradycardia, and
hypotension.
Naloxone is the antidote for morphine overdose.
Naltrexone is useful in cases of substance abuse.
Flumazenil is used to treat overdose with benzodiazepines.

Go to the next page if you knew the correct answer, or click the link images
below to further research the concepts in this question (if desired).

Research Concepts:
Opioid Toxicity:

Tap flag to report any problems with this question.


Question 1739: The direct cardiac effects of dobutamine would be blocked
by which one of the following agents?

Choices:
1. Prazosin
2. Metoprolol
3. Clonidine
4. Isoproterenol
Answer: 2 - Metoprolol
Explanations:
Dobutamine directly stimulates beta 1 receptors and improves cardiac
output
Metoprolol is a selective beta 1 receptors blocker and can antagonize the
effects of dobutamine
Prazosin is an Alpha one blocker
Clonidine is an alpha two agonist

Go to the next page if you knew the correct answer, or click the link images
below to further research the concepts in this question (if desired).

Research Concepts:
Dobutamine:

Selective Beta-1-Blockers:

Tap flag to report any problems with this question.


Question 1740: Which of the following is not a component of Shone
syndrome?

Choices:
1. Parachute mitral valve
2. Aortic stenosis
3. Coarctation of aorta
4. Supravalvular mitral membrane
Answer: 2 - Aortic stenosis
Explanations:
Shone syndrome is a congenital heart complex disorder consisting of 4
cardiac defects.
There is a supravalvular mitral membrane, parachute mitral valve, subaortic
stenosis, and coarctation of the aorta.
The lesion is characterized by obstruction of both the inflow and the
outflow of the left ventricle.

Go to the next page if you knew the correct answer, or click the link images
below to further research the concepts in this question (if desired).

Research Concepts:
Shone Syndrome:

Tap flag to report any problems with this question.


Question 1741: At what factor do the extrinsic and intrinsic clotting
pathways meet?

Choices:
1. Factor 2
2. Factor 5
3. Factor 7
4. Factor 10
Answer: 4 - Factor 10
Explanations:
Both the intrinsic and extrinsic clotting pathways proceed through the
common pathway, forming activated factor X.
Prothrombin is cleaved by factor X to yield thrombin.
Thrombin then cleaves fibrinogen, which then forms fibrin monomers.
Thrombin also activates factors 8 and 5 and factor 13, which then forms
covalent bonds that cross-link fibrin polymers.

Go to the next page if you knew the correct answer, or click the link images
below to further research the concepts in this question (if desired).

Research Concepts:
Coagulation Pathways:

Tap flag to report any problems with this question.


Question 1742: The major medical use of nitric oxide has been to decrease
which of the following?

Choices:
1. Inflammation
2. Peptic ulcer disease
3. Pulmonary artery pressure
4. Incidence of asthma
Answer: 3 - Pulmonary artery pressure
Explanations:
Nitric oxide has found use in cardiovascular surgery. It has been used
successfully when there is pulmonary hypertension.

Go to the next page if you knew the correct answer, or click the link images
below to further research the concepts in this question (if desired).

Research Concepts:
Nitric Oxide:

Tap flag to report any problems with this question.


Question 1743: In a patient with stable angina, which of the following
should be of concern?

Choices:
1. Pain when patient goes outside on a cold day
2. Pain after a big meal
3. Change in pattern of pain
4. Pain after raking the leaves
Answer: 3 - Change in pattern of pain
Explanations:
The patient should report a change in the pattern of chest pain.
It may indicate increasing severity of coronary artery disease.
Pain is expected with exertion or situations that decrease blood flow to the
heart (big meal) or increase peripheral resistance (cold weather).

Go to the next page if you knew the correct answer, or click the link images
below to further research the concepts in this question (if desired).

Research Concepts:
Angina, Stable:

Tap flag to report any problems with this question.


Question 1744: The artery which arises as a branch of right coronary
artery opposite the origin of the posterior interventricular branch supplies which
of the following structures?

Choices:
1. SA node
2. Bundle of His
3. Pectinate muscle
4. AV node
Answer: 4 - AV node
Explanations:
The atrioventricular node is supplied by the posterior interventricular artery.
This is a branch of right coronary artery in those who have right dominance.
The rest of patients have supply from the left circumflex via the posterior
interventricular artery.

Go to the next page if you knew the correct answer, or click the link images
below to further research the concepts in this question (if desired).

Research Concepts:
Heart Anatomy:

Tap flag to report any problems with this question.


Question 1745: Which of the following correctly describes the action of
clonidine?

Choices:
1. Prevents uptake of catecholamines
2. Prevents release of catecholamines from the nerve endings
3. Acts as an agonist on presynaptic receptors
4. Increases breakdown of catecholamines
Answer: 3 - Acts as an agonist on presynaptic receptors
Explanations:
Clonidine works in the brain as an alpha agonist.
By stimulating the presynaptic alpha-receptors, there is a decrease in
sympathetic stimulation and consequently a lowering of blood pressure.

Go to the next page if you knew the correct answer, or click the link images
below to further research the concepts in this question (if desired).

Research Concepts:
Clonidine:

Tap flag to report any problems with this question.


Question 1746: Which of the following types of chest pain is most
indicative of ischemic heart disease?

Choices:
1. Dull, boring pain
2. Pain lasting for more than 4 hours
3. Pain occurring at rest
4. Chest pain radiating to the jaw or left arm
Answer: 4 - Chest pain radiating to the jaw or left arm
Explanations:
In at least 10-19 percent of patients with jaw or left arm pain, there is a
strong correlation with ischemic chest pain.
In general, any type of chest pain must be investigated.

Go to the next page if you knew the correct answer, or click the link images
below to further research the concepts in this question (if desired).

Research Concepts:
Coronary Artery Disease:

Chest Pain:

Tap flag to report any problems with this question.


Question 1747: All of the following are appropriate uses of vancomycin
except:

Choices:
1. Treatment of serious infections secondary to Gram-positive bacteria that are
beta-lactam-resistant
2. Treatment of infections secondary to Gram-positive bacteria in those with
anaphylaxis to beta-lactam antibiotics
3. Empiric antimicrobial therapy for a febrile neutropenic patient
4. Antibiotic prophylaxis for implantation of devices or prosthetic materials at
hospitals with high incidence of MRSA infections
Answer: 3 - Empiric antimicrobial therapy for a febrile neutropenic patient
Explanations:
The Centers for Disease Control and Prevention have established guidelines
for situations in which the use of vancomycin is appropriate as well as
situations in which the use of vancomycin is discouraged
"Empiric antimicrobial therapy for a febrile neutropenic patient, unless
initial evidence indicates that the patient has an infection caused by gram-
positive microorganisms (e.g., at an inflamed exit site of Hickman catheter)
and the prevalence of infections caused by MRSA in the hospital is
substantial"

Go to the next page if you knew the correct answer, or click the link images
below to further research the concepts in this question (if desired).

Research Concepts:
Vancomycin:

Tap flag to report any problems with this question.


Question 1748: What is the treatment for left ventricular dysfunction with
mitral valve regurgitation?

Choices:
1. Calcium channel blockers and diuretics
2. Antiarrhythmics and diuretics
3. Beta blockers and diuretics
4. ACE inhibitors and diuretics
Answer: 4 - ACE inhibitors and diuretics
Explanations:
ACE inhibitors and diuretics are the primary treatment for left ventricular
dysfunction with mitral valve regurgitation.
An ARB can be substituted if the ACEI is not tolerated.
Nitrates are occasionally used.

Go to the next page if you knew the correct answer, or click the link images
below to further research the concepts in this question (if desired).

Research Concepts:
Mitral Regurgitation:

Left Ventricular Dysfunction:

Tap flag to report any problems with this question.


Question 1749: What is the best procedure for a patient with Eisenmenger
syndrome?

Choices:
1. Single lung transplant
2. Bilateral lung transplant
3. Heart and lung transplant
4. Heart transplant
Answer: 3 - Heart and lung transplant
Explanations:
The heart and lung transplant is the procedure of choice if the underlying
cardiac defect cannot be fixed.
Bilateral lung transplant is considered if the cardiac defect is very simple.
Repair of the cardiac defect alone is not recommended.

Go to the next page if you knew the correct answer, or click the link images
below to further research the concepts in this question (if desired).

Research Concepts:
Eisenmenger Syndrome:

Tap flag to report any problems with this question.


Question 1750: Central venous pressure is reflective of:
Choices:
1. Heart failure
2. Function of the SA node
3. Pressure of blood returning to the heart
4. Hyperthermia
Answer: 3 - Pressure of blood returning to the heart
Explanations:
Normal CVP: 5-12 mm Hg
The pressure measurement reflects how ventricles pump blood out of the
heart and pressure of the blood returning

Go to the next page if you knew the correct answer, or click the link images
below to further research the concepts in this question (if desired).

Research Concepts:
Central Venous Pressure:

Tap flag to report any problems with this question.


Question 1751: Which is the appropriate initial way to control bleeding in
most body areas?

Choices:
1. Saline irrigation
2. Apply direct pressure
3. Apply indirect pressure
4. Use a tourniquet
Answer: 2 - Apply direct pressure
Explanations:
Direct pressure on a wound is the most appropriate way to control bleeding
in most body areas.

Go to the next page if you knew the correct answer, or click the link images
below to further research the concepts in this question (if desired).

Research Concepts:
Hemorrhage Control:

Tap flag to report any problems with this question.


Question 1752: Risk factors for contrast-induced nephropathy do NOT
include which of the following?

Choices:
1. Age
2. IDDM
3. Repeat contrast doses
4. NSAID use
Answer: 4 - NSAID use
Explanations:
Risk factors for CIN are subcateogrized as patient related, procedure
related, and contrast related.
Patient related factors include patient age, the presence of chronic renal
disease, IDDM, or HTN, metabolic syndrome, multiple myeloma, anemia,
renal transplant, or hypoalbuminemia. It also includes cases of hypovolemia
or decreased effective circulating volumes as evidence by CHF, EF < 40%,
hypotension, or need for IABP.
Procedure related risk factors include urgent rather than elective imaging,
arterial rather than venous contrast administration, and diagnostic rather
than therapeutic contrast administration.
Contrast-related risk factors are related to contrast volume, contrast
osmolarity, ionicity, and viscosity.

Go to the next page if you knew the correct answer, or click the link images
below to further research the concepts in this question (if desired).

Research Concepts:
Contrast-Induced Nephropathy:

Tap flag to report any problems with this question.


Question 1753: After how many year of smoking cessation is the risk of
cardiovascular disease equal to the normal population?

Choices:
1. 5
2. 10
3. 15
4. 20
Answer: 3 - 15
Explanations:
Studies have shown that smoking associated risks are the same as normal
population after 15 year of cessation

Go to the next page if you knew the correct answer, or click the link images
below to further research the concepts in this question (if desired).

Research Concepts:
Smoking Cessation:

Cardiovascular Disease:

Tap flag to report any problems with this question.


Question 1754: Electrocautery does not usually have adverse cardiac
effects because it is used at:

Choices:
1. High frequency
2. Low frequency
3. High voltage
4. Low voltage
Answer: 1 - High frequency
Explanations:
The frequency of electrocautery is above 100 hertz.

Go to the next page if you knew the correct answer, or click the link images
below to further research the concepts in this question (if desired).

Research Concepts:
Electrocautery:

Tap flag to report any problems with this question.


Question 1755: A 71-year-old was brought to the ER with severe
hypotension (BP 60/20). ECG revealed the absence of P-waves with a heart rate
of 180 bpm. Which of the following is the best initial management of this
patient?

Choices:
1. Stat cardioversion
2. ECHO
3. Intravenous Lidocaine
4. IV amiodarone
Answer: 1 - Stat cardioversion
Explanations:
The ECG findings suggest atrial fibrillation with rapid ventricular rate.
Immediate electrical defibrillation is indicated if the patient is
hemodynamically unstable.

Go to the next page if you knew the correct answer, or click the link images
below to further research the concepts in this question (if desired).

Research Concepts:
Hypotension:

Atrial Fibrillation:

Tap flag to report any problems with this question.


Question 1756: The proximal aorta is derived from which structure?
Choices:
1. First aortic arch
2. Truncus arteriosus
3. Second brachial pouch
4. Sinus venosus
Answer: 2 - Truncus arteriosus
Explanations:
The truncus arteriosus is a primative heart tube that gives rise to the
proximal aorta.
The truncus arteriosus also gives rise to the proximal pulmonary artery.
The sinus venosus is the smooth part of the right atrium.
The first aortic arch gives rise to the external carotid arteries.

Go to the next page if you knew the correct answer, or click the link images
below to further research the concepts in this question (if desired).

Research Concepts:
Great Vessel Embryology:

Tap flag to report any problems with this question.


Question 1757: Which of the following is NOT a symptom of acute
cardiogenic pulmonary edema?

Choices:
1. Rales
2. Desaturation
3. Respiratory alkalosis
4. Angina
Answer: 3 - Respiratory alkalosis
Explanations:
Acute pulmonary edema can present with respiratory acidosis.

Go to the next page if you knew the correct answer, or click the link images
below to further research the concepts in this question (if desired).

Research Concepts:
Pulmonary Edema, Cardiogenic:

Tap flag to report any problems with this question.


Question 1758: Where is the site of action of furosemide?
Choices:
1. Collecting duct
2. Descending limb, loop of Henle
3. Distal convoluted tubule
4. Ascending limb, loop of Henle
Answer: 4 - Ascending limb, loop of Henle
Explanations:
Furosemide acts on ascending limb of loop of Henle
Ascending loop of Henle is impermeable to water and permeable to
electrolytes
Furosemide inhibits absorption of electrolytes by blocking a common co-
transporter
Furosemide is a sulfa drug and it is not to be given in patients with sulfa
allergy

Go to the next page if you knew the correct answer, or click the link images
below to further research the concepts in this question (if desired).

Research Concepts:
Furosemide:

Tap flag to report any problems with this question.


Question 1759: Which of the following drugs is an antidote for heparin
overdose?

Choices:
1. Flumazenil
2. Warfarin
3. Protamine sulfate
4. Vitamin K
Answer: 3 - Protamine sulfate
Explanations:
Protamine sulfate quickly terminates the action of heparin on a 1:1 mole
basis
Vitamin K is an antidote for warfarin poisoning
Flumazenil is an antidote for diazepam overdose

Go to the next page if you knew the correct answer, or click the link images
below to further research the concepts in this question (if desired).

Research Concepts:
Protamine:

Tap flag to report any problems with this question.


Question 1760: What serum BNP level is seen with constrictive
pericarditis?

Choices:
1. 100 ng per L
2. 300 ng per L
3. 500 ng per L
4. 700 ng per L
Answer: 1 - 100 ng per L
Explanations:
A BNP level greater than 650 ng per L is indicative of restriction
cardiomyopathy
BNP lower than 150 ng per L is indicative of constriction cardiomyopathy

Go to the next page if you knew the correct answer, or click the link images
below to further research the concepts in this question (if desired).

Research Concepts:
Constrictive Pericarditis:

Tap flag to report any problems with this question.


Question 1761: What is NOT a feature on ECG in a patient with atrial
fibrillation?

Choices:
1. Absent P waves
2. Irregular ventricular rate
3. Rapid oscillation f waves
4. Wide QRS
Answer: 4 - Wide QRS
Explanations:
ECG in a patient with atrial fibrillation will show absent P waves, irregular
heart rate, and rapid oscillation f waves.
The ventricular response is usually irregular in atrial fibrillation.
The QRS is normal in patients with atrial fibrillation. Otherwise, the rhythm
would be similar to ventricular fibrillation.

Go to the next page if you knew the correct answer, or click the link images
below to further research the concepts in this question (if desired).

Research Concepts:
Atrial Fibrillation:

Tap flag to report any problems with this question.


Question 1762: Sodium polystyrene sulfonate (Kayexalate) administration
results in which of the following electrolyte changes?

Choices:
1. Hypokalemia
2. Hypomagnesemia
3. Hyponatremia
4. Hypocalcemia
Answer: 1 - Hypokalemia
Explanations:
Sodium polystyrene sulfonate is a type of polymer and ionomer based on
polystyrene.
The mechanism of action is mediated by ion exchange of potassium with
sodium.
The resin absorbs 1 mEq of potassium per gram and releases 1 mEq of
sodium.
It is effective as a spermicide and a topical microbicide.

Go to the next page if you knew the correct answer, or click the link images
below to further research the concepts in this question (if desired).

Research Concepts:
Sodium Polystyrene Sulfonate:

Tap flag to report any problems with this question.


Question 1763: Which one of the following medications can cause acute
exacerbation of gout?

Choices:
1. Amiloride
2. Furosemide
3. Acetazolamide
4. Spironolactone
Answer: 2 - Furosemide
Explanations:
Hyperuricemia can result from thiazide or loop diuretic therapy
Potent diuretics decrease urate excretion by increasing net urate
reabsorption
Volume depletion appears to play an important role in this response
Amiloride and spironolactone are potassium sparing diuretics

Go to the next page if you knew the correct answer, or click the link images
below to further research the concepts in this question (if desired).

Research Concepts:
Gout:

Furosemide:

Tap flag to report any problems with this question.


Question 1764: A patient with a CABG done 3 years ago had a LIMA to
LAD anastomosis, SVG to RCA and SVG to circumflex. Now he has severe
mitral regurgitation. Cardiac catheterization reveals all patent grafts. What is the
best exposure to this patient?

Choices:
1. Left thoracotomy
2. Median sternotomy
3. Right thoracotomy
4. Partial sternal split
Answer: 3 - Right thoracotomy
Explanations:
In a patient with patent grafts and now needing a REDO for a valve, the
right thoracotomy provides excellent exposure.
The aorta can be cross-clamped but the best way to do the procedure is
under hypothermic fibrillatory arrest.
Left thoracotomy is another choice.

Go to the next page if you knew the correct answer, or click the link images
below to further research the concepts in this question (if desired).

Research Concepts:
Mitral Regurgitation:

Coronary Artery Bypass Graft:

Tap flag to report any problems with this question.


Question 1765: Which of the following is the primary reason why
prophylactic antibiotics often are not productive?

Choices:
1. Inappropriate antibiotic
2. Under-dosing
3. Wrong timing of administration
4. Wrong site of administration
Answer: 3 - Wrong timing of administration
Explanations:
The most common error in prophylactic antibiotic administration is
erroneous timing in administration, which occurs in more than one third of
cases.
An effective antibiotic is selected in the majority of cases.

Go to the next page if you knew the correct answer, or click the link images
below to further research the concepts in this question (if desired).

Research Concepts:
Prophylactic Antibiotics:

Tap flag to report any problems with this question.


Question 1766: The aortic arch is derived from:
Choices:
1. Aortic arch 1
2. Aortic arch 2
3. Aortic arch 3
4. Aortic arch 4
Answer: 4 - Aortic arch 4
Explanations:
The left fourth aortic arch contributes to a small part of the aortic arch
between the origin of the left carotid artery and the termination of the
ductus arteriosus.
The right subclavian artery to the origin of the internal mammary branch is
formed from the right fourth arch.
The third aortic arch forms the origin of the internal carotid artery.
A remnant of the 1st arch forms part of the maxillary artery.

Go to the next page if you knew the correct answer, or click the link images
below to further research the concepts in this question (if desired).

Research Concepts:
Aortic Arch:

Tap flag to report any problems with this question.


Question 1767: The estimated time a person's brain can be anoxic from
cardiopulmonary failure and not develop permanent brain damage is:

Choices:
1. 10 minutes
2. 5 minutes
3. 2 minutes
4. 20 minutes
Answer: 2 - 5 minutes
Explanations:
After a person is without cardiopulmonary function for 4 to 6 minutes,
permanent brain damage is almost certain

Go to the next page if you knew the correct answer, or click the link images
below to further research the concepts in this question (if desired).

Research Concepts:
Anoxic Encephalopathy:

Circulatory Arrest:

Tap flag to report any problems with this question.


Question 1768: Viral-induced myocarditis is most frequently associated
with:

Choices:
1. Coxsackievirus infections
2. Arenavirus infections
3. Echovirus infections
4. Enterovirus infections
Answer: 1 - Coxsackievirus infections
Explanations:
Coxsackie B viruses are cardiotropic and can infect myocytes.
Myocarditis can lead to dilated cardiomyopathy.
The mechanism of action is probably both direct viral cytotoxicity and
immune-mediated.
Adenovirus, cytomegalovirus (CMV), enterovirus, HIV, and parvovirus also
can be cardiotropic.

Go to the next page if you knew the correct answer, or click the link images
below to further research the concepts in this question (if desired).

Research Concepts:
Myocarditis, Viral:

Coxsackie Viruses:

Tap flag to report any problems with this question.


Question 1769: You receive a prescription to administer 2 chewable
aspirin "stat" for your patient. The patient is suspected of having an acute
myocardial infarction. What is the purpose of this drug in this situation?

Choices:
1. Decrease fever
2. Decrease pain in the chest
3. Decrease local discomfort caused by invasive procedures
4. Affect blood clotting
Answer: 4 - Affect blood clotting
Explanations:
Aspirin acts against platelet aggregation and impedes blood clotting.
Aspirin administration during the acute management of a myocardial
infarction may decrease mortality.
Aspirin is a non-opioid analgesic, non-steroidal anti-inflammatory,
antipyretic, anti-platelet medication. Although aspirin is an antipyretic, it
would not be given for this reason in this client's situation.
Aspirin is an analgesic. However, in this situation it would not be given for
its analgesic properties.
There is no indication that the client is having an invasive procedure.

Go to the next page if you knew the correct answer, or click the link images
below to further research the concepts in this question (if desired).

Research Concepts:
Aspirin:

Myocardial Infarction, Acute:

Tap flag to report any problems with this question.


Question 1770: An adult developed right sided valvular endocarditis after
intravenous drug abuse. After 8 weeks of intravenous antibiotics, the fever still
persists and blood cultures remain positive. What is the next step in his
management?

Choices:
1. Change antibiotics
2. Continue antibiotics for a longer course
3. Consult surgery for excision of the valve without replacement
4. Consult surgery for endovascular resection of the vegetation
Answer: 3 - Consult surgery for excision of the valve without replacement
Explanations:
Once the infection is cleared, valve replacement can be considered.

Go to the next page if you knew the correct answer, or click the link images
below to further research the concepts in this question (if desired).

Research Concepts:
Endocarditis, Libman-Sacks:

Tap flag to report any problems with this question.


Question 1771: Which is NOT TRUE of cardiopulmonary bypass?
Choices:
1. Pulsatile
2. Adjustable
3. Peripheral resistance is a factor
4. Non-pulsatile
Answer: 1 - Pulsatile
Explanations:
The flow in cardiopulmonary bypass is non-pulsatile.

Go to the next page if you knew the correct answer, or click the link images
below to further research the concepts in this question (if desired).

Research Concepts:
Cardiopulmonary Bypass:

Tap flag to report any problems with this question.


Question 1772: A patient has an incompetent tricuspid valve. Which of the
following signs are you most likely to see in this patient?

Choices:
1. Lancisi
2. Austin flint
3. Ladin
4. Larrey
Answer: 1 - Lancisi
Explanations:
Lancisi's sign is usually a large V wave in the jugular wave tracing
It is due to an incompetent tricuspid valve
The V wave occurs when blood flows back into the jugular vein

Go to the next page if you knew the correct answer, or click the link images
below to further research the concepts in this question (if desired).

Research Concepts:
Tricuspid Regurgitation:

Tap flag to report any problems with this question.


Question 1773: Postoperative retention of urine can be treated with which
of the following drugs?

Choices:
1. Bethanechol
2. Prazosin
3. Oxybutynin
4. Atropine
Answer: 1 - Bethanechol
Explanations:
Urinary retention is a common complication during the post operative
period
Cholinergic stimulation helps to contract the detrusor muscle and relieve
the retention
Bethanechol is a cholinergic agent that is selective for M3 muscarinic
receptors
Oxybutynin is an anticholinergic used for treatment of urge incontinence

Go to the next page if you knew the correct answer, or click the link images
below to further research the concepts in this question (if desired).

Research Concepts:
Post-op Urinary Retention:

Bethanechol:

Tap flag to report any problems with this question.


Question 1774: All of the following nursing measures are aimed at
preventing acute respiratory failure EXCEPT:

Choices:
1. Suctioning
2. Administration of oxygen
3. Administration of aspirin
4. Keeping airway open
Answer: 3 - Administration of aspirin
Explanations:
Nursing assessment of the patient at risk for acute respiratory failure is key
in preventing the disease
Some preventative measures include keeping the airway open, suctioning,
positioning, administration of oxygen and aerosol therapy
Administration of aspirin is not a preventative measure with regards to
acute respiratory failure

Go to the next page if you knew the correct answer, or click the link images
below to further research the concepts in this question (if desired).

Research Concepts:
Respiratory Distress/Failure:

Tap flag to report any problems with this question.


Question 1775: Which lung disorder can increase alveolar-arterial oxygen
gradient?

Choices:
1. Bronchitis
2. Pleuritis
3. Pulmonary edema
4. Asthma
Answer: 3 - Pulmonary edema
Explanations:
Alveolar-arterial oxygen gradient increases when there is pathologic
shunting.
This can be caused by atelectasis, pneumonia, or pulmonary edema leading
to under ventilation of alveoli.
The alveolar-arterial oxygen gradient is the difference between alveolar
PO2 and arterial PO2 and normally is 5 to 10 mm Hg.

Go to the next page if you knew the correct answer, or click the link images
below to further research the concepts in this question (if desired).

Research Concepts:
Alveolar to Arterial Oxygen Gradient:

Pulmonary Edema:

Tap flag to report any problems with this question.


Question 1776: The most common digestive complaint in the U.S. is:
Choices:
1. Constipation
2. Diarrhea
3. Flatulence
4. None of the above
Answer: 1 - Constipation
Explanations:
Constipation is the most common digestive complaint
No formal definition for constipation exists
Most practitioners define constipation as <3 bowel movements per week
Rome III criteria are currently the research-standard definition of
constipation

Go to the next page if you knew the correct answer, or click the link images
below to further research the concepts in this question (if desired).

Research Concepts:
Constipation:

Tap flag to report any problems with this question.


Question 1777: Which of the following is the LEAST common
complication of sarcoidosis?

Choices:
1. Splenomegaly
2. Liver involvement
3. Congestive heart failure
4. Erythema nodosum
Answer: 3 - Congestive heart failure
Explanations:
Sarcoidosis is a multisystem disease that most commonly involves the
lungs.
Only 2 to 9% of patients have clinical evidence of cardiac involvement.
20% of patients have cardiac involvement on autopsy. Congestive heart
failure is a rare complication.
Splenic involvement, not always symptomatic, is found in over half of
sarcoidosis patients. The liver is involved with sarcoidosis by biopsy in 60-
90%, but most are asymptomatic, only 20-30% showing clinical liver
involvement.
Sarcoidosis involves the skin in about 25%, with erythema nodosum being
the most common presentation.

Go to the next page if you knew the correct answer, or click the link images
below to further research the concepts in this question (if desired).

Research Concepts:
Sarcoidosis:

Tap flag to report any problems with this question.


Question 1778: Which antihypertensive medication is not recommended
in individuals with severe peripheral vascular disease?

Choices:
1. Captopril
2. Metoprolol
3. Nifedipine
4. Aspirin
Answer: 2 - Metoprolol
Explanations:
Beta blockers are common medications used in patients with hypertension.
Beta blockers are not indicated in patients with severe peripheral vascular
disease.
Blocking beta receptors leads to unopposed alpha action and severe
vasoconstriction.
Beta blockers are also not recommended in patients with a history of
asthma.

Go to the next page if you knew the correct answer, or click the link images
below to further research the concepts in this question (if desired).

Research Concepts:
Beta-Blockers:

Peripheral Vascular Disease:

Tap flag to report any problems with this question.


Question 1779: A 59 year old is admitted to the ICU for chest pain. ECG
revealed Q waves and an ST depression in the anterior leads. His cardiac
enzymes were elevated and he was treated with nitroglycerin and inotropic
support for a low BP. He was discharged 7 days later and was again seen in the
emergency department 3 weeks later. His complaints were of a low-grade fever,
chest pain, nausea, and generalized malaise. He complained of pain radiating to
his neck. ECG revealed generalized ST segment elevation. What is the most
likely diagnosis?

Choices:
1. Mycardial infarction
2. Dressler syndrome
3. Pleuritis
4. Pneumonia
Answer: 2 - Dressler syndrome
Explanations:
Dressler syndrome is a secondary form of pericarditis.
Dressler often occurs after a myocardial infarction (MI) and due to
inflammation of the pericardium.
Dressler is self-limiting and may present with low grade fever, chest pain,
and a friction rub.
Dressler tends to occur 7-14 days after an MI and may present with
elevated ESR.

Go to the next page if you knew the correct answer, or click the link images
below to further research the concepts in this question (if desired).

Research Concepts:
Dressler Syndrome:

Tap flag to report any problems with this question.


Question 1780: What is true about the amount of insulin required to
maintain normal blood glucose after surgery?

Choices:
1. Increased
2. Decreased
3. Same
4. No relationship
Answer: 1 - Increased
Explanations:
Insulin requirement is increased due to stress response.

Go to the next page if you knew the correct answer, or click the link images
below to further research the concepts in this question (if desired).

Research Concepts:
Diabetes Mellitus, Type 2:

Diabetes Mellitus, Type 1:

Tap flag to report any problems with this question.


Question 1781: When comparing saphenous vein grafts and internal
mammary conduits, which of the following statements is TRUE?

Choices:
1. IMA has faster atherosclerosis
2. IMA experiences spasm
3. IMA and artery diameter are less equal
4. IMA thrombosis is more common
Answer: 2 - IMA experiences spasm
Explanations:
Patency at 10 years is greater than 90% for IMA grafts and only 40% SVG.

Go to the next page if you knew the correct answer, or click the link images
below to further research the concepts in this question (if desired).

Research Concepts:
Coronary Artery Bypass Graft:

Tap flag to report any problems with this question.


Question 1782: Which of the following local anesthetics possess
antiarrhythmic activity?

Choices:
1. Procaine
2. Lidocaine
3. Bupivacaine
4. Tetracaine
Answer: 2 - Lidocaine
Explanations:
Lidocaine is a local anesthetic with anti arrhythmic properties
Lidocaine is classified as a class 1b anti arrhythmic drug and is used for
ventricular arrhythmias
Lidocaine is also effective in treating jellyfish stings

Go to the next page if you knew the correct answer, or click the link images
below to further research the concepts in this question (if desired).

Research Concepts:
Antiarrhythmic Medication:

Local Anesthetics:

Tap flag to report any problems with this question.


Question 1783: What is the most common primary malignant tumor of the
heart?

Choices:
1. Melanoma
2. Squamous cell carcinoma
3. Angiosarcoma
4. Myxoma
Answer: 3 - Angiosarcoma
Explanations:
Cardiac sarcomas are the most common primary malignant tumors of the
heart.
It should be noted that metastatic tumors are 30 times more common than
primary heart tumors.
Angiosarcomas can occur in any age group and often occur in the right
ventricle.
The symptoms may vary from chest pain, dyspnea, tachycardia, and
hypertension. The prognosis is very poor.

Go to the next page if you knew the correct answer, or click the link images
below to further research the concepts in this question (if desired).

Research Concepts:
Angiosarcoma:

Tap flag to report any problems with this question.


Question 1784: Which group has the highest risk for complications after
minor surgery?

Choices:
1. Males
2. Diabetics
3. Young adults
4. Hypertensives
Answer: 2 - Diabetics
Explanations:
Diabetic patients are at increased risk for surgery.

Go to the next page if you knew the correct answer, or click the link images
below to further research the concepts in this question (if desired).

Research Concepts:
Surgery, Risks:

Tap flag to report any problems with this question.


Question 1785: Which of the following structures forms the right border
of the cardiomediastinal silhouette?

Choices:
1. Left atrium
2. Right atrium
3. Left ventricle
4. Right ventricle
Answer: 2 - Right atrium
Explanations:
The cardiomediastinal silhouette normally occupies half of the chest width.
The right border is formed by the right atrium.
The left atrium is the most posterior cardiac chamber. The left ventricle is
left border forming, and the right ventricle is the most anterior cardiac
chamber.
The right heart border, along the cardiophrenic angle, is a common location
to see a pericardial cyst.

Go to the next page if you knew the correct answer, or click the link images
below to further research the concepts in this question (if desired).

Research Concepts:
Chest Radiograph:

Tap flag to report any problems with this question.


Question 1786: Select the correct statement about myocardial infarction
(MI) in those greater than 70 years.

Choices:
1. Aspirin associated risks are greater than benefit in these patients
2. Patients can present with confusion only
3. Beta blockers are contraindicated due to the high incidence of AV nodal
disease
4. Males are 3 times more likely than females to have an MI in this group
Answer: 2 - Patients can present with confusion only
Explanations:
The elderly can present with atypical symptoms including CVA, confusion,
or a silent MI
Aspirin and beta blocker are both useful for this group
Males are only 1.5 times more likely than females to have an MI in this age
group
Greater than 85 of patients that die from CAD are over 65 years

Go to the next page if you knew the correct answer, or click the link images
below to further research the concepts in this question (if desired).

Research Concepts:
Acute Myocardial Infarction:

Tap flag to report any problems with this question.


Question 1787: The drug that inhibits 5-hydroxy tryptamine 3 receptors
and works as an anti emetic agent is:

Choices:
1. Apomorphine
2. Ondansetron
3. Metoclopramide
4. Prochlorperazine
Answer: 2 - Ondansetron
Explanations:
Ondansetron is a 5HT3 receptor blockers and widely used to treat nausea
and vomiting
Ondansetron is used to treat the severe nausea and vomiting which occurs
in patients receiving cancer chemotherapy
Because of expense, the use of this drug is limited for routine nausea and
vomiting

Go to the next page if you knew the correct answer, or click the link images
below to further research the concepts in this question (if desired).

Research Concepts:
Ondansetron:

Tap flag to report any problems with this question.


Question 1788: A patient with elevated cholesterol, triglycerides, and low
density lipoproteins (LDL) has not shown any decrease in his lipid profile with
diet and exercise. Which of the following medications should be initiated first?

Choices:
1. Ezetimibe
2. Colesevelam
3. Simvastatin
4. Niacin
Answer: 3 - Simvastatin
Explanations:
Simvastatin is an HMG-CoA reductase inhibitor, commonly known as a
"statin." It is a first-line medication in the treatment of hyperlipidemia.
Statins have been shown to decrease LDL, triglycerides, and coronary
events.

Go to the next page if you knew the correct answer, or click the link images
below to further research the concepts in this question (if desired).

Research Concepts:
Hyperlipidemia:

Tap flag to report any problems with this question.


Question 1789: In a patient with a post-transplant cancer, what is the most
important aspect of management?

Choices:
1. Administer radiation
2. Surgical excision
3. Reduce dose of immunosuppression
4. Increase dose of chemotherapeutic drugs
Answer: 3 - Reduce dose of immunosuppression
Explanations:
The first step of management in patients with post-transplant lymphomas is
to reduce the dose of immunosuppression.
Most centers reduce the dose of cyclosporine.
Corticosteroids are usually continued.

Go to the next page if you knew the correct answer, or click the link images
below to further research the concepts in this question (if desired).

Research Concepts:
Posttransplantation Lymphoproliferative Disorders:

Tap flag to report any problems with this question.


Question 1790: Within what ranges is coronary perfusion pressure auto-
regulated?

Choices:
1. 40-120 mmHg
2. 50-130 mmHg
3. 60-160 mmHg
4. 90-180 mmHg
Answer: 3 - 60-160 mmHg
Explanations:
Coronary perfusion pressure (CorPP)= Aortic diastolic pressure (AoDP) -
Left ventricular end diastolic pressure (LVEDP).
LVEDP reflects the pulmonary capillary wedge pressure (PCWP).
Coronary blood flow is largely determined by myocardial metabolism.

Go to the next page if you knew the correct answer, or click the link images
below to further research the concepts in this question (if desired).

Research Concepts:
Cardiac Physiology:

Tap flag to report any problems with this question.


Question 1791: Which vitamin is decreased with the administration of
antibiotic?

Choices:
1. A
2. C
3. E
4. K
Answer: 4 - K
Explanations:
Vitamin K is reduced with antibiotics
Results from reducing colonic bacteria
This can be important in patients on anticoagulation

Go to the next page if you knew the correct answer, or click the link images
below to further research the concepts in this question (if desired).

Research Concepts:
Antibiotics:

Antibiotics:

Vitamin K:

Tap flag to report any problems with this question.


Question 1792: Prostaglandin analogs can be used for all of the following,
except:

Choices:
1. Migraine headaches
2. Pulmonary hypertension
3. Induction of abortion
4. Prevention of closure of a patent ductus
Answer: 1 - Migraine headaches
Explanations:
Bosentan, a prostaglandin analog is used for pulmonary hypertension
Prostaglandins can induce abortion and preterm labor
It can also be used to keep a patent ductus open
Some prostaglandins have a vasodilatory effect and may aggravate migraine
headaches

Go to the next page if you knew the correct answer, or click the link images
below to further research the concepts in this question (if desired).

Research Concepts:
Prostaglandins:

Tap flag to report any problems with this question.


Question 1793: The Air Force/Texas Coronary Atherosclerosis Prevention
Study (AFCAPS/TexCAPS) concluded lovastatin:

Choices:
1. Decreased the risk for first acute major coronary events
2. Reduced HDL-C levels
3. Increased LDL-C levels
4. None of the above
Answer: 1 - Decreased the risk for first acute major coronary events
Explanations:
Lovastatin reduced the risk for first acute major coronary event in men and
women with average TC and LDL-C levels and below average HDL-C
levels.
First acute major coronary events defined as fatal or nonfatal MI, unstable
angina or sudden cardiac death.
Lovastatin reduced LDL-C levels by 25%. It also increased HDL-C levels
by 6% .
AFCAPS/TexCAPS Downs JR et al. JAMA 1998;279:1615-1622

Go to the next page if you knew the correct answer, or click the link images
below to further research the concepts in this question (if desired).

Research Concepts:
Statin Medication:

Tap flag to report any problems with this question.


Question 1794: Which is false about infectious endocarditis (IE)?
Choices:
1. In the elderly patient, endocarditis is most commonly due to rheumatic
heart disease
2. Atrial septal defect is rarely associated with IE
3. In IV cases of endocarditis, no valvular abnormality is ever seen
4. Mechanical valves tend to get infected within the first 3 months after
implantation
Answer: 1 - In the elderly patient, endocarditis is most commonly due to
rheumatic heart disease

Explanations:
Rheumatic heart disease still accounts for many cases of endocarditis in
immigrant populations. However, in the elderly it is calcific aortic stenosis
that is the common pathology
Congenital heart defects can also acquire IE and bicuspid aortic valve is the
most common. Of all the congenital heart defects, ASD is least affected by
IE
The biggest risk factor for IE is residual valve damage caused from a
previous attack of rheumatic fever
In IV drug abusers, there are never any underlying valve abnormalities
visualized and the majority of cases involve the tricuspid valve

Go to the next page if you knew the correct answer, or click the link images
below to further research the concepts in this question (if desired).

Research Concepts:
Endocarditis, Infectious:

Tap flag to report any problems with this question.


Question 1795: In a patient that expired 24-48 hours following occlusion
of the coronary artery, what will a microscopic examination of the myocardial
infarction reveal?

Choices:
1. Granulation tissue
2. Necrosis of myocardium and infiltrates of polymorphonuclear leukocytes
3. Lymphocytic infiltrate
4. Excessive calcium accumulation
Answer: 2 - Necrosis of myocardium and infiltrates of polymorphonuclear
leukocytes

Explanations:
Autopsy done within 24-48 hrs after an MI will typically reveal
accumulation of PMNs, as well as some necrosis.
On gross examination, the infarct is usually not identifiable within the first
12 hours.
After the first 5-7 days, the necrotic areas are invaded by granulation tissue.
Ultimately, the infarct is replaced by a scar.

Go to the next page if you knew the correct answer, or click the link images
below to further research the concepts in this question (if desired).

Research Concepts:
Myocardial Infarction, Acute:

Tap flag to report any problems with this question.


Question 1796: Which of the following must be present to diagnose
malignant hypertension?

Choices:
1. Papilledema
2. Hyperthyroidism
3. Thrombocytopenia
4. None of the above
Answer: 1 - Papilledema
Explanations:
Papilledema must be present for a diagnosis of malignant hypertension
Hyperthyroidism can cause hypertension, but does not need to be present
for a definitive diagnosis
Thrombocytopenia is not related to malignant hypertension

Go to the next page if you knew the correct answer, or click the link images
below to further research the concepts in this question (if desired).

Research Concepts:
Hypertension, Malignant:

Tap flag to report any problems with this question.


Question 1797: Which monoclonal antibody is directed against the
glycoprotein platelet receptor?

Choices:
1. Abciximab
2. Ticlopidine
3. OKT3
4. Hirudin
Answer: 1 - Abciximab
Explanations:
Abciximab is a glycoprotein IIb/IIIa receptor antagonist and a monoclonal
antibody.
It is used to decrease platelet aggregation in patients with unstable angina or
undergoing percutaneous coronary intervention.
It is also used off label in conjunction with thrombolytics.

Go to the next page if you knew the correct answer, or click the link images
below to further research the concepts in this question (if desired).

Research Concepts:
Antiplatelet Medications:

Abciximab:

Tap flag to report any problems with this question.


Question 1798: Adverse effects of nitroglycerin include all of the
following except:

Choices:
1. Headache
2. Hypotension
3. Diarrhea
4. Dizziness
Answer: 3 - Diarrhea
Explanations:
Nitroglycerin can cause a headache, dizziness and hypotension
These side effects are related to its vasodilating properties

Go to the next page if you knew the correct answer, or click the link images
below to further research the concepts in this question (if desired).

Research Concepts:
Nitroglycerin:

Side Effects:

Tap flag to report any problems with this question.


Question 1799: What sympathomimetic amine can be used to pace the
transplanted heart?

Choices:
1. Epinephrine
2. Isoproterenol
3. Dobutamine
4. Norepinephrine
Answer: 2 - Isoproterenol
Explanations:
Isoproterenol is frequently used as an IV drip to pace the transplanted heart
The transplanted heart is devoid of any neural activity since the nerves have
been transected
Isoproterenol can also be used to for serious episodes of heart block and
Adams-Stokes attacks (except when caused by ventricular tachycardia or
fibrillation)

Go to the next page if you knew the correct answer, or click the link images
below to further research the concepts in this question (if desired).

Research Concepts:
Heart Transplantation:

Tap flag to report any problems with this question.


Question 1800: Which of the following is a false statement regarding latex
allergies?

Choices:
1. Approximately 10% of health-care workers experience some form of
allergic reaction to latex
2. Latex is not found in catheters
3. Persons allergic to latex also may be sensitive to fruits such as bananas,
kiwis, pears, pineapples, grapes, and papayas
4. Latex allergies became an important problem with the institution of
universal precautions
Answer: 2 - Latex is not found in catheters
Explanations:
Latex allergy is a significant problem as up to 17% of health-care workers
have allergic reactions to latex.
Reactions can vary from contact dermatitis to anaphylaxis.
Latex can be found in catheters, gloves, and many other medical supplies.
Those allergic to latex may be sensitive to kiwis, bananas, grapes,
pineapples, and papayas.

Go to the next page if you knew the correct answer, or click the link images
below to further research the concepts in this question (if desired).

Research Concepts:
Latex Allergy:

Tap flag to report any problems with this question.


Section 10
Question 1801: Which is NOT a feature of Marfan syndrome?
Choices:
1. Aortic regurgitation
2. Arachnodactyly
3. Dislocated lens
4. Mitral stenosis
Answer: 4 - Mitral stenosis
Explanations:
Marfan syndrome is associated with aortic dissection and valvular
dysfunction.

Go to the next page if you knew the correct answer, or click the link images
below to further research the concepts in this question (if desired).

Research Concepts:
Marfan Syndrome:

Tap flag to report any problems with this question.


Question 1802: The flushing associated with the use of niacin is caused by
which of the following endogenous substances?

Choices:
1. Prostaglandin
2. Leukotriene
3. Thromboxane
4. Histamine
Answer: 1 - Prostaglandin
Explanations:
Prostaglandin release during niacin use is responsible for the flushing side
effect that is frequently seen
This is why aspirin can easily reduce the symptoms if given before niacin
administration

Go to the next page if you knew the correct answer, or click the link images
below to further research the concepts in this question (if desired).

Research Concepts:
Niacin:

Tap flag to report any problems with this question.


Question 1803: A patient treated with nitroglycerin becomes cyanotic
despite adequate blood gases. In addition to stopping the nitroglycerin, which of
the following is also appropriate management of this patient?

Choices:
1. Administration of methylene blue
2. Administration of sodium thiosulfate
3. Administration of 100% oxygen
4. Use of a hyperbaric chamber
Answer: 1 - Administration of methylene blue
Explanations:
Nitroglycerin can cause cyanosis by production of methemoglobinemia and
can be fatal.
Methemoglobin levels can be measured. Treatment is by administering
methylene blue.

Go to the next page if you knew the correct answer, or click the link images
below to further research the concepts in this question (if desired).

Research Concepts:
Methemoglobinemia:

Nitroglycerin:

Tap flag to report any problems with this question.


Question 1804: Which type of cardiac pathology can develop in a 74 year
old bed ridden patient?

Choices:
1. Hypertrophic cardiomyopathy
2. Cardiac atrophy
3. Hypotension
4. Supraventricular tachycardia
Answer: 2 - Cardiac atrophy
Explanations:
Older adults are at risk for changes in the myocardium secondary to
deconditioning.
The left ventricle will have decreased ability to respond to increased
demands.

Go to the next page if you knew the correct answer, or click the link images
below to further research the concepts in this question (if desired).

Research Concepts:
Cardiac Atrophy:

Tap flag to report any problems with this question.


Question 1805: What is the cardiac lesion most commonly found in an
infant with congenital rubella?

Choices:
1. Tetralogy of Fallot
2. Ventricular septal defect
3. Patent ductus arteriosus
4. Coarctation of the aorta
Answer: 3 - Patent ductus arteriosus
Explanations:
Congenital rubella is associated with patent ductus arteriosus (PDA) and
peripheral pulmonary artery stenosis.
The murmur of a patent ductus arteriosus is a continuous "washing
machine" murmur best heard over the left upper sternal border.

Go to the next page if you knew the correct answer, or click the link images
below to further research the concepts in this question (if desired).

Research Concepts:
Patent Ductus Arteriosus:

Congenital Rubella:

Tap flag to report any problems with this question.


Question 1806: Which of the following medications can inhibit the
metabolism of bradykinin?

Choices:
1. Celecoxib
2. Famotidine
3. Phenelzine
4. Captopril
Answer: 4 - Captopril
Explanations:
Captopril can increase bradykinin levels by inhibiting angiotensin
converting enzyme
Celecoxib is a COX-2 inhibitor
Famotidine is an H2 blocker
Phenelzine is a non-selective MAO inhibitor

Go to the next page if you knew the correct answer, or click the link images
below to further research the concepts in this question (if desired).

Research Concepts:
Bradykinin:

ACE Inhibitors:

Tap flag to report any problems with this question.


Question 1807: Pulmonary edema is most often cause by:
Choices:
1. ARDS
2. Right-sided heart failure
3. Left-sided heart failure
4. Aspiration
Answer: 3 - Left-sided heart failure
Explanations:
Edema due to increased hydrostatic pressure can be seen in left sided heart
failure (most common cause of pulmonary edema), volume overload and
mitral stenosis.
Decreased oncotic pressure also favors an edematous state and can be seen
with nephrotic syndrome and cirrhosis of the liver.
Edema due to microvascular injury or alveolar injury is commonly seen
with infections, aspiration, drugs and high altitude.

Go to the next page if you knew the correct answer, or click the link images
below to further research the concepts in this question (if desired).

Research Concepts:
Congestive Heart Failure And Pulmonary Edema:

Tap flag to report any problems with this question.


Question 1808: What is the cause of Meleney gangrene?
Choices:
1. S. aureus
2. Bacteroides
3. Clostridia
4. E. coli
Answer: 1 - S. aureus
Explanations:
Meleney is a postoperative infection that results in gangrene.
The gangrene is often the result of interaction between staphylococcus and
streptococcus.
Meleney gangrene is very painful and requires urgent debridement with
broad spectrum antibiotic coverage.

Go to the next page if you knew the correct answer, or click the link images
below to further research the concepts in this question (if desired).

Research Concepts:
Meleney Gangrene:

Meleney Gangrene:

Tap flag to report any problems with this question.


Question 1809: What is the next best step in a symptomatic patient with
severe aortic regurgitation and ejection fraction of zero point five?

Choices:
1. Aortic valve replacement
2. High dose digoxin
3. Echocardiogram
4. Cardiac catheterization
Answer: 4 - Cardiac catheterization
Explanations:
Patient with symptomatic aortic regurgitation needs AVR
Perform cardiac catheterization first

Go to the next page if you knew the correct answer, or click the link images
below to further research the concepts in this question (if desired).

Research Concepts:
Aortic Regurgitation:

Tap flag to report any problems with this question.


Question 1810: What listed comorbidity will increase operative risk the
most?

Choices:
1. Obesity
2. Age greater than 75
3. Male sex
4. Distant smoking history
Answer: 1 - Obesity
Explanations:
Obesity increases risk of pulmonary complications whereas as age often is
associated preexisting comorbid conditions but does not increase operative
risk on its own

Go to the next page if you knew the correct answer, or click the link images
below to further research the concepts in this question (if desired).

Research Concepts:
Obesity:

Operative Risk:

Tap flag to report any problems with this question.


Question 1811: During a stress test, when should the test be prematurely
stopped?

Choices:
1. When there is sweating
2. When the temperature increases
3. When the heart rate increases
4. When the blood pressure drops by more than 10 mmHg
Answer: 4 - When the blood pressure drops by more than 10 mmHg
Explanations:
The stress test should be halted if there is onset of chest pain, a drop in
blood pressure of more than 10 mmHg, there is dyspnea, or ST segment
elevation or depression.
Other indications to stop the stress test include syncope, dizziness, or ST
elevation of at least 1 mm.
Presence of VT or numerous PVCs should also be an indication to stop the
test.
If the patient does not feel well, stop the test.

Go to the next page if you knew the correct answer, or click the link images
below to further research the concepts in this question (if desired).

Research Concepts:
Treadmill Stress Testing:

Tap flag to report any problems with this question.


Question 1812: A patient was found to have allergy for the local anesthetic
lidocaine. Which of the following anesthetics are good alternatives for this
patient?

Choices:
1. Bipivicaine
2. Articaine
3. Tetracaine
4. Etidocaine,
Answer: 3 - Tetracaine
Explanations:
Lidocaine is an amide anesthetic and so no amides should be used in this
case. Other amides include bupivacaine, etidocaine, cinchocaine, and
articaine.
Esther anesthetics are good alternatives if a patient is allergic to amides.
These include benzocaine, procaine, chloroprocaine, cocaine,
cyclomethycaine, piperocaine, and tetracaine.

Go to the next page if you knew the correct answer, or click the link images
below to further research the concepts in this question (if desired).

Research Concepts:
Local And Regional Anesthesia:

Tap flag to report any problems with this question.


Question 1813: The common carotid artery is derived from which
structure?

Choices:
1. Aortic arch 1
2. Aortic arch 2
3. Aortic arch 3
4. Aortic arch 4
Answer: 3 - Aortic arch 3
Explanations:
The first arch gives rise to the external carotid arteries.
The second arch regresses.
The third pair of arches make up the proximal part of internal carotid
arteries.
The fourth arch on the left forms the aortic arch and on the right forms the
right subclavian artery.

Go to the next page if you knew the correct answer, or click the link images
below to further research the concepts in this question (if desired).

Research Concepts:
Great Vessel Embryology:

Tap flag to report any problems with this question.


Question 1814: Abscesses and pus consist of which of the following?
Choices:
1. Dead neutrophils and protein rich fluid
2. Serosanguinous fluid
3. Lymphocytes
4. Giant cells
Answer: 1 - Dead neutrophils and protein rich fluid
Explanations:
Pus is the yellowish liquid at the center of the abscess.
The pus content is a protein-rich fluid with dead .
Pus or abscesses are common after Staphylococcus infections.
Pus can appear green when the infection is caused by Pseudomonas.

Go to the next page if you knew the correct answer, or click the link images
below to further research the concepts in this question (if desired).

Research Concepts:
Abscess:

Tap flag to report any problems with this question.


Question 1815: Hyperventilation results in all of the following except:
Choices:
1. Hypocapnia
2. Alkalosis
3. Increased cerebrovascular resistance
4. Increased cerebral blood flow
Answer: 4 - Increased cerebral blood flow
Explanations:
Carbon dioxide is important in controlling cerebral blood flow
An increase in arterial pCO2 dilates cerebral blood vessels and a decrease
in pCO2 causes vasoconstriction
This decreased cerebral blood flow may result in a sensation of "faintness"

Go to the next page if you knew the correct answer, or click the link images
below to further research the concepts in this question (if desired).

Research Concepts:
Hyperventilation:

Tap flag to report any problems with this question.


Question 1816: Which is the least likely to trigger an myocardial
infarction?

Choices:
1. Stress
2. Exercise
3. Dream
4. Hyperthermia
Answer: 3 - Dream
Explanations:
Sleeping is the least energy consuming activity

Go to the next page if you knew the correct answer, or click the link images
below to further research the concepts in this question (if desired).

Research Concepts:
Coronary Artery Disease:

Risk Factors For Coronary Artery Disease:

Tap flag to report any problems with this question.


Question 1817: A patient in the ICU with sudden onset of confusion and
visual hallucination is considered to be:

Choices:
1. Lethargic
2. Delirious
3. Dissociative
4. Psychotic
Answer: 2 - Delirious
Explanations:
Delirium is characterized by state of confusion with hallucination.
Lethargic patients have a lowered level of consciousness, with drowsiness,
listlessness, and apathy. It is not characterized by hallucinations.

Go to the next page if you knew the correct answer, or click the link images
below to further research the concepts in this question (if desired).

Research Concepts:
Delirium:

Tap flag to report any problems with this question.


Question 1818: Which population carries the highest mortality rate with
Eisenmenger syndrome?

Choices:
1. Young
2. Pregnant
3. Male
4. Old
Answer: 2 - Pregnant
Explanations:
Pregnancy is absolutely contraindicated in Eisenmenger syndrome
Mortality rates is over 50 percent

Go to the next page if you knew the correct answer, or click the link images
below to further research the concepts in this question (if desired).

Research Concepts:
Eisenmenger Syndrome:

Pregnancy:

Tap flag to report any problems with this question.


Question 1819: The most common cause of congestive heart failure is:
Choices:
1. Cardiomyopathy
2. Valvular heart disease
3. Hypertension
4. Coronary artery disease
Answer: 4 - Coronary artery disease
Explanations:
The leading cause of congestive heart failure is coronary artery disease.

Go to the next page if you knew the correct answer, or click the link images
below to further research the concepts in this question (if desired).

Research Concepts:
Heart Failure, Congestive:

Coronary Artery Disease:

Tap flag to report any problems with this question.


Question 1820: Which sympathomimetic agent is most likely to cause
cardiac arrhythmias?

Choices:
1. Norepinephrine
2. Cocaine
3. Dopamine
4. Dobutamine
Answer: 2 - Cocaine
Explanations:
Cocaine prevents reuptake of catecholamines resulting in higher
sympathetic tone triggering arrhythmias.
It also probably had direct effects on cardiac muscle.
It is theorized that cocaine alters myocardial automaticity directly, that it
causes central autonomic dysregulation with increased neurohumoral and
adrenergic stimulation, and that it causes vasospasm and ischemia.
This results in electrical disturbances and potentiates re-entrant arrhythmia.

Go to the next page if you knew the correct answer, or click the link images
below to further research the concepts in this question (if desired).

Research Concepts:
Toxic, Cocaine:

Tap flag to report any problems with this question.


Question 1821: What does the maze procedure for atrial fibrillation
involve?

Choices:
1. Drug
2. Surgical manipulation
3. Shock
4. Anesthesia
Answer: 2 - Surgical manipulation
Explanations:
Atrial fibrillation post maze procedure is due to surgical manipulation

Go to the next page if you knew the correct answer, or click the link images
below to further research the concepts in this question (if desired).

Research Concepts:
Atrial Fibrillation:

Tap flag to report any problems with this question.


Question 1822: Which is the most likely consequence of high-dose
intravenous epinephrine?

Choices:
1. Tachycardia
2. Bronchospasm
3. Decreased urine output
4. Hypotension
Answer: 1 - Tachycardia
Explanations:
Epinephrine is a sympathomimetic medication.
It causes tachycardia and bronchodilation.
Low doses may cause a decrease in blood pressure due to peripheral
vasodilation.
However, moderate and high doses cause hypertension.

Go to the next page if you knew the correct answer, or click the link images
below to further research the concepts in this question (if desired).

Research Concepts:
Epinephrine:

Tap flag to report any problems with this question.


Question 1823: A neonate develops respiratory distress and cyanosis and
is noted to have a respiratory rate of 80. Cardiovascular exam shows a soft
holosystolic murmur. Oxygen saturation on room air is 68% and does not
improve on 100% oxygen. The chest x-ray is normal. What should be done?

Choices:
1. Begin administration of prostaglandin E1
2. Obtain an electrocardiogram
3. Begin administration of 100% oxygen
4. Call cardiology for an emergency consultation
Answer: 1 - Begin administration of prostaglandin E1
Explanations:
Cyanosis in a newborn that does not improve with 100% oxygen is most
probably due to congenital heart disease
There is most likely a right to left shunt, secondary to obstruction of the
blood flow from the right ventricle
The patient requires immediate intervention to prevent hypoxic ischemic
damage. Prostaglandin E1 opens the ductus arteriosus and allows enhanced
pulmonary blood flow improving oxygenation.
A cardiology consult should be obtained after urgent treatment

Go to the next page if you knew the correct answer, or click the link images
below to further research the concepts in this question (if desired).

Research Concepts:
Congenital Heart Disease:

Tap flag to report any problems with this question.


Question 1824: After a motor vehicle accident a 25 year old male has
fasciotomy for compartment syndrome of the lower extremities.
Rhabdomyolysis develops causing acute renal failure. ECG shows short ST
segment in anterior leads. Select the most appropriate treatment.

Choices:
1. Sodium polystyrene sulfonate
2. Loop diuretics and intravenous fluids
3. Hemodialysis
4. Spiral CT of the chest
Answer: 2 - Loop diuretics and intravenous fluids
Explanations:
Hypercalcemia shortens the duration of repolarization
It is treated with loop diuretics and fluids
Hemodialysis is not needed in most cases
Hyperkalemia would show peaked T waves not shortening of the ST
segment

Go to the next page if you knew the correct answer, or click the link images
below to further research the concepts in this question (if desired).

Research Concepts:
Hypercalcemia:

Rhabdomyolysis:

Tap flag to report any problems with this question.


Question 1825: Which of the following is not commonly used in the
treatment of congestive heart failure?

Choices:
1. Metoprolol
2. Furosemide
3. Captopril
4. Clonidine
Answer: 4 - Clonidine
Explanations:
Congestive heart failure is the inability of the heart to supply sufficient
blood flow to meet the needs of the body.
Treatments include medications (ACE inhibitors, ARBs, beta blockers,
spironolactone, diuretic and digoxin), devices (implantable defibrillator or
biventricular pacemakers) and transplantation.
African American patients have improved prognosis when treated with
hydralazine and nitrates
Clonidine is a direct-acting alpha-2 adrenergic agonist that is used to treat
hypertension, neuropathic pain, opioid detoxification and migraine
headaches.

Go to the next page if you knew the correct answer, or click the link images
below to further research the concepts in this question (if desired).

Research Concepts:
Heart Failure, Congestive:

Tap flag to report any problems with this question.


Question 1826: A patient is on droplet precaution. What personal
protective equipment is needed if working within 3 feet of the patients?

Choices:
1. Mask
2. Mask and eye protection
3. Mask, eye protection, and gloves
4. Mask, eye protection, gloves, and gown
Answer: 2 - Mask and eye protection
Explanations:
Droplet precautions mean that the patient's secretions in droplet form are
contagious
These droplets do not travel more than 3 feet
Healthcare workers should wear masks and eye protection when working
with these patients
The patient should wear a mask if being moved about the facility

Go to the next page if you knew the correct answer, or click the link images
below to further research the concepts in this question (if desired).

Research Concepts:
Droplet Precautions:

Tap flag to report any problems with this question.


Question 1827: A 48 year old male is evaluated for the presence of a
holosystolic murmur. Transthoracic echocardiography shows mitral regurgitation
with dilated left ventricle and left atrium and an ejection fraction of 40%. He
says he feels fine and has never had any shortness or breath or palpitations. What
should be recommended?

Choices:
1. Warfarin prophylaxis
2. Repeat TTE in one year
3. TEE and surgical consult
4. Watchful waiting and endocarditis prophylaxis
Answer: 3 - TEE and surgical consult
Explanations:
Patients with severe gradual onset mitral valve regurgitation with ejection
fraction <50% should undergo a more definitive transesophageal
echocardiogram in preparation for possible valve replacement surgery
Endocarditis prophylaxis is advisable but surgical consult should not be
delayed
As there is no indication of atrial fibrillation there is no need for
anticoagulation

Go to the next page if you knew the correct answer, or click the link images
below to further research the concepts in this question (if desired).

Research Concepts:
Mitral Regurgitation:

Tap flag to report any problems with this question.


Question 1828: A 75-year-old female presents to the emergency
department with a myocardial infarction. She develops asystole and resuscitation
is achieved after a prolonged period. She remains unconscious for 72 hours but
gradually improves slightly. She breathes independently and will swallow food
placed in her mouth. She does not speak or interact. Painful stimuli results with
withdrawal. A feeding tube is placed and there is no change for a few months.
What is the diagnosis?

Choices:
1. Locked-in syndrome
2. Vegetative state
3. Central pontine myelinolysis
4. Severe depression
Answer: 2 - Vegetative state
Explanations:
The patient has no evidence of cognitive function but has sustained
autonomic activity
This can occur secondary to severe damage to the cerebral cortex and intact
brainstem
MRI will reveal the ischemic damage to the cerebral cortex
Protracted hypoxemia or near drowning can cause this condition

Go to the next page if you knew the correct answer, or click the link images
below to further research the concepts in this question (if desired).

Research Concepts:
Vegetative State:

Tap flag to report any problems with this question.


Question 1829: Which of the following agents is not used to treat
Pseudomonas infections?

Choices:
1. Quinolones
2. Sulfonamides
3. Imipenem
4. Aminoglycosides
Answer: 2 - Sulfonamides
Explanations:
Pseudomonas is resistant to many antibiotics. When antibiotics are used
empirically, cultures should be obtained and then the choice of antibiotic
narrowed.
Currently, imipenem and quinolones are used in combination with
aminoglycosides to treat Pseudomonas infections.
Other methods to prevent resistance include probiotic prophylaxis, which
may delay onset of pseudomonas infection in the intensive care unit.

Go to the next page if you knew the correct answer, or click the link images
below to further research the concepts in this question (if desired).

Research Concepts:
Pseudomonas Infection:

Tap flag to report any problems with this question.


Question 1830: Your patient has been discharged with a prescription for
amlodipine besylate (Norvasc). Which of the following will you include in your
discharge instructions?

Choices:
1. The medication may cause bradycardia
2. The medication may cause a drop in blood pressure
3. The medication cannot cause angina
4. The medication is contraindicated while taking sublingual nitroglycerin
Answer: 2 - The medication may cause a drop in blood pressure
Explanations:
Amlodipine besylate (Norvasc) may cause a drop in blood pressure.
Amlodipine besylate is an anti-hypertensive and anti-anginal medication.
It decreases myocardial contractility and oxygen demand by inhibiting the
transport of calcium ions into myocardial and smooth muscle cells. It is
important to teach patients about potential postural hypotension and
measures that avoid syncope or faintness that may occur with sudden
position changes.
Amlodipine besylate (Norvasc) may cause palpitations that are perceptible
to the patient, but does not cause bradycardia. Amlodipine besylate
(Norvasc) can cause angina.
Amlodipine besylate (Norvasc) causes a decrease in frequency and severity
of angina attacks because of its coronary vasodilatory effects. Patient Need:
Health Promotion/Maintenance

Go to the next page if you knew the correct answer, or click the link images
below to further research the concepts in this question (if desired).

Research Concepts:
Calcium Channel blockers:

Tap flag to report any problems with this question.


Question 1831: AV nodal conduction can be reduced with which of the
following medications?

Choices:
1. Digoxin
2. Lidocaine
3. Amlodipine
4. Nifedipine
Answer: 1 - Digoxin
Explanations:
Digoxin is the only drug mentioned from the above list that can reduce A-V
node conduction
Lidocaine is a class I antiarrhythmic that is preferred for use in myocardial
infarction
Amlodipine and nifedipine do not affect AV node function

Go to the next page if you knew the correct answer, or click the link images
below to further research the concepts in this question (if desired).

Research Concepts:
Digoxin:

Tap flag to report any problems with this question.


Question 1832: A patient, who has required epinephrine to maintain
adequate blood pressure after cardiopulmonary bypass, develops lactic acidosis
without any indicators of hypoperfusion. Which type of lactic acidosis does this
patient have?

Choices:
1. It is probably a laboratory error
2. Type A
3. Type B
4. Both type A and type B
Answer: 3 - Type B
Explanations:
Catecholamines are known to cause type B lactic acidosis (not associated
with tissue hypoxia).

Go to the next page if you knew the correct answer, or click the link images
below to further research the concepts in this question (if desired).

Research Concepts:
Inotropes And Vasopressors:

Lactic Acidosis:

Tap flag to report any problems with this question.


Question 1833: Large A waves in the jugular veins are typically seen in
patients with which condition?

Choices:
1. Aortic stenosis
2. Aortic regurgitation
3. Atrial septal defect
4. Tricuspid stenosis
Answer: 1 - Aortic stenosis
Explanations:
Patients with tricuspid stenosis often develop large A waves in the jugular
veins

Go to the next page if you knew the correct answer, or click the link images
below to further research the concepts in this question (if desired).

Research Concepts:
Tricuspid Stenosis:

Tap flag to report any problems with this question.


Question 1834: All of the following may be characteristics of acute
pulmonary embolism (PE) EXCEPT:

Choices:
1. Tachypnea
2. Tachycardia
3. Sudden onset
4. Fever
Answer: 4 - Fever
Explanations:
Fever is not a characteristic of acute PE
Tachypnea, tachycardia and sudden onset are characteristics of acute PE
pulmonary embolism
Chest pain may also be a characteristic of acute PE

Go to the next page if you knew the correct answer, or click the link images
below to further research the concepts in this question (if desired).

Research Concepts:
Pulmonary Embolism:

Tap flag to report any problems with this question.


Question 1835: A patient involved in a motor vehicle accident has
sustained traumatic aortic rupture to the proximal descending aorta. The right
lung is heavily contused. What is the best approach for the repair of this aortic
segment?

Choices:
1. Partial heart bypass with left lung collapse
2. Use of Gott shunt
3. Clamp and sew technique with left lung collapse
4. Femoral-femoral bypass
Answer: 4 - Femoral-femoral bypass
Explanations:
With the right lung damaged, poor oxygenation will occur with one-lung
ventilation (as you have already collapsed the left lung to get exposure to
the thoracic aorta).
Both the Gott shunt and partial heart bypass cannot fully oxygenate the
patient. The patient will need complete bypass and so femoral-femoral
bypass is the technique used.
A femoral-femoral bypass will require full anti-coagulation, but one may
use heparinized-coated catheters.
Stenting may be helpful in such a situation.

Go to the next page if you knew the correct answer, or click the link images
below to further research the concepts in this question (if desired).

Research Concepts:
Chest Trauma:

Tap flag to report any problems with this question.


Question 1836: During an attack of hereditary angioedema, what is the
most common blood finding?

Choices:
1. Elevated white blood cell count
2. Decreased hemoglobin
3. Elevated blood urea nitrogen (BUN) and creatinine
4. Hyperkalemia
Answer: 3 - Elevated blood urea nitrogen (BUN) and creatinine
Explanations:
The results of blood work during an attack of hereditary angioedema are
normal.
The condition results in intravascular fluid sequestration and thus a patient
will show elevated hematocrit or evidence of prerenal failure.
The levels of eosinophils or erythrocyte sedimentation rate are not elevated
in hereditary angioedema.
Levels of complement factors C3 and C1q are normal but C4 may be low.
Sometimes a bowel obstruction may be observed during an attack.

Go to the next page if you knew the correct answer, or click the link images
below to further research the concepts in this question (if desired).

Research Concepts:
Hereditary Angioedema:

Tap flag to report any problems with this question.


Question 1837: A patient is on warfarin for a valve replacement. Her INR
is 15. She has rectal bleeding. What is the best approach?

Choices:
1. Hold next dose of warfarin, give Vitamin K, repeat INR, restart lower dose
warfarin
2. Investigate dietary habits and check for antibodies to heparin
3. Stop warfarin, give FFP, Vitamin K, PCC, repeat INR
4. Hold next dose of warfarin, repeat INR, restart lower dose warfarin
Answer: 3 - Stop warfarin, give FFP, Vitamin K, PCC, repeat INR
Explanations:
In a patient with elevated INR<5 without bleeding the next dose of warfarin
should be held and INR rechecked the following day
In the nonbleeding patient with INR 5-9, Vitamin K should also be added
In the nonbleeding patient with INR 9-20, Vitamin K should be given, and
warfarin held until INR is 2-3
In the nonbleeding patient with INR>20, or if the patient is bleeding,
warfarin should be stopped, and Vitamin K, FFP, PCC given

Go to the next page if you knew the correct answer, or click the link images
below to further research the concepts in this question (if desired).

Research Concepts:
Warfarin:

Gastrointestinal Bleeding, Lower:

Tap flag to report any problems with this question.


Question 1838: Atropine is associated with all of the following side
effects, except:

Choices:
1. Pupillary dilation
2. Excessive salivation
3. Tachycardia
4. Urinary retention
Answer: 2 - Excessive salivation
Explanations:
Atropine is an anticholinergic that is often used prior to surgery to dry up
secretions
Pupillary dilation, tachycardia, and urinary retention are some side effects
of atropine

Go to the next page if you knew the correct answer, or click the link images
below to further research the concepts in this question (if desired).

Research Concepts:
Atropine:

Tap flag to report any problems with this question.


Question 1839: What is the normal PR interval duration on an ECG?
Choices:
1. 0.02-0.08 sec
2. 0.08-0.12 sec
3. 0.04-0.12 sec
4. 0.12-0.20 sec
Answer: 4 - 0.12-0.20 sec
Explanations:
The normal PR interval duration is 0.12-0.20 sec
The P wave represents atrial depolarization
The T wave represents ventricular repolarization

Go to the next page if you knew the correct answer, or click the link images
below to further research the concepts in this question (if desired).

Research Concepts:
Electrocardiogram:

Tap flag to report any problems with this question.


Question 1840: The highest pulmonary flow is seen in patients with?
Choices:
1. Tetralogy Of Fallot
2. Atrial septal defect
3. Ventricular septal defect
4. Aortic stenosis
Answer: 3 - Ventricular septal defect
Explanations:
VSD is associated with highest pulmonary flow

Go to the next page if you knew the correct answer, or click the link images
below to further research the concepts in this question (if desired).

Research Concepts:
Ventricular Septal Defect:

Tap flag to report any problems with this question.


Question 1841: Which cerebral parameter is NOT affected
byhypothermia?

Choices:
1. Less glucose requirement
2. Less oxygen demand
3. Increases blood flow
4. Increases vascular resistance
Answer: 3 - Increases blood flow
Explanations:
In hypothermia state, there is reduced glucose and oxygen demand.
Cerebral blood flow also drops.

Go to the next page if you knew the correct answer, or click the link images
below to further research the concepts in this question (if desired).

Research Concepts:
Hypothermia:

Tap flag to report any problems with this question.


Question 1842: Which of the following can occur when PEEP is used for
patients who are on a mechanical ventilator?

Choices:
1. Increase cardiac output
2. Decrease intrathoracic pressure
3. Increase venous return
4. Increase functional residual capacity
Answer: 4 - Increase functional residual capacity
Explanations:
PEEP shifts lung water from alveoli not the perivascular interstitial space
PEEP can help recruitment of alveoli
PEEP prevents a decrease in FRC
PEEP can increase intrathoracic pressure and decreased venous return

Go to the next page if you knew the correct answer, or click the link images
below to further research the concepts in this question (if desired).

Research Concepts:
Positive End-Expiratory Pressure (PEEP):

Tap flag to report any problems with this question.


Question 1843: Which ECG change is associated with hyperkalemia?
Choices:
1. U wave
2. Tall T wave
3. Long PR interval
4. Absence P wave
Answer: 2 - Tall T wave
Explanations:
Hyperkalemia is associated with wide QRS
Tall T waves

Go to the next page if you knew the correct answer, or click the link images
below to further research the concepts in this question (if desired).

Research Concepts:
Hyperkalemia:

Hyperkalemia:

Tap flag to report any problems with this question.


Question 1844: The phrenic nerve from C3, C4, and C5 innervates what
structure?

Choices:
1. Internal intercostals
2. Diaphragm
3. External intercostals
4. Abdominal intercostals
Answer: 2 - Diaphragm
Explanations:
The diaphragm is the most important muscle during inspiration.
The external intercostals also participate in inspiration.

Go to the next page if you knew the correct answer, or click the link images
below to further research the concepts in this question (if desired).

Research Concepts:
Phrenic Nerve:

Tap flag to report any problems with this question.


Question 1845: Unilateral diaphragm paralysis is due to:
Choices:
1. Phrenic nerve injury
2. Thoracic nerve damage
3. Intercostal nerve damage
4. Costal nerve injury
Answer: 1 - Phrenic nerve injury
Explanations:
Unilateral diaphragmatic paralysis is usually the result of phrenic nerve
injury.

Go to the next page if you knew the correct answer, or click the link images
below to further research the concepts in this question (if desired).

Research Concepts:
Phrenic Nerve Injury:

Diaphragm Disorders:

Tap flag to report any problems with this question.


Question 1846: In which of the following is diaphragmatic pacing not
contraindicated?

Choices:
1. Extensive pulmonary parenchymal disease
2. Muscular atrophy of the diaphragm
3. Phrenic nerve paralysis after congenital heart surgery
4. Diabetic with high quadriplegia
Answer: 4 - Diabetic with high quadriplegia
Explanations:
An intact phrenic nerve in quadriplegic diabetics is not a contraindication to
pacing.
If the lung is diseased or the diaphragm muscle atrophied, or if there is
temporary phrenic nerve paralysis after a coronary artery bypass graft,
pacing is not indicated.
When the diaphragm is paced, a permanent tracheostomy is needed to
prevent upper airway obstruction from discoordinated diaphragmatic and
laryngeal muscle activity.

Go to the next page if you knew the correct answer, or click the link images
below to further research the concepts in this question (if desired).

Research Concepts:
Diaphragmatic Pacing:

Tap flag to report any problems with this question.


Question 1847: Which of the following is FALSE about amiodarone?
Choices:
1. Is used for both atrial and ventricular arrhythmias
2. Blocks sodium, calcium, and potassium channels
3. Can cause corneal deposits
4. Has a very short half life
Answer: 4 - Has a very short half life
Explanations:
Amiodarone is very effective for all arrhythmias but is limited in its used by
its toxicity.
Amiodarone acts by blocking the sodium, calcium, and potassium channels.
Amiodarone has a number of toxicities including pulmonary fibrosis,
corneal deposits, hypothyroidism and tremor.
Amiodarone has a very long half life.

Go to the next page if you knew the correct answer, or click the link images
below to further research the concepts in this question (if desired).

Research Concepts:
Amiodarone:

Tap flag to report any problems with this question.


Question 1848: Which is LEAST likely to be seen on a chest radiograph
of a patient with mitral stenosis?

Choices:
1. Prominent pulmonary arteries
2. Right ventricular distension
3. Pulmonary edema
4. Elevated left main stem bronchus
Answer: 3 - Pulmonary edema
Explanations:
Chest radiograph may show left atrial, right ventricle, and pulmonary artery
enlargement.
Rarely calcification may be visible on the mitral valve annulus.
Pulmonary edema is rarely seen on CXR in this context.
The left atrium sits below the left mainstem bronchus. So left atrial
enlargement can cause an elevation of the left main stem bronchus. The
patient may have a constant cough.

Go to the next page if you knew the correct answer, or click the link images
below to further research the concepts in this question (if desired).

Research Concepts:
Mitral Stenosis:

Tap flag to report any problems with this question.


Question 1849: Which of the following is true of coronary artery disease
(CAD) and smoking?

Choices:
1. Smokers had more frequent ischemic episodes of a shorter duration when
compared to nonsmokers
2. Nonsmokers had less frequent ischemic episodes of a longer duration when
compared to smokers
3. Smokers had more frequent ischemic episodes of a longer duration when
compared to nonsmokers
4. Nonsmokers had more frequent ischemic episodes of shorter duration when
compared to smokers
Answer: 3 - Smokers had more frequent ischemic episodes of a longer
duration when compared to nonsmokers

Explanations:
Cigarette smoking is an established risk factor for CAD
Patients with CAD who smoked had a frequency of angina 3X greater and a
duration 12X greater than those nonsmokers with CAD

Go to the next page if you knew the correct answer, or click the link images
below to further research the concepts in this question (if desired).

Research Concepts:
Smoking:

Coronary Artery Disease:

Tap flag to report any problems with this question.


Question 1850: Cyanosis is a result of:
Choices:
1. Saturated hemoglobin
2. Acidosis
3. Alkalosis
4. Hypoxia
Answer: 4 - Hypoxia
Explanations:
Cyanosis, due to poorly oxygenated blood, may produces bluish purple skin
discoloration and is a late sign of hypoxia.

Go to the next page if you knew the correct answer, or click the link images
below to further research the concepts in this question (if desired).

Research Concepts:
Cyanosis:

Tap flag to report any problems with this question.


Question 1851: Which antihypertensive agent is associated with hair
growth?

Choices:
1. Prazosin
2. Minoxidil
3. Metoprolol
4. Diltiazem
Answer: 2 - Minoxidil
Explanations:
Minoxidil is a vasodilator used in the treatment of hypertension.
Reflex tachycardia can occur with minoxidil. Concomitant use of a beta-
blocker is required.
Minoxidil can lead to water retention and should be used with caution in
patients with congestive heart failure.
Minoxidil can cause hypertrichosis (growth of hair) and has been used in
the treatment of male pattern baldness.

Go to the next page if you knew the correct answer, or click the link images
below to further research the concepts in this question (if desired).

Research Concepts:
Antihypertensive Medications:

Tap flag to report any problems with this question.


Question 1852: Paradoxical splitting of S2 in a patient is most likely due
to which of the following?

Choices:
1. Pulmonary hypertension
2. Left bundle branch block
3. Mitral stenosis
4. Aortic insufficiency
Answer: 2 - Left bundle branch block
Explanations:
When P2 precedes A2 such that the split is maximized in expiration and
decreases in inspiration there is paradoxical splitting of S2
A fixed split in S2 is seen with atrial septal defect
A loud P2 is seen with pulmonary hypertension
Delayed excitation of the left ventricle as in left bundle block causes the
paradoxical splitting of S2

Go to the next page if you knew the correct answer, or click the link images
below to further research the concepts in this question (if desired).

Research Concepts:
Bundle Branch Block, Left:

Tap flag to report any problems with this question.


Question 1853: Hirudin is derived from what source?
Choices:
1. Leech
2. Worm
3. Horse
4. Grapes
Answer: 1 - Leech
Explanations:
Hirudin is an anticoagulant derived from leech saliva
As it is difficult to obtain naturally in large quantities, recombinant
technology is used

Go to the next page if you knew the correct answer, or click the link images
below to further research the concepts in this question (if desired).

Research Concepts:
Hirudin:

Tap flag to report any problems with this question.


Question 1854: Which of the following is most likely to result in behavior
changes in people with multiple risk factors for coronary artery disease?

Choices:
1. Verbal disappointment directed at the patient regarding the behaviors
2. Explanation of why the behaviors are risk factors for coronary artery
disease
3. Warning the patient of the chances of heart attacks and heart failure should
the patient continue their behaviors
4. Verbal praise for changing their behaviors which put them at risk for
coronary artery disease
Answer: 4 - Verbal praise for changing their behaviors which put them at
risk for coronary artery disease

Explanations:
A basic principle of behavior modification is that behavior is learned.
New behavior is more likely to persist if it is rewarded.

Go to the next page if you knew the correct answer, or click the link images
below to further research the concepts in this question (if desired).

Research Concepts:
Primary And Secondary Prevention Of Coronary Artery
Disease:

Tap flag to report any problems with this question.


Question 1855: Which of the following is least likely to cause interstitial
nephritis?

Choices:
1. Methicillin
2. Sulfonamides
3. Asprin
4. Neomycin
Answer: 4 - Neomycin
Explanations:
Hypersensitivity reaction is the most common cause of interstitial nephritis.
Interstitial nephritis can be caused by drugs, penicillins, sulfa drugs, and
nonsteroidal anti-inflammatory drugs.

Go to the next page if you knew the correct answer, or click the link images
below to further research the concepts in this question (if desired).

Research Concepts:
Nephritis, Interstitial:

Tap flag to report any problems with this question.


Question 1856: Which medication would help stabilize blood pressure in a
person already taking isosorbide for angina?

Choices:
1. Ramipril
2. Hydrochlorothiazide
3. Amlodipine
4. Metoprolol
Answer: 4 - Metoprolol
Explanations:
A patient with hypertension and angina should be on a beta blocker.
An ACE inhibitor should be added, if possible.
If needed a low dose of a thiazide diuretic would be the next step.

Go to the next page if you knew the correct answer, or click the link images
below to further research the concepts in this question (if desired).

Research Concepts:
Essential Hypertension:

Angina, Stable:

Tap flag to report any problems with this question.


Question 1857: Which of the following conditions will have a continuous
machinery murmur?

Choices:
1. Atrial septal defect
2. Patent ductus arteriosus
3. Coarctation of aorta
4. Ventricular septal defect
Answer: 2 - Patent ductus arteriosus
Explanations:
Continuous machinery murmur occurs if there is communication in the
circulation and continuous pressure gradient through the cardiac cycle.
Continuous murmurs are often loudest during late systole. This finding
often helps the examiner discriminate between a continuous murmur and an
ejection murmur associated with an early diastolic murmur.
Possible causes of a continuous murmur include pulmonary artery to aorta
communication as in patent ductus arteriosus with is maximal inferior to the
left clavicle.
Another rare cause of continuous murmur includes a coronary artery fistula.

Go to the next page if you knew the correct answer, or click the link images
below to further research the concepts in this question (if desired).

Research Concepts:
Patent Ductus Arteriosus:

Tap flag to report any problems with this question.


Question 1858: Coarctation of the aorta just distal to the subclavian
arteries is diagnosed in a one-week-old neonate. There is a drop in blood
pressure of 50% distal to the lesion. Which value would be increased in this
patient?

Choices:
1. Renal excretion of sodium
2. Glomerular filtration rate
3. Blood flow in the lower body
4. Plasma levels of angiotensin II
Answer: 4 - Plasma levels of angiotensin II
Explanations:
The coarctation is proximal to the kidneys, resulting in activation of the
renin-angiotensin system.
Activation of this system would result in increased angiotensin II and
decreased renal excretion of sodium.
The arterial pressure would be decreased in the lower body.
Glomerular filtration would be reduced compared to a patient with a normal
aorta.

Go to the next page if you knew the correct answer, or click the link images
below to further research the concepts in this question (if desired).

Research Concepts:
Aortic Coarctation:

Tap flag to report any problems with this question.


Question 1859: What is the most successful treatment modality for
ventricular fibrillation (VF)?

Choices:
1. Chest compressions
2. Calling 911
3. Aspirin
4. External electrical defibrillation
Answer: 4 - External electrical defibrillation
Explanations:
External electrical defibrillation with AEDs is the most successful treatment
modality for VF
ACLS is indicated in the setting of VF
It is recommended to use the minimum amount of energy required to
achieve defibrillation

Go to the next page if you knew the correct answer, or click the link images
below to further research the concepts in this question (if desired).

Research Concepts:
Ventricular Fibrillation:

Defibrillation And Cardioversion:

Tap flag to report any problems with this question.


Question 1860: The great cardiac vein empties into the:
Choices:
1. Superior vena cava
2. Inferior vena cava
3. Coronary sinus
4. Thebesian veins
Answer: 3 - Coronary sinus
Explanations:
The great cardiac vein drains the apex of the heart and curves to the left to
empty into the coronary sinus

Go to the next page if you knew the correct answer, or click the link images
below to further research the concepts in this question (if desired).

Research Concepts:
Heart Anatomy:

Tap flag to report any problems with this question.


Question 1861: Select the intervention for congestive heart failure that
reduces hospital admissions and health care costs.

Choices:
1. Echocardiography every 6 months
2. Daily weights with care by a multidisciplinary team
3. Long term skilled nursing care
4. Dabigatran
Answer: 2 - Daily weights with care by a multidisciplinary team
Explanations:
It has been shown that regular monitoring of a patient with CHF allows
initiating care before hospitalization is required
Multidisciplinary teams with active patient involvement has shown to be
most effective
Periodic echocardiograms do not make a difference
Many medications decrease exacerbations and hospitalizations but not
dabigatran

Go to the next page if you knew the correct answer, or click the link images
below to further research the concepts in this question (if desired).

Research Concepts:
Heart Failure, Congestive:

Multidisciplinary Team:

Tap flag to report any problems with this question.


Question 1862: A pulmonary artery catheter is usually inserted to assess
which of the following?

Choices:
1. Pressures with in the different heart chambers
2. Carbon dioxide concentration of the blood in different heart chambers
3. Degree of methemoglobinemia, if any
4. Wall motion abnormalities of the left ventricle
Answer: 1 - Pressures with in the different heart chambers
Explanations:
The pulmonary capillary wedge pressure reflects the left ventricular end-
diastolic pressure and guides the physician in determining fluid
management for the patient

Go to the next page if you knew the correct answer, or click the link images
below to further research the concepts in this question (if desired).

Research Concepts:
Pulmonary Artery Catheterization:

Tap flag to report any problems with this question.


Question 1863: Stenosis of the aorta can cause which of the following?
Choices:
1. Right ventricular atrophy
2. Right ventricular hypertrophy
3. Left ventricular atrophy
4. Left ventricular hypertrophy
Answer: 4 - Left ventricular hypertrophy
Explanations:
Blood leaves the left ventricle through the aortic valve and enters the aorta.
Aortic stenosis restricts the flow of blood and can cause left ventricular
hypertrophy.
Later complications can include chest pain, heart failure, syncope, and
death.
The most common cause of aortic stenosis is bicuspid valve, followed by
degenerative, and postinflammatory,

Go to the next page if you knew the correct answer, or click the link images
below to further research the concepts in this question (if desired).

Research Concepts:
Aortic Stenosis:

Tap flag to report any problems with this question.


Question 1864: Anti-ischemic medications should be continued after
hospital discharge in all of the following unstable angina (UA) patients,
EXCEPT:

Choices:
1. No coronary revascularization procedure
2. Unsuccessful coronary revascularization
3. Successful coronary revascularization
4. Recurring anginal symptoms after revascularization
Answer: 3 - Successful coronary revascularization
Explanations:
ACC/AHA guidelines recommend the continuation of anti-ischemic
medications after revascularization in all, except those undergoing
successful procedures
All UA/NSTEMI patients should receive prescriptions for nitroglycerin
upon discharge
Easily understandable medication directions should also be provided

Go to the next page if you knew the correct answer, or click the link images
below to further research the concepts in this question (if desired).

Research Concepts:
Angina, Unstable:

Revascularization Procedures In Coronary Artery


Disease:

Tap flag to report any problems with this question.


Question 1865: Early signs and symptoms of alcohol withdrawal:
Choices:
1. Decreased urine output
2. Diarrhea
3. Convulsions
4. Agitation
Answer: 4 - Agitation
Explanations:
Patients can develop alcohol withdrawal symptoms as early as 12 hours
after their last drink
Nausea, vomiting, changes in vital signs can accompany agitated behavior

Go to the next page if you knew the correct answer, or click the link images
below to further research the concepts in this question (if desired).

Research Concepts:
Alcohol Withdrawal:

Tap flag to report any problems with this question.


Question 1866: A local anesthetic will usually block ion channels at what
location?

Choices:
1. In the myelinated area
2. Inside of nerve
3. Outside of nerve
4. Only the axon
Answer: 2 - Inside of nerve
Explanations:
Local anesthetics block from inside of the nerve cell in the unmyelinated
area.
Unmyelinated areas are known as the nodes of Ranvier.

Go to the next page if you knew the correct answer, or click the link images
below to further research the concepts in this question (if desired).

Research Concepts:
Local Anesthetics:

Tap flag to report any problems with this question.


Question 1867: A 72-year-old male is admitted to the ICU with dyspnea,
crackles, and chest pain. His blood pressure is 70/40 and he has a 3rd heart
sound. His cardiac enzymes are elevated. Which is not true of this disorder?

Choices:
1. There may be ECG changes
2. The SVR is usually low
3. Dopamine may be used for treatment
4. The JVP is elevated
Answer: 2 - The SVR is usually low
Explanations:
In carcinogenic shock, there may be many signs and symptoms. The patient
may develop a third heart sound and there maybe crackles on auscultation.
The myocardium usually is severely damaged and unable to pump blood.
Most patients have a low cardiac output, high SVR and an elevated JVP.
The condition is treated with drugs like dopamine, dobutamine, or
milrinone or an after load-reducing agent like ACEI.
Besides oxygen, other patients may benefit from intravenous morphine.
Some patient may benefit from an intra aortic balloon pump. However,
cardiogenic shock carries a very high mortality, irrespective of the type of
treatment.

Go to the next page if you knew the correct answer, or click the link images
below to further research the concepts in this question (if desired).

Research Concepts:
Shock, Cardiogenic:

Tap flag to report any problems with this question.


Question 1868: What is the best initial treatment for a geriatric female
with history of hypertension on hydrochlorothiazide who presents with severe
chest pain that radiates to her back and dyspnea who is found to have a BP of
205/98, pulmonary rales, an ECG with ST changes, and a chest x-ray with a
widened mediastinum?

Choices:
1. Immediate cardiac catheterization with likely stent placement
2. IV nitroprusside and an urgent transesophageal echocardiogram
3. IV morphine and nitrates, oxygen and aspirin
4. Serial cardiac enzymes and IV beta-blockers and heparin
Answer: 2 - IV nitroprusside and an urgent transesophageal echocardiogram
Explanations:
This case is suggestive of aortic dissection that can be diagnosed with
transesophageal echo
Beta-blockers are for rate control and nitroprusside for BP control
A proximal, Type A, dissection is associated with aortic insufficiency and
hypertension and requires urgent repair while a distal dissection is
associated with arterial obstruction leading to renal failure and bowel
ischemia

Go to the next page if you knew the correct answer, or click the link images
below to further research the concepts in this question (if desired).

Research Concepts:
Dissection, Aortic:

Tap flag to report any problems with this question.


Question 1869: Aspirin after coronary artery bypass grafting:
Choices:
1. Improves patency at 1 year post bypass
2. Is effective only if begun within 24 hours postop
3. Improves patency at 3 years post bypass
4. Should be used with ticlopidine for at least one year
Answer: 4 - Should be used with ticlopidine for at least one year
Explanations:
Studies have shown that there is no additional benefit of adding ticlopidine
to aspirin after CABG

Go to the next page if you knew the correct answer, or click the link images
below to further research the concepts in this question (if desired).

Research Concepts:
Ticlopidine:

Coronary Artery Bypass Graft:

Tap flag to report any problems with this question.


Question 1870: A 71 year old has suddenly developed congestive heart
failure (CHF). An echocardiogram shows an ejection fraction of <25%. Which
of the following drugs is most likely to precipitate CHF?

Choices:
1. ACE inhibitor
2. Thiazide
3. ASA
4. Calcium channel blocker
Answer: 4 - Calcium channel blocker
Explanations:
Congestive heart failure can be precipitated or exacerbated by beta
blockers, non-dihydropyridine calcium channel blockers, NSAIDS, and
alcohol.
These agents should be avoided if possible.

Go to the next page if you knew the correct answer, or click the link images
below to further research the concepts in this question (if desired).

Research Concepts:
Heart Failure, Congestive:

Tap flag to report any problems with this question.


Question 1871: Most common cause of acquired mitral regurgitation in
North America is?

Choices:
1. Rheumatic fever
2. Degenerative disease
3. Endocarditis
4. Coronary heart disease
Answer: 2 - Degenerative disease
Explanations:
Mitral valve degenerative disease or myxomatous disease is the most
common cause of MR in America
All the chordae are thinned and elongated with billowing of the mitral valve
The posterior annular dilatation is symmetric and involves both
commissures equally

Go to the next page if you knew the correct answer, or click the link images
below to further research the concepts in this question (if desired).

Research Concepts:
Mitral Regurgitation:

Tap flag to report any problems with this question.


Question 1872: Which is associated with poor outcome with acute
myocardial infarction?

Choices:
1. High hemoglobin
2. High WBC
3. High hematocrit
4. High platelet
Answer: 2 - High WBC
Explanations:
WBC correlates with mortality
CRP also correlates with mortality rate

Go to the next page if you knew the correct answer, or click the link images
below to further research the concepts in this question (if desired).

Research Concepts:
Acute Myocardial Infarction:

Leukocytosis:

Tap flag to report any problems with this question.


Question 1873: Which is most suggestive of digoxin toxicity?
Choices:
1. Yellow vision
2. Galactorrhea
3. Constipation
4. Seizures
Answer: 1 - Yellow vision
Explanations:
Digoxin toxicity can produce visual changes, such as blurred or yellow
vision
Long-term use can lead to gynecomastia in men, but not galactorrhea
Central nervous system changes include confusion, apathy, dizziness,
depression, and anxiety
Digoxin may also cause palpitations, symptoms of heart block, vomiting,
and diarrhea

Go to the next page if you knew the correct answer, or click the link images
below to further research the concepts in this question (if desired).

Research Concepts:
Digoxin Toxicity:

Tap flag to report any problems with this question.


Question 1874: A 6 year scheduled for elective outpatient surgery presents
with productive cough with yellow sputum. What should be done?

Choices:
1. Postpone surgery
2. Proceed with surgery
3. Start PO antibiotics
4. IV antibiotics
Answer: 1 - Postpone surgery
Explanations:
URI can lead to bronchospasm.
It is recommended to postpone elective surgery.

Go to the next page if you knew the correct answer, or click the link images
below to further research the concepts in this question (if desired).

Research Concepts:
Preoperative Evaluation And Management:

Upper Respiratory Tract Infection:

Tap flag to report any problems with this question.


Question 1875: Which structures below is least likely to be injured during
mitral valve surgery?

Choices:
1. Circumflex coronary artery
2. Aortic valve
3. AV node
4. SA node
Answer: 4 - SA node
Explanations:
All the structures above except the SA node may be damaged during mitral
valve surgery

Go to the next page if you knew the correct answer, or click the link images
below to further research the concepts in this question (if desired).

Research Concepts:
Mitral Valve Replacement:

Cardiac Valvular Surgery:

Tap flag to report any problems with this question.


Question 1876: Which is not a characteristic of Tetralogy of Fallot?
Choices:
1. Right ventricular hypertrophy
2. Ventricular septal defect
3. Overriding aorta
4. Obstructive pulmonary vein
Answer: 4 - Obstructive pulmonary vein
Explanations:
TOF is characterized by obstruction of pulmonary artery

Go to the next page if you knew the correct answer, or click the link images
below to further research the concepts in this question (if desired).

Research Concepts:
Tetralogy Of Fallot:

Tap flag to report any problems with this question.


Question 1877: Bypass of minimally stenosed vessels with the internal
mammary artery results in which of the following?

Choices:
1. Augmented perfusion of the heart
2. String sign
3. Improved ejection fraction
4. Occlusion of the coronary artery
Answer: 2 - String sign
Explanations:
Bypass of minimally stenosed vessels with an arterial conduit like the IMA
will result in competitive flow and cause atrophy of the arterial graft or a
string sign as seen on an angiogram.

Go to the next page if you knew the correct answer, or click the link images
below to further research the concepts in this question (if desired).

Research Concepts:
Internal Mammary Artery Bypass:

Tap flag to report any problems with this question.


Question 1878: Cardiogenic shock is associated with pulmonary capillary
wedge pressure of:

Choices:
1. Greater than 15 mm Hg
2. Lesser than 15 mm Hg
3. Greater than 5 mm Hg
4. Lesser than 5 mm Hg
Answer: 1 - Greater than 15 mm Hg
Explanations:
Cardiogenic shock is characterized by pulmonary arterial sedge pressure
greater than 15 mm Hg

Go to the next page if you knew the correct answer, or click the link images
below to further research the concepts in this question (if desired).

Research Concepts:
Cardiogenic Shock:

Tap flag to report any problems with this question.


Question 1879: Which of the following is the most common valvular
disorder in American children?

Choices:
1. Infectious endocarditis
2. Atrial myxomas
3. Mitral valve prolapse
4. Aortic stenosis
Answer: 3 - Mitral valve prolapse
Explanations:
Mitral valve prolapse is the most common valvular defect in children in the
United States.
Rheumatic valvular disease is the most common valvular disease in the
developing world.
Myxomas are usually found in the left atrium.

Go to the next page if you knew the correct answer, or click the link images
below to further research the concepts in this question (if desired).

Research Concepts:
Mitral Valve Prolapse:

Tap flag to report any problems with this question.


Question 1880: A male is examined by a medical student for heart
problems. The student reclines the patients and holds the arm up vertically. He
then notices and feels a tapping impulse transmitted through the bulk of the
muscles. Based on these findings, what condition does this patient have?

Choices:
1. Aortic stenosis
2. Aortic regurgitation
3. Thoracic outlet syndrome
4. Raynaud's syndrome
Answer: 2 - Aortic regurgitation
Explanations:
A water hammer pulse is bounding and very forceful.
A water hammer pulse is often seen in aortic regurgitation because of the
increased stroke volume of the left ventricle.
The wide pulse pressure often results in large amount of blood being
pumped out during each heart beat.
Other disorders with water hammer pulse include PDA, pregnancy, fever,
and anemia.

Go to the next page if you knew the correct answer, or click the link images
below to further research the concepts in this question (if desired).

Research Concepts:
Aortic Regurgitation:

Tap flag to report any problems with this question.


Question 1881: A 56-year-old male is placed on heparin after developing
an upper extremity deep vein thrombosis. Two days later, he is found to have
profound melena and you decide to reverse the heparin. Which of the following
drugs do you select?

Choices:
1. Warfarin
2. Cryoprecipitate
3. Protamine
4. Vitamin K
Answer: 3 - Protamine
Explanations:
Protamine antagonizes the anticoagulant effects of heparin.
Protamine is derived from fish sperm and has a high arginine content.
The positively charged protein interacts with the negatively charged
heparin.
Adverse effects of protamine include hypersensitivity, bronchoconstriction,
flushing, and hypotension.

Go to the next page if you knew the correct answer, or click the link images
below to further research the concepts in this question (if desired).

Research Concepts:
Anticoagulation:

Tap flag to report any problems with this question.


Question 1882: Which of the following drugs should be used acutely to
treat Wolff-Parkinson-White syndrome?

Choices:
1. Verapamil
2. Adenosine
3. Digoxin
4. None of the above
Answer: 2 - Adenosine
Explanations:
Adenosine blocks the conduction of impulses through the AV node.
Adenosine increases the efflux of potassium, hyperpolarizing the nodal
cells. It is effective in 90% of cases.
Procainamide and esmolol are other anti-arrhythmic medications that are
considered second line.
Patients should be checked for electrolyte imbalances, including
magnesium. Abnormalities should be corrected.

Go to the next page if you knew the correct answer, or click the link images
below to further research the concepts in this question (if desired).

Research Concepts:
Wolff-Parkinson-White Syndrome:

Tap flag to report any problems with this question.


Question 1883: Which of the following is most important to prevent
central line abscess?

Choices:
1. Prophylactic antibiotics
2. Changing the line frequently
3. Aseptic technique for dressing changes
4. Avoid use for parenteral nutrition or transfusions
Answer: 3 - Aseptic technique for dressing changes
Explanations:
A central line needs to be handled using aseptic technique
Each lumen should be assigned on purpose
Prophylactic antibiotics are not of help
Changing the line frequently introduces other risks

Go to the next page if you knew the correct answer, or click the link images
below to further research the concepts in this question (if desired).

Research Concepts:
Central Line Management:

Tap flag to report any problems with this question.


Question 1884: Chronic transplant rejection occurs in:
Choices:
1. Minutes
2. Days
3. Months
4. Seconds
Answer: 3 - Months
Explanations:
Hyperacute transplant rejection occurs within minutes
Acute rejection occurs within days
Chronic rejection can occur in months to years

Go to the next page if you knew the correct answer, or click the link images
below to further research the concepts in this question (if desired).

Research Concepts:
Chronic Transplant Rejection:

Tap flag to report any problems with this question.


Question 1885: Dullness on percussion of the chest occurs with:
Choices:
1. Acute bronchitis
2. Pneumothorax
3. Pleural effusion
4. Healthy lungs
Answer: 3 - Pleural effusion
Explanations:
Chest percussion can yield 3 sounds-dull, normal or hyperresonant
Dullness on percussion is indicative of pleural effusion or lobar pneumonia
Normal percussion is seen in healthy lungs or bronchitis
Hyperresonance is indicative of emphysema or pneumothorax

Go to the next page if you knew the correct answer, or click the link images
below to further research the concepts in this question (if desired).

Research Concepts:
Effusion, Pleural:

Tap flag to report any problems with this question.


Question 1886: Which clinical feature is NOT seen after administration of
morphine?

Choices:
1. Tachycardia
2. Hypotension
3. Rashes
4. Respiratory depression
Answer: 1 - Tachycardia
Explanations:
Morphine causes histamine release and results in hypotension and
bradycardia.

Go to the next page if you knew the correct answer, or click the link images
below to further research the concepts in this question (if desired).

Research Concepts:
Morphine:

Tap flag to report any problems with this question.


Question 1887: Where is a cut down on the saphenous vein done?
Choices:
1. Knee
2. At the groin
3. At the ankle
4. In the mid thigh
Answer: 3 - At the ankle
Explanations:
The greater saphenous vein is very superficial but is best cut down at the
ankle just above the middle malleolus
The saphenous vein joins the femoral vein at the saphenofemoral junction
in the groin and can be deep and difficult to access percutaneously
In the fossa ovalis, the saphenous vein has a few branches including the
superficial epigastric, superficial iliac circumflex and the superficial
pudendal veins
The greater saphenous vein is the conduit of choice for peripheral artery
disease bypass procedures

Go to the next page if you knew the correct answer, or click the link images
below to further research the concepts in this question (if desired).

Research Concepts:
Cutdown, Saphenous Vein:

Tap flag to report any problems with this question.


Question 1888: Which of the following is not an acceptable parameter
prior to extubation?

Choices:
1. pH 7.35
2. PCO2 48
3. D(A-a)O2 < 350 torr on 100% O2
4. PaO2 >70 torr on FIO2 0.5
Answer: 2 - PCO2 48
Explanations:
PCO2 levels greater than 45 torr may indicate that the patient is still not
breathing well and is therefore not ready for extubation.
However, in a CO2 retainer, this level may be acceptable.

Go to the next page if you knew the correct answer, or click the link images
below to further research the concepts in this question (if desired).

Research Concepts:
Ventilator Management:

Tap flag to report any problems with this question.


Question 1889: Which of the following agents is not recommended in
patients with a low ejection fraction and congestive heart failure?

Choices:
1. Carvedilol
2. Captopril
3. Furosemide
4. Diltiazem
Answer: 4 - Diltiazem
Explanations:
Diltiazem is a calcium channel (L- type) blocker prototype
Like verapamil, diltiazem has more depressant effect on heart than
dihydropyridines (e.g., nifedipine)
Diltiazem can cause AV block, CHF, edema and constipation
Diltiazem is widely used to treat arrhythmias, hypertension and angina

Go to the next page if you knew the correct answer, or click the link images
below to further research the concepts in this question (if desired).

Research Concepts:
Heart Failure, Congestive:

Calcium Channel blockers:

Tap flag to report any problems with this question.


Question 1890: What is not a common cause of dilated cardiomyopathy?
Choices:
1. Alcohol
2. Viral infection
3. Genetic disease
4. Hypertension
Answer: 4 - Hypertension
Explanations:
Hypertension does not cause dilated cardiomyopathy

Go to the next page if you knew the correct answer, or click the link images
below to further research the concepts in this question (if desired).

Research Concepts:
Cardiomyopathy, Dilated:

Tap flag to report any problems with this question.


Question 1891: Which of the following will be found in a patient with
erythrocytosis?

Choices:
1. Red rash
2. Erythema nodosum
3. Polycythemia
4. Neutropenia
Answer: 3 - Polycythemia
Explanations:
Erythrocytosis, or polycythemia, is an excess of red blood cells.
In primary polycythemia, there is no identifiable cause of red blood cell
increase, whereas in secondary polycythemia the hormone erythropoietin is
increased.
Other causes of polycythemia include smoking, high altitude living, chronic
carbon monoxide exposure, and disorders of the kidney.
Symptoms of polycythemia include purpura, bruising, tender skin rashes,
thrombosis of small vessels, heat intolerance, weight loss, itching, and
excess warmth in the body.

Go to the next page if you knew the correct answer, or click the link images
below to further research the concepts in this question (if desired).

Research Concepts:
Polycythemia:

Tap flag to report any problems with this question.


Question 1892: Which second messenger system is associated with
nitroglycerin?

Choices:
1. cAMP
2. ATP
3. cGMP
4. GTP
Answer: 3 - cGMP
Explanations:
Nitroglycerin activates guanylate cyclase and converts GTP to cGMP.

Go to the next page if you knew the correct answer, or click the link images
below to further research the concepts in this question (if desired).

Research Concepts:
Nitroglycerin:

Tap flag to report any problems with this question.


Question 1893: Which is not a complication of IV therapy?
Choices:
1. Air embolism
2. Phlebitis
3. Infection
4. Hemangioma
Answer: 4 - Hemangioma
Explanations:
Air embolism can occur during placement of a subclavian catheter.

Go to the next page if you knew the correct answer, or click the link images
below to further research the concepts in this question (if desired).

Research Concepts:
Intravenous Infusion:

Tap flag to report any problems with this question.


Question 1894: Which of the following is an absolute contraindication to
thrombolytic therapy for ST-segment elevation myocardial infarction (STEMI)?

Choices:
1. Prior intracranial hemorrhage (ICH)
2. Active bleeding
3. Known malignant intracranial neoplasm
4. All of the above
Answer: 4 - All of the above
Explanations:
Prior ICH, active bleeding or bleeding diathesis (excluding menses) and
known malignant intracranial neoplasm are absolute contraindications to
thrombolysis in STEMI
Some others are known structural cerebral vascular lesion and ischemic
stroke within 3 months
Lastly, others are suspected aortic dissection, and significant closed-head
trauma or facial trauma within 3 months

Go to the next page if you knew the correct answer, or click the link images
below to further research the concepts in this question (if desired).

Research Concepts:
Thrombolytic Therapy:

Acute ST-elevation Myocardial Infarction (STEMI):

Tap flag to report any problems with this question.


Question 1895: A patient with Prinzmetal angina is started on
nitroglycerin. At what time should the nitrate free period occur?

Choices:
1. Late afternoon
2. Early morning
3. At night
4. Evening
Answer: 1 - Late afternoon
Explanations:
Prinzmetal angina typically occurs in the early morning.
Many people who use nitrates develop tolerance and a drug free period
should be established.
The drug free period should not be in the morning as this is the time when
the attacks usually occur.
The patient should be advised to have a drug free period in the late
afternoon.

Go to the next page if you knew the correct answer, or click the link images
below to further research the concepts in this question (if desired).

Research Concepts:
Angina, Prinzmetal:

Tap flag to report any problems with this question.


Question 1896: Prior to administering tissue plasminogen activator (TPA),
it is most important to obtain which blood test?

Choices:
1. Complete blood count (CBC)
2. Serum sodium
3. Serum creatinine
4. Serum cross-reactive protein
Answer: 1 - Complete blood count (CBC)
Explanations:
TPA is a thrombolytic agent, which can cause bleeding
It is contraindicated in patients with low platelet counts
A CBC should therefore be performed to assess the platelet count
It will also provide a baseline hemoglobin for comparison purposes if
bleeding occurs

Go to the next page if you knew the correct answer, or click the link images
below to further research the concepts in this question (if desired).

Research Concepts:
TPA Therapy:

Tap flag to report any problems with this question.


Question 1897: Which of the following family of viruses are the most
common cause of viral cardiomyopathy?

Choices:
1. Adenoviridae
2. Hepadnaviridae
3. Picornaviridae
4. Reoviridae
Answer: 3 - Picornaviridae
Explanations:
Enteroviruses, members of Picornaviridae, account for at least half of all
cases of acute myocarditis
Of the individual members, group B coxsackie viruses are the most
common causes
Myocarditis due to group A coxsackie viruses, echoviruses, and
polioviruses is also well documented.

Go to the next page if you knew the correct answer, or click the link images
below to further research the concepts in this question (if desired).

Research Concepts:
Myocarditis, Viral:

Tap flag to report any problems with this question.


Question 1898: The following populations are at high risk for aortic
dissection except:

Choices:
1. Patients positive for syphilis
2. Patients with Turner syndrome
3. Adults with Marfan syndrome
4. Patients with mitral valve stenosis
Answer: 4 - Patients with mitral valve stenosis
Explanations:
Patients with Turner syndrome, Marfan syndrome, previous syphilis
infection, and aortic stenosis are highly predisposed to aortic dissection
Blunt chest trauma can result in aortic dissection due to rapid deceleration

Go to the next page if you knew the correct answer, or click the link images
below to further research the concepts in this question (if desired).

Research Concepts:
Aortic Dissection:

Tap flag to report any problems with this question.


Question 1899: Which of the following is NOT a feature of pericardial
cysts?

Choices:
1. Majority occur in the right cardiophrenic angle
2. They communicate with the pericardium
3. Do not require excision
4. Can be aspirated
Answer: 2 - They communicate with the pericardium
Explanations:
Pericardial cysts do not communicate with the pericardium and are
asymptomatic.
They present as a unilocular mass and an occasional patient may present
with chest heaviness.
Diagnosis is usually evident on a chest x-ray and a CT scan. Majority are
located in the right cardiophrenic angle.
VATs is fast becoming an effective technique for removal of these cysts if
the diagnosis is not certain.

Go to the next page if you knew the correct answer, or click the link images
below to further research the concepts in this question (if desired).

Research Concepts:
Pericardial Cyst:

Tap flag to report any problems with this question.


Question 1900: In a patient receiving an intravenous infusion of
nitroglycerin, how often should the blood pressure be measured?

Choices:
1. Every 4 hours
2. Every 2 hours
3. Every 1 hour
4. Continuously
Answer: 4 - Continuously
Explanations:
Intravenous nitroglycerin can produce profound hypotension
The blood pressure should be monitored continuously in patients receiving
an intravenous nitroglycerin infusion.

Go to the next page if you knew the correct answer, or click the link images
below to further research the concepts in this question (if desired).

Research Concepts:
Nitroglycerin, Intravenous:

Tap flag to report any problems with this question.


Question 1901: Which of the following is not true of heparin?
Choices:
1. It is a mucopolysaccharide
2. It is often administered two to three times daily
3. It binds to antithrombin III for its mode of action
4. Prothrombin time should be monitored in patients receiving heparin
Answer: 4 - Prothrombin time should be monitored in patients receiving
heparin

Explanations:
Heparin is a mucopolysaccharide, which is found in basophils and mast
cells.
Heparin acts like an anticoagulant and prevents formation of blood clots.
Unlike fibrinolytics, heparin does not break down clots that are already
formed.
Heparin is not used orally and its activity is monitored using activated
partial thromboplastin time (aPTT) levels.

Go to the next page if you knew the correct answer, or click the link images
below to further research the concepts in this question (if desired).

Research Concepts:
Heparin:

Tap flag to report any problems with this question.


Question 1902: A 54-year-old patient taking nonsteroidal
antiinflammatory drugs for arthritis is scheduled for urgent surgery after
suffering a gunshot to his legs. He is scheduled for a peripheral bypass
procedure. Which laboratory test should be ordered prior to surgery?

Choices:
1. Bleeding time
2. Prothrombin time
3. Euglobin time
4. International normalized ratio
Answer: 1 - Bleeding time
Explanations:
Bleeding time is affected by platelet function. Von Willebrand disease is a
common disorder that affects bleeding time.
Other disorders that cause prolonged bleeding time include
thrombocytopenia, disseminated intravascular coagulation, Bernard Soulier
syndrome, and Glanzmann thrombasthenia.
Aspirin and other non-steroidal anti-inflammatory drugs inhibit cyclo-
oxygenase and prolong bleeding time.
Individuals with Von Willebrand disease experience prolonged bleeding
time. Patients who are on aspirin or non-steroidal anti-inflammatory drugs
are usually told to stop the medication for at least 7-10 days prior to any
elective surgery.

Go to the next page if you knew the correct answer, or click the link images
below to further research the concepts in this question (if desired).

Research Concepts:
Bleeding Time:

Coagulopathy:
Tap flag to report any problems with this question.
Question 1903: Which of the following are common side effects high level
of statin drugs?

Choices:
1. Atrial fibrillation
2. Skin rash and gastrointestinal bleeding
3. Thrombocytopenia
4. Skeletal muscle injury and hepatic dysfunction
Answer: 4 - Skeletal muscle injury and hepatic dysfunction
Explanations:
Muscle injury and hepatic dysfunction are the most common
Muscle injury occurs because of inhibition of dolichol phosphate
Muscle injury can manifest with weakness and myalgias

Go to the next page if you knew the correct answer, or click the link images
below to further research the concepts in this question (if desired).

Research Concepts:
Statin Medication:

Myopathy:

Tap flag to report any problems with this question.


Question 1904: Mitral stenosis causes a high pressure in which heart
chamber?

Choices:
1. Right atrium
2. Right ventricle
3. Left atrium
4. Left ventricle
Answer: 3 - Left atrium
Explanations:
Mitral stenosis causes a higher pressure in the left atrium.

Go to the next page if you knew the correct answer, or click the link images
below to further research the concepts in this question (if desired).

Research Concepts:
Mitral Stenosis:

Tap flag to report any problems with this question.


Question 1905: What is the most common cause of secondary amenorrhea
in a 25-year old female?

Choices:
1. Polycystic ovarian syndrome
2. Pregnancy
3. Premature menopause
4. Bulimia
Answer: 2 - Pregnancy
Explanations:
Secondary amenorrhea is often caused by pregnancy (most common cause)
Secondary amenorrhea is the absence of menses for 3 months in a woman
with previously normal menstruation or 9 months for women with a history
of oligomenorrhea.

Go to the next page if you knew the correct answer, or click the link images
below to further research the concepts in this question (if desired).

Research Concepts:
Amenorrhea, Secondary:

Pregnancy:

Tap flag to report any problems with this question.


Question 1906: Which of the following wound closure choices is NOT an
adequate replacement for suture technique?

Choices:
1. Fibrin tissue adhesive
2. Skin staples
3. Porous strips
4. Healing by secondary intention
Answer: 4 - Healing by secondary intention
Explanations:
While each method has its purpose, they are not interchangeable, and
healing by secondary intention is limited to distinct situations
In general, healing by secondary intention is not an alternative to suture
techniques

Go to the next page if you knew the correct answer, or click the link images
below to further research the concepts in this question (if desired).

Research Concepts:
Wound Closure Technique:

Tap flag to report any problems with this question.


Question 1907: Depolarizing neuromuscular blocking agents provide
skeletal muscle relaxation for endotracheal intubation. Which of the following
medication is in this group?

Choices:
1. Cisatracurium
2. Succinylcholine
3. Atracurium
4. Vecuronium
Answer: 2 - Succinylcholine
Explanations:
Succinylcholine is the only depolarizing neuromuscular-blocking agent in
clinical use
It mimics acetylcholine by binding to the alpha subunits of nicotinic
receptors
The other choices are nondepolarizing neuromuscular blocking drugs

Go to the next page if you knew the correct answer, or click the link images
below to further research the concepts in this question (if desired).

Research Concepts:
Depolarizing Neuromuscular Blocking Drugs:

Tap flag to report any problems with this question.


Question 1908: Which of the following is not true concerning congestive
heart failure?

Choices:
1. ACE inhibitors increase the progression of disease
2. Ejection fraction is the best predictor for mortality rate
3. Beta blockers are beneficial
4. Five year mortality rate is 50 percent
Answer: 1 - ACE inhibitors increase the progression of disease
Explanations:
Studies have shown ACE inhibitors and beta blockers lower the progression
of disease and improve the overall function

Go to the next page if you knew the correct answer, or click the link images
below to further research the concepts in this question (if desired).

Research Concepts:
Heart Failure, Congestive:

Tap flag to report any problems with this question.


Question 1909: Which is true of pulmonary embolism?
Choices:
1. Is seen in a high number of postmortem cases
2. Is diagnosed by x-ray
3. Majority are due to paradoxical emboli
4. Are best treated with thrombectomy
Answer: 1 - Is seen in a high number of postmortem cases
Explanations:
Pulmonary embolism is presence in nearly 70% of individual with a DVT
but more than 50% of these are asymptomatic
Autopsy studies show that nearly 60% of patients who die have pulmonary
embolism and the diagnosis was missed in the majority of these individuals
The chief cause of PE is blood clots that arise from the legs. Immobilization
is a major cause for DVT in post operative patients
Since there is no clinical reliable test for PE, a number of criteria have been
established

Go to the next page if you knew the correct answer, or click the link images
below to further research the concepts in this question (if desired).

Research Concepts:
Pulmonary Embolism:

Tap flag to report any problems with this question.


Question 1910: A 17 year old collapses while playing basketball. He is
found to be in ventricular fibrillatory arrest. What is the most common cause of
ventricular fibrillation in this age group?

Choices:
1. Congenital prolonged QT syndrome
2. Wolff-Parkinson-White syndrome
3. Tetralogy of Fallot
4. Hypertrophic cardiomyopathy
Answer: 4 - Hypertrophic cardiomyopathy
Explanations:
Hypertrophic cardiomyopathy is the most common cause of ventricular
fibrillation in people younger than 30.
Hypertrophic cardiomyopathy is the most common cause of ventricular
fibrillation in athletes.
It has autosomal dominant inheritance.
The hypertrophy of the left ventricle, most commonly of the septum.

Go to the next page if you knew the correct answer, or click the link images
below to further research the concepts in this question (if desired).

Research Concepts:
Cardiomyopathy, Hypertrophic:

Tap flag to report any problems with this question.


Question 1911: Ventricular fibrillation (VF) is treated the same as:
Choices:
1. Nonperfusing ventricular tachycardia
2. Atrial tachycardia
3. Asystole
4. None of the above
Answer: 1 - Nonperfusing ventricular tachycardia
Explanations:
VF and nonperfusing ventricular tachycardia are treated identically
Both require immediate defibrillation
VF is the most common arrhythmia found in sudden cardiac arrest patients

Go to the next page if you knew the correct answer, or click the link images
below to further research the concepts in this question (if desired).

Research Concepts:
Ventricular Fibrillation:

Tap flag to report any problems with this question.


Question 1912: How much fluid does it take to cause cardiac tamponade?
Choices:
1. 50 ml
2. 150 ml
3. 300 ml
4. 500 ml
Answer: 2 - 150 ml
Explanations:
The rate of accumulation of fluid is more important than the volume in the
pathogenesis of cardiac tamponade
As little as 150 ml of fluid can cause clinically significant symptoms while
as much as a liter can slowly accumulate if the pericardium is distensible

Go to the next page if you knew the correct answer, or click the link images
below to further research the concepts in this question (if desired).

Research Concepts:
Cardiac Tamponade:

Tap flag to report any problems with this question.


Question 1913: A patient is seen for lightheadedness and palpitations. The
heart rate is greater than 200 beats per minute and the blood pressure is 70/45
mm Hg. Which of the following is a correct statement?

Choices:
1. Diastolic time decreases more than systolic time decreases
2. Systolic time decreases more than diastolic time decreases
3. Diastolic time decreases but systolic time stays the same
4. Diastolic time stays the same but systolic time decreases
Answer: 1 - Diastolic time decreases more than systolic time decreases
Explanations:
Normally, the cardiac cycle is two thirds diastolic and one third systolic.
During tachycardia diastole is shortened preferentially.

Go to the next page if you knew the correct answer, or click the link images
below to further research the concepts in this question (if desired).

Research Concepts:
Tachycardia:

Cardiac Physiology:

Tap flag to report any problems with this question.


Question 1914: What is the treatment of choice for a hypotensive patient
with atrial fibrillation?

Choices:
1. Amiodarone
2. Quinidine
3. Lidocaine
4. Cardioversion
Answer: 4 - Cardioversion
Explanations:
In any hypotensive patient with an arrhythmia, cardioversion should be the
first step.

Go to the next page if you knew the correct answer, or click the link images
below to further research the concepts in this question (if desired).

Research Concepts:
Atrial Fibrillation:

Tap flag to report any problems with this question.


Question 1915: An intravenous drug user presents with low-grade fever
and painful red lesions on the palm and soles. He says the lesions appeared a few
days ago. He has no appetite and feels tired. He most likely has developed which
of the following?

Choices:
1. Heberden nodes
2. Bouchard nodes
3. Osler nodes
4. Janeway lesions
Answer: 3 - Osler nodes
Explanations:
Osler nodes are reddish painful lesions on the palms and soles.
Osler nodes are usually associated with infective endocarditis.
Other features of endocarditis include Roth spots and Janeway lesions.
Janeway lesions also occur on the palms but are not painful.

Go to the next page if you knew the correct answer, or click the link images
below to further research the concepts in this question (if desired).

Research Concepts:
Endocarditis, Bacterial:

Tap flag to report any problems with this question.


Question 1916: What is the best indicator of tissue oxygenation?
Choices:
1. Saturation
2. PvO2
3. PaCO2
4. PvO2
Answer: 2 - PvO2
Explanations:
PvO2 is a sample from the pulmonary artery
It is an indicator of mixed venous oxygen level
Normal values are between 35 to 40 mmHg

Go to the next page if you knew the correct answer, or click the link images
below to further research the concepts in this question (if desired).

Research Concepts:
Pulmonary Artery Catheterization:

Tap flag to report any problems with this question.


Question 1917: Which sympatholytic has both alpha and beta blocking
activity?

Choices:
1. Pindolol
2. Propranolol
3. Labetalol
4. Acebutolol
Answer: 3 - Labetalol
Explanations:
Labetalol has both alpha and beta blocking ability. It has a short half-life.
It is sometimes used for hypertension in pregnancy although it is considered
to be a category C pregnancy drug.

Go to the next page if you knew the correct answer, or click the link images
below to further research the concepts in this question (if desired).

Research Concepts:
Labetalol:

Tap flag to report any problems with this question.


Question 1918: What IV fluid is to be given with blood products?
Choices:
1. Normal saline
2. Ringers lactate
3. 5% dextrose in water
4. Half normal saline with 20 mEq/L of KCl
Answer: 1 - Normal saline
Explanations:
IV fluids are often used in a Y tubing with blood products
Dextrose containing fluids may lead to activation of coagulation and/or
hemolysis and hypotonic solutions may cause hemolysis.
Normal saline is used to irrigate IV lines during blood transfusion
Lactated Ringers solution has calcium that may cause coagulation.

Go to the next page if you knew the correct answer, or click the link images
below to further research the concepts in this question (if desired).

Research Concepts:
Blood Transfusion:

Tap flag to report any problems with this question.


Question 1919: Stents can be employed in which of the following?
Choices:
1. LIMA
2. Acute thrombus
3. Restenosis
4. All of the above
Answer: 4 - All of the above
Explanations:
SVGs and long lesions may also be stented.

Go to the next page if you knew the correct answer, or click the link images
below to further research the concepts in this question (if desired).

Research Concepts:
Intracoronary Stents:

Tap flag to report any problems with this question.


Question 1920: A patient is taking aspirin for headaches. He is scheduled
for surgery. Which test should be ordered prior to surgery?

Choices:
1. Aspirin platelet function test
2. Prothrombin time
3. Reptilase time
4. Platelet count
Answer: 1 - Aspirin platelet function test
Explanations:
The aspirin platelet function test can be ordered for preoperative evaluation.
The test can also be used for evaluation of women with menorrhagia,
detecting drug-induced platelet dysfunction, and checking patient
compliance with antiplatelet drugs.
It may be appropriate to assist in evaluation of bleeding patients.
It may detect patients with von Willebrand disease and assist in their
treatment with DDAVP.

Go to the next page if you knew the correct answer, or click the link images
below to further research the concepts in this question (if desired).

Research Concepts:
Aspirin:

Tap flag to report any problems with this question.


Question 1921: During admission assessment, your patient tells you she
takes quinidine 4 times a day. Which of the following is true of this medication?

Choices:
1. It is used in the treatment of malaria
2. It normally causes a widened QRS
3. It rarely causes toxic side effects
4. It is a Class 1 anti-arrhythmic
Answer: 4 - It is a Class 1 anti-arrhythmic
Explanations:
Quinidine is a Class 1 anti-arrhythmic. It is indicated for atrial, interatrial,
and ventricular arrhythmias.
Quinine is used in the treatment of malaria, not quinidine. A wide QRS is
not normal and might indicate quinidine toxicity.
Quinidine has numerous side effects and serum levels should be monitored.
Patient Need: Physiological Integrity

Go to the next page if you knew the correct answer, or click the link images
below to further research the concepts in this question (if desired).

Research Concepts:
Antiarrhythmic Medication:

Tap flag to report any problems with this question.


Question 1922: When administering warfarin, how is therapy assessed?
Choices:
1. Start with low dose warfarin
2. Start with high dose warfarin
3. Base therapy on INR
4. Start therapy based on PT levels
Answer: 3 - Base therapy on INR
Explanations:
Warfarin therapy must be monitored with INR. The dose should be adjusted
until the INR is between 2-3.
Low dose warfarin is often combined with aspirin but has not found to be
effective.
High dose warfarin without monitoring can lead to bleeding.
In some poor countries, a standard dose of warfarin is administered when
there is no easy access to a laboratory.

Go to the next page if you knew the correct answer, or click the link images
below to further research the concepts in this question (if desired).

Research Concepts:
Anticoagulation:

Tap flag to report any problems with this question.


Question 1923: What is the treatment of choice for a hypotensive patient
with atrial fibrillation?

Choices:
1. Amiodarone
2. Quinidine
3. Lidocaine
4. Cardioversion
Answer: 4 - Cardioversion
Explanations:
In any hypotensive patient with an arrhythmia, cardioversion should be the
first step of treatment.

Go to the next page if you knew the correct answer, or click the link images
below to further research the concepts in this question (if desired).

Research Concepts:
Cardiogenic Shock:

Atrial Fibrillation:

Tap flag to report any problems with this question.


Question 1924: What is the best position for a hospitalized patient
experiencing difficulty breathing?

Choices:
1. Lithotomy
2. Trendelenburg
3. Reverse Trendelenburg
4. High Fowler's
Answer: 4 - High Fowler's
Explanations:
High Fowler's position is best for a hospitalized patient experiencing
difficulty breathing
The patient's upper torso is positioned between 60 to 90o in relation to the
lower body
It is also useful for patient's who are on feeding precautions
It is also useful when needing dependent drainage after abdominal surgery

Go to the next page if you knew the correct answer, or click the link images
below to further research the concepts in this question (if desired).

Research Concepts:
Respiratory Distress/Failure:

Tap flag to report any problems with this question.


Question 1925: What is most important in the initial management of
severe hemorrhagic shock?

Choices:
1. Place a central venous pressure line early for fluid resuscitation and
monitoring
2. Type and crossmatch
3. Secure the airway and ventilate adequately
4. Rapidly infuse colloids
Answer: 3 - Secure the airway and ventilate adequately
Explanations:
The airway must be established.
Identification and control of bleeding are done simultaneously with IV
access and fluid resuscitation.
Type specific or O-negative blood is given if hemodynamic instability
persists after the infusion of 2 L of crystalloid.

Go to the next page if you knew the correct answer, or click the link images
below to further research the concepts in this question (if desired).

Research Concepts:
Shock, Hemorrhagic:

Tap flag to report any problems with this question.


Question 1926: Which of the following is an absolute contraindication to
cardiopulmonary testing?

Choices:
1. Unstable angina
2. Hypertension
3. Asthma
4. Chronic obstructive pulmonary disease
Answer: 1 - Unstable angina
Explanations:
Unstable angina is an absolute contraindication to cardiopulmonary
exercise testing
Other absolute contraindications include uncontrolled asthma, uncontrolled
hypertension, aortic stenosis, febrile illness, and O2 sat <85% at rest
Relative contraindications include hypertension, heart disease, epilepsy, and
inability to exercise

Go to the next page if you knew the correct answer, or click the link images
below to further research the concepts in this question (if desired).

Research Concepts:
Cardiopulmonary Exercise Testing:

Tap flag to report any problems with this question.


Question 1927: Laboratory findings associated with hemophilia A include
which of the following?

Choices:
1. Prolonged prothrombin time (PT)
2. Prolonged partial thromboplastin time (PTT)
3. Prolonged thrombin time
4. Prolonged fibrin time
Answer: 2 - Prolonged partial thromboplastin time (PTT)
Explanations:
Factor VIII deficiency results in a prolonged PIT, with a normal PT and
thrombin time
Fibrin time is not a true test. Factor VIII is involved in the intrinsic
coagulation cascade, which is initiated by factor XII interaction with high
molecular weight kinins or kallikrein and resulting in serial activation of
factors XI and IX, with factor VIII acting as a cofactor with activated factor
IX for activation of factor X
The PTT measures the function of the intrinsic cascade, and is therefore
prolonged in cases of factor VIII or IX deficiency. The PT measures the
extrinsic clotting cascade, with factor VII interacting with tissue factor to
activate factor X
Activated factor X can then activate factor V, which then activates thrombin
(factor 11), which then cleaves fibrinogen to form fibrin. Factor XIII is then
required for fibrin cross-linking to form a stable blood clot

Go to the next page if you knew the correct answer, or click the link images
below to further research the concepts in this question (if desired).

Research Concepts:
Hemophilia A:

Tap flag to report any problems with this question.


Question 1928: Which of the following is not required during medical
research on human subjects?

Choices:
1. Informed consent
2. Compensation
3. Refusal of treatment
4. Choose alternate treatments
Answer: 2 - Compensation
Explanations:
Human subjects participating in medical research must have informed
consent.
They can refuse treatment or choose alternate treatments.
Some research studies do offer compensation but this is not a legal
requirement.

Go to the next page if you knew the correct answer, or click the link images
below to further research the concepts in this question (if desired).

Research Concepts:
Ethics and Legal Issues:

Research Study:

Tap flag to report any problems with this question.


Question 1929: Select the finding that would help differentiate acute
respiratory distress syndrome from cardiogenic pulmonary edema.

Choices:
1. Hypoxemia
2. Bilateral infiltrates
3. Decreased lung compliance
4. Pulmonary capillary wedge pressure of 16 mm Hg
Answer: 4 - Pulmonary capillary wedge pressure of 16 mm Hg
Explanations:
Cardiogenic pulmonary edema will have PCWP greater than 18 mm Hg
Both conditions may have hypoxemia, bilateral infiltrates, and decreased
lung compliance

Go to the next page if you knew the correct answer, or click the link images
below to further research the concepts in this question (if desired).

Research Concepts:
Acute Respiratory Distress Syndrome:

Pulmonary Edema, Cardiogenic:

Tap flag to report any problems with this question.


Question 1930: A 65-year-old undergoes an exercise stress test, but the
results are inconclusive. What is the next step in his management?

Choices:
1. Holter monitor
2. Transthoracic echocardiogram
3. Exercise scintigraphy
4. CT scan
Answer: 3 - Exercise scintigraphy
Explanations:
If a patient has an inconclusive exercise stress test, one must rule out
coronary artery disease.
One can either proceed with a stress dobutamine echo or nuclear
scintigraphy.
A coronary angiogram can be performed if suspicion is high.
The use of CT scan to assess calcium deposition in the coronary vessels is
still being questioned. The results are hard to interpret.

Go to the next page if you knew the correct answer, or click the link images
below to further research the concepts in this question (if desired).

Research Concepts:
Treadmill Stress Testing:

Tap flag to report any problems with this question.


Question 1931: Which of the following radiological signs is NOT seen on
chest x-ray in a patient with a suspected traumatic aortic dissection?

Choices:
1. Double aortic knob sign
2. Tracheal displacement to the right
3. Displacement of nasogastric tube
4. Notching of the ribs
Answer: 4 - Notching of the ribs
Explanations:
The most common finding of aortic dissection on chest x-ray is mediastinal
widening.
Other features on chest x-ray include a double aortic knob sign, inward
displacement of aortic wall calcification, tracheal displacement, left side
pleural effusion, cardiac enlargement, and left apical opacity.
Notching of ribs is seen in coarctation of the aorta.
Occasionally, mediastinal fat may cause widening on chest x-ray and lead
to a false positive diagnosis. The best test for a stable patient with suspected
dissection is either a chest CT or a transesophageal echocardiogram.

Go to the next page if you knew the correct answer, or click the link images
below to further research the concepts in this question (if desired).

Research Concepts:
Aorta, Trauma:

Tap flag to report any problems with this question.


Question 1932: In a 66 year old patient with the point of maximal impulse
(PMI) palpated below the 5th intercostals space, what should be suspected?

Choices:
1. Right atrial enlargement
2. Right lung collapse
3. Left ventricular enlargement
4. Left atrial enlargement
Answer: 3 - Left ventricular enlargement
Explanations:
The apical impulse is normally at the left midclavicular line in the fifth
interspace.
Displacement laterally and inferiorly can occur with left ventricular
enlargement.

Go to the next page if you knew the correct answer, or click the link images
below to further research the concepts in this question (if desired).

Research Concepts:
Left Ventricular Enlargement:

Left Ventricular Enlargement:

Tap flag to report any problems with this question.


Question 1933: Which of the following is a metabolite of sodium
nitroprusside?

Choices:
1. Thiosulfate
2. Depoprusside
3. Acetaldehyde
4. Cyanide
Answer: 4 - Cyanide
Explanations:
Nitroprusside is a drug that is used for malignant hypertension and in severe
congestive heart failure since it reduces both preload and after load.
Nitroprusside degrades in the human body to release cyanide.
Rhodanese is a mitochondrial enzyme that detoxifies the cyanide by
reaction with thiosulfate.
Without adequate thiosulfate the cyanide ions can rapidly reach toxic levels.

Go to the next page if you knew the correct answer, or click the link images
below to further research the concepts in this question (if desired).

Research Concepts:
Sodium Nitroprusside:

Tap flag to report any problems with this question.


Question 1934: Which of the following does NOT call for the use of
epinephrine?

Choices:
1. Wheezing
2. Prolonging the action of local anesthetics
3. Hypotension
4. Pulmonary edema
Answer: 4 - Pulmonary edema
Explanations:
Epinephrine can induce pulmonary edema in some patients.
Pulmonary edema is best treated with diuretics.

Go to the next page if you knew the correct answer, or click the link images
below to further research the concepts in this question (if desired).

Research Concepts:
Epinephrine:

Tap flag to report any problems with this question.


Question 1935: A patient undergoes abdominal surgery for a splenic
rupture and receives many units of transfused blood. Post-surgery he complains
of numbness around his mouth and a carpopedal spasm. An electrocardiogram
reveals a prolonged QT interval. Which of the following would be the best
intravenous treatment for this patient?

Choices:
1. Bicarbonate
2. Potassium
3. Calcium
4. Digoxin
Answer: 3 - Calcium
Explanations:
Chvostek sign occurs when tapping the skin over the facial nerve causes
ipsilateral contraction of facial muscles. This is a suggestive sign of
hypocalcemia.
Trousseau sign occurs when the blood pressure cuff is inflated on the arm
and causes a flexion of the wrist and extension of interphalangeal joints.
The thumb usually adducts. Trousseau sign is more specific for
hypocalcemia.
Other abnormalities that may occur in hypocalcemia include athetosis,
dystonic spasm, Parkinson disease, and hemiballismus.
Coarse hair, brittle nails, and dry skin are other features of long-standing
hypocalcemia.

Go to the next page if you knew the correct answer, or click the link images
below to further research the concepts in this question (if desired).

Research Concepts:
Hypocalcemia:

Tap flag to report any problems with this question.


Question 1936: New York Heart Association (NYHA) Class III states:
Choices:
1. Patients have no limitation of physical activity
2. Patients have slight limitation of physical activity
3. Patients have marked limitation of physical activity
4. Patients have symptoms at rest and are unable to carry out any physical
activity without discomfort
Answer: 3 - Patients have marked limitation of physical activity
Explanations:
NYHA classifies heart failure and comprises 4 classes. Class I patients have
no limitation of physical activity.
Class II patients have slight limitation of physical activity. Class III patients
have marked limitation of physical activity.
Heart Failure Society of America. Retrieved from
www.abouthf.org/questions_stages.htm

Go to the next page if you knew the correct answer, or click the link images
below to further research the concepts in this question (if desired).

Research Concepts:
Heart Failure, Congestive:

Tap flag to report any problems with this question.


Question 1937: Which of the following is a component of a stress
reaction?

Choices:
1. Release of adrenaline
2. Relaxation of the bronchial tree
3. Release of ACTH
4. All of the above
Answer: 4 - All of the above
Explanations:
Components of a stress reaction include release of adrenalin and ACTH and
relaxation of the bronchial tree
Another component of the reaction is slowing of the digestive process
The stress reaction prepares the body to react efficiently to a stressor

Go to the next page if you knew the correct answer, or click the link images
below to further research the concepts in this question (if desired).

Research Concepts:
Stress Reaction:

Tap flag to report any problems with this question.


Question 1938: How many hours should patients be NPO before surgery?
Choices:
1. At least 12 hours
2. At least 8 hours
3. At least 6 hours
4. At least 24 hours
Answer: 2 - At least 8 hours
Explanations:
Overnight fasting of at least 8 hours is recommended before surgery
Risk of aspiration is high if patients are not NPO before surgery
In emergency surgery gastric contents can be evacuated through NG tube

Go to the next page if you knew the correct answer, or click the link images
below to further research the concepts in this question (if desired).

Research Concepts:
Preoperative Evaluation And Management:

Nil Per Os (NPO):

Tap flag to report any problems with this question.


Question 1939: What medication should be used to perfuse the heart <6
hours after myocardial infarction?

Choices:
1. ASA
2. Plavix
3. Coumadin
4. Alteplase (tPA)
Answer: 4 - Alteplase (tPA)
Explanations:
Streptokinase is often used to break down clots in patients with an acute
myocardial infarction, but allergic reactions limit use.
The major application of thrombolytic agents is in the emergency treatment
of myocardial infarction. Under ideal conditions (within 6 hours following
an MI), these agents will lyse the thrombus in the coronary artery and
reanalyze the vessel.
Primary percutaneous coronary intervention is the preferred procedure. If
this procedure is not possible, fibrinolysis is the best option.
Streptokinase induces activation of plasminogen to produce plasmin, which
breaks up blood clots.

Go to the next page if you knew the correct answer, or click the link images
below to further research the concepts in this question (if desired).

Research Concepts:
Myocardial Infarction, Acute:

Tap flag to report any problems with this question.


Question 1940: Paraplegia after aortic surgery is usually due to which of
the following?

Choices:
1. Thrombus
2. Emboli
3. Hypotension
4. Spinal ischemia
Answer: 4 - Spinal ischemia
Explanations:
A common complication of aortic surgery is low flood in the radicular
arteries and leads to spinal cord ischemia.

Go to the next page if you knew the correct answer, or click the link images
below to further research the concepts in this question (if desired).

Research Concepts:
Paraplegia:

Tap flag to report any problems with this question.


Question 1941: Which of the following is not a typical feature of a
pulmonary embolism?

Choices:
1. Mismatch on an isotopic ventilation/perfusion scan
2. Pleuritic chest pain
3. Peripheral cyanosis
4. Tachypnea
Answer: 3 - Peripheral cyanosis
Explanations:
Peripheral cyanosis is not a feature of a pulmonary embolism.
Most common presenting features of a pulmonary embolism include
dyspnea, chest pain, cough, hemoptysis, and fever.
Depending on the size of the pulmonary embolus, the symptoms may vary.
Friction rub, loud P2, or a right ventricular gallop are heard in only a few
patients.

Go to the next page if you knew the correct answer, or click the link images
below to further research the concepts in this question (if desired).

Research Concepts:
Pulmonary Embolism:

Tap flag to report any problems with this question.


Question 1942: All of the following are classic signs and symptoms of
right ventricular heart failure EXCEPT:

Choices:
1. Tachycardia
2. Peripheral edema
3. Jugular venous distention (JVD)
4. Carotid bruits
Answer: 4 - Carotid bruits
Explanations:
Tachycardia, peripheral edema and JVD are classic signs and symptoms of
right ventricular heart failure
Carotid bruits are not signs of right ventricular failure
Carotid bruits are associated with atherosclerotic carotid artery disease

Go to the next page if you knew the correct answer, or click the link images
below to further research the concepts in this question (if desired).

Research Concepts:
Right Heart Failure:

Tap flag to report any problems with this question.


Question 1943: Which is true of aspiration pneumonia?
Choices:
1. Rare occurrence during anesthesia
2. Can occur after head injuries
3. Due to gram-negative organisms
4. Most occur in the left lung
Answer: 2 - Can occur after head injuries
Explanations:
Aspiration pneumonia presents an acute lung injury from inhalation of
gastric contents. The disorder may occur in individuals with seizures, loss
of consciousness, stroke, drug intoxication, and head trauma.
Aspiration of large amounts of gastric contents is also known as
Mendelssohn syndrome. It can result in severe respiratory compromise.
Aspiration pneumonia are predominantly caused by anaerobic organisms.
Hospitalized patients may also acquired aspiration pneumonia from gram-
negative organisms.
The right middle and lower lung lobes are most common sites of infiltrates
due to the more vertical orientation. The right lobe is involved in patients
who aspirate while in a prone position.

Go to the next page if you knew the correct answer, or click the link images
below to further research the concepts in this question (if desired).

Research Concepts:
Pneumonia, Aspiration:

Tap flag to report any problems with this question.


Question 1944: What is the most common cause of a holosystolic murmur
post acute myocardial infarction?

Choices:
1. Anterior wall infarction
2. Septal wall infarction
3. Inferior wall infarction
4. Normal
Answer: 2 - Septal wall infarction
Explanations:
Holosystolic murmur is likely a VSD
Echo is needed to further evaluate the condition

Go to the next page if you knew the correct answer, or click the link images
below to further research the concepts in this question (if desired).

Research Concepts:
Interventricular Septum Damage:

Acute Myocardial Infarction:

Tap flag to report any problems with this question.


Question 1945: The single greatest determinant for REDO coronary artery
surgery is which of the following?

Choices:
1. Young patients
2. Female patients
3. Diabetics
4. Use of vein grafts
Answer: 4 - Use of vein grafts
Explanations:
Patients requiring REDO surgery today are much older, increasing number
have diabetes, and diffuse peripheral vascular disease.
REDO surgery is undertaken for graft closure, worsening atherosclerosis,
choice of conduit, kinking, etc
The number one factor which determines if a REDO is going to occur is the
choice of conduit.
Late vein graft atherosclerosis is seen on angiography on practically all vein
grafts after 3-5 years.

Go to the next page if you knew the correct answer, or click the link images
below to further research the concepts in this question (if desired).

Research Concepts:
Redo CABG:

Tap flag to report any problems with this question.


Question 1946: PTCA is more cost effective than coronary artery bypass
graft for:

Choices:
1. 2 years
2. 4 years
3. 5 years
4. 10 years
Answer: 3 - 5 years
Explanations:
PTCA is more cost effective for the first 5 years in multi-vessel disease
After 5 years, CABG becomes more cost effective due to rehospitalization
and revascularization

Go to the next page if you knew the correct answer, or click the link images
below to further research the concepts in this question (if desired).

Research Concepts:
Percutaneous Transluminal Coronary Angioplasty:

Coronary Artery Bypass Graft:

Tap flag to report any problems with this question.


Question 1947: Patients on bed rest may be encouraged to periodically
plantar and dorsiflex their ankles. What is the rationale for this?

Choices:
1. To prevent blood clots
2. To prevent muscle atrophy
3. To promote cardiac contractility
4. To increase blood flow to the legs
Answer: 1 - To prevent blood clots
Explanations:
Plantar and dorsal flexion of the ankles promotes venous return from the
lower extremities.
Venous stasis is a risk factor for deep venous thrombosis (DVT)
development.

Go to the next page if you knew the correct answer, or click the link images
below to further research the concepts in this question (if desired).

Research Concepts:
Deep Venous Thrombosis Prophylaxis:

Tap flag to report any problems with this question.


Question 1948: A patient with chronic obstructive pulmonary disease
would show a decrease of which value?

Choices:
1. Total lung capacity
2. Vital capacity
3. Residual volume
4. Functional residual capacity
Answer: 2 - Vital capacity
Explanations:
In obstructive lung disease all parameters increases except vital capacity

Go to the next page if you knew the correct answer, or click the link images
below to further research the concepts in this question (if desired).

Research Concepts:
Obstructive Pulmonary Disease, Chronic (COPD):

Vital Capacity:

Tap flag to report any problems with this question.


Question 1949: A patient develops extreme muscle rigidity soon after
surgery is started. The patient is tachycardic and has a fever of 104 degrees.
What is the most appropriate treatment for this condition?

Choices:
1. Barbiturates
2. Muscle relaxants
3. Dantrolene
4. Acetaminophen
Answer: 3 - Dantrolene
Explanations:
Dantrolene and observation for the next 24 hours are recommended. The
condition can recur.
The patient has malignant hyperthermia.

Go to the next page if you knew the correct answer, or click the link images
below to further research the concepts in this question (if desired).

Research Concepts:
Malignant Hyperthermia:

Tap flag to report any problems with this question.


Question 1950: Morphine may be best characterized by which of the
following statements?

Choices:
1. It is a pure opioid antagonist at the mu, kappa and delta receptors
2. It has an addiction potential equal to that of codeine
3. At high doses it causes death by respiratory depression
4. It is classified as a mixed agonist -antagonist drug
Answer: 3 - At high doses it causes death by respiratory depression
Explanations:
Morphine is the prototype of many narcotic drugs that interact
predominantly with the opioid mu-receptor
These mu-binding sites are discretely distributed in the human brain, spinal
cord and spinal nucleus of trigeminal nerve.

Go to the next page if you knew the correct answer, or click the link images
below to further research the concepts in this question (if desired).

Research Concepts:
Morphine:

Tap flag to report any problems with this question.


Question 1951: Electrocautery current differs from wall current in which
property?

Choices:
1. Resistance
2. Voltage
3. Frequency
4. Amplitude
Answer: 3 - Frequency
Explanations:
Electrocautery current frequency is 105 Hz.

Go to the next page if you knew the correct answer, or click the link images
below to further research the concepts in this question (if desired).

Research Concepts:
Electrocautery:

Tap flag to report any problems with this question.


Question 1952: Which of the following condition is common in patients
with heart failure?

Choices:
1. Constipation from immobility
2. Activity intolerance from left ventricular dysfunction
3. Hypoxia from restricted lung function
4. Cholestasis from diffuse edema
Answer: 2 - Activity intolerance from left ventricular dysfunction
Explanations:
Decreased cardiac output associated with heart failure leads to reduced
oxygen and fatigue
These patients have minimal exercise tolerance

Go to the next page if you knew the correct answer, or click the link images
below to further research the concepts in this question (if desired).

Research Concepts:
Heart Failure:

Tap flag to report any problems with this question.


Question 1953: Which is the LEAST likely complication in patients with
Eisenmenger syndrome?

Choices:
1. Gallstones
2. Renal stones
3. Thrombotic stroke
4. Peripheral vascular disease
Answer: 4 - Peripheral vascular disease
Explanations:
There are many complications that can occur in patients with Eisenmenger
syndrome.
CNS complications include brain abscess, thrombotic strokes, and
hemorrhage.
Both gallstones and renal stones occur with increased frequency in this
condition.
Transient visual loss is a common complaint. Peripheral vascular disease is
not likely.

Go to the next page if you knew the correct answer, or click the link images
below to further research the concepts in this question (if desired).

Research Concepts:
Eisenmenger Syndrome:

Tap flag to report any problems with this question.


Question 1954: Which is the false statement about lung transplant?
Choices:
1. Done for cystic fibrosis
2. Single lung transplant is required for CF patients
3. Contraindicated in pulmonary hypertension
4. More economical than heart transplant
Answer: 2 - Single lung transplant is required for CF patients
Explanations:
Cystic fibrosis patients require a double lung transplant
A single lung transplant in these patients will quickly lead to contamination
of the transplanted lung

Go to the next page if you knew the correct answer, or click the link images
below to further research the concepts in this question (if desired).

Research Concepts:
Transplantation:

Tap flag to report any problems with this question.


Question 1955: In what class of medication is prazosin?
Choices:
1. Beta-blocker
2. Phosphodiesterase inhibitor
3. Calcium channel blocker
4. Alpha-blocker
Answer: 4 - Alpha-blocker
Explanations:
Prazosin is an alpha-blocker that is not widely used.
The medication is sometimes used to treat high blood pressure, but the side
effects limit its use.
Prazosin can cause orthostatic hypotension and dizziness.
The drug may relieve symptoms of benign prostatic hypertrophy.

Go to the next page if you knew the correct answer, or click the link images
below to further research the concepts in this question (if desired).

Research Concepts:
Antihypertensive Medications:

Tap flag to report any problems with this question.


Question 1956: Which is FALSE about endothelial-derived relaxation
factor (EDRF)?

Choices:
1. Causes smooth muscle to relax
2. Most well known is nitrous oxide
3. Prostacyclin is considered to an EDRF
4. Endothelial derived hyperpolarizing factor is thought to be potassium
Answer: 2 - Most well known is nitrous oxide
Explanations:
EDRF are factors produced by the endothelium that cause relaxation of
smooth muscle.
The most well known EDRF is nitric oxide, which induces vasodilatation.
Other EDRF include Prostacyclin and it is felt that another endothelium
derived hyperpolarizing factor may be potassium.
Heme proteins can also induce formation of heat shock proteins that
generate carbon monoxide and relax smooth muscle.

Go to the next page if you knew the correct answer, or click the link images
below to further research the concepts in this question (if desired).

Research Concepts:
Endothelial-derived Relaxation Factor:

Tap flag to report any problems with this question.


Question 1957: Which electrolyte abnormality can predispose to digoxin
toxicity?

Choices:
1. Hyperchloremia
2. Hypernatremia
3. Hypokalemia
4. Hyperkalemia
Answer: 3 - Hypokalemia
Explanations:
Low potassium levels predispose to digoxin toxicity

Go to the next page if you knew the correct answer, or click the link images
below to further research the concepts in this question (if desired).

Research Concepts:
Digoxin Toxicity:

Tap flag to report any problems with this question.


Question 1958: What is the advantage of using pressure-controlled
ventilation (PCV) over assist control ventilation (ACV)?

Choices:
1. There is a decrease in lung over-inflation
2. It is used to assist spontaneous breathing
3. It is more effective in patients with decreased lung compliance
4. None of the above
Answer: 1 - There is a decrease in lung over-inflation
Explanations:
Over-inflation and concomitant lung injury is prevented with PCV by
delivering pressure cycled mechanical breaths
PCV is not suitable in case of intrinsic pulmonary dysfunction because it
delivers variable inflation volumes depending on lung compliance

Go to the next page if you knew the correct answer, or click the link images
below to further research the concepts in this question (if desired).

Research Concepts:
Mechanical Ventilation:

Tap flag to report any problems with this question.


Question 1959: What is the average intrinsic rate of the bundle of His?
Choices:
1. 15 beats per minute
2. 35 beats per minute
3. 80 beats per minute
4. 100 beats per minute
Answer: 1 - 15 beats per minute
Explanations:
The intrinsic rate of the bundle of His is about 15 beats per minute

Go to the next page if you knew the correct answer, or click the link images
below to further research the concepts in this question (if desired).

Research Concepts:
Bundle of His:

Tap flag to report any problems with this question.


Question 1960: What is the mechanism of action of aspirin?
Choices:
1. Increase cyclic GMP
2. Increase thromboxane formation
3. Inhibit cyclooxygenase
4. Inhibit ADP induced aggregation
Answer: 3 - Inhibit cyclooxygenase
Explanations:
Aspirin blocks thromboxane synthesis by irreversibly inhibiting the enzyme
cyclooxygenase.
The resulting effects of aspirin last for 7-10 days, which is the life span of
platelets.
ASA is currently used in the treatment of transient ischemic attacks, post
myocardial infarction, and post coronary artery bypass surgery.

Go to the next page if you knew the correct answer, or click the link images
below to further research the concepts in this question (if desired).

Research Concepts:
Aspirin:

Tap flag to report any problems with this question.


Question 1961: What should a healthcare provider do if a psychologically
competent patient refuses treatment?

Choices:
1. Have a judge order treatment
2. Discuss implications with the family
3. Review the case with the ethics committee
4. Respect the patients wishes
Answer: 4 - Respect the patients wishes
Explanations:
The competent patient has the right to refuse treatment.
Discussing with the family and ethics committee cannot overrule the
patient.
Patient confidentiality would prevent discussion with family without the
patient's permission.

Go to the next page if you knew the correct answer, or click the link images
below to further research the concepts in this question (if desired).

Research Concepts:
Refusal of Care:

Tap flag to report any problems with this question.


Question 1962: Which class of antihypertensive agents has been linked to
angioneurotic edema?

Choices:
1. Diuretics
2. Calcium channel blockers
3. ACE inhibitors
4. Beta-blockers
Answer: 3 - ACE inhibitors
Explanations:
Side effects of ACE inhibitors include dry cough, rashes, fever, altered
taste, hypotension, and hyperkalemia.
Potassium levels must be monitored and potassium supplementations are
contraindicated in patients who take ACE inhibitors.
Angioedema is a rare but potentially life threatening reaction.

Go to the next page if you knew the correct answer, or click the link images
below to further research the concepts in this question (if desired).

Research Concepts:
Angiotensin Converting Enzyme Inhibitors (ACEI):

Tap flag to report any problems with this question.


Question 1963: Which of the following electrolyte abnormalities are
commonly seen after insulin infusion?

Choices:
1. Hypernatremia
2. Hyponatremia
3. Hyperkalemia
4. Hypokalemia
Answer: 4 - Hypokalemia
Explanations:
Insulin infusion commonly results in intracellular shift of potassium
Insulin infusion is often used to reduce hyperkalemia
Sodium bicarbonate can also cause hypokalemia by inducing intracellular
shift of potassium

Go to the next page if you knew the correct answer, or click the link images
below to further research the concepts in this question (if desired).

Research Concepts:
Hypokalemia:

Insulin:

Tap flag to report any problems with this question.


Question 1964: In a patient with liver failure requiring a local anesthetic,
what is the best choice?

Choices:
1. Lidocaine
2. Bupivacaine
3. Mepivacaine
4. Tetracaine
Answer: 4 - Tetracaine
Explanations:
Amide local anesthetics are hydrolyzed in the liver. Liver dysfunction may
increase the elimination half-life of amide local anesthetics.
Lidocaine, bupivacaine, and mepivacaine are amides and need to be used
with caution in those with hepatic disease.
Patients with severe liver disease may develop toxic levels.
Tetracaine is an ester and is safer in these patients.

Go to the next page if you knew the correct answer, or click the link images
below to further research the concepts in this question (if desired).

Research Concepts:
Local Anesthetics:

Tap flag to report any problems with this question.


Question 1965: Your patient has a 6 beat run of ventricular tachycardia. A
lidocaine infusion is prescribed at 2 mg/min. The infusion is mixed with 2 g/500
cc of lactated Ringer solution. What is the appropriate rate of delivery?

Choices:
1. 3 cc per hour
2. 6 cc per hour
3. 30 cc per hour
4. 60 cc per hour
Answer: 3 - 30 cc per hour
Explanations:
30 cc per hour will be delivered. 60 min/1 hr X 2mg/1 min X 1q/1000mg X
500cc/2 gm = 30 cc per hour.

Go to the next page if you knew the correct answer, or click the link images
below to further research the concepts in this question (if desired).

Research Concepts:
Antiarrhythmic Medication:

Ventricular Tachycardia:

Tap flag to report any problems with this question.


Question 1966: Select the best study to identify a pericardial effusion.
Choices:
1. Echocardiography
2. CT scan
3. MRI
4. Chest radiograph
Answer: 1 - Echocardiography
Explanations:
MRI and CT may detect smaller effusions but do not provide the functional
data that echocardiography does

Go to the next page if you knew the correct answer, or click the link images
below to further research the concepts in this question (if desired).

Research Concepts:
Pericardial Effusion:

Tap flag to report any problems with this question.


Question 1967: Which legislation supports the use of advanced directives?
Choices:
1. The Civil Rights Act
2. Rehabilitation Act of 1973
3. The Patient Self-Determination Act
4. Family Leave Act of 1993
Answer: 3 - The Patient Self-Determination Act
Explanations:
The Patient Self-Determination Act protects the rights of people in making
their own health care decisions.
Advanced directives allow patient to communicate their desires if they
become incapable of making decisions.
Power of attorney for health care names another to make such decisions.

Go to the next page if you knew the correct answer, or click the link images
below to further research the concepts in this question (if desired).

Research Concepts:
Patient Self-Determination Act:

Advanced Directives:

Tap flag to report any problems with this question.


Question 1968: Which of the following conditions requires intervention
prior to surgery?

Choices:
1. First degree heart block
2. Wenckebach (Mobitz type I)
3. Classical (Mobitz type II)
4. Sinus bradycardia
Answer: 3 - Classical (Mobitz type II)
Explanations:
Mobitz type II can have serious complications.
Placing a pacemaker prior to surgery may be required.

Go to the next page if you knew the correct answer, or click the link images
below to further research the concepts in this question (if desired).

Research Concepts:
Heart Block, Second Degree:

Tap flag to report any problems with this question.


Question 1969: In a patient with stable coronary artery disease, which
statement is TRUE?

Choices:
1. Best treatment is percutaneous intervention
2. These patients do better with open-heart surgery
3. The patients do best with medical therapy
4. There is no difference in prognosis if the medical therapy is optimal
Answer: 4 - There is no difference in prognosis if the medical therapy is
optimal

Explanations:
In patients with stable angina, if optimal medical therapy is provided there
is no difference in outcome compared to percutaneous intervention or
surgery.
Evidence now shows that elderly patients with stable CAD do better with
medical therapy.
Percutaneous intervention may help in the short term, but stent occlusion is
a chronic problem.
Open-heart surgery is best reserved for patients with depressed LV function
and triple vessel disease.

Go to the next page if you knew the correct answer, or click the link images
below to further research the concepts in this question (if desired).

Research Concepts:
Coronary Artery Disease:

Tap flag to report any problems with this question.


Question 1970: A 71-year-old patient develops an ischemic stroke after
prolonged bed rest. Which of the following is the most likely cause?

Choices:
1. Atrial septal defect
2. Atrial embolism
3. Coronary atherosclerosis
4. Underlying malignancy
Answer: 1 - Atrial septal defect
Explanations:
Atrial septal defect is a congenital heart defect in the interatrial septum.
It allows blood flow between the atria.
It can result in paradoxical emboli, a clot traveling from the right atrium to
the left and into systemic circulation. This can cause a cerebrovascular
accident.
Other complications of ASD are pulmonary hypertension, Eisenmenger
syndrome, atrial flutter, atrial fibrillation, and right sided heart failure.

Go to the next page if you knew the correct answer, or click the link images
below to further research the concepts in this question (if desired).

Research Concepts:
Atrial Septal Defect:

Stroke, Ischemic:

Tap flag to report any problems with this question.


Question 1971: What effect does digoxin have on the ECG?
Choices:
1. Prolonged PR
2. Widened QRS
3. Tented T wave
4. Delta wave
Answer: 1 - Prolonged PR
Explanations:
Digoxin blocks AV conduction and prolongs the PR interval.

Go to the next page if you knew the correct answer, or click the link images
below to further research the concepts in this question (if desired).

Research Concepts:
Digoxin:

Tap flag to report any problems with this question.


Question 1972: Once aortic stenosis patients begin to develop symptoms
their mortality rate over the next three years is:

Choices:
1. 25%
2. 50%
3. 75%
4. nearly 100%
Answer: 3 - 75%
Explanations:
Unless they undergo valve replacement therapy, the three-year mortality
rate after symptoms develop is 75%.

Go to the next page if you knew the correct answer, or click the link images
below to further research the concepts in this question (if desired).

Research Concepts:
Aortic Stenosis:

Tap flag to report any problems with this question.


Question 1973: Which is NOT an abnormal breath sounds?
Choices:
1. Rales
2. Wheezes
3. Stridor
4. Sibilance
Answer: 4 - Sibilance
Explanations:
Rales, wheezes and stridor are abnormal breath sounds
Rhonchi are also abnormal breath sounds
Sibilance is not an abnormal breath sound

Go to the next page if you knew the correct answer, or click the link images
below to further research the concepts in this question (if desired).

Research Concepts:
Lung Exam:

Tap flag to report any problems with this question.


Question 1974: Which medication has not shown to reduce the risk of MI
in patients with coronary disease?

Choices:
1. Aspirin
2. Nifedipine
3. Atorvastatin
4. Metoprolol
Answer: 2 - Nifedipine
Explanations:
Calcium channel blockers have not been shown to reduce risk in patients
with coronary disease

Go to the next page if you knew the correct answer, or click the link images
below to further research the concepts in this question (if desired).

Research Concepts:
Coronary Artery Disease:

Tap flag to report any problems with this question.


Question 1975: A 33 year old G1 P0 woman at 29 weeks gestation
presents with dyspnea at rest, orthopnea and cough. She has a prior history of
mitral stenosis secondary to rheumatic fever. On exam she is tachycardic with
irregular pulse and blood pressure is 135/85 mmHg. She has a low diastolic
rumble heard best at the apex, prominent neck veins and bibasilar rales.
Echocardiogram reveals normal left ventricular ejection fraction and mitral valve
area of 0.8 cm2. She fails to improve on diltiazem and furosemide. Which is the
indicated course of treatment for this patient?

Choices:
1. Start an amiodarone drip
2. Urgent cesarean section
3. Change to metoprolol for rate control
4. Percutaneous mitral balloon valvulotomy
Answer: 4 - Percutaneous mitral balloon valvulotomy
Explanations:
If a patient with severe mitral stenosis with a valve area of less than 1.0 cm
is experiencing symptomatic decompensation with maximal medical
therapy, invasive therapy is indicated
The risks of procedural radiation to the developing fetus are outweighed at
this stage by the risks of extreme prematurity from an early delivery

Go to the next page if you knew the correct answer, or click the link images
below to further research the concepts in this question (if desired).

Research Concepts:
Mitral Stenosis:

Pregnancy:

Tap flag to report any problems with this question.


Question 1976: What is the term for occurrence of psoriasis at sites of
trauma?

Choices:
1. Sister Mary Joseph phenomenon
2. Chadwick sign
3. Koebner sign
4. Nikolsky sign
Answer: 3 - Koebner sign
Explanations:
Koebner sign refers to development of new lesions at sites of injury. Any
type of trauma can lead to new psoriatic skin lesions.
In general, causes of Koebner reaction are due to scratching rather than the
actual infection or chemical injury.
Other disorders besides psoriasis that exhibit Koebner reaction are
molluscum contagiosum, warts, and toxidodendron dermatitis.
For this reason, all psoriatic patients should be aware that any skin damage
could lead to new lesions.

Go to the next page if you knew the correct answer, or click the link images
below to further research the concepts in this question (if desired).

Research Concepts:
Psoriasis:

Tap flag to report any problems with this question.


Question 1977: What is normal urine output for a young infant?
Choices:
1. Less than 1 mL per kg per hour
2. 2 mL per kg per hour
3. 4 mL per kg per hour
4. 6 mL per kg per hour
Answer: 2 - 2 mL per kg per hour
Explanations:
A normal urine output of a young infant is 2 mL per kg per hour

Go to the next page if you knew the correct answer, or click the link images
below to further research the concepts in this question (if desired).

Research Concepts:
Urine Output:

Tap flag to report any problems with this question.


Question 1978: A 73 year old female has diabetes and chronic non-
valvular atrial fibrillation on metoprolol for rate control. She has not had a CVA
or TIA. Which of the following should be started for prevention of strokes?

Choices:
1. Aspirin
2. Aspirin and dipyridamole
3. Warfarin
4. Nothing
Answer: 3 - Warfarin
Explanations:
Patients with chronic non-valvular atrial fibrillation have a 5 fold increased
risk for stroke
Unless contraindicated, warfarin has been the drug of choice for stroke
prevention in moderate to high risk patients
Dabigatran and rivaroxaban are indicated for prevention of stroke in these
patients
They have the advantage of not needing protime monitoring but have not
shown superiority in stroke prevention

Go to the next page if you knew the correct answer, or click the link images
below to further research the concepts in this question (if desired).

Research Concepts:
Atrial Fibrillation:

Tap flag to report any problems with this question.


Question 1979: How can hypoxemia be rapidly measured?
Choices:
1. ABG
2. VBG
3. Arterial line
4. Pulse oximetry
Answer: 4 - Pulse oximetry
Explanations:
Pulse oximetry detects hypoxemia faster than transcutaneous methods

Go to the next page if you knew the correct answer, or click the link images
below to further research the concepts in this question (if desired).

Research Concepts:
Hypoxia:

Tap flag to report any problems with this question.


Question 1980: A 72-year-old male is admitted to the hospital for elective
bowel surgery. The patient claims that his wife died of a pulmonary embolus and
it is decided to treat the patient prophylactically to prevent a deep vein
thrombosis. Which of the following is the least effective way to prevent a deep
vein thrombosis?

Choices:
1. Minidose heparin
2. Graded stockings
3. Dextran
4. Acetylsalicylic acid
Answer: 4 - Acetylsalicylic acid
Explanations:
Mechanical compression with compression stockings is widely used but the
latest evidence indicates that it must be combined with a pharmacological
method to prevent deep vein thrombosis.
Aspirin alone is not recommended and it does not help prevent deep vein
thrombosis. Compared to low molecular weight heparin, aspirin fared
worse than a placebo.
Both heparin and low molecular weight heparin can be used as prophylaxis
against deep vein thrombosis. Low molecular weight heparin has a long
half-life and less tendency to cause heparin-induced thrombocytopenia.
Low molecular weight heparin does not need any monitoring and is cost
effective.
Graded intermittent pneumatic stockings are also widely used and effective
in preventing deep vein thrombosis.

Go to the next page if you knew the correct answer, or click the link images
below to further research the concepts in this question (if desired).

Research Concepts:
Deep Venous Thrombosis Prophylaxis:

Tap flag to report any problems with this question.


Question 1981: Which of the following is not part of treatment after a
massive air embolism during cardiopulmonary bypass?

Choices:
1. Administer 100% oxygen
2. Lower perfusion pressure
3. De air coronaries by antegrade perfusion
4. Administer steroids and phenytoin
Answer: 3 - De air coronaries by antegrade perfusion
Explanations:
After a massive air embolus, it is the CNS effects, which will predominate.
The air from the coronaries will slowly leak out and into the aortic vent but
the air in the CNS is hard to get out.
Hyperbaric oxygen has been tried out in selected cases
Retrograde perfusion via the coronary sinus may assist in removing air
from the coronary

Go to the next page if you knew the correct answer, or click the link images
below to further research the concepts in this question (if desired).

Research Concepts:
CPB, Air Embolism:

Tap flag to report any problems with this question.


Question 1982: Which of the following findings is not associated with
severe mitral stenosis?

Choices:
1. Pulmonary vascular congestion
2. Atrial fibrillation
3. Decreased left atrial pressure
4. Jugular venous distention\
Answer: 3 - Decreased left atrial pressure
Explanations:
There is increased left atrial pressure with severe mitral stenosis
Elevated left atrial pressure causes the mitral valve to open quickly
This results in a short interval between A2 and the opening snap
Severe mitral stenosis can result in elevated jugular pressure, atrial
fibrillation, and pulmonary vascular congestion

Go to the next page if you knew the correct answer, or click the link images
below to further research the concepts in this question (if desired).

Research Concepts:
Mitral Stenosis:

Tap flag to report any problems with this question.


Question 1983: A patient with a "Do Not Resuscitate" order is in
respiratory distress secondary to acute pulmonary edema. What is the best
management option?

Choices:
1. Provide oxygen and furosemide
2. Do nothing
3. Endotracheal intubation
4. Call the patient's family to get permission to treat
Answer: 1 - Provide oxygen and furosemide
Explanations:
A DNR order does not mean to withhold treatment
Oxygen and furosemide would be appropriate for this patient suffering from
acute pulmonary edema. If there is not an intravenous line, oral morphine
would be an option.
Endotracheal intubation would be too aggressive with a DNR order
Calling a patient's family to inform them of treatment is not required with a
DNR order

Go to the next page if you knew the correct answer, or click the link images
below to further research the concepts in this question (if desired).

Research Concepts:
Respiratory Distress/Failure:

Do Not Resuscitate (DNR):

Tap flag to report any problems with this question.


Question 1984: What is the drug of choice for pseudomembranous
enterocolitis?

Choices:
1. Metronidazole
2. Vancomycin
3. Minocycline
4. Ceftriaxone
Answer: 1 - Metronidazole
Explanations:
Both metronidazole and vancomycin are effective against Clostridium
difficile
Metronidazole is much less expensive
Vancomycin can select for vancomycin resistant enterococci

Go to the next page if you knew the correct answer, or click the link images
below to further research the concepts in this question (if desired).

Research Concepts:
Pseudomembranous Colitis:

Tap flag to report any problems with this question.


Question 1985: Which of the following is NOT a sign of digitalis toxicity?
Choices:
1. Prolonged PR interval
2. Nausea and vomiting
3. Hair loss
4. Atrial or ventricular dysrhythmias
Answer: 3 - Hair loss
Explanations:
Digitalis lowers the heart rate by working on phase 4 of the cardiac cycle,
decreases automaticity.
Digitalis inhibits the sodium/potassium pump.

Go to the next page if you knew the correct answer, or click the link images
below to further research the concepts in this question (if desired).

Research Concepts:
Digoxin:

Tap flag to report any problems with this question.


Question 1986: Which of the following is most specific test for diagnosis
of subacute bacterial endocarditis?

Choices:
1. Transesophageal echocardiogram
2. Elevated erythrocyte sedimentation rate
3. Positive blood culture
4. Leukocytosis with left shift
Answer: 3 - Positive blood culture
Explanations:
The most specific test or SBE is a positive blood culture
Vegetations seen on echocardiogram are often positive with this condition
but the absence does not rule it out

Go to the next page if you knew the correct answer, or click the link images
below to further research the concepts in this question (if desired).

Research Concepts:
Endocarditis, Bacterial:

Tap flag to report any problems with this question.


Question 1987: Which substance has no alpha-receptor activity?
Choices:
1. Phenylephrine
2. Clonidine
3. Labetalol
4. Metoprolol
Answer: 4 - Metoprolol
Explanations:
Labetalol is an alpha and beta-blocker
Metoprolol is a selective beta 2 blocker

Go to the next page if you knew the correct answer, or click the link images
below to further research the concepts in this question (if desired).

Research Concepts:
Metoprolol:

Tap flag to report any problems with this question.


Question 1988: Propranolol works to reduce angina by what mechanism
of action?

Choices:
1. Decreasing production of catecholamines
2. Dilating coronary arteries
3. Decreasing oxygen requirements
4. Increases peripheral resistance
Answer: 3 - Decreasing oxygen requirements
Explanations:
Propranolol is a non-selective beta-blocker with little or no intrinsic
sympathetic activity.
Propranolol helps decrease angina by blocking catecholamines and
reducing the work of the heart.

Go to the next page if you knew the correct answer, or click the link images
below to further research the concepts in this question (if desired).

Research Concepts:
Beta-Blockers:

Tap flag to report any problems with this question.


Question 1989: Coarctation of the aorta is NOT typically associated with
which of the following?

Choices:
1. Ventricular septal defect
2. Bicuspid aortic valve
3. Patent ductus arteriosus
4. Atrial septal defect
Answer: 4 - Atrial septal defect
Explanations:
Coarctation of the aorta accounts for 5-8% of congenital heart defects.
It is most commonly associated with bicuspid aortic valve, ventricular
septal defect, and patent ductus arteriosus.
Coarctation of the aorta often presents with upper extremity hypertension
and radial-femoral pulse delay.
The disorder can be surgically treated but endovascular techniques are also
being applied.

Go to the next page if you knew the correct answer, or click the link images
below to further research the concepts in this question (if desired).

Research Concepts:
Aortic Coarctation:

Tap flag to report any problems with this question.


Question 1990: What is the best medication for glottic edema post
extubation?

Choices:
1. Racemic epinephrine
2. Albuterol
3. Ipratropium
4. Steroid
Answer: 1 - Racemic epinephrine
Explanations:
Racemic epinephrine has alpha receptor activity
Can result in vasoconstriction and decrease glottic swelling

Go to the next page if you knew the correct answer, or click the link images
below to further research the concepts in this question (if desired).

Research Concepts:
Extubation:

Glottic Edema:

Tap flag to report any problems with this question.


Question 1991: Which of the following statements is not true in regards to
the hypertrophic cardiomyopathy disease?

Choices:
1. Charaterized by asymmetric hypertrophy
2. It is an autosomal recessive disorder
3. Histology reveals disarray of myocardium fibers
4. Calcium channel blockers are first line pharmacologic agents
Answer: 2 - It is an autosomal recessive disorder
Explanations:
Hypertrophic cardiomyopathy is an autosomal dominant disorder
characterized by asymmetric septal hypertrophy and myocardial fiber
disarray
Beta blockers and calcium channel blockers are usually the first line agents

Go to the next page if you knew the correct answer, or click the link images
below to further research the concepts in this question (if desired).

Research Concepts:
Hypertrophic Obstructive Cardiomyopathy:

Tap flag to report any problems with this question.


Question 1992: What is the agent of choice to reverse heparin?
Choices:
1. Warfarin
2. Protamine
3. Albumin
4. Cryoprecipitate
Answer: 2 - Protamine
Explanations:
Protamine reverses the action of heparin

Go to the next page if you knew the correct answer, or click the link images
below to further research the concepts in this question (if desired).

Research Concepts:
Protamine:

Tap flag to report any problems with this question.


Question 1993: Which of the following antiarrhythmics can be used to
terminate serious ventricular arrhythmias?

Choices:
1. Adenosine
2. Digoxin
3. Amiodarone
4. Propranolol
Answer: 3 - Amiodarone
Explanations:
Amiodarone is used for pharmacologic cardioversion in hemodynamically
stable patients
Lidocaine is used to terminate ventricular tachycardias and prevent
ventricular fibrillation after cardioversion
Paroxysmal supraventricular tachycardias are best treated with adenosine
Propranolol is a beta blockers used to reduce AV nodal conductivity

Go to the next page if you knew the correct answer, or click the link images
below to further research the concepts in this question (if desired).

Research Concepts:
Amiodarone:

Tap flag to report any problems with this question.


Question 1994: Collagen synthesis in a wound after 10 to 13 days would
have collagen synthesis. Which of the following would be evidence of this?

Choices:
1. Monocytes
2. Polymorphonuclear cells
3. Hydroxyproline
4. Fibrin
Answer: 3 - Hydroxyproline
Explanations:
Collagen synthesis would be evidenced by presence of hydroxyproline

Go to the next page if you knew the correct answer, or click the link images
below to further research the concepts in this question (if desired).

Research Concepts:
Wound Healing:

Tap flag to report any problems with this question.


Question 1995: Which of the following is the most common site of intimal
tears with an aortic dissection?

Choices:
1. Just distal to the left subclavian artery
2. Within a few centimeters of the proximal ascending aorta
3. At the proximal arch
4. Distal to the descending aorta
Answer: 2 - Within a few centimeters of the proximal ascending aorta
Explanations:
Type I dissection involves the entire aorta. Type ll involves only the
ascending aorta and type lll involves the descending aorta.
Cystic medial degeneration plays a role in the pathology of aneurysmal
formation and dissection in the ascending aorta.
Other factors related to dissection in the ascending aorta are hypertension,
cannulation during cardiopulmonary bypass, Marfan, pregnancy, retrograde
dissection, and a bicuspid aortic valve.
The majority of dissections (95%) occur in 2 places. The most common is
several centimeters about the atrioventricular valve and the second most
common is just beyond the ligamentum arteriosum in the descending aorta.
The tear is usually transverse and can progress in either a retrograde or an
antegrade fashion.

Go to the next page if you knew the correct answer, or click the link images
below to further research the concepts in this question (if desired).

Research Concepts:
Dissection, Aortic:

Tap flag to report any problems with this question.


Question 1996: Which of the following is an aldosterone antagonist used
in the treatment of congestive heart failure (CHF)?

Choices:
1. Nitroprusside
2. Eplerenone
3. Losartan
4. Isosorbide dinitrate
Answer: 2 - Eplerenone
Explanations:
Eplerenone is an aldosterone antagonist used in the treatment of CHF
Nitroprusside is a vasodilator used in the treatment of CHF
Losartan is an angiotensin receptor blocker used in the treatment of CHF
Isosorbide dinitrate is a nitrate used in the treatment of CHF

Go to the next page if you knew the correct answer, or click the link images
below to further research the concepts in this question (if desired).

Research Concepts:
Potassium Sparing Diuretics:

Heart Failure, Congestive:

Tap flag to report any problems with this question.


Question 1997: A 65-year-old presents with chest pain that started a few
hours ago. Blood work is quickly drawn and it is observed that his enzymes
peaked at 24-48 hours and lasted for several days. Which of the following
enzymes were measured?

Choices:
1. Troponin
2. CPK
3. LDH
4. AST
Answer: 2 - CPK
Explanations:
CPK levels usually start to rise within 4 hours after an acute MI
The levels of CPK peak at about 24-36 hours and the en subside over the
next 3-4 days
Occasionally small infarcts can be missed by CPK and thus troponin levels
should be measured in patients with suspected MI but have negative serial
CPK

Go to the next page if you knew the correct answer, or click the link images
below to further research the concepts in this question (if desired).

Research Concepts:
Myocardial Infarction, Acute:

Tap flag to report any problems with this question.


Question 1998: All of the following are options for treatment of a patent
ductus arteriosus except?

Choices:
1. Intravenous NSAIDs
2. Intravenous digoxin
3. Catheter closure
4. Surgical ligation
Answer: 2 - Intravenous digoxin
Explanations:
Option include intravenous ibuprofen or indomethacin, surgical ligation,
catheter closure, and waiting for spontaneous closure
Surgical ligation requires a thoracotomy
Spontaneous closure of PDA is common

Go to the next page if you knew the correct answer, or click the link images
below to further research the concepts in this question (if desired).

Research Concepts:
Patent Ductus Arteriosus:

Tap flag to report any problems with this question.


Question 1999: Which route of body heat loss during general anesthesia is
least likely?

Choices:
1. Respiration
2. Skin
3. Urine
4. Soft tissue exposure
Answer: 3 - Urine
Explanations:
Temperature must be monitored for the entire duration of the case.

Go to the next page if you knew the correct answer, or click the link images
below to further research the concepts in this question (if desired).

Research Concepts:
Hypothermia:

General Anesthesia:

Tap flag to report any problems with this question.


Question 2000: What is the most common cause of subacute endocarditis?
Choices:
1. Staphylococcus
2. Streptococcus
3. Pseudomonas
4. E coli
Answer: 2 - Streptococcus
Explanations:
The most common cause of subacute endocarditis is Streptococcus viridans
The most common cause of acute endocarditis is staphylococcus aureus

Go to the next page if you knew the correct answer, or click the link images
below to further research the concepts in this question (if desired).

Research Concepts:
Endocarditis, Bacterial:

Tap flag to report any problems with this question.


Knowledge Testing develops Kindle eBooks for Medical Professionals.

Here is a list other books available on Kindle. We also have fully interactive
apps with questions and answers available for the Ipad, IPhone, Android Phones,
and Android Tablets. These are available on the Google Play store and the Itunes
Connect. Please visit our website at http://www.knowledgetesting.com for more
information.

Allied Health

Alternative/Chinese Certification Review


Anesthesia Technician Certification Review
Apheresis Technician Certification Review
Athletic Trainer Certification Review
Audiology Certification Review
Bioengineering and Biomedicine Certification Review
Blood Banking Certification Review
Cardiovascular Technician Certification Review
Certified Clinical Nutritionist Certification Review
Certified Medical Assistant (CMA) Certification Review
Certified Nursing Assistant (CNA) Certification Review
Chemistry, C (ASCP) Certification Review
Chiropractic Certification Review
Clinical Neuropsychology Certification Review
Cytotechnologist Certification Review
Dialysis Technician Certification Review
Dietitian Certification Review
EKG Technician Certification Review
EMT (Basic) Certification Review
EMT (Intermediate) Certification Review
EMT (Paramedic) Certification Review
Exercise Physiology Certification Review
Hemapheresis Practitioner Certification Review
Hematologist Certification Review
Histotechnologist Certification Review
Holistic Medicine Certification Review
Laboratory Safety Certification Review
Massage Therapist Certification Review
Medical Administrative Certification Review
Medical Billing and Coding Certification Review
Medical Lab Assistant Certification Review
Medical Laboratory Scientist Certification Review
Medical Laboratory Technician Certification Review
Medical Physics Certification Review
Medical Technologist Certification Review
Mental Health Technician Certification Review
Microbiology, M (ASCP) Certification Review
Molecular Pathology, MP (ASCP) Certification Review
Naturopathic Certification Review
Occupational Therapy Certification Review
Orthotics and Prosthetics Certification Review
Pathologists' Assistant Certification Review
Patient Care Technician Certification Review
Perfusionist Certification Review
Personal Training Certification Review
Pharmacy Technician (PTCB) Certification Review
Phlebotomy Technician Certification Review
Physical Therapy Certification Review
Physiotherapy Certification Review
Radiation Therapist Certification Review
Radiology Technology – Cardiovascular Technologists Certification
Review
Radiology Technology – CT Certification Review
Radiology Technology – Densitometrists Certification Review
Radiology Technology – Dosimetrists Certification Review
Radiology Technology – Mammography Certification Review
Radiology Technology – MRI Certification Review
Radiology Technology – Nuclear Medicine Certification Review
Radiology Technology – Quality Management Certification Review
Radiology Technology Certification Review
Registered Diagnostic Cardiac Sonographer Certification Review
Registered Vascular Technologist Certification Review
Respiratory Therapist Certification Review
Social Work Certification Review
Speech Pathology Certification Review
Surgical Technician Certification Review
Ultrasound Technologist Certification Review

Health Professional
Advanced Cardiac Life Support (ACLS) Review Course
Pediatric Advanced Life Support (PALS) Review Course

Nursing
CNS-Adult Health Certification Review
CNS-Adult Psychiatric and Mental Health Certification Review
CNS-Child/Adolescent and Mental Health Certification Review
CNS-Diabetes Management Certification Review
CNS-Gerontological Certification Review
CNS-Home Health Certification Review
CNS-Pediatric Certification Review
CNS-Public/Community Health Certification Review
MS-Nurse Anesthetist (CRNA) Certification Review
NP-Acute Care Certification Review
NP-Adult Certification Review
NP-Adult Psychiatric and Mental Health Certification Review
NP-Diabetes Management Certification Review
NP-Family Certification Review
NP-Family Psych and Mental Health Certification Review
NP-Gerontological Certification Review
NP-Midwife Certification Review
NP-Pediatric Certification Review
RN-Addictions Registered (CARN) Certification Review
RN-Administrator (NE-BC) Certification Review
RN-Aids/HIV Registered (ACRN/AACRN) Certification Review
RN-Ambulatory Care Certification Review
RN-Ambulatory Perianesthesia/Post Anesthesia (CAPA/CPAN)
Certification Review
RN-Anticoagulation Certification Review
RN-Bariatric (CBN) Certification Review
RN-Breast Care Certification Review
RN-Cardiovascular (CVN/CVRN) Certification Review
RN-Case Management Certification Review
RN-Continence Care (CCCN) Certification Review
RN-Corrections (CCN) Certification Review
RN-Critical Care (CCRN) Certification Review
RN-Dermatology (DNC) Certification Review
RN-Developmental Disabilities (CDDN)
RN-Diabetes Educator (CDE)
RN-Eating Disorders (CEDRN) Certification Review
RN-Emergency (CEN) Certification Review
RN-Flight Registered (CFRN) Certification Review
RN-Foot and Nail Care Nurse (CFCN) Certification Review
RN-Gastroenterology (CGN/CGRN) Certification Review
RN-Genetics Clinical (GCN) Certification Review
RN-Gerontology (NCA) Certification Review
RN-Hemodialysis (CHN) Certification Review
RN-Holistic (HN-BC) Certification Review
RN-Hospice and Palliative (CHPN) Certification Review
RN-Hyperbaric Registered (CHRN) Certification Review
RN-Infection Control (CIC) Certification Review
RN-Informatics Certification Review
RN-Infusion (CRNI) Certification Review
RN-Lactation (CLA) Certification Review
RN-Low Risk Neonatal (LRN) Certification Review
RN-Managed Care (CMCN) Certification Review
RN-Maternal Newborn (MN) Certification Review
RN-Medical-Surgical Certification Review
RN-NCLEX - LPN/LVN Certification Review
RN-NCLEX - PN Certification Review
RN-Neonatal Intensive Care Review (NICU) Certification Review
RN-Nephrology (CNN) Certification Review
RN-Neuroscience Registered (CNRN) Certification Review
RN-Occupational & Environmental Health (COHN) Certification Review
RN-Oncology (OCN) Certification Review
RN-Operating Room (CNOR) Certification Review
RN-Ophthalmology (CRNO) Certification Review
RN-Orthopedic (ONC) Certification Review
RN-Ostomy Care (COCN) Certification Review
RN-Otorhinolaryngology (CORLN) Certification Review
RN-Pain Management (PMCN) Certification Review
RN-Pediatric (CPN) Certification Review
RN-Pediatric Critical Care (CCRN-P) Certification Review
RN-Pediatric Emergency (CEN-P) Certification Review
RN-Pediatric Oncology (CPON) Certification Review
RN-Perinatal Nurse/InPatient OB Nurse Certification Review
RN-Peritoneal Dialysis (CPDN) Certification Review
RN-Plastic Surgery (CPSN) Certification Review
RN-Professional Development Certification Review
RN-Professional Healthcare Quality (CPHQ) Certification Review
RN-Progressive Care (PCCN) Certification Review
RN-Psychiatric and Mental Health (PMHN) Certification Review
RN-Radiologic (CRN) Certification Review
RN-Rehabilitation Registered (CRRN) Certification Review
RN-Risk Management (CPHRM) Certification Review
RN-School (NCSN) Certification Review
RN-Sexual Assault (SANE) Certification Review
RN-Transplant (CTN) Certification Review
RN-Transport (CTRN) Certification Review
RN-Trauma (TNCC) Certification Review
RN-Urologic Registered (CURN) Certification Review
RN-Wound Care (CWCN) Certification Review

MD/DO/PA/OD/DDS/Pharmacist/PsyD
Adolescent Medicine Board Review
Aerospace Medicine Board Review
Allergy and Immunology Board Review
Ambulatory and Urgent Care Board Review
Anesthesiology Board Review
Anesthesiology-Cardiothoracic Board Review
Anesthesiology-Pediatric Board Review
ATLS Board Review
Cardiology Board Review
Cardiology-Electrophysiology Board Review
Cardiology-Heart Failure and Transplant Board Review
Cardiology-Interventional Board Review
Critical Care Board Review
Dental School (NBDE Part I) Board Review
Dermatology Board Review
Emergency Medicine Board Review
Endocrinology, Diabetes, and Metabolism Board Review
Family Medicine Board Review
Gastroenterology Board Review
Genetics Board Review
Geriatrics Board Review
Hematology Board Review
Hospice and Palliative Medicine Board Review
Infectious Disease Board Review
Internal Medicine Board Review
Medical School (USMLE I) Board Review
Medical School (USMLE II) Board Review
Medical Student (USMLE III) Board Review
Nephrology Board Review
Neurology Board Review
Neurology-Headache Board Review
Neurology-Neurodevelopmental Board Review
Neurology-Neuromuscular Board Review
Obesity Medicine Board Review
Occupational Medicine Board Review
Oncology Board Review
Optometry Board Review
Osteopathic School (COMLEX I) Board Review
Osteopathic School (COMLEX II) Board Review
Osteopathic School (COMLEX III) Board Review
Pain Medicine Board Review
Pance and Panre Board Review
Pathology-Anatomic and Clinical Board Review
Pediatric Board Review
Pediatric-Cardiology Board Review
Pediatric-Child Abuse Board Review
Pediatric-Critical Care Board Review
Pediatric-Developmental/Behavioral Board Review
Pediatric-Emergency Medicine Board Review
Pediatric-Endocrinology Board Review
Pediatric-Gastroenterology Board Review
Pediatric-Hematology/Oncology Board Review
Pediatric-Infectious Diseases Board Review
Pediatric-Neonatal/Perinatal Board Review
Pediatric-Nephrology Board Review
Pediatric-Neurology Board Review
Pediatric-Pulmonology Board Review
Pediatric-Rehabilitation Board Review
Pediatric-Rheumatology Board Review
Pediatric-Sports Medicine Board Review
Pediatric-Transplant Hepatology Board Review
Pharmacist (NAPLEX) Board Review
Physical Medicine and Rehabilitation Board Review
Preventive Medicine Board Review
Psychiatry Board Review
Psychiatry-Addiction Board Review
Psychiatry-Child and Adolescent Board Review
Psychiatry-Forensic Board Review
Psychiatry-Geriatric Board Review
Psychologist Board Review
Psychosomatic Medicine Board Review
Public Health Board Review
Pulmonary Board Review
Radiation Oncology Board Review
Radiology Board Review
Radiology Physics Board Review
Radiology-Abdominal Board Review
Radiology-Endovascular Surgical Board Review
Radiology-Musculoskeletal Board Review
Radiology-Neuroradiology Board Review
Radiology-Nuclear Medicine Board Review
Radiology-Pediatric Board Review
Radiology-Vascular and Interventional Board Review
Rheumatology Board Review
Sleep Board Review
SPEX Board Review
Spinal Cord Injury Board Review
Sports Medicine Board Review
Surgery-Cardiac Board Review
Surgery-Colon and Rectal Board Review
Surgery-Complex General Surgical Oncology Board Review
Surgery-Craniofacial Board Review
Surgery-General Board Review
Surgery-Gynecologic Oncology Board Review
Surgery-Hand Board Review
Surgery-Maternal and Fetal Board Review
Surgery-Neurosurgery Board Review
Surgery-Obstetrics and Gynecology Board Review
Surgery-Ophthalmology Board Review
Surgery-Orthopaedic Board Review
Surgery-Otolaryngology Board Review
Surgery-Pediatric Board Review
Surgery-Plastic Board Review
Surgery-Podiatry Board Review
Surgery-Reproductive Board Review
Surgery-Thoracic Board Review
Surgery-Urologic Board Review
Surgery-Urologic-Pediatric Board Review
Surgery-Vascular Board Review
Toxicology Board Review
Transplant Hepatology Board Review
Undersea and Hyperbaric Board Review

You might also like